You are on page 1of 488

UGC NTA-NET/JRF

ENGLISH
Solved Papers
Also Useful for Other State Level Competitions
SLET/SET/CRET/RET/PSC/PGT

Chief Editor
A. K. Mahajan
Compiled and Edited by
Youth Competition Times Exam Expert Team
Editorial Office
12, Church Lane Prayagraj -211002
Mob. : 9415650134
Email : yctap12@gmail.com
website : www.yctbooks.com/ www.yctfastbooks.com
 All Rights Reserved with Publisher
Publisher Declaration
Edited and Published by A.K. Mahajan for YCT Publication Pvt. Ltd.
and printed by Roop Printing Press, Prayagraj. In order to Publish the book,
full care has been taken by the Editor and the Publisher,
still your suggestions and queries are welcomed. Rs. : 695/-
In the event of any dispute, the Judicial area will be Prayagraj.
CONTENT
University Grant Commision (UGC) NTA NET ENGLISH
UGC NTA NET/JRF ENGLISH Exam New Syllabus ------------------------------------------------------------------- 3
UGC NET/JRF Exam, English December 2004 Solved Paper II --------------------------------------------------- 4-8
UGC NET/JRF Exam, English June 2005 Solved Paper II --------------------------------------------------------- 9-13
UGC NET/JRF Exam, English December 2005 Solved Paper II ------------------------------------------------ 14-18
UGC NET/JRF Exam, English June 2006 Solved Paper II ------------------------------------------------------- 19-22
UGC NET/JRF Exam, English December 2006 Solved Paper II ------------------------------------------------ 23-27
UGC NET/JRF Exam, English June 2007 Solved Paper II ------------------------------------------------------- 28-32
UGC NET/JRF Exam, English December 2007 Solved Paper II ------------------------------------------------ 33-37
UGC NET/JRF Exam, English June 2008 Solved Paper II ------------------------------------------------------- 38-42
UGC NET/JRF Exam, English December 2008 Solved Paper II ------------------------------------------------ 43-46
UGC NET/JRF Exam, English June 2009 Solved Paper II ------------------------------------------------------- 47-52
UGC NET/JRF Exam, English December 2009 Solved Paper II ------------------------------------------------ 53-58
UGC NET/JRF Exam, English June 2010 Solved Paper II ------------------------------------------------------- 59-63
UGC NET/JRF Exam, English December 2010 Solved Paper II ------------------------------------------------ 64-68
UGC NET/JRF Exam, English June 2011 Solved Paper II ------------------------------------------------------- 69-73
UGC NET/JRF Exam, English December 2011 Solved Paper II ------------------------------------------------ 74-79
UGC NET/JRF Exam, English June 2012 Solved Paper III ------------------------------------------------------ 80-90
UGC NET/JRF Exam, English June 2012 Solved Paper II ------------------------------------------------------- 91-95
UGC NET/JRF Exam, English December 2012 Solved Paper III ---------------------------------------------- 96-105
UGC NET/JRF Exam, English December 2012 Solved Paper II --------------------------------------------- 106-111
UGC NET/JRF Exam, English June 2013 Solved Paper III --------------------------------------------------- 112-121
UGC NET/JRF Exam, English June 2013 Solved Paper II ---------------------------------------------------- 122-126
UGC NET/JRF Exam, English December 2013 Solved Paper III -------------------------------------------- 127-138
UGC NET/JRF Exam, English December 2013 Solved Paper II --------------------------------------------- 139-145
UGC NET/JRF Exam, English June 2014 Solved Paper III --------------------------------------------------- 146-157
UGC NET/JRF Exam, English June 2014 Solved Paper II ---------------------------------------------------- 158-164
UGC NET/JRF Exam, English December 2014 Solved Paper III -------------------------------------------- 165-173
UGC NET/JRF Exam, English December 2014 Solved Paper II --------------------------------------------- 174-178
UGC NET/JRF Exam, English June 2015 Solved Paper III --------------------------------------------------- 179-187
UGC NET/JRF Exam, English June 2015 Solved Paper II ---------------------------------------------------- 188-193
UGC NET/JRF Exam, English December 2015 Solved Paper III -------------------------------------------- 194-203
UGC NET/JRF Exam, English December 2015 Solved Paper II --------------------------------------------- 204-209
UGC NET/JRF Exam, English July-2016 Solved Paper III --------------------------------------------------- 210-218
UGC NET/JRF Exam, English July-2016 Solved Paper II ---------------------------------------------------- 219-223
UGC NET/JRF Exam, English January -2017 Solved Paper III --------------------------------------------- 224-232
UGC NET/JRF Exam, English January-2017 Solved Paper II ----------------------------------------------- 233-237
UGC NET/JRF Exam, English November -2017 Solved Paper III ------------------------------------------- 238-245
2
UGC NET/JRF Exam, English November-2017 Solved Paper II--------------------------------------------- 246-250
UGC NET/JRF Exam, English July-2018 Solved Paper II ---------------------------------------------------- 251-266
UGC NTA NET/JRF Exam, English December-2018 Solved Paper II -------------------------------------- 267-286
UGC NTA NET/JRF Exam, English June-2019 Solved Paper II--------------------------------------------- 287-298
UGC NTA NET/JRF Exam, English December-2019 Solved Paper II -------------------------------------- 299-314
UGC NTA NET/JRF Exam, English June-2020 Shift-II Solved Paper II ---------------------------------- 315-328
UGC NTA NET/JRF Exam, English June -2020 Shift-I Solved Paper II ----------------------------------- 329-343
UGC NTA NET/JRF Exam, English December-2020- June 2021 Shift-II Solved Paper II ------------- 344-359
UGC NTA NET/JRF Exam, English December-2020-June 2021 Shift-I Solved Paper II --------------- 360-376
UGC NTA NET/JRF Exam, English December-2021- June 2022 Shift-II Solved Paper II ------------- 377-397
UGC NTA NET/JRF Exam, English December-2021-June 2022 Shift-I Solved Paper II --------------- 398-416
UGC NTA NET/JRF Exam, English December-2022 Shift-I Solved Paper II ----------------------------- 417-436
UGC NTA NET/JRF Exam, English December-2022 Shift-II Solved Paper II ---------------------------- 437-456
UGC NTA NET/JRF Exam, English June-2023 Shift-I Solved Paper II .................................................. 457-470
UGC NTA NET/JRF Exam, English June-2023 Shift-II Solved Paper II ..................................................471-488

Subject: English Code No. : 30

Unit –I : Drama
Unit –II : Poetry
Unit –III : Fiction, short story
Unit –IV : Non-Fictional Prose
NOTE : The first four units must also be tested through comprehension passages
to assess critical reading, critical thinking and writing skills. These four
units will cover all literatures in English.
Unit –V : Language: Basic concepts, theories and pedagogy. English in Use.
Unit –VI : English in India: history, evolution and futures
Unit –VII : Cultural Studies
Unit –VIII : Literary Criticism
Unit –IX : Literary Theory post World War II
Unit –X : Research Methods and Materials in English
3
UGC NET/JRF Exam, December-2004
ENGLISH
Solved Paper-II
Note : This paper contains fifty (50) objective 5. Which of the following plays has a
type questions, each question carrying two (2) marks. Machiavellian hero?
Attempt all questions. (a) Tamburlaine Part I (b) Dr. Faustus
1. In Langland's 'Piers the Plowman' Piers (c) Jew of Malta (d) Edward II
appears finally as : Ans : (c) “The Jew of Malta” is a play by Christopher
(a) Charity (b) The Holy Trinity Marlowe probably written in 1589 or 1590. Plot is an
(c) Jesus (d) The Good Samaritan original story of religious conflict, intrigue, and
Ans : (d) “Piers Plowman” (written in 1370-90) is a revenge, set against a backdrop of the struggle for
Middle English allegorical narrative poem by William supremacy between Spain and the Ottoman Empire, in
Langland. It is considered by many critics to be one of the Mediterranean that takes place on the Island of
the greatest works of English Literature of the Middle Malta. The character Barabas is described in the play’s
Ages, along with Chaucer’s 'Canterbury Tales' and the prologue as “a sound Machiavil” which means he is
extremely Machiavellian.
anonymously authored Pearl and Sir Gawain and the
Green Knight. 6. Which of the following is written by Samuel
2. It was decided that each Canterbury pilgrim Butler?
would tell in all : (a) Religio Laici
(a) One story (b) Two stories (b) David Simple
(c) Three stories (d) Four stories (c) Hudibras
(d) Journal of the Plague Year
Ans : (d) “The Canterbury Tales” is a collection of
24 stories that runs to over 17,000 lines written in Ans : (c) “Hudibras” is an English mock heroic poem
from the 17th century written by Samuel Butler.
Middle English by Geoffrey Chaucer. In this Chaucer
himself is one of the pilgrims. That evening the host of 7. Which of the following poems did Milton write
the Tabard Inn suggested that each member of the group in Octosyllabic Couplets?
tell tales on the way to and from Canterbury in order to (a) IL Penseroso
make the time pass more pleasantly. (b) "On His Blindness"
3. 'Venus and Adonis' is a long narrative poem by : (c) "On the Late Massacre in Piedmont"
(d) Lycidas
(a) Shakespeare (b) Marlowe
(c) Drayton (d) Sydney Ans : (a) “IL Penseroso” is a vision of poetic
melancholy by John Milton first found in the 1645-1646
Ans : (a) “Venus and Adonis” is a poem by quarto of verses “The Poems of Mr. John Milton, both
Shakespeare written in 1592-1593, with a plot based on English & Latin” published by Humphrey Moseley.
passages from Ovid’s 'Metamorphoses'. It is a
8. Which of the following plays in not written by
complex Kaleidoscopic work, using constantly shifting
zone and perspective to present contrasting views of the Congreve?
nature of love. (a) The Way of the World
(b) The Old Bachelor
4. The total number of poems in Shakespeare's
(c) Love for Love
'Sonnets' is : (d) The Relapse
(a) 123 (c) 104
Ans : (d) “The Relapse” or virtue in danger is a
(b) 142 (d) 154 restoration comedy from 1696 written by John
Ans : (d) Shakespeare’s 'Sonnets' is the title of a Vanbrugh. The play is a sequel to Colley Cibber’s
collection of 154 sonnets by Shakespeare, which covers “Love’s Last Shift,” or “The Fool in Fashion.”
themes such as the passage of time, love, beauty and 9. Dryden's 'All for Love' is an adaptation of:
morality. The first 126 sonnets are addressed to a young
(a) Philaster (b) Romeo and Juliet
man; the last 28 to a dark lady.
(c) Antony and Cleopatra (d) Edward II
UGC NET/JRF English Paper II, December 2004 4 YCT
Ans : (c) “All For Love” or the "World Well Lost" is a 14. Pope's 'An Essay on Man' is based on the ideas
heroic drama by John Dryden written in 1677. It is an of :
acknowledged imitation of Shakespeare’s 'Antony and (a) Lord Petrie (b) Theobald
Cleopatra', and focuses on the last hours of the lives of (c) Lord Bolingbroke (d) Lord Harvey
its hero and heroine.
Ans : (c) "The Essay on Man" is a philosophical
10. Which of the following books proposes a poem, written, characteristically, in heroic couplets, and
political theory? published between 1732 and 1734, written by
(a) Principia Alexander Pope. The “Essay” consists of four epistles,
(b) Leviathan addressed to Lord Bolingbroke, and derived, to some
(c) Antony and Melancholy extent, from some of Bolingbroke’s own fragmentary
(d) Liberty of Prophesying philosophical writing.
Ans : (b) “Leviathan” is a book written by Thomas 15. Which of the following works by Johnson is an
Hobbes (1588-1679) and published in 1651. Its name imitation of the tenth satire of Juvenal?
derives from the biblical Leviathan. The work concerns (a) London
the structure of society and legitimate government. (b) Vanity of Human Wishes
11. Which of the following books is written by a (c) The Life of Savage
woman? (d) Rasselas
(a) A Vindication of the Rights of Women Ans : (b) 'The Vanity of Human Wishes: The Tenth
(b) Social Contract Satire of Juvenal Imitated' is a poem by the English
(c) A Treatise of Human Nature author Samuel Johnson written in 1749, it was the first
(d) The Wealth of Nations published work to include Johnson’s name on title page.
Ans : (a) 'A Vindication of the Rights of Women' 16. The final version of Wordsworth's 'The
(1792), written by the 18th century British philosopher Prelude' appeared in :
Mary Wollstonecraft, is one of the earliest works of (a) 1798 (b) 1806
feminist philosophy. (c) 1850 (d) 1860
12. Which of the following books by Jonathan Ans : (c) “The Prelude or Growth of a poet’s Mind"; is
Swift is a religious allegory?
an autobiographical poem in blank verse by the English
(a) The Battle of the Books poet William Wordsworth intended as the introduction
(b) A Modest Proposal to the more philosophical 'Recluse', which Wordsworth
(c) Gulliver's Travels never finished. 'The Prelude' is an extremely personal
(d) A Tale of a Tub and revealing work on the details of Wordsworth's life.
Ans : (d) “A Tale of a Tub” was the first major work 17. "To Suffer woes which Hope thinks infinite" is
written by Jonathan Swift arguably his most difficult
written by :
satire and perhaps his most masterly. The Tale is a
(a) Shelley (b) Wordsworth
prose parody divided into sections each delving into the
(c) Keats (d) Byron
morals and ethics of English.
Ans : (a) Percy Bysshe Shelley gives an inspirational
13. Which of the following is a "Visionary" work
quote, talking about the value of Love. “To suffer woes
by William Blake? which Hope thinks infinite; To forgive wrongs darker
(a) The Song of Los than death or night; To defy power", these lines occur in
(b) Songs of Experience “Prometheus Unbound.”
(c) Poetical Sketches
18. "A thing of beauty is a joy for ever" occurs in :
(d) The Vision of the Daughters of Albion
(a) "Ode on a Grecian Urn"
Ans : (b) 'Songs of Innocence and of Experience' is
(b) "Ode to Autumn"
an illustrated collection of poems by William Blake. It
(c) "Ode to Psyche"
appeared in two phases. A few first copies were printed
and illuminated by William Blake himself in 1789. (d) "Endymion"
“Songs of Experience” is a poetry collection of 26 Ans : (d) “Endymion” is a poem by John Keats first
poems forming the second part of William Blake’s published in 1818. It begins with the line “A thing of
'Songs of Innocence and Experience'. The poem was beauty is a joy for ever.” It is written in rhyming
published in 1794. couplets in iambic pentameter.

UGC NET/JRF English Paper II, December 2004 5 YCT


19. Which of the following novels is a satire on the 24. The novel ' Mary Barton' is written by :
Gothic novel? (a) Mrs. Gaskell (b) George Eliot
(a) Pride and Prejudice (c) Emily Bronte (d) Dickens
(b) Emma Ans : (a) “Mary Barton” is the first novel by English
(c) Sense and Sensibility author Elizabeth Gaskell, published in 1848. The story
(d) Northanger Abbey is set in the English city of Manchester between 1839
Ans : (d) “Northanger Abbey” was the first of Jane and 1842, and deals with the difficulties faced by the
Austen’s novels to be completed for publication, though Victorian lower class.
she had previously made a start on “Sense and
25. The line "Poetry is a criticism of life" occurs in :
Sensibility” and “Pride & Prejudice.” Most literary
critics refer to “Northanger Abbey” as Jane Austen’s (a) Culture and Anarchy (b) Modern Painters
'Gothic Parody' because it satirizes the form and (c) The Study of Poetry (d) Sartor Resartus
conventions of the Gothic novels that were popular Ans : (c) 'The Study of poetry' (1880) is written by
during the time when Austen wrote this. Matthew Arnold. It is Arnold’s most famous piece of
20. Who distinguished between "the literature of literary criticism. In this work, Arnold is fundamentally
Knowledge" and "the literature of Power"? concerned with poetry’s “high destiny.”
(a) Coleridge (b) De Quincey 26. 'Martha Quest' was written by :
(c) Hazlitt (d) Lamb (a) Jean Rhys (b) Doris Lessing
Ans : (b) Thomas De Quincey, “The Literature of (c) Iris Murdoch (d) Nadine Gordimer
Knowledge & the Literature of Power,” first Ans : (b) “Martha Quest” (1952) is the second novel
published in the North British Review in August, 1848 of British Nobel Prize in Literature winner Doris
as part of a critical essay on Alexander Pope. Lessing. Martha Quest is the main character of five
21. Who among the following Victorian poets is the semi-autobiographical novels called 'The Children of
most sensitive to the conflict between the old Violence' series, which traces Martha’s life from
and the new? girlhood to middle age.
(a) Tennyson (b) Rossetti
27. The term "Stream of Consciousness" was
(c) Browning (d) Swinburne
taken from the book :
Ans : (a) Alfred Tennyson (6 Aug, 1809- 6 Oct, 1892)
(a) The Human Mind
was a Poet Laureate of Great Britain and Ireland during
much of Queen Victoria’s reign and remains one of the (b) The Principles of Psychology
most popular British Poets. (c) The Mind of Man
22. Under the Greenwood Tree is written by: (d) Modes of Human Behaviour
(a) Mrs. Gaskell (b) George Eliot Ans : (b) In Literary criticism, stream of consciousness,
(c) Thomas Hardy (d) Emily Bronte also known as interior monologue, is a narrative mode
Ans : (c) "Under the Greenwood Tree, A Rural Painting that depicts the multitudinous thoughts and feelings
of the Dutch School" is a novel by Thomas Hardy, which pass through the mind. The term was coined by
published anonymously in 1872. It was Hardy’s second William James in 1890 in his “Principles of
published novel, the last to be printed without his name, Psychology.”
and the first of his great series of Wessex Novels. 28. G. S. Fraser's 'The Golden Bough' focuses on :
23. The Office of Circumlocution occurs in: (a) Images (b) Metaphors
(a) David Copperfield (c) Symbols (d) Archetypes
(b) Bleak House Ans : (c) “The Golden Bough: A Study in
(c) Great Expectations Comparative Religion” (1890) is a wide ranging
(d) Hard Times comparative study of mythology and religion written by
Ans : (b) “Bleak House” was the ninth novel from the Scottish anthropologist Sir James George Frazer
Charles Dickens intended to illustrate the evils caused (1854-1941). It attempts to define the shared elements
by long, drawn-out legal cases in the Courts of of religious belief and scientific thought, discussing
Chancery. The novel is considered one of Dickens' fertility rites, human sacrifices, the dying god, the
finest work although has not been his most popular. scapegoat and many other symbols.

UGC NET/JRF English Paper II, December 2004 6 YCT


29. Arthur Miller's 'Death of a Salesman' relies for Ans : (d) “Half a Life” is a 2001 novel by Nobel
its tragic seriousness on the fate of : Laureate V.S. Naipaul published by Alfred A Knop.
(a) Willy Loman (b) Estragon The novel is set in India, Africa & Europe. “Half a
(c) Vladimir (d) Lucky Life” was long listed for the Man Booker Prize.
Ans : (a) Willy Loman, the main character in “Death 35. Which of the following books by Salman
of a Salesman” is a complex and fascinating tragic Rushdie refers to the 15th Century Spain as a
character. He is a man struggling to hold onto what starting point?
dignity he has left in a changing society that no longer (a) Haroun and the Sea of Stories
values the ideals he grew up to believe in.
(b) The Moor's Last Sigh
30. The character Leopold Bloom makes an (c) Shame
appearance in the novel : (d) Grimus
(a) The Sound and the Fury Ans : (b) “The Moor’s Last Sigh” is the fifth novel by
(b) Ulysses Salman Rushdie, published in 1995. The title is taken
(c) To the Lighthouse from the story of Boabdil (Abu Abdullah Muhammad)
(d) The Europeans the last Moorish King of Granada in the 15th century
Ans : (b) “Ulysses” is a modernist novel by Irish writer Spain and tells about the exclusion of Jews & Muslims
James Joyce. It was first serialised in parts in the from Spain and other events.
American Journal “The Little Review” from March 36. 'Who's Afraid of Virginia Woolf' is written by :
1918 to December 1920 and then published in its
entirety by Sylvia Beach in February 1922, in Paris. (a) Arthur Miller (b) Eugene O'Neil
(c) Edward Albee (d) Tennessee Williams
31. Who of the following authors represents the Sri
Lankan diaspora? Ans : (c) 'Who’s Afraid of Virginia Woolf?' is a 1962
(a) Cyril Dabydeen (b) Michael Ondaatje play by Edward Albee. It examines the breakdown of
(c) Arnold H. Itwaru (d) M.G. Vassanji the marriage of a middle aged couple, Martha &
George.
Ans : (b) Philip Michael Ondaatje is a Sri Lankan born
Canadian novelist and poet. He won the Booker Prize 37. Imamu Amiri Baraka is :
for his novel 'The English Patient' (1992). (a) A Caribbean writer
32. Australian aborigines receive a sympathetic (b) An American writer
treatment in : (c) An Arab writer
(a) Les Murray (b) Gwen Harwood (d) A Sri Lankan writer
(c) Judith Wright (d) A.D. Hope Ans : (b) Amiri Baraka, formerly known as LeRoi
Ans : (c) Judith Wright (31 May 1915-25 June 2000) Jones and Imamu Amiri Baraka, was an African-
was an Australian poet, environmentalist and American writer of poetry, drama, fiction, essays and
campaigner for Aboriginal land rights. music criticism.
33. Margaret Atwood's 'Survival' makes a case for : 38. 'The Miscellany' was published from :
(a) Canadian literary studies (a) Sahitya Akademi
(b) Canadian nationalism (b) The Writers Workshop
(c) The Future of Canadian Literature
(c) PEN
(d) The Past of Canadian Literature
(d) Dhwanyalok
Ans : (a) “Survival: A Thematic Guide to Canadian
Ans : (b) 'The Miscellany' published by Writer's
Literature” is a survey of Canadian Literature by
Workshop, Calcutta is a bimonthly publication of
Margaret Atwood, one of the best-known Canadian
creative writing from 1968-1980.
authors. It was first published by House of Anansi in
1972. 39. Who of the following writers recreates the life
34. V.S. Naipaul's latest book is : of the Yoruba/Ibo community?
(a) 'The Mystic Masseur' (a) Derek Walcott
(b) 'A Bend in the River' (b) Wole Soyinka
(c) 'Among the Believers' (c) Chinua Achebe
(d) 'Half a Life' (d) Okot
UGC NET/JRF English Paper II, December 2004 7 YCT
Ans : (b) Akinwande Oluwole “Wole” Babatunde (c) The Metaphysical Poets
Soyinka is a Nigerian playwright and poet. He was (d) Hamlet
awarded the 1986 Nobel Prize in Literature, the firstAns : (d) “Hamlet and His Problems” is an essay
African to be honored in that category. written by Eliot in 1919 that offers a critical reading of
40. Who of the following White female authors are Hamlet. The essay first appeared in Eliot’s 'The Sacred
sympathetic to the cause of the Blacks? Wood' essays on poetry & criticism in 1920.
(a) Margaret Drabble (b) Nadine Gordimer 46. Sprung Rhythm is an example of :
(c) Muriel Spark (d) Jean Rhys (a) Verse (b) Syllable
Ans : (b) Nadine Gordimer (1923-2014) was a South (c) Stress (d) Meter
African writer, political activist & recipient of the 1991
Ans : (a) Sprung Rhythm is a poetic rhythm designed
Nobel Prize in Literature.
to imitate the rhythm of natural speech. It is constructed
41. New Criticism considers text as a : from feet in which the first syllable is stressed & may
(a) Cultural construct be followed by a variable number of unstressed
(b) Historical construct syllables.
(c) Linguistic construct
47. "More is thy due than more than all can pay"
(d) Autotelic
is an example of :
Ans : (d) New Criticism was a formalist movement in
(a) Weak-ending (b) Inversion
literary theory that dominated American literary
criticism, in the middle decades of the 20th century. It (c) Alexandrine (d) Extra Syllable
emphasized close reading, particularly of poetry to Ans : (b) Inversion also known as 'anastrophe' is a
discover how a work of literature functioned as self literary technique in which the normal order of words is
contained, self-referential aesthetic object. reversed in order to achieve a particular effect of
42. 'Mythologies' was written by: emphasis or meter, example-
(a) Roland Barthes (b) Jacques Derrida What a beautiful picture it is!
(c) Homi K. Bhabha (d) Ernest Dowson Note : Inversion was a common practice in the days of
Ans : (a) “Mythologies” is a 1957 book by William Shakespeare.
Roland Barthes. It is a collection of essays taken from 48. Unrhymed metrical composition consisting of
his letters, 'nouvelles' examining the tendency of five iambic measures in each line is called :
contemporary social value systems to create modern (a) Rhyme royal (b) Run-on-lines
myths.
(c) Blank verse (d) Spenserian stanza
43. The word "Catharsis" signifies :
Ans : (c) Blank verse is poetry written with regular
(a) Pontification (b) Personification metrical but unrhymed lines, almost always in iambic
(c) Purgation (d) Publication pentameter.
Ans : (c) Catharsis is the purification and purgation of 49. Verse stories dealing with chivalry, Knight,
emotions especially pity and fear through art or any
errantry, enchantments, and love are known as :
extreme change in emotion that results in renewal &
restoration. (a) The epic (b) The ballad
(c) The ode (d) The metrical romances
44. The rejection of "Universalism" is a mark of :
Ans : (d) A Metrical romance is otherwise known as
(a) Deconstruction (b) New Historicism
romantic poetry. The poem tells a story in verse form
(c) Structuralism (d) Postcolonial criticism
and depicts the adventures of romantic poetry.
Ans : (a) Deconstruction is a critical outlook concerned
50. "He is a citizen of no mean city" is an example
with the relationship between text and meaning. Jacques
Derrida's 1976 work 'Of Grammatology' introduced the of :
majority of ideas influential within deconstruction. (a) Periphrasis (b) Tautology
45. Eliot's theory of "objective correlative" (c) Prolepsis (d) Litotes
appeared in his essay entitled : Ans : (d) Litotes is an ironical understatement in which
(a) Three voices of Poetry an affirmative is expressed by the negative of its
(b) Tradition and the Individual Talent contrary example- “Not a bad singer”.

UGC NET/JRF English Paper II, December 2004 8 YCT


UGC NET/JRF Exam, June-2005
ENGLISH
Solved Paper-II
Note: This paper contains fifty (50) multiple Ans: (c) “The Fortunes and Misfortunes of the Famous
choice questions, each carrying two (2) marks. Moll Flanders” is a novel by Daniel Defoe, first
Attempt all of them. published in 1722. It purports to be the true account of
1. The Nun’s Priest’s Tale had its origin in : the life of the eponymous Moll, detailing her exploits
(a) The French Roman de Renart from birth until old age.
(b) The Italian Boccaccio's 'Teseide'
7. The expression “ancestral voices prophesying
(c) The English John Gower’s 'Confessio Amantis'
war” occurs in :
(d) The German Goethe’s 'Faust'
(a) ‘Kubla Khan’
Ans: (a) “The French Roman de Renart” is the main (b) ‘Frost at Midnight’
character in a literary cycle of allegorical Dutch, (c) 'Christabel'
English, French and German fables. Geoffrey Chaucer (d) 'Rime of the Ancient Mariner'
used ‘Reynard’ material in Canterbury Tales; in “The
Nun’s Priest’s Tale”, Reynard appears as “Rossel” and Ans: (a) “Kubla Khan” is a poem written by Samuel
an ass as “Brunel”. Taylor Coleridge, in 1797 and published in 1816.
2. The First Folio of Shakespeare’s plays appeared 8. The posthumously published novel of Jane
in : Austen is :
(a) 1664 (b) 1631 (a) Sense and Sensibility (b) Mansfield Park
(c) 1623 (d) 1650 (c) Emma (d) Northanger Abbey
Ans: (c) “The First Folio” is the first collected edition Ans: (d) Jane Austen’s first major novel “Northanger
of William Shakespeare’s plays, published in 1623, Abbey” was written in 1798-99. Her brother
seven years after his death. He wrote around 37 plays, Henry Austen published the novel, after her death in
36 of which are contained in the “First Folio.” 1817.
3. Restoration comedy begins with: 9. Carlyle’s 'Sartor Resartus' means :
(a) Congreve (b) Sheridan (a) Satan’s story retold
(c) Dryden (d) Etherege (b) The tailor retailored
Ans: (a) Restoration comedy refers to English comedies (c) I know not where
written and performed in the restoration period from (d) a set of elegant clothes
1660 to 1710. During 1690s the “softer” comedies of Ans: (b) 'Sartor Resartus' (meaning ‘The tailor
William Congreve reflected mutating cultural retailored’) is an 1836 novel by Thomas Carlyle. The
perceptions and great social change. novel purports to be a commentary on the thought and
4. The author of 'The Progress of the Soul' is : early life of a German philosopher called Diogenes
(a) John Bunyan (b) John Donne Teufelsdrockh.
(c) Henry Vaughan (d) Richard Crashaw 10. The character not created by Hardy is :
Ans: (b) The author of “Of the Progress of the Soul” (a) Sue Bridehead (b) Bathsheba Everdene
is John Donne, written in 1896. Donne was an English (c) Betsy Trotwood (d) Thomasin
poet. He is considered the pre-eminent representative of Ans: (c) Betsy Trotwood is a fictional character from
the metaphysical poets. Charles Dickens’s 1850 novel 'David Copperfield'.
5. Dr. Johnson’s 'The Lives of The Poets' is an
11. The poet who described poetry as “Inspired
example of :
mathematics” is :
(a) Psychological criticism
(a) T.S. Eliot (b) Hopkins
(b) Biographical criticism
(c) Archibald Macleish (d) Ezra Pound
(c) Historical criticism
(d) Archetypal criticism Ans: (d) “Poetry is a sort of inspired mathematics,
Ans: (b) “Lives of the Most Eminent English Poets” which gives us equations, not for abstract figures,
(1779-81) comprised short biographies of 52 poets most triangles, squares, and the like, but for the human
of whom lived during 18th century. emotions.” - Ezra Pound.
6. The picaresque novel with a female picaroon is: 12. The woman character who is an artist by
(a) Tom Jones (b) Clarissa profession in Virginia Woolf’s 'To the
(c) Moll Flanders (d) Amelia Lighthouse' is :
UGC NET/JRF English Paper II, June 2005 9 YCT
(a) Lily Briscoe (b) Mrs. Ramsay (a) Antithesis (b) Bathos
(c) Mrs. Dalloway (d) Miriam (c) Tautology (d) Litotes
Ans: (a) Lily Briscoe is a painter and one of the central Ans: (a) In Antithesis, one idea or word is set against
character’s in the novel “To the Lighthouse” (1927) by another for the sake of emphasis. Examples- “God made
Virginia Woolf. Lily represents Woolf’s idea of an the country and man made the town,” “Prosperity gains
artist, who mingles “masculine” rationality with friends, adversity tries them.”
“feminine” sympathy. 20. A metre in which an unaccented syllable
13. The poet who said, “My poems are not about precedes the accented is called :
violence, but vitality”, is: (a) anapaestic (b) dactylic
(a) Philip Larkin (b) Ted Hughes (c) catalectic (d) iambic
(c) C.D. Lewis (d) Thom Gunn Ans: (d) Anapaest: two unaccented syllables followed
by one stressed syllable.
Ans: (b) The major theme of Ted Hughes’s poetry is of
course man, that is, the question of man's existence. He Dactylic: one stressed syllable followed by two
unstressed syllables.
said, 'My Poems are not about violence but vitality'.
Catalectic: a line lacking one syllable in the last foot.
14. Pinter’s 'Care Taker' can be called a :
Iambic: an unaccented syllable precedes (comes before)
(a) comedy of manners (b) comedy of menace
an accented syllable.
(c) comedy of errors (d) comedy of humours
Choose the correct chronological sequence in
Ans: (b) “The Care Taker” (1960) is a play in three question numbers 21-30:
acts by Pinter. Harold Pinter usually used comedy of 21. (a) Northanger Abbey, Pride and Prejudice, Sense
menace in his plays. and Sensibility, Mansfield Park.
15. Toni Morrison used male narrator for the first (b) Mansfield Park, Sense and Sensibility,
time in : Northanger Abbey, Pride and Prejudice
(a) Song of Solomon (b) Tar Baby (c) Pride and Prejudice, Northanger Abbey,
(c) Jazz (d) The Bluest Eye Mansfield Park, Sense and Sensibility
Ans: (a) “Song of Solomon” (1977) is Toni Morrison's (d) Sense and Sensibility, Pride and Prejudice,
first novel to be written through a male protagonist's Mansfield Park, Northanger Abbey
view. It tells us how it is like to be a young black male Ans: (d) The correct chronological sequence is:
living in a white male-dominant society through the ♦ “Sense and Sensibility” - Published in 1811
story of Macon ‘Milkman’. ♦ “Pride & Prejudice” - Published in 1813
16. The author of 'The Hungry Tide' is : ♦ “Mansfield Park” - Published in 1814
(a) Vikram Seth (b) Shobha De ♦ “Northanger Abbey” - Published in 1817
(c) Amitav Ghosh (d) Upamanyu Chatterjee 22. Shakespeare criticism by :
Ans: (c) 'The Hungry Tide' (2004) is the sixth novel by (a) Spurgeon- T.S. Eliot- Stephen Greenblatt-
Indian-born author, Amitav Ghosh. It won the Bradley
Crossword Book Award for fiction. (b) Bradley- Spurgeon- T.S. Eliot- Stephen
Greenblatt
17. The soul of tragedy, according to Aristotle is:
(c) T.S. Eliot- Stephen Greenblatt- Bradley-
(a) Thought (b) Character Spurgeon
(c) Plot (d) Spectacle (d) Stephen Greenblatt- Bradley- T.S. Eliot-
Ans: (c) According to Aristotle, tragedy has six main Spurgeon
elements: Plot, Character, Diction, Thought, Spectacle Ans: (b) The correct chronological sequence is:
(scenic effect), and Song (music). Aristotle also writes: ♦ A.C. Bradley : ‘Shakespearean Tragedy’ (1904)
“The plot then is the first principle and as if it were the ♦ T.S. Eliot : “Hamlet and his Problems” (1919)
soul of a tragedy.”
♦ Caroline Spurgeon : ‘Shakespeare’s Imagery’ (1935)
18. The discussion of Fabula/Syuzhet occurs in : ♦ Stephen Greenblatt : “On Poetics of Culture and the
(a) New criticism (b) Deconstruction Interpretation of Shakespeare.” (1987)
(c) Structuralism (d) Formalism 23. (a) Pre-Raphaelite-Brotherhood, Oxford
Ans: (d) Originating in Russian Formalism, the terms Movement, Movement Poetry, Imagism
'fabula' and 'sujet' (Syuzhet) describes two different (b) Oxford Movement, Pre-Raphaelite
aspects of the timeline of event in a narrative. The Brotherhood, Imagism, Movement Poetry
“fabula” is the raw material of a story and “Syuzhet” the (c) Imagism, Movement Poetry, Pre-Raphaelite
way a story is organised. Brotherhood, Oxford Movement
19. “United we stand, divided we fall” is an example (d) Movement-Poetry, Pre-Raphaelite
of : Brotherhood, Oxford Movement, Imagism
UGC NET/JRF English Paper II, June 2005 10 YCT
Ans: (b) The correct chronological sequence is: Ans: (d) The correct chronological sequence is:
♦ Oxford Movement -1833-45 ♦ Toru Dutt - (1856-1877)
♦ Pre - Raphaelite Brotherhood - 1848 ♦ Sarojini Naidu - (1879-1949)
♦ Imagism - 1912 ♦ Kamala Das - (1934-2009)
♦ Movement Poetry - 1954 ♦ Meena Alexander - (1951)
24. (a) Closet drama, Epic Theatre, Theatre of the 28. (a) Jude, Lady Havisham, Dorothea, Mrs. Morel
Absurd, Portable Theatre (b) Dorothea, Mrs. Morel, Jude, Lady Havisham
(b) Epic Theatre, Portable Theatre, Theatre of the (c) Dorothea, Jude, Mrs. Morel, Lady Havisham
Absurd, Closet drama (d) Lady Havisham, Dorothea, Jude, Mrs. Morel
(c) Portable Theatre, Closet drama, Epic Theatre, Ans: (d) The correct chronological sequence is :
Theatre of the Absurd
♦ Miss Havisham – Charles Dickens’ 'Great
(d) Theatre of the Absurd, Portable Theatre, Closet
Expectations' (1861)
drama, Epic Theatre
♦ Dorothea – George Eliot’s 'Middlemarch' (1871-72)
Ans: (a) The correct chronological sequence is: ♦ Jude – Thomas Hardy’s 'Jude the Obscure' (1895)
♦ Closet drama - Late 18th century
♦ Mrs. Morel – D. H. Lawrence’s 'Sons and Lovers'
♦ Epic Theatre - 1924-27 (1913)
♦ Theatre of Absurd - 1942 29. (a) The Well-Wrought Urn, The Verbal Icon,
♦ Portable Theatre - Late 1960s and 1970s Theory of Literature, Literary Theory : An
Introduction
25. (a) Thomas Nashe, Ben Jonson, Kyd, Marlowe (b) The Well-Wrought Urn, Theory of Literature,
(b) Ben Jonson, Thomas Kyd, Marlowe, Thomas The Verbal Icon, Literary Theory: An
Nashe Introduction
(c) Thomas Kyd, Marlowe, Thomas Nashe, Ben (c) The Verbal Icon, The Well-Wrought Urn,
Jonson Literary Theory: An Introduction, Theory of
(d) Marlowe, Thomas Nashe, Thomas Kyd, Ben Literature
Jonson (d) Literary Theory: An Introduction, The Well-
Ans: (c) The correct chronological sequence is: Wrought Urn, Theory of Literature, The Verbal
♦ Thomas Kyd - (1558-1594) Icon
♦ Christopher Marlowe - (1564-1593) Ans: (b) The correct chronological sequence is:
♦ Thomas Nashe - (1567-1601) ♦ “The Well Wrought Urn: Studies in the structure of
♦ Ben Jonson - (1572-1637) poetry is a 1947 collection of essays by Cleanth
26. (a) Essay on Dramatic Poesy, Areopagitica, Urn Brooks.
Burial, Religio Medici ♦ “The Verbal Icon”: Studies in the meaning of poetry,
(b) Areopagitica, Urn Burial, Religio Medici, published in 1954, written by William K. Wimsatt.
Essay on Dramatic Poesy ♦ “Theory of Literature”: is a book on literary
(c) Religio Medici, Areopagitica, Urn Burial, scholarship by Rene Wellek, published in 1948.
Essay on Dramatic Poesy ♦ “Literary theory : An Introduction” - by Terry
(d) Urn Burial, Essay on Dramatic Poesy, Eagleton and published in 1983.
Areopagitica, Religio Medici 30. Nobel Prize winners in Literature:
Ans: (c) The correct chronological sequence is: (a) Seamus Heaney, T.S. Eliot, Nadine Gordimer,
♦ 'Areopagitica' (1644) - John Milton W.B. Yeats
♦ 'Urn Burial' (1658) - Sir Thomas Browne (b) W.B. Yeats, T.S. Eliot, Nadine Gordimer,
♦ 'Religio Medici' (1643) - Thomas Browne Seamus Heaney
(c) T.S. Eliot, Seamus Heaney, W.B. Yeats,
♦ 'Essay on Dramatic Poesy' (1668) - John Dryden
Nadine Gordimer
27. (a) Kamala Das, Sarojini Naidu, Toru Dutt,
(d) Nadine Gordimer, Seamus Heaney, W.B.
Meena Alexander
Yeats, T.S. Eliot
(b) Meena Alexander, Toru Dutt, Sarojini Naidu,
Kamala Das Ans: (b) The correct chronological sequence is:
(c) Sarojini Naidu, Kamala Das, Meena ♦ W.B. Yeats - 1923
Alexander, Toru Dutt ♦ T.S Eliot - 1948
(d) Toru Dutt, Sarojini Naidu, Kamala Das, Meena ♦ Nadine Gordimer - 1991
Alexander ♦ Seamus Heaney - 1995
UGC NET/JRF English Paper II, June 2005 11 YCT
Select the matching pair in question numbers 37. (a) T.S. Eliot - The Birthday Party
31 to 40 : (b) Osborne - The Entertainer
31. (a) 'Idylls of the King' - Browning (c) Bernard Shaw - Luther
(b) 'The Diverting History of John Gilpin' - (d) Tom Stoppard - Lear
William Cowper Ans: (b) 'The Entertainer' is a three-act play by John
(c) 'The Tower' - T.S. Eliot
Osborne, first produced in 1957.
(d) 'The Fall of Hyperion' - Shelley
38. (a) Periodical Essays - Bacon
Ans: (b) The correct pair is option (b).
(b) Confessional Poetry - Ted Hughes
♦ 'Idylls of the King' (1859) - Alfred Lord Tennyson (c) Science Fiction - David Lodge
♦ ‘The Diverting History of John Gilpin' (1782) - (d) Pre-Raphaelites - William Morris
William Cowper
Ans: (a) The correct pair is option (a).
♦ 'The Tower' (1928) - William Butler Yeats
♦ 'The Fall of Hyperion' - John Keats ♦ Periodical Essays - Bacon
Note: There is also a “The Fall of Hyperion” novel ♦ Confessional Poetry - Sylvia Plath
written by Dan Simmons in 1990. ♦ Science Fiction - Hugo Gernsback
32. (a) 'Hard Times' - Psychological novel ♦ Pre-Raphaelites - William Holman Hunt
(b) 'To The Light house' - Picaresque novel 39. (a) Nissim Ezekiel - Persian
(c) 'The Castle of Otranto'- Gothic novel (b) Gieve Patel - Gujarati
(d) 'Wuthering Heights' - Historical novel (c) Dilip Chitre - Sanskrit
Ans: (c) “The Castle of Otranto” is a 1764 novel by (d) Adil Jussawalla - Urdu
Horace Walpole. It is generally regarded as the first Ans: (a) Nissim Ezekiel (1924-2004) was an Indian
gothic novel. Jewish poet. He was a foundational figure in
33. (a) Emily Bronte - Yorkshire Moors Post-Colonial India’s literary history, specifically for
(b) Hardy - Scotland Indian writers.
(c) Walter Scott - Ireland 40. (a) Pearl - The Scarlet Letter
(d) Mark Twain - Yoknapatawfa
(b) Raka - The God of Small Things
Ans: (a) The Bronte Country or Moors is a name given (c) Raphael - The Great Expectations
to an area of South Pennine hills, west of Bradford in (d) Pip - Fire on the Mountain
West Yorkshire, England.
Ans: (a) In Nathaniel Hawthorne's “The Scarlet
34. (a) Surrealism - Tristan Tzara
Letter” (1850), Pearl is a symbol of sin and adultery.
(b) Imagism - Spender
(c) Naturalism - Yeats 41. The assertion, “We had a very restful holiday,”
(d) Magic Realism - Gabriel Garcia Marquez implies :
(a) We didn’t exert ourselves
Ans : (d) It is often said that the works of Colombian
novelist Gabriel Garcia Marquez are examples of (b) We did nothing
“magic realism” fiction that integrates elements of (c) We were very lazy
fantasy into otherwise realistic settings. (d) We had a very dull time
35. (a) Victor Shklovsky - Carnivalesque Ans: (a) We had a very restful holiday implies that we
(b) Stanley Fish - Aphasia didn’t exert ourselves.
(c) Hjelmslev - Glossematics 42. “The progress of an artist is a continual self
(d) Roland Barthes - Affective Stylistics sacrifice, continual extinction of personality.
Ans: (b) The correct pair is (b) Stanley Fish - Aphasia. This assertion implies”
He is an American literary critic associated with (a) Merely by a continual extinction of personality
Reader-response criticism. an artist is sure to make progress
36. (a) Bessie Head - New Zealand (b) An artist is likely to make progress through
(b) Derek Walcott - South Africa continual self sacrifice and extinction of
(c) A.D. Hope - Australia personality
(d) Ondaatje - Nigeria (c) Continual self sacrifice and extinction of
Ans: (c) The correct pair is (c) A.D. Hope - Australia. personality will undermine the progress of the
♦ Bessie Head (1937-1986) - Botswana’s writer artist
♦ Derek Walcott (b. 1930) - Saint Lucia's poet & (d) An artist must have a personality to create art
playwright. Ans: (b) The above line implies that an artist is likely to
♦ A.D. Hope (1907-2000) - Australian writer make progress through continual self sacrifice and
♦ Ondaatje (b. 1943) - Sri Lankan writer extinction of personality.
UGC NET/JRF English Paper II, June 2005 12 YCT
43. “The best poetry will be found to have a power In a broader sense, “Poetic License” is applied not
of forming, sustaining and delighting us”. This only to language, but to all the ways in which poets and
assertion implies : other literacy authors are held to be free to violate, for
(a) Poetry has multiple functions to perform special effects, the ordinary norms and only of common
(b) Poetry is more useful than other arts discourse but also but also of literal and historical truth,
(c) All other arts including poetry have their including the devices of metre and rhyme, the recourse to
limitations
literary conventions, and representation of fictional
(d) Poetry has no role to play
characters and events.
Ans: (a) The multiple functions of poetry are implied in
46. ‘Poetic license’ means:
the above lines.
(a) liberty with diction, alone
44. “Human beings and especially human beings as (b) liberty with diction and norms of common
an integral part of a social organisation are
discourse
regarded as primary subject matter of
(c) liberty with historical truth
literature”. This assertion implies:
(a) Human beings alone can be the subject matter (d) liberty with representations of fictional
of literature characters
(b) All living beings-animal and human, contribute Ans: (b) According to the passage option (b) is correct.
towards the creation of literature (Para 2)
(c) Humans as social beings are the nucleus of all 47. ‘Linguistic freedom’ is:
literary exercise (a) freedom with diction, newly-coined words,
(d) Literature transcends the human and the non- syntax
human.
(b) freedom with the use of colloquial language
Ans: (c) Humans as social beings, are the nucleus of all (c) freedom with the use of figurative construction
literary exercise. (d) freedom with literal truth
45. “We must learn to see more, to hear more, to
Ans: (a) According to the passage, Linguistic freedom
feel more”. The assertion implies :
(a) Human beings have only three faculties at their is freedom with diction, newly coined words, syntax.
command to comprehend all knowledge (lines 8-9, Para 1)
(b) A sharpening of three faculties mentioned 48. How do you justify the linguistic freedom taken?
would help human beings to become better. (a) on the basis of scholarship embedded
(c) Only with the combination of all senses, we (b) on the basis of form
may become better (c) on the basis of the success of the effect
(d) Seeing, hearing and feeling are not enough to (d) on the basis of the thematic grandeur
become better human beings.
Ans: (c) According to the passage option (c) is correct.
Ans: (b) The correct implication is option (b). (Para 1, last line)
(46-50) Read the passage below and answer the 49. “Diction” means:
questions that follow based on your understanding of
(a) severity of prose
the passage:
(b) devices of metre and rhyme
John Dryden in the late seventeenth century defined
(c) poetic license
poetic license as “The liberty which poets have assumed
to themselves, in all ages, of speaking things in verse (d) syntax and word order
which are beyond the severity of prose.” In its most Ans: (d) Diction means syntax and word order. (line 8-
common use the term is confined to diction alone, to 9, Para 1)
justify the poet’s departure from the rules and 50. “Poetic license” applies to :
conventions of standard spoken and written prose in (a) Poets alone
matters such as syntax, word order, the use of archaic or (b) All literary authors
newly coined words, and the conventional use of eye- (c) Dramatists only
rhymes. The degree and kinds of linguistic freedom
(d) Epic writers only
assumed by poets have varied according to the
conventions of each age, but in every case the Ans: (b) 'Poetic license' applies to all literary authors, in
justification of the freedom lies in the success of the all ages who have assumed the liberty of speaking
effect. freely for special effects. (Para 2)
UGC NET/JRF English Paper II, June 2005 13 YCT
UGC NET/JRF Exam, December-2005
ENGLISH
Solved Paper-II
Note : This paper contains fifty (50) multiple 6. The main idea of 'The Dunciad' was taken from:
choice questions, each question carrying two (2) (a) The Hind and the Panther
marks. Attempt all of them. (b) Religio Laici
1. Chaucer’s 'The Knight’s Tale' is a high romance (c) Mac-Flecknoe
told in : (d) The Medal
(a) rhyme royal (b) terza rima Ans: (c) ‘The Dunciad’ is a satire by Alexander Pope
(c) heroic libre (d) verse libre
published in three different versions at different times,
Ans: (c) 'The Knight’s Tale' is the first tale from during 1728, 1729 and 1742. The main idea of the
Geoffrey Chaucer’s 'The Canterbury Tales'. The story Dunciad was taken from 'Mac-Flecknoe'.
introduces various typical aspects of knighthood such as
7. The character of the leech gatherer appears in :
courtly love and ethical dilemmas. The story is written
(a) The Recluse
in iambic pentameter end-rhymed couplets. Libre means
(b) The Prelude Book I
rhyme or metre in a musical composition.
(c) Laodamia
2. Marlowe’s first original work was :
(d) Resolution and Independence
(a) Tamburlaine the Great
(b) The Tragical History of D. Faustus Ans: (d) “Resolution and Independence” is a lyric
(c) The Jew of Malta poem of the English Romantic poet William
(d) The troublesome Raigne and Lamentable Wordsworth, composed in 1802 and published in 1807.
Death of Edward the second 8. 'Table-Talk' is a collection of essays by :
Ans: (a) “Tamburlaine the Great” is a play in two (a) Lamb (b) Hunt
parts by Christopher Marlowe. It is loosely based on the (c) Hazlitt (d) De Quincey
life of the Central Asian emperor, Timur. Written in Ans: (c) “Table–Talk" is a collection of essays by the
1587 or 1588, the play is a milestone in Elizabethan English cultural critic and social commentator William
public drama. Hazlitt. It was originally published as two volumes, the
3. Marvell pays his homage to the Protector and a first of which appeared in April 1821. The essay deals
tribute to the royal dignity of Charles I in : with topics such as art, literature and philosophy.
(a) “The Garden” 9. Carlyle’s 'Sartor Resartus' was written under
(b) “The Picture of T.C.” the influence of :
(c) “Bermudas” (a) Italian romance (b) German romance
(d) “Horatian ode upon Cromwell’s Return from (c) French romance (d) British romance
Ireland”
Ans: (b) “Sartor Resartus” is an 1836 novel by
Ans: (d) Marvell begins the poem “Horatian Ode Thomas Carlyle, first published as a serial in 1833-34 in
upon Cromwell’s Return from Ireland” by presenting
Fraser’s Magazine. The novel purports to be a
Oliver Cromwell, Lord Protector of the Commonwealth,
commentary on the thought and early life of a German
as a “forward youth”, who must once again engage in
philosopher called Diogenes Teufelsdrockh.
military conflict and achieve glory.
4. 'The Life and Death of Mr. Badman' was 10. The image of the Neptune taming the sea horse
written by: appears in :
(a) Sir Henry Wotton (b) John Bunyan (a) “Abt Vogler” (b) “Prospice”
(c) Jeremy Taylor (d) Richard Baxter (c) “Andrea del Sarto” (d) “My Last Duchess”
Ans: (b) ‘The Life and Death of Mr. Badman’ is a Ans: (d) “My Last Duchess”, a poem by Robert
1680 book by John Bunyan. It was designed as a Browning, frequently anthologized, is an example of the
companion to 'The Pilgrim’s Progress'. dramatic monologue. It first appeared in 1842 in
5. Dr. Johnson’s 'A Dictionary of the English Browning’s 'Dramatic Lyrics'. The poem is written in
Language' was published in : 28 rhymed couplets of iambic pentameter.
(a) 1755 (b) 1756 11. T.S. Eliot’s 'The Waste Land' is dedicated to II
(c) 1757 (d) 1758 miglior fabro (“The better Craftsman”) :
Ans: (a) Published on 15 April 1755 and written by (a) Ezra Pound (b) Baudelaire
Samuel Johnson, 'A Dictionary of the English (c) G.M. Hopkins (d) Dante
Language', sometimes published as 'Johnson’s Ans: (a) Eliot dedicated his poem Waste Land by
Dictionary', is among the most influential dictionaries in writing a letter in which he wrote “for Ezra Pound: iI
the history of the English Language. miglior fabbro”.
UGC NET/JRF English Paper II, December 2005 14 YCT
12. The locale of 'Riders to the Sea' is : 18. The concept of “arche writing” is developed by :
(a) Dublin (b) Aran Island (a) Fish (b) Foucault
(c) Galway (d) Belfast (c) Derrida (d) Paul de Man
Ans: (b) “Riders to the Sea” is a play written by Irish Ans: (c) According to Derrida, “Arche-writing” is a
Literary Renaissance playwright John Millington form of language that can’t be conceptualized within the
Synge, first performed on 25 Feb, 1904. A one act “metaphysics of presence”. It is an original form of
tragedy, the play is set in the Aran Island. language that is not desired from speech, and it is
13. The “Bog” poems are associated with : unhindered by the difference between speech and
(a) Ted Hughes (b) Elizabeth Jennings writing.
(c) Tony Harrison (d) Seamus Heaney 19. A figure of speech in which two terms opposite
in meaning are placed side by side in one phrase
Ans: (d) “North” (1975) is a collection of poems
is known as :
written by Seamus Heaney, who received the 1995 (a) paradox (b) oxymoron
Nobel Prize in Literature. The collection is divided into (c) sarcasm (d) antithesis
two parts of which the first is more symbolic, dealing
with themes such as “the bog bodies of Northern Ans: (b) An 'oxymoron' is a figure of speech that
juxtaposes elements in two words that appear to be
Europe”. “Bog bodies” inspire four poems in this
contradictory, but which contains a concealed point, e.g.
volume: “Bog Queen”, “The Grauballe Man”,
“The same difference”, 'faithful unfaithful', kept him
“Punishment”, and “Strange Fruit”.
'falsely true'.
14. Edward Bond’s 'Bingo' deals with the life of : 20. A stanza of eight iambic pentameters on the
(a) Dryden (b) Shakespeare pattern of ab, ab, ab, cc is known as :
(c) Ben Jonson (d) Marlowe (a) Rhyme royal (b) Ottava rima
Ans: (b) “Bingo, Scenes of Money & Death" is a (c) Tennysonian stanza (d) Spenserian stanza
1973 play by English Marxist playwright Edward Bond. Ans: (b) “Ottava rima” is a rhyming stanza form of
It depicts an aging William Shakespeare at his Italian origin. Originally used for long poems on heroic
Warwickshire home in 1615 and 1616."Bingo" is a themes. It is a form of poetry consisting of stanzas of
political drama heavily influenced by Bertolt Brecht and eight lines of ten or eleven syllables, rhyming
Epic theatre. “abababcc”.
15. Arthur Miller's 'The Death of a Salesman' is Choose the correct chronological sequence in
mainly about : question numbers 21 to 30 :
(a) American dream 21. (a) Love’s Labour’s Lost, Twelfth Night, Othello,
(b) American imperialism The Tempest
(c) American pragmatism (b) Twelfth Night, Love’s Labour’s Lost. The
(d) American transcendentalism Tempest Othello
Ans: (a) “Death of a Salesman” is a 1949 play written (c) Love’s Labour’s Lost, Othello, The Tempest,
by Arthur Miller. It was the recipient of the 1949 Twelfth Night
Pulitzer Prize for Drama. It is widely considered to be (d) Othello, Twelfth Night, Love’s Labour’s Lost.
one of the greatest plays of the 20th century. The Tempest
16. The patient in Michael Ondaatje’s 'The English Ans: (a) The correct chronological sequence of the
Patient' is : plays is–
(a) Almasy (b) Caravaggio ♦ 'Love’s Labour’s Lost' – 1594
(c) Kirpal Singh (d) Hana ♦ 'Twelfth Night' – 1599
Ans: (a) 'The English Patient' is a 1992 novel. The ♦ 'Othello' – 1604
book follows four dissimilar people brought together at ♦ 'The Tempest' – 1611
an Italian Villa during the Italian campaign of World 22. (a) Ralph Roister Doister, Utopia, Astrophel and
War-II, first one is an unrecognizably burned man Stella, Shepherds Calendar
Almasy, the titular patient, presumed to be English, (b) Astrophel and Stella, Ralph Roister Doister,
others being his Canadian army nurse, a Sikh British Shepherds Calendar
Army Sapper and a Canadian thief. (c) Shepherds Calendar, Astrophel and Stella,
17. Mimetic criticism views literary work as : Utopia, Ralph Roister Doister
(a) personalisation (b) depersonalisation (d) Utopia, Ralph Roister Doister, Shepherds
(c) imitation (d) interpretation Calendar, Astrophel and Stella
Ans: (c) Mimesis criticism is a method of interpreting Ans: (d) The correct chronological sequence is–
texts in relation to their literary or cultural models. ♦ 'Utopia' (1516) – Thomas More
Mimesis or imitation was a widely used rhetorical tool ♦ 'Ralph Roister Doister' (1552) – Nicholas Udall
in antiquity up until the 18th century’s romantic ♦ 'Shepherds Calendar' (1579) – Edmund Spenser
emphasis on originality. ♦ 'Astrophel & Stella' (1580) – Philip Sidney
UGC NET/JRF English Paper II, December 2005 15 YCT
23. (a) Sonnet, periodical essay, gothic novel, absurd 27. (a) “Sign, Structure and Play,” "Signs Taken for
play Wonder", “The Death of the Author”, “Two
(b) Gothic novel, periodical essay, sonnet, absurd Uses of Language”
play (b) “Two Uses of Language”, “The Death of the
(c) Periodical essay, gothic novel, absurd play, Author”, "Sign, Structure and play”, "Signs
sonnet Taken for Wonder"
(d) Sonnet, gothic novel, periodical essay, absurd (c) “The Death of the Author”, "Two Uses of
play Language”, “Signs Taken for Wonder”, “Sign,
Structure and Play”
Ans: (a) The correct sequence is–
(d) “Two Uses of Language”, “The Death of the
♦ Sonnet - During 13th century Author”, “Sign, Structure and Play”, "Signs
♦ Periodical Essay - During early 18th century Taken for Wonder"
♦ Gothic novel - 1764 Ans: (b) The correct sequence is–
♦ Absurd play - Late 1950S ♦ “Two uses of Language” (Principles of Literary
24. (a) Stephen Spender, T.S. Eliot, Philip Larkin, Ted Criticism, 1924) - I.A. Richards
Hughes ♦ “Sign, Structure & Play” (1966) - Jacques Derrida
(b) T.S. Eliot, Stephen Spender, Philip Larkin, Ted ♦ “The Death of the Author” (1967) - Roland Barthes
Hughes ♦ “Signs Taken for Wonder” (1985) - Homi K. Bhabha
(c) Philip Larkin, T.S. Eliot, Ted Hughes, Stephen 28. (a) “The Burial of the Dead”, “A Game of Chess”,
Spender “Fire Sermon”, "Death by Water"
(d) T.S. Eliot, Philip Larkin, Ted Hughes, Stephen (b) “A Game of Chess”, “The Burial of the Dead”,
Spender “Fire Sermon”, “Death by Water”
Ans: (b) The correct sequence is– (c) “Fire Sermon”, “The Burial of the Dead”,
♦ T.S. Eliot - 1888-1965 “Death by Water”, “A Game of Chess”
♦ Stephen Spender - 1909-1995 (d) “The Burial of the Dead”, “Fire Sermon”,
♦ Philip Larkin - 1922-1985 “Death by water”, “A Game of Chess”
♦ Ted Hughes - 1930-1998 Ans: (a) The correct sequence of the sections of "The
25. (a) Negative capability, sublime, dissociation of Waste Land" (1922) by T.S. Eliot are–
sensibility, heteroglossia ♦ The First section - The Burial of the Dead.
(b) Sublime, negative capability, heteroglossia, ♦ The Second section - A Game of Chess.
dissociation of sensibility ♦ The Third section - Fire Sermon.
(c) Sublime, negative capability, dissociation of ♦ The Fourth section - Death by Water.
sensibility, heteroglossia ♦ The Fifth section - What the Thunder Said.
(d) Heteroglossia, dissociation of sensibility, 29. (a) Midnight’s Children, Nectar in a Sieve,
sublime, negative capability Kanthapura, Calcutta Chromosome
Ans: (c) The correct sequence is– (b) Kanthapura, Midnight’s Children, Nectar in a
♦ “Sublime” is associated with Gothic Literature. (18th Sieve, Calcutta Chromosome
century) (c) Kanthapura, Midnight’s Children, Calcutta
Chromosome, Nectar in a Sieve
♦ “Negative capability” is associated with John Keats.
(d) Kanthapura, Nectar in a Sieve, Midnight’s
(early 19th century)
Children, Calcutta Chromosome
♦ “Dissociation of sensibility” is related with T.S.
Eliot. (early 20th century) Ans: (d) The correct order of publication is–
♦ “Heteroglossia” is related to Mikhail Bakhtin. (Mid ♦ 'Kanthapura' (1938) - Raja Rao
20th century) ♦ 'Nectar in a Sieve' (1954) - Kamla Markandaya
26. (a) Thyrsis, Adonais, Lycidas, In Memory of W.B. ♦ 'Midnight’s Children' (1981) - Salman Rushdie
Yeats ♦ 'Calcutta Chromosome' (1995) - Amitav Ghosh
(b) Lycidas, Thyrsis, Adonais, In Memory of 30. (a) The English Novel : Form and Function, The
W.B. Yeats Craft of Fiction, Aspects of the Novel, The
(c) Lycidas, Adonais, Thyrsis, In Memory of W.B. Sense of an Ending
Yeats (b) Craft of Fiction, Aspects of the Novel, The
(d) Adonais, In Memory of W.B. Yeats, Lycidas, English Novel: Form and Function The Sense
Thyrsis of an Ending
Ans: (c) The correct sequence is– (c) The Sense of an Ending, The English Novel:
Form and Function, Craft of Fiction, Aspects
♦ 'Lycidas' (1637) - John Milton
of the Novel
♦ 'Adonais' (1821) - P.B. Shelley (d) Aspects of the Novel, Craft of Fiction, The
♦ 'Thyrsis' (1866) - Matthew Arnold Sense of an Ending, The English Novel: Form
♦ 'In Memory of W.B. Yeats' (1940) - W.H. Auden and Function.
UGC NET/JRF English Paper II, December 2005 16 YCT
Ans: (b) The correct sequence is– Ans: (c) The correct matches are–
♦ “The English Novel: form and function” (1953) - "The Old Bachelor" is the first play written by British
Dorothy Bendon Van Ghent. playwright William Congreve, produced in 1693.
♦ “The Craft of Fiction” (1921) - Percy Lubbock 'Faust' - Goethe;
♦ “Aspects of the Novel” (1927) - E.M. Forster 'The White Devil' - John Webster;
♦ “The Sense of an Ending” (2011) – Julian Barnes 'The Maid's Tragedy' - Beaumont and Fletcher.
Select the matching pairs in question numbers 37. (a) Nadine Gordimer-Nigeria
31 to 40: (b) Chinua Achebe-Kenya
31. (a) Sohrab and Rustum-Arnold (c) Judith Wright-Australia
(b) The Princess-Browning (d) Peter Carey-Canada
(c) Hugh Selwyn Mauberly-Hopkins
(d) The Excursion-Shelley Ans: (c) The correct pairs are–
Ans: (a) The correct matches are– ♦ Nadine Gordimer- South African Writer
♦ 'Sohrab and Rustum' (1853) - Arnold ♦ Chinua Achebe - Nigerian Novelist
♦ 'The Princess' (1847) - Tennyson ♦ Judith Wright- Australian Novelist
♦ 'Hugh Selwyn Mauberly' - Ezra Pound ♦ Peter Carey - Australian novelist
♦ 'The Excursion' - Wordsworth 38. (a) Campus novel-Margaret Drabble
32. (a) Middlemarch-Picaresque (b) Travelogue - Macaulay
(b) Women in Love-Historical (c) Diary writing-Samuel Pepys
(c) Pamela-Epistolary novel (d) Periodical essay-Lamb
(d) Pride and Prejudice-Autobiographical Ans: (c) The correct pairs are–
Ans: (c) The correct matches are– ♦ Campus Novel - Mary McCarthy
♦ 'Middlemarch' (1871) - George Eliot ♦ Travelogue - Pausanias
♦ 'Women in Love' (1920) - D.H. Lawrence ♦ Diary writing - Samuel Pepys
♦ 'Pamela' (1740) - Samuel Richardson [Epistolary - in ♦ Periodical essay - Joseph Addison & Richard Steele
form of letters] 39. (a) Girish Karnad-Kannada
♦ 'Pride and Prejudice' (1813) - Jane Austen (b) A.K. Ramanujan-Telugu
33. (a) Dickens-Manchester (c) Kamala Das-Tamil
(b) Faulkner-Yoknapatawfa (d) R. Parthasarathy-Malayalam
(c) Joyce-Belfast Ans: (a) Girish Karnad (born 19 May, 1938) is a
(d) Lawrence- Birmingham playwright who predominantly works in South Indian
Ans: (b) Yoknapatawpha county is a fictional county Kannada Language.
created by the American author William Faulkner. He A.K. Ramanujan - Kannada
set three of his novels in the county.
Kamala Das - Malayalam
34. (a) Naturalism-Zola
R. Parthasarathy - Tamil
(b) Symbolism-T.E. Hulme
(c) Expressionism-V. Woolf 40. (a) Mrs. Malaprop-The School for Scandal
(d) Magic realism- Graham Greene (b) Nora-The Seagull
(c) Lydia Languish-She Stoops to Conquer
Ans: (a) Naturalism is a literary movement that
(d) Eliza Doolittle-Pygmalion
emphasizes observation and the scientific method in the
fictional portrayal of reality. Emile Zola was the Ans: (d) Eliza Doolittle is a character that appeared in
founder of this movement. the play 'Pygmalion', written by George Bernard Shaw.
Symbolism – Charles Baudelaire It was first presented on stage to the public in 1913.
Expressionism – Edward Munch 41. In the assertion “Four out of five people suffer
Magic Realism - Alejo Carpentier from dreaded pyorrhea”, the writer wants to
35. (a) Audrey Thomas-The Stone Angel arouse the feeling of :
(b) Robert Kroetsch-The Burning Water (a) Sympathy (b) Fear
(c) Margaret Lawrence - What the Crow Said (c) Hatred (d) Ill-will
(d) Margaret Atwood-The Blind Assassin Ans: (b) The writer is trying to arouse the feeling of
Ans: (d) The correct matches are– fear. Fear is an unpleasant emotion caused by the threat
♦ 'The Stone Angel' (1964) - Margaret Lawrence of danger, pain or harm.
♦ 'The Burning Water' (1980) – George Bowcring 42. “John is six feet tall and 240 Ib” is an assertion
♦ 'What the Crow Said' (1978) - Robert Kroetsch of–
♦ 'The Blind Assassin' (2000) - Margaret Atwood (a) a fact (b) a judgment
36. (a) Marlowe-Faust (c) an opinion (d) an inference
(b) Fletcher-The White Devil Ans: (d) An inference is an assumption or conclusion
(c) Congreve-The Old Bachelor that is rationally or logically made, based on the given
(d) Ben Jonson-The Maid’s Tragedy facts or circumstances as in the above statement.
UGC NET/JRF English Paper II, December 2005 17 YCT
43. X : "He’s mean and stingy". 47. What is the meaning of intellectuals being
Y : “Oh, I wouldn’t say that. He is just thrifty”. ‘servants’?
The above dialogue asserts that he : (a) The intellectual may be appropriated by his
(a) is too careful with his money tradition, historical and other actualities of his
(b) never spends money nation and society
(c) is so careful with his money that everyone (b) The intellectual may be inappropriately Co-
admires him for good management opted by agencies of the government
(d) is careful with his money (c) The intellectual may be sent into exile and
Ans: (a) The above dialogue asserts that he is too made marginal
careful with his money. (d) The intellectual may be forced into accepting
44. “I wandered lonely as a cloud” makes an the unacceptable propositions
assertion that : Ans: (a) The appropriate answer seems to be option (a)
(a) The poet travelled with the cloud The intellectual may be appropriated by his tradition,
(b) The poet moved aimlessly with the cloud historical and other actualities of his nation and society.
(c) Both the poet and the cloud were lonely 48. What are four important institutions that Co-
(d) The poet moved as aimlessly as the cloud opt an intellectual?
Ans : (d) “I wandered lonely as a cloud” makes an (a) society, institutions, worldly powers, and truth
assertion that the poet moved as aimlessly as the cloud. (b) academy, church, professional guild, and
45. “Death is here, and death is there worldly power
Death is busy everywhere (c) society, professional guild, worldly power,
All around, within, beneath, truth
Above, is death-and we are death” (d) academy, worldly power, truth, government
The effect of rhythm, sound, word-order and Ans: (b) Four important institutions that co-opt an
stress in the above lines intellectual are academy, church, professional guild and
(a) assist the communication of meaning worldly powers.
(b) hinder the communication of meaning
49. What is the meaning of ‘relative independence’?
(c) reflect meaning and mood
(a) liberating oneself from the pressure of
(d) reflect a mechanical regularity
government and institutions
Ans: (c) It reflects the meaning and mood of Death. (b) liberating oneself from the pressures of religion
(46-50) Read the following passage and answer and state
the questions that follows based on your (c) liberating oneself from the pressures of
understanding of the passage. institutions and worldly powers
All of us live in a society, and are members of a (d) liberating oneself from all religious and secular
nationality with its own language, tradition, historical pressures
situation. To what extent are intellectuals servants of
Ans: (c) According to the passage, the answer is (c)
these actualities, to what extent enemies? The same is
liberating oneself from the pressures of institutions and
true of intellectuals relationship with institutions
worldly powers.
(academy, church, professional guild) and with worldly
powers, which in our times have co-opted the 50. What is the duty of an intellectual and how
intelligentsia to an extraordinary degree. Thus in my many identities does he acquire to perform his
view the principal intellectual duty is the search for role?
relative independence from such pressures. Hence my (a) to achieve complete independence and be
characterization of the intellectual as an exile and characterized as an exile, marginal, and
marginal, as amateur, and as the author of a language amateur
that tries to speak the truth to power. (b) to achieve partial independence and be
46. Name four important sources to which an characterized as the author of a language
intellectual is related basically : (c) to manoeuvre independence and be
(a) Society, institutions, worldly powers, and characterized as a keeper of his own
government conscience
(b) Institutions, language, truth, and power (d) to search for relative independence and be
(c) Nationality, language, tradition, and historical characterized as exile and Marginal, as
situation amateur, and author
(d) Nationality, truth, language and tradition Ans: (d) The duty of an intellectual is to search for
Ans: (c) According to the above passage, nationality, relative independence and be characterized as exile and
language tradition and historical situation are four Marginal, as amateur and author of a language capable
important sources. of speaking the truth to power.
UGC NET/JRF English Paper II, December 2005 18 YCT
UGC NET/JRF Exam, June-2006
ENGLISH
Solved Paper-II
Note: The paper contains fifty (50) multiple (c) A satire on the True Blue Protestant Poets
choice questions, each question carrying two (2) (d) A poem
marks. Attempt all of them. Ans : (d) “Absalom and Achitophel” is a landmark
1. Which one of the following author-book pair is poetic political satire by John Dryden (1631-1700). The
correctly matched? poem exists in two parts. The first part, of 1681, is
(a) J.M. Coetzee-Shame undoubtedly by Dryden. The second part, of 1682, was
(b) Saul Bellow-Herzog written by another hand, most likely Nahum Tate. It is
(c) Salman Rushdie-Disgrace subtitled 'A Poem'.
(d) Elfriede Jelinek-The Pianist 7. Who of the following is not a periodical essayist?
Ans : (b) “Herzog” is a 1964 novel by Saul Bellow, (a) Jonathan Swift (b) Joseph Addison
composed in large part of letters from the protagonist (c) Richard Steel (d) Lancelot Andrews
Moses E. Herzog. It won the U.S. National Book Award Ans : (d) Joseph Addison and Richard Steele are
for Fiction and the Prix International. generally regarded as the most significant figures in the
2. Which novel has a nameless narrator? development of the 18th century periodical. Together
(a) Invisible Man they published the ‘Tatler’ (1709-11), 'The Spectator’
(b) The Grapes of Wrath (1711-12) and the ‘The Guardian’ (1713). Jonathan
(c) Moby Dick Swift is considered as the foremost prose satirist of
(d) Anna Karenina eighteenth century and wrote under the pseudonym of
Ans : (a) “Invisible Man” is a novel by Ralph Ellison Isaac Bickerstaff. Lancelot Andrews was among the
about an African American man whose color renders translators of the Authorized Version of King James
Bible published in 1611. He does not belong to
him invisible, published by Random House in 1952.
eighteenth century.
3. Samuel Beckett wrote :
8. John Eveyln and Samuel Pepys were the famous
(a) Endgame
writers of :
(b) Volpone
(a) essays (b) editorials
(c) Mother Courage and Her Children
(c) letters (d) diaries
(d) A Doll’s House
Ans : (d) John Evelyn was an English writer, gardener
Ans : (a) “Endgame”, by Samuel Beckett, is a one - act and diarist. Evelyn’s diaries, or memoirs are largely
play with four characters. It was originally written in contemporaneous with those of his rival diarist, Samuel
French in 1957 (entitled 'Fin de Partie'); Beckett himself Pepys, and cast considerable light on the art, culture
translated it into English. and politics of the time.
4. Willy Loman is a character in: 9. Samuel Butler's 'Hudibras' is modeled upon :
(a) A Doll’s House (a) “Annus Mirabilis” (b) Endymion
(b) The Cherry Orchard (c) Don Quixote (d) Pilgrim’s Progress
(c) Waiting for Godot
Ans : (c) ‘Hudibras’ is in English mock heroic
(d) The Death of a Salesman
narrative poem from the 17th century. Butler is clearly
Ans : (d) “Death of a Salesman” is a 1949 play written influenced by Cervantes’ 'Don Quixote'. ‘Hudibras’
by American playwright Arthur Miller. It was the was written in an iambic tetrameter in closed couplets
recipient of the 1949 Pulitzer Prize for Drama. with surprising feminine rhymes.
5. 'The Plough and the Stars' was written by : 10. Who was the last of the Christian Humanists?
(a) G.B. Shaw (b) J.M. Synge (a) Oliver Cromwell (b) John Milton
(c) Sean O Casey (d) Lady Gregory (c) John Bunyan (d) Richard Crashaw
Ans : (c) “The Plough and the Stars” is a play by the Ans : (b) ‘Christian humanism’ emphasizes the
Irish writer Sean O' Casey first performed on February humanity of Jesus, his social teachings and his
8, 1926 at the Abbey Theatre in the writer’s native propensity to synthesize human spirituality and
Dublin. It is the third of his well known “Dublin materialism. The term ‘humanism’ was coined by
Trilogy” - the other two being ‘The Shadow of a Theologian Friedrich-Niethammer at the beginning of
Gunman’ (1923) and ‘Juno and the Pay cock’ (1924). the 19th century.
6. The subtitle of Dryden’s Absalom and 11. The narrative of Raja Rao’s 'Kanthapura' is
Achitophel is : based on :
(a) There was no subtitle (a) Puranas (b) Shastras
(b) A satire (c) The Ramayana (d) The Mahabharata
UGC NET/JRF English Paper II, June 2006 19 YCT
Ans : (a) Rao’s first novel, “Kanthapura”, is the story (c) secondary imagination
of how a small, sleepy, south Indian village is caught in (d) esemplastic imagination
the whirlpool of the Indian freedom struggle. These Ans : (b) The Above Statement of Coleridge is taken
local Sthala-Puranas are modeled on the ancient Indian from “Biographia Literaria” which was one of
Puranas. . Coleridge’s main critical studies. In this work, he
12. Which of the following author-book pair is discussed the elements of writing and what writing
correctly matched? should be to be considered genius.
(a) David Malour-The City of Djins 18. “Did he who made the Lamb made thee”
(b) C.L.R. James-The English Patient appears in :
(c) Shashi Tharoor-Trotter Nama (a) “The Tyger” (b) “Chimney Sweeper”
(d) Arundhati Roy-Algebra of Infinite Justice (c) “London” (d) “Introduction”
Ans : (d) The correctly matched pair is (d). Ans : (a) The above line is from “The Tyger”, is a
(a) ‘The City of Djins’ (1994) - William Dalrymple poem by William Blake published in 1794 as part of the
(b) ‘The English Patient’ (1992) - Michael Ondaatje ‘Songs of Experience’ collection. It is one of Blake’s
(c) ‘Trotter Nama’ (1988) - Allan Sealy most reinterpreted and arranged works.
(d) 'Algebra of Infinite Justice' (2001)- Arundhati Roy 19. “Essays of Elia” are :
13. Who wrote “A tiger does not proclaim its (a) political ideology (b) economic disparity
(c) literary criticism (d) personal impressions
tigretude”?
(a) Ngugi (b) Achebe Ans : (d) “Essay of Elia” is a collection of essays
(c) Soyinka (d) Derek Walcott written by Charles Lamb; it was first published in book
form in 1823, with a second volume, 'Last Essays of
Ans : (c) The above quote is by Wole Soyinka (born
Elia' published in 1833.
1934), a Nigerian playwright, poet, novelist, essayist
and pro-democracy activist. He was awarded Nobel 20. Who among the following is a writer of
Prize for Literature in 1986. historical romances?
(a) Emily Bronte (b) Jane Austen
14. “Jindiworobak” movement relates to : (c) Walter Scott (d) Walter Savage Landor
(a) Australian literature
(b) Canadian literature Ans : (c) Historical romance (also historical novel) is a
(c) New Zealand literature broad category of fiction in which the plot takes place in
(d) Caribbean literature a setting located in the past which Walter Scott helped
popularize in the early 19th century.
Ans : (a) “Jindiworobak” movement was an Australian
literary movement of the 1930s and 1940s whose white 21. Which of the following thinker-concept pairs is
members, mostly poets, sought to contribute to a rightly matched?
uniquely Australian culture through the integration of (a) Stanley Fish-Reader Response
Indigenous Australian subject, language and mythology. (b) Jacques Derrida-New Historicism
(c) Northrop Frye-Practical Criticism
15. The Montreal group of poets championed the (d) I.A. Richards-Archetypal Criticism
cause of :
(a) Nature poetry (b) Symbolist poetry Ans : (a) The rightly matched pair is (a).
(c) Imagist poetry (d) Modernist poetry (a) Reader Response - Stanley Fish
Ans : (d) ‘Montreal Group’ was a circle of Canadian (b) New Historicism - Stephen Greenblatt
modernist writers formed in the mid-1920s at Mc Gill (c) Practical Criticism - I.A. Richards
University in Montreal, Quebec, which included Leon (d) Archetypal Criticism - Northrop Frye
Eder, John Glassco, A.M. Klein, F.R. Scott and A.J.M. 22. Which of the following thinker concept pairs is
Smith Most of the group’s members attended Mc Gill as rightly matched?
undergraduates. Due to this connection, the group is (a) Vaman-Dhwanyaloka
also referred to as the Mc Gill Group or Mc Gill (b) Bharata-Natya Shastra
Movement. (c) Mamata-Vakrokti
16. The figure of the “Abyssinian maid” appears in: (d) Abhinava Gupta-Kavya Alankar
(a) “Frost at midnight” (b) “Christabel” Ans : (b) The rightly matched pair is (b).
(c) “Kubla Khan” (d) “Dejection : an Ode” (a) Dhwanyaloka - Anandavardhana
Ans : (a) “Kubla Khan; or A Vision in a Dream: A (b) Natya Shastra - Bharata Muni
Fragment” is a poem written by Samuel Taylor (c) Vakrokti - Kuntaka
Coleridge, completed in 1797 and published in 1816. (d) Kavya Alankar - Bhamaha
According to Coleridge’s Preface to “Kubla Khan” the 23. Choose the correct sequence of the following
poem was composed one night after he experienced an schools of criticism :
opium-influenced dream after reading a work describing (a) Structuralism, New Criticism, Deconstruction,
Xanadu, the summer palace of the Mongol ruler and Reader Response
emperor of China, Kubla Khan. (b) New Criticism, Structuralism, Deconstruction,
17. Coleridge's statement that imagination Reader Response
“dissolves, diffuses, dissipates in order to (c) Reader Response, Deconstruction,
recreate” relates to : Structuralism, New Criticism
(a) fancy (d) Deconstruction, New Criticism, Structuralism,
(b) primary imagination Reader Response
UGC NET/JRF English Paper II, June 2006 20 YCT
Ans : (a) The correct sequence is as follows– Ans : (d) “The Madwoman in the Attic” published in
(1) Structuralism - Early 1900s 1979, examines Victorian Literature from a feminist
(2) New Criticism - 1941 perspective. Authors Sandra Gilbert and Susan Gubar
(3) Deconstruction - 1967 draw their title from Charlotte Bronte’s “Jane Eyre” in
which Rochester’s wife Bertha Mason is kept locked in
(4) Reader Response - 1960s - 1970s
the attic by her husband.
24. “Peripetia” means :
30. Who among the Victorians is called “the prophet
(a) purgation of emotion (b) tragic flaw
of modern society”?
(c) reversal of fortune (d) recognition of error
(a) Ruskin (b) Carlyle
Ans : (c) Peripetia is a reversal of circumstances, or (c) Macaulay (d) Arnold
turning point. The term is primarily used with reference Ans : (a) Ruskin is often considered as “the prophet of
to works of literature. The Anglicized form of Peripetia modern society.” His first two lectures in 'Sesame and
is Peripety. Lilies' gives his views on wealth, books, education,
25. “Gynocriticism” focuses on : labor, woman’s sphere and human society.
(a) Criticism on women 31. Who among the following is not a pilgrim in
(b) Criticism by women 'The Canterbury Tales'?
(c) Criticism of male writers by women writers (a) the Haberdasher (b) the Tapyser
(d) Woman as writers (c) the Blacksmith (d) the Summoner
Ans : (d) “Gynocriticism” or gynocritics is the term Ans : (b) “The Canterbury Tales” is a collection of 24
coined in the seventies by Elaine Showalter to describe stories that run over 17,000 lines written in Middle
a new literary project intended to construct “a new English by Geoffrey Chaucer. In 1386 Chaucer became
female frame work for the analysis of women’s Controller of Customs and Justice of Peace and three
literature.” years later, clerk of the king’s work in 1389.
26. Which of the following sequences is correct? 32. Bosola is the executioner in :
(a) Vanity Fair, Henry Esmond, Middlemarch, The (a) The Spanish Tragedy (b) The Duchess of Malfi
Return of the Native (c) The White Devil (d) The Jew of Malta
(b) Henry Esmond, Vanity Fair, Middlemarch, The Ans : (b) “The Duchess of Malfi” is a macabre, tragic
Return of the Native play written by the English dramatist John Webster in
(c) Middlemarch, The Return of the Native, 1612-13. Published in 1623, the play is loosely based on
Vanity Fair, Henry Esmond events that occurred between 1508-1513.
(d) The Return of the Native Middlemarch, Vanity 33. The mystery plays deal with :
Fair, Henry Esmond (a) the life of Christ (b) the New Testament
Ans : (a) The correct sequence is as follows– (c) Psalms (d) Apocrypha
(1) 'Vanity Fair' (1847-48) - William Makepeace Ans : (b) Mystery plays also called miracle plays
Thackeray focused on the representation of Bible stories in
(2) 'Henry Esmond' (1852) - William Makepeace churches as tableaux with accompanying antiphonal
Thackeray song i.e. the New Testament.
(3) 'The Return of the Native' (1878) - Thomas Hardy 34. The Faerie Queene is based on :
27. Queen Victoria’s reign, after whom the (a) Utopia (b) Tottel's Miscellany
Victorian period is named, spans : (c) Morte d’ Arthur (d) Orlando Furioso
(a) 1833-1901 (b) 1837-1901 Ans : (c) Thomas Malory’s “Le Morte d’ Arthur” was
(c) 1840-1905 (d) 1842-1905 written a century earlier than Spenser’s ‘Faerie Queene’
Ans : (b) The Victorian Era of British history was the and is written in prose and somewhere Spenser was
period of Queen Victoria’s reign from 20 June 1837 trying to capture this story while writing ‘Faerie
until her death, on 22 January 1901. It was a long period Queene’.
of peace, prosperity, refined sensibilities and national 35. Choose the correct chronological sequence of the
self-confidence for Britain. following plays :
28. Pre-Raphaelite poetry is mainly concerned with: (a) King Lear, Othello, Macbeth, Hamlet
(a) narrative and style (b) narrative and nature (b) Othello, Macbeth, King Lear, Hamlet
(c) form and design (d) form and value (c) Hamlet, Othello, King Lear, Macbeth
(d) Hamlet, King Lear, Othello, Macbeth
Ans : (c) Pre-Raphaelite Brotherhood (Later known as
the Pre-Raphaelites) was a group of English painters, Ans : (c)
poets, and critics, founded in 1848 by William Holman (1) Hamlet - 1600 (2) Othello - 1604
Hunt, John Everett Millais and Dante Gabriel Rossetti. (3) King Lear - 1605 (4) Macbeth - 1605-06
29. The concept of “mad woman in the attic” can be 36. Pope’s “Essay on Criticism” sums up the art of
traced to : poetry as taught first by :
(a) The Tenant of Wild fell Hall (b) Villette (a) Aristotle (b) Horace
(c) Wuthering Heights (d) Jane Eyre (c) Longinus (d) Plato
UGC NET/JRF English Paper II, June 2006 21 YCT
Ans : (b) “An Essay on Criticism” is written in a type Ans : (b) 'Out of Africa' film is adapted from Karen
of rhyming verse called heroic couplets written in 1709. Blixen's novel of the same title.
It is a verse essay written in the Horatian mode and is 44. The Empire writes Back was written by :
primarily concerned with writers and critics in the new (a) Bill Ashcroft, Helen Tiffin, Ngugi Wa Thiongo
literary commerce. (b) Bill Ashcroft, Helen Tiffin, Stephen Slemon
37. Swift’s 'Tale of a Tub' is a satire on : (c) Bill Ashcroft, Gareth Griffiths, Chinua Achebe
(a) science and morality (d) Bill Ashcroft, Helen Tiffin, Gareth Griffiths
(b) art and morality Ans : (d) 'The Empire Writes Back: Theory and
(c) dogma and superstition Practice in Post-Colonial Literature' is a 1989 non-
(d) fake morals and manners fiction book on post-colonialism, penned by Bill
Ans : (c) “A Tale of a Tub” is a prose parody Ashcroft, Gareth Griffiths and Helen Tiffin.
composed between 1694 and 1697, it was eventually 45. The theatre of cruelty is associated with :
published in 1704. The ‘tale’ presents a consistent satire (a) Stanislavosky (b) Grotovsky
of religious excess. (c) Antonin Artaud (d) Eugino Barba
38. Dr. Johnson started : Ans : (c) The Theatre of Cruelty is a form of theatre
(a) The Postman (b) The Spectator developed by avant-garde playwright, actor, essayist
(c) The Rambler (d) The Tatler and theorist, Antonin Artaud in ‘The Theatre and its
Double’.
Ans : (c) “The Rambler” was a periodical by
Dr. Johnson. It was published on Tuesdays and 46. A particle is :
(a) a patchwork of words, sentences, passages
Saturdays from 1750 to 1752 and totals 208 articles.
(b) a satirical poem
“The Rambler” was written in elevated prose. It (c) a love song
discussed subjects such as morality, literature, society, (d) a collection of lines from different poems
politics and religion.
Ans : (a) A particle is a word that has grammatical
39. Who among the following cautioned against the functions but doesn’t fit into the main parts of speech.
dangers of popular liberty? Particles don’t change.
(a) Mary Wollstonecraft (b) Edmund Burke
(c) Thomas Hobbes (d) John Locke 47. “Careless she is with artful Care/Affecting to
seem unaffected” is an example of :
Ans : (b) Edmund Burke was an Irish Statesman born in (a) irony (b) paradox
Dublin, as well as an author, orator, political theorist (c) simile (d) metaphor
and philosopher who after moving to London served as
Ans : (b) A paradox is a statement that is self
a Member of Parliament with the Whig Party for many contradictory because it often contains two statements
years. that are both true, but in general, can’t both be true at
40. Which famous American classic opens with the same time.
“Call me Ishmael”? 48. A metrical foot containing a stressed, followed
(a) Rip Van Winkle (b) The Scarlet Letter by an unstressed, syllable is :
(c) The Grapes of Wrath (d) Moby Dick (a) anapaest (b) iamb
Ans : (d) Ishmael is a fictional character in Herman (c) trochee (d) dactyl
Melville’s 'Moby-Dick' (1851). Ishmael, the only Ans : (c) In poetic meter, a trochee is a metrical foot
surviving crew member of the Pequod, is the narrator of consisting of a stressed syllable followed by an
the book. As a character he is only a minor participant unstressed one, in English. A trochee is the reverse of
in the action. an iamb.
41. Allen Ginsberg’s vision of America is inspired 49. The rhyme scheme of a Spenserian sonnet is :
by : (a) abba, cbcb, cdcd, ee (b) abab, bccb, ccdd, ee
(a) Walt Whitman (b) Robert Frost (c) aabb, bcbc, cdcd, ee (d) abab, bcbc, cdcd, ee
(c) Ralph Waldo Emerson (d) Edgar A. Poe Ans : (d) The Spenserian sonnet is a variation of the
Ans : (a) “America” is a poem by Allen Ginsberg, English sonnet with the rhyme scheme. ABAB, BCBC,
written in 1956. It appears in his collection “Howl and CDCD, EE in which the quatrains are interlinked by a
other poems.” It is presented in a somewhat rambling continuation of one end rhyme from the previous
stream of consciousness format. quatrain.
42. Who among the following represents the Sri 50. Using the expression “Crown” for the monarchy
Lankan diaspora? is an example of :
(a) M.G. Vassanji (b) Cyril Debydeen (a) Metonymy (b) Synecdoche
(c) Michael Ondaatje (d) Arnold H. Itwaru (c) Irony (d) Metaphor
Ans : (c) Philip Michael Ondaatje (born 12 Sep, 1943), Ans : (a) Metonymy is a figure of speech in which a
is a Sri Lankan - born Canadian novelist and poet. He thing or concept is called not by its own name but rather
won the Booker Prize for his novel ‘The English by the name of something associated in meaning with
Patient’ (1992). that thing or concept. For example, Wall Street is often
43. 'Out of Africa' is a film adaptation of a work by: used metonymically to describe the U.S. financial and
(a) Alice Walker (b) Margaret Lawrence corporate banking sector, while Hollywood is used as a
(c) Margaret Atwood (d) Alice Munro metonym for the U.S. film industry.
UGC NET/JRF English Paper II, June 2006 22 YCT
UGC NET/JRF Exam, December-2006
ENGLISH
Solved Paper-II
1. The title 'The Sound and the Fury' is taken (c) "Voyage to Brobdingnag"
from: (d) "Voyage to Lilliput"
(a) Hamlet (b) Macbeth Ans: 6 (a) Travels into Several Remote Nations of the
(c) The Tempest (d) King Lear World in Four Parts by Lemuel Gulliver, first a
Ans: (b) Title of the novel “The Sound and the Fury” Surgeon, and then a Captain of several ships, commonly
is taken from Macbeth's famous soliloquy of Act 5, known as Gulliver’s Travels (1726, amended 1735), is a
scene 5 of the play 'Macbeth' by William Shakespeare. satire by Anglo-Irish writer and Clergyman Jonathan
The above titled novel by William Faulkner was Swift.
published in 1929 and was his fourth novel. Part I : A Voyage to Lilliput
2. Pecola is a character in : Part II : A Voyage to Brobdingnag
(a) The Bluest Eye (b) Oliver Twist Part III : A Voyage to Laputa, Balnibarbi, Luggnagg
(c) Don Quixote (d) Beloved and Glubbdubdrib
Ans: (a) Pecola is a character in “Bluest Eye” (1970), Part IV : A Voyage to the country of the Houyhnhnms
first novel by American author Toni Morrison. The 7. Who edited The Tatler?
story is about a year in the life of a young black girl (a) Steele and John Locke
named Pecola who develops an inferiority complex due (b) Addison and Dryden
to her eye color and skin appearance. (c) Addison and Blackmore
3. Which of the following was associated with the (d) Addison and Steele
"Bloomsbury Group". Ans: (d) The Tatler was a British Literary and society
(a) T.S. Eliot (b) W.B. Yeats Journal begun by Richard Steele in 1709 and published
(c) T. E. Hulme (d) Virginia Woolf for two years.
Ans: (d) The Bloomsbury Group was an influential 8. John Locke's "Essay Concerning Human
group of associated English writers, intellectuals, Understanding" is about :
philosophers and artists, like Virginia Woolf, John (a) nature of human behaviour
Maynard Keynes, E.M. Forster and Lytton Strachey. (b) nature of the human mind
4. Which of the following characters appear in (c) nature of human society
'Waiting for Godot' : (d) nature of human ideology
(a) Jerry (b) Lucky Ans: (b) “An Essay Concerning Human
(c) Jimmy Porter (d) Ham Understanding” is a work concerning the foundation
Ans: (b) ‘Lucky’ is a character from Samuel Beckett’s of human knowledge, first appeared in 1689.
“Waiting for Godot” (1953). He is a slave to the 9. Restoration Comedy marks the restoration of :
character Pozzo. (a) women's rights (b) democracy
5. About whom did T.S. Eliot write "A thought to (c) monarchy (d) human rights
him was an experience"? Ans: (c) Restoration Comedy refers to the English
(a) Herbert (b) Marvell comedies written and performed in the restoration
(c) Donne (d) Crashaw period from 1660 to 1710. Comedy of manners is used
Ans: (c) T.S. Eliot used term Dissociation of Sensibility as a synonym of Restoration Comedy.
first in his essay “The Metaphysical Poets.” It refers to 10. Which of Alexander Pope's poems begins with
the way in which intellectual thought was separated the line "Shut, shut the door, good John,
from the experience of feeling in 17th century poetry. In fatigued I said":
this essay, Eliot expresses his views for Donne. (a) "Epistle to Dr. Arbuthnot"
6. The last book of Gulliver's Travels is : (b) "Dunciad"
(a) "Voyage to Houyhnhnms" (c) "Epistles"
(b) "Voyage to Laputa" (d) "Rape of the Lock"
UGC NET/JRF English Paper II, December 2006 23 YCT
Ans: (a) The “Epistle to Dr. Arbuthnot” is a satire in Ans: (d) “A House For Mr. Biswas” is a 1961 novel
poetic form written by Alexander Pope and addressed to of V.S. Naipaul, significant as Naipaul’s first work to
his friend John Arbuthnot, a physician. It was first achieve acclaim worldwide. In the novel the Tulsi
published in 1735. family lives in Hanuman House.
11. The statement "One has to convey in a 16. The quotation "a repetition in the finite mind
language that is not one's own the spirt that is of the eternal act of creation in the infinite I
one's own" appears in : AM" appears in :
(a) Ice-Candy man (b) The Guide (a) Lyrical Ballads
(c) Nagamandala (d) Kanthapura (b) Biographia Literaria
Ans: (d) Raja Rao’s first novel 'Kanthapura' (1938) is (c) " In Defense of Poetry"
the story of a village in South India named Kanthapura. (d) Letters of Keats.
The novel was an account of the impact of Gandhi’s Ans: (b) “Biographia Literaria” or 'Biographical
teaching on nonviolent resistance against the British.Sketches of My Literature, Life and Opinions', is an
12. Which of the following author-book pair is autobiography in discourse by Samuel Taylor
correctly matched : Coleridge, which he published in 1817, in two volumes.
(a) Arundhati Roy – The Autumn of the It has 23 chapters and features the above quote.
Patriarch 17. "Fearful Symmetry" appears in the poem :
(b) Gabriel Garcia – Love in the Time of
(a) "Introduction" (b) "Chimney Sweeper"
Marquez Cholera
(c) "The Tyger" (d) "London"
(c) Umberto Eco – The Tin Drum
Ans: (c) “The Tyger” is a poem by English poet
(d) Jhumpa Lahiri – Beloved
William Blake published in 1794, "The Tyger" is the
Ans: (b) The correct book-author pairs are– sister poem to “The Lamb”, a reflection of similar
'The Autumn of the Patriarch' (1975) – by Gabriel ideas from a different perspective. Blake believes that to
Garcia Marquez seek out, the hand that created “The Lamb”, one must
'Love in the Time of Cholera' (1985) – by Gabriel also seek the other that created “The Tyger.”
Garcia Marquez
18. The quotation "when a man is capable of being
'The Tin Drum' (1959) – by Gunter Grass
in uncertainties, mysteries, doubts, without any
'Beloved' (1987) – by Toni Morrison
irritable reaching after fact and reason" is a
13. Which of the following women writers did not definition of :
receive the Nobel Prize : (a) Negative capability
(a) Toni Morrison (b) Nadine Gordimer (b) Secondary imagination
(c) Buchi Emecheta (d) Doris Lessing (c) Criticism of life
Ans: (c) Buchi Emecheta (born 21 July 1944) is a (d) Dissociation of sensibility
Nigerian novelist based in Britain who has published Ans: (a) Negative capability was a phrase first used by
more than 20 books, including “Second Class Citizen” Romantic poet John Keats in 1817 to characterize the
(1974), “The Bride Price” (1976), “The Slave Girl” capacity of the greatest writers to pursue a vision of
(1977) and “The Joys of Motherhood” (1979). artistic beauty even when it leads them into intellectual
14. Which of the following is not an Australian confusion and uncertainty.
author : 19. Which of the following prose-writers do not
(a) Margaret Laurence (b) David Malouf belong to the Romantic Period :
(c) Mudrooroo Narogin (d) Peter Carey (a) Peacock (b) De Quincey
Ans: (a) Margaret Laurence (1926-1987) was a (c) Hazlitt (d) Gibbon
Canadian novelist and Short story writer, and is one of
Ans: (d) Edward Gibbon (1737-1794) was an English
the major figures in Canadian literature. historian, writer and Member of Parliament. His most
15. The Tulsis of Naipaul's 'A House for Mr. important work, “The History of the Decline and Fall
Biswas' lived in : of the Roman Empire,” was published in 6 volumes
(a) Pagotes House (b) Hanuman Mansion between 1776 and 1788 and is known for quality and
(c) Tulsiana (d) Hanuman House irony of its prose.

UGC NET/JRF English Paper II, December 2006 24 YCT


20. In 'Pride and Prejudice' Lydia and Wickham Ans: (d) “Hamartia” derives from Greek, which
eloped to : means “Miss to Mark” or “to err”, first used by
(a) Barchester (b) Bath Aristotle. It is associated with Greek tragedy, an error of
(c) Gretna Green (d) Glasgow judgement by the protagonist which leads to downfall,
such as that of Macbeth.
Ans: (c) “Pride & Prejudice” is a novel of manners by
Jane Austen first published in 1813. 25. The term "gynocriticism" was coined by :
21. Which of the following thinker-concept pairs is (a) Betty Friedman (b) Elaine Showalter
correctly matched : (c) Luce Irigarey (d) Susan Sontag
(a) Frye..........Mysticism Ans: (b) Gynocriticism is a term coined by
(b) Derrida...........Deconstruction Elaine Showalter, to describe new literary project,
intended to construct “a female framework for the
(c) I. A. Richards.......Archetypal Criticism
analysis of Women’s Literature.”
(d) Eagleton...........Psychological Criticism
26. Which is the correct sequence :
Ans: (b) Deconstruction is a critical outlook concerned
with the relationship between text and meaning. Jacques (a) D.G. Rossetti, George Eliot, Bronte Sisters,
Derrida’s 1967 work “Of Gram-matology” introduces Thackeray
the majority of ideas influential with in deconstruction. (b) George Eliot, D.G. Rossetti, Bronte Sisters,
Thackeray
22. Which of the following thinker-concept pairs is
(c) Thackeray, Bronte Sisters, George Eliot,
correctly matched :
D.G. Rossetti
(a) Abhinava Gupta.........Dhwanyaloka
(d) Bronte Sisters, George Eliot, Thackeray, D
(b) Vaman............Kavya Alankar
G. Rossetti
(c) Mamata............Kavya Prakash
Ans. (c) The sequence is option (c) is correct.
(d) Bharata........Vakrokti
William Makepeace Thackeray → 1811 - 1863
Ans: (c) Kavya Prakash is a concept by Mamata is the Bronte Sisters - Charlotte → 1816-1855
correct pair.
Emily → 1818-1848
'Dhwanyaloka' - Ananda Vardhana Anne → 1820 - 1849
'Kavya Alankar' - Pandit Var Shri Vaman George Eliot → 1819 - 1880
'Kavya Prakash' - Mamta D.G. Rossetti → 1828 - 1882
'Vakrokti' - Kuntaka
27. Which of Dickens's novel that opens with the
23. Choose the correct sequence of the following
words "It was the best of times, it was the
schools of criticism : worst of time...." is :
(a) Structuralism, Deconstruction, Reader- (a) A Tale of Two Cities
Response, New Historicism (b) Oliver Twist
(b) New Historicism, Reader-Response, (c) Pickwick Papers
Deconstruction, Structuralism (d) Hard Times
(c) Deconstruction, New Historicism, Ans. (a) “A Tale of Two Cities” (1859) is a novel by
Structuralism, Reader-Response Charles Dickens set in London and Paris before and
(d) Reader-Response, Deconstruction, New during the French Revolution.
Historicism, Structuralism
28. The term "The Fleshly School of Poetry" is
Ans: (a) The sequence in option (a) is correct. associated with the :
Structuralism - 1879 (a) Chartists (b) Pre-Raphaelites.
Deconstruction - 1967 (c) Symbolists (d) Imagists
Reader Response Theory – 1960s & 1970s Ans. (b) “The Fleshly School of Poetry” is a fierce
New Historicism - 1980s attack on the Pre-Raphaelite School written in 1871.
24. "Hamartia" means : The essay was first published in the Contemporary
(a) reversal of fortunes Review under the pseudonym, ‘Thomas Maitland.’
(b) purgation of emotions 29. The line "The sea is calm tonight" occurs in :
(c) depravity (a) Tennyson's "Maude"
(d) error of judgement (b) Arnold's "Thyrsis"
UGC NET/JRF English Paper II, December 2006 25 YCT
(c) Tennyson's "The Lotos-Eaters" 36. The author of Ars Poetica is :
(d) Arnold's "Dover Beach" (a) Plato (b) Horace
Ans. (d) “Dover Beach” is a lyric poem first published (c) Virgil (d) Aristotle
in 1867 in the collection new poems. Ans. (b) “Ars Poetica”, or “The Art of Poetry”, is a
30. The term "gothic", a category of fiction, also poem written by Horace C. 19 BC, in which he advises
applies to : poets on the art of writing poetry and drama.
(a) architecture (b) painting 37. Which of the following is not a work by Dr.
(c) music (d) theater Johnson :
Ans. (a) Gothic architecture is a style of architecture (a) Preface to the English Dictionary
that flourished during the high and late medieval period. (b) Preface to Shakespeare
It evolved from Romanesque architecture and was (c) Lives of English Poets
succeeded by renaissance architecture. (d) Cowley
31. The gap-toothed character in "Prologue" to Ans. (d) Samuel Johnson, often referred to as Dr.
The Canterbury Tales is : Johnson, was an English writer who made lasting
(a) the Prioress (b) the Nun contributions to English Literature as a poet, essayist,
(c) the Wife of Bath (d) the Narrator literary critic, biographer, editor and lexicographer.
Ans. (c) The Wife of Bath’s Tale is among the best 38. Which novel of Daniel Defoe was considered to
known of Geoffrey Chaucer’s 'Canterbury Tales'. It be the best by E. M. Forster?
provides insight into the role of women in the late (a) Colonel Jack (b) Robinson Crusoe
Middle Ages. (c) Captain Singleton (d) Moll Flanders
32. Which of the following is not a Revenge Tragedy : Ans. (d) “The Fortunes and Misfortunes of the
(a) Duchess of Malfi (b) Volpone Famous Moll Flanders” is a novel by
(c) Hamlet (d) Gorboduc Daniel Defoe, first published in 1722. E.M. Forster
Ans. (b) “Volpone” is a comedy by English playwright considers it his best.
Ben Jonson first produced in 1605-06, drawing on 39. Edmund Burke denounced the French
elements of comedy and beast fable. Revolution in :
33. Miracle plays are based on the lives of : (a) Political Philosophy
(a) Knights (b) Crusaders (b) A Philosophical Enquiry into the Origin of
our Ideas of the sublime and the Beautiful
(c) Pilgrims (d) Saints
(c) Reflection
Ans. (d) Miracle Plays, also called Saint’s Play,
(d) The Annual Register
presents a real or fictitious account of the life, Miracles,
or martyrdom of a saint. Ans. (c) “Reflection on the Revolution in France” is
a political Pamphlet written by the Irish Statesman
34. The Red-cross Knight in Spenser's Faerie
Edmund Burke and published in November, 1790.
Queene represents : Reflection was one of the best-known intellectual
(a) Temperance (b) Chastity attacks against the French Revolution.
(c) Truth (d) Falsehood 40. The line "A man can be destroyed but not
Ans. (c) “The Faerie Queene” is an incomplete defeated" appears in :
English epic poem by Edmund Spenser. It is divided (a) For Whom the Bell Tolls
into VI Books, Book I tells the story of the Knight of
(b) The Old Man and the Sea
Holiness or Truth, the Red-cross Knight.
(c) The Snows of Kilimanjaro
35. The line "Present fears/Are less than horrible (d) The Sun also Rises
imaginings" appear in : Ans. (b) “The Old Man and the Sea” is a short novel
(a) Macbeth (b) King Lear written by the American author Ernest Hemingway in
(c) Othello (d) Julius Caesar 1951 in Bimini, Bahamas and published in 1952. It tells
Ans. (a) “Macbeth” is believed to have been written the story of Santiago, an aging Cuban fisherman who
between 1599 and 1606. It was first published in the struggles with a giant Marlin far out in the Gulf Stream
Folio of 1623. off the coast of Florida.

UGC NET/JRF English Paper II, December 2006 26 YCT


41. Who among the following is called "A New 46. The relationship between Othello and Iago is
England Poet": an example of :
(a) Robert Frost (a) inversion (b) irony
(b) Edwin Arlington Robinson (c) innuendo (d) invective
(c) William Carlos Williams
Ans. (a) “Othello” was written in 1603, and is set in
(d) Allen Ginsberg
Venice. The relationship consists of jealousy and malice
Ans. (a) Robert Frost (1874-1963) was an American from Iago’s side and trust and nobility from Othello.
poet. His work was initially published in England
47. A metrical foot consisting of an unstressed
before it was published in America.
syllable followed by a stressed syllable is :
42. Which of the following is not a play by
Tennessee Williams : (a) dactyl (b) trochee
(a) Night of the Iguana (c) iamb (d) anapaest
(b) A Streetcar named Desire Ans. (c) An iamb is a metrical foot used in various
(c) Cat on a Hot Tin Roof types of poetry. Originally the term referred to one of
(d) The Zoo Story the feet of the quantitative meter of classical Greek
Ans. (d) “The Zoo Story”, originally titled Peter and prosody: a short syllable followed by a long syllable (as
Jerry, is a one-act play by American playwright Edward in “delay’).
Albee. His first play, it was written in 1958 and 48. The rhyme scheme of a Shakespearean sonnet
completed in just Three Weeks. is :
43. Margaret Atwood's 'Survival' is : (a) abab, cdcd, efef, gg
(a) a critical assessment of Canadian writing (b) abba, cddc, dffe, gg
(b) a thematic guide to Canadian literature (c) abcd, efgh, effe, hh
(c) a critique of Canadian polity (d) abca, abca, bcab, dd
(d) a exposition of Canadian history Ans. (a) The Shakespearean Sonnet has the rhyme
Ans. (b) "Survival: A Thematic Guide to Canadian scheme of ABAB CDCD EFEF GG, forming three
literature" is a survey of Canadian literature by quatrains (four line in a group) and a closing couplet
Margaret Atwood, one of the best-known Canadian (two rhymed lines).
authors. It was first published by House of Anansi in 49. Using "the Bench" for the judiciary is an
1972.
example of :
44. The term "Negritude" was coined by :
(a) metaphor (b) irony
(a) Frantz Fanon and Homi Bhabha (c) Synecdoche (d) metonymy
(b) Ngugi Wa' Thiongo and Wole Soyinka
Ans. (d) In Metonymy, (literally change of name) an
(c) Aime Cesaire and Leopold Senghor
object is designated by the name of something with
(d) K. Alfred Memi and Chinua Achebe
which it is associated with,
Ans. (c) Negritude is a literary and ideological Example- The Bench, for the Judges.
philosophy, developed by francophone African
The House, for the Members of Parliament.
intellectuals, writers and politicians in France during
1930s. Its initiators included Aime Cesaire and Leopold 50. Four feet, comprising a monosyllable, trochee,
Sedar Senghor. dactyl and first paeon is often called?
45. Bertolt Brecht's concept of theatre was (a) running rhythm (b) Sprung rhythm
influenced by : (c) blank verse (d) rhymed verse
(a) Irwin Piscator (b) Antonin Artaud Ans. (b) Sprung rhythm is a poetic rhythm designed to
(c) Peter Brook (d) Eugino Barda imitate the rhythm of natural speech. First paeon or foot
Ans. (a) Irwin Piscator (1893-1966) was a German comprises of one long syllable and three short syllables.
Theater director and producer and along with Bertolt In English, the use of sprung rhythm is rarely found out
Brecht, the foremost exponent of epic theatre, a form of that order except in G.M. Hopkins who used the
that emphasizes the socio-political contents of drama, second and third paeons in combination with other feet
rather than its emotional manipulation of the audience in his poem 'The Windhover' and others. It was
or on the production of formal beauty. popularized by him.
UGC NET/JRF English Paper II, December 2006 27 YCT
UGC NET/JRF Exam, June-2007
ENGLISH
Solved Paper-II
Note : This paper contains fifty (50) multiple- 6. The Grand Inquisitor is a character in :
choice questions, each question carrying two (2) (a) Crime and Punishment
marks. Attempt all of them. (b) Notes from the Underground
1. The lines : (c) Brothers Karamazov
'Even, I, a dunce of more renown than they, (d) The Idiot
Was sent before but to prepare thy way' are
Ans : (c) “The Grand Inquisitor” is a character in
quoted from :
Fyodor Dostoyevsky’s novel 'The Brothers Karamazov'
(a) Pope's 'Dunciad' (1879-1880).
(b) Dryden's 'Absalom and Achitophel'
7. Which modern critic described value
(c) Dryden's 'Mac Flecknoe'
(d) Swift's 'A Tale of A Tub' judgments as 'the donkey's carrot of literary
criticism' ?
Ans : (c) “Mac Flecknoe” is a verse mock-heroic
(a) T.S. Eliot (b) I. A. Richards
satire written by John Dryden. It is a direct attack on
Thomas Shadwell, another prominent poet of the time. (c) William Empson (d) Northrop Frye
Ans : (d) Northrop Frye had described value judgment
2. Fanny Burney's 'Evelina' is about :
as ‘the donkey’s carrot of literary criticism’ in his book
(a) a young lady's entry into English fashionable
‘The Anatomy of Criticism’ (1957).
society
(b) English refugees in Paris 8. Select the matching pair :
(c) an English enthusiast for revolutionary (a) 'The Book of the Duchess' : Blanche of
liberty Leicester
(d) money and the world of the country house (b) 'The Canterbury Tales' : The Host of the
Ans : (a) “Evelina, or the History of a Young Lady’s Tabard
Entrance into the World” is a novel written by (c) 'Troilus and Criseyde' : Squire
English author Fanny Burney, and first published in (d) 'The Parliament of Birds' : St. Agnes's Eve
1778. Ans : (a) “The Book of the Duchess” also known as
3. The unexpurgated text of 'Lady Chatterley's 'The Deth of Blaunche' is the earliest of Chaucer’s
major poems, preceded only by his short poem, 'An
Lover' was published after Obscenity trial in :
ABC' and possibly by his translation of 'Romance of the
(a) 1958 (b) 1965
Rose'. Blanche was the wife of his patron John of Gaunt
(c) 1960 (d) 1962 who died in 1368.
Ans : (c) “Lady Chatterley’s Lover” is a novel by
9. 'The Winter Morning' forms part of a longer
D.H. Lawrence first published in 1928, privately. An
unexpurgated edition could not be published openly in poem by :
the U.K. until 1960. (a) Cowper (b) Blake
(c) Burns (d) Byron
4. Sir Andrew Freeport is a character in :
Ans : (a) “The Task: A Poem, in Six Books” is a
(a) Humphry Clinker
poem in blank verse by William Cowper published in
(b) Joseph Andrews 1785. Its six books are called. “The Sofa”, “The
(c) The Coverley Papers Timepiece”, “The Garden”, “The Winter Evening”,
(d) Clarissa “The Winter Morning Walk” and “The Winter Walk at
Ans : (c) The character Sir Andrew Freeport, is in the Noon.”
Coverley essays, which is best recognized by the most 10. Bradley Pearson is the narrator of Iris
popular section “The Spectator Club”.
Murdoch's novel :
5. In which of the following novels does Stein (a) Under the Net (b) Bruno's Dream
feature as a significant character? (c) The Bell (d) The Black Prince
(a) Under Western Eyes (b) Lord Jim Ans : (d) “The Black Prince” is Iris Murdoch’s 15th
(c) Heart of Darkness (d) Nostromo novel, first published in 1973. The Black Prince is
Ans : (b) “Lord Jim” is a novel by Joseph Conrad remarkable for the structure of its narrative. The main
originally published as a serial in Blackwood's character Bradley Pearson, is an author, and the whole
Magazine from October 1899 to November 1900. story moves around him.
UGC NET/JRF English Paper II, June 2007 28 YCT
11. 'Victorian Compromise' is an expression first Ans : (c) Scholar F.O. Matthiessen originated the
used by : phrase “American Renaissance” in his book “American
(a) David Cecil (b) G. K. Chesterton Renaissance (1941).
(c) Lytton Strachey (d) Vincent Buckley 17. 'Gladly would he learn and gladly teach' is a
Ans : (b) In 1913 the Home University Library
line from :
published Chesterton’s “The Victorian Age in
Literature.” The Literature of 18th century England (a) Spenser's 'Fairie Queene'
was revolution but the politics were not. Puritan (b) Goldsmith's 'The Deserted Village'
Theology was rejected but Puritan Practices remained in (c) Chaucer's 'Prologue to Canterbury Tales'
all. Though the society tried to maintain a moral face, (d) Langland's 'Piers Plowman'
underneath was a heartless philosophy known as Ans : (c) Chaucer’s “Prologue to Canterbury Tales”
utilitarianism. Chesterton calls this great gap between
is a collection of 24 stories that runs to over 17,000
theory and practice the ‘Victorian Compromise.’
lines written in Middle English by Geoffrey Chaucer.
12. More's Latin masterpiece 'Utopia' was
translated into English in : 18. Which of the following arrangement of the
(a) 1551 (b) 1498 English plays is in correct chronological order?
(c) 1516 (d) 1532 (a) Justice-The Family Reunion-Saint Joan-The
Ans : (a) “Utopia” is a work of fiction and political Playboy of the Western World
philosophy by Thomas More (1478-1535) published in (b) Saint Joan - Justice - The Playboy of the
1516 in Latin and in English in 1551. Western World - Family Reunion
13. The Anxiety of Influence: A Theory of Poetry (c) The Family Reunion - Saint Joan - Justice -
is written by : The Playboy of the Western World
(a) Maud Bodkin (b) Stephen Spender (d) The Playboy of the Western World - Justice -
(c) Harold Bloom (d) Frank Kermode Saint Joan - The Family Reunion
Ans : (c) “The Anxiety of Influence: A theory of Ans : (d) The correct sequence is in option (d) :
Poetry” is a 1973 book by Harold Bloom. It was the 'The Playboy of the Western World' - 1907 - by
first in a series of books that advanced a new J.M. Synge
“revisionary” or antithetical approach to literary 'Justice' - 1910 - by John Galsworthy
criticism. 'Saint Joan' - 1924 - by G. B. Shaw
14. Who among the following was not a member of 'The Family Reunion' - 1939 - by T.S. Eliot
the group, 'The University Wits'?
19. The second part of The Pilgrim's Progress was
(a) Thomas Nashe (b) Ben Jonson
(c) George Peele (d) Samuel Daniel published in :
(a) 1690 (b) 1678
Ans : (b) The University Wits is a phrase used to name
a group of late 16th century English playwrights who (c) 1686 (d) 1684
were educated at the universities like (Oxford or Ans : (d) “The Pilgrim’s Progress” is written by
Cambridge) who became popular secular writers. John Bunyan. It was published in two parts: the first
Prominent writers of this group were Christopher part in 1678 and the second part in 1684.
Marlowe, Robert Greene, Thomas Nashe (from
20. 'The Egoist' is written by :
Cambridge) and George Peele, Lodge and Lyly (from
Oxford). They emerged in the period from 1580s to 90s (a) Blackmore (b) William Thackeray
and wrote in pre-Shakespearean style. Jonson wrote (c) Meredith (d) Hardy
during and after Shakespeare's era. Ans : (c) "The Egoist" is a tragicomical novel by
15. William Beckford's oriental fantasy Vathek George Meredith published in 1879.
was originally written in : 21. Which is the correct chronological sequence of
(a) Spanish (b) German the following novels?
(c) French (d) Italian (a) Decline and Fall - The Time Machine
Ans : (c) ‘Vathek’ is a Gothic novel written by Nineteen Eighty four - Brave New World
William Beckford. It was composed in French (b) Nineteen Eighty four - Decline and Fall- The
beginning in 1782, and then translated into English. Time Machine - Brave New World
16. The term 'American renaissance' was first used (c) Brave New World - The Time Machine -
by Nineteen Eighty four - Decline and Fall
(a) R. W. B. Lewis (b) Leo Marx (d) The Time Machine - Decline and Fall - Brave
(c) F. O. Matthiessen (d) Richard Chase New World - Nineteen Eighty four
UGC NET/JRF English Paper II, June 2007 29 YCT
Ans : (d) Novels Publishing year (c) Mill : 'On Liberty'
'The Time Machine' by R.L. Stevenson - 1895 (d) Ruskin : 'Unto This Last'
'Decline and Fall' by Evelyn Waugh - 1928 Ans : (d) “Unto this Last” is an essay and book on
'Brave New World' by Aldous Huxley - 1932 economy by John Ruskin, first published in
'Nineteen eighty four' by George Orwell - 1949 December, 1860.
22. Roland Barthes is the author of one of the 28. 'Three Guineas' is the title of a book by :
following texts : (a) E. M. Forster (b) Virginia Woolf
(a) The Death of Tragedy (c) George Orwell (d) G. B. Shaw
(b) The Death of a Hero Ans : (b) “Three Guineas” is a book-length essay by
(c) The Death of the Author Virginia Woolf, published in June 1938.
(d) The Death of Literature
29. Harold Pinter 's first four plays are :
Ans : (c) “The Death of the Author” is a 1967 essay
by French Literary critic and theorist Roland Barthes. (a) The Caretaker, The Room, The Home-
coming, The Birthday Party
23. The author of the Elizabethan sonnet sequence,
(b) The Room, The Dumb Waiter, The Birthday
'Idea', is : Party, The Caretaker
(a) Samuel Daniel (b) Michael Drayton (c) The Homecoming, The Caretaker, Old
(c) Edmund Spenser (d) Fulke Greville Times, Betrayal
Ans : (b) Michael Drayton (1562-1619) was an English (d) The Dumb Waiter, The Caretaker, No Man's
poet and historian who wrote 'Idea', a sonnet sequence Land, Betrayal
published first in 1593 and revised in further editions.
Ans : (b) Pinter's first four plays are–
Samuel Daniel's sequence 'Delia' was published in
The Room -1957
1592.
The Birthday Party -1958
24. Muriel Spark's The Prime of Miss Jean Brodie
The Dumb Waiter - 1959
is a rewriting of the Victorian novel : The Caretaker - 1960
(a) Jane Eyre (b) Villette
30. Identify the odd character out :
(c) Wuthering Heights (d) North and South
Ans : (a) “The Prime of Miss Jean Brodie” is a novel (a) Bosola (b) De Flores
by Muriel Spark, published as a book in 1961 by (c) Iago (d) Kent
Macmillan. In this novel, six ten-year old girls Sandy, Ans : (d) All the characters have a grey shade with
Rose, Mary, Jenny, Monica and Eunice are assigned to some selfish feeling except Kent. He was totally
Miss Jean Brodie, who described herself as being “in devoted to his king.
my prime” as their teacher, during 1930s Edinburgh. 31. Select the matching pair :
25. 'The Romantic Imagination' is the title of a (a) 'The Great Gatsby' : Chicago
book by : (b) 'The Old Man and the Sea' : Cuba
(a) Harold Bloom (b) Graham Hough (c) 'For Whom the Bell Tolls' : Italy
(c) C. M. Bowra (d) M. H. Abrams (d) 'The Sound and the Fury' : Boston
Ans : (c) Maurice Bowra (1898-1971) was an English Ans : (b) “The Old Man and the Sea” is a short novel
classical scholar and known for his wit. He served as written by American author Ernest Hemingway in 1951.
the Vice-Chancellor of the University of Oxford from It tells the story of Santiago, an aging Cuban fisherman
1951 to 1954. His 'The Romantic Imagination' was who struggles with a giant marlin (whale).
published in 1949. 32. 'The page is printed'. This is the last line in a
26. 'Ode on the Spring' was written by : poem by :
(a) Thomas Gray (b) John Keats (a) Sylvia Plath (b) Dylan Thomas
(c) Abraham Cowley (d) William Collins (c) Philip Larkin (d) Ted Hughes
Ans : (a) Thomas Gray (26 December, 1716 - 30 July, Ans : (d) The above line has been taken from the poem
1771) was an English poet, letter-writer and Professor at "The Thought Fox" (1984) by Ted Hughes.
Cambridge University. He is widely known for his 33. T. S. Eliot's 'The Wasteland ' was first
“Elegy Written in a Country Churchyard”,
published in 1751. published in :
(a) The Criterion (b) The Dial
27. Which of the following books was not
(c) The Yale Review (d) New Yorker
published in 1859?
Ans : (a) Wasteland was published in 1922, the 434
(a) Darwin : 'The Origin of Species'
line poem first appeared in The Criterion.
(b) George Eliot : 'Adam Bede'
UGC NET/JRF English Paper II, June 2007 30 YCT
34. 'Relationship' is a long poem by : Ans : (d) The correct sequence is in option (d):
(a) A. K. Ramanujan (b) R. Parthasarathy Beatrice - From Shakespeare's “Much Ado about
(c) Jayanta Mahapatra (d) Kamala Das Nothing” (1599)
Vittoria Corombona - From John Webster's “The
Ans : (c) Jayanta Mahapatra's 'Relationship' was
White Devil” (1612)
published in 1980 in New York.
Hermione – From Shakespeare's “The Winter’s
Kamala Das (31 March 1934-31 May 2009), also Tale” (1623)
known by her one-time pen name Madhavikutty and Christiana – From John Bunyan's "Pilgrim's
Kamala Surayya, was an Indian English poet, litterateur Progress" (1678)
and leading Malayalam author from Kerala, India.
38. Which of the following is in correct
A.K. Ramanujan and R. Parthasarathy along with
Chronological sequence?
Jayanta Mahapatra form the trio of great Indian poets in
(a) 'In Memoriam' - 'Lycidas' - 'An Elegy
English.
Written in a Country Churchyard' - 'Adonais'
35. The phrase, 'bottomless perdition' occurs in (b) 'Adonais' - 'In Memoriam' - 'Lycidas'- 'An
Milton's 'Paradise Lost' in : Elegy Written in a Country Churchyard'
(a) Book I (b) Book IV (c) 'An Elegy Written in a Country Churchyard' -
(c) Book VI (d) Book XII 'In Memoriam' - 'Adonais' - 'Lycidas'
Ans : (a) “Paradise Lost” (1667, 1674) is divided into (d) 'Lycidas' - 'An Elegy Written in a Country
Churchyard' - 'Adonais' - 'In Memoriam'
twelve books and was written by John Milton. The
above phrase appears in Book I, line 47. Ans : (d) The correct sequence is in option (d):
Milton's 'Lycidas' - 1637
36. Which of the following arrangements of
Thomas Gray's 'An Elegy written in a Country
American plays is in the correct chronological Churchyard'- 1750
sequence? Shelley's 'Adonais' - 1821
(a) Mourning Becomes Electra - The Hairy Ape Tennyson's 'In Memoriam' - 1849
- Death of a Salesman - A Streetcar Named
39. The Chartist Demonstration in London
Desire
involving the third presentation of Charter
(b) The Hairy Ape - Death of a Salesman - took place in :
Mourning Becomes Electra - A Streetcar (a) 1842 (b) 1846
Named Desire (c) 1848 (d) 1851
(c) A Streetcar Named Desire - The Hairy Ape -
Ans : (c) Chartism was a working class movement for
Mourning Becomes Electra - Death of a
political reform in Britain which existed from
Salesman 1838-1858. Support for the movement was at its highest
(d) The Hairy Ape - Mourning Becomes Electra in 1839, 1842 and 1848, when petitions signed by
- A Streetcar Named Desire - Death of a millions of working people were presented to the
Salesman House of Commons.
Ans : (d) The correct sequence is in option (d) : 40. 'Life, like a dome of many-coloured glass,
'The Hairy Ape' - 1922 - by Eugene O'Neill Stains the white radiance of Eternity, Until
'Mourning Becomes Electra' – 1931 - by Eugene Death tramples it into fragments'. The above
O'Neill lines occur in :
'A Streetcar Named Desire' – 1947 - by Tennessee (a) 'Dejection: An Ode' (b) 'Adonais'
Williams (c) 'In Memoriam' (d) 'Thyrsis'
'Death of a Salesman' - 1949 - by Arthur Miller Ans : (b) The above lines occur in pastoral elegy
37. Which of the following arrangements of "Adonais: An Elegy on the Death of John Keats",
written by P.B. Shelley for John Keats in 1821.
famous characters is in the correct
41. Arrange the following characters in
chronological order?
chronological sequence :
(a) Vittoria Corombona - Beatrice - Christiana - (a) Mr. Rochester - David Copperfield -
Hermione Rosamond - Bathsheba
(b) Beatrice - Hermione - Vittoria Corombona - (b) David Copperfield - Rosamond - Mr.
Christiana Rochester - Bathsheba
(c) Hermione - Beatrice - Vittoria Corombona - (c) Bathsheba - Mr. Rochester - David
Christiana Copperfield - Becky Sharp
(d) Beatrice - Vittoria Corombona - Hermione - (d) David Copperfield - Bathsheba - Mr.
Christiana Rochester- Rosamond
UGC NET/JRF English Paper II, June 2007 31 YCT
Ans : (a) The correct sequence is in option (a): 47. The Gutenberg Bible was first published in :
Mr. Rochester - From “Jane Eyre” (1847) by (a) 1456 (b) 1516
Charlotte Bronte (c) 1449 (d) 1498
David Copperfield - From “David Copperfield”
Ans : (a) The Gutenberg Bible is also known as the 42
(1850) by Charles Dickens
line Bible, was the first major book, which marked the
Rosamond - From “Middlemarch” (1871) by
George Eliot start of “Gutenberg Revolution”. It came out from
Bathsheba - From “Far From the Madding Mainz, Germany in around 1454-56.
Crowd” (1874) by Thomas Hardy 48. Identify the odd one out :
42. The book, 'The Religion of Man' is written by : (a) 'Persuasion' : Anne Tilney
(a) Sri Aurobindo (b) 'Northanger Abbey' : Catherine Price
(b) Rabindranath Tagore (c) 'Emma' : Jane Fairfax
(c) A. K. Coomaraswamy (d) 'Mansfield Park' : Fanny Dean
(d) V.K. Gokak
Ans : (c) All the characters are protagonists of their
Ans : (b)“The Religion of Man” (1931) is a novels except Jane Fairfax in 'Emma', a novel by Jane
compilation of lectures by Rabindranath Tagore, edited Austen published in 1815. In the novel, Emma
by him and drawn largely from his Hibbert Lectures
Woodhouse is the protagonist and Jane is an orphan.
given at Oxford University in May 1930.
49. Which among the following is in the correct
43. In the poem 'Windhover' Hopkins uses :
chronological sequence?
(a) Alternate Rhyme (b) Disyllabic Rhyme
(a) Sexual Politics - Thinking About Women -
(c) Cross Rhyme (d) Split Rhyme
The Second Sex - The Prisoner of Sex
Ans : (d) “The Windhover” (1877) written by
G.M. Hopkins is a poem written in “Sprung rhythm”, a (b) Thinking About Women - The Prisoner of
meter in which the number of accents in a line are Sex - Sexual Politics - The Second Sex
counted but the number of syllables doesn’t matter. The (c) The Second Sex- Thinking About Women -
poem uses broken or split rhyme produced by dividing a Sexual Politics - The Prisoner of Sex
word at the line break of a poem to make a rhyme with (d) The Prisoner of Sex - The Second Sex -
the end word of another line. For eg. the word 'King- Sexual Politics - Thinking About Women
dom' at the end of the first line is divided to rhyme with Ans : (b) The correct sequence in descending order is
'Wing' in the fourth line.
in option (b):
44. Which Dickens novel attacks the New Poor (1) Thinking About Women - By Margaret L. Andersen
Law of 1834 in the opening chapters ? published in 1983
(a) Great Expectations (2) The Prisoner of Sex - By Norman Mailer published
(b) Hard Times in 1971.
(c) Oliver Twist (3) Sexual Politics - By Kate Millett published in 1970.
(d) Dombey and Son
(4) The Second Sex - By Simon De Beauvoir published
Ans : (c) Charles Dickens's 'Oliver Twist' (1839) in 1949.
attacks the New Poor Law of 1834. The New Poor Law
was really a series of measures that were enacted in 50. Coleridge's 'Kubla Khan' remains' 'a
1834. fragment' because :
45. 'Throw away thy rod, throw away thy wrath, O (a) He was called by Wordsworth who was
my God, take the gentle path' living in Porlock at that time
These lines are taken from a poem by : (b) Dorothy Wordsworth was upset over their
(a) Herbert (b) Donne love affair
(c) Crashaw (d) Vaughan (c) He was interrupted by a caller, a person on
Ans : (a) The above lines are taken from the poem business from Porlock
“Discipline” written by Herbert Spencer (1820-1903). (d) He ran out of his stock of Opium
46. 'Epithalamium' is a : Ans : (c) “Kubla Khan” is a poem written by
(a) song of mourning (b) song of eulogy Coleridge, composed in 1797 and published in 1816. It
(c) nuptial song (d) funeral song is said that Coleridge's sequence of thoughts was broken
Ans : (c) An Epithalamium is a poem written by a caller on business from Porlock and he could not
specifically for the bride on the way to her marital remember it afterwards and so 'Kubla Khan' remained a
chamber. fragment.

UGC NET/JRF English Paper II, June 2007 32 YCT


UGC NET/JRF Exam, December-2007
ENGLISH
Solved Paper-II
Note : This paper contains fifty (50) multiple- Ans : (c) Feminine ending, in grammatical gender, is
choice questions, each question carrying two (2) the final syllable or suffixed letters that mark words as
marks. Attempt all of them. feminine. It can also refer to a feminine ending, in meter
1. The author of The 'Provok'd Husband' was : (Poetry), a line of verse that ends with an unstressed
(a) Etherege (b) Colley Cibber syllable.
(c) Wycherley (d) Vanbrugh 5. The essay 'The Death of the Author' is written
Ans : (d) Sir John Vanbrugh (1664-1726) was an by :
English architect and dramatist. He wrote two (a) Michel Foucault (b) Jacques Derrida
argumentative and outspoken Restoration comedies, (c) Roland Barthes (d) Alvin Kernan
“The Relapse” (1696) and the “Provoked Wife” Ans : (c) “The Death of the Author” is a 1967 essay by
(1697) which have become enduring stage favourites the French Literary critic and theorist Roland Barthes.
but originally occasioned much controversy. He was Barthes' essay argues against traditional literary
knighted in 1714. His fragment of 'The Provok'd criticism’s practice of incorporating the intentions and
biographical context of an author in an interpretation of
Husband' was completed by Cibber in 1728.
a text.
2. Who among the boys in Golding's 'Lord of the
6. Salman Rushdie's 'Shame' is set in :
Flies' is associated with Christ?
(a) Piggy (b) Ralph (a) East Pakistan (b) India and Pakistan
(c) Pakistan (d) None of the above
(c) Jack (d) Simon
Ans : (c) “Shame” is Salman Rushdie’s third novel
Ans : (d) “Lord of the Flies” is a 1954 novel by Nobel
published in 1983. Like most of Rushdie’s work, this
prize-winning English author William Golding, about a book was written in the style of magic realism. The
group of British boys stuck on an uninhabited island, central theme of the novel is that violence is born out of
who try to govern themselves with disastrous results. shame. Rushdie wrote “Shame” after his second novel
3. The complete title of Sterne's novel 'Tristram “Midnight’s Children.”
Shandy' is : 7. Choose the correct chronological sequence in :
(a) The Strange and Surprising Adventures of (a) Lucy Hutchinson's 'Memoirs of the life of,
Tristram Shandy, Gentleman Colonel Hutchinson'-Milton's 'Paradise Lost'-
(b) A True Account of The Life of Tristram Bunyan's 'Pilgrim's Progress'-Dryden's 'The
Shandy, Gentleman Hind and the Panther'
(c) The Life and Opinions of Tristram Shandy, (b) Hutchinson's 'Memoirs'- Bunyan's 'Pilgrim's
Gentleman Progress'-Dryden's 'Hind and the Panther'-
Milton's 'Paradise Lost'
(d) The Strange and Surprising Opinions of
(c) Milton's 'Paradise Lost'-Bunyan's 'Pilgrim's
Tristram Shandy, Gentleman
Progress'-Dryden's 'Hind and the Panther'-
Ans : (c) “The Life and Opinions of Tristram Hutchinson's 'Memoirs'
Shandy, Gentleman” is a humorous novel by Laurence (d) Dryden's 'Hind and the Panther'-Bunyan's
Sterne. It was published in 9 volumes. It purports to be 'Pilgrim's Progress' Hutchinson's 'Memoirs'-
a biography of the eponymous character. Its style is Milton's 'Paradise Lost'
marked by digression, double entendre and graphic Ans : (c) The correct sequence is in option (c).
devices. Milton’s “Paradise Lost” - 1667
4. Feminine ending refers to : Bunyan’s “Pilgrim’s Progress”- 1678
(a) a stressed final syllable in a line of verse Dryden’s “Hind and the Panther”- 1687
(b) the ending of a poem in a stressed syllable Hutchinson’s “Memoirs”- 1806
(c) the ending of a poem in an unstressed 8. 'The Little Minister' is a novel by :
syllable (a) John Galsworthy (b) H.G. Wells
(d) an unstressed final syllable in a line of verse (c) James M. Barrie (d) Rudyard Kipling

UGC NET/JRF English Paper II, December 2007 33 YCT


Ans : (c) “The Little Minister” is a popular (c) The individuating character of a natural thing
sentimental novel by J.M. Barrie, published in 1891. (d) The moment of release from the material
The plot is set in Thrums, a Scottish weaving village world
based on Barrie’s birthplace and a young impoverished Ans : (c) ‘Inscape’ refers to the individuating character
minister with his first congregation. of a natural thing.
9. Which Augustan writer's epitaph reads: 'one 14. In which of these plays does Edward Albee use
who strove with all his might to champion the 'success' myth?
liberty'? (a) A Zoo Story
(a) Alexander Pope (b) Jonathan Swift (b) Who's Afraid of Virginia Woolf?
(c) Henry Fielding (d) Daniel Defoe (c) American Dream
(d) The Death of Bessie Smith
Ans : (b) Jonathan Swift was an Anglo - Irish satirist,
essayist, political pamphleteer, poet and cleric who Ans : (b) “Who’s Afraid of Virginia Woolf?” is a 1966
became Dean of St. Patrick’s Cathedral, Dublin. American black comedy-drama, by Edward Albee. It
examines the breakdown of the marriage of a middle-
10. In which of the following novels incidents aged couple Martha and George.
relating to the declaration of Emergency in 15. 'The voice of poetry comes from a region above
India in 1975 figure?
us, a plane of our being above and beyond our
(a) Farrukh Dhondy's 'Bombay Duck' personal intelligence'. Who among the
(b) Vikram Seth's 'A Suitable Boy' following is the author of the above lines?
(c) Upamanyu Chatterjee's 'English August: An (a) Rabindranath Tagore
Indian Story' (b) A. K. Coomaraswamy
(d) Rohinton Mistry's 'Such Long Journey' (c) Sri Aurobindo
Ans : (d) “Such a Long Journey” is a 1991 novel by (d) Sisir Kumar Ghose
Rohinton Mistry. It was short listed for Booker Prize, Ans : (c) Sri Aurobindo (1872-1950) was and Indian
and won several other awards. In 2010, the book made nationalist Hindu Philosopher, Yogi, Guru and Poet. He
headlines when it was withdrawn from the University of joined the Indian movement for independence from
Mumbai’s English syllabus after complaints from the British rule. His main literary works are “The Life
family of the Hindu nationalist politician Bal Divine.”
Thackeray. 16. The number of poems in Sidney's sonnet
11. Identify the matching pair : sequence Astrophel and Stella is :
(a) Edward II : Zenocrate (a) 99 (b) 47
(b) The Jew of Malta : Barabas (c) 112 (d) 108
(c) The Spanish Tragedy : Horatio Ans : (d) Probably composed in the 1580s,
(d) Tamburlaine : Gaveston Philip Sidney’s "Astrophel and Stella" is an English
sonnet sequence containing 108 sonnets and 11 songs.
Ans : (b) “The Jew of Malta” is a play by
The name derives from the two Greek words, ‘aster’
Christopher Marlowe, probably written in 1589 or 1590. (star) and ‘Phil’ (lover).
Its plot is an original story of religious conflict, intrigue,
17. J. M. Coetzee's 'Foe' is a postmodern retelling
and revenge, and struggle for supremacy between Spain
and the Ottoman Empire that took place on the island of of :
Malta. The title character, Barabas, dominates the play. (a) Ivanhoe (b) Evelina
(c) Robinson Crusoe (d) The Moonstone
12. The future ruin of Troy and the murder of
Agamemnon are referred to by W. B. Yeats in : Ans : (c) “Foe” is a 1986 novel by South African born
Nobel laureate J.M. Coetzee. Woven around the
(a) 'The Second Coming'
existing plot of Robinson Crusoe, “Foe” is written from
(b) 'Circus Animals Desertion' the perspective of Susan Barton.
(c) 'When You Are Old'
18. Johnson's edition of Shakespeare appeared in :
(d) 'Leda and Swan'
(a) 1752 (b) 1765
Ans : (d) “Leda and Swan” is a story and subject in (c) 1791 (d) 1760
art from Greek mythology in which the God Zeus, in
Ans : (b) 'The Plays of William Shakespeare' was an
the form of a swan, seduces or rapes Leda.
18th century edition of the dramatic works of
13. Inscape refers to : Shakespeare, edited by Samuel Johnson and George
(a) The indwelling presence of God in nature Stevens. His work finally appeared in an eight-volume
(b) The universal character of a natural thing edition in 1765.
UGC NET/JRF English Paper II, December 2007 34 YCT
19. The main character in Gogol's 'Dead Souls' is : Ans : (d) “Howards End” is a novel by E.M. Forster,
(a) Oblomov (b) Bazarov first published in 1910, about social conventions, codes
(c) Alyosha (d) Chichikov of conduct, and personal relationships in turn - of - the
Ans : (d) “Dead Souls” is a novel by Nikolai Gogol, century England.
first published in 1842, and widely regarded as an 25. The expression "Thy hand, great Anarch"
exemplar of 19th century Russian Literature. The occurs in a satire by :
purpose of the novel was to demonstrate the flaws and (a) Dryden (b) Pope
faults of the Russian mentality and character Pavel (c) Johnson (d) Swift
Ivanovich. Chichikov is the main character of the novel.
Ans : (b) “The Dunciad” is a landmark literary satire
20. After Shakespeare made his debut as a London by Alexander Pope, published in 1728, 1729 and 1742.
playwright, he was described as an 'upstart The poem celebrates the Goddess Dulness and the
crow' by : progress of her chosen agents as they bring decay
(a) Robert Greene (b) Thomas Lodge imbecility and tastelessness to the Kingdom of Britain.
(c) Christopher Marlowe (d) John Lyly 26. In which of the following novels by Graham
Ans : (a) The first reference to Shakespeare as an Greene does the little girl Brigitta appear?
actor/playwright was in 1592. He was attacked by
(a) The Heart of the Matter
Robert Greene in 1592 as an “Upstart Crow.” An
‘Upstart’ is characteristic of someone who has risen (b) The Power and the Glory
economically or socially but lacks the social skills (c) Brighton Rock
appropriate for this new position. (d) The Quiet American
21. What was the first play of Mrs. Dalloway Ans : (b) “The Power and the Glory” (1940) is a
called? novel by British author Graham Greene. It was initially
(a) Clarissa (b) Hours published in the United States with the title, “The
(c) The Big Ben (d) The Party Labyrinthine Ways.”
Ans : (a) “Mrs. Dalloway”, published on 14 May 27. The author of 'A Satire against Reason and
1925, is a novel by Virginia Woolf that details a day in Mankind' is :
the life of Clarissa Dalloway, a fictional high society (a) Rochester (b) Dryden
woman in post-first World War England. It is one of (c) Gray (d) Swift
Woolf’s best known novels.
Ans : (a) “A Satyre Against Reason and Mankind”
22. Which of the following Caribbean novels (1674) is a satirical poem by the English Restoration
makes intertextual references to Jane Eyre? poet John Wilmot, 2nd Earl of Rochester.
(a) No Telephone to Heaven
28. 'Anagnorisis' is a term used by Aristotle for
(b) Wide Sargasso Sea
(c) Crick Crack Monkey describing :
(d) Between Two Worlds (a) the moment of discovery by the protagonist
(b) the reversal of fortune for the protagonist
Ans : (b) An important dimension of the story
“Wide Sargasso Sea” is related to “Jane Eyre.” It is (c) the happy resolution of the plot
certain that Jean Rhys herself expected that her readers (d) the convergence of the main plot and the sub
had a passing knowledge of “Jane Eyre” even if they plot
didn’t know it in detail. Ans : (a) Anagnorisis is a moment in the play or other
23. The term 'metaphysical poets', was first used work when a character makes a critical discovery.
by : Anagnorisis was the hero’s sudden awareness of a real
(a) Ben Jonson (b) Dr. Johnson situation, the realization of the things as they stood, and
finally the hero’s insight awakens.
(c) Helen Gardner (d) Dryden
Ans : (b) The metaphysical poets is a term coined by 29. In which play by Shakespeare do we find
the poet and critic Samuel Johnson to describe a loose widowed queens questioning the assumptions
group of English lyric poets of the 17th century, whose of male politics?
work was characterized by the inventive use of conceits, (a) Henry V (b) Richard III
and by speculation about topics such as love or religion. (c) Anthony and Cleopatra (d) Hamlet
24. 'Only connect' is the epigraph to a novel by : Ans : (b) “Richard III”, is written approx in 1592,
(a) George Orwell (b) Joseph Conrad depicts the Machiavellian rise to power and subsequent
(c) D. H. Lawrence (d) E. M. Forster short reign of King Richard III of England.

UGC NET/JRF English Paper II, December 2007 35 YCT


30. Which of the following feminist critics used the places them in a very different paradigm. Who
expression 'Gynocriticism' for the first time? is the French poet?
(a) Kate Millet (b) Simone de Beauvoir (a) Baudelaire (b) Mallarme
(c) Elaine Showalter (d) Mary Ellmann (c) Verlaine (d) Apollinaire
Ans : (c) Gynocriticism is the term coined in the 70s Ans : (a) “Black Venus” is an anthology of short
by Elaine Showalter, to construct “a female framework fiction by Angela Carter. It was first published in U.K.
for the analysis of women’s literature.” in 1985. The “Black Venus” of the title story is Jeanne
31. John Keats's poem 'Ode to a Nightingale' was Duval, the lover of poet Charles Baudelaire.
composed in : 37. Strophe, antistrophe and epode form a three-
(a) 1818 (b) 1819 part structure in :
(c) 1820 (d) 1821 (a) a classic ode
Ans : (b) “Ode to a Nightingale” is written in 1819, (b) a Greek chorus
and is a personal poem that describes Keats's Journey (c) a medieval ballad
into the state of ‘negative capability.’ (d) a Petrarchan sonnet
32. The Female Quixote was written by : Ans : (a) “A strophe (anti-clockwise movement from
(a) Henry Fielding (b) Tobias Smollett right to left) is a poetic term originally referring to the
(c) Charlotte Lennox (d) Aphra Behn first part of the Ode in Ancient Greek, followed by the
Ans : (c) "The Female Quixote; or the Adventures of antistrophe (clockwise movement from left to right) and
Arabella" was a novel written by Charlotte Lennox epode (stand still). The term has been extended to also
imitating and parodying the ideas of Miguel de mean a structural division of a poem containing stanzas
Cervantes' "Don Quixote". It was published in 1752. of varying line length.”
33. Which contemporary British poet has Note: Greek Chorus also had the same form of structure
and hence can also be correct.
translated Beowulf?
(a) Thom Gunn (b) Alan Lewis 38. The words 'where are the songe of spring?
(c) Edward Thomas (d) Seamus Heaney Ay, where are they?' occur in :
Ans : (d) Seamus Justin Heaney (1939-2013) was an (a) 'Ode to the West Wind'
Irish poet and recipient of the 1995 Nobel Prize in (b) 'The Seasons'
Literature who is credited to have translated the epic (c) 'Ode to Autumn'
Beowulf. (d) 'Resolution and Independence'
34. 'The Praise of Chimney-Sweepers' is : Ans : (c) “Ode to Autumn” is a poem by Keats,
(a) a poem by William Blake composed in 1819 and published in 1820. This poem is
(b) an elegy by William Wordsworth the final work in a group of poems known as Keats's
(c) an essay by Charles Lamb “1819 odes.”
(d) an essay by William Hazlitt 39. 'Music that gentle on the spirit lies than tired
Ans : (c) “The Chimney Sweepers” is the title of two eyelids upon tired eyes' the above lines occur in
poems by William Blake, published in “Songs of Tennyson's :
Innocence” in 1789 and “Songs of Experience” in (a) 'Tears, Idle Tears' (b) In Memoriam
1794. The poem “The Chimney Sweepers” is set against (c) 'Maud' (d) 'The Lotos Eaters'
the dark background of child labour that was prominent Ans : (d) “The Lotos Eaters” is a poem by Tennyson,
in England. published in 1832 poetry collection. The poem
35. 'The Loneliness of a Long Distance Runner' is describes a group of mariners who, upon eating the
a novel by : Lotos, are put into an altered state and isolated from the
(a) Kingsley Amis (b) Alan Sillitoe outside world.
(c) John Braine (d) John Osborne 40. Which of the following pairs is correctly
Ans : (b) “The Loneliness of the Long Distance matched?
Runner” is a short story by Alan Sillitoe, published in (a) Robert Southey : 'Lady of the Lake'
1959. The work focuses on Smith, a poor Nottingham (b) T.S. Eliot : 'Lake Isle of Innisfree'
teenager from a dismal home in a working class area. (c) A. C. Swinburne : 'The Lady of Shallott'
36. In 'Black Venus' Angela Carter takes elements (d) Thomas De Quincey : 'Recollections of the
from the poetry of a famous French poet and Lakes and the Lake Poets'

UGC NET/JRF English Paper II, December 2007 36 YCT


Ans : (d) The correct pair is option (d). Ans : (c) The correct sequence is–
“Lady of the Lake” is written by Sir Walter Scott. 'Things Fall Apart' - 1958
“Lake Isle of Innisfree” is written by W.B. Yeats. 'No Longer At Ease' - 1960
“The Lady of Shallott” is written by Tennyson. 'Arrow of God' - 1964
“Recollections of the Lakes & the Lake Poets” is 46. Which are the figures of speech used in the
written by Thomas De Quincey.
following lines by Blake?
41. Which famous English novel opens with a
"Tyger, tyger, burning bright
young woman who is 'handsome, clever and
In the forest of the night,
rich,?
(a) Middlemarch (b) Wuthering Heights What immortal hand or eye
(c) Moll Flanders (d) Emma Could frame thy fearful symmetry?''
Ans : (d) “Emma” by Jane Austen is a novel about (a) simile and personification
youthful hubris and the perils of misconstrued (b) irony and synecdoche
romance. The novel was first published in 1815. (c) apostrophe and synecdoche
42. It appears that in Paradise Lost Book I 'Milton (d) metonymy and apostrophe
belongs to the Devil's party without knowing Ans : (d) Metonymy is figure of speech in which a
it'. Who among the following made this thing or concept is called not by its own name but rather
statement? by the name of something associated in meaning. An
(a) Frank Kermode (b) William Empson Apostrophe is a diacritical mark, in language the
(c) C. S. Lewis (d) William Blake marking of the omission of one or more letters.
Ans : (d) It was William Blake who observed that 47. In which of the following American Novels
Milton belonged to the Devil’s party without knowing does 'the Valley of Ashes' occur?
it. The remark implies that Milton unconsciously
(a) Huck Finn
glorified Satan, especially in Book-I of Paradise Lost.
(b) The Red Badge of Courage
43. 'Live Like Pigs' is : (c) Invisible Man
(a) a humorous poem by Pope (d) The Great Gatsby
(b) an allegorical narrative by Orwell
Ans : (d) “The Great Gatsby” is a 1925 novel written
(c) a play by Arden
by American author F. Scott Fitzgerald.
(d) a satirical sketch by Swift
48. To whom is Chaucer referring when he says
Ans : (c) John Arden (1930-2012) was an English
Marxist playwright. The play “Live Like Pigs” was 'He knew the tavern well in every town'?
written in 1956 and was published in 1961. (a) Pardoner (b) Monk
44. 'A woman drew her long black hair out tight (c) Squire (d) Friar
and fiddled whisper music on those strings' Ans : (d) Chaucer refers to ‘Friar’ by saying ‘He knew
From which section of Eliot's The Waste Land the Tavern well in every town’ in his “Canterbury
are the above lines taken? Tales”.
(a) A Game of Chess 49. 'Poetry is a criticism of life under the
(b) What the Thunder Said conditions fixed for such a criticism by laws of
(c) Burial of the Dead poetic truth and poetic beauty'. Who among
(d) Fire Sermon the following, made the above statement?
Ans : (b) The above lines are taken from the section (a) Dr. Johnson (b) Sidney
‘What the Thunder Said.” (c) Matthew Arnold (d) Wordsworth
45. Which is the correct sequence of Achebe's Ans : (c) The above statement is made by Matthew
African Trilogy? Arnold in “The Study of Poetry” (1880).
(a) Things Fall Apart- Arrow of God - No 50. 'She is inspired but diabolically inspired'. Who
Longer At Ease is this lady?
(b) No Longer At Ease - Arrow of God - Things
(a) Candida (b) Major Barbara
Fall Apart
(c) Saint Joan (d) Ann
(c) Things Fall Apart - No Longer At Ease -
Arrow of God Ans : (c) “Saint Joan” is a play by G.B. Shaw, based
(d) Arrow of God - Things Fall Apart - No on the life and trial of Joan of Arc. It was published in
Longer At Ease 1924.
UGC NET/JRF English Paper II, December 2007 37 YCT
UGC NET/JRF Exam, June-2008
ENGLISH
Solved Paper-II
Note : This paper contains fifty (50) multiple- 6. "Ripeness is all" occurs in :
choice questions, each question carrying two (2) (a) King Lear (b) Hamlet
marks. Attempt all of them. (c) Macbeth (d) Julius Caesar
1. Tennyson's poem about women's rights and Ans : (a) “Ripeness is all” occurs in “King Lear”
women's sphere is : Act V, Scene II, written by William Shakespeare. Edgar
(a) Maud (b) In Memoriam said these lines to Gloucester.
(c) Idylls of the king (d) The Princess 7. A. C. Bradley's 'Shakespearean Tragedy' was
Ans : (d) “The Princess” is a Serio-Comic blank published in :
verse narrative poem published in 1847. Tennyson was (a) 1903 (b) 1904
Poet Laureate of U.K. from (1850 to 1892 and remains (c) 1905 (d) 1906
one of the most popular English poets). Ans : (b) Andrew Cecil Bradley (1851-1935) was an
2. "Hymn To Adversity" is a poem by : English literary scholar, best remembered for his work
(a) Thomas Gray (b) Edward Gibbon on Shakespeare. His two major works are
(c) Alexander Pope (d) William Blake Shakespearean Tragedy (1904) and Oxford Lectures
Ans : (a) The allegorical ode “Hymn to Adversity”- on Poetry (1909).
(1753) was written by the poet and literary scholar, 8. "Topsy" appears in :
Thomas Gray (1716-1771). He is widely known for his (a) Uncle Tom's Cabin
“Elegy written in a Country Churchyard” published (b) History of the United States
in 1751. (c) Walden
3. The King James Bible was published in : (d) Tom Sawyer
(a) 1609 (b) 1610 Ans : (a) 'Uncle Tom’s Cabin; or Life among the
(c) 1611 (d) 1612 Lowly' is an anti-slavery novel by American author
Ans : (c) The King James Version (KJV), also Harriet Beecher Stowe. Published in 1852, the novel
known as the Authorized Version (AV) or King “helped lay the groundwork for the Civil War,”
James Bible (KJB) is an English translation of the according to Will Kaufman.
Christian Bible for the Church of England begun in 9. A poem that captures the essence of a moment
1604 and completed in 1611. The books of KJV in a simple image is :
include the 39 books of Old Testament, and 27 books (a) Lyric (b) Ballad
of New Testament. (c) Ode (d) Haiku
4. "IL Migilor Fabro" is the expression Eliot used Ans : (d) “Haiku” is a traditional form of Japanese
for : poetry. Haiku poems consist of 3 lines. The first and last
(a) W. B. Yeats (b) Samuel Beckett lines have 5 syllables and the middle line had 7. The
(c) W. H. Auden (d) Ezra Pound lines rarely rhyme.
Ans : (d) T.S. Eliot used the above expression for 10. Which of the following Shakespearean plays
Ezra Pound for his role as an editor of “The Waste are in the correct chronological sequence?
Land” (1992). (a) The Merchant of Venice-Henry IV Part I-
5. "The Figure a poem Makes" is an essay by : Romeo and Juliet-Richard II
(a) Henry James (b) Sylvia Plath (b) Richard II-Henry IV Part I-Romeo and Juliet-
(c) Robert Frost (d) Wallace Stevens The Merchant of Venice
Ans : (c) Robert Frost (1874-1963) was an American (c) Henry IV Part I-Romeo and Juliet-The
poet. He is highly-regarded for his realistic depiction Merchant of Venice-Richard II
of rural life and his command of American colloquial (d) Romeo and Juliet-Richard II-Henry IV Part I-
speech. The Merchant of Venice

UGC NET/JRF English Paper II, June 2008 38 YCT


Ans : (d) The following list is in order of the first Ans : (b) Gestalt theory emphasized the organization
performance of Shakespeare's plays: and meaning imposed on sensory data during the
• 'Romeo and Juliet' - 1595 process of perception. An often quoted summary of
• 'Richard II' - 1595 Gestalt theory (Gestalt = whole or pattern) is the phrase
• 'The Merchant of Venice' - 1598 ‘the whole is more than the sum of the parts.
• 'Henry IV, Part I' -1600 17. Margaret Laurence is a novelist from :
11. The word 'nature' in the eighteenth century (a) Australia (b) The U.S.A.
literature stands for : (c) Canada (d) Britain
(a) Nature of writing (b) External nature Ans : (c) Margaret Laurence (1926-1987) was a
(c) Human nature (d) The Universe Canadian novelist and short story writer. She was also a
Ans : (c) Note: There is no such evidence but during founder of the Writer’s Trust of Canada, a non-profit
18th century, Romanticism used to rule over human literary organization that seeks to encourage Canada's
beings. Hence, it can be said that human nature is the writing.
best answer among all.
18. 'Sartor Resartus' is a text by :
12. Who is given credit for first using the term
"romantic"? (a) Ruskin (b) Arnold
(a) Friedrich Schlegel (b) Kant (c) Carlyle (d) Burke
(c) Coleridge (d) Schiller Ans : (c) “Sartor Resartus” (meaning ‘The tailor
Ans : (a) Friedrich Schlegel (1772-1829) was a German retailored’) is an 1836 novel by Thomas Carlyle, first
Poet, literary critic and philosopher. He was a zealous published as a serial in 1833-34 in Fraser's Magazine.
promoter of the Romantic Movement. The novel purports to be a commentary on the thoughts
13. Gudrun is a character in a novel by and early life of a German philosopher called Diogenes
(a) James Joyce (b) Virginia Woolf Teufelsdrockh.
(c) D. H. Lawrence (d) E. M. Forster 19. Who of the following is not a university wit?
Ans : (c) Gudrun is a character of the novel “Women (a) Webster (b) Robert Greene
in Love” published in 1920 by D.H. Lawrence. It is a (c) Kyd (d) Marlowe
sequel to his earlier novel “The Rainbow” (1975).
Ans : (a) The University Wits is a phrase used to name
14. 'July's People' is a novel by : a group of late 16th century English playwrights who
(a) Margaret Atwood (b) V. S. Naipaul were educated at the universities (Oxford or
(c) Wole Soyinka (d) Nadine Gordimer Cambridge), who became popular secular writers.
Ans : (d) "July’s People" is a 1981 novel by the Prominent members of this group were Christopher
South African writer Nadine Gordimer. Gordimer wrote Marlowe, Robert Greene, Thomas Nashe (from
the book before the end of apartheid in her prediction of Cambridge) and George Peele, Lyly and Lodge (from
how it would end. The book was notably banned in Oxford). Kyd is sometimes included in this list although
South Africa after its publication.
he wasn't from a university. Webster does not belong to
15. Heroic Couplet is a pair of : this group.
(a) Rhyming iambic pentameter lines
20. Bosola is a character in a play by :
(b) Unrhyming iambic pentameter lines
(a) Ben Jonson (b) Webster
(c) Rhyming iambic hexameter
(c) Christopher Marlowe (d) Thomas Middleton
(d) Unrhyming iambic hexameter
Ans : (a) A Heroic Couplet is a traditional form for Ans : (b) Bosola is a character of the novel “The
English poetry, commonly used in epic and narrative Duchess of Malfi” (1612-13). It is a tragic play written
poetry, and consisting of a rhyming pair of lines in by the English dramatist John Webster in 1612-13. The
iambic pentameter. play begins as a love story with a Duchess.
16. "Gestalt" theory of literature considers text as : 21. "Bliss was it in that dawn to be alive, But to be
(a) a structure of metaphors young was very heaven". This occurs in a poem
(b) a unified whole by :
(c) an experimentation in form (a) William Wordsworth (b) S. T. Coleridge
(d) construction of history (c) Byron (d) Shelley
UGC NET/JRF English Paper II, June 2008 39 YCT
Ans : (a) The above lines occur in the poem, “The Ans : (a) “A Portrait of the Artist as a Young Man”
French Revolution as it Appeared to Enthusiasts at (1916) is the first novel of Irish writer James Joyce.
Its Commencement”, written by William Wordsworth. Published in serial form in 1914-1915, this novel draws
The above lines occur in the 4th -5th line of the poem. on many details of Joyce’s early life.
22. 'A Dance of the Forest' is written by : 27. Victorian Age witnessed a clash between :
(a) Margaret Atwood (b) Nadine Gordimer (a) faith and reason
(c) Chinua Achebe (d) Wole Soyinka (b) tradition and modernity
Ans : (d) “A Dance of the Forest” is one of the most (c) oriental and occidental civilization
recognized of Wole Soyinka’s plays, first presented in (d) romanticism and neo romanticism
1960. In it, Soyinka espouses a unique vision for a New
Ans : (a) Victorian Era (1837-1901) was the period of
Africa, one that is able to forge a new identity free from
Queen Victoria. It was a long period of peace,
the influence of European imperialism.
prosperity, refined sensibilities and national
23. The first Canadian poet is : self-confidence for Britain.
(a) Charles Sangster
28. "For gold in Physique is Cordial/Therefore, he
(b) Oliver Goldsmith loved gold in special" relates to Chaucer's :
(c) Charles Heavysege
(a) Friar (b) Monk
(d) Alexander Machlachlan
(c) Doctor (d) Pardoner
Ans : (c) Charles Heavysege (1816-1874) was a
Ans : (c) Chaucer wrote these lines for the character of
Canadian Poet and dramatist. He was one of the first
Doctor in his “The Canterbury Tales.” These lines
serious poets to emerge in Canada, and his play “Saul”
occur in the description of “The Physician” from line
was hailed on its appearance as the greatest verse drama
413-446.
in English since the time of Shakespeare.
29. The historical novel began in :
24. Heroic quatrain is :
(a) Restoration Period (b) Augustan Age
(a) a stanza in blank verse
(c) Victorian Period (d) Romantic Period
(b) eight line stanza in iambic hexameter
(c) four line stanza in iambic pentameter Ans : (d) Historical fiction rose to prominence during
(d) six line stanza in iambic pentameter the early 19th century as a part of the Romantic reaction
to the Enlightenment. Even though Jane Porter’s 1809
Ans : (c) Heroic quatrain is a poetic stanza consisting of
novel ‘Thaddeus of Warsaw’ is one of the earliest
four lines of iambic pentameter rhyming alternately.
examples, The first true historical novel in English was
The first and third lines and the second and fourth lines
in fact Maria Edgeworth’s “Castle Rackrent” (1800).
rhyme similarly. Every line has ten syllables and every
alternate syllable is stressed starting with the second. 30. The term "Campus novel" is associated with :
25. "Bildungsroman" translated literally means : (a) Graham Greene (b) Kingsley Amis
(a) Development novel (c) Margaret Drabble (d) William Golding
(b) Psychological novel Ans : (b) A Campus novel, also known as an academic
(c) Autobiographical novel novel, is a novel whose main action is set in and around
(d) Campus novel the campus of a university. “The Groves of Academe”
by Mary McCarthy was published in 1952. Kingsley
Ans : (a) In Literary Criticism, a Bildungsroman novel
Amis' “Lucky Jim” is comic & satirical campus novel.
is a story of general growth rather than self-cultivation.
Hence, it is also known as ‘developmental novel. 31. Which of the following author-book pair is
26. A book that faithfully renders a young man's correctly matched?
confused images of love and rejection is : (a) Hard Times – George Eliot
(a) A Portrait of the Artist as a Young Man (b) Heroes and – Walter Pater
(b) Lucky Jim Hero Worship
(c) Daisy Miller (c) Sohrab and Rustom – Mathew Arnold
(d) The Brave New World (d) Ethics of the Dust – Macaulay

UGC NET/JRF English Paper II, June 2008 40 YCT


Ans : (c) The correct option is (c) “Sohrab and 36. The line "I am no Prince Hamlet nor was
Rustom” (1853) is a book by Matthew Arnold. meant to be...." appears in T. S. Eliot's.
th
• “Hard Times” (1854) is the 19 novel by Charles (a) Gerontion
Dickens. (b) The Love Song of J. Alfred Prufrock
• “On Heroes, Hero Worship, and the Heroic in (c) Four Quartets
History” (1841) is a book by Thomas Carlyle. (d) The Waste-Land
• “Ethics of the Dust” (1875) is a book by John Ans : (b) “Love Song of J. Alfred Prufrock” was first
Ruskin. published in 1915 in the issue 'Poetry: A Magazine of
32. The title of William Faulkner's 'The Sound and Verse', later in 1917 it was published as 'Prufrock and
the Fury' is derived from a play by : other Observations'. The poem was highly influenced
(a) William Shakespeare by Dante Alighieri and makes several references to the
(b) Christopher Marlowe Bible and other literary works like Shakespeare’s
'Henry IV Part II', 'Twelfth Night', and 'Hamlet'.
(c) John Webster
(d) Ben Jonson 37. "Fancy" deals with :
(a) "Fixities and definities"
Ans : (a) “The Sound and the Fury” is actually a
reference to one of the most famous ending soliloquies (b) "Imagination and Reason"
in Shakespeare’s play “Macbeth” (Act V, Scene V) (c) "Judgement and Memory"
(d) "Structure and Superstructure"
33. The new humanism school of philosophy and
literary criticism was popular in America Ans : (b) “Fancy” deals with Imagination and Reason
during : while writing.
(a) 1920-1940 (b) 1910-1930 38. Swift's 'Modest Proposal' is written in the form
(c) 1930-1940 (d) 1900-1910 of a:
Ans : (b) The New Humanism School of Philosophy (a) Project in political economy
and Literary Criticism was popular in America during (b) Political allegory
1910-1930. This philosophy is mainly based on the (c) Social Satire
literary and social theories of the English poet and critic (d) Old-Testament history
Matthew Arnold's new humanistic philosophy as a Ans : (c) “A Modest Proposal” is an essay that uses
reaction against the scientifically oriented philosophies satire to make its points. The essay was originally
of literary realism and naturalism. printed in the form of a pamphlet. At the time of its
34. Internal rhyme is : publication, 1729, a pamphlet was a short work that
(a) the basic rhythmic structure of a poem took a stand on a political, religious or social issue, or
(b) rhyming of two words in alternative lines any other issue of public interest.
(c) rhyming of two or more words in the same 39. The main idea of Pope's 'The Dunciad' was
line of poetry taken from :
(d) all the lines of a poem ending with the same (a) Absalom and Achitophel
line pattern (b) Mac-Flecknoe
Ans : (c) Internal rhyme is a poetic device which can be (c) The Medal
defined as metrical lines in which its middle words and (d) An Epistle to Dr. Arbuthnot
Ans : (b) 'Dunciad' was published in various forms
its end words rhyme with each other. It is also called
middle rhyme, since it comes in the middle of lines. from 1728 to 1743. The main idea was taken from John
35. The macabre element in drama was introduced Dryden’s “Mac Flecknoe” (1682), in which the
by : poetaster Thomas Shadwell is crowned ruler of the
(a) John Lyly (b) Marlow Kingdom of Nonsense.
(c) Ben Jonson (d) John Webster 40. Which of the following is not a Browning's
Ans : (d) In works of art, macabre is the quality of work?
having a grim or ghastly atmosphere. Macabre works (a) Dramatic Lyrics (b) Dramatic Personae
emphasize the details of symbols of death. The (c) Men and Women (d) The Palace of Art
outstanding instances in English Literature are John Ans : (d) “The Palace of Art” is an 1832 poem by
Webster, Charles Dickens, Thomas Hardy and many Tennyson. In the poem a man constructs a palace of art
more. for his soul with any amount of art.
UGC NET/JRF English Paper II, June 2008 41 YCT
41. The most obvious feature of Johnson's 'The 46. Sheridan's first play was :
Lives of the Poets' is the equipoise between : (a) The Rivals
(a) Language and form (b) School for Scandal
(b) Style and content (c) St. Patrick's Day
(c) Biography and criticism (d) A Trip to Scarborough
(d) Myth and archetype Ans : (a) Richard Brinsley Butler Sheridan (1751-1816)
Ans : (c) “Lives of the Most Eminent English Poets” was an Irish satirist, a playwright and poet. 'The Rivals',
(1779-81) is a work by Samuel Johnson, comprising his first play, is a comedy of manners, in five acts.
short biographies and critical appraisals of 52 poets, 47. Anti-sentimental comedy is a criticism of :
most of whom lived during the 18th century. It is (a) loss of moral purpose
arranged, approximately by date of death. Six of the (b) excess of emotion
lives has been singled out as the most “important”: John
(c) excess of reason
Milton, John Dryden, Alexander Pope, Joseph Addison,
(d) loss of human feelings
Jonathan Swift and Thomas Gray.
Ans : (b) Anti-sentimental comedy is a reaction against
42. "The Kelson of creation is love". The line
sentimental comedy. The comedy of humor which
occurs in Walt Whitman's
Goldsmith and Sheridan cultivated in 18th century was
(a) Paumonak (b) Passage to India
the reaction against the sentimental comedy of Cibber,
(c) O Captain, My Captain (d) Song of Myself Steele and Kelly.
Ans : (d) “Song of Myself” is a poem by Whitman that 48. Which of the following novel-novelist pair is
is included in his work 'Leaves of Grass'. It has been correctly matched ?
credited as "representing the lore of Whitman’s poetic
(a) Bhabani Bhattacharya – All About H. Hatter
vision".
(b) Nayantara Sahgal – Cry, the Peacock
43. With whom was Dr. Johnson intimately (c) Bhagwandas Gidwani – A Bend in the
associated in his personal life?
Ganges
(a) Boswell (b) Dryden
(d) Arun Joshi – The Apprentice
(c) Alexander Pope (d) Lord Bolingbroke
Ans : (d) Arun Joshi (1939-1993) is mainly known for
Ans : (a) James Boswell (1740-1795) was a Scottish his novels “The Strange case of Billy Biswas” and
biographer and diarist. He is best known for the “The Apprentice.” He won the Sahitya Akademi
biography he wrote of one of his contemporaries Award for his novel, 'The Last Labyrinth' in 1982.
Samuel Johnson.
49. The Indian English poet who addressed the
44. The early religious drama is associated with: question 'of time' in his poetry is :
(a) Superstitions and beliefs (a) Nissim Ezekiel (b) R. Parthsarathy
(b) Mysteries and histories (c) A. K. Ramanujan (d) Gieve Patel
(c) Interludes and mysteries
Ans : (a) Nissim Ezekiel (1924-2004) was an Indian
(d) Miracles and morality Jewish poet. He was a foundational figure in
Ans : (d) Liturgical drama or religious drama, in its post colonial India’s literary history, specifically for
various Christian contexts, originates from the Mass Indian writing in English.
itself, and usually presents a relatively complex ritual 50. Symbolist movement was influenced by:
that includes theatrical elements. (a) Poetic theory of Edgar Allan Poe
45. 'The Tale of Two Cities' has : (b) Stephane Mallarme's Poetry
(a) a sentimental buffoon with a moral purpose (c) Prose of Emerson
(b) a courageous lady in pain (d) Ezra Pound's Cantos
(c) an optimist on verge of collapse Ans : (a) Edgar Allan Poe (1809-1849) was the
(d) a romantic hero with a weakness principal forerunner of the “art for art’s sake”
Ans : (d) “A Tale of Two Cities” (1859) is a novel by movement in 19th century European Literature. Poe’s
Charles Dickens set in London and Paris before and poetry and short stories greatly influenced the French
during the French Revolution. Symbolists of the late 19th century.
UGC NET/JRF English Paper II, June 2008 42 YCT
UGC NET/JRF Exam, December-2008
ENGLISH
Solved Paper-II
Note : This paper contains fifty (50) multiple 7. A "Foot" in prosody is a basic unit of:
choice questions, each question carrying two (2) (a) rhyme (b) length
marks. Attempt all of them. (c) rhythmic measurement (d) height
1. The Victorian period refers to the reign of Ans : (c) The Foot is the basic metrical unit that forms
Queen Victoria of England during: part of a line of verse in most western traditions of
(a) 1830 - 1890 (b) 1837 - 1905 poetry. The unit is composed of syllables the number of
(c) 1837 - 1901 (d) 1850 - 1910 which is limited.
Ans : (c) The Victorian era of British history was the 8. Who of the following is known for aphoristic
period of Queen Victoria’s reign from 20 June 1837 prose style?
until her death, on 22 January 1901. It was a long period (a) William Hazlitt (b) Francis Bacon
of peace, prosperity, refined sensibilities and national (c) John Ruskin (d) G.K. Chesterton
self confidence for Britain.
Ans : (b) Francis Bacon is the father of modern English
2. The Rambler appeared every: prose. He evolved a prose style that proved for the first
(a) Tuesday and Saturday time that English could also be used to express the
(b) Sunday and Wednesday subtle thoughts.
(c) Friday and Monday 9. 'The Confessions of an English Opium Eater'
(d) Thursday and Monday was written by:
Ans : (a) 'The Rambler' was a periodical, strictly a (a) William Hazlitt (b) S.T. Coleridge
series of short papers by Samuel Johnson, published on (c) Landor (d) De Quincey
Tuesday and Saturday from 1750 to 1752 and totals 208 Ans : (d) 'Confessions of an English Opium Eater'
articles. (1821) is an autobiographical account written by
3. "Tottel's Miscellany" contained: Thomas De Quincey. It is about his addictions and its
(a) 30 sonnets (b) 40 sonnets effects on his life.
(c) 50 sonnets (d) 60 sonnets 10. Ireland emerges as the most important
Ans : (d) “Songes and Sonettes”, usually called metaphor in:
'Tottel’s Miscellany', was the first printed anthology of (a) Seamus Heaney (b) Elizabeth Jennings
English poetry, first published by Richard Tottel in (c) Arnold Wesker (d) Edward Albee
1557 in London. Ans : (a) Seamus Heaney (1939-2013) was an Irish
4. "Imagism" is associated with: poet, playwright and lecturer. He is the recipient of the
(a) T.S. Eliot (b) D.H. Lawrence 1995 Nobel Prize in Literature.
(c) E.E. Cummings (d) T.E. Hulme 11. Which of the following Shakespearean plays is
bAns : (d) Imagism was a movement in early 20th in the correct chronological order?
century Anglo American poetry that favored precision (a) King Lear, Hamlet, Much Ado...., Troilus
of imagery and clear sharp language. Thomas Ernest and Cressida
Hulme was an aesthetic philosopher and the ‘father of (b) Much Ado......., Hamlet, King Lear, Troilus
imagism.’ and Cressida
5. The title 'Things Fall Apart' is drawn from a (c) Troilus and Cressida, King Lear, hamlet,
poem by : Much Ado.......
(a) W.B. Yeats (b) Ted Hughes (d) Hamlet, Much Ado...., King Lear, Troilus
(c) W.H. Auden (d) Robert Lowell and Cressida
Ans : (a) “Things Fall Apart” is a post colonial novel Ans : (b) . "Much Ado About Nothing" written in
written by Nigerian author Chinua Achebe in 1958. It is 1598-1599 and published in 1600.
seen as the archetypal modern African novel in English. • “Hamlet” was written between 1599 and published in
The title 'Things Fall Apart' is drawn from a poem 'The 1603.
Second Coming' by W.B. Yeats. • “King Lear” was written between 1605 or 1606 and
6. "Formal Criticism" relates to the structure of: published in 1608.
(a) Literary devices (b) Myths • “Troilus and Cressida” believed to have been
(c) Content (d) Form written in 1603 and was published in 1609.
Ans : (a) Formalism is a school of literary criticism and 12. The major contribution of the Restoration
literary theory having mainly to do with structural period is in the field of:
purposes of a particular text. It is the study of a text (a) Philosophical writings (b) Poetry
without taking into account any outside influence. (c) Drama (d) Letters

UGC NET/JRF English Paper II, December 2008 43 YCT


Ans : (b) Restoration Literature is written during the Ans : (a) “The Loneliness of the Long Distance Runner”
historical period commonly referred to as the English is a short story by Alan Sillitoe, published in 1959.
Restoration (1660-1689). John Milton’s 'Paradise Lost' 19. "Rugby Chapel" is a poem by Matthew Arnold
and all the major poets of the period like Samuel Butler, in the memory of his:
John Dryden, Alexander Pope etc wrote poetry during (a) mother (b) brother
this period. (c) father (d) sister
13. The correct chronological order of the Ans : (c) Matthew Arnold (1822-1888) was a British
following poets is: Poet and critic during the Victorian era. In 1837, Arnold
(a) Byron, Shelley, Keats, Walter Scott first attended the Rugby school where his father was the
(b) Shelley, Walter Scott, Keats, Byron headmaster.
(c) Keats, Byron, Walter Scott, Shelley 20. The earliest woman novelist of significance in
(d) Walter Scott, Byron, Shelley, Keats the 18th century is:
Ans : (d). Walter Scott was a Scottish novelist, born in (a) Mary Edgeworth (b) Aphra Behn
1771 and died in 1832. (c) Mary Russell (d) Mrs. Gaskell
• Lord Byron was an English poet born in 1788 to Ans : (a) Mary Edgeworth (1768-1849) was an Anglo
1824. Irish writer of adults and children’s literature. She was
• P.B. Shelley was an English Romantic poet born in one of the first realist writers in children’s literature.
1792 to 1822. 21. "Cut is the branch that might have grown full
• John Keats was an English Romantic poet born in straight" is a line that occurs in:
1795-1821. (a) Dr. Faustus (b) Hamlet
14. Where 'Angels Fear to Tread' is a novel by: (c) Macbeth (d) The Spanish Tragedy
(a) Virginia Woolf (b) E.M. Forster Ans : (a) “The Tragical History of the Life and
(c) D.H. Lawrence (d) James Joyce Death of Doctor Faustus” commonly referred to
Ans : (b) “Where Angels Fear to Tread” (1905) is a simply as “Doctor Faustus” is a play by Christopher
novel by E.M. Forster, originally entitled Monteriano. Marlowe, first published in 1604.
The title comes from a line in Alexander Pope’s “An 22. Pope's "Essay on Man" can best be read as a
Essay on Criticism,” “For fools rush in where angels poem of:
fear to tread.” (a) Classical understanding of nature
15. The plays of Edward Albee deal with: (b) anti-romantic view of life
(a) problems of middle- class (c) sociological estimate of man
(b) hypocracy of aristocracy (d) philosophical apprehension of life
(c) mechanizations of politics Ans : (d) “An Essay on Man” is a poem published by
(d) simplicity of lower-class Alexander Pope in 1733-1734. It is an effort to
Ans : (b) Edward Franklin Albee III (1928) is an rationalize or rather “vindicate the ways of God to
American playwright. His works are often considered as man”, a variation of John Milton’s claim in the opening
a well crafted realistic examination of the modern lines of Paradise Lost that he will “justify the ways of
conditions. His well known works are “The Sandbox” God to Man.”
(1959), “The Zoo Story” (1958), and "Who’s Afraid of 23. The term "Victorian" evokes the attitudes of:
Virginia Woolf" (1962). (a) philistinism (b) moral earnestness
16. Heptameter consists of: (c) licentiousness (d) transcendentalism
(a) five metrical feet (b) six metrical feet Ans : (b) The term “Victorian” was first used during
(c) seven metrical feet (d) eight metrical feet The Great Exhibition in London (1851), where Victorian
Ans : (c) Heptameter is a type of meter where each line inventions and morals were shown to the world.
of verse contains seven metrical feet. It was used 24. Larry Slade is a character in:
frequently in classical prosody, and in English the line (a) Desire Under the Elms
was used frequently in narrative poetry since the (b) The Emperor Jones
Romantics. (c) The Iceman Cometh
17. In Formalistic school of criticism art is: (d) Hairy Ape
(a) entertainment (b) preaching Ans : (c) 'The Iceman Cometh' is a play written by
(c) matter (d) style American playwright Eugene O’ Neill in 1939 and
Ans : (d) Formalistic school is the base line of published in 1946. The character Larry Slade is a
Formalism of literary criticism and literary theories. In former syndicalist anarchist in the play.
art history, formalism is the study of art by analyzing 25. "Iambus" is a metrical foot consisting of:
and comparing form and style, the way objects are made (a) two syllables (b) three syllables
and their purely visual aspects. (c) four syllables (d) one syllables
18. 'The Loneliness of the Long-Distance Runner' Ans : (a) An Iamb or iambus is a metrical foot used in
is a novel by: various forms of poetry. It refers to a foot comprising
(a) Alan Sillitoe (b) Paul Scott two syllables, an unstressed syllable followed by a
(c) Peter Porter (d) Muriel Spark stressed syllable.
UGC NET/JRF English Paper II, December 2008 44 YCT
26. The lines "Not that he wished is greatness to (a) a study of myths and mythology
create / For politicians neither love nor hate", (b) archetypes of spiritual experience
occur in: (c) recurrence of archetypal patterns
(a) The Rape of the Lock (d) the confluence of different traditions
(b) Absalom and Achitophel Ans : (c) Myth Criticism is mainly known as
(c) Mac Flecknoe Archetypal Literary Criticism. Archetypal literary
(d) Essay on Man criticism is a type of critical theory that interprets a text
Ans : (b) 'Absalom and Achitophel' is a poetic political by focusing on recurring Myths and archetypes in the
satire by John Dryden (1631-1700). The poem is an narratives, symbols, images and character types in
allegory that uses the story of the rebellion of Absalom literary work. As a form of literary criticism, it dates
against King David. back to 1934 when Maud Bodkin published
“Archetypal Patterns in Poetry.”
27. 11,396 definitions of romanticism were given by:
(a) Friedrich Schlegel (c) Edger Allan Poe 33. The phrase "disassociation of sensibility" was
(b) Victor Hugo (d) F.L. Lucas first used by:
(a) Philip Sydney (b) T.S. Eliot
Ans : (d) In “The Decline and Fall of the Romantic (c) John Dryden (d) Mathew Arnold
Ideal” (1948) F.L. Lucas counted 11,396 definitions of
romanticism. Lucas (1894-1967) was an English Ans : (b) Disassociation of sensibility is a literary term
classical scholar and literary critic. first used by T.S. Eliot in his essay “The Metaphysical
Poets” (1921). It refers to the way in which intellectual
28. The term "a stream of consciousness" is thought was separated from the experience of feeling in
derived from the writing of: 17th century poetry.
(a) Mary Sinclair (b) Dorothy Richardson
34. An "Idyll" is usually a poem about a:
(c) William James (d) Gertrude Stein
(a) picturesque city life
Ans : (c) In Literary criticism stream of consciousness, (b) panoramic view of nature
also known as interior monologue. It is a narrative mode (c) picture of industrial society
or device that depicts the multitudinous thoughts and
(d) picturesque country life
feelings which pass through the mind. The term was
coined by William James in 1890 in his Ans : (d) An Idyll or Idyl is a short poem, descriptive of
“The Principles of Psychology.” rustic life written in the style of Theocritus' short
pastoral poems, the “Idylls”. Unlike Homer, Theocritus
29. Sean O' Casey's 'Juno and the Paycock' is: didn’t engage in heroes and warfare. His idylls are
(a) a romantic comedy limited to a small intimate world, and describe scenes
(b) a historical tragedy from everyday life.
(c) a mythical reconstruction 35. "The Lost Generation" refers to the generation
(d) a tragi-comedy that came to maturity in the :
Ans : (d) “Juno and the Paycock” is a play by Sean (a) 1920s (b) 1930s
O’ Casey. The play is highly regarded and often (c) 1910s (d) 1940s
performed in Ireland. It is set in the working class Ans : (a) The “Lost Generation” was the generation that
tenements of Dublin in the early 1920s, during the Irish came of age during World War - I. The term was
Civil War period. popularized by Ernest Hemingway, who used it as one
30. The 'Reader-Response Theory' implies that: of two contrasting epigraphs for his novel, “The Sun
(a) there is no one correct meaning of the text Also Rises” (1926).
(b) the readers of an age construct the meaning 36. The French Revolution had a significant
(c) beliefs determine meaning impact on:
(d) a style is the hallmark of the text (a) Victorian Literature
Ans : (b) Reader-response criticism is a school of (b) Romantic Literature
literary theory that focuses on the reader and their (c) Neo-classic Literature
experience of a literary work. Modern Reader-response (d) Modern Literature
criticism began in the 1960s and 70s particularly in the Ans : (b) The French Revolution was a period of far
US and Germany in works by Roland Barthes etc. reaching social and political upheaval in France that
31. Which of the following author-book pair is lasted from 1789 until 1799.
correctly matched? 37. In which poem does the following line appear?
(a) Walter Pater - 'Unto This Last' "Our birth is but a sleep and a forgetting." :
(b) Browning - 'The Ring and the Book' (a) "Michael" (b) "Immortality Ode"
(c) M. Arnold - 'Idylls of the King' (c) "Rejection: Ode" (d) "Tintern Abbey"
(d) Thackeray - 'Bleak House' Ans : (b) “Ode: Intimations of Immortality from
Ans : (b) “The Ring and the Book” is a long dramatic Recollections of Early Childhood” also known as
narrative poem, and, more specifically, a verse novel of Immortality Ode is a poem by William Wordsworth,
21,000 lines, written by Robert Browning. It was completed in 1804.
published in four volumes from 1868 to 1869 by Smith 38. 'Tale of a Tub' is about:
Elder & Co. (a) Warring political factions
32. "Myth Criticism" focuses on: (b) Struggling lower- class people
UGC NET/JRF English Paper II, December 2008 45 YCT
(c) Controversial philosophical documents Ans : (b) Samuel Richardson (1689-1761) is also
(d) Contending religious parties known as the father of the novel. His major novels were
Ans : (d) “A Tale of a Tub” was the first major work 'Pamela' (1740) and 'Clarissa' (1747-48). Richardson
written by Jonathan Swift, his most difficult satire and was Jane Austen’s favourite novelist.
perhaps his most masterly. The tale is a prose parody 45. The most important of the 'evolutionists'
divided into sections. Composed in 1694 and 1697, it during the Victorian period was:
was eventually published in 1704. It was long regarded (a) Erasmus Darwin (b) Robert Chambers
as a satire on religion. (c) Charles Darwin (d) Alfred Russell Wallace
39. Congreve's 'The way of the world' ends with: Ans : (c) In England on 1859 during the Victorian
(a) a dance party period Charles Darwin published a book titled “On the
(b) punishment of Lady Wishfort Origin of Species by Means of Natural Selection.” It
made the Victorians "experience a great age of doubt".
(c) sending of Mr. Fainall to prison
(d) reconciliation of Petulant Whitwood 46. A philosophical attitude pervading much of
modern literature is:
Ans : (d) “The Way of the World” is a play written (a) Absurdism (b) Dadaism
by the English playwright William Congreve. It
(c) Imagism (d) Surrealism
premiered in early March 1700. It is widely regarded as
one of the best restoration comedies. It ends with Ans : (a) Absurdism is a philosophical attitude
reconciliation of Petulant Whitwood. pervading much of modern drama and fiction, which
underlines the isolation and alienation that human
40. On seeing whom does Miranda exclaim, "O, beings experience.
father, surely that is a spirit. Lord! How it
47. The term "magic realism" was first introduced
looks about?"
by :
(a) Caliban (b) Ferdinand (a) Hannah Arendt (b) Franz Roh
(c) Alonso (d) Stephano (c) Jean Arp (d) Peter Behrens
Ans : (b) The above lines have been taken from Ans : (b) The term 'magical realism' first appeared in
Shakespeare’s famous play “The Tempest,” written in 1955. Although, the term 'Magischer Realismus'
1610-11. translated as 'magic realism', was first used by German
41. Secular influences on the early English drama art critic Franz Roh in 1925. “Magic realism is an
were: acceptance of magic in the rational world.” It is also
(a) political squabbles, religious sermons and known as 'fabulism' or 'marvelous realism'.
social customs 48. The Indian English novelist who, for the first
(b) rural politicking, hypocracy of the elite and time addressed the question of language and
falsity of aristocracy indigenous experience was:
(c) village festivals, folk plays and minstrels (a) Mulk Raj Anand (b) R.K. Narayan
(d) middle-class life, moral beliefs and uprising (c) Arun Joshi (d) Raja Rao
of the subalterns Ans : (b) R.K. Narayan (1906-2001), full name
Ans : (d) Secular influences on the early English drama Rasipuram Krishnaswami Iyer Narayanswami, was an
were middle-class life, moral beliefs and uprising of the Indian writer, who has been compared to American
subalterns. William Faulkner, who also created a fictional town that
stood for reality. Narayan is the first to address the
42. John Bunyan's 'The Pilgrim's Progress' was question to language and indigenous experience in
written while he was: Indian English fiction.
(a) in prison (b) on a pilgrimage
49. G.V. Desani's All About H. Hatterr is written
(c) on a social mission (d) in a church in the :
Ans : (a) “The Pilgrim’s Progress” is a 1678 Christian (a) stream of consciousness mode
allegory written by John Bunyan. It is regarded as one (b) first person narrative mode
of the most significant works of religious English (c) picaresque mode
Literature. Bunyan wrote this while he was in Bedford Jail. (d) naturalistic mode
43. In Juvenalian satire the speaker is: Ans : (c) “All About H. Hatterr” (1948) is a novel by
(a) a political orator (b) a propagandist G.V. Desani chronicling the adventures of an Anglo
(c) a social revolutionary (d) a serious moralist Malay man in search of wisdom and enlightenment.
Ans : (d) Juvenalian satire came into existence after 50. The rhyme scheme of the Shakespearean
Roman satirist Juvenal. He wrote a collection of sonnet is:
satirical poems, named “Satires” in the late 1st and (a) abab, cdcd, efef, gg (b) abba, cddc, effe, gg
early 2nd centuries AD. (c) abab, cdcd, efef, gh (d) aabb, ccdd, eeff, gg
44. Jane Austen's Pride and Prejudice most clearly Ans : (a) The Shakespearean Sonnet has the rhyme
shows the influence of: scheme abab, cdcd, efef, gg, forming three quatrains
(a) Fielding (b) Richardson (four lines in a group) and a closing couplet (two
(c) Smollett (d) Sterne rhymed lines).
UGC NET/JRF English Paper II, December 2008 46 YCT
UGC NET/JRF Exam, June-2009
ENGLISH
Solved Paper-II
Note: This paper contains fifty (50) multiple- oppressive patriarchal society and the status of women.
choice questions. Each question carrying two (2) It is a prequel or response to Charlotte Bronte’s novel
marks. Attempt all the questions. "Jane Eyre" (1847).
1. In an 1817 review of Coleridge’s 'Biographia 4. “His life was gentle and the elements/So mixed
Literaria', Francis Jeffrey coined the term ‘Lake in him, that Nature might stand up/And say to
School of Poets’ grouping… all the world, This was a man!”
(a) Wordsworth, Coleridge and Crabbe Who is the speaker, and about whom is this
(b) Wordsworth, Coleridge and Byron spoken?
(c) Wordsworth, Coleridge and Hazlitt (a) Enobarbus on Antony
(d) Wordsworth, Coleridge and Southey (b) Brutus on Caesar
Ans : (d) The Lake School of Poets is a group in (c) Cleopatra on Antony
which were listed the poets who lived in the Lake (d) Marc Antony on Caesar
District of England during the beginning of 19th Ans : (d) Marc Antony has said the above sentence to
century. They are mainly poets of Romantic Brutus in 'Julius Caesar', in Act V Scene V.
Age/movement. 5. “When my love swears that she is made of truth/
2. “I am the enemy you killed, my friend/I knew I do believe her, though I know she lies.”
you in this dark…” The author of these lines is…
The above lines are taken from… (a) Philip Sidney
(a) “The Soldier” (b) Edmund Spenser
(b) “Dulce et Decorum Est.” (c) Christopher Marlowe
(c) “To His Dead Body” (d) William Shakespeare
(d) “Strange Meeting” Ans : (d) Shakespeare wrote these lines in his Sonnet
Ans : (d) The above lines are taken from the poem 138. Theme in this Sonnet is the comfort that 'lies'
'Strange Meeting' written by Wilfred Owen. He also bring to an insecure mind.
wrote "In-sensibility", “Anthem for Doomed 6. The poetry of Wordsworth and Coleridge was
Youth” and "Futility". notably influenced by…
3. Below are two sets of texts one of which has (a) The Napoleonic Wars
inspired the other. Match the text with its (b) The Glorious Revolution
inspiration : (c) The French Revolution
(i) Coral Island (d) Poor Laws
(ii) The Odyssey Ans : (c) French Revolution exerted a profound
(iii) The Mahabharat influence on the poetries of Romantic Age. It
(iv) Jane Eyre signalized the arrival of a new era of fresh thinking and
(v) The Great Indian Novel introspection.
(vi) Wide Sargasso Sea 7. “Great wits are sure to madness near allied
(vii) Omeroos And thin partitions do their bounds divide.”
(viii) Lord of the Flies The above lines appear in…
(a) (i)- (v), (ii)-(vii), (iii)- (viii), (iv)- (vi) (a) Mac Flecknoe
(b) (iv)- (vii), (iii)-(vi), (i)- (viii), (ii)-(v) (b) Absalom and Achitophel
(c) (iii)- (v), (iv)-(vi), (i)-(vii), (ii)-(viii) (c) Essay on Man
(d) (i)-(viii), (ii)-(vii), (iii)-(v), (iv)- (vi) (d) Alexander’s Feast
Ans : (d) Ans : (b) 'Absalom and Achitophel' (1681) is a
(i-viii) "Coral Island" is a children’s fiction that tells landmark poetic political satire by John Dryden. The
the adventure of three boys. This adventure literature poem is an allegory that uses the story of the rebellion
inspired William Golding’s "Lord of the Flies". of Absalom against King David attacking his
(ii-vii) "Omeroos" (1990) means Homer in Greek and contemporary rivals Shadwell and others.
has little touch of Homer’s Iliad and 'Odyssey'. 8. Who among the following developed the term
(iii-v) "Mahabharat" and "Great Indian Novel" are strategic essentialism?
attached to one another because of common (a) Edward Said
background. (b) Gayatri Chakravorty Spivak
(iv-vi) "Wide Sargasso Sea" (1966) written by Jean (c) Homi Bhabha
Rhys deals with the background of marriage, (d) Aijaz Ahmed
UGC NET/JRF English Paper II, June 2009 47 YCT
Ans; (b) Gayatri Chakravorty Spivak is an Indian Ans : (b) 'The Somonyng of Everyman' (The
literary critic and theorist, who introduced this summoning of Everyman), usually referred to simply
strategic essentialism concept in post colonial theory as 'Everyman', is a late 15th century morality play.
during 1980. 14. Which of the following sets would you call the
9. David Malouf's 'An Imaginary Life' is a retelling poets of the Movement?
of the story of : (a) Elizabeth Jennings, Philip Larkin, John Wain
(a) Aristotle (b) Juvenal (b) W. H. Auden, Cecil Day Lewis, Stephen
(c) Ovid (d) Horace Spender
(c) T.S. Eliot, Richard Aldington, Ezra Pound
Ans : (c) 'An Imaginary Life' (1978) is written by (d) Alan Brownjohn, C.H. Sissons, Anthony
Australian writer David Malouf. It tells the story of the Thwaite
Roman poet Ovid during his exile in Tomis.
Ans : (a) The Movement was a term coined in 1954 by
10. ‘Jabberwocky’ is a character in… J.D. Scott, to describe a group of writers including
(a) The Importance of Being Earnest Jennings, Larkin, Wain, Kingsley Amis, Thom Gunn,
(b) Fra Lippo Lippi Donald Davie etc. The Movement poets were
(c) Through the Looking Glass considered anti-romantic. To these poets, good poetry
(d) Goblin Market meant simple, sensuous content and traditional,
Ans : (c) 'Through the looking Glass' (1871) is a conventional and dignified form.
novel by Lewis Carroll. It’s the sequel to 'Alice’s 15. Doris Lessing’s interest in………is widely
Adventures in Wonderland' (1865). The novel recognized.
belongs to children's fiction. (a) Hinduism (b) Sufism
11. Which of the following statements is the most (c) Zen (d) Judaism
accurate regarding Edwards Said’s thesis in Ans : (b) Doris Lessing (1919-2013) was a British
Orientalism? novelist, poet, and playwright. Lessing was awarded
(i) The Europeans used the East dialectically to the 2007 Nobel Prize in Literature.
describe their self-image as irrational and 16. Periphrasis, which is a roundabout way of
primitive. speech/writing, is also known as…
(ii) The Oriental people used the West (a) synecdoche (b) allusion
dialectically to define their self-image as (c) understatement (d) circumlocution
irrational and primitive. Ans : (d) Periphrasis originates from a Greek word
(iii) The Europeans used the East oppositionally to 'periphrazein' which means “talking around.” It is a
define their self-image as rational and modern. stylistic device that can be defined as the use of
(iv) The Oriental people used the West excessive and longer words to convey a meaning
oppositionally to define their self-image as which could have been conveyed with a shorter or in a
rational and modern. few words expressed by circumlocution.
(a) (iii) (b) (iv) 17. Arrange the following in chronological order…
(c) (i) and (iv) (d) (ii) and (iii) (i) The death of Shakespeare
(ii) Accession of James I to the English throne
Ans : (c) Statements (i) and (iv) are most accurate
(iii) Caxton and the printing press
regarding Said's thesis in ‘Orientalism’.
(iv) The Norman Conquest of England
12. Assertion (AST): Literary and historical (a) (iv) (iii) (ii) (i)
periodization often has nothing to do with the (b) (iii) (iv) (ii) (i)
lifetime of writers. Thus we see two writers born in (c) (iii) (iv) (i) (ii)
the same year belonging to two separate periods. (d) (iv) (iii) (i) (ii)
Reasoning (R) Example: Thomas Carlyle and Ans : (a) The correct sequence is:
John Keats were born in 1795. In standard literary (iv) Norman Conquest of England (1066). Normans
histories, Keats is a Romantic and Carlyle, a took possession of England after the decisive Battle of
Victorian. Hastings in 1066 under the leadership of King
(a) (AST) and (R) are correct. William.
(b) (AST) is correct; (R) is incorrect. (iii) Caxton and the printing press (1474). William
(c) (AST) and (R) are incorrect. Caxton set up a press in Bruges about 1474, and the
(d) (R) does not follow from (AST). first book printed in English, 'Recuyell of The History
Ans : (a) Assertion and Reason are both correct. Troye', was published there in 1475.
13. Everyman is… (ii) Accession of James I to the English throne (1603).
(a) a medieval play based on an episode from the (i) Death of Shakespeare (1616).
Bible 18. “The Muse of History” is a classic postcolonial
(b) a medieval morality play essay by :
(c) a Tudor interlude (a) Ngugi wa Thiongo (b) Chinua Achebe
(d) a miracle play (c) Wilson Harris (d) Derek Walcott
UGC NET/JRF English Paper II, June 2009 48 YCT
Ans : (d) Sir Derek Alton Walcott received Nobel 22. 'Playing in the Dark' by Toni Morrison is a
Prize in 1992, and is currently Professor of Poetry at series of reflections on :
the University of Essex. His works include the (a) Jazz music
Homeric epic poem "Omeroos' (1990) (b) Disability sports
19. “Do I contradict myself? (c) Whiteness and the literary imagination
Very well then, I contradict myself, (I am large, (d) Black American folklore
I contain multitudes.)” Ans : (d) “Playing in the Dark” focuses primarily on
The above lines are from… the literary imagination of European Americans and
(a) Walt Whitman how it has been impacted by the coexistence of
(b) Edgar Allan Poe Africans and Europeans in this country. Morrison
(c) Ralph Waldo Emerson examines the kind of roles African American
(d) John Greenleaf Whittier characters have been given in novels.
Ans : (a) The above lines are from the poem “Song of Note - “Whiteness and the literary imagination” is the
Myself” by Walt Whitman. The poem was first complete name of the novel, and not a reflection.
published without sections as the first of twelve 23. “He’s not the brightest man in the world” is an
untitled poems in the first (1855) edition of 'Leaves of example of :
Grass'. (a) Chiasmus (b) Hyperbole
20. “Verses on the Death of Dr. Swift” was written (c) Litotes (d) Simile
by… Ans : (c) Litotes is a special form of understatement.
(a) Jonathan Swift (b) Alexander Pope The assertion of an affirmative by negating its
(c) Samuel Johnson (d) James Boswell contrary: “He’s not the brightest man in the world”
Ans : (a) The satirist Jonathan Swift wrote the poem meaning “He is stupid.”
“Verses on the Death of Dr. Swift” just to describe 24. The term ‘horizon of expectations’ is associated
what his contemporaries' reaction to his death will with…
probably be 'Why wait until you die to describe your (a) Wolfgang Iser (b) Stanley Fish
own funeral'? Jonathan Swift answered by writing a (c) Harold Bloom (d) H.R. Jauss
verse on his own death.
Ans : (d) The phrase “Horizons of Expectation” is a
21. Match the following elegies with the persons for term fundamental to German academic Hans Robert
whom they were written: Jauss’s Reception theory. According to Jauss, the
(i) ‘Lycidas’ reader approaches a text armed with the knowledge
(ii) Arthur Hugh Clough
and experience gained from interactions with other
(iii) ‘Adonais’
texts. These earlier texts arouse familiarity for the
(iv) A.H. Hallam
reader based on expectations and rules of genre and
(v) In Memoriam
style.
(vi) Edward King
(vii) Thyrsis 25. The following writers have something in
(viii) Keats common:
(a) (i)- (vi), (iii)-(iv); (vii)- (ii); (v)- (vi) Mary Seacole J.A. Froudes
(b) (iii)- (viii); (i)-(iv); (iii)- (ii); (v)-(ii) Mary Kingsley Anthony Trollope
(c) (i)-(vi); (iii)-(viii); (v)-(iv); (vii)- (ii) What is it?
(d) (v)- (vi); (i)-(viii); (iii)-(ii); (vii)-(iv) (i) They are all Victorians
(ii) They are all writers of children’s fiction
Ans : (c) The correct sequence is:
(iii) They are all members of one literary guild
(i-vi) “Lycidas” is written by John Milton in 1637. It
(iv) They are all travel writers
is a Pastoral elegy. Milton dedicates this poem to his
(a) (i) and (ii) (iii) and (iv)
friend Edward King, who drowned when his ship sank
(b) (ii) and (iv) (i) and (iv)
in the Irish Sea off the coast of Wales in Aug 1637.
(iii-viii) “Adonais” is a pastoral elegy written by Ans : (d) All of them are Victorian writers and can be
Percy Bysshe Shelley in 1821, on the death of John classified under travel or adventure writers. Mary
Keats. Seacole (1805-1881) was a Jamaican woman of
(v-iv) “In Memoriam” is written by Alfred Lord Scottish and African descent. Her biography,
Tennyson for his Cambridge friend Arthur Henry 'Wonderful Adventures of Mrs. Seacole in Many
Hallam who died in 1833. The poem was completed in Lands' (1857) is one of the earliest biographies.
1849. James Anthony Froudes (1818-1894) was an
(vii-ii) “Thyrsis” is written by Matthew Arnold in 1865 English historian known for his 'History of England
for his friend, the poet Arthur Hugh Clough, who had from the fall of Wolsey to the Defeat of the Spanish
died in 1861. Armada' (1893).
UGC NET/JRF English Paper II, June 2009 49 YCT
Mary Henrietta Kingsley (1862-1900) was an (i) literal
English ethnographic and scientific writer and explorer (ii) metaphorical
whose travels throughout West Africa resulted in (iii) ambiguous
shaping European perceptions of African cultures and (iv) neither literal nor metaphorical
British Imperialism. She wrote 'Travels in West (a) (i) and (ii) are correct.
Africa' (1897). (b) (i) is correct
Anthony Trollope (1815-1882) was one of the (c) (ii) is correct
most successful, prolific and respected English (d) (iii) and (iv) are correct.
novelists. Among his best loved works is a series of Ans : (b) In the above statement 'Wooden leg' has a
novels collectively known as the Chronicles of literal sense, although 'Wooden leg' is a phrase which
Barsetshire which includes his best work ‘Barchester originates from the tales of pirates. Usually, there used
Towers’ published in 1857. to be one character who had a wooden leg and could
26. The immediate source of Christopher Marlowe’s drink like no other. Hence, this term is used for
'Doctor Faustus' is… someone who drinks a lot.
(a) A French narrative Note- It can be metaphorical as well as literal, depends
(b) A Dutch narrative on the usage.
(c) A German narrative
31. Prosody studies :
(d) None of the above
(a) Line endings (b) Meanings of words
Ans : (c) The immediate source of Christopher (c) Patterns of prose (d) Metrics
Marlowe’s 'Doctor Faustus' is a German work 'Historia
Ans : (d) “Prosody” signifies the systematic study of
Von D. Iohan Fausten' (1587), which was translated
versification in poetry; that is, the principles and
into English in 1592. It is this English translation from
which Marlowe took the idea. practice of meter, rhyme, and stanza forms.
Sometimes, it is extended to include also the study of
27. Who among the following were associated with
speech sound patterns and effects such as alliteration
the Irish Dramatic Movement?
assonance, euphony and onomatopoeia.
(a) Lady Gregory, W. B. Yeats, J.M. Synge
(b) Jonathan Swift, R. B. Sheridan, G.B. Shaw 32. Which of the following is a major Jacobean
(c) W.B. Yeats, J.M. Synge, G.B. Shaw play?
(d) W.B. Yeats, Patrick J. Kavanagh, Seamus (a) Everyman (b) Gorboduc
Heaney (c) Romeo and Juliet (d) The Duchess of Malfi
Ans : (a) or (c) Irish Dramatic Movement was Ans : (d) 'The Duchess of Malfi' is a tragic play
essentially a folk art which combined realism of written by the English dramatist John Webster in 1612-
environment and character of their native Island. 13. The play begins as a love story, with a Duchess.
Note - The answer for this can be (a) and (c), both. Jacobean drama continued the trend of stage violence
28. The term 'diaspora' was originally applied to the and horror as in the Duchess of Malfi.
following ethnic group : 33. 'Understanding Poetry' used to be a classic
(a) Jews (b) Muslims textbook that encapsulates the principles of…
(c) Hindus (d) French Canadians (a) New Historicism (b) New Aristotelianism
Ans : (a) The Jewish diaspora or Exile, to which the (c) New Criticism (d) The New Left
term was originally applied, refers to the dispersion of Ans : (c) “Understanding Poetry” was an influential
Israelites, Judahites, and later Jews out of what is book by Cleanth Brooks and Robert Penn Warren, first
considered their ancestral homeland and the published in 1938. The book influenced New Criticism,
communities built by them across the world. and went through its fourth edition in 1976.
29. Who among the following is NOT a 'University 34. What century is variously called The Age of
Wit'? Enlightenment, the Age of Sensibility, The
(a) Christopher Marlowe (b) George Peele Augustan Age and the Age of Prose and Reason?
(c) Robert Greene (d) Ben Jonson (a) sixteenth century (b) seventeenth century
Ans : (d) The University Wits is a phrase used to name (c) eighteenth century (d) nineteenth century
a group of late 16th century English playwrights who Ans : (c) The 18th century in English literature is
were educated at the universities (Oxford or known as the Augustan Age, Neo- Classical Age and
Cambridge), who became popular secular writers. the Age of Reason. The Enlightenment thinkers of the
Prominent members of this group were Christopher 18th century such as Voltaire and Rousseau
Marlowe, Robert Greene, Thomas Nashe and George popularized the idea of the Age of Reason.
Peele. Enlightenment movement advocated scientific
30. When a person has a wooden leg, we are apt to rationalism and empirical knowledge.
say, ‘He has a wooden leg.’ Now this wooden leg 35. What is common to the following poems?
is ….... Wordsworth’s “The Recluse”
UGC NET/JRF English Paper II, June 2009 50 YCT
Shelley’s “The Triumph of Life” 40. In which eclogue of 'The Shepheardes Calender'
Byron’s “Don Juan” does Spenser praise Queen Elizabeth I?
Keats’ “Hyperion” (a) January (b) April
(a) They are all elegies (c) August (d) November
(b) They are all unfinished poems Ans : (b) "The Shepheardes Calender" was Edmund
(c) They are all divided into cantos Spenser’s first major poetic work, published in 1579.
(d) They are women-centred poems The poem introduces Colin Clout a folk character and
Ans : (b) They are all unfinished poems. depicts his life as a shepherd through the twelve
• Wordsworth had for years been making plans to months of the year. While the January pastoral tells of
write a long philosophical poem in three parts, the unhappy love of Colin for Rosalind, the spring time
which he intended to call “The Recluse.” of April calls for a song in praise of Elizabeth.
• Shelley’s “The Triumph of Life” was the last 41. Which of the following is NOT the opening of
major work, before his death in 1822. The work the well-known Romantic poem?
was deft unfinished. (a) My heart aches, and a drowsy numbness pains/
• Byron’s “Don Juan” is a satiric poem. Byron my sense
completed 16 cantos leaving an unfinished 17th (b) Hail to thee, blithe spirit!
canto before his death in 1824. (c) Margaret, are you grieving/ Over Golden grove
• Keats’s “Hyperion” is an abandoned epic poem. unleaving?
He wrote the poem from late 1818 until the spring (d) The world is too much with us
of 1819 when he gave it up. Ans : (c) The opening lines in option (c) “Margaret,…”
36. Who among the following called the novel ‘the belongs to Victorian era and not Romantic period.
bright book of life’? • "My heart aches, and a drowsy numbness pains" are
(a) D. H. Lawrence (b) James Joyce the opening lines of the poem “Ode to a
(c) Virginia Woolf (d) Aldous Huxley Nightingale” by Keats (1795-1821).
Ans : (a) David Herbert Richards Lawrence • "Hail to thee, blithe spirit…." are the opening lines of
(1885-1930) was an English novelist, poet, playwright, the poem “To a Skylark” by P.B. Shelley
essayist, literary critic and painter. His works represent (1792-1822).
an extended reflection upon the dehumanizing effects • "Margaret, are you grieving Over Golden grove
of modernity and industrialization. The above quote is unleaving? …." are the opening lines taken from the
attributed to him. poem “Spring and Fall” (1880) by Hopkins (1884-
37. “Ripeness is all” is a line from… 1887).
(a) Hamlet (b) King Lear • "The world is too much with us ….." are the opening
(c) Othello (d) Macbeth lines of the poem “The world is too much with us”
Ans : (b) This line is from “King Lear” Act 5, Scene 2 by William Wordsworth (1770-1850).
“Men must endure their going hence, even as their 42. “Politics and the English Language” is an essay
coming wither: Ripeness is all". It speaks to being by :
ready for inevitable death, but also, savoring your life (a) F. R. Leavis (b) Terry Eagleton
and making the most of every moment until the end. (c) George Orwell (d) Raymond Williams
38. U. R. Ananthamurthy’s 'Samskara' was Ans : (c) “Politics and the English Language” (1946)
translated by… is an essay by George Orwell that criticizes the “ugly
(a) Himself (b) Girish Karnad and inaccurate” written English of his time and
(c) H. S. Shivaprakash (d) A. K. Ramanujan examines the connection between political orthodox and
Ans : (d) 'Samskara' is a Kannada play by U.R. the debasement of language.
Ananthamurthy. A.K. Ramanujan translated it into 43. “The mind-forged manacles” is phrase from:
English in 1978. The source of title is from a Sanskrit (a) “London” (b) “Eternity”
word that means “rite of passage.” (c) “A Poison Tree” (d) “I Asked a Thief”
39. Abel Whittle is a character in: Ans : (a) “The mind forged manacles” is the fourth line
(a) The Return of the Native of the second quatrain in William Blake’s poem
(b) The Mayor of Caster-bridge "London". This metaphorical expression represents the
(c) Far from the Madding Crowd mental state the people of London are inflicting on
(d) Tess of the D’ Urbervilles themselves due to their own perceived social status.
Ans : (b) 'The Mayor of Caster-bridge' (1886), 44. “He is not fully recognized at home; he is not
subtitled “The Life and Death of a Man of Character”, recognized at all abroad. Yet I firmly believe
is a novel by British author Thomas Hardy. The book is that the poetical performance of ….is, after that
one of Hardy’s Wessex novels, all set in a fictional of Shakespeare and Milton, undoubtedly most
rural England. considerable in our language.”
UGC NET/JRF English Paper II, June 2009 51 YCT
To whom does Matthew Arnold refer in the 46. This passage casts doubt on :
above statement? (a) the assumption called literature.
(a) Edmund Spenser (b) John Keats (b) the idea of literature.
(c) William Wordsworth (d) S.T. Coleridge (c) the institution of literature.
Ans : (c) Arnold refers to Wordsworth in the above (d) the notion of literature.
lines. Ans : (d) This passage casts doubt on the notion of
45. The Globe Theatre opened in: literature. (line 2).
(a) 1585 (b) 1593 47. 'Literature' is unsustainable because…
(c) 1599 (d) 1603 (a) we are unclear as to what it means.
Ans : (c) The Globe Theatre was a theatre in London (b) we are unsure as to its message.
associated with Shakespeare. It was built in 1599 by (c) we are not persuaded that the claims made for
it are allowable and acceptable.
Shakespeare’s playing company.
(d) we cannot prove that its definitions are the
(46-50) Read the following passage carefully, rights and the only possible ones.
and select the right answers from the alternatives
Ans : (d) According to the passage, we cannot prove
given below in the questions 46 to 50:
that its definitions are right and the only possible ones.
We need to begin by casting doubt on the
(End of the second Para).
legitimacy of the notion of literature. The mere fact that
the word exists, or that an academic institution has been 48. How does the writer argue that the existence of
built around it, does not mean that the thing itself is self- literature is hardly self-evident?
evident. (i) by citing reasons for its non-existence
Reasons-perfectly empirical ones, to begin with- (ii) by citing reasons for interrogating its
are not hard to find. The full history of the word legitimacy
literature and its equivalents in all languages and all eras (iii) by citing reasons and proving by argument that
its legitimacy can be interrogated.
has yet to be written, but even a perfunctory look at the
(iv) by citing reasons to show that the label does
question makes it clear that the term has not been around
not match the thing we know to be literature
for ever. In the European languages, the word literature
(a) (i) (b) (i) and (ii)
is its current sense is quite recent: it dates back-just
(c) (iii) (d) (iii) and (iv)
barely- to the nineteenth century. Might we be dealing
with a historical phenomenon rather than an ‘eternal’ Ans : (d) Third para illustrates both statements (iii)
one? Moreover, many languages (many African and (iv).
languages, for example) have no generic term covering 49. “Might we be dealing with a historical
all literary productions. To these initial observations we phenomenon rather than an ‘eternal’ one”?
may add the fragmentation characteristics of literature to- What makes this a reasonable question to
day. Who dares specify what is literature and what is not, consider in this context?
given the irreducible variety of the writing that tends to (a) A historical phenomenon lends itself to better
be attached to it, from vastly different perspectives? empirical verification than an ‘eternal’ one.
The argument is not conclusive: a notion may (b) A historical phenomenon has more legitimacy
legitimately exist even if there is no specific term in the than an ‘eternal’ one.
lexicon for it. But we have been led to cast the first (c) A historical phenomenon can be debated and
shadow of doubt over the ‘naturalness’ of literature. A possibly settled while an ‘eternal’ one must be
theoretical examination of the problem proves no more taken on trust or not at all.
reassuring. Where do we come by the conviction that (d) A historical phenomenon is well above
there is indeed such a thing as literature? From disputation while an ‘eternal’ one is not.
experience. We study ‘literary’ works in school, then in Ans : (c) According to the third Para of given passage,
college; we find the ‘literary type of book in specialized statement (c) seems most correct.
stores; we are in the habit of referring to ‘literary’ 50. What does “the fragmentation characteristic of
authors in everyday conversation. An entity called literature today” suggest to the writer?
‘literature’ functions at the level of inter-subjective and (a) the fragmentation of modern consciousness.
social relations; this much seems beyond question. Fine, (b) the divided perceptions of literature by its
but what have we proved? That in the broader system of readers.
a given society or culture, an identifiable element exists (c) the lack of specificity of literature.
that is known by the label literature. Have we thereby (d) the blur that frustrates further investigation
demonstrated that all the particular products that take on into this concept.
the function of ‘literature’ possess common Ans : (b) The fragmentation characteristics of literature
characteristics, which we can identify with legitimacy? today suggest the divided perceptions of literature by
Not at all. its readers, according to the given passage.
UGC NET/JRF English Paper II, June 2009 52 YCT
UGC NET/JRF Exam, December-2009
ENGLISH
Solved Paper-II
Note : This paper contains fifty (50) objective (a) Ivanhoe
type questions, each question carrying two (2) marks. (b) Lady of the Lake
Attempt all the questions. (c) Heart of Midlothian
1. A classical influence on Ben Jonson's Volpone (d) The English Mail Coach
is: Ans : (d) The 'English Mail Coach' is an essay by the
(a) Juvenal (b) Aristophanes English author Thomas De Quincey. It is a “three part
(c) Plautus (d) Terence masterpiece” and one of his most magnificent works. It
Ans : (b) Aristophanes (circa. 446 BC- 386 BC), was a first appeared in 1849. The essay is divided into 3
comic playwright of ancient Athens. Eleven of his forty sections:
plays survive virtually complete. These provide the only Part I - “The Glory of Motion”
real examples of a genre of comic drama known as old Part II- “The Vision of Sudden Death”
comedy. He is also known as the ‘Father of Comedy’, Part III- “Dream Fugue, Founded on the Preceding
and ‘The Prince of Ancient Comedy.’ Theme of Sudden Death”
2. Kipling's "The White Man's Burden" is 6. "Provincializing Europe" is a concept
addressed to : propounded by :
(a) The American imperial mission in the (a) Edward Said
Philippines. (b) Paul Gilroy
(b) The Belgian colonial expansion in the Congo. (c) Abdul R. Gurnah
(c) The British Imperial presence in Nigeria. (d) Dipesh Chakravarty
(d) The British colonial entry into Afghanistan. Ans : (d) Provincializing Europe : Post Colonial
Ans : (a) 'The White Man’s Burden: The United Thought and Historical difference, addresses the
Status and the Philippine Islands' (1899), by Mythical figure of Europe that is often taken to be the
Rudyard Kipling, is a poem about the Philippine- original site of modernity in many histories of capitalist
American wars (1899-1902). transition in non-western countries.
7. The earliest tract on feminism is :
3. 'Poetry: A Magazine of Verse' was founded by
Harriet Monroe in : (a) Simone de Beauvoir's 'The Second Sex'
(a) 1922 (b) 1020 (b) Virginia Woolf's 'A Room of One's Own'
(c) Mary Wollstonecraft's 'A Vindication of the
(c) 1918 (d) 1912
Rights of Woman'
Ans : (d) Poetry: A Magazine of Verse published in
(d) Mary Astell's 'A Serious Proposal to the
Chicago since 1912, is one of the leading monthly Ladies'
poetry journals in the English speaking world.
Ans : (d) Mary Astell’s (1666-1731) was one of the
4. Who among the following was Geoffrey first English women to advocate the idea that women
Chaucer's contemporary? were just as rational as men, and just as deserving of
(a) Thomas Chatterton education. 'A Serious Proposal' was first published in
(b) John Gower 1694.
(c) Thomas Shadwell 8. Match the imaginary location with its creator:
(d) John Gay 1. Emily Bronte
Ans : (b) John Gower (c.1330-1408) was an English 2. Thomas Hardy
Poet, a contemporary and a personal friend of 3. Lowood Parsonage
Geoffrey Chaucer. He is remembered primarily for 3 4. Charles Dickens
major works, the 'Mirour de l' Omme', 'Vox 5. Wessex
Clamantis' and 'Confessio Amantis'. 6. Egdon Heath
5. Which of the following is NOT written by 7. Coketown
Walter Scott? 8. Charlotte Bronte
UGC NET/JRF English Paper II, December 2009 53 YCT
(a) 1-7-2-5-4-6-3-8 (c) A study of the media
(b) 1-6-2-5-3-8-4-7 (d) An analysis of poetic ambivalence
(c) 1-5-2-6-3-8-4-7 Ans : (d) 'Seven types of Ambiguity' is a work of
(d) 2-5-1-7-3-4-6-8 literary criticism by William Empson which was first
Ans : (b) published in 1930. The book is organized around seven
types of ambiguity that Empson finds in the poetry he
• Emily Bronte set her novel’s background in Egdon
criticises.
Heath, although Egdon Heath is also a fictitious area
of Thomas Hardy’s Wessex. 14. Who among the following is associated with the
ideology of Utilitarianism?
• Thomas Hardy set all his major novels in South and
South West of England. He named the area 'Wessex'. (a) J.A Froude (b) Charles Kingsley
(c) J.S. Mill (d) Cardinal Newman
• Charlotte Bronte set Lowood Parsonage as the
background in her novel Jane Eyre. Ans : (c) John Stuart Mill (1806-1873) was an English
philosopher, political economist, feminist and
• Charles Dickens used Coketown as a set in his novel
civil servant. He was a proponent of utilitarianism.
“Hard Times” (1854)
15. The 'Condition of England' literature refers to:
9. Which Chaucerian text parodies Dante's 'The
(a) The literature written by the labour class.
Divine Comedy'?
(b) The literature of England extolling living
(a) The Canterbury Tales
conditions.
(b) The Book of the Duchess
(c) The literature of England depicting the
(c) The House of Fame vulnerability of labour classes.
(d) Legend of Good Women (d) The literature of England depicting the
Ans : (c) 'The House of Fame' is a Middle English imperial projects abroad.
poem by Geoffrey Chaucer, probably written between Ans : (c) The term “Condition of England novels”
1379 and 1380. refers to a body of narrative fiction, also known as
10. 'Essays of Elia' was published in : industrial novels or social problem novels, published in
(a) 1800 (b) 1823 Victorian England, depicting the vulnerability of labour
(c) 1827 (d) 1850 classes in England.
Ans : (b) 'Essays of Elia' is a collection of essays 16. Philip Sidney wrote 'An Apology for Poetry' in
written by Charles Lamb. It was first published in a immediate response to :
book form in 1823. (a) Plato's 'Republic'
(b) Aristotle's 'Poetics'
11. Which of the following is an example of
homosexual fiction? (c) Stephen Gosson's 'The School of Abuse'
(a) The Well of Loneliness (d) Jeremy Collier's 'Immorality and profaneness
of the English Stage'
(b) Maurice
(c) Orlando Ans : (c) 'An Apology for Poetry' (or, The Defence of
Poesie) is a work of literary criticism, written in approx
(d) The Ballad of the Reading Gaol
1579 and published in 1595. It is generally believed that
Ans : (b) Maurice is a novel by E.M. Forster. It was he was at least partly motivated by Stephen Gosson.
written in 1913-1914. It is a tale of same-sex love in
17. 'Silence! The Court is in Session' is a........
early 20th century England.
(a) Gujarati (b) Bengali
12. W.B. Yeats' "Easter 1916" is: (c) Marathi (d) Kannada
(a) a response to a major political uprising
Ans : (c) 'Silence! The Court is in Session' is a Marathi
(b) a reminiscence of his visit to a nursery school play written by playwright Vijay Tendulkar and first
(c) a love poem for Maud Gonne performed in 1968. The play was written in 1963.
(d) an ode to his native country 18. Arrange the following in ascending order in
Ans : (a) 'Easter, 1916' is a poem, describing the terms of size:
poet’s torn emotions regarding the events of the Easter 1. epic 2. epigram
Rising staged in Ireland against British rule. 3. stanza 4. sonnet
13. William Empson's 'Seven Types of Ambiguity' (a) 1 2 3 4
is : (b) 2 1 3 4
(a) A structuralist study of narrative (c) 2 3 4 1
(b) A piece of psychoanalytic criticism (d) 1 3 4 2
UGC NET/JRF English Paper II, December 2009 54 YCT
Ans : (c) The sequence according of their size is: 23. 'The Temple' is a collection of poems by :
• 2. Epigram is a brief interesting, memorable and (a) Thomas Carew
sometimes surprising or satirical statement. (b) Robert Herrick
• 3. In poetry, a stanza is a grouped set of lines within (c) George Herbert
a poem. (d) Richard Crashaw
• 4. A Sonnet is a poetic form which signified a poem Ans : (c) George Herbert (1593-1633) was a Welsh
of fourteen lines that follows a strict rhyme poet, orator and Anglican priest. In 1633 all of
scheme and specific structure. Herbert’s English poems were published in 'The
• 1. An epic poem is a lengthy narrative poem, Temple: sacred poems and Private Ejaculations'.
ordinarily concerning a serious subject containing
24. Ben Jonson's comedies are:
details of heroic deeds and events.
(a) Volpone, Bartholomew Fair, The
19. "Fail I alone in words and deeds?/Why, all
Shoemaker's Holiday
men strive and who succeeds?" These lines are
(b) Volpone, The Alchemist, Epicoene
from:
(c) Volpone, The Alchemist, The Knight of the
(a) "Rabbi Ben Ezra"
Burning Pestle
(b) "Fra Lippo Lippi"
(c) "Caliban upon Setebos" (d) Volpone, Epicoene, The Shoemaker's
Holiday
(d) "The Last Ride Together"
Ans : (d) “The Last Ride Together” (1891) by Ans : (b)
Robert Browning is a monologue of a rejected lover • “Volpone” is a comedy first performed in 1605-06
exploring the end of a love affair. drawing on elements of city comedy and beast fable.
20. Dr. Johnson's "The Vanity of Human Wishes" • “The Alchemist” is a comedy first performed in 1610
expresses : by the King’s Men.
(a) Epicureanism (b) Humanism “Epicoene” or “The Silent Woman” was originally
(c) Stoicism (d) Cynicism performed by the Blackfriar's Children or Children of
Ans : (d) "The Vanity of Human Wishes: The Tenth the Queen’s Revels in 1609.
Satire of Juvenal Imitated" is written in 1749. As the 25. What is 'L' Allegro's' companion piece called?
subtitle suggests, it is an imitation of satire by the Latin (a) Lamia (b) Hyperion
poet Juvenal. (c) Il Penseroso (d) Thyrsis
21. "A trivial comedy for serious people" was the Ans : (c) ‘L’ Allegro’ is a pastoral poem by John
subtitle for : Milton published in his 1645 Poems. ‘L’ Allegro (which
(a) Everyman in His Humour means “the happy man” in Italian) is invariably paired
(b) Blythe Spirit with the contrasting pastoral poem, “Il Penseroso” (or
(c) The Way of the World 'the melancholy man'), which depicts a similar day spent
(d) The Importance of Being Earnest. in contemplation and thought.
Ans : (d) 'The Importance of Being Earnest, A 26. Match the character with the novel :
Trivial Comedy for Serious People' is a play by Oscar 1. Caddy
Wilde. It was first performed in 1895 and is a farcical
2. Lennie
comedy.
3. Jake Barnes
22. Which famous elegy closes with the following
4. Tommy Wilhelm
lines?
5. The Sound and the Fury
"In the deserts of the heart/Let the healing
fountain start/In the prison of his days,/Teach 6. Of Mice and Men
the free man how to praise." 7. The Sun Also Rises
(a) In Memoriam 8. Seize the Day
(b) Thyrsis Codes:
(c) "In Memory of W.B. Yeats" (a) 1-5 2-6-3-7 4-8
(d) "Verses on the Death of T.S. Eliot" (b) 2-7 1-6 8-3 5-6
Ans : (c) Poem “In Memory of W.B. Yeats” is an (c) 3-5 4-6 2-8 1-7
elegy written by W.H. Auden. (d) 4-5 3-8 2-7 1-8
UGC NET/JRF English Paper II, December 2009 55 YCT
Ans : (a) (b) Greenblatt, Sinfield, Butler
• “The Sound and the Fury” (1929) is a novel written (c) Greenblatt, Montrose, Goldberg
by American writer William Faulkner. Caddy (d) Williams, Greenblatt, Belsey
Compson appears as a character in the novel. Ans : (c) New Historicism is school of literary theory
• “Of Mice and Men” is a novella written by another which first developed in the 1980, primarily through the
John Steinbeck, published in 1937. It tells the story of work of Stephen Greenblatt.
Lennie and George. 32. Wallace Stevens' "The Man with the Blue
• “The Sun Also Rises” is a 1926 novel written by Guitar" may be linked to the work of the
American author Ernest Hemingway. Jake Barnes is following artist:
the protagonist and narrator in this novel. (a) Modigliani (b) Chagall
• “Seize the Day” first published in 1956 written by (c) Picasso (d) Cezanne
Saul Bellow. Tommy is a character in this novel. Ans : (c) “The Man with the Blue Guitar” is a poem
27. Who among the following writers belonged to published in 1937 by “Wallace Stevens.”It is divided
the American Beat movement? into 33 lengthy section or cantos and takes the form of
(a) Allen Ginsberg (b) Mark Beard an imaginary conversation with the subject of Pablo
(c) Isaac Mc Caslih (d) Charles Beard Picasso’s Painting 'The Old Guitarist'.
Ans : (a) The American Beat Movement is also called 33. The author of 'Gender Trouble' is :
Beat Generation. The Beat Generation was a group of (a) Elaine Showalter (b) Helene Cixous
authors whose literature explored and influenced (c) Michele Barrett (d) Judith Butler
American culture in the post World War- II era. Alan Ans : (d) “Gender Trouble: Feminism and the
Ginsberg's 'Howl and other Poems' is often considered Subversion of identity” is a 1990 book by Butler. The
representative of the Beat poets. The term 'beat work is influential in feminism, women’s studies,
generation' was coined by Jack Kerouac. Lesbian and gay studies and queer theory.
28. "The Lost Generation" is a name applied to 34. The structural analysis of signs was practised
the disillusioned intellectuals and aesthetes of by :
the years following the First World War. Who (a) Michel Foucault (b) Jacques Lacan
called them "The Lost Generation"? (c) Julia Kristeva (d) Roland Barthes
(a) H.L. Mencken (b) Willa Cather
Ans : (d) Roland Gerard Barthes (1915-1980) was a
(c) Jack London (d) Gertrude Stein
French Literary Theorist and Philosopher. Barthes'
Ans : (d) The “Lost Generation” was the generation “Introduction to the Structural Analysis of
that came of age during World War-I. The term was Narratives” is concerned with examining
popularized by Ernest Hemingway in 'The Sun also correspondence between structure and narratives.
Rises', who credits the phrase to Gertrude Stein, who
35. Which of the following is a spoof of a Gothic
was then his mentor and patron.
novel?
29. Hyperbole is: (a) Frankenstein (b) Northanger Abbey
1. an extravagant exaggeration (c) Castle of Otranto (d) Mysteries of Udolfo
2. a racist slur
Ans : (b) “Northanger Abbey” was the first of Jane
3. a metrical skill Austen’s novels to get published, written around
4. a figure of speech 1798- 99.
(a) 1 is correct (b) 1 and 4 are correct
36. The "madwoman in the attic" is a specific
(c) 1 and 3 are correct (d) 3 is correct
reference to :
Ans : (b) “Hyperbole” is the use of exaggeration as a (a) The narrator of "Goblin market"
rhetorical device or figure of speech. (b) August Egg's 1858 narrative painting
30. "Imagined Communities" is a concept (c) The Heroine of The Yellow Wallpaper
propounded by : (d) Bertha Manson of 'Jane Eyre'
(a) Benedict Anderson (b) Homi Bhabha
Ans : (d) “The Madwoman in the Attic” (1979),
(c) Aijaz Ahmed (d) Partha Chatterjee examines Victorian literature from a feminist
Ans : (a) Anderson coined this term in his book perspective. The writers draw their title from Bronte’s
“Imagined Communities” in which he explains the “Jane Eyre” in which Rochester’s wife Bertha Mason
concept in depth, first published in 1983. is kept locked in the attic by her husband.
31. The New Historicists include : 37. Assertion (A) : Dr. Johnson's 'The Lives of the
(a) Greenblatt, Showalter, Montrose Poets' carries critical and biographical studies of
UGC NET/JRF English Paper II, December 2009 56 YCT
poets he admired. It does not, however, carry a Ans : (b) The psychological concept of The Uncanny as
life of William Wordsworth. something that is strangely familiar, rather than just
Reason (R) : Dr. Johnson singled out poets whom mysterious, was first introduced by Sigmund Freud in
he not only admired but also adored. This his essay “Das Unheimliche.”
explains his omission of Wordsworth. 41. He wrote an essay called "Conrad's Darkness"
(a) (A) is wrong but (R) is correct. where he praises the earlier writer for offering
(b) (A) is true but (R) is false him a vision of the world's "half-made
(c) (A) and (R) are true. societies". Identify the writer.
(d) neither (A) nor (R) is true. (a) Chinua Achebe (b) V.S. Naipaul
Ans : (d) “Lives of the Most Eminent English Poets” (c) Salman Rushdie (d) Ngugi wa Thiongo
(1779-81) is a work by Samuel Johnson comprising Ans : (b) Sir Vidiadhar Surajprasad Naipaul (1932) is a
short biographies and critical appraisals of 52 poets, Trinidadian Nobel Prize winning British writer, known
most of whom lived in 18th century. for his comic early novels set in Trinidad and Tobago.
38. What is the correct chronological sequence of His essay 'Conrad's Darkness' is popular among critics
of Conrad's 'Heart of Darkness'.
the following?
(a) Moll Flanders, Pamela, Joseph Andrews, 42. "Magic Realism" is closely associated with :
Tristram Shandy (a) Italo Calvino
(b) Joseph Andrews, Tristram Shandy, Pamela, (b) Gabriel Garcia Marquez
Moll Flanders (c) Anita Desai
(c) Tristram Shandy, Moll Flanders, Pamela, (d) Rohinton Mistry
Joseph Andrews Ans : (b) Magic Realism in literature is that portrays
(d) Pamela, Moll Flanders, Joseph Andrews, magical or unreal elements as a natural part in our
Tristram Shandy environment. It accepts some magical element in the
Ans : (a) rational world. It was associated with Gabriel Garcia
Marquez in Latin American Literature.
• "The Fortunes and Misfortunes of the famous Moll
Flanders" is a novel by Daniel Defoe published in 43. Who among the following combines
1722. anthropology, history and fiction?
(a) Kamala Markandya
• “Pamela, or Virtue Rewarded” is an epistolary
novel by Samuel Richardson, published in 1740. (b) Mulk Raj Anand
(c) Upamanyu Chatterjee
• “Joseph Andrews” was published in 1742 and
(d) Amitav Ghosh
written by Henry Fielding.
• “Tristram Shandy” is a humorous novel by Ans : (b) Mulk Raj Anand (1905-2004) was an Indian
writer in English. Anand is admired for his novels and
Lawrence Sterne. It was published in 1759.
short stories, which have acquired the status of being
39. "How can what an Englishman believes by classic works of modern Indian English Literature.
heresy? It is a contradiction in terms." This
44. Which of the following is NOT a Partition
means :
novel?
(a) An Englishman does not know what heresy
(a) Train to Pakistan
is.
(b) Sunlight on a Broken Column
(b) An Englishman has no beliefs.
(c) The Shadow Lines
(c) And, therefore, there is no question of his
(d) In Custody
heresy.
Ans : (d)
(d) And, therefore, there cannot be any question
of his acting his beliefs. • “Train to Pakistan” by Khushwant Singh, published
in 1956, is based on India-Pakistan partition.
(a) 1 and 4 are correct (b) 2 and 1 are correct
• "Sunlight on a Broken Column" by Attia Hossain,
(c) 1 and 3 are correct (d) 2 and 4 are correct
published in 1961, is also a partition novel.
Ans : (c) This means that an Englishman doesn’t know
• “The Shadow Lines” (1988) by Amitav Ghosh is set
what heresy is, and therefore, there is no question of his
against the backdrop of Indian independence, second
heresy. world war, partition and riots in Dhaka.
40. Which of the following is an essentially • “In Custody” (1984) by Anita Desai is a story of
Freudian concept? Deven trying to get an interview of an Urdu poet Nur
(a) Archetype (b) The Uncanny Shahjahanabadi, set in old Delhi. It is not a partition
(c) The Absurd (d) The Imaginary novel.
UGC NET/JRF English Paper II, December 2009 57 YCT
45. Which of the following options is correct? meaning, different sets of values. This process allows us
(a) Transcendentalism was a philosophical and to show how language is implicated in our construction
literary movement. of different 'selves': different discourses position us in
(b) It flourished in the Southern States of different ways in relation to the world.
America in the 19th century. 46. Which of the following is True in the light of
(c) It was a reaction against 18th century this passage?
rationalism and the skeptical philosophy of (a) Language is inaccurate.
Locke. (b) Discourse is accurate
(d) Among the major texts of Transcendentalist (c) Language comprises discourse
thought are the essays of Emerson, Thoreau's (d) Discourse comprises language
Walden and the writings of Margaret Fuller.
Ans : (d) According to the passage, discourse
(a) (i) and (iv) are correct.
comprises language, is correct.
(b) (ii) and (iii) are correct.
47. What words/phrases suggest the plurality of
(c) (iii) and (iv) are correct.
discourse in this passage?
(d) (iv) is correct.
I. different selves
Ans : (a) Transcendentalism was a philosophical
II. range
movement that developed in the late 1820s and 1830s in
III. system of statements
the eastern region of the U.S., the movement was a
reaction to or protest against the general state of IV. heterogeneous collection
intellectualism and spirituality. (a) II and IV (b) II and III
(46-50) Read the following passage carefully, and (c) II and IV (d) I
select the right answer from the alternatives given Ans : (a) Plurality of discourse is suggested by the word
below in the question 46 to 50 : range and heterogeneous collection.
It would be more accurate to say that discourse, 48. Having called language "something of a
rather than language, plays a crucial part in structuring fiction", how does the author suggest its
our experience. The whole idea of 'language' is opposite?
something of a fiction: what we normally refer to as By using the phrase
'language' can more realistically be seen as (a) conceptualized as a system
heterogeneous collection of discourses. Each of us has
(b) More accurate to say
access to a range of discourses, and it is these different
(c) range of discourses
discourses which give us access to, or enable us to
perform, different 'selves'. A discourse can be (d) more realistically be seen
conceptualized as a 'system of statements which cohere Ans : (a) The author suggested its opposite by using the
around common meaning and values'. So. for example, in phrase conceptualized as a system.
contemporary Britain there are discourses which can be 49. Which among the following statements in NOT
labelled 'conservative' -that is, discourses which true?
emphasize values and meanings where the status quo is (a) Conservative discourses plead for the status
cherished : and there are discourses which can be quo.
labelled 'patriarchal' - That is, discourses which
(b) Patriarchal discourses privilege male values.
emphasize meanings and values which assume the
(c) Dominant discourses are natural.
superiority of males. Dominant discourses such as these
appear 'natural': they are powerful precisely because they (d) Dominant discourses seem natural.
are able to make invisible many different discourses. Ans : (c) According to the passage option (c) is correct.
Theorizing language in this way is still new in 50. What does this passage plead for?
linguistics (to the extent that many linguists would not (a) Theorizing language in a new way.
regard analysis in terms of discourses as being part of (b) Theorizing language in terms of discourses.
linguistics). One of the advantages of talking about (c) Studying language as discourses
discourses rather than about language is that the concept
(d) Studying discourse as language.
'discourse' acknowledges the value-laden nature of
language. There is no neutral discourse: whenever we Ans : (b) This passage pleads for Theorizing language
speak we have to choose between different systems of in terms of discourses.

UGC NET/JRF English Paper II, December 2009 58 YCT


UGC NET/JRF Exam, June-2010
ENGLISH
Solved Paper-II
Note : This paper contains fifty (50) objective (a) Keats, Wordsworth and Coleridge
type questions, each question carrying two (2) marks. (b) Wordsworth, Byron and Coleridge
Attempt all questions. (c) Blake, Wordsworth and Coleridge
1. The epithet "a comic epic in prose" is best (d) Wordsworth, Coleridge and Southey
applied to : Ans : (d) The term ‘Lake Poets' is used for the poets
(a) Richardson's 'Pamela' Wordsworth, Coleridge and Southey, who used to live
(b) Sterne's 'A Sentimental Journey' in the Lake district of England, and shared their
(c) Fielding's 'Tom Jones' common habitation.
(d) Defoe's 'Robinson Crusoe' 6. Which of the following novels is not by Patrick
Ans : (c) Comic prose epic is derived from serious epic White?
that satirizes contemporary ideas or conditions in a form (a) The Vivisector
and style burlesquing the serious epic. Fielding's 'Tom (b) The Tree of Man
Jones' is also known as mock epic in prose. (c) Voss
2. Muriel Spark has written a dystopian novel (d) Oscar and Lucinda
called :
Ans : (d) “Oscar & Lucinda” is a novel by Australian
(a) 'Memento Mori' author Peter Carey which won the 1988 Booker Prize,
(b) 'The Prime of Miss Jean Brodie' the 1989 Miles Franklin Award and was shortlisted for
(c) 'Robinson' the Best of the Booker.
(d) 'The Ballad of Peckham Rye' 7. The famous line "... where ignorant armies
Ans : (b) Dystopian novel (unlike utopian novel) is a clash by night" is taken from a poem by :
novel where life is worse than reality. “The Prime of (a) Wilfred Owen (b) W.H. Auden
Miss Jean Brodie” (1961) is the best example of this
(c) Siegfried Sassoon (d) Matthew Arnold
type.
Ans : (d) “Dover Beach” is a lyric poem by the poet
3. Samuel Butler's 'Erewhon' is an example of : Matthew Arnold. It was first published in 1867 in the
(a) Feminist Literature (b) Utopian Literature collection 'New Poems'.
(c) War Literature (d) Famine Literature 8. Which among the following novels is not
Ans : (b) A Utopia is a community or society written by Margaret Atwood?
possessing highly desirable or perfect qualities. It is a (a) Surfacing
common literary theme, especially in speculative fiction
(b) The Blind Assassin
and science fiction. Butler's 'Erewhon' is an example of
(c) The Handmaid's Tale
utopian literature.
(d) The Stone Angel
4. The line "moments of unageing intellect"
Ans : (d) "The Stone Angel" first published in 1964
occurs in Yeats's : by Mc Clelland and Stewart, is perhaps the best known
(a) Byzantium of Margaret Laurence set in the fictitious town of
(b) Among School Children Manakawa. It tells the story of Hagar Currie Shipley. In
(c) Sailing to Byzantium the present day narrative, 90 year old Hagar is
(d) The Circus Animals' Desertion struggling against being put in a nursing home which
Ans : (c) “Sailing to Byzantium” is a poem by she sees as a symbol of death.
William Butler Yeats first published in 1928 collection 9. The term 'theatre of cruelty' was coined by :
“The Tower.” It comprises 4 stanzas in Ottava rima (a) Robert Brustein (b) Antonin Artaud
each made up of 8-10 syllable lines. (c) Augusto Boal (d) Luigi Pirandello
5. In his 1817 review of Coleridge's Biographia Ans : (b) “The Theatre of cruelty” is a form of Theatre
Literaria, Francis Jeffrey grouped the following developed by avant-garde playwright actor Antonin
poets together as the 'Lake School of Poets' : Artaud in 'The Theatre and its Double' (1938).
UGC NET/JRF English Paper II, June 2010 59 YCT
10. The verse form of Byron's 'Childe Harold' was (a) William Godwin (b) Edmund Burke
influenced by : (c) Thomas Paine (d) Edward Gibbon
(a) Milton (b) Spenser Ans : (c) “The Age of Reason” is an influential work
(c) Shakespeare (d) Pope written by English and American political activist
Ans : (b) “Childe Harold’s Pilgrimage” is a lengthy Thomas Paine. It was published in three parts in 1794,
narrative poem in four parts written by Lord Byron. It 1795 and 1807 and originally distributed as unbound
pamphlets.
was published between 1812 and 1818. The rhyme
scheme is ABABBCBCC, which was believed to be 15. The Restoration comedy has been criticized
Spenser’s rhyme scheme with an additional mainly for its :
'alexandrine' line. (a) excessive wit and humour
11. Tennyson's 'Ulysses' is : (b) bitter satire and cynicism
I. a poem expressing the need for going (c) indecency and permissiveness
forward and braving the struggles of life (d) superficial reflection of society
II. a dramatic monologue Ans : (c) Restoration comedy refers to English
III. a morbid poem comedies written and performed in the restoration
IV. a poem making extensive use of satire period from 1660 to 1710. Comedy of manners is used
The right combination for the above statement, as a synonym of Restoration comedy.
according to the code, is : 16. 'Ideology and Ideological State Apparatuses' is
(a) I and IV (b) II and III an essay by :
(c) III and IV (d) I and II (a) Terry Eagleton (b) Karl Marx
Ans : (d) “Ulysses” is a modernist novel by Irish writer (c) Raymond Williams (d) Louis Althusser
James Joyce. It was first serialized in parts in the Ans : (d) "Ideology and Idelogical State Apparatuses"
American Journal 'The Little Review' from 1918 to is an essay by the French Marxist Philosopher Louis
1920. Althusser. First published in 1970, it advances
12. Which post-war British poet was involved in a Althusser’s theory of ideology.
disastrous marriage with Sylvia Plath? 17. Sexual possessiveness is theme of
(a) Philip Larkin (b) Ted Hughes Shakespeare's :
(c) Stevie Smith (d) Geoffrey Hill (a) Coriolanus
Ans : (b) Edward James “Ted” Hughes (1930-1998) (b) Julius Caesar
was an English Poet and children’s writer. Critics (c) Henry IV Part-I
frequently rank him as one of the best poets of his (d) A Midsummer Night's Dream
generation. Hughes was married to American poet Ans : (d) “A Midsummer Night’s Dream” is a comic
Sylvia Plath from 1956 until her suicide in 1963, at the play by Shakespeare written between 1590 and 1597. It
age of 30. portrays the events surrounding the marriage of
13. Chaucer's 'Parliament of Fowles' is in part : Theseus, the Duke of Athens, to Hippolyta.
I. a puzzle 18. The term 'Cultural Materialism' is associated
II. a debate with :
III. a threnody (a) Stephen Greenblatt (b) Raymond Williams
IV. a beast fable (c) Matthew Arnold (d) Richard Hoggart
The correct combination for the above Ans : (b) Raymond Williams (1921-1988) was a Welsh
statement, according to the code, is : academic, novelist and critic. His writings on politics,
(a) I, II and IV (b) II, III and IV culture, the mass media and literature are a significant
(c) III and IV (d) II and IV contribution to the Marxist critique of culture and the
Ans : (a) 'The Parlement of Fowles' is a poem by arts. His work laid the foundations for the field of
cultural studies and the cultural materialist approach.
Geoffrey Chaucer made up of approximately 700 lines.
The poem is in the form of a dream vision in rhyme 19. Which of the following author-book pair is
royal stanza. The poem is not a threnody (lament). correctly matched?
14. Who among the following wrote a book with (a) Muriel Spark – Under the Net
the title 'The Age of Reason'? (b) William Golding – Girls of Slender Means

UGC NET/JRF English Paper II, June 2010 60 YCT


(c) Angus Wilson – Lucky Jim 25. Which of the following plays is not written by
(d) Doris Lessing – The Grass is Singing Wole Soyinka?
Ans : (d) (a) The Lion and the Jewel
♦ 'Under the Net' (1954) - Iris Murdoch (b) The Dance of the Forests
♦ 'Girls of Slender Means' (1963) - Muriel Spark (c) Master Harold and the Boys
♦ 'Lucky Jim' (1954) - Kingsley Amis (d) Kongi's Harvest
♦ 'The Grass is Singing' (1950) - Doris Lessing Ans : (c) “Master Harold and the Boys” is a play by
20. Who among the following is a Canadian critic? Athol Fugard set in 1950, it was first produced in 1982.
(a) I.A. Richards (b) F.R. Leavis The play takes place in South Africa during apartheid
(c) Cleanth Brooks (d) Northrop Frye era, and depicts how institutionalized racism, bigotry or
Ans : (d) Northrop Frye (1912-1991) was a Canadian hatred can become absorbed by those who live under it.
Literary critic and literary theorist, considered one of 26. Which of the following plays by William
the most influential of the 20th century. Wycherley is in part an adaptation of Moliere's
21. Sethe is a character in : 'The Misanthrope'?
(a) The Colour Purple (a) The Plain Dealer
(b) The Women of Brewster Place (b) The Country Wife
(c) Beloved (c) Love in a Wood
(d) Lucy (d) The Gentleman Dancing Master
Ans : (c) Sethe is the central character in Toni Ans : (a) “The Plain Dealer” is a restoration comedy
Morrison's 'Beloved', who kills her daughter and tries to by William Wycherley, first performed in 1676. The
kill her other three children. It was written by Toni play is based on Moliere’s 'The Misanthrope' and is
Morrison in 1987 inspired by the African-American generally considered Wycherley’s finest work along
slavery during the American Civil War. with 'The Country Wife'.
22. 'Imagined Communities' is a book by :
27. 'Inversion' is the change in the word order for
(a) Aijaz Ahmad (b) Edward Said
creating rhetorical effect, e.g. this book I like.
(c) Perry Anderson (d) Benedict Anderson
Another term for inversion is :
Ans : (d) “Imagined Communities” by Benedict
(a) Hypallage (b) Hubris
Anderson was first published in 1983. Anderson
(c) Haiku (d) Hyperbaton
believes that a nation is a socially constructed
community, imagined by the people who perceive Ans : (d) Hyperbaton is an inversion of the normal
themselves as part of that group. order of words especially for the sake of emphasis, as in
the sentence “This I must See”.
23. Who among the following is a Cavalier poet?
28. The phrase 'the willing suspension of disbelief'
(a) Henry Vaughan (b) Richard Crashaw
(c) John Suckling (d) Anne Finch occurs in :
Ans : (c) Sir John Suckling (1609-1641) was an (a) Biographia Literaria
English poet and a prominent figure among those (b) Preface to Lyrical Ballads
renowned for careless gaiety and wit, considered as the (c) In Defence of Poetry
accomplishments of a Cavalier poet. (d) Poetics
24. Which play of Wilde has the subtitle, 'A Trivial Ans : (a) The term “Suspension of disbelief” or
Comedy for Serious People'? "willing suspension of disbelief" has been defined as a
(a) A Woman of No Importance willingness to suspend one’s critical faculties and
(b) Lady Windermere's Fan believe the unbelievable, to sacrifice logic for the sake
(c) The Importance of Being Earnest of enjoyment. The phrase was coined in 1817 by S.T.
(d) An Ideal Husband Coleridge in 'Biographia Literaria'.

Ans : (c) 'The Importance of Being Earnest' subtitled 29. The religious movement Methodism in the 18th
'A Trivial Comedy for Serious People', is a play by century England was founded by :
Oscar Wilde, first performed in 1895 at the St. James’s (a) John Tillotson (b) Bishop Butler
Theatre in London. It is a farcical comedy or farce. (c) Bernard Mandeville (d) John Welsey
UGC NET/JRF English Paper II, June 2010 61 YCT
Ans : (d) Methodism, a worldwide protestant Ans : (b) The King James Version, also known as the
movement dating from 1729, started when a group of Authorised Version or King James Bible, is an English
students at the University of Oxford, England, began to translation of the Christian Bible for the Church of
assemble for worship, study and Christian Service. England, begun in 1604 and completed in 1611.
30. 'My First Acquaintance with Poets', an 36. The Court of Chancery is a setting in Dickens' :
unforgettable account of meeting with literary (a) Little Dorrit (b) Hard Times
heroes, is written by :
(c) Dombey and Son (d) Bleak House
(a) Charles Lamb (b) Thomas de Quincey
(c) Leigh Hunt (d) William Hazlitt Ans : (d) “Bleak House” was first published in a serial
form between 1852 and 1853 and it is one of Charles
Ans : (d) “My First Acquaintance with Poets” (1823)
is written by William Hazlitt. The essay was first Dickens's major novels.
published in ‘The Literal.’ 37. Which romantic poet coined the famous phrase
31. The figure of the Warrior Virgin in Spenser's 'spots of time'?
'Faerie Queene' is represented by the character : (a) John Keats (b) William Wordsworth
(a) Britomart (b) Gloriana (c) S.T. Coleridge (d) Lord Byron
(c) Cynthia (d) Duessa Ans : (b) William Wordsworth, English romantic poet,
Ans : (a) The hero of Book III, the female warrior wrote about the concept of “spots of time” in 'The
Virgin is represented by Chastity. Prelude' book twelfth.
32. The book 'Speech acts' is written by :
38. The statement 'I think, therefore, I am' is by :
(a) John Austin
(b) John Searle (a) Schopenhauer (b) Plato
(c) Jacques Derrida (c) Descartes (d) Sartre
(d) Ferdinand de Saussure Ans : (c) French philosopher, scientist and
Ans : (b) “A Speech act” in linguistics and the mathematician, Rene Descartes published philosophical
philosophy of language is an utterance that has essays in 1637. Descartes applies the rational inquiry of
performative function. John R. Searle is the author of science to philosophy and argues that one can be certain
'Expression and Meaning: Studies in the Theory of of one’s own existence. All else derives from this basic
Speech Acts', published in 1999. premise which Descartes states as cogito egro sum
33. Which among the following is not a sonnet (Latin for 'I think, therefore I am').
sequence? 39. Verse that has no set theme no regular meter,
(a) Philip Sydney – Astrophel and Stella rhyme or stanzaic pattern is :
(b) Samuel Daniel – Delia I. open form II. flexible form
(c) Derek Walcott – Omeroos III. free verse IV. blank verse
(d) D.G. Rossetti – The House of Life The correct combination for the statement,
Ans : (c) “Omeroos” is an epic poem by Caribbean according to the code, is :
writer Derek Walcott, first published in 1990. The work (a) I, II and III are correct.
is divided into “seven books” containing a total of 640
(b) III and IV are correct.
chapters.
(c) II, III and IV are correct.
34. 'Incunabula' refers to :
(d) I and III are correct.
(a) books censured by the Roman Emperor
Ans : (d) Open form and free verse are terms that are
(b) books published before the year 1501
sometimes described as totally synonymous.
(c) books containing an account of myths and
rituals 40. Which is the correct sequence of publication of
(d) books wrongly attributed to an author Pinter's plays?
Ans : (b) “Incunabula’ refers to an early printed book, (a) The Room, One for the Road, No Man's
especially one printed before 1501. Land, The Homecoming
35. The most notable achievement in Jacobean (b) The Homecoming, No Man's Land, The
prose was : Room, One for the Road
(a) Bacon's 'Essays' (c) The Room, The Homecoming, No Man's
(b) King James' translation of the Bible Land, One for the Road
(c) Robert Burton's 'Anatomy of Melancholy' (d) One for the Road, The Room, The
(d) None of the above Homecoming, No Man's Land
UGC NET/JRF English Paper II, June 2010 62 YCT
Ans : (c) The correct sequence of Pinter's plays is– Ans : (d) “A Portrait of the Artist as Young Man” is
♦ 'The Room' - 1957 a semi-autobiographical novel and “Ulysses” is focused
♦ 'The Homecoming' - 1964 on the events of a single day and related to one another
♦ 'The Basement' - 1966 in thematic pattern, based on Greek mythology. In both,
♦ 'No Man’s Land' - 1974 Stephen Dedalus figures in the lead role.
♦ 'One for the Road' - 1984 46. In 'Moby Dick' Captain Ahab falls for his :
41. Johnson's 'Dictionary of the English Language' (a) ignorance (b) pride
was published in the year : (c) courage (d) drunkenness
(a) 1710 (b) 1755 Ans : (b) “Moby Dick” or 'The whale' is a novel by
(c) 1739 (d) 1759 American writer Herman Melville, published in 1851
Ans : (b) Published on 15 April, 1755 and written by during the period of the American Renaissance.
Samuel Johnson, 'A Dictionary of the English 47. The first complete printed English Bible was
Language', sometimes published as Johnson’s produced by :
Dictionary, is among the most influential dictionaries in
(a) William Tyndale (b) William Caxton
the history of English language.
(c) Miles Coverdale (d) Roger Ascham
42. The literary prize, Booker of Bookers, was
Ans : (c) Miles Coverdale’s complete Bible in print
awarded to :
was published in 1535.
(a) J.M. Coetzee (b) Nadine Gordimer
♦ John Wycliff - First hand-written Bible in manuscript
(c) Martin Amis (d) Salman Rushdie
(1380).
Ans : (d) Salman Rushdie's second novel, the ♦ Gutenberg - First printed Bible in Latin (1450 ).
s
acclaimed "Midnight’s Children" (1981), won the
♦ Tyndale - First man to print New Testament in
Booker Prize for fiction in 1981. In 1993, it was judged
English (1525). It didn't have Old Testament.
to be the ‘Booker of Bookers’ and in 2008 the 'Best of
the Bookers', the best novel to have won the Booker 48. Elizabeth Gaskell's novel 'Mary Barton' is sub-
Prize for fiction in the award’s 25 and 40 years history. titled :
43. In Keats' poetic career, the most productive (a) The Two Nations
year was : (b) A Tale of Manchester Life
(a) 1816 (b) 1817 (c) A Story of Provincial Life
(c) 1820 (d) 1819 (d) The Factory Girl
Ans : (d) John Keats' (1795-1821) great creative Ans : (b) “Mary Barton” sub-titled as “A Tale of
outpouring came in the year of 1819 when he composed Manchester Life” (1848) examines the schism between
a group of five odes. rich and poor in Industrial Manchester, England, during
1840s.
44. Pope's 'The Rape of the Lock' was published in
49. Some of the Jacobean playwrights were
1712 in :
(a) three cantos (b) four cantos prolific. One of them claimed to have written
200 plays. The playwright is :
(c) five cantos (d) two cantos
(a) John Ford (b) Thomas Dekker
Ans : (d) “The Rape of the Lock” is a mock-heroic
(c) Philip Massinger (d) Thomas Heywood
narrative poem. It was first published in Pope's
'Miscellaneous poems and translations' (May 1712) in Ans : (d) Thomas Heywood (e. 1574-1641), according
two cantos, 1334 lines. to his own testimony, wrote more than 220 plays for the
English stage. Although not always tightly constructed,
45. Stephen Dedalus is a fictional character
his plays exhibit a remarkable talent.
associated with :
I. A Portrait of the Artist as a young Man 50. The concept of "Star-equilibrium" in
II. Sons and Lovers connection with man-woman relationship
III. Ulysses appears in :
IV. The Heart of Darkness (a) Women in Love (b) Maurice
The correct combination for the above (c) Mrs. Dalloway (d) The Old wives' Tales
statement, according to the code, is : Ans : (a) “Women in Love” discloses Lawrence’s
(a) I and II (b) I, II and III assertion of establishing the equilibrium in Male/Female
(c) III and IV (d) I and III relationship.

UGC NET/JRF English Paper II, June 2010 63 YCT


UGC NET/JRF Exam, December-2010
ENGLISH
Solved Paper-II
Note : This paper contains fifty (50) objective Ans : (c) “Voss” (1957) is the fifth published novel of
type questions, each question carrying two (2) marks. Patrick White. It is based upon the life of the 19th
Attempt all questions. century Prussian explorer and naturalist Ludwig
1. Jeremy Collier's 'A Short View of the Leichhardt who disappeared whilst on an expedition
Immorality and Profaneness of the English into the Australian outback.
Stage' attacked among others.
6. Although Nobel Laureate Seamus Heaney
(a) John Bunyan (b) Thomas Rhymer
(c) William Congreve (d) Henry Fielding writes in English, in voice and subject matter,
his poems are :
Ans : (c) In March 1698, Jeremy Collier published his
anti-theatre pamphlet, “A Short view of the (a) Welsh (b) Scottish
Immorality and Profaneness of the English Stage.” (c) Irish (d) Polish
In the pamphlet Collier attacks a number of playwrights Ans : (c) Seamus Justin Heaney (1939-2013), MRIA
like William Wycherley, John Dryden, William (Member of the Royal Irish Academy), was an Irish
Congreve, John Vanbrugh and Thomas D’Urfey. poet, playwright, translator and lecturer and the
2. The Crystal Palace, a key exhibit of the Great recipient of the 1995 Nobel Prize in Literature.
Exhibition, was designed by : 7. To whom is Mary Shelley's famous work
(a) Charles Darwin (b) Edward Moxon
Frankenstein dedicated?
(c) Joseph Paxton (d) Richard Owen
(a) Lord Byron (b) Claire Clairmont
Ans : (c) The Crystal Palace was a cast iron and plate
(c) William Godwin (d) P.B. Shelley
glass structure originally built in Hyde Park, London to
house the Great exhibition of 1851. Designed by Ans : (c) "Frankenstein or the modern Prometheus" is a
Joseph Paxton, the Great Exhibition building was 1,851 novel written by the English author Mary
feet (564m) long with an interior height of 128 feet Wollstonecraft Shelley that tells the story of a young
139m. science student Victor Frankenstein published in 1818.
3. Influence of the Indian Philosophy is seen in It is an early example of science fiction. It is also a
the writings of : Gothic novel.
(a) G.B. Shaw (b) Noel Coward 8. Which among the following poems by Philip
(c) Tom Stoppard (d) T.S. Eliot Larkin records his impressions while travelling
Ans : (d) T.S. Eliot wrote (1933) that the great to London by train?
philosophers of India “make most of the great European (a) "Aubade"
Philosophers look like school boys” a very great (b) "Church Going"
remark. (c) "The Whitsun Wedding"
4. In which of his voyages, Gulliver discovered (d) "An Arundel Tomb"
mountain-like beings? Ans : (c) “The Whitsun Wedding” is a collection of
(a) The land of the Lilliputians 32 poems by Philip Larkin. It was first published by
(b) The land of the Brobdingnagians Faber and Faber in U.K. on 28 Feb, 1964.
(c) The land of the Laputans 9. The English satirist who used the sharp edge of
(d) The land of the Houyhnhnms
praise to attack his victims was :
Ans : (b) 'By Lemuel Gulliver, First a Surgeon and then (a) Ben Jonson (b) John Donne
a Captain of Several Ships', commonly known as
(c) John Dryden (d) Samuel Butler
"Gulliver Travels" (1726, amended 1735), is a prose
satire by Irish writer and clergyman Jonathan Swift that Ans : (c) John Dryden (1631-1700) was an English
is both a satire on human nature and the “travellers poet, literary critic, translator and playwright who was
tales” literary subgenre. made England’s first Poet Laureate in 1668.
5. Patrick White's Voss is a novel about : 10. One of the most famous movements of direct
(a) the sea (b) the capital market address to the reader-"Reader, I married
(c) the landscape (d) the judicial system him"-occurs in :
UGC NET/JRF English Paper II, December 2010 64 YCT
(a) Henry Fielding's Tom Jones 15. Which of the following is not a Revenge
(b) Charlotte Bronte's Jane Eyre Tragedy?
(c) Laurence Sterne's Tristram Shandy (a) The White Devil
(d) George Eliot's Middlemarch (b) The Duchess of Malfi
Ans : (b) “Jane Eyre” is a novel by English writer (c) Doctor Faustus
Charlotte Bronte. It was published on 16 Oct 1847. (d) The Spanish Tragedy
“Jane Eyre” follows the emotions and experiences of
its eponymous heroine including her growth to Ans : (c) 'The Tragic History of the Life and Death of
adulthood and her love. Doctor Faustus', is a play by Christopher Marlowe,
based on the German story 'Faust', first published in
11. Langland's Piers Plowman is a satire on :
1604, eleven years after Marlowe's death.
(a) aristocracy (b) chivalry
(c) peasantry (d) clergy 16. Who of the following playwrights rejects the
Ans : (d) 'Piers Plowman' (written c. 1370-90 or Visio Aristotelian concept of tragic play as imitation
Willelmi de Petro Ploughman), is a Middle English of reality?
allegorical narrative poem. It is in unrhymed alliterative (a) G.B. Shaw (b) Arthur Miller
verse divided into sections called 'passus'. It is (c) Bertolt Brecht (d) John Galsworthy
considered by many critics to be one of the greatest Ans : (c) Eugen Bertolt Friedrich Brecht (1898-1956)
works of English literature of the Middle Ages, along was a German Poet, playwright and theatre director of
with Chaucer’s 'Canterbury Tales'. 20th century. He made contributions to dramaturgy and
12. Which of the following thinker-concept pair is theatrical production.
correctly matched?
17. The label 'Diasporic Writer' can be applied to :
(a) I.A. Richards – Archetypal Criticism
(b) Christopher Frye – Mysticism I. Meena Alexander II. Arundhati Roy
(c) Jacques Derrida – Deconstruction III. Kiran Desai IV. Shashi Deshpande
(d) Terry Eagleton – Psychological Criticism The correct combination for the statement,
Ans : (c) Jacques Derrida was a French Philosopher, according to the code, is :
born in Algeria. Derrida is best known for developing a (a) I and IV are correct.
form of semiotic analysis known as deconstruction (b) II and III are correct.
which is discussed in numerous texts, and developed in (c) I, II and IV are correct.
the context of Phenomenology. His books are “Of (d) I and III are correct.
Grammatology”, (1967), 'Margins of Philosophy' Ans : (d) Meena Alexander, born in 1851 in Allahabad
(1962), 'Dissemindio' (1972) etc.
and raised in India and Sudan, is an internationally
13. Sexual jealousy is a theme in Shakespeare's : acclaimed poet, scholar and writer; Kiran Desai (b.
(a) The Merchant of Venice 1971) is an Indian author. She left India at 14, and she
(b) The Tempest and her mother then lived in England.
(c) Othello 18. The letter 'A' in 'The Scarlet Letter' stands for
(d) King Lear
1. Adultery 2. Able
Ans : (c) 'Othello, or the Moor of Venice' is a tragedy 3. Angel 4. Appetite
by Shakespeare believed to have been written in 1603.
The correct combination for the statement,
It is based on the story 'Un Capitano Moro' (a Moorish
according to the code, is :
captain) by Cinthio, a disciple of Boccaccio, first
published in 1565. (a) I and II are correct.
(b) II and III are correct.
14. The title, 'The New Criticism', published in
(c) I, II and IV are correct.
1941, was written by :
(d) I, II and III are correct.
(a) Cleanth Brooks
(b) John Crowe Ransom Ans : (d) “The Scarlet Letter: A Romance” is an
(c) Robert Penn Warren 1850 work of fiction in a historical setting, written by
(d) Allan Tate American author Nathaniel Hawthorne. It tells the story
of Hester Prynne who conceives a daughter through an
Ans : (b) John Crowe Ransom was an American
affair and struggles to create a new life of repentance
educator, scholar, literary critic, poet, essayist and
editor. He is considered to be a founder of the New and dignity.
Criticism School of Literary Criticism. He authored the 19. A monosyllabic rhyme on the final stressed
above book. syllable of two lines of verse is called :
UGC NET/JRF English Paper II, December 2010 65 YCT
(a) monorhyme (b) feminine rhyme Ans : (d) John Harvey (1696-1743), English courtier
(c) masculine rhyme (d) eye rhyme and political writer was the son of John Harvey, 1st Earl
Ans : (c) Masculine Rhyme is a rhyme between final of Bristol. He was known as Lord Harvey from 1723,
stressed syllables. Example blow/flow and but he never became Earl of Bristol.
confess/redress. 25. The hero of Marlowe's 'Tamburlaine' was born
20. A fatwa was issued in Salman Rushdie's name as a :
following the publication of : (a) carpenter (b) goldsmith
(a) Midnight's Children (b) Shame (c) shepherd (d) fisherman
(c) Satanic Verses (d) Grimus Ans : (c) 'Tamburlaine the Great' is a play in two parts
Ans : (c) 'Satanic Verses' is Rushdie’s 4th novel, by Marlowe. It is loosely based on the life of the Central
published in 1988 and inspired in part from the life of Asian emperor, Timur (1405). The play is a milestone
Muhammad. in Elizabethan public drama.
21. "There is nothing outside the text" is a key 26. In a letter to his brother George in September
statement emanating from :
1819, John Keats has this to say about a fellow
(a) Feminism (b) New Historicism romantic poet: "He describes what he sees-I
(c) Deconstruction (d) Structuralism describe what I imagine - Mine is the hardest
Ans : (c) Deconstruction is a critical outlook concerned task." The poet under reference is :
with the relationship between text and meaning. Jacques (a) Wordsworth (b) Coleridge
Derrida’s 1967 work “Of Grammatology” introduced (c) Byron (d) Southey
the majority of ideas influential within deconstruction.
Ans : (c) The rivalry and dislike between George
22. The Augustan Age is called so because : Gordon Byron and John Keats has been much discussed
(a) King Augustus ruled over England during in his own time, however, it was felt far more keenly by
this period Keats.
(b) The English writers imitated the Roman
27. A sequence of repeated consonant sounds in a
writers during this period
(c) The English King was born in the month of stretch of language is :
August (a) alliteration (b) acrostic
(d) This was an age of sensibility (c) assent (d) syllable
Ans : (b) Augustan Age is a style of British literature Ans : (a) Alliteration is the repetition of consonant
produced during the reigns of Queen Anne, King sounds.
George I and II in the first half of 18th century, ending 28. Reformation was predominantly a movement
in the 1740s, with the deaths of Alexander Pope and in :
Jonathan Swift, in 1744 and 1745, respectively.
(a) politics (b) literature
23. One of the important texts of Angry Young (c) religion (d) education
Men Movement is : Ans : (c) The Reformation was a religious movement in
(a) 'Time's Arrow' by Martin Amis the 16th century that divided Christianity between
(b) 'A Portrait of the Artist as a Young Man' by Roman Catholics and Protestants.
James Joyce
29. The motto "only connect" is taken from :
(c) 'Lucky Jim' by Kingsley Amis
(d) 'The French Lieutenant's Woman' by John (a) Joseph Conrad's 'Nostromo'
Fowles (b) Rudyard Kipling's 'Kim'
Ans : (c) 'Lucky Jim' is a novel by Kingsley Amis, (c) H.G. Well's 'The History of Mr. Polly'
published in 1954 by Victor Gollancz. It is Amis' first (d) E. M. Forster's 'Howards End'
novel and won the Somerset Maugham Award for Ans : (d) “Howards End” is a novel by Forster, first
fiction. published in 1910 about social conventions, codes of
24. Whom does Alexander Pope satirise in the conduct and personal relationships in turn-of the
portrait of Sporus? century England.
(a) Lady Wortley Montague 30. English Iambic Pentameter was brought to its
(b) Joseph Addison first maturity in :
(c) Lord Shaftsbury (a) sonnet (b) dramatic verse
(d) Lord Harvey (c) lyric (d) elegy
UGC NET/JRF English Paper II, December 2010 66 YCT
Ans : (a) The word sonnet is derived from the Italian Ans : (d) “Juno and the Paycock” is a play by Sean
word “Sonetto.” It means a small or little song or lyric. O’ Casey and is highly regarded and often performed in
In poetry a sonnet has 14 lines and is written in Iambic Ireland. It was first staged at the Abbey Theatre in
Pentameter, each line having 10 syllables. Dublin in 1924.
31. Who among the following was not a member of 37. The title of William Faulkner's 'The Sound and
the Bloomsbury Group? the Fury' is taken from a play by :
(a) Lytton Strachey (b) Clive Bell (a) Christopher Marlowe
(c) E. M. Forster (d) Winston Churchill (b) William Shakespeare
Ans : (d) The Bloomsbury Group was an influential (c) Ben Jonson
group of associate English writers, intellectuals,
(d) John Webster
philosophers and artists the best known members of
which included Virginia Woolf, John Maynard, E. M. Ans : (b) “The Sound and the Fury” (1959) is a novel
Forster and Lytton Strachey. written by the American author William Faulkner. It
employs a number of narrative styles including the
32. The concept of human mind as tabula rasa or
technique known as stream of consciousness etc.
blank tablet was propounded by :
(a) Bishop Berkley (b) David Hume 38. "Silverman has never read Browning." This is
(c) Francis Bacon (d) John Locke an example of :
Ans : (d) John Locke in his Essay “Concerning (a) chiasmus (b) conceit
Human Understanding” restated that the human (c) zeugma (d) metonymy
minds on birth is a complete, but receptive, blank slate Ans : (d) Metonymy is a figure of speech in which a
upon which experience imprints knowledge. thing or concept is called not by its own name but rather
33. The terms 'resonance' and 'wonder' are by the name of something associated in meaning with
associated with : that thing or concept.
(a) Stephen Greenblatt (b) Terence Hawkes 39. The term 'International Fallacy' is first used by :
(c) Terry Eagleton (d) Ronald Barthes (a) William Empson
Ans : (a) “Resonance and Wonder” is an essay by (b) Northrop Frye
Stephen Greenblatt (born 1943). He is an American (c) Wellek and Warren
thinker, Shakespearean, literary historian and Pulitzer (d) Wimsatt and Beardsley
Prize winning author.
Ans : (d) In mid 20th century William K. Wimsatt Jr.
34. The rhetorical pattern used by Chaucer in 'The and Monroe C Beardsley published The Intentional
Prologue to Canterbury Tales' is : Fallacy. In it they counter the contemporary assumption
(a) ten-syllabic line (b) eight-syllabic line that the original creator’s intention for a work was equal
(c) rhyme royal (d) ottava rima to the meaning and merit of the work.
Ans : (c) Chaucer first used the rhyme royal stanza in 40. "Recessional: A Victorian Ode," Kipling's
his long poems “Troilus and Criseyde” and well-known poem,
'Parlement of Foules'. He also used it for four of the
I. laments the end of an Era
Canterbury Tales: The Man of Law’s Tale, The
II. marks a new commitment to scientific
Prioress' Tale, the Clerk’s Tale and the second Nun’s
knowledge
Tale.
III. expresses the sincerity of his religious
35. Charles Darwin's 'Origin of the Species' was
devotion
published in the year : IV. was occasioned by Queen Victoria's 1897
(a) 1859 (b) 1879 Jubilee Celebration
(c) 1845 (d) 1866 The correct combination for the statement,
Ans : (a) 'On the Origin of Species', published Nov according to the code, is :
1859, is a work of scientific literature by Darwin, which (a) I, II and III are correct.
is considered to be the foundation of evolutionary
(b) III and IV are correct.
biology.
(c) I and IV are correct.
36. Who of the following is the author of 'Juno and (d) I, III and IV are correct.
the Paycock'?
Ans : (b) “Recessional” is a poem by Rudyard Kipling,
(a) Lady Gregory (b) W. B. Yeats which he composed for the occasion of Queen
(c) Oscar Wilde (d) Sean O' Casey Victoria’s Diamond Jubilee in 1897.
UGC NET/JRF English Paper II, December 2010 67 YCT
41. Who among the following is not a Restoration second expanded edition followed in 1809, with Byron
playwright? identified as the author. The poem was appreciated for
(a) William Congreve the survey of English poetry and the description of
(b) William Wycherley contemporary literary scene.
(c) Ben Jonson 46. Which Eliotian character utters the question
(d) George Etherege "Do I eat a peach"?
Ans : (c) Restoration dramatists include William (a) Marina (b) Prufrock
Wycherley, George Etherege, Thomas Otway, William (c) Sweeney (d) Stetson
Congreve, and George Farquhar. Ans : (b) “The Love Song of J. Alfred Prufrock”
42. Which famous Romantic poem begins with the (1915) by Eliot, published in the issue of poetry : 'A
line : 'Hail to thee, blithe spirit!/Bird thou Magazine of Verse at the instigation of Ezra Pound'
never wert'? (1885-1972).
(a) "Ode to a Nightingale" 47.
Which among the following works by Daniel
(b) "To the Cuckoo" Defoe landed him in prison and the pillory?
(c) "To a Skylark" (a) The True-Born Englishman
(d) "To the Daisy" (b) Captain Singleton
Ans : (c) “To a Skylark” is a poem completed by (c) The Shortest Way with Dissenters
Percy Bysshe Shelley in late June 1820. (d) Moll Flanders
43. Who among the following Victorian poets Ans : (c) “The Shortest Way with the Dissenters” is
disliked his middle name? a pamphlet by Daniel Defoe, first published in 1702.
(a) Arthur Hugh Clough 48. The arrival of printing in fifteenth century
(b) Dante Gabriel Rossetti England was engineered by :
(c) Gerard Manley Hopkins (a) Sir Thomas Malory (b) John Gower
(d) Algernon Charles Swinburne (c) John Barbour (d) William Caxton
Ans : (b) Dante Gabriel Rossetti (1828-1882) was an Ans : (d) William Caxton was an English merchant,
English poet, illustrator, painter and translator. He diplomat, writer and printer, thought to be the first
founded the Pre-Raphaelite Brotherhood in 1848 with English person to work as a printer and the first to
William Holman Hunt and John Everett Millais. introduce a printing press in England which he did in
44. Aston is a character in Pinter's : 1476.
(a) The Birthday Party 49. About which nineteenth century English writer
(b) The Caretaker was it said that "He had succeeded as a writer
(c) The Dumb Waiter not by conforming to the Spirit of the Age, but
(d) The Homecoming in opposition to it"?
Ans : (b) "The Caretaker" (1960) is a play in 3 acts (a) Lord Byron on Coleridge
by Harold Pinter. It was his sixth of his major works for (b) Coleridge on Keats
stage and TV. (c) Hazlitt on Lamb
45. Byron's 'English Bards and Scottish (d) De Quincey on Crabbe
Reviewers' is about : Ans : (c) William Hazlitt (1778-1830) was an English
I. the survey of English poetry writer, drama and literary critic and philosopher. He is
II. evangelism in English poetry now considered one of the greatest critic and essayist in
III. contemporary literary scene the history of the English language placed in company
of Samuel Johnson and George Orwell.
IV. the early English travellers
The correct combination for the statement, 50. The Restoration comedy, 'The Double Dealer'
according to the code, is : was written by :
(a) III and IV are correct. (a) John Dryden
(b) II, III and IV are correct. (b) William Wycherley
(c) I and II are correct. (c) William Congreve
(d) I and III are correct. (d) George Etherege
Ans : (d) “English Bards and Scotch Reviewers” Ans : (c) “The Double Dealer” is a comic play written
(1809) is a satirical poem written by Lord Byron. It was by English playwright William Congreve, first
first published, anonymously in March 1809, and a produced in 1693.
UGC NET/JRF English Paper II, December 2010 68 YCT
UGC NET/JRF Exam, June-2011
ENGLISH
Solved Paper-II
Note : This paper contains fifty (50) objective 6. The plan of Arthurian stories has influenced
type questions, each question carrying two (2) marks. the composition of Tennyson's :
Attempt all questions. (a) In Memoriam (b) Idylls
1. Little Nell is a character in Dickens's : (c) "Maud" (d) "Locksley Hall"
(a) Hard Times
Ans : (b) "Idylls of the King" published between 1859
(b) Great Expectations and 1885 is a cycle of twelve narrative poems by the
(c) Oliver Twist English poet Alfred Lord Tennyson which retells the
(d) The Old Curiosity Shop legends of King Arthur.
Ans : (d) "The Old Curiosity Shop" (1841) is a novel 7. There are two lists given below. Match the
by Charles Dickens. The plot follows the life of Nell
authors in List-I with their nationality in List-
Trent and her grandfather, both residents of the Old
II by choosing the right option against the
Curiosity Shop in London.
code.
2. Who, among the following Indian writers in List -I List-II
English, has created an identifiable imagined (Author) (Nationality)
locale?
(I) Patrick White (1) Canada
(a) Mulk Raj Anand (b) Raja Rao
(II) Nadine Gordimer (2) New Zealand
(c) R.K. Narayan (d) Anita Desai
(III) Margaret Atwood (3) Australia
Ans : (c) "Malgudi Days" (1943) is a collection of
short stories by R.K. Narayan published by Indian (IV) Keri Hulme (4) South Africa
Thought Publication. The book includes 32 stories all Code :
set in the fictional town of Malgudi, located in South (I) (II) (III) (IV)
India. (a) (2) (1) (4) (3)
3. Who among the following is not a formalist (b) (4) (3) (2) (1)
critic? (c) (3) (4) (1) (2)
(a) Allen Tate (b) Cleanth Brooks (d) (3) (2) (4) (1)
(c) Stanley Fish (d) William Empson Ans : (c)
Ans : (c) Stanley Eugene Fish (1938) is an American (I) Patrick White – (3) Australia
literary theorist. He is associated with reader-response (II) Nadine Gordimer – (4) South Africa
theory. (III) Margaret Atwood – (1) Canada
4. The rhyme scheme of the Spenserian sonnet is : (IV) Keri Hulme – (2) New Zealand
(a) abab bcbc cdcd ee (b) abab cdcd efef gg 8. A Shakespearean sonnet has the following
(c) abba cddc effe gg (d) abba abba cde cde rhyme scheme :
Ans : (a) Spenserian sonnet is a sonnet in which the (a) ABBA, ABBA, CDCDCD
lines are grouped into three interlinked quatrains and a (b) ABAB, BCBC, CD CD EE
couplet and the rhyme scheme is abab bcbc cdcd ee. (c) ABAB, CDCD, EFEF, GG
5. Who among the following Marlovian (d) ABBA, ABBA, CDCD, EE
characters is consumed by greed? Ans : (c) Shakespearean sonnets are written
(a) Barabas (b) Tamburlaine predominantly in a meter called Iambic pentameter, a
(c) Doctor Faustus (d) Mephistopheles rhyme scheme in which each line consists of five feet or
Ans : (a) "The Jew of Malta" is a play by Christopher 10 syllables with alternate unstressed followed by
Marlowe, probably written in 1589 or 1590. A stressed syllable.
Marlovian tragic hero belongs to humble family but he 9. "The future of poetry is immense, because in
is a great man because he possesses great quality. poetry..... our race, as time goes on, will find an
UGC NET/JRF English Paper II, June 2011 69 YCT
ever surer and surer stay."–This claim for Ans : (c) Curial sonnet is an eleven line sonnet but
poetry is made in : rather than the first eleven lines of a Standard Sonnet it
(a) Arnold's "The Study of Poetry" consists of precisely 3/4 of the structure of a Petrarchan
(b) Shelley's "A Defence of Poetry" sonnet with the octave becoming a sestet and the sestet
(c) Sidney's "An Apology for Poetry" becoming a quatrain with an additional tail line.
(d) Eliot's "Of Poetry and Poets" 14. The author of the pamphlet "Short View of
Ans : (a) Mathew Arnold's most famous piece of Immorality and Profaneness of the English
literary criticism is his essay "The Study of Poetry". In Stage" (1698) was :
this, Arnold is fundamentally concerned with poetry's (a) John Bunyan (b) Jeremy Collier
"high seriousness". (c) William Wycherley (d) John Vanbrugh
10. Which of the following is not about a dystopia? Ans : (b) Jeremy Collier (1650-1726) was an English
(a) George Orwell's 'Nineteen Eighty-Four' theatre critic, non-Juror bishop and theologian.
(b) Aldous Huxley's 'Brave New World' 15. Identify a play in the following list that is not
(c) William Golding's 'Lord of the Flies' written by Oscar Wilde :
(d) R.M. Ballantyne's 'The Coral Island' (a) A Woman of No Importance
Ans : (d) "The Coral Island: A tale of the Pacific (b) The Importance of Being Earnest
Ocean" (1858) is a novel written by Scottish author (c) Saints and Sinners
R.M. Ballantyne. It is one of the first works of juvenile
(d) An Ideal Husband
fiction to feature exclusively juvenile heroes.
Ans : (c) "Saints and Sinners", an 1884 play, is written
11. Who among the following is not associated with
by Henry Arthur Jones (1851-1929). He was an English
the translation of the Bible?
dramatist.
(a) Miles Coverdale (b) William Tyndale
16. Put the following novels by Charles Dickens in
(c) John Wycliffe (d) Thomas Browne
a sequential order with the help of the code :
Ans : (d) Sir Thomas Browne was an English
1. Great Expectations 2. Hard Times
Polymath and a learned scholar in various fields
including science and medicine, religion and the 3. Bleak House 4. A Tale of Two Cities
esoteric. He is not associated with translation of the Codes :
Bible. (a) 3, 2, 4, 1 (b) 2, 4, 3, 1
12. Arrange the following stages in a sequence in (c) 1, 2, 4, 3 (d) 4, 2, 1, 3
which all Shakespearean tragedies are Ans : (a) 3. 'Bleak House' – 1853
structured. Use the code given below : 2. 'Hard Times' – 1854
I. Denouement II. Conflict 4. 'A Tale of two cities' – 1859
III. Exposition IV. Climax 1. 'Great Expectation' – 1860
Codes: 17. Thomas Kyd's The Spanish Tragedy was
(a) III, II, IV, I (b) III, IV, II, I influenced by :
(c) II, IV, III, I (d) II, IV, I, III (a) Seneca (b) Tertullian
Ans : (a) Stages of Shakespearean tragedies is mainly (c) Virgil (d) Plautus
in the following sequence :
Ans : (a) Thomas Kyd's "Spanish Tragedy" is an
III. Exposition Elizabethan tragedy written between 1582 and 1592. It
II. Conflict is a revenge tragedy. The play was mostly influenced by
IV. Climax Seneca. Seneca (c.4BC-65AD) was Roman Stoic
I. Denouement philosopher, statesman and dramatist of the Silver Age
13. The term, 'curtal sonnet', was coined by : of Latin Literature.
(a) John Milton 18. In its final published version, Eliot's 'The
(b) William Blake Waste Land' contains a total of :
(c) Gerard Manley Hopkins (a) 334 lines (b) 433 lines
(d) Matthew Arnold (c) 373 lines (d) 423 lines
UGC NET/JRF English Paper II, June 2011 70 YCT
Ans : (b) "Waste Land" is widely regarded as one of the 25. 'You Can't Do Both' is a novel by :
most important poems of the 20th century, published in (a) John Fowles (b) Doris Lessing
1922. (c) Kingsley Amis (d) Iris Murdoch
19. Jean Rhys's 'Wide Sargasso Sea' is set in : Ans : (c) "You can't do Both" is a semi-
(a) The Congo region autobiographical novel published in 1994 by Kingsley
(b) The Niger Delta Amis. Amis's first novel "Lucky Jim" (1954) is the most
(c) The Caribbean famous novel.
(d) The African Savannah 26. The character, Nathan Zuckerman, is
Ans : (c) 'Wide Sargasso Sea' is a 1966 post colonial
associated with the fiction of :
novel by Dominica born British author Jean Rhys. The
(a) Norman Mailer (b) Saul Bellow
novel parallels Charlotte Bronte's 'Jane Eyre'.
(c) Philip Roth (d) Bernard Malamud
20. Hamlet, lying wounded, says to his friend,
Ans : (c) Nathan Zuckerman is a fictional character
"Horatio, I am dead." This is an example of :
who appears as the narrator and protagonist in many of
(a) protasis (b) anacrusis
Philip Roth's novels. He appears first in 'My life as a
(c) prolepsis (d) pun
Novel' (1974), then in 'The Ghost Writer' (1979) and
Ans : (c) Prolepsis is a Figurative device by which a
also in 'Zuckerman Unbound' (1981).
future event is presumed to have already occurred.
27. Plato censured poetry because he believed it :
21. 'The Castle of Otranto' is an example of :
(a) Gothic (b) Romance (a) eliminates the ego.
(c) Comic fiction (d) Bildungsroman (b) promotes sensuality.
Ans : (a) "The Castle of Otranto" is a 1764 novel by (c) distorts reality.
Horace Walpole. It is generally regarded as the first (d) cripples the imagination.
gothic novel. Ans : (c) Plato (428/424 or 424/423–348/347 BC) was
22. "The City of Dreadful Night", a long poem a philosopher in classical Greece and the founder of the
depicting the late Victorian sense of gloom and Academy in Athens. Plato has also often been cited as
despondency, is written by : one of the founders of Western religion and spirituality.
(a) Matthew Arnold (b) Robert Browning
28. Which of the following Tennyson poems is a
(c) James Thomson (d) John Davidson
dramatic monologue?
Ans : (c) 'The City of Dreadful Night' is a long poem
by the Scottish poet James Thomson written between (a) In Memoriam
1870 and 1873 and published in 1874 and in 1880 in a (b) "The Charge of the Light Brigade"
book. (c) "Crossing the Bar"
23. Which of the following novels by V. S. Naipaul (d) "Tithonus"
is set in Africa and carries echoes of Joseph Ans : (d) "Tithon or Tithonus" was composed by
Conrad? Alfred Lord Tennyson, written in 1833 and completed
(a) The Mystic Masseur in 1859. It first appeared in the February edition of the
(b) A Bend in the River Cornhill Magazine in 1860.
(c) A House for Mr. Biswas 29. The character Giovanni features in one of the
(d) The Mimic Men following texts :
Ans : (a) "The Mystic Masseur" is a comic novel by (a) John Cleland's 'Fanny Hill: Memoirs of a
V.S. Naipaul. It is set in Colonial Trinidad and was Woman of Pleasure'
published in London in 1957.
(b) John Ford's 'Tis Pity She's a Whore
24. In 'The Rape of the Lock', Belinda's lapdog is
(c) John Braine's 'Room at the Top'
named : (d) John Evelyn's 'Diaries'
(a) Luck (b) Shock
Ans : (b) 'Tis Pity She's a Whore is a tragedy written
(c) Pluck (d) Muck
by John Ford. The play was first published in 1663.
Ans : (b) "Rape of the Lock" is a mock heroic narrative
Ford dedicated the play to John Mordaunt, 1st Earl of
poem written by Alexander Pope, first published in
Peterborough and Baron of Turvey, Bedfordshire.
1712 in two cantos (334 lines). The poem satirises a
minor incident by comparing it to the epic world of the 30. Which of the following poems features the
Gods. phrase, "the still, sad music of humanity"?

UGC NET/JRF English Paper II, June 2011 71 YCT


(a) "Ode: Intimations of Immortality from (a) Kim, A Passage to India, Sons and Lovers,
Recollections of Early Childhood" Brave New World
(b) "Michael: A Pastoral Poem" (b) Sons and Lovers, A Passage to India, Kim,
(c) "The Solitary Reaper" Brave New World
(d) "Tintern Abbey" (c) Kim, Sons and Lovers, A Passage to India,
Brave New World
Ans : (d) "Tintern Abbey" (1798) was written by
(d) Brave New World, Kim, Sons and Lovers, A
William Wordsworth after a walking tour with his
Passage to India
sister. The description of his encounters with the
Ans : (c) Kim – 1901
countryside on the banks grows into an outline of his
Sons and Lovers – 1913
general philosophy.
A Passage to India – 1924
31. Molly Bloom is a character in James Joyce's :
Brave New World – 1932
(a) A Portrait of the Artist as a Young Man
36. "Verses on the Death of Dr. Swift" is written
(b) Dubliners
by :
(c) Ulysses
(a) Alexander Pope (b) Samuel Johnson
(d) Exiles
(c) John Gay (d) Jonathan Swift
Ans : (c) Molly Bloom is the wife of the main
Ans : (d) "Verses on the Death of Dr. Swift" is written
character Leopold Bloom in 'Ulysses'. It is a modernist by Jonathan Swift in 1731 and published in 1739.
novel by Irish writer James Joyce, published in 1922.
37. 'Widower's Houses' was written by :
32. Eliot uses the term "objective correlative" in
(a) Oscar Wilde (b) T.S. Eliot
his essay :
(c) John Galsworthy (d) G.B. Shaw
(a) "The Metaphysical Poets"
Ans : (d) "Widower's Houses" is the first play written
(b) "Hamlet"
by G.B. Shaw to be staged. The play was published in
(c) "Tradition and the Individual Talent" 1892.
(d) "Dante"
38. Who among the following Marxist critics has
Ans : (b) Objective correlative is the term referring to a
reconsidered the classic problem of 'base and
symbolic article used to show inexplicable feelings like superstructure' in relation to literature?
emotion. The term was popularized by Eliot in the essay (a) Edmund Wilson
"Hamlet and His Problems". The term was first used by (b) Raymond Williams
Washington Allston about 1840. (c) Lucien Goldmann
33. "Seamus Heaney was awarded the Nobel Prize (d) Walter Benjamin
for literature in the year : Ans : (b) In Marxist theory human society consists of
(a) 1995 (b) 1996 two parts: The base (or infrastructure) and
(c) 1997 (d) 1998 superstructure. The base comprises the employer-
employee work and the superstructure of a society
Ans : (a) Seamus Justin Heaney (1939-2013) was an
includes its culture, institutions, political power etc.
Irish poet, playwright and the recipient of the 1995
Nobel Prize. 39. "Heteroglossia" refers to:
34. The pamphlet on the Irish condition, "An (a) the multiple readings of a text.
(b) the juxtaposition of multiple voices in a text.
Address to the Irish People" was composed by :
(c) the comments on the margins of a text.
(a) W.B. Yeats (b) P.B. Shelley
(d) the gloss or commentary relating to a text.
(c) Jonathan Swift (d) G.B. Shaw
Ans : (a) Heteroglossia means a diversity of voices,
Ans : (b) P. B. Shelley wrote the above pamphlet and style of discourse of points of view in a literary work
published it in 1812. Shelley visited Ireland and and especially a novel.
immediately became aware of the situation of Irish
40. Margaret Drabble is the author of :
Catholics and wrote a pamphlet.
(a) The Memoirs of a Survivor
35. Which of the following arrangements of
(b) The Witch of Exmoor
English novels is in the correct chronological (c) The Service of Clouds
sequence? (d) The Godless in Eden
UGC NET/JRF English Paper II, June 2011 72 YCT
Ans : (b) 'The Witch of Exmoor" is a 1997 novel by 46. "The Rime of Ancient Mariner" is about :
Margaret Drabble. The novel is a social novel with a (a) a perilous adventure in the sea
focus on exploring the state of post-Thatcher Britain
(b) the accidental killing of an octopus
through the Dickensian satire of the Palmer family.
(c) the curse of a sea God
41. 'MacFlecknoe' is an attack on Dryden's
(d) the guilt and expiation of the Ancient
literary rival :
Mariner
(a) Richard Flecknoe
(b) Thomas Shadwell Ans : (d) 'The Rime of the Ancient Mariner' is a longer
(c) John Wilmot poem by the English poet Coleridge written in 1797-98
and published in 1798, in the first edition of Lyrical
(d) Matthew Prior
Ballads.
Ans : (b) MacFlecknoe is a verse mock-heroic satire by
John Dryden. It is a direct attack on Thomas Shadwell. 47. "To Daffodils" is a poem, written by :
The verse was written about in 1678 but not published (a) Robert Herrick
until 1682. (b) William Wordsworth
42. Eighteenth century writers used satire (c) John Keats
frequently for : (d) P.B. Shelley
(a) attacking human vices and follies.
Ans : (a) Robert Herrick (1591-1674) was a 17th
(b) inciting the reading public.
century lyric poet. He is best known for "Hesperides" a
(c) glorifying the culture of the upper classes.
book of poems. 'To Daffodils' is a short lyric by him.
(d) pleasing their women readers.
48. Which of the following novels reconstructs the
Ans : (a) Eighteenth century writers used satire
frequently for attacking human vices and follies. historical events of the Indian Mutiny?
(a) The Jewel in the Crown
43. Byron's "The Vision of Judgement" is a satire
(b) The Siege of Krishnapur
directed against :
(a) Charles Lamb (b) John Keats (c) The Day of the Scorpion
(c) Henry Hallam (d) Robert Southey (d) The Towers of Silence
Ans : (d) "The Vision of Judgment" (1822) is a Ans : (a) 'The Siege of Krishnapur' (1971) is a novel
satirical poem by Byron, which depicts a dispute in by J.G. Farrell. The book details the siege of a fictional
Heaven over the fate of George III's soul. It was written Indian town Krishnapur during the Indian rebellion of
in response to the Poet Laureate Robert Southey's "A 1857.
Vision of Judgement" (1821) which had imagined the 49. "England, my England" is a poem by :
soul of King George entering the heaven triumphantly
(a) W.E. Henley (b) A.E. Housman
to have his due.
(c) R.L. Stevenson (d) Rudyard Kipling
44. Tom Paine's 'The Rights of Man' was
Ans : (a) William Ernest Henley was an influential
published in :
poet critic and editor of the late-Victorian era in
(a) 1790 (b) 1791
England whose poem 'England, my England' became
(c) 1792 (d) 1793
popular in First World War.
Ans : (b) "The Rights of Man" (1791), a book by
50. Shelley was expelled from the Oxford
Thomas Paine including 21 articles, posits that popular
political revolution is permissible when a government University due to the publication of :
doesn't safeguard the natural rights of its people. (a) The Revolt of Islam
45. Andrew Marvell's "An Horatian Ode upon (b) The Necessity of Atheism
Cromwell's Return from Ireland" was written (c) The Triumph of Life
in : (d) The Masque of Anarchy
(a) 1647 (b) 1649 Ans : (b) 'The Necessity of Atheism' is an essay on
(c) 1650 (d) 1648 atheism by the English poet P.B. Shelley printed in
Ans : (c) The above poem is in the form of a political 1811 for which he, along with his friend Thomas Hogg,
address published around 1650. was expelled from Oxford.

UGC NET/JRF English Paper II, June 2011 73 YCT


UGC NET/JRF Exam, December-2011
ENGLISH
Solved Paper-II
Note : This paper contains fifty (50) objective Ans : (c) A heroic couplet is a traditional form for
type questions, each question carrying two (2) marks. English poetry commonly used in epic and narrative
Attempt all questions. poetry and consisting of a rhyming pair of lines in
1. 'Poems Descriptive of Rural Life and Scenery' iambic pentameter.
is written by : 5. Who, among the following women writers,
(a) William Wordsworth famously imagined the plight of Shakespeare's
(b) Robert Southey sister?
(c) John Clare (a) George Eliot (b) Virginia Woolf
(c) Iris Murdoch (d) Frances Burney
(d) Thomas Gray
Ans : (b) Virginia Woolf invented a fictional character
Ans : (c) John Clare (1793-1864) was an English poet,
Judith as Shakespeare's sister in her essay "A Room of
the son of a farm labourer. His poetry underwent a One's Own" first published in 1929.
major revaluation in the late 20th century and he is
6. Read the following statement and the reason
among the most significant 19th century English poets
given for it. Choose the right response.
who wrote about the English countryside and its
Assertion (A) : Dickens's novels are called
disruption. His above mentioned collection was
'Newgate Novels'.
published in 1820.
Reason (R) : They are called so, because
2. Hemingway's novel 'A Farewell to Arms' is Dickens adulates in these
divided into : novels the careers and
(a) two books (b) three books adventures of criminals.
(c) four books (d) five books (a) Both (A) and (R) are true and (R) is the
Ans : (d) "A Farewell to Arms" is a novel by correct explanation.
Hemingway set during the Italian campaign of world (b) Both (A) and (R) are true, but (R) is not the
correct explanation.
war. The book was published in 1929 and the title is
(c) (A) is true, but (R) is false.
taken from a poem by 16th century dramatist George
(d) (A) is false. but (R) is true.
Peele.
Ans : (a) The Newgate novels or old Baily novels were
3. "Panopticism is the title of a chapter in a well-
novels published in England from the late 1820s till
known book by : 1840. They were thought to glamorise the lives of the
(a) Roman Jakobson (b) Jacques Lacan criminals they portrayed.
(c) Michel Foucault (d) Jacqes Derrida 7. Who among the following writers does not
Ans : (c) "Panopticism" is a social theory named after belong to the group, the University Wits?
the Panopticon, originally developed by French (a) John Lyly (b) Thomas Nashe
philosopher Michel Foucault in his book "Discipline (c) George Peele (d) Thomas Kyd
and Punish" (1975). He viewed the panopticon as a Ans : (d) University Wits is a phrase used to name a
symbol of the disciplinary society of surveillance.group of late 16th century English playwrights who were
educated at the Universities of Oxford or Cambridge
4. The lines, "She was a worthy woman al hir
and who became popular writers. Among the wits were
lyve:/ Housbondes at cherche dore she hadde Marlowe, Greene and Nashe from Cambridge and Lyly,
five", are an example of : Lodge and Peele from Oxford. Kyd was not from
(a) blank verse (b) clerihew University although his name is also sometimes
(c) heroic couplet (d) free verse associated with the University Wits.

UGC NET/JRF English Paper III, December 2011 74 YCT


8. Which of the following characters of Webster's 13. Who famously said, "Three or four families in
'The White Devil' utters the memorable words : a Country Village is the very thing to work
"Oft gay and honour'd robes those tortures try : on"?
We think cag'd birds sing, when indeed they cry." (a) Clara Reeve (b) Maria Edgeworth
(a) Vittoria Corombona (c) Frances Burney (d) Jane Austen
(b) Bracciano Ans : (d) Jane Austen wrote this sentence while writing
(c) The Cardinal her novel "Sense and Sensibility". In this novel Jane
(d) Flamineo Austen collects her country families to explore family
Ans : (d) "The White Devil" (1612) is a revenge relationships, class snobbery and the tension between
tragedy by John Webster (1508-1634). The play was humanistic and economic values.
successfully revived in 1630 by Queen Henrietta's Men 14. Ikemefuna is a character in the novel:
at the Cockpit Theatre and published again in 1631.
(a) When Rain Clouds Gather
9. "All great literature is, at bottom, a criticism of (b) The Mimic Men
life"–this statement is attributed to : (c) Things Fall Apart
(a) Thomas Carlyle (b) Matthew Arnold (d) The Interpreters
(c) J.S. Mill (d) John Ruskin
Ans : (c) "Thing Fall Apart" is a post-colonial novel
Ans : (b) Mathew Arnold (1822-1888) was an English written by Nigerian author Chinua Achebe in 1958. It is
Poet and Cultural critic who worked as an inspector of seen as the archetypal modern African novel in English,
school. In 1865 Arnold published 'Essays in Criticism' one of the first to receive global critical acclaim.
which contains the above lines.
15. A foot consisting of a strong syllable followed
10. Who amongst the following is not a Jewish-
by a weak syllable is called :
American novelist?
(a) Trochee (b) Iambic
(a) J.D. Salinger (b) Henry Greene
(c) Spondee (d) Terza Rima
(c) William Faulkner (d) Philip Roth
Ans : (a) In poetic meter, a trochee is a metrical foot
Ans : (b) Henry Greene (1905-1973) was an English
consisting of a stressed syllable followed by an
author best remembered for the novels "Party Going
unstressed one in English prosody.
and Loving".
16. What is it that Chaucer focuses on in the
11. Which among the following plays by
Christopher Marlowe has epic features? depiction of the Wife of Bath in the Canterbury
(a) Doctor Faustus (b) Edward II Tales?
(c) Hero and Leander (d) Tamburlaine (a) Meekness (b) Defiance
Ans : (d) "Tamburlaine the Great" is a play in two parts (c) Chastity (d) Experience
written by Christopher Marlowe. It is loosely based on Ans : (b) The Wife of Bath's Tale is among the best
the life of the central Asian emperor Timur-Lung. known of Geoffrey Chaucer's Canterbury Tales. It
12. Sir Fopling is a character in : provides insight into the role of women in the late
(a) Wycherley's 'The Plain Dealer' Middle Ages. The tale is often regarded as the first of
the so-called "marriage group" of tales which includes
(b) Congreve's 'The Way of the World'
the Clerk's, the Merchant's and the Franklin's tales.
(c) Etherege's 'The Man of Mode'
(d) Davenant's 'The Platonick Lovers' 17. Put the following books of Pope in a sequence
Ans : (c) "The Man of Mode" or "Sir Fopling Flutter" is of publication. Answer the question with the
a restoration comedy by George Etherege, written in help of the Code given below :
1676. Despite the subtitle the fop Sir Fopling is only (i) The Dunciad
one of several marginal characters, the rake Dorimant is (ii) The Rape of the Lock
the protagonist. (fop – a cheat or foolish person; rake - a (iii) An Essay on Man
licentious or immoral person) (iv) An Essay on Criticism
UGC NET/JRF English Paper II, December 2011 75 YCT
Code : (a) George Villiers (b) John Marston
(a) (ii), (iii), (i), (iv) (b) (i), (ii), (iii), (iv) (c) John Webster (d) Thomas Middleton
(c) (iv), (ii), (i), (iii) (d) (ii), (i), (iv), (iii) Ans : (a) George Villiers (1628-1687), 2nd Duke of
Ans : (c) (iv) An Essay on Criticism – 1711 Buckingham, was an English statesman and poet. He
(ii) The Rape of the hock – 1712 wrote occasional verses, pamphlets, satires and plays
(i) The Dunciad – 1728 and his combined works were published in 1704.
(iii) An Essay on Man – 1735 23. The author of 'Nation and Narration' is :
18. Dinah Morris is a character in George Eliot's (a) Edward Said
novel : (b) Gayatri Chakravorty Spivak
(a) Middlemarch (b) Silas Marner (c) Frantz Fanon
(c) Daniel Deronda (d) Adam Bede (d) Homi Bhabha

Ans : (d) Adam Bede, the first novel written by George Ans : (d) 'Nation and Narration' (1990) is written by
Homi K. Bhabha. He is the Anne F. Rothenberg
Eliot (the pen name of Mary Ann Evans) was published
Professor of English and American Literature. He has
in 1859. It was published pseudonymously.
developed a number of key concepts such as hybridity,
19. The Booker Prize is awarded by a panel of
mimicry, difference and ambivalence.
judges to the best novel by a citizen of :
24. Which of the following novels has a great
(a) the United Kingdom
impact on the formal experimentation in
(b) the British Commonwealth
contemporary fiction?
(c) the United Kingdom or the British
(a) Thomas Nashe's 'The Unfortunate Traveller'
Commonwealth
(b) Henry Fielding's 'Tom Jones'
(d) the United Kingdom or the British
(c) Laurence Sterne's 'Tristram Shandy'
Commonwealth or the Republic of Ireland
(d) Samuel Richardson's 'Pamela'
Ans : (b) The Man Booker Prize is a literary prize
Ans : (c) "The Life and Opinions of Tristram Shandy or
awarded each year for the best original novel written in
Gentle Man" is a humorous novel by Laurence Sterne.
English language and published in the U.K. and all the
It was published in nine volumes, the first two
nations of the British Commonwealth.
appearing in 1759 and seven others following over the
20. A 'curtal sonnet' consists of : next Seven years.
(a) 11 lines (b) 12 lines 25. The phrase 'Only Connect' is associated with:
(c) 13 lines (d) 14 lines (a) D.H. Lawrence (b) James Joyce
Ans : (a) The curtal sonnet is a form invented by (c) E.M. Forster (d) Virginia Woolf
Gerard Manley Hopkins and used in three of his poems. Ans : (c) E.M. Forster (1879-1970) was an English
It is an 11 line or more accurately a 10 line sonnet. But novelist, short story writer etc. Forster's humanistic
rather than the first eleven lines of a standard sonnet it impulse towards understanding and sympathy may be
consists of precisely 3/4 of the structure of a Petrarchan aptly summed up in the epigraph to his 1910 novel
sonnet shrunk proportionally. 'Howard's End: Only Connect'.
21. "The Unfortunate Traveller" has been 26. Which of the following books is by Margaret
authored by :
Atwood?
(a) Robert Greene (b) Thomas Deloney
(a) The Stone Angel (b) No Fixed Address
(c) Thomas Nashe (d) Thomas Lodge
(c) The Edible Woman (d) Halfbreed
Ans : (c) "The Unfortunate Traveller or life of Jack
Ans : (c)
Wilton" is a picaresque novel by Thomas Nashe, first
'The Stone Angel' (1964) – Margaret Lawrence
published in 1564 but set during the reign of Henry VIII
'No Fixed Address' (1998) – Aritha van Herk
of England.
'The Edible Woman' (1969) – Margaret Atwood
22. Who, among the following, is not a practitioner
'Half Breed' (1973) – Maria Campbell
of Jacobean tragedy?
UGC NET/JRF English Paper II, December 2011 76 YCT
27. The expression "murderous innocence" is an 32. Match the following authors with their
example of : respective works with the help of the code
(a) Oxymoron (b) Zeugma given below :
(c) Chiasmus (d) Pun List-I List-II
I. Oliver Goldsmith 1. The Vanity of
Ans : (a) Oxymoron is a figure of speech in which
apparently contradictory terms appear in conjunction. Human Wishes
Example: faith unfaithful kept him falsely true. II. John Gay 2. The Vicar of
Wakefield
28. Read the following statement and the reason
III. Samuel Johnson 3. She Stoops to
given for it. Choose the right response :
Conquer/
Assertion (A) : Othello killed Desdemona.
School for Scandal
Reason (R) : Because Desdemona
IV. Richard Sheridan 4. The Beggar's Opera
committed infidelity.
Code :
(a) Both (A) and (R) are true and (R) is the
I II III IV
correct explanation.
(a) 1 4 3 2
(b) Both (A) and (R) are true, but (R) is not the
(b) 2 4 1 3
correct explanation.
(c) 3 2 4 1
(c) (A) is true, but (R) is false.
(d) 4 3 2 1
(d) (A) is false. but (R) is true.
Ans : (b) "The Vanity of Human Wishes" (1749) by
Ans : (c) Othello killed Desdemona because he
Samuel Johnson
believed that Desdemona is unfaithful to him with
"The Vicar of Wakefield" (1766) by Oliver Goldsmith
Cassio.
"She Stoops to Conquer" (1773) by Oliver Goldsmith
29. The Enlightenment believed in the universal
"The Beggar's Opera" (1728) by John Gay
authority of :
Note: Richard Sheridan has written 'School for
(a) Religion (b) Tradition
Scandal'.
(c) Reason (d) Sentiments
33. The term "egotistical sublime" was coined by :
Ans : (c) The age of Enlightenment sometimes called
the Age of Reason advocating reason as a means of (a) S.T. Coleridge (b) John Keats
establishing an authoritative system. (c) William Wordsworth (d) William Hazlitt

30. Which of the following works of John Milton is Ans : (b) The phrase was first used by Keats in a letter
to Richard Woodhouse dated 27 Oct. 1818 saying that a
an elegy?
poet 'has no self'.
(a) Lycidas (b) L' Allegro
34. Put the following novels of George Eliot in a
(c) Camus (d) Paradise Lost
sequential order. Answer the question with the
Ans : (a) "Lycidas" is a poem by John Milton written in
help of the code :
1637 as a pastoral elegy. It first appeared in 1638.
(i) Middlemarch (ii) Daniel Deronda
31. Which of the following poem by Keats uses the
(iii) Felix Holt, the Radical (iv) Romola
Spenserian stanza?
Code :
(a) Endymion
(a) (i), (iii), (iv), (ii) (b) (ii), (i), (iii), (iv)
(b) The Fall of Hyperion
(c) (iv), (iii), (i), (ii) (d) (iv), (i), (iii), (ii)
(c) The Eve of St. Agnes
(d) Lamia Ans : (c) (iv) 'Romola' – 1862
(iii) 'Felix Holt, The Radical' – 1866
Ans : (c) "The Eve of St. Agnes" is a poem (42 stanzas)
by John Keats written in 1819 and published in 1820. (i) 'Middlemarch' – 1871-72
The poem is in Spenserian stanzas. (ii) 'Daniel Deronda' – 1876

UGC NET/JRF English Paper II, December 2011 77 YCT


35. Who, among the following writers, is known (a) Richard Steele
for his unforgettable sense of humour and (b) John Dennis
comedy? (c) John Locke
(a) D.H. Lawrence (d) Joseph Addison
(b) P.G. Wodehouse Ans : (d) "Pleasures of Imagination" first published in
(c) Thomas Hardy the Spectator series in 1712, was written by Joseph
(d) John Galsworthy Addison.
Ans : (d) John Galsworthy (1867-1933) was an English 39. Read the following statement and the reason
novelist and playwright. Notable works include "The given for it. Choose the right response:
Forsyte Saga" (1906-1921) and its sequels, 'A Modern
Assertion (A) : 'Gulliver's Travels' earned
Comedy' and 'End of the Chapter'. He won the Nobel
Jonathan Swift the bad
Prize in Literature in 1932.
name of being a misanthrope.
36. Which of the following is not an apocalyptic Reason (R) : Swift in the novel was neutral
novel? to the image of man.
(a) Doris Lessing's 'The Four-Gated City' (a) Both (A) and (R) are true and (R) is the
(b) L.P. Hartley's 'Facial Justice' correct explanation.
(c) Anthony Burgess's 'The Wanting Seed' (b) Both (A) and (R) are true, but (R) is not the
(d) V.S. Naipaul's 'A House for Mr. Biswas' correct explanation.
Ans : (d) D. Lessing's 'The Four-gated City' (1969), a (c) (A) is true, but (R) is false.
dystopian novel, is the last of her five-volume series (d) (A) is false, but (R) is true.
'The Children of Violence' began in 1952. L. P. Ans : (b) "Gulliver's Travels" examines human nature
Hartley's 'Facial Justice' (1960) depicts a (dystopian)
through a misanthropic lens and through satires
post-apocalyptic society seeking to banish and envy.
examines the changes English society was undergoing.
"The Wanting Seed" is a dystopian novel by the
40. Who, amongst the following, does not belong to
English author Anthony Burgess (1917-1993) written in
1962. the 'Great Tradition', enunciated by F. R.
Naipaul's 'A House for Mr. Biswas' (1961) looks at life Leavis?
in the post-colonial world through Mohun Biswas. It is (a) Joseph Conrad
not a dystopian novel. (b) James Joyce
37. Identify the author of the following lines: (c) Jane Austen
(d) George Eliot
Let sea-discoverers to new worlds have gone,
Let Maps to other, worlds on worlds have shown Ans : (b) James Joyce (1882-1941) was an Irish
Let us possess one world, each hath one, and is novelist and poet. He contributed to the modernist
one. avant-garde and is regarded as one of the most
(a) Shakespeare influential and important authors of the 20th century. He
does not belong to Leavis' 'Great Tradition'.
(b) George Herbert
(c) John Donne 41. Isaac Bashevis Singer is an :
(d) Henry Vaughan (a) African-American writer
Ans : (c) "The Good -Morrow" is published in Donne's (b) American-Jewish writer
1633 collection 'Songs and Sonnets'. The above lines (c) American-Indian writer
have been taken from this poem. (d) American-Asian writer
38. In the summer of 1712, The Spectator Ans : (b) Isaac Bashevis Singer (1902-1991) was a
published a series of essays on "The Pleasures Polish-born Jewish author in Yiddish, awarded the
of Imagination," written by : Nobel Prize in 1978.

UGC NET/JRF English Paper II, December 2011 78 YCT


42. Samuel Beckett's 'Waiting for Godot' has : 47. "On the Knocking at the Gate in Macbeth" is a
(a) three Acts (b) five Acts longer essay by :
(c) four Acts (d) two Acts (a) G. Wilson Knight
Ans : (d) "Waiting for Godot", premiered in 1953, is an (b) A.C. Bradley
absurd play written by Samuel Beckett. (c) Thomas De Quincey
43. James Joyce's 'Exiles' is a : (d) F.R. Leavis
(a) Short Story (b) Poem Ans : (c) "On the knocking at the Gate in Macbeth" is
(c) Play (d) Novel an essay in Shakespearean Criticism first published in
Ans : (c) 'Exiles' is a play by James Joyce. It draws on 1823.
the story of "The Dead", the final story in Joyce's story 48. The expression, "dreaming house" is an
collection, 'Dubliners'. The play was published in 1918. example of :
44. "It was a bright cold day in April and the (a) Zeugma
clocks were striking thirteen"– is the opening (b) Transferred epithet
sentence of : (c) Chiasmus
(a) Ulysses (d) Apostrophe
(b) Nostromo Ans : (b) A transferred epithet is a figure of speech in
(c) Chrome Yellow which an epithet grammatically qualifies a noun other
(d) Nineteen Eighty-Four than the person or thing it is actually describing, also
Ans : (c) 'Nineteen Eighty-Four' often published as known in rhetoric as 'hypallage'.
'1984', is a dystopian novel by English author George 49. The term 'Practical Criticism' is coined by :
Orwell published in 1949. (a) William Empson
45. The subtitle of William Godwin's Caleb (b) W.K. Wimsatt, Jr.
Williams is : (c) I.A. Richards
(a) Man As He Is Not (d) F.R. Leavis
(b) Man As He Is Ans : (c) I.A. Richards was an English educator,
(c) Things As They Are literary critic, whose work contributed to the foundation
(d) The Pupil of Nature of the New Criticism. He wrote the book "Principles of
Ans : (c) "Things as They Are or The Adventures of Literary Criticism" in which he coined the above
Caleb Williams" by William Godwin is a three volume phrase.
novel published in 1794. 50. Victor Shklovsky's name is associated with :
46. Who amongst the following belongs to the (a) Post-modernism
group of radical feminists? (b) New Historicism
(a) Helene Cixous (c) Reader Response Theory
(b) Monica Wittig (d) Russian Formalism
(c) Simone de Beauvoir Ans : (d) Russian Formalism (1910-1930) was an
(d) Luce Irigaray influential school of Literary Criticism in Russia.
Ans : (a) Helene Cixous (b. 1937) is a professor Russian Formalism exerted a major influence on
Algerian/ French feminist writer and literary critic. She thinkers like Mikhail Bakhtin and on Structuralism as a
founded the first centre of feminist studies at the whole. Shklovsky was a Russian formalist best known
European University of Paris. She is best known for her for the concept of 'defamiliarization' or 'ostranenie'
article "The Laugh of the Medusa", which established (translated as 'estrangement') in literature. His work
her as one of the pioneers of poststructuralist feminist pushes Russian Formalism towards understanding
theory. literary activity as an integral part of social practice.

UGC NET/JRF English Paper II, December 2011 79 YCT


UGC NET/JRF Exam, June-2012
ENGLISH
Solved Paper-III
Note : This paper contains seventy five (75) 4. Items in a published book appear in the
objective type questions of two (2) marks each. All following order:
questions are compulsory. (a) Index, Copyright Page, Bibliography,
1. In Ben Jonson's Volpone, the animal imagery Footnotes
includes- (b) Copyright Page, Bibliography, Index,
(A) the fox and the vulture Footnotes
(B) the fly and the cockroach (c) Copyright Page, Footnotes, Bibliography,
(C) the fly, the crow and the raven Index
(D) the fox, the vulture and the goat (d) Bibliography, Copyright Page, Index,
(a) (A) and (B) are correct. Footnotes
(b) only (D) is correct. Ans: (c) The correct order of items in a book –
(c) (B) and (D) are correct. Copyright Page → Footnotes → Bibliography → Index
(d) (A) and (C) are correct. Hence option (c).
Ans: (d) In Ben Jonson’s ‘Volpone’ (1605-06) the 5. Match the following :
animal imagery includes – The fox, vulture, fly, crow (I) James Thomson, (a) Metaphysical
and the raven. Oliver Goldsmith, poets
Hence option (d). William Cowper,
George Crabbe
2. Salman Rushdie's "Imaginary Homelands"
is____. (II) George Herbert, (b) Transitional
(a) a discussion of imperialist assumptions. Henry Vaughan, poets
(b) an essay that propounds an anti-essentialist Andrew Marvell,
view of place. Abraham Cowley,
(c) an existential lament on triumphant John Donne
colonialism. (III) Rupert Brooke, (c) War poets
(d) an orientalist description of his favourite Wilfred Owen,
homelands. Siegfried Sassoon,
Ans: (b) Rushdie’s ‘Imaginary Homelands' is an essay Edmund Blunden,
that propounds an anti-essentialist view of place. Hence Robert Graves,.
option (b). (IV) W. H. Davies, (d) Georgians
Walter de la Mare,
3. Identify the incorrect statement below:
John Drinkwater,
(A) BASIC was an experiment initiated by C. K.
Ogden and I. A. Richards from 1926 to about Rupert Brooke
1940. (I) (II) (III) (IV)
(B) Expanded, BASIC read: Broadly Ascertained (a) (d) (a) (c) (b)
Scientific International Course. (b) (d) (b) (d) (a)
(C) BASIC English was an attempt to reduce the (c) (b) (a) (c) (d)
number of essential words to 850. (d) (a) (c) (d) (b)
(D) while keeping to normal construction, Ans: (c) The correct matches are –
BASIC failed as an experiment because its I. J. Thomson, O. Goldsmith, (b) Transitional
documents were far too complicated and W. Cowper, G. Crabbe Poets
technical to understand. II. G. Herbert, H. Vaughan, (a) Metaphysical
(a) (A) and (B) (b) (B) and (D) A. Marvell, A. Cowley, Poets
(c) (A) and (C) (d) (C) and (D) J. Donne
Ans: (b) BASIC is expanded as British Scientific III. R. Brooke, W. Owen, (c) War Poets
International Commercial and Basic English was an 850 S. Sassoon, E. Blunden
word simplified auxiliary language presented in R. Graves,
Ogden's 1930 book 'Basic English: A General IV. W.H. Davies, W. de la Mare, (d) Georgians
Introduction with Rules and Grammar'. J. Drinkwater, R. Brooke
Hence option (b). Hence option (c).

UGC NET/JRF English Paper III, June 2012 80 YCT


6. The following phrases from Shakespeare have (a) (A) and (B) (b) (C) and (D)
become the titles of famous works. Identify the (c) (B) and (C) (d) (A) and (D)
correctly matched group. Ans: (c) The emergence of the concept of ‘World
(I) Pale Fire (a) Thomas Literature’ is associated with Goethe and Herder.
Hardy Hence option (c).
(II) The Sound and (b) Somerset 9. Gunter Grass's 'Tin Drum' is part of a trilogy
the Fury Maugham known as the Danzig trilogy. The other two
(III) Rosencrantz and (c) William novels are:
Guildenstern are Faulkner (a) The Flounder and Dag years
Dead (b) The Rat and Cat and Mouse
(IV) Under the (d) Tom (c) Cat and Mouse and Dog Years
Greenwood Tree Stoppard (d) Crabwalk and The Rat
(V) Of Cakes and Ale (e) Vladimir Ans: (c) Gunter Grass' Danzig Trilogy has three novels-
Nabokov 'Tin Drum', 'Cat and Mouse' and 'Dog Years'.
(I) (II) (III) (IV) (V) Hence option (c).
(a) (e) (d) (c) (a) (b) 10. The hostess proudly announces that the family
(b) (d) (e) (b) (c) (a) can afford a servant and her daughters have
(c) (e) (c) (d) (a) (b) nothing to do with the kitchen. Who is the
(d) (c) (d) (b) (e) (a) proud mother in this Jane Austen novel?
Ans: (c) The correctly matched pairs are – (a) Mrs. Morland
I. 'Pale Fire' (1962) – Vladimir (b) Lady Catherine de Burgh
Nabokov (c) Mrs. Bennet
II. 'The Sound and (d) Mrs. Dashwood
the Fury' (1929) – William Faulkner Ans: (c) Mrs. Bennet in Austen’s ‘Pride and Prejudice’
III. 'Rosencrantz and – Tom Stoppard (1813) proudly announces that their family could afford
Guildenstern are a servant and so her daughters had nothing to do with
Dead' (1966) the kitchen.
IV. 'Under the Greenwood – Thomas Hardy Hence option (c).
Tree' (1872) 11. When Keats writes about the "beaker full" of
V. 'Of Cakes and Ale' – Somerset "The blushful Hippocrene", Hippocrene is:
(1930) Maugham (a) the fountain of the horse
Hence option (c). (b) a spring sacred to the Muses
7. Identify the statement that is NOT TRUE (c) Mount Helicon produced from a blow of
among those that explain "stage directions" in Pegasus
drama. (d) Both (a) and (b)
(a) Stage directions inform readers how to stage, Ans: (d) The term Hippocrene as used by Keats in one
perform or imagine the play. of his poems means both the fountain of the horse and a
(b) The place. time of action, design of the set spring sacred to the Muses.
and at times characters' actions or tone of Hence option (d).
voice are indicated by stage directions. 12. Which of the following statements on The
(c) Stage directions are often italicized in the text Prelude by William Wordsworth is/are not
of a play in order to be spoken aloud. true?
(d) Stage directions may appear at the beginning (A) The Prelude was published posthumously.
of a play, before a scene or attached to a line
(B) In this poem, Wordsworth records his
of dialogue.
development as a poet.
Ans: (c) The incorrect statement is (c) because stage (C) The poem runs to 14 books; at crucial stages
directions are not to be spoken aloud, they are just the poet celebrates the sublime natural
indicators. scenery in developing his spiritual, moral and
Hence option (c). imaginative nature.
8. The emergence of the concept of "World (D) Poems like' "Michael", "The Old Cumberland
Literature" is associated with: Beggar", "She dwelt among the untrodden
(A) Friedrich Schiller ways", "Nutting" etc. are the highlights of
(B) Johann Wolfgang von Goethe this volume.
(C) Johann Gottfried Herder (a) (A) to (D) are true. (b) (A) is not true.
(d) Immanuel Kant (c) (D) is not true. (d) Only (C) is true.

UGC NET/JRF English Paper III, June 2012 81 YCT


Ans: (c) The poems like ‘Michael’ ‘The Old Ans: (c) The above view is known as – The Great
Cumberland Beggar’, ‘She dwelt among the untrodden Chain of Being.
ways’, ‘Nutting’ etc are the part of 1800 edition of Hence option (c).
‘Lyrical Ballads.’ The first edition of ‘The Prelude’ 17. In Virginia Woolf's To the Lighthouse the
came out in 1805 with 13 books which were lighthouse does not symbolize :
subsequently revised and published posthumously by
(a) permanence at the heart of change.
his wife in 1850. It is a poem and not a volume of
poems. (b) change in the unchanging world.
Hence option (c). (c) celebration of life in the heart of death.
(d) celebration of order in the heart of chaos.
13. Assertion (A) : At the end of Heart of Darkness,
Marlow tells a lie to the Intended about Kurtz Ans: (b) In V. Woolf’s 'To the Lighthouse' (1927) the
when he tells her "The last word he pronounced lighthouse does not symbolize change in the
was your name". unchanging world. It symbolizes the other three
Reason (R) : Marlow tells this lie because he is statements.
secretly in love with the Intended and tells her Hence option. (b)
what she wants to hear. 18. "Can one imagine any private soldier, in the
(a) Both (A) and (R) are true: (R) is the correct nineties or now, reading Barrack Room Ballads
explanation. and feeling that here was a writer who spoke
(b) Both (A) and (R) are true, but (R) is not the for him? It is very hard to do so. [....] When he
correct explanation. is writing not of British but of "loyal" Indians
(c) (A) is true, but (R) is false. he carries the 'Salaam, Sahib' motif to
(d) (A) is false, but (R) is true. sometimes disgusting lengths. Yet it remains
Ans: (b) Both (A) and (R) are true but (R) is not the true that he has far more interest in the
correct explanation of (A). common soldier, far more anxiety that he shall
Hence option (b). get a fair deal, than most of the "liberals" of
his day and our own. He sees that the soldier is
14. Ear-training in ELT is easily achieved by:
neglected, meanly underpaid and
(A) composition (B) dictation hypocritically despised by the people whose
(C) cloze tests (D) listening exercises incomes he safeguards".
(E) precis writing (a) This is E. M. Forster's "India, Again".
(a) (A) and (E) (b) (A), (C) and (E) (b) This is Malcolm Muggeridge on E. M.
(c) (B), (C) and (D) (d) (B) and (D) Forster's India.
Ans: (d) Ear training in ELT is easily achieved by – (c) This is T. S. Eliot on Rudyard Kipling.
dictation and listening exercises. (d) This is George Orwell on Rudyard Kipling.
Hence option (d). Ans: (d) The above is a commentary on Rudyard
15. William Shakespeare's Julius Caesar, Antony Kipling by George Orwell.
and Cleopatra and Coriolanus are based on____ Hence option (d).
(a) Holinshed's Chronicles
19. In the well-known poem "To his coy mistress",
(b) Folk-tales and legends
the word coy means
(c) Older Roman Plays
(a) shy (b) timid
(d) Plutarch's Lives
(c) voluptuous (d) sensuous
Ans: (d) Shakespeare’s ‘Julius Caesar’, ‘Antony and
Ans: (a) The word ‘coy’ means 'shy'.
Cleopatra’ and ‘Coriolanus’ are based on Plutarch’s
‘Lives’. Hence option (a).
Hence option (d). 20. From the following list, identify
"backformation": Sulk, bulk, stoke, poke,
16. The basic concept that creation was ordered,
swindle, bundle.
that every species exists in a hierarchy of
status, from God to the lowest creature, was (a) Sulk, bulk, stoke, poke
prevalent in the Renaissance. In this (b) Stoke, poke, swindle, bundle
hierarchical continuum, man occupies the (c) Sulk, stoke, bundle
middle position between the animal kinds and (d) Bulk, poke, bundle
the angels. This world view is known as: Ans: (*) UGC has considered all options correct and
(a) Humanism marks was awarded to every candidate.
(b) The Enlightenment Back-formation is either the process of creating a new
(c) The Great Chain of Being lexeme by removing actual or supposed affixes, or a
(d) Calvinism neologism formed by such a process.
UGC NET/JRF English Paper III, June 2012 82 YCT
21. "It blurs distinctions among literary, non- (c) It is a set of advanced grammatical principles
literary and cultural texts, showing how all assumed to be fundamental to all natural
three intercirculate, share in, and mutually languages.
constitute each other." What does it in this (d) It is a set of universally respected practices
statement stand for? that have come, in time, to be known as
"grammar".
(a) Marxism (b) Structuralism
(c) Formalism (d) New Historicism Ans: (b) Universal Grammar is so called because it is
a set of basic grammatical principles assumed to be
Ans: (d) The above description is about New fundamental to all natural languages.
Historicism. Hence option (b).
Hence option (d). 27. Identify the novel with the wrong subtitle listed
22. For, though, I've no idea. What this accoutred below :
frowsty___ is worth, It pleases me to stand in (a) Middlemarch, a Study of Provincial Life
silence here. (Fill in the blank) (b) Tess of the D'Urbervilles, A pure Woman
(a) bar (b) barn (c) The Mayor of Casterbridge, A Man of
(c) attic (d) alcove Character
Ans: (b) The above extract is from the 6th stanza of P. (d) Felix Holt, the Socialist
Larkin’s poem ‘Church Going’ and the word in the Ans: (d) The subtitle of George Eliot’s 'Felix Holt'
blank is – ‘barn’. (1866) is ‘The Radical’.
Hence option (b). Hence option (d).
28. Match List-I with List-II
23. Which of the following novels is NOT a
List-I List-II
partition novel ?
(I) David Malouf (a) The Solid
(a) Azadi (b) Tamas
Mandala
(c) Clear Light of the Day (d) That Long Silence (II) Patrick White (b) Wild Cat
Ans: (d) Shashi Deshpande’s ‘That Long Silence’ Falling
(1988) is a story of a middle- class Indian woman’s life (III) Peter Carey (c) Remembering
with a feminist perspective. It is not a partition novel. Babylon
Hence option (d). (IV) Colin Johnson (d) True History of the
24. Of the following characters, which one does not Kelly Gang
belong to A House for Mr. Biswas? (I) (II) (III) (IV)
(a) Raghu (b) Ralph Singh (a) (a) (c) (b) (d)
(c) Dehuti (d) Tara (b) (c) (a) (d) (b)
(c) (b) (c) (a) (d)
Ans: (b) Ralph Singh is the protagonist of V.S.
(d) (c) (d) (b) (a)
Naipaul’s 'The Mimic Men' (1967).
The other characters are from Naipaul’s 'A House for Ans: (b) The correct matches are –
Mr. Biswas' (1961). I. David Malouf – (c) ‘Remembering
Babylon’ (1993)
Hence option (b).
II. Patrick White – (a) ‘The Solid
25. In English literature, the trope of the vampire Mandala’ (1966)
was used for the first time by : III. Peter Carey – (d) ‘True History of
(a) Matthew Gregory Lewis the Kelly Gang’
(b) John Polidori (2000)
(c) John Stagg IV. Colin Johnson – (b) ‘Wild Cat Falling’
(d) Bram Stoker (1965)
Ans: (c) The trope of Vampire was introduced in (Mudrooroo)
English literature by John Stagg. Hence option (b).
Hence option (c). 29. The opening sentence of Tolstoy's Anna
26. Why is "Universal grammar" so called" Karenina, "Happy families are all alike, every
unhappy family is unhappy in its own way".
(a) It is a set of basic grammatical principles The specific cause of the unhappiness in
universally followed and easily recognized Oblonsky's house was the husband's affair with :
by people. (a) a Kitchen- maid
(b) It is a set of basic grammatical principles (b) an English governess
assumed to be fundamental to all natural (c) a French governess
languages. (d) a socialite
UGC NET/JRF English Paper III, June 2012 83 YCT
Ans: (c) In Tolstoy’s ‘Anna Karenina’ (1877) , the 33. What is register?
cause of unhappiness in Oblonsky’s house was the (a) The way in which a language registers in the
husband’s affair with a French Governess. minds of its users.
Hence option (c). (b) The way users of a language register the
30. This periodical had the avowed intention "to nuances of that language.
enliven morality with wit and to temper wit (c) A variety of language used in social
with morality.. to bring philosophy out of the situations or one specially designed for the
closets and libraries, schools and colleges, to subject it deals with.
dwell in clubs and assemblies, at tea-tables and (d) A variety of language used in non-
coffee houses". It also promoted family, professional or informal situations by
marriage and courtesy. The periodical under professionals.
reference is : Ans: (c) A ‘register’ is a variety of language used in
(a) The Tatler social situations or one specially designed for the
(b) The Spectator subject it deals with.
(c) The Gentleman's Magazine Hence option (c).
(d) The London Magazine 34. Jeremy Collier's Short View of the Immorality
Ans: (b) The above described periodical is Addison’s and profaneness of the English Stage (1698)
'The Spectator'. attacked_____
Hence option (b). (a) the practice of mixing tragic and comic
31. Assertion (A) : "Tam O' Shanter" by John Clare themes in Shakespeare's plays.
is about the experience of an ordinary human (b) the bawdiness of "low" characters in
being and became quite popular during that time. Shakespeare's plays.
Reason (R) : John Clare, having suffered bouts of (c) the coarseness and ugliness of Restoration
madness, could really feel for the misery of Theatre.
common man. (d) irreligious themes and irreverent attitudes in
In the context of the two statements, which of the plays of the seventeenth century.
the following is correct ? Ans: (c) Collier’s 1698 work attacked the coarseness
(a) Both (A) and (R) are true and (R) explains and ugliness of Restoration Theatre.
(A).
Hence option (c).
(b) Both (A) and (R) are true, but (R) does not
explain (A). 35. One of the most important themes the speakers
(c) (A) is true but (R) is false. debate in Dryden's An Essay on Dramatic Poesy
is______
(d) (A) is false but (R) is true.
(a) European and non-English perceptions of
Ans: (b) The (R) does not tell us about the points raised reality.
in (A), so, both (A) and (R) are true but (R) does not
(b) English and non-English perceptions of
explain (A).
reality.
Hence option (b).
(c) the relative merits of French and English
32. Alexander Pope's An Essay in Criticism : theatre.
(A) Purports to define "wit" and "nature" as they (d) the relative merits of French and English
apply to the literature of his age. poetry.
(B) Claims no originality in the thought that
Ans: (c) Dryden’s ‘essay’ debates the relative merits of
governs this work.
French and English theatre.
(C) is a prose essay that gives us such quotes as
Hence option (c).
"A little learning is a dangerous thing !"
(D) Appeared in 1701. 36. Identify the correctly matched pair:
(a) (C) and (D) are incorrect. (a) Amitav Ghosh - All About H. Hatterr
(b) (A) and (B) are incorrect. (b) Anita Desai - Inheritance of Loss
(c) (A) to (D) are correct. (c) Shashi Deshpande - A Bend in the Ganges
(d) Only (A) and (C) are correct (d) Salman Rushdie - The Enchantress of
Ans: (*) Alexander Pope's 'An Essay in Criticism' Florence
appeared in 1711 and purports of define 'wit' and Ans: (d) Salman Rushdie – 'The Enchantress of
'nature' as they apply to the literature of his age. It also Florence' (2008) is the correct pair. Other correct pairs
claims no originality in the thought that governs this are:
work and gives us famous quotes such as "A little ‘All About H. Hatterr’ (1948) – G.V. Desani
learning is a dangerous thing". ‘Inheritance of Loss’ (2006) – Kiran Desai
Note- UGC has considered all options corrects and ‘A Bend in the Ganges’ (1965) – Manohar Malgonkar
marks was awarded to all candidates. Hence option (d).

UGC NET/JRF English Paper III, June 2012 84 YCT


37. Match the following correctly : (d) Disneyland is both 'appearance' and 'reality'
(I) Langue / Parole (A) Noam Chomsky in the post modern visual game of handy-
(II) Competence / (B) C. S. Pierce dandy.
performance Ans: (c) Jean Baudrillard adopted Disneyland as his
(III) Iconic / Indexical (C) Ferdinand de own sign because it is an artefact that so obviously
Saussure announces its own fictiveness that it would seem to
imply some counter balancing reality.
(IV) Readerly/Writerly (D) Roland Barthes
Hence option (c).
(I) (II) (III) (IV)
(a) (C) (B) (A) (D) 40. Which of the following statements is NOT
TRUE of Dante Gabriel Rossetti?
(b) (C) (A) (B) (D)
(a) D. G. Rossetti was a Londoner, the son of an
(c) (A) (C) (D) (B)
Italian refugee who taught Italian at King's
(d) (B) (C) (A) (D) college.
Ans: (b) The correct pair will be– (b) Rossetti formed the Pre Raphaelite
I. Langue/Parole C. Ferdinand de Brotherhood with Holman Hunt, Ford
Saussure Maddox Brown and Painter Millais.
II. Competence/Performance A. Noam Chomsky (c) he married Christina Georgina who was a
III. Iconio/Indexical B. C.S. Pierce poet in her right.
(d) Rossetti's "Blessed Damozel" displays his
IV. Readerly/writerly D. Roland Barthes remarkable gifts as a poet and painter.
Hence, option (b) is correct. Ans: (c) Christina Georgina Rossetti was the sister of
38. 1. Joy Kogawa (a) Body Rites D.G. Rossetti who modeled for some of his famous
2. M. G. Vasanjee (b) Obasan paintings and was hailed as Elizabeth Barrett
3. Sky Lee (c) The Gunny Sack Browning’s successor for her poetry.
4. Arnold Itwaru (d) Disappearing Moon Hence option (c).
Cafe 41. Goethe's 'Faust' (Part I, Scene 1) opens in:
1 2 3 4 (a) heaven (b) hell
(a) (d) (a) (b) (c) (c) forest (d) Faust's study
(b) (a) (d) (c) (b) Ans: (d) Goethe’s ‘Faust’ opens in Faust’s study.
(c) (b) (c) (d) (a) Hence option (d).
(d) (a) (b) (c) (d) 42. "Is it their single-mind-sized skulls or a trained
Ans: (c) The correct pairs are – Body, or genius, or a nestful of brats Gives
1. Joy Kogawa – (b) ‘Obasan’ (1981) their days this bullet and automatic purpose..."
(Japanese-Canadian) (Thrushes)
2. M.G. Vassanji. – (c) ‘The Gunny In the above lines what does 'their' refer to and
(Nairobi-Tanzanian) sack’ (2005) what quality of 'their' does the poet speak of?
3. Sky Lee – (d) ‘Disappearing I. Human beings and their intelligence
(Chinese-Canadian) Moon Café’ II. The thrushes and their concentration in
(1990) achieving what they set our for
4. Arnold Itwaru – (a) ‘Body Rites’ III. The efficiency of the thrushes in getting at
(1991, book their prey
of poetry) IV. All the above
Hence option (c). (a) Only III is correct. (b) Only IV is correct.
39. Why does Jean Baudrillard adopt Disneyland (c) I and II is correct. (d) II and III are correct.
as his own sign? Ans: (d) In Ted Hughes' ‘Thrushes’ The word ‘their’
(a) Disneyland is by far the most eminently refers to the thrushes and their concentration in
noticeable cultural sign in the post modern achieving what they set out for and the efficiency of the
world. thrushes in getting at their prey.
(b) Disneyland captures 'essences' and 'non- Hence option (d).
essences' of Reality more convincingly than 43. Find the odd (wo)man our : Belladonna -
other cultural venues. Engenides - The Typist- Marie- Madame
(c) Disneyland is an artefact that so obviously Sosostris- The ruinbibber- Tiresias- the
announces its own fictiveness that it would Youngman Carbuncular
seem to imply some counter balancing (a) Belladonna (b) Madame Sosostris
reality. (c) Tiresias (d) The ruin - bibber
UGC NET/JRF English Paper III, June 2012 85 YCT
Ans: (d) The ruin-bibber is any person who is a regular 47. Which among the following is NOT a typical
drinker of alcoholic beverages. It appears in P. Larkin’s "Indian English Poem" by Nissim Ezekiel?
'Church Going'. (a) "How the English Lessons Ended"
Hence option (d). (b) "The Railway Clerk"
44. Wilkie Collins's novel, The Moonstone (1868) (c) "Goodbye Party for Miss Pushpa T.S."
tells the story of_____ (d) "The Patriot"
(a) a detective's exploits in Victorian England. Ans: (a) Ezekiel’s ‘How the English lessons ended’ is
(b) a doctor's adventures in a Middle-Eastern narrated in a simple tone.
Suburb. Hence option (a).
(c) a fabulous yellow diamond stolen from an 48. Match the correct pair :
Indian shrine. (I) George Eliot 1. Ellis Bell
(d) illegal mining of diamonds in eastern U.P (II) Saki 2. Mary Anne
during British rule. Evans
Ans: (c) Collin’s ‘The Moonstone’ (1868) is the story (III) Emily Bronte 3. Samuel
of a fabulous yellow diamond stolen from an Indian Langhorne
shrine. Clemens
Hence option (c). (IV) Mark Twain 4. H. H. Munro
45. Identify the correctly matched group : (I) (II) (III) (IV)
(I) "Because I could (a) Walt Whitman (a) 2 3 1 4
not stop for death... (b) 2 4 1 3
(II) "O Captain! My (b) William Carlos (c) 1 3 4 2
Captain!" Williams (d) 3 2 1 4
(III) "Two roads (c) Emily Ans: (b) The correct pairs are –
diverged in a Dickinson I. George Eliot – 2. Mary Anne Evans
wood..." II. Saki – 4. H.H. Munro
(IV) "So much depends (d) Robert Frost III. Emily Bronte – 1. Ellis Bell
upon..." IV. Mark Twain – 3. Samuel Langhorne
(I) (II) (III) (IV) Clemens
(a) (a) (b) (c) (d) Hence option (b).
(b) (c) (a) (d) (b) 49. In Canto 17 of the Inferno, the monster Geryon
(c) (a) (c) (b) (d) represents_____
(d) (c) (a) (b) (d) (a) fraud (b) usury
Ans: (b) The correct pairs are – (c) sloth (d) gluttony
I. ‘Because I could not – (c) Emily Dickinson Ans: (a) In Canto 17 of the Inferno from Dante’s 'The
Stop for death....’ Divine Comedy', the monster Geryon represents fraud.
II. ‘O captain! My captain!– (a) Walt Whitman Hence option (a).
III. ‘Two roads diverged’ – (d) Robert frost 50. I. A. Richards's famous experiment with poems
in a wood’ and his Cambridge students is detailed in
IV. ‘So much depends – (b) William Carlos Practical Criticism: A Study of Literary
upon.....’ Williams Judgement (1929). Richards was astonished by-
Hence option (b). (a) the poor quality of his students’ "stock
46. "Now stop your noses, readers, all and some, responses"
For here's a tun of midnight-work to come, (b) the very astute remarks made by his students
Og, from a treason-tavern rolling home. (c) the non-availability of poems, worthy of
class-room attention
Round as a globe and liquor'd e'vry chink,
(d) the success of his experiment
Goodly and great he rails behind his link".
In the above passage from Absalom and Ans: (a) Richards was astonished by the poor quality
Achitophel, 'link' means : of his Cambridge student’s ‘stock responses.’
(a) a connection in the court Hence option (a).
(b) a hired servant who carries a lighted torch 51. Based on the following description, identify the
(c) a social tie text in reference:
(d) a rich patron This is a play in which no one comes, no one
goes, nothing happens. In its opening scene a
Ans: (b) 'Link' in Dryden’s 'Absalom and Achitophel'
man struggles hard to remove his boot. The
(1681) refers to a hired servant who carries a lighted play was originally written in French, later
torch. translated into English. It was first performed
Hence option (b). in 1953.
UGC NET/JRF English Paper III, June 2012 86 YCT
(a) Look Back in Anger (b) Waiting for Godot 56. Which of the following is NOT TRUE of Ralph
(c) The Zoo Story (d) The Birthday Party Waldo Emerson?
Ans: (b) The above description is of Samuel Beckett’s (a) He wrote essays on New England scenery,
‘Waiting for Godot’ (1953). woodcraft and plantations.
Hence option (b). (b) He was an eloquent pulpit orator, a member
52. One of the following Canterbury Tales is in of the Unitarian Church under William
prose, identify. Chawming.
(a) The Pardoner's Tale (b) The Parson's Tale (c) In essays like "Nature", he elaborates on the
(c) The Monk's Tale (d) The Knight's Tale importance of seeing familiar things in new
Ans: (b) In Chaucer’s ‘Canterbury Tales’, the Parson’s ways.
Tale and Tale of Melibeus are the two tales in prose. (d) His famous "American Scholar" was
Hence option (b). delivered as an address before the Phi Beta
53. In his distinction between imagination and Kappa Society at Cambridge in 1837.
fancy, Coleridge identifies the following : Ans: (a) The false statement about Ralph Waldo
(A) it dissolves, diffuses, dissipates, in order to Emerson is that he wrote essays on New England
recreate. scenery, woodcraft and plantations. He was the member
(B) it has aggregative and associative power. of Transcendental club.
(C) it plays with fixities and definites. Hence option (a)
(D) it has shaping and modifying power. 57. "Exorcism" is the title of Act III of Who's
The correct combination reads : Afraid of Virginia Woolf? What is the
(a) (A) and (B) for fancy; (C) and (D) for significance of 'exorcism' in the context of the
imagination. play?
(b) (A) and (C) for fancy; (B) and (D) for (a) The casting out of evil spirits
imagination. (b) Deconstructing of myths involving marriage,
(c) (B) and (C) for fancy; (A) and (D) for fertility and sons
imagination. (c) Facing life without illusions
(d) (C) and (D) for fancy; (A) and (B) for (d) Exposing all attempts at illusion-making
imagination.
Ans: (d) Exorcism, in Edward Albee’s ‘Who’s Afraid
Ans: (c) Coleridge differentiated between Imagination of Virginia Woolf’ (1962), signifies exposing all
and Fancy by postulating that ‘fancy’ has aggregative
attempts at illusion making.
and associative power and plays with fixities and
definites; ‘imagination’ dissolves, diffuses, dissipates in Hence option (d).
order to recreate and has shaping and modifying power. 58. "Womanist is to feminist as purple is to
Hence option (c). lavender". This is an important statement
54. Julia Kristeva's 'Intertextuality' derives from: defining the womanist perspective advanced by
(A) Saussure's deep structure (a) Toni Morison (b) Zora Neale Hurston
(B) Chomsky's deep structure (c) Alice Walker (d) Bell Hooks
(C) Bakhtin's dialogism Ans: (c) Alice Walker gave the womanist perspective
(D) Derrida's difference as "Womanist is to feminist as purple is to lavender".
(a) (A) and (D) (b) (A) and (C) Alice Walker is famed to have coined the term
(c) (C) and (D) (d) (A) and (B) ‘womanism’.
Ans: (b) Julia Kristeva’s ‘Intertextuality’ derives from Hence option (c).
(A) Saussure’s signs and (C) Bakhtin’s dialogism. 59. Identify the mismatched pair in the following
Hence option (b). where characters in Golding's Lord of the
55. Ralph Ellison enjoys subverting myths about Flies' fit the allegorized pattern of virtues and
white purity through characters like : vices.
(A) Norton (B) Bledsoe (a) Ralph - rationality
(C) Rhinehart (D) all of the above (b) Piggy - pragmatism
(a) (A) and (B) (b) (A), (B) and (C) (c) Jack - pity
(c) (B) and (C) (d) (A) and (C) (d) Simon - innocence
Ans: (a) Ralph Ellison subverts myths about white Ans: (c) Jack, in Golding’s ‘Lord of the Flies’ (1954),
purity through characters like Norton and Bledsoe in his symbolizes evil and violence, the dark side of human
novel ‘The Invisible Man’ (1952). nature, which is the opposite of pity.
Hence option (a). Hence option (c).
UGC NET/JRF English Paper III, June 2012 87 YCT
60. A Subaltern perspective is one where 63. Identify the correctly matched pair of
(a) power-structures define and determine your translators and translations.
command of language and language of (I) A. K. (a) The Ramayana
command in an uneven world. Ramanujan
(b) The politically dispossessed could be (II) Manmathanath (b) The Bhagavad
voiceless, written out of the historical record Dutt Gita
and ignored because their activities do not (III) Mohini Chatterjee (c) Speaking of
count for "Cultural" or "Structured". Shiva
(c) Yor don't know what your 'story' is, how to (IV) Romesh Chandra (d) Mahabharata
deal with a 'story' and therefore you are Dutt
forced to put stereotyped situations in it to (I) (II) (III) (IV)
please your listeners. (a) (c) (d) (b) (a)
(d) You begin to see how we live, how we have (b) (d) (c) (a) (b)
been living, how we have been led to imagine (c) (d) (a) (b) (c)
ourselves, how our language has trapped as (d) (b) (a) (d) (c)
well as liberated us. Ans: (a) The correct pairs are –
Ans: (b) A subaltern perspective is the oppressed I. A.K. Ramanujan – (c) ‘Speaking of
viewpoint that speaks of the politically dispossessed Shiva’ (1973)
who could be voiceless, written out of the historical II. Manmathanath Dutt – (d) ‘The Mahabharata'
record and ignored because their activities do not count (1905)
for ‘cultured’ or ‘structured.’ III. Mohini Chatterjee – (b) ‘The Bhagavad
Gita’ (1887)
Hence option (b).
IV. Romesh Chandra Dutt – (a) ‘The Ramayana’
61. (1899)
(A) "Interlanguage" is a term we owe to Hence option (a).
M.A.K. Halliday. 64. Assertion (A) : In The Power and the Glory.
(B) Interlanguage develops an autonomous and Greene shows how the Whisky Priest transcends
self-contained grammatical system. his weakness for drink and his human fears.
(C) It is a distinct stage in a learner's progress in moving towards martyrdom.
the study of a second langrage. Reason (R) : Transcendence in Greene's novels is
(D) It owes nothing at all either to the learner's generally an outcome of love for humanity, but
native or target / second language. pride is also an essential ingredient in the Priest's
character.
(a) (D) is correct.
(a) (A) is true, but (R) is false.
(b) (B) is correct.
(b) (A) is false, but (R) is true.
(c) (A) and (C) are correct. (c) Both (A) and (R) are true, but (R) is not the
(d) (C) and (D) are correct. correct explanation for (A).
Ans: (c) The correct statements are – (d) Both (A) and (R) are true and (R) is the
(A) ‘Interlanguage’ is a term we owe to M.A.K. correct explanation for (A).
Halliday. Ans: (c) Both (A) and (R) are true but (R) does not
(C) It is a distinct stage in a learner’s progress in the correctly explain (A).
study of a second language. Hence option (c).
Hence option (c). 65. Which of the following statements of John
62. In a classic statement that inaugurated Dryden is incorrect?
Feminist thought in English, we read : (A) John Milton and John Dryden were
contemporaries.
"A woman writing thinks back through her
(B) Dryden was a Royalist. while Milton fiercely
mothers" Where does this occur? opposed monarchy.
(a) Virginia Woolf's A Room of One's Own (C) Dryden wrote a play on the Mughal Emperor
(b) Kate Millet's Sexual Politics Humayun.
(c) Gertrude Stein's Three Lives (D) Dryden was appointed the Poet Laureate of
(d) Mary Hiatt's The Way Women Write. England in 1668.
Ans: (a) V. Woolf’s 'A Room of One’s Own' (1929) is (a) (a) is incorrect.
considered the inaugural feminist text and the above (b) (d) is incorrect.
line figures in it. (c) (c) is incorrect.
Hence option (a). (d) (b) and (c) are incorrect.
UGC NET/JRF English Paper III, June 2012 88 YCT
Ans: (c) Dryden wrote a Restoration Drama ‘Aurang- (b) Third Space - Ernst Bloch
zebe’ in 1675 which was published in 1690. Dryden did Hybridity - Edward Said
not write a play on Humayun. Reception - Eve K.
Hence option (c). aesthetics - Sedgwick
66. "Like walking, criticism is a pretty nearly Langue - G. S. Frazer
universal art; both require a constant intricate (c) Third Space - Edward Soja
shifting and catching of balance; neither can be
Hybridity - Homi Bhabha
questioned much in process; and few perform
either really well. For either a new terrain is Reception - Wolfgang Iser
fatiguing and awkward. and in our day most aesthetics -
men prefer paved walks and some form of Langue - Ferdinand de
rapid transport- some easy theory or Saussure
overmastering dogma." (R.P. Blackmur, "A (d) Third Space - G. S. Frazer
Critic's Job of Work") Hybridity - Eve K.
(A) Blackmur compares walking with criticism
Sedgwick
because he considers both to be "arts" of a
similar kind that call for attention to detail Reception - Edward Soja
and utmost care. aesthetics -
(B) Blackmur admits that some people do Langue - Edward Said
however manage to be good critics and good Ans: (c) The correct pairs are –
walkers. Third space – Edward Soja
(C) Critics prefer tried and tested approaches for Hybridity – Homi Bhabha
much the same reason as Walkers would look
Reception Aesthetics – Wolfgang Iser
for paved walks and rapid transport.
(D) Blackmur does not quite give us the Langue – Ferdinand de Saussure
equivalents of "Some paved walks and some Hence option (c).
form of rapid transport" in order to press his 69. Which of the following can be best described
comparison. as: (i) the first statement of Bernard Shaw's
(a) (A) and (D) are correct. idea of Life Force; (ii) a play dealing with a
(b) (A) and (C) are correct. woman's pursuit of her mate; and (iii) a play
(c) only (D) is correct. whose third act called "Don Juan in Hell" is
(d) only (B) is correct. both unconventional and hilarious?
Ans: (b) The correct statements about R.P. Blackmur’s (a) The Devil's Disciple
‘A Critic’s Job of Work’ (1954) are – (b) Man and Superman
(A) Blackmur compares walking with criticism because (c) Candida
he considers both to be ‘arts’ of a similar kind that call (d) Arms and the Man
for attention to detail and utmost care.
Ans: (b) The above description is of Shaw’s 1903 play
(C) Critics prefer tried and tested approaches for much
the same reason as walkers would look for paved walks ‘Man and Superman’.
and rapid transport. Hence option (b).
Hence option (b). 70. Identify the untrue statement of the
67. The world dominated by cold and hypocritical CONTACT ZONE below :
materialists is represented by William Blake in (a) "The contact zone" is a space where disparate
the mythological figure of_____ cultures meet, clash and grapple with each
(a) Urizen (b) Albion other.
(c) Geryon (d) Satan (b) In Postcolonial societies "contact" suggests
Ans: (a) Urizen in Blake’s mythological world the historical moment when settler and
represents cold and hypocritical materialists. indigenous cultures first met.
Hence option (a). (c) The idea of the Contact Zone was first
68. Identify the correctly matched group: proposed and defined by Mary Louise Pratt's
(a) Third Space - Wolfgang Iser Imperial Eyes:
Hybridity - Edward Soja Travel Writing and Transculturation (1992)
Reception - Ferdinand de (d) It is believed that the Contact Zone was
aesthetics - Saussure largely instrumental in spearheading
Langue - Homi Bhabha nationalist movements across the world.
UGC NET/JRF English Paper III, June 2012 89 YCT
Ans: (d) The statements (a), (b), and (c) are correct and Ans: (d) The correct matches are –
(d) is incorrect. I. Tennessee Williams – 2. 'A Streetcar Named
Contact zone marks the beginning of colonialism Desire' (1947)
whereas nationalist movements come at the last or II. Eugene O ‘Neill – 1. 'Emperor Jones'
(1920)
saturation state of oppression in post-colonial societies.
III. Lorraine Hansberry – 4. 'A Raisin in the
Hence option (d). Sun' (1959)
71. Name the novel in which IV. Arthur Miller – 3. 'After the Fall'
I. the protagonist is a war veteran called Tayo. (1964)
II. Tayo returns from World War II, thoroughly Hence option (d).
disillusioned and haunted by his violent 74. Match the correct pair :
actions of war time. I. Theatre of 1. Safdar
III. Tayo seeks consolation and counsel from old Cruelty Hashmi
Betonie. II. Theatre of the 2. Georg Kaiser
IV. The protagonist realizes the importance of Oppressed
harmonizing humanity and the universe. III. Expressionist 3. Jerzy
Theatre Grotowsky
(a) Beloved (b) Ceremony
IV. Agitprop 4. Augusto Boal
(c) Daisy Miller (d) Enter, Conversing I II III IV
Ans: (b) The above description is of Native American (a) 1 2 4 3
writer Leslie Marmon Silko’s novel ‘Ceremony’ (b) 3 4 2 1
published in 1977. (c) 2 3 1 4
Hence option (b). (d) 4 1 3 2
72. One of the following poems in Men and Women Ans: (b) The correct pairs are –
is addressed to Elizabeth Barrett Browning by Name of Theatre Founder of term
the poet. Identify 1. Theatre of cruelty Antonin Artaud
(a) "In Three Days" 2. Theatre of the oppressed Auqusto Boal
(b) " By the Fireside" 3. Expressionist Theatre Frank Wedekind
(c) "One Way of Love" 4. Agitprop Safdar Hashmi
(d) "One Word More" Note– There is no proper harmony between the given
Ans: (d) ‘Men and Women’ is a collection of 51 poems question and required answer. Therefore, NTA has
dropped this question and marks have been given to
in two volumes first published in 1855 by Robert
all the appeared candidates.
Browning. The first fifty poems take in a diverse range
75. Bertolt Brecht's Epic Theatre-
of historical religious or European situations and the
(A) turns the spectator into an observer
fifty-first ‘One Word More’ is dedicated to his wife
(B) wears down the spectator's capacity for
Elizabeth Barrett Browning. action
Hence option (d). (C) relies on argument
73. Match List-I with List-II according to the (D) presents man as a process
codes given below : (a) (A) and (D) are correct; (B) and (C) are
List-I List-II incorrect.
I. Tennessee 1. Emperor (b) (A), (C) and (D) are correct; (B) is wrong.
(c) (B) and (D) are correct; (A) and (C) are
Williams Jones incorrect.
II. Eugene O' Neill 2. A Streetcar (d) (A), (B) and (C) are correct; (D) is incorrect.
Named Desire Ans: (b) Bertolt Brecht’s Epic Theatre turns the
III. Lorraine 3. After the Fall spectator into an observer, relies on argument and
Hansberry presents man as a process. It does not wear down the
IV. Arthur Miller 4. A Raisin in the spectator’s capacity for action, rather encourages it.
Sun Acting in Epic Theatre requires actors to play characters
believably without convincing either the audience or
I II III IV themselves that they have ‘become’ the characters.
(a) 3 1 4 2 Actors address the audience directly and play multiple
(b) 1 3 2 4 roles in trying to develop a style of acting where
(c) 4 2 3 1 characters were choosing one action over another.
(d) 2 1 4 3 Hence option (b).

UGC NET/JRF English Paper III, June 2012 90 YCT


UGC NET/JRF Exam, June-2012
ENGLISH
Solved Paper-II
Note : This paper contains fifty (50) objective Ans: (a) The Irish Dramatic Movement was heralded by
type questions, each question carrying two (2) marks. such figures as – W.B. Yeats, Lady Gregory and
Attempt all the questions. Edward Martyn.
1. To refer to the unresolvable difficulties a text Hence option (a).
may open up, Derrida makes use of the term:
(a) aporia (b) difference 6. Which poem by Chaucer was written on the
(c) erasure (d) supplement death of Blanche, Wife of John of Gaunt?
(a) Troilus and Criseyde
Ans: (a) Derrida used the term ‘aporia’ to refer to the
unresolvable difficulties in a text. (b) The House of Fame
Hence option (a). (c) The Book of Duchess
2. Who among the following English playwrights, (d) The Legend of Good Women
scripted the film Shakespeare in Love? Ans: (c) Chaucer’s ‘The Book of the Duchess’ was
(a) Harold Pinter (b) Alan Bennett written on the death of John of Gaunt’s wife Blanche.
(c) Caryl Churchill (d) Tom Stoppard Hence option (c).
Ans: (d) The film ‘Shakespeare in love’ (1998) was 7. The Tragedy of Ferrex and Porrex is the other
directed by John Madden and scripted by Morc Norman title of
and playwright Tom Stoppard. (a) Gorboduc
Hence option (d). (b) Ralph Roister Doister
3. Arrange the following in the chronological (c) Damon and Pythias
order: (d) Lamentable Tragedy
1. Mary Wollstonecraft's Vindication of the
Rights of Women Ans: (a) ‘The Tragedy of Ferrex and Porrex’ is the
2. Lyrical Ballads subtitle of Sackville and Norton’s ‘Gorboduc’ (1561).
3. French Revolution Hence option (a).
4. Percy's Reliques of Ancient English Poetry 8. Who of the following poets is Australian?
(a) 4, 3, 1, 2 (b) 3, 2, 1, 2 (a) Austin Clarke (b) Judith Wright
(c) 1, 2, 4, 3 (d) 2, 1, 3, 4 (c) Edwin Muir (d) Derek Walcott
Ans: (a) The correct chronological order is – Ans: (b) Judith Wright is of Australian origin,
4. Percy’s 'Reliques of Ancient English Poetry' – 1765 Austin Clarke – Irish,
3. 'French Revolution' – 1789 Edwin Muir - Scottish (Orkney),
1. Mary Wollstonecraft’s 'Vindication of the Rights of Derek Walcott – Caribbean (St. Lucia).
Women' – 1792 Hence option (b).
2. 'Lyrical Ballads' – 1798
Hence option (a). 9. "He found it [English] brick and left it
marble", remarked one great writer on
4. Which ot the following employs a narrative another. Who were they?
structure in which the main action is relayed at
second hand through an enclosing frame story? (a) Milton on Shakespeare
(a) Sons and Lovers (b) Dryden on Shakespeare
(b) Ulysses (c) Johnson on Dryden
(c) The Power and the Glory (d) Jonson on Shakespeare
(d) Heart of Darkness Ans: (c) Samuel Johnson remarked on Dryden that “he
Ans: (d) In Joseph Conrad’s ‘Heart of Darkness’ (1902) found it (English) brick and left it marble.”
the main action is relayed through an enclosing frame Hence option (c).
story. 10. Who, among the following, is a Poet Laureate?
Hence option (d). (a) Tony Morrison (b) Seamus Heaney
5. The Irish Dramatic movement was heralded by (c) Ted Hughes (d) Geoffrey Hill
such figures as
Ans: (a & b) Ted Hughes was appointed Poet Laureate
(a) W. B. Yeats, Lady Gregory and Edward
Martyn in 1984 and held the office until his death. He was an
English poet, translator and children's writer.
(b) Jonathan Swift and his contemporaries
(c) H. Drummond, Edward Irving and John Hence, option (a & b) is correct.
Ervine Note– UGC has accepted option (a & b), but the right
(d) Oscar Wilde and his contemporaries answer will be Ted Hughes.

UGC NET/JRF English Paper II, June 2012 91 YCT


11. List-I List-II Ans: (a) Mrs. Malaprop appears in R.B. Sheridan’s
I. "Because I could a. Robert Frost ‘The Rivals’ (1775).
not stop for death..." Hence option (a).
II. "O Captain! b. William Carlos 16. Which of the following statements about
My Captain!" Williams Christopher Marlowe are true?
III. "Two roads diverged c. Emily Dickinson I. Edward II was written in the last year of
in a wood....." Marlowe's life
IV. "So much d. Walt Whitman II. Many critics consider Doctor Faustus to be
depends/upon" Marlowe's best play.
The correctly matched series would be: III. His Spanish Tragedy comes a close second.
(a) I-d, II-c, III-b, IV-a IV. Marlowe was less educated than
(b) I-a, II-b, III-c, IV-d Shakespeare.
(c) I-b, II-a, III-d, IV-c (a) I and II are true. (b) II and III are true.
(d) I-c, II-d, III-a, IV-b (c) II and IV are true. (d) III and IV are true.
Ans: (d) The correct matches are – Ans: (a) Marlowe was a University Wit who was killed
I. “Because I could not – c. Emily Dickinson in a tavern brawl in 1593, the year in which ‘Edward II’
stop for death.” was written and published next year. ‘Doctor Faustus’
II. “O Captain! – d. Walt Whitman (pub. 1604) is considered his best play. The ‘Spanish
My Captain!” Tragedy’ was authored by Thomas Kyd.
III. “Two roads diverged – a. Robert Frost Hence option (a).
in a wood.” 17. "Art for Art's Sake" became a rallying cry for:
IV. “ So much depends – b. William Carlos (a) the Aesthetes
upon” Williams. (b) the Symbolists
Hence option. (d) (c) the Imagists
12. The predominant tone and thrust of Jonathan (d) the Art Noveau School
Swift's "A Modest Proposal" are
Ans: (a) ‘Art for Art’s sake’ became a rallying cry for
(a) comic (b) solemn
The Aesthetes in 1880s.
(c) hortatory (d) irony
Hence option (a).
Ans: (d) The predominant tone and thrust of J. Swift’s
‘A Modest Proposal’ (1729) is irony. 18. Confessions of an English Opium Eater is a
Hence option (d). literary work by:
(a) S.T. Coleridge (b) P. B. Shelley
13. I sit in one of the dives
(c) Thomas De Quincey (d) Lord Byron
On Fifty Second Street,
Uncertain and afraid Ans: (c) Thomas De Quincey wrote 'Confessions of an
As the clever hopes expire English Opium Eater' in 1822.
Of a low dishonest decade. Hence option (c).
So begins Auden's "September 1, 1939". 19. Which of the following statements about The
What is the meaning of the "word in italics? Canterbury Tales is true?
(a) bench (b) night club (a) "The General Prologue' is appended to The
(c) house (d) park Canterbury Tales.
Ans: (b) In the above extract from Auden’s ‘September (b) In all, Chaucer tells thirty tales in this work.
1, 1939’ the word ‘dives’ refers to a night club. (c) The Canterbury Tales remained unfinished at
Hence option (b). the time of its author's death.
14. C. K. Ogden and I. A. Richards ware reputed (d) The Wife of Bath, The Clerk, Sir Gawain and
in the 1930s for introducing The Franklin are characters and tale-tellers in
(a) Practical Criticism this work.
(b) New Criticism Ans: (c) The true fact from the above statements is that
(c) Standard English Project ‘The Canterbury Tales’ remained unfinished at the time
(d) Basic English Project of its author’s death.
Ans: (d) C.K. Ogden and I.A. Richards were reputed in Hence option (c).
the 1930s for introducing – Basic English project. 20. Who, among the following was a Catholic
Hence option (d). novelist, an Intelligence Officer, a film critic
15. In which of the following works does Mrs. and set his fictions in far-away places wrecked
Malaprop appear? by political conflicts?
(a) The Rivals (a) Anthony Powell (b) Evelyn Waugh
(b) She Stoops to Conquer (c) William Golding (d) Graham Greene
(c) The Mysteries of Udolpho Ans: (d) The above description is of Graham Greene.
(d) The Way of the World Hence option (d).

UGC NET/JRF English Paper II, June 2012 92 YCT


21. List-I List-II Ans : (c) The novels of Dickens and Scott were
1. Good sense is the I. Brooks, The illustrated by George Cruikshank.
body of poetic genius Formalist Critic Hence option (c).
2. Poetry is the breath II. Sidney, Defence/ 25. The last of Gulliver's Travels is to :
and a finer spirit of An Apology (a) The Land of the Houyhnhnms
all knowledge. for Poetry
(b) The Land of Homosapiens
3. Literary criticism III. Wordsworth,
description and Preface (c) The Land of the Hurricanes
evaluation of its object (d) The Newfound Land
4. Nature never set IV. Coleridge, Ans : (a) The last journey of Gulliver in Swift’s
forth the earth in as Biographia 'Gulliver’s Travels' (1726) is to the land of the
rich a tapestry as Literaria Houyhnhnms.
diverse poets have Hence option (a).
done 26. Madam Merle is a character in
1 2 3 4 (a) The Great Gatsby
(a) IV III I II (b) The Portrait of a Lady
(b) II IV III I (c) The Jungle
(c) III II I IV (d) The Heart is a Lonely Hunter
(d) IV II I III Ans: (b) Madam Merle is a character in Henry James'
Ans: (a) The correct matches are – 'The Portrait of a Lady' (1881).
1. Good sense is the – IV. Coleridge, Hence option (b).
body of poetic genius "Biographia
27. In which of the following scenes of The Waste
Literaria" Land do we have a departure from Standard
2. Poetry is the breath – III. Wordsworth English?
and a finer spirit 'Preface' (a) The typist scene
of all
(b) The pub scene
3. Literary criticism is the – I. Brooks, "The
(c) The hyacinth garden scene
description and Formalist Critic"
(d) The Chapel Perilous scene
evaluation of its object
4. Nature never set forth – II. Sidney, "Defence/ Ans: (b) The departure from Standard English is
the earth in as rich a An Apology" observed in the pub scene of ‘The Waste Land’ (1992)
tapestry as diverse by T.S. Eliot.
poets have done Hence option (b).
Hence option (a). 28. The words "If it were done when tis done, then
22. In which of the following travel books does twere well/ It were done quickly..." are uttered
Mark Twain give an account of his visit to by
India? (a) Hamlet (b) Lear
(a) A Tramp Abroad (c) Othello (d) Macbeth
(b) Roughing It Ans: (d) The above is a part of one of the famous
(c) The Innocents Abroad soliloquies from Shakespeare’s play ‘Macbeth’, spoken
(d) Following the Equator by Macbeth.
Ans: (d) Mark Twain gave an account of his visit to Hence option (d).
India in his book ‘Following the Equator’ (1897). 29. John Dryden's Absalom and Achitophel is a :
Hence option (d). (a) religious tract (b) political allegory
23. William Blake's famous poems such as (c) comic verse epic (d) comedy
"London", "The Sick Rose", and "The Tyger"
appear in Ans: (b) John Dryden’s 'Absalom and Achitophel'
(1681) is a political allegory. It satirises the then
(a) Songs of Innocence
political figures like Shaftesbury (as Achitophel) and
(b) Songs of Experience
Duke of Monmouth (as Absalom).
(c) The Marriage of Heaven and Hell
Hence option (b).
(d) Vision of the Daughters of Albion
Ans: (b) W. Blake’s above mentioned poems appear in 30. The term 'the comedy of menace' is associated
the collection ‘Songs of Experience’. with the early plays of
Hence option (b). (a) Arnold Wesker (b) John Arden
24. Who among the following English artists (c) Harold Pinter (d) David Hare
illustrated the novels of Dickens and Scott ? Ans: (c) Harold Pinter’s early plays like ‘The Birthday
(a) Richard Hogarth (b) Joshua Reynolds Party’ (1958) are categorised as the comedy of menace.
(c) George Cruikshank (d) John Tennial Hence option (c).

UGC NET/JRF English Paper II, June 2012 93 YCT


31. Examine the following statements and identify I. Let us go then, a. Moby dick
one of them which is not true. you and I.....
(a) Rudyard Kipling died in the year 1936. II. Call me b. Macbeth
(b) He was born in India but schooled in Ishmael.....
England. III. When shall we c. The Love Song
(c) He returned to India as a police constable in three meet again? of J. Alfred
Burma. Prufrock
(d) He is the author of Jungle Book and Barrack IV. He disappeared d. Tristram Shandy
Room Ballads.
in the dead of winter
Ans: (c) The incorrect statement about Kipling is (c)
V. I wish either.... e. In Memory of
because he worked as a journalist in India and not as a
constable in Burma. begot me.... of W. B. Yeats
Hence option (c). (a) I-c; II-a; III-b; IV-e;V-d
32. What is the correct combination of the (b) I-e; II-b; III-a; IV-c;V-d
following? (c) I-b; II-a; III-d; IV-e;V-c
I. Balachandra a. The Tamarind (d) I-b; II-e; III-d; IV-c;V-a
Rajan Tree Ans: (a) The correct matches are –
II. R. K. Narayan b. The Coffer I. "Let us go then, you – (c) T.S. Eliot’s 'The
Dams and I" Love Song of J.
III. Kamala c. The Dark Dancer Alfred Prufrock'
Markandaya (1915)
IV. Romen Basu d. The Dark Room II. “Call me Ishmael ..” – (a) H. Melville’s
(a) I-c; II-d; III-b; IV-a 'Moby Dick' (1851)
(b) I-d; II-a; III-b; IV-c III. "When shall we – (b) Shakespeare’s
(c) I-c; II-a; III-d; IV-b three meet again?" ‘Macbeth’ (1606)
(d) I-d; II-c; III-a; IV-b IV. "He disappeared in – (e) W.H. Auden’s
(d) The Newfound Land the dead of winter" 'In memory of
Ans: (*) Note– Marks was awarded to all the W.B. Yeats' (1939)
appeared candidates. V. "I wish either . . . – (d) L. Sterne’s
Author Work Year
begot me . . ." 'Tristram Shandy'
I. Balachandra C. The Dark 1958
(1759-67)
Rajan Dancer
Hence option (a).
II. R.K. Narayan D. The Dark Room 1938
36. Which of the following is not a revenge
III. Kamala B. The Coffer Dams 1969 tragedy?
Markandaya (a) Hamlet (b) The Duchess of Malfi
IV. Romen A. The Tamarind 1975 (c) Volpone (d) Gorboduc
Basu Tree
Ans: (c) ‘Volpone’ was a comedy by Ben Jonson
33. Name the poet who chooses his successor and printed in 1607.
the successor-poet whom Dryden satirises in
his famous poem. Hence option (c).
(a) James Shirley and Chris Shirley 37. What is a neologism ?
(b) Henry Treece and Charles Triesten (a) A word with roots in a native language
(c) Richard Flecknoe and Thomas Shadwell (b) A word whose meaning changes with every
(d) Thomas Percy and Samuel Pepys renewed use
Ans: (c) In his ‘Absalom and Achitophel’ (1681) (c) A word newly coined or used in a new sense
Dryden satirises Richard Flecknoe and Thomas (d) An obsession with new words and phrases
Shadwell. Ans: (c) Neologism is a word newly coined or used in a
Hence option (c). new sense.
34. "If______ comes, can _____ be far behind ?" Hence option (c).
(Shelley, "Ode to the West Wind") 38. Which of the following is not true of Edward
(a) winter, spring (b) autumn, summer Said's Orientalism?
(c) wind, rains (d) spring, winter (a) Makes use of Foucault's concept of
Ans: (a) In his poem ‘Ode to the West Wind’ Shelley discursive formulation
says – “If winter comes, can spring be far behind.” (b) Is one of the founding texts of Postcolonial
Hence option (a). theory
35. The following passages are the very first lines (c) Makes use of Barthes' concept of writerly
of well-known works. Match the lines and the text
works: (d) Utilizes the Gramscian notion of hegemony
UGC NET/JRF English Paper II, June 2012 94 YCT
Ans: (c) Said does not use Barthes’ concept of writerly 45. Entries in The Diary of Samuel Pepys begins
text in ‘Orientalism: Western Conceptions of the after:
Orient’ (1978). (a) The Restoration
Hence option (c). (b) The Glorious Revolution
39. Thomas Love Peacock classified poetry into 4 (c) The Reformation
periods. They are : (d) The French Revolution
(a) carbon, gold, silver and brass Ans: (a) 'The Diary of Samuel Pepys' begins in the
(b) brass, silver, gold and diamond Restoration year 1660 and continues till 1669.
(c) iron, gold, silver and brass Hence option (a).
(d) gold, platinum, silver and diamond 46. In a poem, a line may either be end stopped or:
Ans: (c) Peacock classified poetry into– iron, gold, (a) rhymed (b) broken
silver and brass periods. (c) accented (d) run-on
Hence option (c). Ans: (d) In a poem a line can be end stopped or run-on.
40. Which among the following novels has more Hence option (d).
than one ending? 47. Which of the following poets wrote the essay
(a) Lucky Jim "Naipaul's India and Mine"?
(b) The Prime of Jean Brodie (a) Kamala Das (b) R. Parthasarthy
(c) The French Lieutenant's Woman (c) A.K. Ramanujan (d) Nissim Ezekiel
(d) The Clockwork Orange Ans: (d) Nissim Ezekiel wrote ‘Naipaul’s India and
Ans: (c) John Fowle’s ‘The French Lieutenants’s Mine’ (1965), an essay to criticise Naipaul’s view of
Woman’ (1969) has several endings. India in ‘An Area of Darkness’ (1964).
Hence option (c). Hence option (d).
41. "You have seen how a man was made a slave; 48. Match the following:
you shall see how a slave was made a man" is I. James Joyce 1. Peter Ackroyd
an example of II. T. S. Eliot 2. James Boswell
(a) Bathos (b) Apostrophe III. Life of Johnson 3. Samuel Johnson
(c) Chiasmus (d) Anti-climax IV. Lives of poets 4. Richard Ellman
Ans: (c) The above statement is an example of criss- (a) I-3; II-4; III-1; IV-2
cross or Chiasmus. (b) I-4; II-1; III-2; IV-3
Hence option (c). (c) I-1; II-2; III-3; IV-4
42. Which of the following statements is NOT (d) I-2; II-3; III-1; IV-4
correct? Ans: (b) The correct matches are –
(a) Chaucer used the rhyme royal, a stanzaic I. ‘James Joyce’ – 4. Richard Ellman
form in some of his major poems. II. ‘T.S. Eliot’ – 1. Peter Ackroyd
(b) Chaucer was the author of The Legend of III. ‘Life of Johnson’ – 2. James Boswell
Good Women. IV. ‘Lives of Poets’ – 3. Samuel Johnson.
(c) Chaucer wrote in English when the court Hence option (b).
poetry of his day was written in Anglo- 49. "The pen is mightier than the sword" is an
Norman and Latin. example of :
(d) Chaucer wrote The Book Named the (a) simile (b) image
Governor
(c) conceit (d) metonymy
Ans: (d) Thomas Elyot is the author of ‘The Book
Ans: (d) The above statement is an example of
named the Governour’ (1531) and not Chaucer. metonymy. In this figure of speech a thing or concept is
Hence option (d). not called by its own name but by the name of
43. Material feminism studies inequality in terms something closely associated with that thing or concept.
of Hence option (d).
(a) only gender (b) only class 50. An epilogue is :
(c) both class and gender (d) only patriarchy (a) prefixed to a text which it introduces.
Ans: (c) Material feminism studies inequality in terms (b) suffixed to a text which it sums up or
of both class and gender. extends.
Hence option (c). (c) a piece of writing or speech that formally
44. Who among the following is not an Irish begins a book
writer? (d) a piece of writing or speech that bears no
(a) Oscar Wilde (b) Oliver Goldsmith relation to the text at hand.
(c) Edmund Burke (d) Thomas Gray Ans: (b) An epilogue is – suffixed to a text which it
Ans: (d) Thomas Gray is an English writer from Eton, sums up or extends at the end of a work. Prologue is in
England. the beginning of a work or prefixed to a text.
Hence option (d). Hence option (b).

UGC NET/JRF English Paper II, June 2012 95 YCT


UGC NET/JRF Exam, December-2012
ENGLISH
Solved Paper-III
Note : This paper contains seventy five (75) Codes :
objective type questions of two (2) marks each. All i ii iii iv v
questions are compulsory. (a) 1 2 3 4 5
1. Which of the following book by V.S. Naipaul is (b) 5 1 2 3 4
subtitled The Caribbean Revisited? (c) 1 3 2 4 5
(a) In a Free State (d) 5 1 2 4 3
(b) A Bend in the River
(c) The Middle Passage Ans: (b ) The following poems of Milton are correctly
matched as follows –
(d) An Area of Darkness
(i) ‘L’ Allegro and Il Penseroso (1645) – 5. Companion
Ans: (c) V.S. Naipaul’s ‘The Middle Passage’ is poems in octo syllabic couplets
subtitled ‘The Caribbean Revisited.’ It is a non-fiction
(ii) 'Lycidas' (1638) – 1. Pastoral Elegy
book length essay which was published in 1962 as
‘Impression of five societies: British, French and Dutch (iii) 'Comus' (1637) – 2. Masque
in West Indies and South America.’ (iv) 'On His Blindness' (1673) – 3. Sonnet
Hence option (c). (v) 'Areopagitica' (1644) – 4. Prose tract
2. 'Fluency' in Language is the same as Hence option (b).
(a) The ability to put oneself across comfortably 5. The Pre-Raphaelite brotherhood - The
in speech and/or writing. University Wits- The Rhymers' Club- The
(b) the ability to command language rather than Scottish Chaucerians.
language commanding the user. The right chronological sequence would be
(c) glibness (a) The Scottish Chaucerians- The University
(d) accuracy Wits- The Transitional Poets- The Pre-
Ans: (a) In language, fluency is same as the ability to Raphaelite brotherhood- The Rhymers' Club.
put oneself across comfortably in speech and/or writing. (b) The Rhymers' Club- The University Wits-
Hence option (a). The Scottish Chaucerians- The Pre-
Raphaelite brotherhood.
3. Which of the following statements on Pathetic
Fallacy is NOT TRUE ? (c) The Pre-Raphaelite brotherhood- The
(a) This term applies to descriptions that are not Rhymers' Club- The Transitional Poets- The
true but imaginary and fanciful. Scottish Chaucerians- The University Wits.
(b) Pathetic Fallacy is generally understood as (d) The University Wits- The Scottish
human traits being applied or attributed to Chaucerians- The Pre- Raphaelite
non-human things in nature. brotherhood- The Transitional Poets- The
(c) In its first use, the term was used with Rhymers' Club.
disapproval because nature cannot be equated Ans: (a) The correct chronological sequence is–
with the human in respect of emotions and 1425-1650 – The Scottish Chaucerians – James I,
responses. Dunbar etc.
(d) The term was originally used by Alexander 1585-1600 – The University Wits – Marlowe, Kyd etc.
Pope in his Pastorals (1709). Mid 18th Cent – The Transitional Poets – Gray, Blake etc.
Ans: (d) The term ‘Pathetic fallacy’ was originally Mid 19th Cent – The Pre-Raphaelite Brotherhood –
used by Pope in his ‘Pastorals’ (1709). It is a literary Hunt, Millais, Rossetti etc.
device in which the author attributes human emotions 1890s – The Rhymer’s Club – Yeats, Rhys etc.
and traits to nature or inanimate objects. Hence option (a).
Hence option (d).
6. 'Aucitya' refers to:
4. Identify the correctly matched group: I. Decorum II. Propriety
List-I List-II III. Proportion IV. Accuracy
i. 'L' Allegro and 1. pastoral elegy (a) I and IV are correct.
Il’ Penseroso’ (b) I and III are correct.
ii. 'Lycidas' 2. masque
(c) II is correct.
iii. ‘Comus’ 3. Sonnet
(d) II and IV are correct.
iv. 'On His Blindness' 4. Prose tract
v. ‘Areopagitica’ 5. Companion poems Ans: (c) ‘Auchitya’ refers to propriety or
in octo-syllabic appropriateness.
couplets Hence option (c).
UGC NET/JRF English Paper III, December 2012 96 YCT
7. In the closing paragraph of The Trial two men 11. Raskolnikov murders the old lady :
accompany Joseph K to a part of the city to I. to get her money and achieve his ambition in
eventually execute him. life
The place is II. to achieve his political goal as an extremist
(a) a Public Park (b) a church and a nihilist
(c) a Quarry (d) an Abandoned Factory III. to prove his superiority over other young men
Ans: (c) The place of murder in the closing para of of the time.
Kafka's novel ‘The Trial’ (1925) is a quarry. IV. All of the above
Hence option (c). Find the correct combination according to the
8. Match List-I with List-II according to the code code :
given below : (a) I and II are correct.
List-I List-II (b) I and III are correct.
(Character) (Work) (c) II and III are correct.
i. Telemachus 1. Notes from (d) I. II and III are correct.
underground
Ans: (b) Raskolnikov, in Dostoevsky’s ‘Crime and
ii. Anya 2. Old Goriot
Punishment’ (1866), murders the old lady to get her
iii. Zverkov 3. The Cherry
money to achieve his ambition in life and to prove his
Orchard superiority over other young men of the time.
iv. Rastignac 4. The Odyssey
Hence option (b).
Codes:
i ii iii iv 12. In his preface to The Order of Things,
(a) 4 1 2 3 Foucault mentions being influenced by a Latin
(b) 3 1 4 2 American writer and his work.
(c) 2 4 1 3 Choose the correct answer:
(d) 4 3 1 2 (a) Marquez - "The Solitude
Ans: (d) The correct matches are – of Latin America"
(i) Telemachus – 4. ‘The Odyssey’ by Homer (b) Borges - "Chinese
(8th cent BC) Encyclopedia"
(ii) Anya – 3. ‘The Cherry orchard’ by (c) Juan Rulfo - Pedro Paramo
Chekhov (1904) (d) Alejo - "On the Marvelous
(iii) Zverkov – 1. ‘Notes from underground’ Carpentier in America"
by Dostoevsky (1864) Ans: (b) In his preface to ‘The Order of Things’,
(iv) Rastignac – 2. ‘Old Goriot’ by Balzac Foucault mentions being influenced by Louis Borges'
(1835) ‘Chinese Encyclopedia’.
Hence option (d). Hence option (b).
9. This renowned German poet was born in 13. Here is a list of Partition novels which have
Prague and died of Leukemia. When young he 'violence on the women's body' as a significant
met Tolstoy and was influenced by him. The theme. Pick the odd one out:
titles of his last two works contain the (a) The Pakistani Bride
words"sonnets" and "elegies"
(b) What the Body Remembers
He is
(a) Herman Hesse (c) Train to Pakistan
(b) Heinrich Heine (d) The Ice-Candy Man
(c) Joseph Freiherr Von Eichendorff Ans: (c) 'Train to Pakistan' is the odd one in the list as it
(d) Raine Marie Rilke is by a male author- Khushwant Singh.
Ans: (d) The above description is of Raine Marie Rilke. ‘The Pakistani Bride’ (1990) – Bapsi Sidhwa
His last two works were – ‘Duino Elegies’ and ‘Sonnets ‘What the Body Remembers’ (1999) – Shauna Singh
to Orpheus’. Baldwin
Hence option (d). ‘The Ice-Candy Man’ (1991) – Bapsi Sidhwa
10. Which of the following plays gained notoriety Hence option (c).
for its caricature of the philosopher Socrates? 14. Match the translators in List-I with the English
(a) 'The Birds' (b) 'The Wasps' translation of Indian literarure texts in List-II
(c) 'The Clouds' (d) 'The Frogs' according to the code given below :
Ans: (c) Aristophanes’ comedy ‘The Clouds’ (423 BC) List-I List-II
gained notoriety for its caricature of Socrates and is i. K.B. Vaid 1. Says Tuka
mentioned in Plato’s ‘Apology’ as a contributor to the ii. O.V. Vijayan 2. The Diary of a
philosopher’s trial & execution. It can be considered the Maid Servant
world’s first extant (surviving) ‘comedy of ideas’ iii. Dilip Chitre 3. Samskara
Hence option (c). iv. A.K. Ramanujan 4. Dharmapuri
UGC NET/JRF English Paper III, December 2012 97 YCT
Codes : Ans: (c) The word resurrect is an example of back
i ii iii iv formation which is a process of creating a new lexeme
(a) 4 1 2 3 usually by removing the suffixes (like-ion removed
(b) 3 2 1 4 from resurrection, a Latin word). The term ‘back-
(c) 2 4 1 3 formation’ was coined by James Murray in 1889.
(d) 1 2 3 4 Hence option (c).
Ans: (c) The correct matches are – 19. Match List-I with List-II according to the code
given below :
(i) K.B. Vaid – 2. 'The Diary of a Maid
Servant' (2007) List-I List-II
i. Annie John 1. Picaresque
(ii) O.V. Vijayan – 4. 'Saga of Dharmapuri'
ii. Tom Jones 2. Bildungsroman
(1985)
iii. The Sorrows of 3. Gothic
(iii) Dilip Chitra – 1. 'Says Tuka' (1991)
Young Werther
(iv) A.K. Ramanujan – 3. 'Samskara' iv. Vathek 4. Epistolary
(Eng. Trans. 1976) Codes:
Hence option (c). i ii iii iv
15. In his poem "A Morning Walk", Nissim (a) 1 2 3 4
Ezekiel talks about a 'Barbaric City sick with (b) 2 1 4 3
slums/ Deprived of seasons, Blessed with rains? (c) 4 3 2 1
Its hawkers, beggars, iron- Lunged/ (d) 3 4 1 2
Processions led by frantic drums.' Identify the Ans: (b) The correct matches are –
city : (i) Jamaica Kincaid’s ‘Annie John’–2. Bildungsroman
(a) Calcutta (b) Banaras (1985)
(c) Bombay (d) Agra (ii) Henry Fielding’s ‘Tom Jones’–1. Picaresque
Ans: (c) Ezekiel talks of Bombay in the above lines (1749)
from his poem ‘A Morning Walk’. (iii) Goethe’s ‘The Sorrows of Young Werther’
Hence option (c). (1774) – 4. Epistolary
16. In Practical Criticism I.A. Richards Iinks four (iv) William Beckford’s ‘Vathek’ (1786) – 3. Gothic
kinds of meanings in most human utterances to Hence option (b).
four aspects. 20. Ted Hughes's poem "The Thought-Fox" is :
These are I. About Thought as Fox
(a) Sense, Feeling, Tone, Intention II. About the Fox as Thought
(b) Sound, Feeling, Nuance, Intention III. About the process of writing poetry.
(c) Sense, Voice, Emotion, Intention IV. About thought entering the poet's brain like
the Fox emerging from darkness.
(d) Sense, Image, Tone, Intention
Find the most appropriate combination
Ans: (a) The aspects mentioned by I.A. Richards in according to the code :
‘Practical Criticism’ (1929) are sense, feeling, tone and (a) I and II are correct.
intention. (b) I and III are correct.
Hence option (a). (c) I and IV are correct.
17. In 'Christabel' after Geraldine enters Sir (d) I, III and IV are correct.
Leoline's castle on her way to Christabel's Ans: (d) Ted Hughes' poem ‘The Thought Fox’ (1957)
chamber there are several ill omens which is about thought as fox and the process of writing
warn the reader about Geraldine. Pick out the poetry. It is about Thought entering the poet’s brain like
phrase which does not serve as an omen : the fox emerging from darkness.
(a) 'the angry moan of the ailing mastiff bitch' Hence option (d) I, III and IV are correct.
(b) "The Owlet's Scritch' 21. In Aristotle's Poetics we read that it is the
(c) "The Moaning Wind" imitation of an action that is complete and
(d) 'a tongue of light, a fit of flame' whole, and of a certain magnitude... having a
beginning, a middle, and an end'. What is 'it'?
Ans: (c) The phrase ‘The moaning wind’ does not (a) tragedy (b) epic
appear as an ill omen in Coleridge’s ‘Christabel’( 1797- (c) poetry (d) farce
1800).
Ans: (a) In ‘Poetics’ Aristotle has defined tragedy as
Hence option (c). the imitation of an action that is complete and whole
18. The word resurrect is and of a certain magnitude, having a beginning, a
(a) an abbreviation (b) a spurious verb middle and an end.
(c) a back-formation (d) a disguised compound Hence option (a).

UGC NET/JRF English Paper III, December 2012 98 YCT


22. According to Matthew Arnold, 'touchstones' 26. The following works and phrases, 'peace
help us test truth and seriousness that makers', 'help-meet', 'the fat of the land', 'a
constitute the best poetry. What are the labour of love', 'the eleventh hour' and 'the
'touchstones'? shadow of death' were made current by
(a) The purple passages of lyric poetry (a) the British Greek scholars like Roger
(b) Passages from ancient poets Ascham
(c) The lines and expressions of the great (b) the fifteenth century British prelates
masters (c) the Puritan tractarians
(d) Passages of epic strength and vigour (d) the sixteen-century translators of the Bible
Ans: (c) According to Arnold, the touchstones are the
Ans: (d) The above words and phrases were made
lines and expressions of the great masters to test the
current by the sixteenth century translators of the Bible.
relative merit of poetry.
Hence option (c). Hence option (d).
23. 'An extremely simplified form of language used 27. Who among the following writers asserted
for oral, verbal contact among a community 'Commonwealth Literature' does not exist ?
whose members speak different languages but (a) Amitav Ghosh (b) Salman Rushdie
do not share a common language in order to (c) V.S. Naipaul (d) Nirad Chaudhari
fulfill the essential needs of communication', Ans: (b) Salman Rushdie had asserted once that
Which of the following is best described by Commonwealth Literature does not exist.
this definition? Hence option (b).
(a) Creole (b) Pidgin
28. Identify the one in correct chronological
(c) Dialect (d) Lingua franca
sequence :
Ans: (b) A pidgin is an extremely simplified form of (a) The Norman Conquest- The Death of
language used for oral, verbal communication in a Geoffrey Chaucer- William Tyndale's New
community that does not share a common language
Testament- The Birth of William
among its members.
Shakespeare
Hence option (b).
(b) The Death of Geoffrey Chaucer- William
24. What do the prosodic features of a language Tyndale's New Testament- The Birth of
tell us? William Shakespeare - The Norman
(a) The speaker's native language and its cognate Conquest
languages.
(c) The Norman Conquest- William Tyndale's
(b) The speaker's age, emotional state, social
New Testament- The Death of Geoffrey
class, educational background, geographical
provenance etc. Chaucer- The birth of William Shakespeare
(c) The speaker's self-confidence or lack of it. (d) William Tyndale's New Testament - The
(d) The speaker's command of the re-sources of Norman Conquest- The Death of Geoffrey
the language spoken by him/her and their Chaucer- The birth of William Shakespeare
deployment. Ans: (a) The correct chronological sequence is –
Ans: (b) The prosodic features of a language tell us of 1066 – The Norman Conquest
the speaker’s age emotional state social class 1400 – Death of Geoffrey Chaucer
educational background, geographical provenance etc. 1525 – Tyndale’s ‘New Testament’
Hence option (b). 1564 – Birth of William Shakespeare
25. What novel answers to the following Hence option (a).
descriptions? 29. Which of the following arrangements is in the
This was a 1990 best-seller by a British writer. correct chronological sequence?
The work incorporates many genres such as (a) Mary Wollstonecraft's 'A Vindication of the
letters, diaries and poetry as also third-person Rights of Women'- 'Lyrical Ballads' by
narratives.
Wordsworth and Coleridge- 'Lyrical Ballads
The plot here involves two time-periods - with 'Preface', second edition' by
contemporary and Victorian. The work is
Wordsworth and Coleridge- Edmund Burke's
subtitled A Romance.
'Reflections on the Revolution in France'.
(a) The Virgin in the Garden
(b) Edmund Burke's 'Reflections on the
(b) Possession
Revolution in France'- Mary Wollstonecraft's
(c) The Girl in the Polka Dot Dress
'A Vindication of the Rights of Women'-
(d) The Sea Lady.
'Lyrical Ballads' by Wordsworth and
Ans: (b) The above description is of A.S. Byatt’s 1990 Coleridge- 'Lyrical Ballads with Preface,
best seller ‘Possession: A Romance.’ second edition' by Wordsworth and
Hence option (b). Coleridge.
UGC NET/JRF English Paper III, December 2012 99 YCT
(c) 'Lyrical ballads with 'Preface', second edition' (d) The title of the essay, the title of the journal,
by Wordsworth and Coleridge- 'Lyrical volume and issue numbers, the year of
Ballads' by Wordsworth and Coleridge- publication, page number
Edmund Burke's 'Reflection on the Ans: (b) The correct syntax in Bibliography is – The
Revolution in France'- Mary Wollstonecraft's title of the essay or article, the title of the journal,
'A Vindication of the Rights of Women'. volume and issue numbers, the year of publication, page
(d) 'Lyrical Ballads' by Wordsworth and numbers.
Coleridge- 'Lyrical Ballads with 'Preface', Hence option (d).
second edition' by Wordsworth and 33. From the following indicate the work which is
Coleridge- Edmund Burke's 'Reflection on not a Dystopia :
the Revolution in France'- Mary (a) Aldous - A Brave
Wollstonecraft's 'A Vindication of the Rights Huxley New World
of Women'. (b) George - 1984
Ans: (b) The correct chronological sequence is – Orwell
1790 – Edmund Burke’s ‘Reflections on the Revolution (c) Yevgeny - We
in France.’ Zamyatin
1792 – Mary Wollstonecraft’s ‘A Vindication of the (d) Evelyn - Brideshed
Rights of Women’ Waugh Revisited
1798 – Wordsworth and Coleridge’s 'Lyrical Ballads' Ans: (d) Dystopia is a catastrophic term used as an
first edition opposite of Utopia. From the given options, Evelyn
1800 – Second edition of ‘Lyrical Ballads’ with ‘The Waugh's 'Brideshed Revisited' is not a dystopian novel.
Preface’ Hence option (d).
Hence option (b). 34. 'Unless wariness be used, as good almost kill a
man as kill a good book. Who kills a man kills
30. Who is John Keats's 'Sylvan Historian'? a reasonable creature, God's Image, but he
(a) Fanny Brawne who destroys a good book, kills reason itsilf,
(b) Nightingale kills the image of God as if were in the eye.
(c) The Grecian Urn Many a man lives a burden to the earth; but a
(d) The Bridge of Quietness good book is the precious life-blood of a master
spirit.....'
Ans: (c) The Grecian Urn is itself the story teller or
Where is the passage from?
‘sylvan historian’ in John Keats' 'Ode on a Grecian Urn'.
(a) Milton's Areopagitica
Hence option (c). (b) Sidney's Apologie for Poetry
31. This periodical was started in 1709 with a (c) Dryden's 'Preface to the Fables'
motive 'to expose the false arts of life, to pull (d) Marvell's The Rehearsal Transposed
the disguise of cunning, vanity and affectation, Ans: (a) The above passage is taken from Milton’s
and to recommend a general simplicity in our ‘Areopagitica’ (1964).
dress, our discourse and our behaviour'. The Hence option (a).
founder of the periodical wrote under the
pseudonym of Isaac Bickerstaff. 35. Virginia Woolf rubbished the idea of character
and the understanding of realism of writers
The periodical described above is like Arnold Bennett, John Galsworthy and
(a) The Tatler (b) The Spectator H.G. Wells. Her famous essay is called 'Mr.
(c) The Critical Review (d) The Rambler Bennet and Mrs. Brown'. Who is Mrs. Brown?
Ans: (a) Addison and Steele had started the above (a) The name Woolf gives a woman whom she
described periodical in 1709 with a motive ‘to expose happens to meet in a train
the false arts of life….’ (b) A servant in Mr. Bennett's household.
Hence option (a). (c) A character in a Bennett story.
32. Arrange the following in the order in which the (d) Mr. Bennett's neighbour who happens to be a
writer.
details of a research article/essay appear in
your bibliography. Ans: (a) Mrs. Brown, in V. Woolf’s essay, was the
name given by Woolf to a woman whom she met in a
(a) Page numbers, the title of the essay, the title
train. Hence option (a).
of the journal, volume and issue numbers,
year of publication 36. E.M. Forster uses some recurrent images in A
Passage to India. Pick the odd one out:
(b) The title of the essay, page numbers, the title
(a) Wasp (b) Stone
of the journal, volume and issue numbers,
(c) Thunder (d) Echo
year of publication
(c) The title of the journal, the title of the essay, Ans: (c) Thunder is not a recurrent image used by
page numbers, volume and issue numbers, Forster in ‘A Passage to India’ (1924).
year of publication Hence option (c).
UGC NET/JRF English Paper III, December 2012 100 YCT
37. Now stop your noses, readers, all and some, Reason (R) : On her 'broche of gold full shene'
For here's a tun of midnight-work to come, was written Amor Vincit Omnia.
Og, from a treason-tavern rolling home. In the context of the two statements, which one
Round as a globe, and liquor'd ev'ry chink of the following is correct?
Goodly and great he rails behind his link' (a) Both (A) and (R) are true and (R) is the
In the above extract from Absalom and correct explanation of (A).
Achitophel Og is (b) Both (A) and (R) are true but (R) is not the
(a) Elkanah Settle (b) Lord Harvey correct explanation of (A).
(c) Thomas Shadwell (d) Joseph Addison (c) (A) is true but (R) is false.
Ans: (c) ‘Og’ in Dryden’s ‘Absalom and Achitophel’ (d) (A) is false bur (R) is true.
(1681) is Thomas Shadwell.
Hence option (c). Ans: (b) It is clear on reading the two statements that
both (A) and (R) are true but (R) is not the correct
38. D.H. Lawrence uses the expression 'a bright
book of life' to describe explanation of (A).
(a) the novel Hence option (b).
(b) the dramatic monologue 41. Identify the correct statement on Langue and
(c) the Bible Parole below:
(d) the short lyric 1. Langue is the abstract language system, the
Ans: (a) D.H. Lawrence uses the expression ‘a bright grammar of a language.
book of life’ to describe the novel. 2. Parole is the language actually produced by
Hence option (a). its users following langue.
39. Identify the correctly matched group: 3. Langue is the language actually produced by
List-I List-II its users following parole.
i. Where Angels 1. Malay 4. Parole is the abstract language system, the
Fear to Tread grammar of a system.
(E.M. Forster, 1905) (a) 1 and 3 are correct. (b) 1 and 2 are correct.
ii. A Portrait of the 2. Russia (c) 2 and 3 are correct. (d) 2 and 4 are correct.
Artist as a Young Ans: (b) Langue and Parole are syntactic units. Langue
Man is the abstract language system, the grammar of a
(James Joyce, 1916) language; Parole is the language actually produced by
iii. The Plumed 3. Italy its users following language.
Serpent
-Parole [Product that is visible]
(D. H. Lawrence, 1926)
iv. An Outcast of 4. Mexico
the Islands -Langue [abstract system not visible]
(Joseph Conrad, 1896) Hence option (b).
v. Under Western 5. Dublin 42. In Monica Ali's 'Brick Lane' which among the
Eyes following characters has 'a face like a frog' ?
Codes: (a) Nazneen (b) Chanu
i ii iii iv v (c) Hasina (d) Karim
(a) 3 5 4 1 2 Ans: (b) In Monica Ali’s ‘Brick Lane’ (2003), Chanu
(b) 4 3 5 2 1 has a face like a frog.
(c) 5 4 3 2 1
Hence option (b).
(d) 2 1 3 4 5
43. "The grey-eyed morn smiles on the frowning
Ans: (a) The correctly matched pairs are –
night,
(i) ‘Where Angels Fear to Tread’ – 3. Italy
(ii) 'A Portrait of the Artist as – 5. Dublin Check'ring the eastern clouds with streaks of light;
a Young Man' And flecked darkness like a drunkard reels
(iii) ‘The Plumed Serpent’ – 4. Mexico From forth day's path and Titan's burning wheels".
(iv) ‘An Outcast of the Islands’ – 1. Malay (Romeo and Juliet II, 3, 1 - 4)
(v) ‘Under Western Eyes’ – 2. Russia The speaker describes
Hence option (a). (a) The Setting Sun
40. Given below are two statements, one labelled as (b) The Return Home of a Drunkard
Assertion (A) and the other labelled as Reason (c) The Drawing of a New Day
(R): (d) The Rising Sun
Assertion (A) : Chaucer describes 'Madame Ans: (c) Shakespeare in above lines from ‘Romeo and
Eglentyne' Thus: 'She was so charitable and so Juliet’ describes the drawing of a new day.
piteous, she wolde wepe. If that 'she sawe a mous
caught in a trappe' Hence option (c).

UGC NET/JRF English Paper III, December 2012 101 YCT


44. 'How noble in reason! How infinite in faculty! 48. 1. A content word is not a function word.
In form and moving how express and admirable! 2. A content word has lesser meaning than a
In action how like an angel! In apprehension how function word.
like a God!’ 3. A content word has no function.
What does Hamlet marvel at in this passage? 4. A content word bears lexical meaning whereas
(a) His own self (b) His father a function word just about means functionally.
(c) Man (d) Woman Which of these statements are correct ?
Ans: (c) Hamlet marvels at man and philosophises him (a) 1 and 4 are correct. (b) 1 and 2 are correct.
in the above lines. (c) 3 and 4 are correct. (d) 2 and 4 are correct.
Hence option (c). Ans: (a) The correct statements are –
45. Said identifies Orientalism as : 1. A content word is not a function word, and
I. What an Orientalist does. 4. A content word bears lexical meaning whereas a
II. A style of thought based on an ontological function word just about means functionally.
and epistemological distinction made Hence option (a).
between the Orient and the Occident. 49. The year 1828 is a landmark in the history of
III. a discourse dealing with the Orient American language and literature. Identify the
IV. a fact of nature rather than one of human reason from the following:
production (a) Mark Twain's, 'The Adventures of
In the light of the statement above : Huckleberry Finn' was published in that year.
(a) II and III are correct, I and IV are wrong. (b) The Southern Literary Messenger gained
(b) I and III are correct, II and IV are wrong. wide circulation since that year.
(c) I, II and III are correct and IV is wrong. (c) Washington Irving was adjudged the nation's
(d) IV is correct and I, II and III are wrong. greatest writer in that year.
Ans: (c) Said identifies Orientalism as – (d) Noah Webster published 'An American
Dictionary of the English Language' in that
I. What an Orientalist does.
year.
II. A style of thought based on an ontological and
Ans: (d) In 1828 Noah Webster published his landmark
epistemological distinction made between the orient and
'An American Dictionary of the English Language'.
the accident.
III. A discourse dealing with the Orient. Hence option (d).
Orientalism is not a fact of nature but an ideology. 50. What alternative title to her 'Frankenstein' did
Hence option (c). Marry Shelley give?
(a) A Gothic Tale
46. Identify the period during which the Puritans
(b) A Gothic Romance
under the rule of Oliver Cromwell and his
Commonwealth shut down all English theatres (c) The Modern Prometheus
on religious and moral grounds: (d) A Modern Parable
(a) 1640-1660 (b) 1649-1660 Ans: (c) Mary Shelley’s ‘Frankenstein’ (1818) is also
(c) 1649-1659 (d) 1640-1659 known as ‘The Modern Prometheus’.
Ans: (b/a) Note– (NTA has considered both option A Hence option (c).
and B as correct) The Puritans under the rule of Oliver 51. Which of the following statements on George
Cromwell shut down all English theatres on religious Lamming's 'In the Castle of My Skin' [1953] is
and moral grounds during the period of 1640-1660, also not true?
1649-1660 was also included in this period. In 1642, the (a) On one level this is a coming- of - age story.
Puritans ordered the indefinite closure of all London (b) It is an elegiac account of a village's growth
theatres. into awareness in the late colonial period.
47. "To tell the truth Shug act more manly than (c) Its themes parody The Tempest.
rest, men. I mean she upright, honest, speak (d) This was George Lamming's first novel.
her mind....." Ans: (c) George Lamming’s ‘In the Castle of My Skin’
What light does the quotation throw on Shug (1953) does not parody ‘The Tempest’. The other three
Avery? statements correctly describe the novel.
(a) She is a manly woman. Hence option (c).
(b) She is upright and honest in asserting her 52. We are likely to misunderstand an Emily
lesbian identity. Dickinson poem if we take her famous dashes
(c) She is bent on self-assertion to be.......
(d) Both (b) and (c) (a) quite specific and unambiguous
Ans: (d) The extract on Shug Avery asserts that she is (b) ambiguous and indeterminate
upright and honest in asserting her lesbian identity and (c) suggestive of both forward and backward
is bent on self-assertion. movements in terms of sense
Hence option (d). (d) suggestive of links but equivocally
UGC NET/JRF English Paper III, December 2012 102 YCT
Ans: (a) Emily Dickinson’s dashes can be Ans: (c) A mock-epic poem uses a heroic style to deride
misunderstood if taken as quite specific and airs and affectations.
unambiguous. They are as described in the other three Hence option (c).
statements. 57. Which of the following statements is not true of
Hence option (a). Laurence Sterne's Tristram Shandy ?
53. Readers of Tayeb Salih's 'Seasons of Migration (a) It has a linear plot.
to the North' will undoubtedly notice its (b) It opens and ends with the theme of birth.
parallels with the story/stories of: (c) It contains a trip to France.
I. Death in Venice (d) It contains a marbled page.
II. Othello
Ans: (a) The false statement about Laurence Sterne’s
III. Bartleby the Scrivener ‘Tristram Shandy’ (1759-67) is that it has a linear plot.
IV. Heart of Darkness
Hence option (a).
Of the above:
(a) I And II are correct. 58. In drama an aside is addressed...
(b) Only IV is correct. (a) to an audience by an actor, the words so
(c) II and III are correct. spoken are not meant to be heard by other
actors on the stage.
(d) II and IV are correct.
(b) to other actors of the stage; the words so
Ans: (d) Tayib Salih's 'Seasons of Migration to the spoken are not meant to be heard by the
North' has parallels with the stories of ‘Othello’ and
audience.
‘Heart of Darkness’.
(c) by the playwright to the audience.
Hence option (d).
(d) by the protagonist to his/ her antagonist
54. Which statement is not true of Benedict
Ans: (a) In drama an aside is addressed to an audience
Anderson's 'Imagined Communities'?
by an actor, the words so spoken are not meant to be
(a) It is a prosaic response to the myth of El
heard by other actor on stage.
Dorado.
(b) It is subtitled 'Reflections on the Origin and Hence option (a).
Spread of Nationalism'. 59. Match List-I with List-II according to the code
(c) In this book, Anderson advances the view given below:
that nations are not natural entities but List-I List-II
narrative constructs. (Novels) (Last Lines)
(d) In Anderson's view, modern nationalism was i. The Mayor 1. 'He walked
basically a consequence of the convergence of Casterbridge towards the faintly
of capitalism, the new print technology and humming, glowing
the fixity that resulted from print extending to town, quickly.'
'Vernacular' languages. ii. Sons and Lovers 2. 'In Their death,
Ans: (a) The false statement is that it is a prosaic they were not
response to the myth of El Dorado. The other statements divided.'
are correct. iii. The Great Gatsby 3. 'Happiness was but
Hence option (a). the occasional
55. 'By swaggering could I never thrive, for the episode in a
rain it raineth everyday'. general drama of
These lines from Twelfth Night occur in the pain.'
novel: iv. The Mill on the Floss 4. 'So we beat on,
(a) Middlemarch boats against
(b) Vanity Fair Codes:
(c) Our Mutual Friend i ii iii iv
(d) Far From the Madding Crowd (a) 1 2 3 4
Ans: (a) The above lines from ‘Twelfth Night’ occur in (b) 2 1 3 4
the novel ‘Middlemarch’ (1872) by George Eliot. (c) 4 3 2 1
Hence option (a). (d) 3 1 4 2
56. What is a mock-heroic poem? A mock- heroic Ans: (d) The correct matches are–
poem
(a) mocks at heroic pretensions in poets and (i) T. Hardy’s ‘The Mayor 3. “Happiness was but
critics of Casterbridge’ the occasional
(b) mocks heroism, an exaggerated virtue in all (1886) episode in a general
epics drama of pain.”
(c) uses a heroic style to deride airs and (ii) H. Lawrence’s 'Sons 1. “He walked towards
affectations and lovers' (1913) the faintly
(d) uses a mocking style to deride heroes and humming, glowing
hero-worship town, quickly.”
UGC NET/JRF English Paper III, December 2012 103 YCT
(iii) F. Scott Fitzgerald’s 4. "So we beat on, 66. Statement (S) : "Our birth is but a sleep and
'The Great Gatsby' boats against". forgetting."
(1925) Interpretation (I) : The human soul never tires in
(iv) G. Eliot’s ‘The Mill on 2. “In their death, they the course of life, it never dies. Therefore, the
the Floss’ (1860) were not divided.” human life is a long sleep and ephemeral events
Hence option (d). are better forgotten.
60. "There is nothing outside the text," is a (a) (S) is a view and (I) is not correct.
statement by (b) (S) is a view and (I) is correct.
(a) Victor Shklovsky (c) (S) is a poetic view, the (I) does not suit it.
(b) Jacques Derrida (d) (S) is a poetic view and bears no relationship
(c) Roland Barthes to (I).
(d) Ferdinand de Saussure Ans: (b) The interpretation correctly explains the view
in the statement.
Ans: (b) Jacques Derrida has said, ‘there is nothing
outside the text’. Hence option (b).
Hence option. (b) 67. "The parish of rich women, physical decay,/
61. Here is a list of women abandoned by their Yourself...."
lovers in Hardy's novels. Pick the odd one out : What do these make of W.B. Yeats in W.H.
(a) Fanny Robin (b) Tess D' Urberville Auden's view?
(c) Marty South (d) Bathsheba Everdene (a) Proud (b) Vainglorious
Ans: (d) Bathsheba Everdene in ‘Far from the Madding (c) Avaricious (d) Silly
Crowed’ (1874) is the only one in the list who Ans: (d) In above extract from his 'In memory of W.B.
abandoned her suitors instead, while all others were Yeats', W.H. Auden views W.B. Yeats as silly– "You
abandoned by their lovers. were silly like us; your gift survived it all : The Parish
Hence option (d). ....Yourself".
Hence option (d).
62. What is the following a description of?
'a loose sally of the mind; an irregular 68. Who among Charles Dickens's characters is
indigested piece' 'umble' and who 'willin' ?
(a) Essay (b) Autobiography (a) Mr. Pickwick, Mrs. Gamp
(c) Epistolary Fiction (d) Diary (b) Master Humphrey, Nicolas Nickleby
Ans: (a) The 'essay' was defined by a famous critic as a (c) Martin, Little Nell
loose sally of the mind, an irregular indigested piece.’ (d) Uriah Heep, Barkis
Hence option (a). Ans: (d) The above described characters are – Uriah
Heep and Barkis from Dickens’ 'David Copperfield'.
63. From the following indicate the critic who is
not a New Critic: Hence option (d).
(a) Allen Tate 69. "Fourth World Literature" refers to
(b) Robert Penn Warren I. the works of native people living in a land that
(c) Cleanth Brooks has been taken over by non-natives
(d) Claude Levi-Strauss II. the works of black people in the United States.
Ans: (d) Claude Levi-Strauss is associated with III. the literature of the marginalized
structuralism and post-structuralism. IV. refers to the works of non-heterosexuals
Hence option (d). Of the above :
(a) I and II are correct.
64. From the following list, pick out a woman
character who does not belong to Amitav (b) I and III are correct.
Ghosh's novels : (c) II and IV are correct.
(a) IIa (b) Urvashi (d) I, III and IV are correct.
(c) Sonali (d) Piyali Ans: (c) Fourth World Literature refers to the works of
Ans: (b) Urvashi is not a character in Amitav Ghoshi’s black people in the US (II.) and to the works of non-
novels. heterosexuals (IV.).
Hence option (b). Hence option (c).
65. Pick the odd man out of the following members 70. Assertion (A) : In 'The Duchess of Malfi'
of the subaltern group : Ferdinand sets a whole group of mad men on the
Duchess and they dance and sing in a crazy
(a) Ranajit Guha (b) Partha Chatterjee
manner.
(c) Dipesh Chakrabarty (d) Sumit Sarkar
Reason (R) : His desire was to provide a strange
Ans: (d) Sumit Sarkar is a Modern Indian Historian entertainment to drive the Duchess mad.
associated with the Swadeshi Movement. In the context of the two statements, which one
Hence option (d). of the following is correct?
UGC NET/JRF English Paper III, December 2012 104 YCT
(a) (A) is correct, but (R) is wrong. ii. William Congreve,, 2. ThePre-Raphaelite
(b) Both (A) and (R) are correct. William Wycherley Brotherhood
(c) (A) is wrong. but (R) is correct. George Etherege,
(d) Both (A) and (R) are wrong. George Farquhar
Ans: (b) Both (A) and (R) are correct. iii. John Everett 3. The University
Hence option (b). Millais, James Wits
71. Why is 'The Signifying Monkey' of Henry Collinson, Ford
Louis Gates JR. a notable contribution to the Madox Brown,
study of African-American literature? Dante Gabriel
(a) It focuses on largely neglected African- Rossetti
American novelists and poets. iv. Ernest Dowson, 4. The
(b) It offers a theory of African- American Lionel Johnson, Restoration
criticism that draws upon rhetorical and W.B. Yeats Playwrights
signifying practices. Codes:
(c) It offers a theory of African - American films i ii iii iv
and dramatic arts that signify Black ethos. (a) 3 2 1 4
(d) It departs from critical theory of (b) 1 4 3 2
autobiographical narrative involving Black (c) 2 1 4 3
lives and cultural traditions.
(d) 3 4 2 1
Ans: (b) ‘The Signifying Monkey’ by Henry Louis
Gate JR offers a theory of African - American criticism Ans: (d) The correct matches are –
that draws upon rhetorical and signifying practices. (i) George Peele, 3. The University wits
Hence option (b). Robert Greene,
Thomas Lodge
72. This influential critic
Thomas Kyd.
I. wrote influential commentaries on such poets
as Shelley, Blake and Yeats. (ii) William Congreve, 4. The Restoration
II. Published such titles as The Anxiety if William Wycherley, playwrights
Influence, A Map of Misreading, Poetry and George Etherege,
Repression and The Western Canon. George Farquhar.
III. asserted that most literary criticism is but (iii) John Everett Millais, 2. The Pre-Raphaelite
slightly disguised religion and James Collinson, Brotherhood.
IV. is, arguably, the most widely known and Ford Maddox Ford,
contrarian among his American peers in the Brown, D.G.
English Academy, Rossetti.
Identify the critic (iv) Ernest Dowson, 1. The Rhymer’s Club;
(a) Edward Said (b) Geoffrey Chaucer Lionel Johnson, The Decadents of the
(c) Harold Bloom (d) Sven Birkrets W.B. Yeats. 1890’s
Ans: (c) The above description is about Harold Bloom. Hence option (d).
Hence option (c). 75. Combine the statements correctly:
73. According to the Italian Marxist theorist According to Homi Bhabha..................
Antonio Gramsci : 1. mimicry is not mere copying or emulating the
(a) hegemony is synonymous with domination colonizer's culture, behaviour and manners.
(b) hegemony involves a degree of consent on 2. but it is further aimed at perfection and excess.
the part of subject people. 3. mimicry is mere copying the colonizer's
(c) hegemony involves a degree of co-political culture, behaviour and manners................
entity. 4. but is informed by both mockery and a certain
(d) hegemony is synonymous with subjugation. menace.
Ans: (b) According to Gramsci, hegemony involves a (a) 1 and 4 (b) 1 and 2
degree of consent on the part of subject people. (c) 3 and 4 (d) 3 and 2
Hence option (b). Ans: (a) According to Homi Bhabha –
74. Match the following: 1. Mimicry is not mere copying or emulating the
i. George Peele, 1. The Rhymers' colonizer’s culture, behaviour and manners.
Robert Greene, Club/The 4. But it is informed by both mockery and a certain
Thomas Lodge, Decadents of menace.
Thomas Kyd The 1890's Hence option (a).
UGC NET/JRF English Paper III, December 2012 105 YCT
UGC NET/JRF Exam, December-2012
ENGLISH
Solved Paper-II
Note : This paper contains fifty (50) objective 5. One of the following texts was published earlier
type questions, each question carrying two (2) marks. than 1955. Identify the text :
Attempt all the questions. (a) William Golding, The Inheritors
1. Identify the work below that does not belong to (b) Philip Larkin, The Less Deceived
the literature of the eighteenth century : (c) William Empson, Collected Poems
(a) Advancement of Learning (d) Samuel Becket, Waiting for Godot
(b) Gulliver's Travels Ans: (d) Samuel Beckett's, play, 'Waiting for Godot'
was published earlier than 1955. Originally it was
(c) The Spectator known as 'En Attendant Godot', and comes under the
(d) An Epistle to Dr. Arbuthnot category of tragicomedy.
Ans: (a) Francis Bacon’s 'The Advancement of Hence, option (d) will be correct answer.
Learning' was published in 1605 and thus does not 6. Who among the poets in England during the
belong to the 18th century. 1930s had left-leaning tendencies?
Swift’s ‘Gulliver’s Travels’ – 1726, (a) T.S. Eliot, Ezra Pound, Richard Aldington
Addison’s 'The Spectator'– 1711-12, (b) Wilfred Owen, Siegfried Sassoon, Rupert
Brooke
Pope’s 'An Epistle to Dr. Arbuthnot'- 1735.
(c) W.H. Auden, Louis MacNeice, Cecil Day
Hence option (a). Lewis
2. Which, among the following, is a place through (d) J. Fleckner, W.H. Davies, Edward Marsh
which John Bunyan's Christian does NOT Ans: (c) W.H. Auden, Louis MacNeice and Cecil Day
pass? Lewis had left-leaning tendencies in the 1930s English
(a) The Slough of Despond poets. Hence option (c).
(b) Mount Helicon 7. Match the following :
(c) The Valley of Humiliation 1. The Sage of 5. Emily Dickinson
(d) Vanity Fair Concord
Ans: (b) In John Bunyan’s 'Pilgrim's Progress' (1678),
2. The Nun of 6. R.W. Emerson
Christian passes through the Slough of Despond, the Amherst
Valley of Humiliation, Vanity Fair, but not through Mt. 3. Mark Twain 7. T.S. Eliot
Helicon. Hence option (b). 4. Old Possum 8. Samuel L.
Clemens
3. The period of Queen Victoria's reign is (a) 1-6; 2-5; 3-8; 4-7
(a) 1830-1900 (b) 1837-1901 (b) 1-5; 2-6; 3-7; 4-8
(c) 1830-1901 (d) 1837-1900 (c) 1-8; 2-7; 3-6; 4-5
Ans: (b) Queen Victoria reigned from 1837-1901. (d) 1-7; 2-8; 3-5; 4-6
Hence option (b). Ans: (a) The correct matches are:
4. Which of the following statements about the Pen/other names Authors
Lyrical Ballads is NOT true? 1. The sage of concord 6. R.W. Emerson
(a) It carried only one ballad proper, which was 2. The Nun of Amherst 5. Emily Dickinson
Coleridge's The Rime of the Ancient Mariner. 3. Mark Twain 8. Samuel L. Clemens
4. Old Possum 7. T.S. Eliot
(b) It also carried pastoral and other poems.
Hence option (a).
(c) It carried a "Preface" which Wordsworth
8. Name the theorist who divided poets into
added in 1800.
"strong" and "weak" and popularized the
(d) It also printed from Gray's Elegy Written in a practice of misreading :
Country Churchyard. (a) Alan Bloom (b) Harold Bloom
Ans: (d) Gray’s ‘Elegy written in a Country (c) Geoffrey Gartman (d) Stanley Fish
Churchyard’ was published in 1751 and not in ‘Lyrical Ans: (b) Harold Bloom divided poets into ‘strong’ and
Ballads’ (1798). Hence option (d). ‘weak’ and popularized misreading. Hence option (b).
UGC NET/JRF English Paper II, December 2012 106 YCT
9. In The Rape of the Lock Pope repeatedly Ans: (d) A.C. Swinburne (1837-1909) was not a
compares Belinda to contemporary of Wordsworth (1770-1850) and
(a) the sun (b) the moon Coleridge. (1772-1834).
(c) the north star (d) the rose Hence option (d).
Ans: (a) In his mock heroic poem ‘The Rape of the 15. Which of the following statements about
Lock’ (1712), Pope compares Belinda quite often to the Waiting for Godot is true?
sun.
1. It carries a subtitle: "a tragicomedy in two
Hence option (a). acts"
10. Which of the following awards is not given to 2. It carries a subtitle: "a tragicomedy in two
Indian-English writers? scenes"
(a) The Booker prize
3. It carries a subtitle: "a tragicomedy in two
(b) The Sahitya Akademi Award
parts"
(c) The Gyanpeeth
4. It does not carry a subtitle.
(d) Whitbread Prize
(a) 4 (b) 2
Ans: (c & d) The Gyanpeeth Award and Whitbread
(c) 3 (d) 1
Prize are the awards, not given to Indian-English
writers. The Gyanpeeth Award is a prestigious award in Ans: (*) There is no harmony between the question and
India for achievement in Indian literature. given answers. The combination of the statements that
Hence, option (c) & (d) will be correct answer. can justify the answer is not kept in the proper
sequence. Therefore UGC has not taken any answer that
11. Identify the correct statement below :
can support the right answer.
(a) Gorboduc is a comedy, while Ralph Roister
Doister and Gammer Gurton's Needle are 16. The Bloomsbury Group included British
tragedies. intellectuals, critics, writers and artists. Who
(b) Gorboduc is a tragedy, while Ralph Roister among the following belonged to the
Doister and Gammer Gurton's Needle are Bloomsbury Group?
comedies. I. John Maynard Keynes, Lytton Strachey
(c) all of them are problem plays. II. E.M. Forster, Roger Fry, Clive Bell
(d) All of them are farces. III. Patrick Brunty, Paul Haworth
Ans: (b) The correct statement is – ‘Gorboduc’ is a IV. Thomas Hardy, Henry James, Walter Pater
tragedy, while ‘Ralph Roister Doister’ and ‘Gammer (a) I and II (b) I
Gurton’s Needle’ are comedies. (c) II and III (d) IV
Hence option (b).
Ans: (a) The Bloomsbury Group included – J.M.
12. W.M. Thackeray's Vanity Fair owes its title to Keynes, Lytton Strachey, E.M. Forster, Roger Fry and
(a) Browning's Fifine at the Fair Clive Bell, along with Virginia Woolf.
(b) Shakespeare's Merchant of Venice Hence option (a).
(c) Goldsmith's Vicar of Wakefield
17. Who among the following is credited with the
(d) Bunyan's Pilgrim's Progress
making of the first authoritative Dictionary of
Ans: (d) Thackeray picked the title of his novel ‘Vanity the English Language?
Fair or A Novel without a Hero’ (1848) from Bunyan’s’
(a) Bishop Berkeley (b) Samuel Johnson
‘Pilgrim’s Progress’ (1678). Hence option (d).
(c) Edmund Burke (d) Horace Walpole
13. The Puritans shut down all theaters in England
in Ans: (b) Samuel Johnson published the first
(a) 1642 (b) 1640 authoritative ‘Dictionary of the English Language’ in
(c) 1659 (d) 1660 1755. Hence option (b).
Ans: (a) The Puritans shut down all theaters in England 18. In Dryden's Essay of Dramatic Poesy (1668),
in 1642. Hence option (a). who opens the discussion on behalf of the
ancients?
14. Who of the following was not a contemporary
of Wordsworth and Coleridge? (a) Lisideius (b) Crites
(a) Robert Southey (c) Eugenius (d) Neander
(b) Sir Walter Scott Ans: (b) Crites favours the ancients in Dryden’s 'Essay
(c) William Hazlitt of Dramatic Poesy' (1668).
(d) A.C. Swinburne Hence option (b).

UGC NET/JRF English Paper II, December 2012 107 YCT


19. The term 'invective' refers to 23. Which Shakespearean play contains the line: "
(a) the abusive writing or speech in which there ....... there is a special providence in the fall of a
is harsh denunciation of some person or sparrow"?
thing. (a) King Lear (b) Hamlet
(b) an insulting writing attack upon a real person, (c) Coriolanus (d) Macbeth
in verse or prose, usually involving caricature Ans: (b) The above line appears in the play ‘Hamlet’.
and ridicule. Hence option (b).
(c) a written or spoken text in which an 24. Match the following pairs of books and authors :
apparently straightforward statement or event Books Authors
is undermined in its context so as to give it a
I. Condition of the i. John Ruskin
very different significance.
Working Class in
(d) the chanting or reciting of words deemed to
England
have magical power.
II. London Labour and ii. Henry Mayhew
Ans: (a) ‘Invective’ is abusive or venomous speech used
the London Poor
to blame or censure of some person or thing. Hence
option (a). III. Past and Present iii. Thomas
Carlyle
20. Which of the following novels depicts the plight
IV. The Unto This Last iv. Friedrich
of the Bangladeshi immigrants in East
London? Engels
(a) How far can you go Codes:
(b) The White Teeth I II III IV
(c) An Equal Music (a) iv i ii iii
(d) Brick Lane (b) iv ii iii i
(c) ii iv i ii
Ans: (d) The novel that depicts the plight of the
(d) iii ii iv iv
Bangladeshi immigrants in East London is ‘Brick Lane’
(2003) by Monica Ali. Hence option (d). Ans: (b) The correct matches are –
21. The year 1939 proved to be a crucial year for I. 'Condition of the working iv- Friedrich
two important writers in England. Identify the Class in England' (1845) Engels
correct phrase below : II. 'London Labour and the ii. Henry Mayhew
(a) For Yeats who died, for Auden who left London poor' (1851)
England for the U.S. III. 'Past and Present' (1843) iii. Thomas
(b) For Eliot who started publishing verse- Carlyle.
drama, for Hardy, whose 'Wessex Poems' IV. 'Unto this last' (1860) i. John Ruskin.
were published. Hence option (b).
(c) For Evelyn Waugh and Graham Greene, each 25. In which of the following texts do Aston, Davies
for publishing his first novels. and Mick appear as characters?
(d) For Eliot who won the Nobel Prize and (a) Wyndham Lewis's Enemy
Orwell who published his 'Animal Farm'. (b) Harold Pinter's Caretaker
Ans: (a) In 1939, Yeats died and Auden left England (c) Katherine Mansfield's Life of Ma Parker
for U.S. Hence option (a). (d) Graham Greene's Brighton Rock
22. The Enlightenment was characterized by Ans: (b) Aston, Davies and Mick appear as characters
(a) accelerated industrial production and general in Harold Pinter’s ‘Caretaker’ (1960).
well-being of the public. Hence option (b).
(b) a belief in the universal authority of reason 26. What is common to the following writers?
and emphasis on scientific experimentation. Identify the correct description below:
(c) the Protestant work ethic and compliance William Congreve
with Christian values of life. George Etherege
(d) an undue faith in predestination and neglect William Wycherley
of free will.
Thomas Otway
Ans: (b) The Enlightenment was characterized by a (a) All of these were Restoration play-wrights
belief in the universal authority of reason and emphasis (b) All of them were critics of Orwell's regime
on scientific experimentation. (c) All of them edited Shakespeare's plays
Hence option (b). (d) All of them wrote tragedies in the same age
UGC NET/JRF English Paper II, December 2012 108 YCT
Ans: (a) Congreve, Etherege, Wycherley and Otway 31. "Kubla Khan" takes an epigraph from
were all Restoration playwrights. Restoration period (a) Samuel Purchas' Purchas His Pilgrimage
was from 1660-1700. (b) Hakluyt's Voyages
Hence option (a). (c) The Book Named the Governour
27. In which Jane Austen novel do you find the (d) Sir Thomas More's Utopia
characters Anne Elliott, Lady Russell, Louisa Ans: (a) ‘Kubla Khan’ takes an epigraph from Samuel
Musgrove and Captain Wentworth? Purchas' 'Purchas His pilgrimage' (1613). Hence
(a) Emma option (a).
(b) Mansfield Park
32. Which of the following author-theme is
(c) Persuasion correctly matched?
(d) Northanger Abbey
Author Theme
Ans: (c) Anne Elliott, Lady Russell, Louisa Musgrove
(a) 'The Battle of the Quarrel between
and Capt. Wentworth are characters in Jane Austen’s
Books' (J. Swift) ancient and
‘Persuasion’ (1818).
modern authors.
Hence option (c).
(b) 'The Rape of the Tribute to "The
28. In which of his essays does Homi Bhabha
Lock' (A. Pope) rude forefathers
discuss the 'discovery' of English in colonial
of the hamlet".
India?
(a) "Signs taken for Wonders" (c) Gray's "Elegy" Accumulation of
(b) "Mimicry" wealth and loss
of human lives
(c) "Nation and Narration"
and values.
(d) "The Commitment to Theory"
(d) 'The Deserted Quarrel between
Ans: (a) In his essay ‘Signs taken for Wonders’ Homi
Village' (Oliver two families
Bhabha discusses the ‘discovery’ of English in colonial
Goldsmith) caused by Lord
India.
Peter.
Hence option (a).
29. .........was The first Sonnet Sequence in English. Ans: (*) Note– (Marks was awarded to all the appeared
(a) Edmund Spenser's Amoretti candidates)
(b) Philip Sidney's Astrophel and Stella 'The Battle of The Books' by Swift deals with the
madness of pride involved in believing one's own age to
(c) Samuel Daniel's Delia
be supreme and the inferiority of derivative works.
(d) Michael Drayton's Idea's Mirror
'The Rape of The Lock by Pope is a humorous
Ans: (b) Philip sidney's Astrophel and Stella was the
indictment of the vanities and idleness of 18th century
first sonnet sequence in English. The publication of
high society. Thomas Gray's Elegy develops themes
Sidney's Astrophel and Stella (1591) generated an
that emphasizes the commonalities among human
equally extraordinary vogue for the sonnet sequence.
beings.
Hence, the correct answer is option (b).
'The Deserted Village by Oliver Smith was written with
30. Which is the correct sequence of the novels of the express purpose of evoking strong emotions in the
V.S. Naipaul? readers.
(a) The Mystic Masseur- Miguel Street- The
Suffrage of Elvira- A House for Mr. Biswas. 33. Which among the following titles set a course
for academic literary feminism?
(b) Miguel Street- The Mystic Masseur- A
House for Mr. Biswas- The Suffrage of (a) Nostromo
Elvira. (b) From Ritual to Romance
(c) The Suffrage of Elvira- Miguel Street- The (c) A Room of One's Own
Mystic Masseur- A House for Mr. Biswas. (d) A Dance to the Music of Time
(d) The Mystic Masseur- The Suffrage of Elvira- Ans: (c) Feminism as an academic study was
Miguel Street- A House for Mr. Biswas. established by V. Woolf’s essay ‘A Room of One’s
Ans: (d) The correct sequence of V.S. Naipaul’s novels Own’ (1929). Hence option (c).
are – 34. In which play do we see a reworking of E.M.
1957 – ‘The Mystic Masseur’ Forster's A Passage to India as a cameo?
1958 – ‘The Suffrage of Elvira.’ (a) The Birthday Party
1959 – ‘Miguel Street.’ (b) A Resounding Tinkle
1961 – ‘A House for Mr. Biswas’. (c) Indian Ink
Hence option (d). (d) Amadeus
UGC NET/JRF English Paper II, December 2012 109 YCT
Ans: (c) E.M. Forster’s ‘A Passage to India’ (1924) 41. Whom did Keats regard as the prime example
appears as a cameo in the play ‘Indian Ink’ (1995) by of 'negative capability'?
Tom Stoppard. Hence option (c). (a) John Milton
35. Shakespeare's sonnets (b) William Wordsworth
(a) do not carry a dedication. (c) William Shakespeare
(b) are dedicated to James I of England. (d) P.B. Shelley
(c) are dedicated to Mary Arden. Ans: (c) Keats regarded Shakespeare as the Prime
(d) are dedicated to an unknown "Mr. W.H." example of ‘negative capability’.
Ans: (d) Shakespeare’s sonnets are dedicated to an Hence option (c).
unknown Mr. W.H. (Henry Wriothesley) and a dark 42. Charles Dickens's A Tale of Two Cities begins
lady. Hence option (d). with the sentence
36. Which of the following poems uses terza rima? (a) It was the best of times, it was the worst of
(a) John Keats's "Ode to a Nightingale" times.
(b) P.B. Shelley's "Ode to the West Wind" (b) It was the brightest of times, it was the
(c) William Wordsworth's "The Solitary Reaper" darkest of times.
(d) Alfred Tennyson's "Ulysses" (c) It was the richest of times, it was the poorest
Ans: (b) ‘Terza rima’ is a stanza of three lines or tercets of times.
rhyming aba bcb cdc. Out of the options P.B. (d) It was the happiest of times, it was the
Shelley’s ‘Ode to the West Wind’ uses terza rima. saddest of times.
Hence option (b). Ans: (a) Dicken’s ‘A Tale of Two Cities’ (1859) begins
37. When one says that "someone is no more" or with the sentence- “ It was the best of times, it was the
that "someone has breathed his/her last", The worst of times.”
speaker is resorting to Hence option (a).
(a) euphism (b) euphony 43. The works of Gerard Manley Hopkins were
(c) understatement (d) euphemism published posthumously by
Ans: (d) Euphemism is saying something indirectly or (a) Edwin Muir
inoffensively which is the case here. (b) Edward Thomas
Hence option (d). (c) Robert Bridges
38. Which of the following are "companion (d) Coventry Patmore
poems"?
Ans: (c) The works of G.M. Hopkins were published
(a) "Gypsy songs" and "Songs and Sonnets"
long after his death by Robert Bridges. Hence option (c).
(b) "L'Allegro" and "II Penseroso"
44. Which of the following is the correct
(c) "The Good Morrow" and "The Sun Rising"
chronological sequence?
(d) "Full Fathom Five" and "Hark, Hark! the
Lark" (a) A Poison Tree - The Deserted Village - The
Blessed Damozel - Ozymandias
Ans: (b) Milton’s "L’ Allegro" and "II Penseroso" are
(b) The Deserted Village - A Poison Tree -
'companion poems' published in 1645. 'L' Allegro'
Ozymandias - The Blessed Damozel
means the 'happy' man and 'II Penseroso' means the
'melancholy' man. (c) The Blessed Damozel - A Poison Tree - The
Hence option (b). Deserted Village - Ozymandias
(d) The Deserted Village - The Blessed Damozel
39. What does the term 'episteme' signify?
- Ozymandias - A Poison Tree
(a) Knowledge (b) Archive
(c) Theology (d) Scholarship Ans: (b) 1770 – Oliver Goldsmith’s ‘The Deserted
Village’
Ans: (a) ‘episteme’ means knowledge.
1794 – W. Blake’s ‘A Poison Tree.’
Hence option (a).
1818 – P.B. Shelley’s ‘Ozymandias’.
40. Which of the following is a better definition of
1850 –D.G. Rossetti’s 'The Blessed Damozel'. Hence
an image in literary writing?
option (b).
(a) A reflection (b) A speaking picture
(c) A refraction (d) A reflected picture 45. The term homology means a correspondence
between two or more structures. Who of the
Ans: (b) In literary writing an ‘image’ is a speaking following developed a theory of relations
picture. between literary works and social classes in
Hence option (b). terms of homologies?
UGC NET/JRF English Paper II, December 2012 110 YCT
(a) Raymond Williams Codes :
(b) Christopher Caudwell I II III IV
(c) Lucien Goldmann (a) i ii iii iv
(d) Antonio Gramsci
(b) ii iii iv i
Ans: (*) Note– (NTA has considered both option A and
C correct and marks was awarded for both options) (c) iii iv i ii
Raymond Henry Williams was a Welsh socialist writer (d) iv iii i ii
and he was credited to develop the theory of Ans: (b) The correct matches are –
correspondence or homology between literary works
I. Milton’s ‘Areopagitica’ (1644) – (ii) The liberty
and social classes.
for unlicensed printing.
Along with him, Lucien Goldmann believes that there is
a homology between the literary structure and the II. Hobbes ‘Leviathan’ (1651) – (iii) Absolute
existing social structure when the work was created sovereignty
since both are the products of the same structural III. Dryden’s 'Alexander’s Feast' (1697) – (iv) The
process. power of music
46. F. Turner's famous hypothesis is that IV. Congreve’s 'The Way of the World' (1700) – (i)
(a) the Frontier has outlived its ideological utility
Fashion, courtship
in American civilization.
(b) the Frontier has posed a challenge to the Hence option (b).
American creative imagination 49. The preliminary version of James Joyce's
(c) the Frontier has been the one great Portrait of the Artist as a Young Man was called
determinant of American civilization (a) Stephen Hero
(d) the frontier has been the one great deterrent
(b) Bloom's Blunder
to American progress.
(c) A Day in the life of Stephen Dedalus
Ans: (c) F. Turner hypothesized that the Frontier has
been the one great determinant of American civilization. (d) The Dead
Hence option (c). Ans: (a) James Joyce had begun the ‘Portrait’ as
47. Which statement (s) below on the Spenserian ‘Stephen Hero’ in 1903 and abandoned in 1907 and
Stanza is/are accurate? later reworked and serialized in The Egoist from 1914-
I. a quatrain, unrhymed, but alliterative 15. Hence option (a).
II. a stanza of four lines in iambic pentameter
50.
III. an eight-line stanza in iambic pentameter
followed by a ninth in six iambic feet (i) A pastiche is a mixture of themes, stylistic
IV. an eight-line stanza with six iambic feet elements of subjects borrowed from other
followed by a ninth in iambic pentameter works.
(a) I and II (b) II (ii) It is distinguished from parody because not
(c) III (d) IV all parody is pastiche.
Ans: (c) A spensarian stanza is an eight-line stanza in (iii) A pastiche is also known as a 'purple
iambic pentameter followed by a ninth in six iambic passage'.
feet. It has a rhyming scheme ababbcbcc. Hence, option
(iv) A pastiche is given to an elevated style,
(c) is correct.
especially in its use of figurative language.
48. Match the following texts with their respective
themes : (a) (i) and (ii) are correct.
I. Areopagitica i. Fashion, courtship, (b) only (i) is correct.
(Milton) seduction (c) (iii) and (iv) are correct.
II. Leviathan ii. The liberty for (d) only (iv) is correct.
(Hobbes) unlicensed Ans: (a) A Pastiche is an imitation of themes, style or
printing character borrowed from other works and is
III. Alexander's Feast iii. Absolute distinguished from parody as not all parody is pastiche,
(Dryden) sovereignty
which celebrates rather than mocks or ridicules. The
IV. The Way of the iv. The power of
work it imitates as in parody.
World music
(Congreve) Hence option (a) (i) and (ii) are correct.

UGC NET/JRF English Paper II, December 2012 111 YCT


UGC NET/JRF Exam, June-2013
ENGLISH
Solved Paper-III
Note : This paper contains seventy five (75) 3. Which one of the Brecht's works was intended
objective type questions, of two (2) marks each. All to lampoon the conventional sentimental
questions are compulsory. musical opera but the public lapped up the
1. Match the following: work's sentiment and missed the humour?
List -I List-II (a) Man is Man (b) Three Penny Opera
(Browning's poems) (Type of Character) (c) The Mother (d) Life of Galileo
I. Abt Vogler 1. A Medieval Knight Ans: (b) Bertolt Brecht’s ‘Three Penny Opera’ (1928)
II. Andrea del Sarto 2. A Musician was intended to satirize conventional musical opera but
III. Chile Ronald to although quite popular, its humour was missed by the
the Dark Tower public who were only interested in its sentimental
Came 3. A Poet element.
IV. Cleon 4. An Artist Hence option (b).
The right combination according to the code is: 4. Ostensibly a musical treatise, 'The Anatomy of
Code: Melancholy' is a reflection on human learning
I II III IV and endeavour published under the
(a) 4 2 3 1 pseudonym
(b) 2 4 1 3 (a) Vox Populi (b) Epicurus Senior
(c) 3 1 2 4 (c) Democritus Junior (d) Jesting Pilate
(d) 1 3 4 2 Ans: (c) Robert Burton used the persona of Democritus
Junior in the satirical preface to ‘The Anatomy of
Ans: (b) Browning’s poems are based on a certain type
of characters- Melancholy’ (1621) and stated that ‘I write of
melancholy by being busy to avoid melancholy.’
I. ‘Abt Vogler’ (1964) 2. A Musician.
Hence option (c).
II. 'Andrea del Sarto' (1855) 4. An Artist.
III. ‘Childe Ronald to the Dark 1. A Medieval 5. Horace Walpole's novel 'The Castle of Otranto'
tells the story of
Tower came’ (1855) Knight
(a) A defiant and heartless tyrant who kills his
IV. ‘Cleon’ (1855) 3. A Poet.
own son mercilessly.
Hence option (b).
(b) A usurper and a tyrant who kills his own
2. All forms of feminism posit that : daughter by mistake.
Code: (c) A castle that collapses and crushes the young
I. The relationship between the sexes is one of and sickly prince to death.
inequality and oppression. (d) A tyrant who retires to a monastery at the end
II. There should be an end to all wars. and lives happily ever after with his queen.
III. Women need financial independence, Ans: (b) Horace Walpole’s ‘The Castle of Otranto’
IV. All men are prone to violence. (1764) is the story of a usurper and a tyrant (Manfred)
The correct combination according to the who kills his own daughter (Matilda) by mistake.
codes is : Hence option (b).
(a) I and II are correct.
6. In the Literature of Romanticism there was a
(b) III and IV are correct.
widespread frustration with visions
(c) I and III are correct. experienced in dreams, in nightmares and
(d) II and IV are correct. other altered slates. The following list contains
Ans: (c) All forms of the feminism movement hold the poems which illustrate this theme, with one
position that – exception. Identify the exception
I. The relationship between the sexes is one of (a) "Kubla Khan"
inequality and oppression and, (b) "Confessions of an English Opium Eater"
III. Women need financial independence. (c) "The Ruined Cottage"
Hence option (c). (d) "The Fall of Hyperion"
UGC NET/JRF English Paper III, June 2013 112 YCT
Ans: (*) In the literature of Romanticism, there was a 10. Which of the following statements is not true of
widespread frustration with visions experienced in Mahesh Dattani's 'Final Solutions'?
dreams, in nightmares and other altered states. All the (a) The play centres around a middle class Hindu
given options follow this experiences. family during a communal riot.
Dropped by UGC and marks awarded to all appeared (b) It challenges communalism.
candidates. (c) It is concerned with homosexual relationship.
7. The book was for many years banned for (d) It promotes religious pluralism in South Asia.
obscenity in Britain and the United States. The Ans: (c) Mahesh Dattani’s ‘Final Solutions’ (1993)
central character is a Catholic Jew in Ireland. centers around a middle-class Hindu family during a
The author claimed that the book is meant to communal riot. It challenges communalism and
make you laugh. Which is this book? promotes religious pluralism in South Asia. It is not
(a) The Picture of Dorian Gray concerned with homosexual relationship.
(b) Herzog Hence option (c).
(c) Portnoy's Complaint 11. According to Bakhtin the idea of the
(d) Ulysses Carnivalesque represents the following
Ans: (d) ‘Ulysses’ (1922) by James Joyce was banned characteristics except:
for obscenity for some time and its central character is a (a) a liberation from the prevailing truth and
Catholic Jew. established order
Hence option (d). (b) a harking back to the past
8. A.S. Byatt in her famous award winning novel (c) emphasis on play, parody, pleasure and the
of 1990 contrasts past and present involving a body
search for a Victorian poet's past illuminating (d) the suspension of all hierarchical rank,
a contemporary university researcher's life and principles, norms and prohibitions
times. Which is the novel? Ans: (b) According to Bakhtin the idea of
(a) The Virgin in the Garden Carnivalesque does not represent a harking back to the
(b) Possession past. The other options are correct.
(c) Babel Tower Hence option (b).
(d) Still Life 12. Which of the following statements is not true of
Ans: (b) A.S. Byatt’s ‘Possession’ (1990) contrasts the Patrick White?
past and present of the life of a Contemporary (a) He is remembered today for his epic and
University Researcher. psychological narrative art.
Hence option (b). (b) He is the only Australian to receive the Nobel
9. Which of the following statements best Prize in Literature.
describes J.M. Coetzee's 'Disgrace'? (c) He pioneered a new fictional landscape and
(a) It is a murder mystery set in post-apartheid introduced a new continent in literature.
South Africa. (d) His style is noted for lucidity and simplicity.
(b) It is a complex narrative of sin and Ans: (b & d ) Patrick White, the Australian Nobel Prize
redemption which involves both White and winner has pioneered new fictional genres and art forms
Black South Africans. in literature his style is noted for humour, florid prose
(c) The protagonist David Lurie is a priest who and stream of consciousness techniques. Hence, option
brings disgrace to his calling. (b) and (d) both are correct.
(d) Coetzee has a schematic and reductive view Note– NTA has considered both the options (b) and (d)
on the relations between Whites and the as correct answer.
Blacks in South Africa. 13. Conventional scholarship dates 'Early Modern
Ans: (b) J.M. Coetzee’s ‘Disgrace’ (1999) is a complex English' as beginning around
narrative of sin and redemption and revolves around the (a) 450 (b) 1066
life of a South African Professor of English, David (c) 1500 (d) 1800
Lurie in Post-Apartheid South-Africa. It also involves Ans: (c) Modern scholars of English have dated the
both White and Blacks South Africans. birth of early modern English around 1500.
Hence option (b). Hence option (c).
UGC NET/JRF English Paper III, June 2013 113 YCT
14. "Every demon carries within him unknown to Ans: (d) The correct order is –
himself, a tiny seed of self-destruction and goes (III) Anglo Norman → (IV) Early Tudor → (I)
up in thin air at the most unexpected moment." Elizabethan → (II) Caroline
To which of R.K. Narayan's characters the Hence option (d).
above statement applies? 18. Which of the following plays is not written by
(a) Raju - The Guide Rabindranath Tagore?
(b) Jagan - The Sweet Vendor (a) Sacrifice (b) Chandalika
(c) Vasu - Man Eater of Malgudi (c) Muktadhara (d) Eknath
(d) Margayya - The Financial Expert Ans: (d) Tagore has authored Sacrifice, Chandalika and
Ans: (c) The given description applies to Vasu a Muktadhara but not Eknath.
character in R.K. Narayan’s ‘The Man Eater of Hence option (d).
Malgudi’ (1961). 19. Given below are two statements, one is labelled
Hence option (c). as Assertion (A) and the other labelled as
15. Which of the following is not true of post- Reason (R):
structuralism? Assertion (A) : A quarto refers to a text in which
(a) It seeks to undermine the idea that meaning each leaf was a quarter the size of the original
pre-exists its linguistic expression. sheet.
(b) There can be no meaning which is not Reason (R) : Because eight pages of text were
formulated and no language formulation printed on large sheets of paper, which were then
reaches anywhere beyond language. folded four times to produce four leaves.
(c) There is no a-textual 'origin' of a text. In the context of the above statements, which
(d) Every sign refers to every other sign one of the following is correct?
adequately. (a) (A) is correct but (R) is wrong.
Ans: (d) In Post structuralism every sign does not refer (b) Both (A) and (R) are correct.
to every other sign adequately. It is more reflexive than (c) (A) is wrong but (R) is correct.
referential. (d) Both (A) and (R) are wrong.
Hence option (d). Ans: (a) Assertion is true but the Reason is false as in
16. Which of the following statements is not true of Quarto text a sheet is folded twice to form four leaves.
Wole Soyinka's 'The Swamp Dwellers'? Hence option (a).
(a) It talks about the family, the extended family 20. The purpose of the Pre-Raphaelites was
in the African society. primarily to promote
(b) It is a confrontation between the traditional (a) complexity and ambivalence in art and
and modern society. literature.
(c) It talks about the migration of people, (b) simplicity and naturalness in art and
crossing of borders and diasporic anguish. literature.
(d) It is a comment about the city, urban, modern (c) symbolic and classical modes in art and
and the country rural, the swamp, the ancient. literature.
Ans: (c) Wole Soyinka’s ‘The Swamp Dwellers’ (1958) (d) psychological and mythic modes in art and
talks about the extended family in the African society literature.
and comments about the city, urban, rural, swamp and Ans: (b) The purpose of Pre-Raphaelites was primarily
contrasts the modern and ancient society. It does not to promote simplicity and naturalness in art and
talk about diasporic anguish or migration problems literature.
across borders. Hence option (b).
Hence option (c). 21. Which one of the following plays does not use
17. Arrange the following English literary periods the device of the play within the play"?
in the order in which they appeared. Use the (a) Hamlet
codes given below : (b) Women Beware Women
Codes: (c) The Spanish Tragedy
I. Elizabethan II. Caroline (d) A Midsummer Nights' Dream
III. Anglo Norman IV. Early Tudor Ans: (*) Note– NTA has dropped this question and
The correct combination according to the code marks awarded to all appeared candidates.
is All the given plays have 'Play within play' technique.
(a) III, II, IV, I (b) III, IV, II, I The concept of a play within a play is the French saying
(c) II, III, IV, I (d) III, IV, I, II "mise an abyme," or "placed into abyss".
UGC NET/JRF English Paper III, June 2013 114 YCT
22. Given below are two statements, one is labelled 25. Kafka's 'Trial' has all the following
as Assertion (A) and the other labelled as characteristics except :
Reason (R) : (a) Vivid yet surreal
Assertion (A) : In the Absurd plays of Pinter and (b) Dystopian
Beckett, lack of communication seems to be a (c) The use of historical details of setting
predominant theme. (d) The depiction of totalitarian society
Reason (R) : Existentialist philosophy had a Ans: (c) Kafka’s ‘Trial’ (1925) depicts a vivid yet
tremendous influence on the dramatists of the surreal and a dystopian totalitarian society but use of
period, nihilism and meaninglessness of life historical detail is not its characteristic.
taking a front seat. Hence option (c).
In the context of the above statements which 26. Match the following lists :
one of the following is correct. List-I List-II
(a) Both (A) and (R) are true and (R) is the (Phrases from poems) (Titles of poems)
correct explanation of (A). I. "Sound of stick 1. "Byzantium"
(b) Both (A) and (R) are true and (R) is not the upon the floor"
correct explanation of (A). II. "Hade's bobbin 2. "Sailing to
(c) (A) is true but (R) is false. bound in mummy Byzantium"
(d) (A) is false but (R) is true. cloth"
Ans: (a) Both Assertion and Reason are true and (R) is III. "With beauty like 3. "Coole and
the correct explanation of (A). It is a certain fact that a tightened bow" Ballylee, 1931"
existentialist philosophy had a tremendous impact on IV. "A tattered coat 4. "No Second Troy"
the modern dramatists like Pinter and Beckett, whose upon a stick"
plays characterize with nihilism and meaninglessness.
The right combination according to the code is :
Hence option (a). Code :
23. Which of the following observations are true I II III IV
about Beatrice Culleton's 'April Raintree'? (a) 4 1 3 2
I. It is a fictional account of the lives of two (b) 3 2 1 4
metis sisters growing up in Winnipeg. (c) 4 3 2 1
II. April has a darker complexion and identifies (d) 3 1 4 2
herself with Metis population.
Ans: (d) W.B Yeats is the author of all the poems. the
III. The two sisters have been removed from their correct matches of the phrases and poems are–
parents home and placed with a series of
foster families. I. Sound of stick upon the 3. Coole and Ballylee
floor (1931)
IV. Cheryl has a lighter complexion and
identifies herself with white population. II. 'Hade's bobbin bound in 1. Byzantium (1933)
mummy cloth'
The correct combination according to the code
is III.'With beauty like a 4. No Second Troy
(a) I and III are correct. tightened bow' (1916)
(b) I and II are correct. IV.'A tattered coat upon a 2. Sailing to Byzantium
(c) II and III are correct. stick' (1928)
(d) III and IV are correct. Hence, option (d) is correct.
Ans: (a) Beatrice Culleton’s ‘April Raintree’ (1984) 27. Given below are the two statements, one is
presents a fictional account of the lives of two ‘Metis’ labelled as Assertion (A) and the other labelled
sisters, April and Cheryl in Winnipeg and shows how as Reason (R):
they were removed from their parent’s home and placed Assertion (A) : The literature of the Jacobean
with a series of foster families and how they cope up Age is dominated by works revealing symptoms
with it. of melodrama and sensationalism.
Hence option (a) is correct. Reason (R) : The Jacobean Age is generally ruled
by the spirit of decadence.
24. "She dwells with beauty - Beauty that must In the context of the two statements which one of
die", wrote Keats in one of his odes, referring the following is correct.
to (a) Both (A) and (R) are true and (R) is the
(a) Indolence (b) Autumn correct explanation of (A).
(c) Melancholy (d) Psyche (b) Both (A) and (R) are true and (R) is not the
Ans: (c) In his ‘Ode on Melancholy’ (Stanza 3), Keats correct explanation of (A).
wrote the above lines referring to melancholy. (c) (A) is true but (R) is false.
Hence option (c). (d) (A) is false but (R) is true.
UGC NET/JRF English Paper III, June 2013 115 YCT
Ans: (b) Both Assertion and Reason are true but (R) (a) Both (A) and (R) are true and (R) is the
does not explain (A) correctly. (A) is talking about correct explanation of (A).
literature but (R) is talking about the Jacobean Age. (b) Both (A) and (R) are true but (R) is not the
Hence option (b). correct explanation of (A).
28. Which of the following statements best (c) (A) is true but (R) is false.
describes the term "deconstruction"? (d) (A) is false but (R) is true.
(a) It seeks to expose the problematic nature of Ans: (a) Both Assertion and Reason are true and (R) is
'centered' discourses. the correct explanation of (A), as both statements are
(b) It advocates 'subjective' or 'free' talking about ‘bourgeoisie.’
interpretation. Hence option (a).
(c) It emphasizes the importance of historical 32. "A Tun of Man in thy large Bulk is writ.
context. But sure thou'rt but a Kilderkin of wit"
(d) It is a method of critical analysis. In the above lines what does Dryden mean by
Ans: (a) The term ‘deconstruction’ seeks to expose the 'Kilderkin?
problematic nature of ‘centered’ discourses. (a) a trivial instance
Hence option (a). (b) a small barrel of wine
29. Which of these authors is not a writer of (c) kith and kin
African American slave narratives? (d) a small amount, as contrasted with 'tun’
(a) Solomon Northrop Ans: (b) These lines are from Dryden’s poem ‘Mac
(b) Frederick Douglass Flecknoe’ (1692). The word 'Kilderkin' here means a
(c) Phillis Wheatley small barrel of wine.
(d) Sojourner Truth Hence option (b).
Ans: (c) Solomon Northrop, Fredreick Douglass and 33. Which of the following statements is not true of
Sojourner Truth are famous African-American authors Kazuo Ishiguro's 'Remains of the Day'? The
known for slave narratives but Phillis Wheatley is novel
famous for ‘Poems’ (1773). (a) uses a butler as a pivotal character.
Hence option (c). (b) uses the classic English detective story form.
30. "For nature then (c) refers to England in the 1930.
The courser pleasures of my boyish days. And (d) became a very successful film.
their glad animal movements all gone by To me Ans: (b) Kazuo Ishiguro’s ‘Remains of the Day’ (1989)
was all in all". is not a classic English Detective story.
In these lines from "Tintern Abbey Revisited", Hence option (b).
Wordsworth is talking about : 34. "From a Second Space perspective city space
(a) the second stage in his relationship with becomes more of a mental and ideational field,
Nature. conceptualised in imagery, reflexive thought
(b) the first stage in his relationship with Nature. and symbolic representation, a conceived space
(c) both the first and second stages in his of the imagination or what I will henceforth
relationship with Nature. describe as the urban imagery."
(d) the third stage in his relationship with Nature. (Edward Soja, Postmetropolis) Which of the
following statements cannot be applied to
Ans: (b & c) Note– NTA has considered both (b) and
(c) options as correct answer. Soja's proposition on the Second Space?
(a) Second Space perspective tends to be more
In the above lines from the poem 'Tintern Abbey
subjective.
Revisited', Wordsworth is talking about both the first
stage in his relationship with nature and the third stage (b) Second Space perspective is concerned with
in his relationship with nature. symbolic representation of reality.
Hence, option (b) and (c) both are correct. (c) Second Space perspective is concerned with
the fundamentally materialist approach.
31. Assertion (A): One of Flaubert's main
(d) Second Space perspective deals with
motivations in writing the novel Madam Bovary
'thoughts about space.'
was his antipathy for the bourgeoisie.
Ans: (c) The second space perspective in Soja’s
Reason (R): Flaubert strongly believed that
‘Postmetropolis (2000) does not concern with the
bourgeoisie are those who think, feel and act in
fundamentally materialist approach.
terms of utilitarianism and who reject the
humanity and uniqueness of the individual person. Hence option (c).

UGC NET/JRF English Paper III, June 2013 116 YCT


35. "Lightly, O lightly, we bear her along. (c) locating the meaning of a literary work in the
She sways like a flower in the wind of our song; internal relations of the language that
She skims like a bird on the foam of a stream, constitute a text.
She floats like a laugh from the lips of a dream...." (d) evaluating a literary text against a backdrop
These lines occur in the poem of historical events.
(a) "Palanquin bearers" Ans: (c) The basic principle of New Criticism is in
(b) "The Illusion of Love" locating the meaning of a literary work in the internal
(c) "Indian Love Song" relations of the language that constitute a text.
(d) "Cradle Song" Hence option (c).
Ans: (a) The above lines occur in Sarojini Naidu’s 40. Who among the following figures give a
poem ‘Palanquin Bearers’ (1917). preview of Aschenbach's fatal end in 'Death in
Hence option (a). Venice'?
Codes :
36. Which among the following novels of Anita
I. The Graveyard Stranger
Desai is a children's book?
II. The Governess
(a) Fire and The Mountain
III. The barber
(b) Fasting, Feasting
IV. The Gondolier
(c) The Zig zag Way
(d) The Village by the Sea The right combination according to the code is
(a) III and IV are correct.
Ans: (d) Anita Desai’s, ‘The Village by the Sea’ (1982)
(b) I and IV are correct.
is a children’s book.
(c) II and III are correct.
Hence option (d).
(d) I and III are correct.
37. Who among the following writers describes
novels as "not form which you see but emotion Ans: (b) The Graveyard Stranger (I) and The Gondolier
which you feel"? (IV) preview Aschenbach’s fatal end in Thomas Mann’s
‘Death in Venice’ (1912).
(a) D.H. Lawrence (b) Jean Rhys
Hence option (b).
(c) Virginia Woolf (d) Joseph Conrad
41. Jacques Lacan posits three 'orders' which
Ans: (c) In response to Percy Lubbock’s formalist
structure human existence. In the list that
critical approach in ‘The Craft of Fiction’ (1922),
follows: Identify the one that is not included by
Virginia Woolf said, in ‘On Re-Reading Novels’
Lacan :
(1922), that book itself is not form which you see, but
emotion which you feel. (a) Imaginary (b) Unconscious
Hence option (c). (Source: Cambridge Companion to V. (c) Real (d) Symbolic
Woolf) Ans: (b) The Three ‘orders’ posited by Jacques Lacan
are – Imaginary, Real and Symbolic. Unconsciousness
38. In 'Paradise Lost', Milton invokes his
isn’t mentioned in his list.
'Heavenly Muse', 'Urania' at the beginning of :
Hence option (b).
Codes :
42. Given below are two statements, one labelled as
I. Book one II. Book four
Assertion (A) and the other labelled as Reason
III. Book nine IV. Book seven
(R) :
The right combination according to the code is
Assertion (A): Deconstructive reading is
(a) I and II are correct. apolitical.
(b) I, III and IV correct. Reason (R): Because it focuses exclusively on
(c) II and III are correct. language. It primarily holds that all texts or
(d) I and IV are correct. linguistic structures contain within them a
Ans: (d) Milton invokes ‘Urania’ in Book one and principle of destabilisation and hence it is difficult
seven. to pin down meaning. Such a reading, therefore, is
Hence option (d). unable to assign historical agency.
39. Which one of the following best describes the In this context of above statements, identify
basic principle of New Criticism? which one of the following is correct?
(a) an emphasis on the distinctive style and (a) (A) is correct but (R) is wrong.
personality of the authors. (b) Both (A) and (R) are correct.
(b) stressing the virtues of discipline, order and (c) (A) is wrong but (R) is correct.
the ethical mean. (d) Both (A) and (R) are wrong.
UGC NET/JRF English Paper III, June 2013 117 YCT
Ans: (b) Both Assertion and Reason are correct. 46. In relation to Spenser's 'Faerie Queene' which
Hence option (b). of the following character virtue link is rightly
matched?
43. Match the following lists : (a) Justice-Artegall; Courtesy-Guyan;
List -I List-II Temperance-Calidore
(Title of poem) (Poet) (b) Chastity-Britomart; Justice-Guyan;
I. "I hear a fly Buzz" 1. Wallace Stevens Temperance-Talus
II. "Birches" 2. Emily Dickinson (c) Courtesy-Calidore; Temperance-Guyon;
Justice-Artegall
III. "Sunday Morning" 3. Allen Ginsberg
(d) Courtesy-Calidore; Temperance-Artegall;
IV. "A Supermarket in 4. Robert Frost Justice-Britomart
California" Ans: (c) In Spenser’s ‘Faerie Queene’ (1590), Courtesy
The correct combination is : is represented by Calidore, Temperance by Guyon and
Code : Justice by Artegall.
I II III IV Hence option (c).
(a) 2 4 3 1 47. The Divine Comedy is divided into three
(b) 2 1 3 4 canticas, each consisting of
(a) 30 cantos (b) 33 cantos
(c) 2 4 1 3
(c) 24 cantos (d) 28 cantos
(d) 3 2 1 4
Ans: (b) Dante’s ‘Divina Commedia’ or ‘The Divine
Ans: (c) The correct matches are – Comedy’ (1320 wr, 1472 first printed), was divided into
I. ‘I hear a fly Buzz’ (1945) 2. Emily Dickinson three canticas, each divided into 33 cantos. The three
II. ‘Birches’ (1916) 4. Robert Frost canticas were – Hell, Purgatory and Paradise.
III. ‘Sunday Morning’ (1915) 1. Wallace Stevens Hence option (b).
IV. ‘A Supermarket in 3. Allen Ginsberg 48. The Modern Prometheus is the alternative title
of
California’ (1956)
(a) Dracula (b) Frankenstein
Hence option (c). (c) Caleb Williams (d) The Italian
44. 'Lexis' refers to Ans: (b) ‘The Modern Prometheus’ is the alternative
(a) all word forms having meaning or title of Mary Shelley’s ‘Frankenstein’ (1818).
grammatical functions Hence option (b).
(b) the history of words 49. In 'Words Upon Words', Saussure says, "The
(c) study of select word forms actual birth of a new language has never
(d) the selection of words reported in the world" because "we have never
known of a language which was not spoken the
Ans: (a) ‘Lexis’ means all word forms having meaning day before or which was not spoken in the
or grammatical functions. same way the day before". What does he
Hence option (a). mean?
45. The following writers are involved in social (a) Old languages die making way for new ones.
activism in addition to their practice of creative (b) The birth and death of a language are not
subject to human laws.
writing :
(c) Languages do not get borne, they evolve out
Codes : of previously existing linguistic situations.
I. Mahasweta Devi II. Shashi Deshpande (d) Old speech patterns trigger the birth of a new
III. Arundhati Roy IV. Shobha De language.
The correct combination according to the code Ans: (c) In the above lines Saussure means that
is Languages do not get borne but they evolve out of
(a) I and II are correct. previously existing linguistic situations.
(b) III and IV are correct. Hence option (c).
(c) I and III are correct. 50. What did Henry James describe as "Loose
Baggy Monsters"?
(d) II and IV are correct.
(a) Novels (b) The Spaniards
Ans: (c) Mahasweta Devi (I) and Arundhati Roy (III) (c) Epic Poems (d) His trousers
are creative writers who are also involved in social Ans: (a) In 1907, Henry James described the ‘large
activism. The other two are authors only. nineteenth century Novels’ as 'Loose Baggy Monsters'.
Hence option (c). Hence option (a).

UGC NET/JRF English Paper III, June 2013 118 YCT


51. "High above the north pole, on the first day of Ans: (a) By the end of Tennyson’s ‘In Memorium’
1969, two professors of English literature (1849), The speaker re-embraces a Christian vision of
approached each other at a combined velocity after life.
of 1200 miles per hour." Hence option (a).
This is the opening of David Lodge's 56. The system of social rules that a speaker knows
(a) Nice Work about language and uses it is called
(b) Changing Places (a) grammar (b) morphology
(c) Small World (c) orthography (d) pragmatics
(d) The British Museum is Falling Down
Ans: (d) Pragmatics is the system of social rules that a
Ans: (b) The above lines constitute in opening of David speaker knows about language.
Lodge’s ‘Changing Places’ (1975). Hence option (d).
Hence option (b).
57. The term 'ecological imperialism' was coined
52. At the end of 'The Portrait of a Lady' Isabel by
Archer (a) Vandana Shiva (b) Laurence Buell
I. Goes back to the house from the Garden. (c) Paulo Freire (d) Alfred Crosby
II. Accepts the proposal of Casper Goodwood.
Ans: (d) The term ‘ecological imperialism’ was coined
III. Straight away refuses the offer of Goodwood.
by Alfred Crosby.
IV. Probably goes back to Rome and Osmond.
Hence option (d).
Which is the correct combination according to
the code ? 58. Emotional ties and personal relationship play a
minor part in Defoe's works. The following
Codes :
protagonists of Defoe have no family except
(a) I and II are correct.
one who leaves family at an early age. Which is
(b) III and IV are correct. that character?
(c) I and IV are correct. (a) Moll Flanders (b) Colonel Jacque
(d) II and III are correct. (c) Robinson Crusoe (d) Captain Singleton
Ans: (c) At the end of Henry James’s ‘The Portrait of a Ans: (c) Crusoe in Daniel Defoe’s ‘Robinson Crusoe’
Lady’ (1881), Isabel Archer goes back to the house and (1719) leaves his family at an early age to go on a sea
probably to Rome and Osmond. It was left uncertain. adventure and is shipwrecked on an island.
Hence option (c). Hence option (c).
53. "I will put myself in poor and mean attire And 59. Match the following lists :
with a kind of umber smirch my face". The
List -I List-II
word umber means :
(Novels) (Settings)
(a) a dusty yellow or brown pigment
I. The Power and 1. Vietnam
(b) a dark brown pigment
the Glory
(c) light brown powder
II. The Quiet 2. Haiti
(d) yellow paste
American
Ans: (a) The word ‘umber’ means a dusty yellow or
III. The Honorary 3. Paraguay
brown pigment.
Consul
Hence option (a).
IV. The Comedian 4. Mexico
54. Which of the following psychoanalysts rewrote
The right combination according to the code is :
Descarte's dictum: "I think therefore I am" as
'I am not where I think, and I think where I am I II III IV
not'? (a) 4 1 3 2
(a) Lacan (b) Freud (b) 1 2 3 4
(c) Jung (d) Cixous (c) 4 3 2 1
Ans: (a) Descarte’s dictum 'I think therefore I am' was (d) 3 4 1 2
rewritten by Lacan as given above. Ans: (a) The correct settings of Graham Greene’s
Hence option (a). novels are –
55. By the end of 'In Memorium' the speaker I. ‘The Power and the 4. Mexico
(a) re-embraces a Christian vision of after life Glory’ (1968)
(b) re-asserts religious doubts and scientific II. ‘The Quiet American’ (1955) 1. Vietnam
scepticism. III. ‘The Honorary Consul’ (1973) 3. Paraguay
(c) reiterates the Darwinian view of social life. IV. ‘The Comedians’ (1966) 2. Haiti
(d) reaffirms his faith in universal brotherhood. Hence option (a).

UGC NET/JRF English Paper III, June 2013 119 YCT


60. ".............every other stone is god or cousin The right combination according to the code is
there is no crop other than god; and god is (a) I and III are correct.
harvested here around the year." (b) I and IV are correct.
This extract is from : (c) II and III are correct.
(a) Jayanta Mahapatra's "Konarak"
(d) I and II are correct.
(b) Arun Kolatkar's "Jejuri"
(c) P. Lal's "Being Very Simple God" Ans: (b) The intellectual complexity (I) and union of
(d) R. Parthasarathy's "Under Another Sky" thought and passion (IV) in 17th century metaphysical
Ans: (b) The above extract is from Arun Kolatkar’s poetry sparked the enthusiasm of modernist poets and
‘Jejuri’ (1974). critics. Hence option (b).
Hence option (b). 65. The inferior Priestess, at her altar's side,
61. In E.M. Foster's 'A Passage to India' some of Trembling, begins the sacred Rites of Pride.
the major symbols are associated with : In this description of Belinda at the dressing
Codes : table, what does the word Pride refer to?
I. Mountains II. Tigers (a) Vanity
III. Echoes IV. Clouds (b) Pride as the first of man's sins
The right combination according to the code is (c) Both (A) and (B)
(a) I and II are correct. (d) Complacency
(b) I, II and IV are correct.
Ans: (c) In the above description of Belinda in Pope’s
(c) I and III are correct.
‘The Rape of the Lock’ (1712), the word ‘Pride’ refers
(d) II and IV are correct.
to vanity and pride as first of man’s sins.
Ans: (c) In E.M. Forster’s ‘A Passage to India’ (1924)
Hence option (c) Both.
some major symbols are – (I) mountains and echoes;
others are caves and temples. 66. "Cover her face; mine eyes dazzle; she died
Hence option (c). young..... She and I were twins: And should I
62. Which of the following features are present in die this instant, I had lived her time to a
Dostoevsky's 'Crime and Punishment'? minute"
I. Nihilism In the light of the above quotation which of the
II. Utilitarianism following interpretations is not correct?
III. Rationalism (a) The beauty and youth of the Duchess become
IV. Christian Symbolism obvious to Ferdinand when he sees her dead
The correct combination according to the code body.
is (b) Only when he identifies himself with her,
(a) I and II are correct. does he realize the enormity of his crime.
(b) I and IV are correct. (c) When he compares the age of the Duchess
(c) III and IV are correct. with his own and puts himself in her position
(d) I and III are correct. does he realize his guilt?
Ans: (b) In Dostoevsky’s ‘Crime and Punishment’ (d) He wants her face to be covered because it
(1866), Nihilism and Christian Symbolism are present.
reminds him of her infidelity.
Hence option (b).
Ans: (*) Note– NTA has dropped this question and
63. "Count no man happy until he dies, free of
pain at last", is the last line of marks awarded to all appeared candidates.
(a) Oedipus at Colonus (b) Agamemnon These lines have been taken from 'Duchess of Malfi'
(c) Oedipus the King (d) Orestes Ferdinand enters and asks if the Duchess is dead, to
Ans: (c) The above line is the last line of Sophocle's which Bosola responds that she is. Ferdinand says the
‘Oedipus the King.’ death of young wolves is not to be pitied. He stares at
Hence option (c). the Duchess's body, and Bosola asks if this causes him
to weep, since, while other sins speak. Bosola says he
64. What characteristics of 17th century
metaphysical poetry sparked the enthusiasm of thinks she suffered for too long and covers her face.
modernist poets and critics? 67. All except one of the following scholars have
Codes : come up with models which aim to characterise
I. its intellectual complexity world Englishes within one conceptual set.
II. its uncompromising engagement with politics Identify the lone exception.
III. its religious fervour (a) Tom McArthur (b) Noam Chomsky
IV. its union of thought and passion (c) Braj Kachru (d) Manfred Gorlach
UGC NET/JRF English Paper III, June 2013 120 YCT
Ans: (b) All scholars except Noam Chomsky have 72. Which of the following is not true of post-
come up with models to characterize world Englishes structuralism?
within one conceptual set. (a) It seeks to undermine the idea that meaning
Hence option (b). pre-exists its linguistic expression.
68. In the very opening scene of Volpone, the (b) There can be no meaning which is not
protagonist says, "Open the shrine, that I may formulated and no language formulation
see my Saint" By the word 'Saint', Volpone is reaches anywhere beyond language.
referring to (c) There is no a-textual 'origin' of a text.
(a) The Sun (b) Saint Arthur (d) Every sign refers to every other sign
(c) Gold (d) Apollo adequately.
Ans: (c) In the opening scene of Ben Jonson’s Ans: (d) The statement – every sign refers to every
‘Volpone’ (1606), the word ‘Saint’ refers to Gold. other sign adequately, is wrong with reference to
Poststructuralism. The other statements are correct.
Hence option (c).
Hence option (d).
69. A close friend of Dickens objected to the
73. Which of the following second-language
original ending of Great Expectations in which learners would most likely acquire the second
Estella remarries and Pip remains single. language more easily?
Dickens accordingly revised to a more (a) a high school student who has been enrolled
conventional ending which suggests that Pip in mandatory classes in the second language
and Estella will marry. Who was the friend? since elementary school.
(a) Wilkie Collins (b) Thomas Beard (b) a visitor to a country where the second
(c) Thomas Carlyle (d) Richard Bentley language is spoken; he interacts with hotel
Ans: (a) On Wilkie Collins' suggestion, Dickens altered and restaurant personnel using the second
the ending of ‘Great expectations’ (1861). language .
Hence option (a). (c) a business person for whom fluency in the
70. Which of the following statements best second language may lead to career
describes an example of the influence of an advancement.
affective factor on second language (d) an immigrant living in a country where the
acquisition? second language is spoken : he feels accepted
(a) a second language learner makes educated by speakers of the second language .
guesses about word meanings in a text by Ans: (d) An immigrant living in a country where the
recognizing cognates. second language is spoken will acquire the second
(b) a second language learner uses familiar language more easily.
vocabulary to mentally form sentences before Hence option (d).
speaking. 74. In Wuthering Heights, Cathy appears in a
(c) an adult second language learner finds it dream beating at a window, wailing "Let me
impossible to form second language sounds in", blood running down her wrist. Who
that do not occur in his first language . dreams her?
(d) a second language learner employs several (a) Lockwood (b) Nelly
words from the first language when peaking (c) Heathcliff (d) Edgar Linton
the second language but not when writing it. Ans: (a) In Emily Bronte’s ‘Wuthering Heights’ (1847),
Ans: (b) A second language learner uses familiar Lockwood, a tenant there, dreams of Cathy, beating at
vocabulary to mentally form sentences before speaking the window wailing ‘Let me in.’
as an influence on acquisition of second language. Hence option (a).
Hence option (b). 75. Who among the following characters in
71. Marvell's "The Coronet" seeks to explore the Thomas More's 'Utopia' did not correspond in
human condition in terms of the conflict biographical background to an actual
between historical person?
(a) body and soul (b) war and peace (a) Morton (b) Hythloday
(c) nature and grace (d) flesh and spirit (c) Giles (d) More
Ans: (c) Marvell’s ‘The Coronet’ seeks to explore the Ans: (b) Hythloday is the fictional character in More’s
human condition in terms of the conflict between nature ‘Utopia’ (1516 Lat, 1551 Eng.) who is not an actual
and grace. historical person.
Hence option (c). Hence option (b).

UGC NET/JRF English Paper III, June 2013 121 YCT


UGC NET/JRF Exam, June-2013
ENGLISH
Solved Paper-II
Note: This paper contains fifty (50) objective type Ans: (a) W.B. Yeat’s poem ‘Among School Children’
questions, each question carrying two (2) marks. All (1928) employs the ottava rima scheme.
questions are compulsory. Hence option (a).
1. In Pinter's Birthday Party, Stanley is given a 7. John Dryden in his heroic tragedy 'All for
birthday present. What is it? Love' takes the story of Shakespeare's
(a) A toy (b) A piano (a) Troilus and Cressida
(c) A drum (d) A violin (b) The Merchant of Venice
Ans: (c) In Harold Pinter’s ‘The Birthday Party’ (1957) (c) Antony and Cleopatra
Stanley is given a drum as a birthday present. (d) Measure for Measure
Hence option (c). Ans: (c) Dryden’s ‘All for Love’ (1677 wr.) is based on
2. How does Lord Jim end? Shakespeare's ‘Antony and Cleopatra.’
(a) Jim is shot through the chest by Doramin. Hence Option (c).
(b) Jim Kills himself with a last unflinching 8. Arrange the following works in the order in
glance. which they appear. Identify the correct code :
(c) Jim answers "the call of exalted egoism" and I. No Longer at Ease
betrays Jewel.
II. Things Fall Apart
(d) Jim surrenders himself to Doramin.
III. A Man of the People
Ans: (a) Lord Jim is shot through the Chest by Doramin IV. Arrow of God
in Joseph Conrad's 'Lord Jim'. It is a story of a man The correct combination according to the code
named Marlow's struggle to tell and to understand the is :
life story of a man named Jim. It deals with the theme
of fidelity, theme of guilt and atonement. Code :
Hence, option (a) will be correct answer. (a) III, IV, II, I (b) IV, III, I, II
(c) II, I, IV, III (d) I, II, III, IV
3. "Where I lacked a political purpose, I wrote
lifeless books". To which of the following Ans: (c) The correct sequence for Chinua Achebe’s
authors can we attribute the above admission? works are –
(a) Graham Greene (b) George Orwell II. 'Things Fall Apart' - 1958
(c) Charles Morgan (d) Evelyn Waugh I. 'No Longer at Ease' - 1960
Ans: (b) In his essay ‘Why I Write’ dated 1946, George IV. 'Arrow of God' -1964
Orwell admits the above lines. III. 'A Man of the people' - 1966
Hence option (b). Hence option (c) II, I, IV, III.
4. Modernism has been described as being 9. Samuel Pepys kept his diary from
concerned with "disenchantment of our culture (a) 1660 to 1669 (b) 1649 to 1660
with culture itself. Who is the critic? (c) 1662 to 1689 (d) 1660 to 1689
(a) Stephen Spender (b) Malcolm Bradbury Ans: (a) Samuel Pepys kept his diary from 1660-1669
(c) Lionel Trilling (d) Joseph Frank Hence option (a).
Ans: (c) Lionel Trilling criticizes Modernism as being 10. In the Defence of Poetry, what did Sydney
concerned with “disenchantment of our culture with attribute to poetry?
culture itself.” (a) A magical power whereby poetry plays tricks
Hence option (c). on the reader.
5. "Only that film, which fluttered on the grate, (b) A divine power whereby poetry transmits a
Still flutters there, the sole unquiet thing." message from God to the reader.
The above lines are quoted from (c) A moral power whereby poetry encourages
(a) "Tintern Abbey Revisited" the reader to evaluate virtuous models.
(b) "Michael" (d) A realistic power that cannot be made to
(c) "Frost at Midnight" seem like mere illusion and trickery.
(d) "This Lime-Tree Bower, My Prison" Ans: (c) In his ‘Apology or Defence of Poetry’ (1579
Ans: (c) The lines “only that … unquiet thing” have writ, 1595 pub.), Philip Sidney attributed a moral power
been taken from S.T. Coleridge’s ‘Frost at Midnight’ to poetry whereby poetry encourages the reader to
(1798). Hence option (c). evaluate virtuous models. Hence option (c).
6. Which one of the following modern poems 11. An Epistle to Dr. Arbuthnot presents portraits
employs ottava rima ? of the following contemporary individuals :
(a) "Among School Children" (a) Addison and Lord Hervey
(b) "In Praise of Limestone" (b) Dryden and Rochester
(c) "The Wild Swans at Coole" (c) Swift and Steele
(d) "The Shield of Achilles" (d) Smollett and Defoe
UGC NET/JRF English Paper II, June 2013 122 YCT
Ans: (a) Alexander Pope’s ‘An epistle to Dr. 16. "Power circulates in all directions, to and from
Arbuthnot’ (1735) presents the portraits of Addison and all social levels, at all times." Who said this?
Lord Hervey who were his famous contemporaries. (a) Edward Said (c) Jacques Derrida
Hence option (a). (b) Michel Foucault (d) Roland Barthes
12. Match the following authors with their works : Ans: (b) In an interview ‘Truth and Power’ Michael
List -A List-B Foucault said – “Power circulates in all directions…”
(Authors) (Works) Hence option (b).
I. Alice Walker 1. Invisible Man 17. Which one of the following is not an Australian
II. Ralph Ellison 2. The Color Purple Aboriginal writer?
III. Richard Wright 3. Their Eyes Were (a) Kath Walker (b) Peter Carey
Watching God (c) Robert Bropho (d) Jack Davis
IV. Zora Neale 4. Native Son Ans: (*) Kath Walker is an Australian aboriginal writer
Hurston and political activist whose another name is Oodgeroo
Which is the correct combination according to Noonuccal, while Peter Carey was born in Bacchus
the code : Marsh, Victoria in 1943, he is an Australian novelist.
Code : Robert Bropho was Balladrong Noongar Australian
I II III IV Aboriginal writer, while Jack Leonard Davis was an
(a) 2 1 4 3 Australian Aboriginal playwright and poet. Aboriginal
people were traditionally hunters and gatherer who did
(b) 3 4 2 1 not live in one place. Australian aboriginal people
(c) 4 3 1 2 originally came from Asia via insular Southeast Asia.
(d) 1 2 4 3 Dropped by UGC and marks awarded to all appeared
Ans: (*) 'Native Son' is a novel written by an American candidates.
author Richard Wright. It is about an old Bigger 18. Sir Thomas Wyatt and the Earl of Surrey
Thomas, while 'Their Eyes were Watching God' is a jointly brought out Tottel's Miscellany during
novel written by Zora Neale Hurston and it comes under the Renaissance. Identify the name of the Earl
the Harlem Renaissance. 'Color Purple' is an epistolary of Surrey from the following :
novel by Alice Walker which depicts the growing up (a) Thomas Lodge (b) Thomas Nashe
and self-realization of Celie who overcomes oppression
and abuse to find fulfillment and independence. (c) Thomas Sackville (d) Henry Howard
'Invisible Man' is a novel written by Ralph Ellison. Ans: (d) Sir Thomas Wyatt and Henry Howard, the Earl
Dropped by UGC and marks awarded to all appeared of Surrey brought out 'Tottel’s Miscellany' in 1557
candidates. consisting of 310 songs and sonnets by various
contributors.
13. Which of these plays by Shakespeare does not
use 'cross-dressing' as a device? Hence option (d).
(a) As You Like it 19. Match the following lists :
(b) Julius Caesar List -I List-II
(c) Cymbeline (Novelists) (Novels)
(d) Two Gentlemen of Verona I. Margaret 1. Surfacing
Ans: (b) Shakespeare’s ‘Julius Caesar’ doesn’t use Laurence
cross dressing as a device. II. Margaret 2. The Stone Angel
Hence option (b). Atwood
III. Sinclair Ross 3. Medicine River
14. Which of the following works cannot be
categorized under postcolonial theory? IV. Thomas King 4. As for Me and My
(a) Nation and Narration House
(b) Orientalism Which is the correct combination according to
(c) Discipline and Punish the code :
(d) White Mythologies Code :
I II III IV
Ans: (c) Michael Foucault’s ‘Discipline and Punish: (a) 1 4 3 2
The Birth of the Prison’ (1975) is a book on western
penal systems. (b) 3 2 1 4
Hence option (c). (c) 4 3 2 1
(d) 2 1 4 3
15. Locke's Essay Concerning Human
Understanding is a classic statement of _______ Ans: (d) The correct matches are –
Philosophy. I. Margaret Laurence 2. ‘The Angel’(1964).
(a) Aesthetic (b) Empiricist II. Margaret Atwood 1. ‘Surfacing’ (1972).
(c) Nationalist (d) Realist III. Sinclair Ross 4. ‘As for Me and My
Ans: (b) Locke’s ‘An Essay Concerning Human House’ (1941).
Understanding’ (1905) is a classic statement of IV. Thomas King 3. ‘Medicine River’
empiricist philosophy. (1984).
Hence option (b). Hence option (d) 2,1, 4, 3.

UGC NET/JRF English Paper II, June 2013 123 YCT


20. The dramatic structure of Restoration The correct combination according to the code
comedies combines in it the features of is:
I. The Elizabethan Theatre Code:
II. The Neoclassical Theatre of ltaly and France I II III IV
III. The Irish Theatre (a) 3 1 4 2
IV. The Greek Theatre (b) 2 4 1 3
The correct combination according to the code (c) 4 1 2 3
is : (d) 3 2 1 4
Code: Ans: (a) The correct matches are –
(a) I and IV are correct. I. Formalism 3. Victor Shklovsky
(b) III and IV are correct.
II. New Critics 1. John Crowe Ransom
(c) II and III are correct.
III. Psychological Theory of 4. I. A. Richards
(d) I and II are correct.
the Value of Literature
Ans: (d) The dramatic structure of Restoration IV. Literary Art as 2. The Jungians
Comedies combines in it the features of the Elizabethan
Theatre and the Neoclassical Theatre of Italy and Archetypal Image
France. These theatres differed in that it had a main Hence option (a) 3, 1, 4, 2.
platform, an inner stage, and an upper stage level that 26. In the late seventeenth century a "Battle of
made movement possible in all directions instead of Books" erupted between which two groups?
simply along the length of a narrow stage. (a) Cavaliers and Roundheads
Hence, option (d) will be correct answer. (b) Abolitionists and Enthusiasts for slaves
21. Which American poet wrote: "I sound my (c) Champions of Ancient and Modern Learning
barbaric yawp over the roofs of the world"? (d) The Welsh and the Scots
(a) Robert Lowell (b) Walt Whitman Ans: (c) 'The Battle between the Ancient and the
(c) Wallace Stevens (d) Langston Hughes Modern Books in St. James' Library' (1704) is a book
Ans: (b) Walt Whitman in his ‘Song of Myself’ (pub. by Swift which described the battle between the
first in 1855 in ‘Leaves of Grass’ collection and in 1881 champions of ancient and modern learning.
edition with this title ‘Song of Myself’), wrote “I sound Hence option (c).
my barbaric yawp over the roofs of the world.” 27. "Everything that man esteems endures a
Hence option (b). moment or a day
22. The etymological meaning of the word "trope" Love's pleasure drives his love away..."
is In the above quote the last line is an example of
(a) gesture (b) turning (a) allusion (b) pleonasm
(c) mirror (d) desire (c) paradox (d) zeugma
Ans: (b) The etymological (study of origin) meaning of Ans: (c) This is a case of paradoxical statement which
‘trope’ is turning or that which turns. Hence option (b). means a contradictory statement. How can 'love's
23. Who among the following English poets pleasure' drive his ‘love away’? There is a
defined poetic imagination as "a repetition in contradiction.
the finite mind of the eternal act of creation in Hence option (c).
the infinite 'I AM"?
(a) Blake (c) Coleridge 28. Match the author with the work :
(b) Wordsworth (d) Shelley List -I (Authors) List-II (Works)
Ans: (c) S.T. Coleridge in his ‘Biographia Literaria’ I. Kingsely Amis 1. Saturday and
(1817) defined poetic imagination as a “repetition in the Sunday Morning
infinite mind of the eternal act of creation in the infinite II. Allan Sillitoe 2. The Golden
‘I am.” Hence option (c). Note Book
24. Little Nell is a character in Dickens' III. Doris Lessing 3. The Left Bank
(a) David Copperfield IV. Jean Rhys 4. Lucky Jim
(b) The Old Curiosity Shop Which is the correct combination according to
(c) Bleak House the code :
(d) Great Expectations Code:
Ans: (b) Little Nell is a character in Dickens ‘The Old I II III IV
Curiosity Shop’ (1841). Hence option (b). (a) 3 4 1 2
25. Match the following : (b) 4 1 2 3
List -A List-B (c) 2 3 1 4
(Schools/ (Critics) (d) 1 2 3 4
Concept Criticism) Ans: (b) The correct matches are –
I. Formalism 1. John Crowe Ransom I. Kingsley Amis 4. ‘Lucky Jim’ (1954)
II. New Critics 2. The Jungians II. Allan Sillitoe 1. 'Saturday and Sunday
III. Psychological 3. Victor Shklovsky Morning' (1958)
Theory of the III. Doris Lessing 2. ‘The Golden Notebook’
Value of Literature (1962)
IV. Literary Art as 4. I. A. Richards IV. Jean Rhys 3. ‘The Left Bank’ (1927)
Archetypal Image Hence option (b) 4, 1, 2, 3.

UGC NET/JRF English Paper II, June 2013 124 YCT


29. In which of Hardy's novels does the character 34. Which of the following characters from Eliot's
Abel Whittle appear? Waste Land is not correctly mentioned?
(a) Far from the Madding Crowd (a) The typist
(b) The Return of the Native (b) Madam Sosostris
(c) A Pair of Blue Eyes (c) The Merchant from Eugenides
(d) The Mayor of Casterbridge (d) The Young Man Carbuncular
Ans: (d) Abel Whittle appears in Hardy’s novel ‘The Ans: (c) In T.S. Eliot’s ‘The Waste Land’ (1922)
Mayor of Casterbridge’ (1886).
Mr. Eugenides is a smyrna merchant.
Hence option (d).
Hence option (c).
30. The phrase "dark Satanic mills" has become
the most famous description of the force at the 35. Which one of the following best describes the
centre of the industrial revolution. The phrase general feeling expressed in literature during
was used by the last decade of the Victorian era?
(a) William Wordsworth (a) Studied melancholy and aestheticism
(b) William Blake (b) The triumph of science and morbidity
(c) Thomas Carlyle (c) Sincere earnestness and Protestant zeal
(d) John Ruskin (d) Raucous celebration combined with paranoid
Ans: (b) William Blake used the phrase ‘dark, Satanic interpretation
mills’ in his poem ‘Jerusalem.’ Ans: (a) The general feeling towards the last decade of
Hence option (b). Victorian era was of studied melancholy and
31. "Five miles meandering with a mazy motion aestheticism.
Through wood and dale the sacred river ran." Hence option (a).
Where does this ‘sacred river’ directly run to? 36. Which poem by Shelley bears the alternative
(a) A lifeless ocean title, "The Spirit of Solitude"?
(b) The caverns measureless (a) Mont Blanc
(c) A fountain (b) "Hymn to Intellectual Beauty"
(d) The waves (c) "Adonais"
Ans: (a & b) According to the context of the given (d) Alastor
stanza, 'sacred river' directly runs through a lifeless
ocean. The caverns measureless in the option justifies Ans: (d) Shelley’s ‘Alastor’ (1816) bears the alternative
the required answer. title of ‘The Spirit of Solitude.’
Hence, option (a) & (b) will be correct answer. Hence option (d).
32. Who is the twentieth century poet, a winner of 37. Which tale in The Canterbury Tales uses the
the Nobel Prize for literature who rejected the tradition of the Beast Fable?
label "British" though he has always written in (a) The Knight's Tale (b) The Monk's Tale
English rather than his regional language? (c) The Nun's Tale (d) The Miller's Tale
(a) Douglas Dunn (b) Seamus Heaney Ans: (c) ‘The Nun’s Priest’s Tale’ in Chaucer’s ‘The
(c) Geoffrey Hill (d) Philip Larkin Canterbury Tales’ uses the tradition of the Beast Fable.
Ans: (b) Seamus Heaney was awarded the Nobel Prize Hence option (c).
for literature in 1995. He had rejected the ‘British’
Label although he wrote in English. 38. At the end of Sons and Lovers Paul Morel
Hence option (b). (a) sets off in quest of life away from his mother.
33. Which of the following statements best (b) considers the option of committing suicide.
describes Sir Thomas Browne's 'Religio (c) joins his elder brother William in London.
Medici'? (d) embraces a Schopenhauer-like nihilism.
(a) It is a story of conversion or providential Ans: (a) At the end of D.H. Lawrence’s ‘Sons and
experiences. Lovers’ (1913) Paul Morel sets off in quest of life away
(b) It emphasizes Browne's love of mystery and from his mother.
wonder. Hence option (a).
(c) It is full of angst, melancholy and dread of
death. 39. When you say "I love her eyes, her hair, her
nose, her cheeks, her lips" you are using a
(d) It reports the facts of Browne's life.
rhetorical device of
Ans: (a & b) Sir Thomas Browne's 'Religio Medici',
(a) Enumeration (b) Antanagoge
moves around the importance of separating one's
attitudes on science and religion. It is a story of (c) Parataxis (d) Hypotaxis
conversion or providential experiences and emphasizes Ans: (a) In the above case ‘Enumeration’ (or listing)
Browne's love of mystery and wonder. device is being used.
Hence, option (a) & (b) will be correct answer. Hence option (a).

UGC NET/JRF English Paper II, June 2013 125 YCT


40. The following are two lists of plays and The correct combination according to the code
characters. Match them. is
List-I List-II Code :
(Plays) (Characters) (a) I and IV are correct.
I. Women Beware 1. Malevole (b) I and III are correct.
Women (c) I and II are correct.
II. The Malcontent 2. Beatrice (d) II and III are correct.
III. The City Madam 3. Bianca Ans: (b) Moloch and Belial reappear in Milton’s
IV. The Changeling 4. Doll Tearsheet ‘Paradise Lost’ book two.
Which is the correct combination according to Hence option (b).
the code :
Code : 45. When Chaucer describes the Friar as a "noble
pillar of order", he is using
I II III IV
(a) 3 1 4 2 (a) irony (b) simile
(b) 2 1 3 4 (c) understatement (d) personification
(c) 1 2 3 4 Ans: (a) There is irony in Chaucer’s statement when he
(d) 4 3 2 1 describes the Friar as a ‘noble pillar of order’ but is
actually criticising the Friar for his malpractices.
Ans: (a) The correct matches are –
I. T. Middleton’s ‘Women 3. Bianca Hence option (a).
Beware Women’ (1657) 46. John Osborne's 'Look Back in Anger' is an
II. J. Marston’s ‘The 1. Malevole example of
Malcontent’ (1603) (a) drawing room comedys
III. P. Massinger’s ‘The City 4. Doll Tearsheet (b) kitchen-sink drama
Madam’ (1632) (c) absurd drama
IV. T. Middleton and 2. Beatrice (d) melodrama
W. Rowley’s ‘The Changeling’ Ans: (b) John Osborne’s 'Look Back in Anger' (1956) is
(1653) an example of kitchen-sink drama, a movement in
Hence option (a) 3, 1, 4, 2. 1950s and 60s in which the protagonists were described
41. With Bacon the essay form is as angry young men who portrayed anger against
(a) an intimate, personal confession society. Hence option (b).
(b) witty and boldly imagistic 47. Which character in Jane Eyre uses religion to
(c) the aphoristic expression of accumulated justify cruelty?
public wisdom (a) Blanche Ingram (b) Mr. Brocklehurst
(d) homely and vulgar (c) Sir John Rivers (d) Eliza Reed
Ans: (c) Bacon’s essays are rich in aphoristic Ans: (b) Mr. Brocklehurst in Charlotte Bronte’s ‘Jane
expression of accumulated public wisdom. Eyre’ (1847) used religion to justify cruelty.
Hence option (c). Hence option (b).
42. Evelyn Waugh's Trilogy published together as 48. Which Romantic poet defined a slave as 'a
Sword of Honour is about person perverted into a thing'?
(a) The English at War (a) Blake (b) Coleridge
(b) The English Aristocracy (c) Keats (d) Shelley
(c) The Irish question Ans: (b) S. T. Coleridge defined a slave as ‘a person
(d) Scottish nationalism perverted into a thing.
Ans: (a) Evelyn Waugh’s trilogy ‘Sword of Honour’ Hence option (b).
(1952, 55, 61) is about the English at war. 49. John Suckling belongs to the group of
Hence option (a). (a) Metaphysical poets (b) Cavalier poets
43. Who coined the phrase "The Two Nations" to (c) Neo-classical poets (d) Religious poets
describe the disparity in Britain between the
Ans: (b) John Suckling belongs to the group of Cavalier
rich and the poor?
poets along with R. Herrick, R. Lovelace and T. Carew.
(a) Charles Dickens (b) Thomas Carlyle Hence option (b).
(c) Benjamin Disraeli (d) Frederick Engels
50. Sir Thomas More creates the character of a
Ans: (c) Benjamin Disraeli coined the phrase ‘The Two
Nations’ to describe the disparity in Britain between the traveller into whose mouth the account of
rich and the poor. Utopia is put. His name is
Hence option (c). (a) Michael (c) Henry
44. Milton introduces Satan and the fallen angels (b) Raphael (d) Thomas
in the Book I of 'Paradise Lost'. Two of the Ans: (b) In Sir Thomas More’s ‘Utopia’ (1516 Lat.
chief devils reappear in Book II. They are 1551 Eng.) he narrates the story through the character of
I. Moloch II. Clemos Raphael Hythloday.
III. Belial IV. Thamuz Hence option (b).
UGC NET/JRF English Paper II, June 2013 126 YCT
UGC NET/JRF Exam, December-2013
ENGLISH
Solved Paper-III
Note : This paper contains seventy five (75) iii. Sir Charles Lyell is 4. ‘The Principles of
objective type questions of two (2) marks each. All the author of Geology’ (1830-3)
questions are compulsory. iv. Sir James George 2. ‘The Golden Bough’
1. In which of the following novels Harikatha is Frazer authored (1890)
strategically used as a medium of Hence option (d) is correct.
'consciousness raising'?
4. Which of the following poems DOES NOT
(a) Waiting for the Mahatma begin in the first person pronoun?
(b) The Serpent and the Rope (a) Shelley's "Adonais"
(c) A Bend in the Ganges (b) Byron's "Don Juan"
(d) Kanthapura (c) Keats's "Lamia"
Ans: (d) In Raja Rao’s ‘Kanthapura’ (1938), Harikatha (d) Coleridge's 'The Aeolian Harp'
is used as a medium of consciousness raising.
Ans: (c) Shelley’s ‘Adonais’ begins as –
Hence option (d) is correct.
“I weep for Adonais (Keats) – he is dead!”
2. Identify the text in the following list which Byron’s 'Don Juan: Dedication' begins as-
offers a fictionalized survey of English “Bob Southey! You're a poet (Poet Laureate). And
Literature from Elizabethan times to 1928 : representative of all the race.”
(a) E.M. Forster, The Eternal Moment
Coleridge's ‘The Aeolian Harp’ begins as- “My pensive
(b) Virginia Woolf, Orlando Sara! Thy soft cheek reclined, Thus on mine arm”. And
(c) Robert Graves, Goodbye to All That Keats’ ‘Lamia’ (1820) begins as – “Upon a time, before
(d) David Jones, In Parenthesis the faery broods”. 'Upon' is not a pronoun.
Ans: (b) Virginia Woolf’s ‘Orlando: A Biography’ Hence option (c) is correct.
(1928) offers a fictionalized survey of English 5. In his 'Anatomy of Melancholy' Robert Burton
Literature from Elizabethan times to 1928. It is one of proposes the following two principal kinds :
Woolf’s most popular satirical novels which describes I. Love II. Death
the adventures of a poet who changes sex from man to
III. Spiritual IV. Religious
woman and live for centuries meeting the key historical
The correct combination according to the code
figures in one lifetime.
is :
Hence option (b) is correct.
(a) I and II are correct.
3. Match List–I with List–II according to the code (b) I and III are correct.
given below :
(c) I and IV are correct.
List–I List–II
(d) II and IV are correct.
i. John Ruskin 1. London Labour and
Ans: (c) In his ‘Anatomy of Melancholy’ (1621) R.
the London Poor
Burton proposed the Love melancholy and Religious
ii. Henry Mayhew 2. The Golden Bough melancholy.
iii. Sir Charles Lyell 3. Unto This Last
Hence option (c) is correct.
iv. Sir James George 4. The Principles of
6. Listed below are some English journals widely
Frazer Geology
read by professionals :
Codes :
Screen, Critical Quarterly, Review of English,
i ii iii iv Wasafiri.
(a) 3 2 1 4 One of the above founded by C.B. Cox, and now
(b) 2 1 3 4 being edited by Colin MacCabe, carries not only
(c) 2 3 4 1 critical and scholarly essays in English Studies but
(d) 3 1 4 2 reviews film, culture, language and contemporary
Ans: (d) political issues. Identify the journal:
i. John Ruskin is the 3. ‘Unto this Last’ (1860) (a) Wasafiri
author of (b) Screen
ii. Henry Mayhew’s 1. ‘London Labour and the (c) Critical Quarterly
London Poor’ (1851) (d) Review of English Studies

UGC NET/JRF English Paper III, December 2013 127 YCT


Ans: (c) ‘Critical Quarterly’ was founded in 1958 by Ans: (*) Open form poetry has certain characteristics,
C.B. Cox and A.E. Dyson and is now being edited by as there is no use of regular rhythmic patterns, varying
Colin MacCabe . line lengths, no set line groupings and usually
Hence option (c) is correct. unrhymed. Among the given options, 'Free Verse' is
7. In Marvell's "A Dialogue between Soul and closely associated with the definition of open form
Body", who/which of the following has the last poetry but it can not justify the right answer.
word? 12. The author of the book observes "I have
(a) Body (b) God attempted, through the medium of biography,
(c) Soul (d) Satan to present some Victorian visions to the
modern eye".
Ans: (a) In Marvell’s ‘A Dialogue between soul and
The four main characters in this book are
Body’, Body has the last word.
Cardinal Manning, Florence Nightingale, Dr.
Hence option (a) is correct. Arnold and General Gordon. Who is this
8. In Blake's poem "A Poison Tree" the speaker's author?
anger grows and becomes ______. (a) Mathew Arnold (b) Robert Browning
(a) a cherry (b) an apple (c) Lytton Strachey (d) Oscar Wilde
(c) an orange (d) a rose Ans: (c) Cardinal Manning, Nightingale, Dr. Arnold
Ans: (b) In Blake’s poem ‘A Poison Tree’ (1794) the and Gen. Gordon are characters in Lytton Strachey's
speaker's anger grows and becomes an apple. ‘Eminent Victorians’ (1918).
Hence option (b) is correct. Hence option (c) is correct.
9. Given below are two statements, one labelled as 13. In his attack delivered on the theatre in 'A
Assertion (A) and the other as Reason (R) : Short View of the immorality and Profaneness
Assertion (A): For deconstructive critics how of the English Stage', Jeremy Collier specially
human beings read and interpret signs they arraigned_____ and _____.
receive will determine their modes of knowing (a) Congreve and Vanbrugh
and being, whether those signs come in the form (b) Farquhar and Vanbrugh
of literary texts or bank statements. (c) Wycherley and Farquhar
Reason (R): The fact of the matter is that human (d) Congreve and Etherege
beings use signs to function in the world and are Ans: (a) In his 'A Short View of the Immorality and
always likely to do so. Profaneness of the English Stage' (1698), Jeremy
In the context of the two statements, which one Collier specially arraigned (accused) Congreve and
of the following is correct? Vanbrugh apart from Wycherly, Dryden and D. Urfey.
(a) Both (A) and (R) are true and (R) is the Hence option (a) is correct.
correct explanation of (A). 14. I.A. Richards' 'Practical Criticism' (1929)
(b) Both (A) and (R) are true and (R) is not the inaugurated a new phase in the history of
correct explanation of (A). English critical thought. What was this book's
(c) (A) is true, but (R) is false. subtitle?
(d) (A) is false, but (R) is true. (a) Studies in Poetry
Ans: (a) The Reason correctly explains the Assertion in (b) A Study in Literary Judgement
the above statements. (c) Essays and Studies
Hence option (a) is correct. (d) A Theoretical Guide
10. Ian McEwan's 'Saturday' spans one day in the Ans: (b) The subtitle of I.A. Richard’s 'Practical
life of Criticism' (1929) is 'A study in Literary Judgement'.
(a) a divorce lawyer Hence option (b) is correct.
(b) an ageing pianist 15. Which of the following arrangements is in the
(c) a London neurosurgeon correct chronological sequence?
(d) a famous poet (a) The Castle of Otranto – Melmoth the
Ans: (c) Ian McEwan’s ‘Saturday’ (2009) spans one Wanderer – The Monk – The Mysteries of
day in the life of a London neurosurgeon. Udolpho
(b) The Castle of Otranto – The Mysteries of
Hence option (c) is correct.
Udolpho – The Monk – Melmoth the
11. "Open Form" as applied to poetry, is the same Wanderer
as _____. It is poetry that is not written (c) The Mysteries of Udolpho – The Castle of
according to traditional fixed patterns. (Fill up) Otranto – The Monk – Melmoth the
(a) Blank verse Wanderer
(b) Concrete poetry (d) Melmoth the Wanderer – The Castle of
(c) L = A = N = G = U = A = G = E poetry Otranto The Mysteries of Udolpho – The
(d) Free verse Monk
UGC NET/JRF English Paper III, December 2013 128 YCT
Ans: (b) The correct chronological sequence is – [Note: Georgian movement is associated with Rupert
1764 – H. Walpole’s ‘The Castle of Otranto.’ Brooke and not Stopford Brooke].
1794 – Ann Radcliffe’s ‘The Mysteries of Udolpho.’ Hence option (b) is correct.
1796 – M.G. Lewis' ‘The Monk.’ 18. During the colonial era, the British used to call
1820 – C. Maturin’s ‘Melmoth the Wanderer’. the Indian Languages 'vernaculars'. We do not
Hence option (b) is correct. use this word for our bhashas because:
16. Select from among the following plays, the one I. we consider English to be equally vernacular.
that best suits the description below : II. 'verna' is, literally a home-born slave.
I. Alyque Padamsee invited its author to write it. III. not all Indian languages are languages of the
II. The play had communalism as its theme. Indo-European family, and therefore not all
III. This play was banned from the Deccan are vernacular.
Herald Theatre Festival for dealing with a IV. the natives of India were never slaves.
sensitive issue. (a) IV (b) II and IV
IV. The play, however, was produced by Playpen (c) III (d) I and III
in Bangalore on July 1993. Ans: (b) We do not call our Bhasas vernaculars because
The play is___. verna literally means a home-born slave and we never
(a) Dance Like a Man were slaves of anyone. Our Pali and Sanskrit predate
(b) Where There's a Will most of the European languages. So, statements II and
(c) Final Solutions IV are correct.
(d) The Wisest Fool on Earth Hence option (b) is correct.
Ans: (c) The above description is of the play ‘Final 19. More's 'Utopia' displays strong influence of
Solutions’ by Mahesh Dattani which was published in I. The Arthurian legends
1993. II. Plato's Republic
Hence option (c) is correct. III. Amerigo Vespucci's account of the travels
17. I have known three generations of John IV. The teachings of John Wycliffe
Smiths. The type breeds true. John Smith II The correct combination according to the code
and III went to the same school, university and is :
learned profession as John Smith I. Yet John (a) I and III are correct.
Smith I wrote pseudo-Swinburne; John Smith (b) II and III are correct.
II wrote pseudo-Brooke; and John Smith III is (c) II and IV are correct.
now writing pseudo-Eliot. But unless John (d) I and IV are correct.
Smith can write John Smith, however
unfashionable the result, why does he bother to Ans: (b) More’s ‘Utopia’ (1516 Latin, 1551 eng.)
write at all? Surely one Swinburne; one displays strong influence of Plato’s ‘Republic’ and
Brooke, and one Eliot are enough in any age? Amerigo Vespucci’s account of travels.
(Robert Graves, "The Poet and His Public") Hence option (b) is correct.
1. Graves is critical of blind adulation and 20. By 'language transfer' is meant
imitation of successful poets. (a) Knowledge generated in the development of
2. Graves is critical of blind conformity to a learner on account of other domains of
standards set by Swinburne, Brooke, and knowledge.
Eliot. (b) The carryover of rules of the mother tongue
3. Swinburne, Brooke, and Eliot represent the syntax, phonology, or semantic system to the
movements: Second language in question.
4. The poets in question are Algernon Charles (c) The carryover of rules of the Second
Swinburne, Stopford Brooke, and Thomas language syntax, to the mother tongue in
Stearns Eliot. question.
(a) Only 1 and 2 are correct. (d) The vocabulary and sentence-structure
(b) Only 4 is incorrect. transferred haphazardly during Second
(c) Only 3 and 4 are correct. language acquisition from any other language
(d) Only 3 is incorrect. accessed by the learner.
Ans: (b) Robert Graves is talking about adulation Ans: (b) Language transfer means the carryover of rules
(flattery) or imitation of the style of the three poets – A. of the mother tongue syntax, phonology, or semantic
C. Swinburne, Rupert Brooke and T.S. Eliot. Statements system to the second language in question.
1, 2, 3 are correct and only 4 is incorrect. Hence option (b) is correct.

UGC NET/JRF English Paper III, December 2013 129 YCT


21. Which of the following descriptions is NOT Ans: (c) The above lines have been taken from Henrik
true of Peter Carey's 'The True History of the Ibsen’s ‘A Doll’s House’ (1879) and the characters are
Kelly Gang'? Helmer and Nora.
(a) It is an epistolary novel. Hence option (c) is correct.
(b) It has such characters as Edward Kelly, his
mother, and his wife. 25. The following statements relate to the early
(c) It is also about the Bush and the frontier. history of the English language. Identify the set
(d) The novel is dedicated to Edward Kelly's that gives INCORRECT statements:
father. 1. English has borrowed words such as sky,
Ans: (d) Peter Carey’s 'True History of the Kelly Gang' give, law, and leg from Norse.
(2000) won Booker Prize in 2001. It is a fiction- 2. English has also borrowed some pronouns
historical novel based on Ned Kelly’s story. It is not like they, their, them from Norse.
dedicated to Edward Kelly’s father. 3. In grammar, Modern English is much more
Hence option (d). highly inflected than Old English.
22. Identify the poem that opens with the lines : 4. After the Norman Conquest, French became
"I walk through the long schoolroom questioning; the language of the court, the language of
A kind old nun in a white hood replies;" nobility and polite society, and literature.
The children learn to cipher and to sing... 5. Following the Norman Conquest, French
(a) "Among the Schoolchildren" virtually replaced English as the language of
(b) "Among School Children" the people.
(c) "A Man Young and Old" 6. Among the French words that came into
(d) "The Man Young and Old" English are: study, logic, grammar, noun, etc.
Ans. (b) W.B. Yeats’ poem ‘Among School Children’ (a) 1, 2, 3 (b) 3, 5
(1928) opens with the above mentioned lines. (c) 4, 5, 6 (d) 2, 4
Hence option (b) is correct. Ans: (b) Statement 3 is wrong because Modern English
23. Which of the following statements is NOT true is free from inflection found in old English; and
of Foucault's position in 'History of Sexuality'? statement 5 is wrong because after Norman Conquest
(a) Modern sexuality is produced through and as for approx 200 years French was the official language
discourse. of the court but Anglo – Saxon was of the people.
(b) The proliferation of modern discourses of Hence option (b) is correct.
sexuality is more striking than their
suppression. 26. Choices of linguistic forms in using a language,
(c) To write historically about sexuality involves or how a language is actually spoken/written,
increasingly direct, immediate knowlege or especially one that differs from its prescribed
understanding of an unchanging sexual grammar, is called
essence. (a) Utterance (b) Use
(d) Modern sexuality is intimately entangled (c) Usage (d) Deviation
with the historically distinctive contexts and Ans: (c) How a language is actually written or spoken
structures now called 'knowledge'. or choice of linguistic forms comes under usage.
Ans: (c) Statement (c) is not true of Foucault’s position Hence option (c) is correct.
in ‘History of sexuality’ (1976).
Hence option (c). 27. Jamaica Kincaid's narrative 'A Small Place'
(a) is all about learning Farsi and meeting young
24. The following is an exchange between two
characters, husband and wife, in a famous people in modern Iran.
play. The lines appear at the very end of an (b) is an essay that discusses the politics of
emotionally-charged sequence of the last scene: tourism and other neo-colonial modes of
"... I've stopped believing in miracles." foreign intervention.
"But I'll believe. Tell me! (c) is a collection of tiny narratives about gender
Transform ourselves to the point that ...?" relations and includes stories concerning the
"That our living together could be a true Sumerian goddess Inanna.
marriage" (d) a novella that looks unblinkingly at marital
(She goes out down the hall.) ceremonies and maternity in Antigua.
Which play? Name the characters. Ans: (b) In her narrative ‘A Small Place’ (1988),
(a) 'Othello'. Othello, Desdemona Jamaica Kincaid expresses her opinions about politics
(b) 'Sure Thing'. Bill Betty of tourism and foreign intervention in post-colonial
(c) 'A Doll's House'. Helmer, Nora Antigua.
(d) 'Death of a Salesman'. Willy, Linda Hence option (b) is correct.

UGC NET/JRF English Paper III, December 2013 130 YCT


28. Identify the correctly-matched poets and their (c) Waverley (in his first novel of that name) is a
works from the following : model hero for the protagonists of Scott's
(a) Nissim Ezekiel – 'Hymns in Darkness', novels.
Kamala Das –'The Sirens', R. Parthasarthy – (d) Scott started his novel-writing career in his
'Rough Passage', A.K. Ramanujan – 'The 43rd year with the novel, Waverley.
Striders' Ans: (d) W. Scott began his novel writing career in his
(b) Nissim Ezekiel – 'The Striders', Kamala Das 43rd year with ‘Waverley’ (1814), the first of the long
– 'Rough Passage', R. Parthasarthy – 'Hymns series of novels that bore the title by the author of
in Darkness', A.K. Ramanujan – 'The Sirens' Waverley (because he hadn’t acknowledged authorship
(c) Nissim Ezekiel – 'The Sirens', Kamala Das – until 1827).
'Hymns in Darkness', R. Parthasarthy – 'The Hence option (d) is correct.
Striders', A.K. Ramanujan – 'Rough Passage'
32. Which of these descriptions/statements best
(d) Nissim Ezekiel – 'Rough Passage', Kamala
suits the idea of the "Renaissance Man' ?
Das – 'The Striders', R. Parthasarthy – 'The
Striders', A.K. Ramanujan – 'Hymns in I. A fop, a scoundrel, who enjoys enormous
Darkness' power in Renaissance courts and aristocratic
families.
Ans: (a) The correctly matched pairs are – Nissim
II. A near-mythical figure : a knight, courtier,
Ezekiel – ‘Hymns in Darkness’ (1976), Kamala Das-
musician, poet, scholar and statesman.
‘The Sirens’ (1964) R. Parthasarthy – ‘Rough Passage’
(1977), A.K. Ramanujan - ‘The striders’ (1966). III. One who ploughs a lonely furrow and keeps
Hence option (a) is correct. away from politicking and scandals.
IV. Someone like Sir Philip Sydney best suits the
29. William Wordsworth had a deep influence on
ideal of the Renaissance Man.
Thomas Hardy.
(a) I (b) IV
According to Hardy a particular poem by
Wordsworth was his 'best cure for despair'. (c) I and III (d) II and IV
Which is that poem? Ans: (d) Renaissance Man pertain to multiple qualities
(a) "Michael" in the protagonist- a near mythical figure, a knight,
(b) "Tintern Abbey Revisited" courtier, musician, poet, scholar and statesman –
(c) "The Idiot Boy" someone like Sir Philip Sidney who was knighted and
possessed all the above mentioned qualities.
(d) "The Leechgatherer"
Hence option (d) is correct.
Ans: (d) Wordsworth’s ‘Resolution and Independence’
published in 1807 in ‘Poems in two volumes’ was 33. Maxim Gorky, the great Russian writer of
originally written in 1802 under the title ‘The fiction and drama, was in real life a man called
Leechgatherer’ which later had a deep influence on _____.
Thomas Hardy. (a) Goliardic Kreshkov
Hence option (d) is correct. (b) Ronsardo Felixikov
30. In Henry James's 'Ambassadors', there is a (c) Malthias Serpieri
character who never appears in the novel. We (d) Aleksei Peshkov
get to know about this significant person, Ans: (d) The real life name of Maxim Gorky was
however, from the other characters. Who is Aleksei Peshkov.
this character? Hence option (d) is correct.
(a) Maria Gostrey 34. After the prediction of the oracle that he was
(b) Madame de Vionette destined to kill his father, Oedipus could have
(c) Mrs. Newsome avoided patricide
(d) Mrs. Sarah Pocock I. had he not determined in horror never to
Ans: (c) In Henry James ‘Ambassadors’ (1903) Mrs. return to the only parents he knew.
Newsome never appears and we get to know about her II. had he been a man of unusual self-control.
from other characters. III. had he remembered the prediction and had he
Hence option (c) is correct. been more cautious having recognized that
31. Why are Scott's novels called "Waverley possibly after all Polybos was not his father.
Novels"? IV. had he never struck any man who was older
(a) His novels are all set in Waverley. than himself saying at the moment of
(b) The Waverley Castle has a significant role in provocation "This insolent man is grey-
his novels. haired; let him have the road".
UGC NET/JRF English Paper III, December 2013 131 YCT
Find the correct combination according to the 39. The play was written in 1881 when its author
code : was in Italy. This is considered to be his most
(a) I, II and III are correct. remarkable intellectual effort. The softening of
the brain as a result of a disease inherited from
(b) I, II and IV are correct.
his father is the subject. Which is the play?
(c) I, III and IV are correct. (a) An Enemy of the People
(d) II, III and IV are correct. (b) Ghosts
Ans: (d) Oedipus had determined never to return to his (c) Rhinoceros
assumed parents and so he left their city Corinth and on (d) Six Characters in Search of an Author
the way to Thebes he accidently killed his real father Ans: (b) The play talked about here is Henrik Ibsen’s
and then unknowingly married his real step mother. So, ‘Ghosts’ (1881).
statement I is wrong. Hence option (b) is correct.
Hence option (d) is correct. 40. In many ways, grammatical categories remain
35. Identify the Post-Apartheid novel by Nadine mysterious. What does it mean to speak a
Gordimer. language that in every sentence requires you to
locate yourself in time, or specify your source
(a) The Conservationist (b) The House Gun of knowledge, or the shape of what you are
(c) The Lying Days (d) Burger's Daughter talking about? We still don't know. But putting
Ans: (b) Nadine Gordimer’s ‘The House Gun’ (1998) is the question like this suggests a clear and
a post apartheid novel. limited way of interpreting the idea that
Hence option (b) is correct. different languages represent different worlds.
Which of the following statements on this
36. The Duchess of Malfi married her steward, passage interprets it most accurately?
Antonio. For the Elizabethan audience her (a) The passage reflects the unreliability of
marriage was a triple offence. Which of the grammatical categories of a language
following is NOT one? generally.
(a) She was a widow marrying a second time. (b) The passage concedes that the Sapir-Whorf
(b) she married on her own outside the Church. hypothesis cannot be discounted entirely.
(c) She married beneath her status in disregard of (c) The passage upholds the reliability of
'degree'. grammatical categories of a language
generally.
(d) She married against the wishes of her
(d) The passage suggests that the Sapir-Whorf
brothers who almost acted like her guardians.
hypothesis is largely discredited today.
Ans: (d) In John Webster’s ‘The Tragedy of the Ans: (b) Edward Spair and B.L. Whorf hypothesized
Duchess of Malfy’ or simply ‘The Duchess of Malfi’ that the structure of a language determines or greatly
(1613 1st performed, 1623 published), the two brothers influences the modes of thought and behavior
did not act as guardians in her marriage but exacted characteristic of the culture in which it is spoken. The
revenge and had themselves destroyed in their pursuit. passage concedes that their hypothesis cannot be
So, statement (d) is not true. discounted entirely.
Hence option (d) is correct. Hence option (b) is correct.
37. Who among the following has written the 41. Tolstoy's 'War and Peace' carries a lengthy
essay, "The Indian Jugglers"? discussion of determinism and free will in____.
(a) its prologue
(a) Charles Lamb
(b) an exchange between Pierre and Natasha
(b) William Hazlitt (c) an exchange between Nikolai Rostof and
(c) Thomas de Quincey Princess Bezukhoi
(d) Thomas Love Peacock (d) its epilogue
Ans: (b) The essay ‘The Indian Jugglers’ was published Ans: (d) Leo Tolstoy’s ‘War and Peace’ (1869) carries
in 1821 in William Hazlitt’s collection ‘Table-Talk’. a lengthy discussion of determinism and free will in its
Hence option (b) is correct. epilogue.
Hence option (d) is correct.
38. How would you best describe George
Meredith's 'Modern Love' (1862)? 42. Which from among the following is NOT true
of Nagmandala?
(a) A ballad (b) A lyric travelogue
(a) It does not have multiple narratives.
(c) A verse romance (d) A sonnet sequence (b) It is open-ended.
Ans: (d) George Meredith’s 'Modern Love' (1862) is a (c) It combines conventional and subversive
16 line Sonnet sequence. modes.
Hence option (d) is correct. (d) Story is personified in the play.
UGC NET/JRF English Paper III, December 2013 132 YCT
Ans: (a) Girish Karnad’s ‘Nagmandala’ (1988) does not 44. Pick out the two relevant and correct
have multiple narratives. descriptions of Caryl Churchill's 'Serious
Hence option (a) is correct. Money' (1987) :
1. This play proposes the foundation of a
43. Arrange the following literary journals
monastery for the education of British
chronologically :
gentlewomen.
(a) The London Magazine
2. This narrative deals with children who are
The Quarterly Review sick of their "enforced idleness."
Blackwood's Magazine 3. This play is subtitled "City Comedy."
The Saturday Review 4. In this play, the state of the British economy
The Tatler is symbolized by a takeover bid by an
(b) The Tatler international cartel.
The Saturday Review 5. This narrative details the adventures of an
Blackwood's Magazine Anglo-Indian orphan.
The Quarterly Review 6. Money is the only criterion for success for
The London Magazine the players in this play's share-market.
(c) The Quarterly Review (a) 1 and 6 are correct. (b) 2 and 5 are correct.
(c) 4 and 6 are correct. (d) 5 and 6 are correct.
Blackwood's Magazine
The Tatler Ans: (c) The two correct descriptions of Caryl
Churchill’s 'Serious Money' (1987) are –
The Saturday Review
4. In this play the state of the British economy is
The London Magazine
symbolized by a takeover bid by an international cartel.
(d) The Tatler 6. Money is the only criterion for success for the players
The London Magazine in this play’s share-market.
The Quarterly Review Hence option (c) is correct.
Blackwood's Magazine 45. Identify from among the following FALSE
The Saturday Review statements :
Ans: (*) The NTA has awarded marks to every 1. Eric Arthur Blair became the famous British
candidates because none of the given options has the novelist, George Orwell.
correct chronological order of the given magazines. 2. Orwell was conversant in Hindustani and
- The Tatler is a British periodical launched in London fond of Indian food.
by the essayist Sir Richard Steele in April 1709. 3. Young Eric Blair lived in Myanmar's trading
• The London Magazine (1732) was originally founded town, Katha.
in 1732 as the "London Magazine, or Gentleman's 4. This town gave him the model for the
Monthly Intelligencer". Another magazine with the fictional district of Kyauktada in Burmese
same name, The London Magazine, (1820-29) by Days.
Josephine Bauer published between 1820 to 1829. 5. Orwell was born on June 25, 1903 in
• "The quarterly Review" (1809) was a literary and Motihari, Bihar.
political periodical. 6. The Orwell Commemorative Committee in
Motihari has been demanding a restoration of
• Blackwood's Magazine was a British Magazine and Orwell's birthplace as a heritage site.
miscellany printed between 1817 and 1980. It was
7. Orwell never returned to his birth place.
founded by William Blackwood.
8. The British journalist Ian Jack was mainly
• The Saturday Review (1855-1938) is an influential responsible for our knowledge of Orwell's
periodical by A.J.B. Beresford Hope. antecedents relating to Katha and Motihari.
• Saturday Review, previously "The Saturday Review (a) 2, 4, 8 are false.
of Literature" was an American magazine founded in (b) 7 and 8 are false.
1924, which began as a purely literary magazine but (c) 3, 6, and 8 are false.
broadened its scope in the 1940s. (d) All statements above are true.
So, None of the option contains the chronological Ans: (d) All the eight statements given above are true.
order of the magazines because there are two
Hence option (d) is correct.
magazines with the same name "London Magazines"
and two magazins with the same name "Saturday 46. Virginia Woolf borrowed the idea of The
common reader from Dr. Johnson. To which
Review"
particular work of Johnson's does she remain
• Hence, none of the options is correct. indebted?
UGC NET/JRF English Paper III, December 2013 133 YCT
(a) The Lives of the Most Eminent English (a) Word, wordling
Poets; the essay on Milton (b) Morpheme, morpheme-bit
(b) The Lives of the Most Eminent English (c) Free morpheme, bound-morpheme
Poets; the essay on Gray (d) Full morpheme, half-morpheme
(c) Preface to Shakespeare Ans: (c) In the word ‘rapidly’ rapid is a free morpheme
(d) The Patriot and 'ly' is a bound morpheme.
Ans: (b) Virginia Woolf’s ‘The Common Reader’ Hence option (c) is correct.
(1925) is indebted to Dr. Johnson’s essay on Gray in
Question Nos. 51 to 55 are based on a poem.
‘The Lives of the Most Eminent English Poets’ (1779-81).
Read the poem carefully and pick out the most
Hence option (b) is correct.
appropriate answers,
47. J.M. Coetzee was the first writer to be awarded It's Your Own Fault
the Booker Prize twice. He won the prize for Of course you can play with them.
(a) 'Life and Times of Michael K.' and 'Disgrace'
There's no harm in them.
(b) 'Dusklands' and 'Disgrace'
They are only words.
(c) 'Foe' and 'Elizabeth Costello'
Words alone are certain good, said someone.
(d) 'Age of Iron' and 'Disgrace'
And someone also said
Ans: (a) J.M. Coetzee was awarded the Booker Prize in
Unlike sticks and stones
1983 for ‘Life and Times of Michael K.’ and in 1999
Words will never break your bones.
for ‘Disgrace’. He was also awarded the Nobel in 2003.
Hence option (a) is correct.
(That is called rhyme. a rhyme is nice to play with
48. After the Norman Conquest England became a
too from time to time.)
three-language nation for at least two
centuries. The three languages were
(a) English, French and German What? They've turned nasty?
(b) English, Latin and German They've clawed you and bitten you?
(c) English, French and Latin Dear me, there's blood all over the place.
(d) English, French and Greek And broken bones.
Ans: (c) The three languages prevalent in England after
the Norman – conquest are – English, French and Latin. They were perfectly tame when I left them.
Hence option (c) is correct. Something they ate might have disagreed with
them.
49. Here are sentences labelled Assertion (A) and
Reason (R) : You mean you fed them on meaning?
Assertion (A) : In 'Who's Afraid of Virginia No wonder then.
Woolf?' George and Martha's blue and green-eyed – D.J. Enright
son is a myth. 51. The poet's remark on 'rhyme' is _____.
Reason (R) : He is a creation of the couple's (a) put in parenthesis (b) put in parentheses
imagination originating from their sense of (c) framed rhetorically (d) put in apposition
sterility and vacuum in life. Ans: (a) In rhetoric the term parenthesis is generally
In the light of (A) and (R), which of the used for brackets, as in lines 9-11.
following is correct? Hence option (a) is correct.
(a) Both (A) and (R) are true and (R) is the
52. The poem is cast in the form of a ______.
correct explanation of (A).
(a) romantic lyric (b) verse epistle
(b) Both (A) and (R) are true, but (R) is not the
correct explanation of (A). (c) dramatic monologue (d) dialogue
(c) (A) is true, but (R) is false. Ans: (c) In a dramatic monologue there is a main
(d) (A) is false, but (R) is true. speaker addressing a passive listener as in this poem
(line 1 – 'of course you can play….')
Ans: (a) The Reason correctly explains the Assertion
that in Edward Albee’s ‘Who’s Afraid of Virginia Hence option (c) is correct.
Woolf?’ (1962), George and Martha’s son is a myth 53. What is the "fault" to which the speaker refers
because he is an imaginary creation. here?
Hence option (a) is correct. (a) Playing with words
50. In the word rapidly, 'ly' is an adverbial suffix (b) Using only words
indicating manner while rapid is a _____, ly is (c) Taking words too seriously
a _____. (d) Reading meanings into words
UGC NET/JRF English Paper III, December 2013 134 YCT
Ans: (d) The second last line 'you fed them on 59. In most people, the first language /dialect
meaning? No wonder then', suggests that the fault is acquired is 'mother tongue'. Among the
reading meanings into words. commonly used terms for mother tongue, one
Hence option (d) is correct. of the following is avoided. Identify the one
54. What tone is most appropriate for reading this term NOT applied to mother tongue :
poem? (a) First language (b) Prime Language
(a) Evasive (b) Plaintive (c) Native language (d) Primary language
(c) Ironic (d) Sarcastic Ans: (b) ‘Prime language’ is not applied for ‘mother
Ans: (c) There is Irony inherent in the tone of this tongue.’
poem. Hence option (b) is correct.
Hence option (c) is correct. 60. Identify the group of critical concepts that
55. "No wonder then." Explain. parenthetically aligns them with their
(a) No wonder that the words here begin to respective theorists :
mean. (a) The Carnivalesqe (Jean Baudrillard), Habitus
(b) No wonder that you now find the words (Pierre Bourdieu), Flaneur (Walter
menacing. Benjamin), Chora (Gayatri C. Spivak),
(c) No wonder that the words find you menacing Simulacrum / Simulacra (Antonio Gramsci),
(d) No wonder the words still mean and are The Subaltern (Mikhael Bakhtin),
tame. Metahistory (Walter Benjamin), Aura (Julia
Ans: (b) Line 12 suggests the words have turned nasty Kristeva), Polyphony (Mikhael Bakhtin),
meaning that having 'fed on meaning, No wonder' that Hegemony (Antonio Gramsci)
we now find the words menacing.
(b) Habitus (Pierre Bourdieu), Flaneur (Walter
Hence option (b) is correct.
Benjamin), Chora (Julia Kristeva),
56. "Nothing odd will do long. ____did not last Simuacrum / Simulacra (Jean Baudrillard),
long. The Subaltern (Gayatri C. Spivak)
Dr. Johnson had this to say about one of the Metahistory (Hayden White), Polyphony
eighteenth century novels. Identify it: (Mikhael Bakhtin), Hegemony (Antonio
(a) Tom Jones Gramsci)
(b) The Female Quixote
(c) Habitus (Julia Kristeva), Flaneur (Walter
(c) Tristram Shandy Benjamin), Chora (Pierre Bourdieu),
(d) Clarissa Simulacrum / Simulacra (Hayden White),
Ans: (c) Dr. Johnson in 1776, said this for Sterne’s ‘The The Subaltern (Gayatri C. Spivak),
Life and Opinions of Tristram Shandy’ that- “Nothing Metahistory (Jean Baudrillard), Polyphony
odd will do long, Tristram Shandy did not last long.” (Mikhael Bakhtin), Hegemony (Antonio
Hence option (c) is correct. Gramsci)
57. Identify the sonnet upon sonnet by William (d) Habitus (Pierre Bourdieu), Flaneur (Antonio
Wordsworth : Gramsci), Chora (Julia Kristeva),
(a) "London, 1802" Simulacrum /Simulacra (Jean Baudrillard),
(b) "The world is too much with us..." The Subaltern (Gayatri C. Spivak),
(c) "Friend! I know not which way..." Metahistory (Hayden White), Polyphony
(d) "Nuns fret not at their convent's narrow (Mikhael Bakhtin), Hegemony (Walter
room..." Benjamin)
Ans: (d) Wordsworth’s 'Nuns Fret not at their Ans: (b) The correct pairs are as follows –
convent’s narrow room' . . . (1807) has sonnet as its
Habitus – Pierre Bourdieu (1930-2002)
subject i.e. it is a sonnet upon sonnet.
Flaneur – Walter Bejamin (1892-1940)
Hence option (d) is correct.
Chora - Julia Kristeva (1941- )
58. Who among the following women writers has
written 'Novel on Yellow Paper'? Simulacrum - Jean Baudrillard (1929-2007)
(a) Elizabeth Smither (b) Stevie Smith The Subaltern- Gayatri C. Spivak (1942)
(c) Zulu Sofola (d) Gita Mehta Metahistory - Hayden white (1928)
Ans: (b) Stevie Smith is the author of ‘Novel on Yellow Polyphony - Mikhael Bakhtin (1895-1975)
Paper’ (1936). Hegemony – Antonio Gramsci (1891-1931)
Hence option (b) is correct. Hence option (b) is correct.

UGC NET/JRF English Paper III, December 2013 135 YCT


61. What was the mandate of the Stationer's The Aesthetes – late 19th century.
Company incorporated in London in 1557? The Georgians – early 20th century
(a) To oversee the affairs of the Royal Registry. The High Modernists – late 1950s and 60s.
(b) To oversee authors' and printers', or printer- Although the correct answer is option (*), yet the NTA
publishers' rights. has awarded the full marks to all the candidates.
(c) To oversee authors' and printers' or printer-
publishers' use of stationery. 64. Which Bible is the earliest English version
(d) To oversee the quality of stationery printed with verse divisions?
harnessed by the Royal Registry. (a) Tyndale's Translation
Ans: (b) The mandate (a command or authorizing (b) The Geneva Bible
power) of the Stationer’s Company incorporated in (c) The Douay-Rheims Version
London in 1557 was to oversee the rights of authors and (d) King James Version
printers or printer publishers. Ans: (b) The first English New Testament to have both
Hence option (b) is correct. chapter and verse divisions was the Geneva Bible
62. One of the following was described by its (1560). Robert Estienne (Stephanus) was the first to
author as "a poem including history." Identify number the verse within each chapter in vernacular
the poem. editions of New Testament in 1551 and Hebrew Bible
(a) Robert Lowell, Life Studies in 1571.
(b) William Carlos Williams, Paterson Hence option (b) is correct.
(c) Elizabeth Bishop, Questions of Travel 65. E.M. Forster's 'Passage to India' begins with a
(d) Ezra Pound, The Cantos description of the city of Chandrapore. It has
Ans: (d) Ezra Pound’s ‘The Cantos’ is a famous an old Indian part and a new part consisting of
modernist poem in 120 sections published from 1922 the British civil station. Which of the following
onwards and described by him as ‘a poem including descriptions of the city is not found in the text ?
history.’ (a) The streets are mean, the temples ineffective.
Hence option (d) is correct.
(b) It is a city of gardens.
63. Arrange the following groups of English (c) It is a tropical pleasaunce washed by a noble
writers in chronological order :
river.
(a) The Metaphysical poets
(d) The new civil station is not sensibly planned
The High Modernists
and not modern.
Transitional poets
The Georgians Ans: (d) The wrong statement is option (d) the new
The Aesthetes civil station is not sensibly planned and not modern.
The University Wits Hence option (d).
(b) The University Wits 66. In which of the following books would you find
The Metaphysical poets the following arguments / observations?
Transitional poets Escapist fiction lacks serious fiction's
The Aesthetes apocalyptic experience of finality. The two
The Georgians versions of literary experience are qualitatively
The High Modernists different; every novel fits one category or the
(c) The High Modernists other, not both. Serious fiction, however,
The Georgians compels our attention by representing
The Aesthetes improvements (the "world of potency") as
Transitional poets being achieved (a "world of act") and by
The Metaphysical poets showing narrative movement "through time to
The University Wits an end, an end, we must sense even if we
(d) The University Wits cannot know it."
The Metaphysical poets (a) Sincerity and Authenticity
The Aesthetes (b) The Sense of an Ending Studies in the
Transitional poets Theory of Fiction
The Georgians (c) Beyond the Apocalypse
The High Modernists (d) The Rhetoric of Fiction
Ans: (*) The correct chronological sequence is– Ans: (b) The above extract of fiction criticism is found
The University Wits – late 16th century. in Frank Kermode’s 1967 work ‘The Sense of an
The Metaphysical Poets – 17th century. Ending: Studies in the Theory of Fiction.’
Transitional Poets – Mid 18th century. Hence option (b).

UGC NET/JRF English Paper III, December 2013 136 YCT


67. Philip Larkin's "The Whitsun Weddings" Ans: (a) The correct pairs are –
I. describes a long train journey I. 'And we are here.....' 3. "Dover Beach"
II. establishes a 'we' voice of collective outlook (1867) by Mathew
III. traces the disfigurement of a sunny landscape Arnold.
on an advertising poster
II. 'Thus, though we 4. ‘To His Coy
IV. gives an account of a drug pusher
cannot make our sun…' Mistress’ (1650s)
The correct combination according to the code
is : by Andrew Marvell.
(a) I and III are correct. III. 'One short sleep past, we 1. ‘Death, be not
(b) I and II are correct. wake eternally, and death proud…’ (1633)
(c) I and IV are correct. shall be no more…….' by John Donne.
(d) II and III are correct. IV. ‘This one last gift I 2. ‘The Great Lover’
Ans: (b) Philip Larkin’s ‘The Whitsun Weddings’ give : that after men (1914) by Rupert
(1964) describes a long train journey and establishes a shall know….’ Brooke.
‘we’ voice of collective outlook. Hence option (a) is correct.
Hence option (b) is correct. 69. The Oxford Companions are handy reference
68. Match the last lines of the poems with their volumes for teachers and students of English.
correct titles : Identify the one volume that has NOT yet
List–I List–I appeared in this series :
(Last lines (Titles of (a) The Oxford Companion to Twentieth-
of poems) poems) Century Literature in English
I. And we are here 1. "Death, be not proud..." (b) The Oxford Companion to The Oxford
as on a darkling Canadian Literature
plain (c) The Oxford Companion to American
Swept with Literature
confused alarms
(d) The Oxford Companion to Indian Literature
of struggle and in English
flight,
Where ignorant Ans: (d) 'The Oxford Companion to Indian Literature in
armies clash by English' has not appeared yet.
night. Hence option (d) is correct.
II. Thus, though we 2. "The Great Lover" 70. While writing or printing, scholarly use prefers
cannot make our titles in italics. Which of the following is the
sun. correct way of writing/printing?
Stand sitll, yet (a) Charles Dicken's Tale of Two Cities
we will make (b) Charles Dickens' Tale of Two Cities
him run. (c) Charles Dickens' A Tale of Two Cities
III. One short sleep 3. "Dover Beach" (d) Charles Dicken's A Tale of Two Cities
past, we wake Ans: (c) According to MLA eighth edition handbook,
eternally, the work should either be in italics or in inverted
And death shall be commas and will come after author's name in
no more; death alphabetical order of second names in the works cited
thou shalt die. list.
IV. This one last gift 4. "To His Coy The correct way of writing/printing the titles is given in
give: that after men Mistress" option (c). Charles Dickens' A Tale of Two Cities is the
Shall know, correct way to write for scholarly use.
and later lovers,
Questions from 71 to 75 are based on the
far-removed, following passage. Read the passage carefully and
Praise you, "All select the most appropriate option :
these were lovely,"
Somewhere, on the edge of consciousness, there is
say, "He loved". what I call a mythical norm, which each one of us within
Codes : our hearts knows "that is not me." In America, this norm
I II III IV is usually defined as white, thin, male, young,
(a) 3 4 1 2 heterosexual, Christian, and financially secure, It is with
(b) 4 3 2 1 this mythical norm that the trappings of power, reside
(c) 2 1 4 1 within the society. Those of us who stand outside that
(d) 1 2 3 4 power often identify one way in which we are different,
UGC NET/JRF English Paper III, December 2013 137 YCT
and we assume that to be the primary cause of all Ans: (*) The author marks her difference from other
oppression, forgetting other distortions around writer on similar issues and underscore her radical style
difference, some of which we ourselves may be typographically by italicizing 'mythical norm' and
practicing. By and large within the women's movement sisterhood, and by using lowercase for proper and
today, white women focus upon their oppression as common nouns and by using phrases like "Those of us
women and ignore differences of race, sexual preference, who stand outside....'.
class, and age. There is a pretense to a homogeneity of There is no option which contains statement 2, 3 and 4.
experience covered by the word sisterhood that does not Hence, the NTA has awarded full marks to all the
in fact exist.
candidates.
(Audre Lorde)
73. That there are levels and grades of
71. A mythical norm is endemic to societies :
powerlessness in societies entertaining 'a
1. where racial myths are prevalent and widely
mythical norm' is indicated
respected and perpetuated through utterances
1. by the overall tone and tenor of the passage.
that establish 'we' and 'they' groups.
2. by the suggestion that 'a mythical norm' is
2. where the superiority of one's own culture
responsible for the unequal distribution of
and nation no longer emphasized openly or
power among people.
straightforwardly.
3. by referring to 'other distortions around
3. where 'difference' has been a preoccupation
difference'.
in the representation of people who are
racially, ethnically, and in terms of gender 4. by referring to white women who narrow
and sexual preference different from an down oppression directed only at white
assumed majority. women.
4. that believe that the norm is part of their right (a) 4 is correct. (b) 1 and 2 are correct.
to defend the ways of life enjoyed by a (c) 3 is correct. (d) 2 is correct.
dominant group, their traditions and customs Ans: (c) Indication of levels and grades of
against outsiders – not because these powerlessness in societies entertaining ‘mythical norm’
outsiders are inferior, but because they is given by referring to ‘other distortions around
belong to other cultures. difference’ in line 15 of the passage.
(a) 1 and 4 are correct. (b) 2 and 3 are correct. Hence option (c) is correct.
(c) Only 4 is correct. (d) Only 3 is correct. 74. Why is the author dismissive about
Ans: (b) Endemic means native or indigenous. A 'sisterhood'?
mythic norm is endemic or indigenous to societies 1. Because it is italicised.
where the consciousness of one’s own culture and 2. Because it does not exist in principle.
nation is not emphasized openly because of trappings of 3. Because it assumes that all 'sisters' are alike.
power residing within certain norms such as (in 4. Because it assumes that all 'sisters' are
America) white, thin, male, financially secure etc. unique.
Those who are outside that power assume their (a) 3 is correct. (b) 1 is correct.
difference blaming it (norms of power) of all (c) 4 is correct. (d) 2 is correct.
oppression. So, this ‘difference’ has been a
Ans: (a) The author is dismissive about ‘sisterhood’
preoccupation in the representation of people who are
because it assumes that all ‘sisters’ are alike. It is a
racially, ethnically and in terms of gender and sexual
pretense that does not exist in practical situation.
preference different from an assumed majority, thus
Hence option (a) is correct.
being endemic.
Hence option (b) is correct. 75. Does the author absolve all women from the
'distortions around difference'?
72. How does the author mark her difference from
1. Yes.
other writers on similar issues and underscore
her radical style typographically? 2. No.
1. By her use of parataxis 3. Not sure.
2. By italicizing 'mythical norm' and 'sisterhood' 4. Yes, in a qualified manner though.
3. By using lowercase for proper and common (a) 1 is correct (b) 2 is correct
nouns (c) 3 is correct (d) 4 is correct
4. By using phrases like 'Those of us who stand Ans: (b) No the author says that we ourselves might be
outside...' practising some of the differences we accuse of
(a) 1 and 4 are correct. (b) 2 is correct. distorting the society.
(c) 3 is correct. (d) 2 and 3 are correct. Hence option (b) is correct.

UGC NET/JRF English Paper III, December 2013 138 YCT


UGC NET/JRF Exam, December-2013
ENGLISH
Solved Paper-II
Note : This paper contains fifity (50) objective 5. "She was a worthy woman al hir lyve,
type questions of two (2) marks each. All questions Housbondes at chirche-dore she hadde fyve,"
are compulsory. In the 'Prologue' Chaucer represents the Wife
1. _____the very word is like a bell of Bath as :
to toll me back from thee to my sole self ! I. crude and vulgar
Which word? II. outspoken and boastfully licentious
(a) Bird (b) Immortal III. a witness to masculine oppression
(c) Forlorn (d) Fancy IV. bubbling with vitality
Ans: (c) The word ‘Forlorn’ completes the extract Find the correct combination according to the
which is from Keats’ 'Ode to a Nightingale' (1819). code :
Thus “Charm’d magic casements, opening on the foam (a) I, II and III are correct
of perilous seas, in faery lands forlorn. (b) I, II and IV are correct
Forlorn! The very word is like a bell (c) I, III and IV are correct
to toll me back from thee to my sole self !” (d) II, III and IV are correct
Hence option (c) is correct. Ans: (b) In his ‘Prologue’ to ‘The Canterbury Tales’
2. In poems like "The Altar" and "Easter Wings" (1475), Chaucer represents the Wife of Bath as crude,
____exploits _____. vulgar, outspoken, licentious and bubbling with vitality.
(a) John Donne, alliteration With her boasting of having five husbands she is not at
(b) Robert Herrick, trimetre all oppressed. So, statement III is wrong.
(c) G.M. Hopkins, sprung rhythm Hence option (b) is correct.
(d) George Herbert, typographic space 6. The novel tells the story of twin brothers,
Ans: (d) In poems like ‘The Altar’ and ‘Easter Wings’ Waldo, the man of reason and intellect, and
(published in 1633), George Herbert exploits the Arthur, the innocent half-wit, the way their
typographical space. lives are inextricably intertwined. Which is the
Hence option (d) is correct. novel?
3. "No, no thou hast not felt the lapse of hours! (a) The Tree of Man (b) Voss
For what wears out the life of mortal men? (c) The Solid Mandala (d) The Vivisector
'Tis that repeated shocks, again, again, Ans: (c) Patrick White's ‘The Solid Mandala’ (1966)
Exhaust the energy of strongest souls tells the story of two twins- Waldo (cold and rational)
And numb the elastic powers..." and Arthur Brown (warm hearted and instinctual), so
Who does the poet address here? that together they represent the two conflicting and
(a) The Scholar Gipsy complementary halves of human nature.
(b) Telemachus Hence option (c) is correct.
(c) The Nightingale 7. Who among the following was NOT a member
(d) The Poet's Sister, Dorothy of the Scriblerus Club?
(a) Thomas Parnell
Ans: (a) In the above lines the poet M. Arnold is
addressing the Scholar Gipsy. The poem’s title is ‘The (b) Alexander Pope
Scholar Gipsy’ (1853). (c) Joseph Addison
Hence option (a) is correct. (d) John Gay
4. The roman-a-clef (French for "novel with a Ans: (c) The Scriblerus Club was an informal group of
key") uses contemporary historical figures as authors founded in 1714 by J. Swift and A. Pope; other
its chief characters. They are of course given members were J. Gay, T. Parnell, Henry, St. John and J.
fictional names. One example is Aldous Arbuthnot. J. Addison was not part of this group.
Huxley's 'Point Counter Point'. Hence option (c) is correct.
Its Mark Rampion is modelled on _____. 8. _____ is a theological term brought into
(a) D.H. Lawrence (b) E. M. Forster literary criticism by ______.
(c) Wyndham Lewis (d) Arnold Bennett (a) Entelechy, St. Augustine
Ans: (a) In Aldous Huxley’s ‘Point Counter Point’ (b) Ambiguity, William Empson
(1928), Mark Rampion is modelled on D.H. Lawrence. (c) Adequation, Fr Walter Ong
Hence option (a) is correct. (d) Epiphany, James Joyce
UGC NET/JRF English Paper II, December 2013 139 YCT
Ans : (d) Epiphany is a theological term brought into Ans: (c) Ralph Ellison’s ‘Invisible Man’ (1952) has the
English literary criticism by James Joyce. Theology is above mentioned scene where African-American
the study and influence of religion. students are made to compete for the gold coins being
Epiphany is the divine manifestation (of three wise men tossed on the electric blanket.
visiting infant Jesus on the twelfth day after Christmas Hence option (c) is correct.
on January 6). 13. G.M. Hopkins's "Windhover" is dedicated :
Hence option (d) is correct. (a) To Christ, our Lord
(b) To Christ our lord
9. _____ the Almighty Power Hurled headlong
flaming from th' Ethereal Sky, (c) to no one
(d) to Christ, the Lord
With hideous ruin and combustion down
To bottomless perdition, there to dwell Ans: (a & b) "The Windhover" is a sonnet by G.M.
Hopkins. It was written on 30 may 1877, but not
In Adamantine Chains and penal Fire published until 1914. Hopkins dedicated the poem "To
Who durst defy th' Omnipotent to arms. Christ our Lord" or To Christ, Our Lord".
(Paradise Lost, I.44-49.) NTA has considered both the options (a) and (b)
Choose the appropriate word : correct. Hence, option (a) & (b) are correct answer.
(a) Him (b) He 14. Match List – I with List – II according to the
(c) Satan (d) Th Fiend code given below :
Ans: (a) In the extract from Milton’s ‘Paradise Lost’, List–I List–I
‘Him’ is the appropriate word in the blank, which (Authors) (Poems)
symbolizes the 'Almighty' God who hurled down Satan i. Ted Hughes 1. "The Otter"
from heaven for his defiance. ii. Seamus Heaney 2. "Snake"
Hence option (a) is correct. iii. W.H. Auden 3. "Ghost Crabs"
iv. D.H. Lawrence 4. "Prevent the Dog
10. Which of the following works does not have a
from Barking
mad woman as a character in it?
with a Juicy Bone."
(a) The Yellow Wallpaper
Codes :
(b) The Mad Woman in the Attic i ii iii iv
(c) Jane Eyre (a) 1 2 4 3
(d) Wide Sargasso Sea (b) 2 3 1 4
Ans: (b) ‘The Mad Woman in the Attic’ (1979) by (c) 3 1 4 2
Sandra Gilbert and Susan Gubar is a critical work on (d) 3 2 1 4
Victorian women and literature. The other works are Ans: (c)
novels with mad woman as a character in their i. Ted Hughes wrote 3. ‘Ghost Crabs’ pub. in 1967
respective stories and under strange situations. ii. Seamus Heaney 1. ‘The Other’ pub. In 1979.
Hence option (b) is correct. wrote
11. Which of the following is NOT a quest iii. W.H. Auden wrote 4. ‘Prevent the Dog from
narrative? barking with a Juicy
(a) Shelley's Alastor Bone’ pub in 1940.
(b) Byron's Manfred iv. D.H. Lawrence 2. ‘Snake’ pub. in 1923.
(c) Coleridge's Christabel wrote
(d) Keats's Endymion Hence option (c) is correct.
Ans: (c) Coleridge’s unfinished long narrative poem 15. "His cooks with long disuse their trade forgot;
‘Christabel’ (written in 1797 (I), 1800 (II) and Cool was his kitchen, though his brains were hot."
published in 1816) is not a quest narrative but a Who is this character whose stinginess passed
complex love poem with Christabel and Geraldine as into a proverb?
main characters. (a) Corah (b) Shimei
Hence option (c) is correct. (c) Zimri (d) Achitophel
12. The novel has a scene where African American Ans: (b) In this extract from Dryden’s ‘Absalom and
Achitophel’ (1681), Shimei is the character whose
students are made to compete and fight with
stinginess passed into a proverb. He represented slings
each other as they rush for the gold coins by Bethel, the Sheriff of London.
tossed on an electric blanket. Identify the Hence option (b) is correct.
novel.
16. "The story and the novel, the idea and the
(a) Richard Wright : Native Son
form, are the needle and thread, and I never
(b) James Baldwin : Another Country heard of a guild of tailors who recommended
(c) Ralph Ellison : Invisible Man the use of the thread without the needle or the
(d) Toni Morrison : Bluest Eye needle without the thread."
UGC NET/JRF English Paper II, December 2013 140 YCT
This famous passage describing the relation of Ans: (b) ‘Late capitalism’ was first popularized by
idea to form is found in Ernst Mandel through his work ‘Der spatkapitalismus’
(a) Sir Philip Sidney, 'An Apology for Poetry' (1972) trans. in English as ‘Late Capitalism’ in 1975 by
(b) Samuel Taylor Coleridge, 'Biographia Joris de Bres. This term was first used by Werner
Literaria' Sombart in 1902 in his work – ‘Der Moderne
(c) Henry James, 'The Art of Fiction'. Kapitalismus’.
(d) I.A. Richards, 'Principles of Literary Hence option (b) is correct.
Criticism' 20. Which of the following arrangements is in the
Ans: (c) The extract given above is taken from Henry correct chronological sequence?
James ‘The Art of fiction’ essay published in (a) 'Native Son' by Richard Wright – 'Invisible
Longman’s Magazine in Sept. 1884. Man' by Ralph Ellison – 'Their Eyes Were
Hence option (c) is correct. Watching God' by Zora Neil Hurston –
17. Identify the correctly matched set below : 'Another Country' by James Baldwin
(a) The Norman Conquest – 1066; William (b) 'Their Eyes Were Watching God' by Zora
Caxton and the introduction of printing – Neil Hurston – 'Native Son' by Richard
1575; The King James Bible – 1611; Dr. Wright – 'Invisible Man' by Ralph Ellison –
Johnson's English Dictionary – 1755; The 'Another Country' by James Baldwin
Commonwealth Period/ the Protectorate – (c) 'Invisible Man' by Ralph Ellison – 'Native
1649- 1660 Son' by Richard Wright – 'Another Country'
(b) The Norman Conquest – 1066; William by James Baldwin – 'Their Eyes Were
Caxton and the introduction of printing – Watching God' by Zora Neil Hurston
1475; The King James Bible – 1611; Dr. (d) 'Their Eyes Were Watching God' by Zora
Johnson's English Dictionary – 1755; The Neil Hurston – 'Another Country by James
Commonwealth Period/ the Protectorate – Baldwin – 'Native Son' by Richard Wright –
1649- 1660 'Invisible Man' by Ralph Ellison
(c) The Norman Conquest – 1016; William Ans: (b) The correct chronological sequence is –
Caxton and the introduction of printing – ‘Their Eyes were Watching God’ by Zora Neil Hurston-
1475; The King James Bible – 1564; Dr. 1937;
Johnson's English Dictionary – 1780; The ‘Native son’ by Richard Wright – 1940;
Commonwealth Period/ the Protectorate –
1649- 1660 ‘Invisible Man’ by Ralph Ellison – 1952 and
(d) The Norman Conquest – 1013; William ‘Another country’ by James Baldwin – 1962
Caxton and the introduction of printing – Hence option (b) is correct.
1575; The King James Bible – 1627; Dr. 21. Metaphor is so widespread that it is often used
Johnson's English Dictionary – 1746; The as an umbrella term to include other figures of
Commonwealth Period/ the Protectorate – speech such as metonyms which can be
1624 - 1660 technically distinguished from it in its
Ans: (b) The correct pair of events and dates are – narrower usage.
The Norman Conquest – 1066 Identify the metaphorical phrase in this
W. Caxton’s Printing Press – 1475-76 sentence:
King James' Bible – 1611 (a) narrower usage
Dr. Johnson’s 'English Dictionary' – 1755 (b) technically distinguished
Commonwealth Period/Protectorate – 1649-60 (c) figures of speech
Hence option (b) is correct. (d) umbrella term
18. Leopold Bloom in Ulysses is Ans: (c & d) Metaphorical phrase is a phrase or group
(a) a Great War veteran of words containing an implied comparison, so in the
(b) a Dublin bar owner above extract 'figures of speech' and 'umbrella term'
represents a metaphorical phrase.
(c) a Jewish advertising agent
NTA has considered both the options (c) & (d) correct
(d) an Irish nationalist
answer.
Ans: (c) In James Joyce’s novel 'Ulysses' (1922),
Leopold Bloom is the fictional protagonist who is a 22. Along the shore of silver streaming Thames;
Jewish advertising agent. Whose rutty bank, the which his river hems,
Hence option (c) is correct. Was painted all with variable flowers,
...
19. "Late capitalism", by which is meant
accelerated technological development and the Fit to deck maidens' bowers
massive extension of intellectually qualified And crown their paramours
labour, was first popularized by ____. Against their bridal day, which is not long;
(a) Terry Eagleton (b) Ernst Mandel Sweet Thames! run softly till I end my song.
(c) Raymond Williams (d) Stanley Fish (Spenser's Prothalamion)
UGC NET/JRF English Paper II, December 2013 141 YCT
Another poet fondly recalls these lines but 27. It is unimaginable that all the following events
cannot conceal their heavily ironic tone in : happened in one year :
(a) Marianne Moore's "Spenser's Ireland" 1. Arthur Evans discovered the first European
(b) Sylvia Plath's "Morning Song" civilization; his excavations in Crete revealed
(c) W.H. Auden's "In Praise of Limestone" a culture that was far older than either Attic
(d) T.S. Eliot's "Waste Land" Greece or Ancient Rome.
Ans: (d) T.S. Eliot recalled the line – "Sweet Thames, 2. Sir Arthur Quiller-Couch published the
run softly till I end my song" in part 3 titled ‘The Fire Oxford Book of English Verse.
Sermon’ in his poem ‘The Waste Land’ (1922) taken 3. Pablo Picasso stepped off the Barcelona train
from Spenser’s ‘Prothalamion’ (1596). at Gare d' Orsay, Paris.
Hence option (d) is correct. 4. Max Planck unveiled the Quantum Theory.
23. The tramp in Pinter's first big hit, 'The 5. Hugo de Vries identified what would later
Caretaker', often travels under an assumed come to be called genes.
name. It is 6. Sigmund Freud published 'The Interpretation
(a) Bernard Jenkins (b) Roly Jenkins of Dreams'.
(c) Jack Jenkins (d) Peter Jenkins 7. Coca-cola arrived in Britain.
Ans: (a) In Pinter’s ‘The Caretaker’ (1960) The tramp Identify the year :
often travels under the assumed name of Bernard (a) 1899 (b) 1900
Jenkins, who is really Mac Davies. (c) 1901 (d) 1903
Hence option (a) is correct. Ans: (b) All the above mentioned famous events
24. Here is a list of early English plays imitating occurred in the year 1900.
Greek and Latin plays. Pick the odd one out : Hence option (b) is correct.
(a) Gorboduc 28. Brother to a prince and fellow to a beggar if he
(b) Tamburlaine be found worthy.
(c) Ralph Roister Doister This is the epigraph to
(d) Gammer Gurton's Needle (a) T.S. Eliot's "The Hollow Men"
Ans: (b) Christopher Marlowe’s ‘Tamburlaine’ first (b) Rudyard Kipling's "The Man Who Would be
published around 1587-88 was a milestone in the King"
Elizabethan Drama marking a turning away from the (c) George Eliot's "Silas Marner"
clumsy language and loose plotting of the early Tudor (d) E.M. Forster's "Howard's End"
tragedies and comedies like – ‘Gorboduc’ (1561) Ans: (b) The above epigraph is of Rudyard Kipling’s
‘Ralph Roister Doister’ (1567) and ‘Grammer Gurton’s ‘The Man Who Would be King’ published in 1888.
Needle’ (1567). The character is based on Timur, the Hence option (b) is correct.
lame a medieval conqueror and portrays the ferocity of
his ambitions and cruelty. The play marked a new 29. Robert Graves's "In Broken Images" ends thus :
interest in fresh and vivid language, memorable action "He in a new confusion of his understanding;
and intellectual complexity. I in a new understanding of my confusion"
Hence option (b) is correct. The figure of speech here is ____.
25. Where does Act I Scene 1 of William (a) Chiasmus (b) Catachresis
Congreve's 'The Way of the World' open? (c) Inversion (d) Zeugma
(a) A Chocolate-House Ans: (a) The figure of speech used in above lines is
(b) A Pub Chiasmus in which two or more clauses are related
(c) A Carrefour through a reversal of structures to make a larger point
(d) The drawing room of Sir Willfull's mansion i.e. inverted parallelism.
Ans. (a) The Act I Sc. I of William Congreve’s ‘The Hence option (a) is correct.
Way of the World’ (1700) opens in a Chocolate – 30. The phrase "leaves dancing" is an example
House. of_____.
Hence option (a) is correct. (a) pathetic fallacy
26. While "a well-boiled icicle" for "a well-oiled (b) hyperbole
bicycle" is an example of Spoonerism, someone (c) pun
saying "Congenital food" for 'Continental (d) conceit
food' is an example of _____. Ans: (a) The phrase ‘leaves dancing’ is an example of
(a) Malapropism (b) Pleonasm pathetic fallacy which is a literary device wherein the
(c) Neologism (d) Archaism author attributes human emotions and traits to nature or
Ans: (a) Malapropism is the intentional misuse of inanimate objects. Eg. smiling skies, angry storm etc.
words. It was popularized by the character Lady Hence option (a) is correct.
Malaprop in Sheridan’s ‘The Rivals’ (1775). So the 31. At the end of 'The Great Gatsby', the narrator
above example is of Malapropism. Nick Carraway observes:
Hence option (a) is correct. "They were careless people". Who were they?
UGC NET/JRF English Paper II, December 2013 142 YCT
(a) Tom and Daisy Ans: (c)
(b) The Wilsons i. The last lines of J. 3. He feels it himself and says
(c) Gatsby and his friends Conrad’s ‘Lord Jim’ often that he is “preparing
(d) The people of East Egg (1900) are to leave all this; preparing
Ans: (a) In the 1925 novel ‘The Great Gatsby’ by F. to leave….”, while he
Scott Fitzgerald, Nick observes Tom and Daisy as waves his hands sadly
‘careless people.’ at his butterflies’.
Hence option (a) is correct. ii. V. Woolf’s ‘To the 1. 'It was done; it was
32. William Wordsworth's statement of purpose in Lighthouse’ (1927) finished yes, she thought
publishing the Lyrical Ballads carries the
laying down her brush in
following phrase. (Complete the phrase
correctly). extreme fatigue, I have
"to choose incidents from common life and to had my vision'.
relate or describe them, throughout, as far as iii. E.M. Forster’s ‘A 4. “No not yet” and the
possible, ______." Passage to India’ sky said, “No, not there.”
(a) in a selection of language really used by men. (1924)
(b) in a relation to language really used by men. iv. James Joyce’s ‘A 2. ‘April 27. Old father, old
(c) in a selection of language really used by Portrait of the Artist artificer, stand me now and
common man. as a young man’ ever in good stead……’
(d) in deference to language actually used by (1916)
men. Hence option (c) is correct.
Ans: (a) Wordsworth’s statement in his 'Preface' to the 34. Identify the incorrect description/s of "Sprung
‘Lyrical Ballads’ (1800) runs as – “to choose incidents Rhythm" from the following :
from common life and to relate or describe them,
throughout, as far as possible, in a selection of language 1. This rhythm causes ideas to spring in our
really used by men.” minds – hence Sprung Rhythm.
Hence option (a) is correct. 2. In Sprung Rhythm the feet are of equal
length.
33. Match List–I with List–II according to the code
given below : 3. A foot may have one to four syllables in
Sprung Rhythm.
List–I List–II
4. Its metre is derived from the metre of Anglo-
( Novels) (Last lines)
Saxon poetry which was based on accent and
i. 'Lord Jim' 1. 'It was done; it was
linked by alliteration.
finished. Yes, she
(a) 4 is incorrect.
thought laying
(b) 1 and 4 are incorrect
down her brush
in extreme fatigue, (c) 3 is incorrect.
I have had my (d) 1 is incorrect.
vision'. Ans: (d) In Sprung Rhythm, the feet are of equal length
ii. 'To the Lighthouse' 2. 'April 27. Old father, having one to four syllables and its metre is derived
old artificer, stand from the metre of Anglo Saxon poetry which was based
me now and ever on accent and linked by alliteration. It doesn’t cause any
idea to spring in our minds.
in good stead...
iii. 'A Passage to India' 3. 'He feels it himself Hence option (d) is correct.
and says often that 35. Who among the following proposes that the
he is "preparing to unconscious comes into being only in language?
leave all this; pre- (a) Sigmund Freud (b) Jacques Lacan
paring to leave,...", (c) Stuart Hall (d) Paul de Man
while he waves his Ans: (b) Jacques Lacan has proposed that the
hands sadly at his unconscious comes into being only in language.
butterflies'. Hence option (b) is correct.
iv. 'A Portrait of the 4. 'No not yet," and the 36. The Elizabethan Settlement established during
Artist as a Young sky said, "No, not the reign of Elizabeth I
Man' there'. I. ensured the supremacy of the Church of
Codes : England.
i ii iii iv II. allowed Christians to acknowledge the
(a) 2 4 3 1 authority of the Pope.
(b) 3 2 4 1 III. allowed the extremer Protestants to be part of
(c) 3 1 4 2 the Anglican Church.
(d) 2 3 1 4 IV. created a group known as the Roundheads.
UGC NET/JRF English Paper II, December 2013 143 YCT
The correct combination according to the code Ans: (c) Northrop Frye in his 1957 work ‘Anatomy of
is : Criticism: Four Essays' gives four generic plots based
(a) I and III are correct on four seasons, which are –
(b) I and II are correct Spring - Comedy
(c) II and III are correct Summer - Romance
(d) III and IV are correct Autumn - Tragedy
Ans: (a) The Elizabethan Settlement–(I) ensured the Winter - Irony and Satire
supremacy of the Church of England and (III) allowed So, Lyric is not mentioned here.
the extremer Protestants to be part of the Anglican Hence option (c) is correct.
Church. 41. Arrange the sections of The Waste Land in the
Hence option (a) is correct. order in which they appear in the poem :
37. Which of the following poems by Tennyson 1. The Fire Sermon
does NOT speak of old age and death? 2. Death by Water
(a) "The Beggar Maid" 3. A Game of Chess
(b) "The Lotus-Eaters" 4. What the Thunder Said
(c) "Ulysses" 5. The Burial of the Dead
(d) "Tithonus" (a) 3, 2, 1, 5, 4 (b) 5, 1, 2, 3, 4
Ans: (a) Tennyson’s ‘The Beggar Maid’ does not speak (c) 5, 2, 3, 1, 4 (d) 5, 3, 1, 2, 4
of old age and death but instead of a young beggar maid Ans: (d) T.S. Eliot’s ‘The Waste Land’ (1922) is
‘more fair than words can say.’ arranged in the following sections:–
Hence option (a) is correct. 1. The Burial of the Dead (5)
38. One English poet addressing another : 2. A Game of Chess (3)
3. The Fire Sermon (1)
Thy soul was like a Star, and dwelt apart;
4. Death by Water (2)
Thou hast a voice whose sound was like the sea:
5. What the Thunder Said (4)
Pure as the naked heavens, majestic, free,
The correct sequence would be – 5, 3, 1, 2, 4
So didst thou travel on life's common way,
Hence option (d) is correct.
In cheerful godliness....
Whose lines are these? To whom are they 42. Sir Plume is a character in___.
addressed? (a) Dryden's 'Absalom and Achitophel'
(a) W.H. Auden – W.B. Yeats (b) Congreve's 'The Way of the World'
(b) P.B. Shelley – William Blake (c) Pope's 'The Rape of the Lock'
(d) Farquhar's 'The Beaux' Strategem'
(c) William Wordsworth –John Milton
(d) Ben Jonson – William Shakespeare Ans: (c) Sir Plume is a character in Pope’s ‘The Rape
of the Lock’ (1712).
Ans. (c) In his poem ‘London, 1802’ Wordsworth is Hence option (c) is correct.
addressing Milton who shoulds’t be living at this hour.
Hence option (c) is correct. 43. Steeling herself to the murder, Lady Macbeth
calls on__to "unsex me here". (Macbeth I.5.39)
39. Samuel Johnson's 'Lives of Poets' (1781) was Choose the right option to fill in the blank :
originally a series of introductions to the poets (a) God
he wrote for a group of London publishers.
(b) the spirits of hell
They were collected as :
(c) the angels in heaven
(a) Lives of English Poets : Critical and (d) no one in particular
Biographical Essays.
(b) Prefaces, Biographical and Critical, to the Ans: (b) Lady Macbeth, in Act I, Sc. V, line 39, calls
on the 'spirits of hell' to ‘unsex me here’ to steel her
Works of English Poets.
heart to commit the murderous deed of killing King
(c) Notes, Biographical and Critical, on the Duncan. Hence option (b) is correct.
Works of English Poets.
44. You will find the following lines in an English
(d) Lives of English Poets: Biographical and
poem :
Critical Notes.
"Thou by the Indian Ganges' side
Ans. (b) Samuel Johnson’s 'Lives of Poets' (1781) were Shoulds’t rubies find; I by the side
originally collected as 'Prefaces, Biographical and Of Humber would complain."
Critical, to the works of English Poets'.
Which poem? Who is the poet?
Hence option (b) is correct.
(a) "Lonely Hearts." Wendy Cope
40. Which of the following is NOT mentioned in (b) "Holy Thursday." William Blake
Northrop Frye's four 'generic plots'? (c) "Tiger Mask Ritual." Chitra Banerjee
(a) The comic (b) The tragic Divakaruni
(c) The lyric (d) The ironic (d) "To His Coy Mistress." Andrew Marvell
UGC NET/JRF English Paper II, December 2013 144 YCT
Ans: (d) The above lines appear in Andrew Marvell’s Ans: (c) Jeanette Winterson’s ‘Written on the Body’
‘To His Coy Mistress’ published in 1650s. (1993) is the postmodernist novel that has an unusual
Hence option (d) is correct. protagonist whose gender is not revealed and is put to
45. "Teach me half the gladness speculation.
That thy brain must know, Hence option (c) is correct.
Such harmonious madness 48. Which novel of Graham Greene in the
From my lips would flow following list does NOT end in some form of
The world should listen then, as I suicide by the protagonist?
am listening now." (a) The Heart of the Matter
Whose lines are these? To whom are they (b) England Made Me
addressed? (c) Brighton Rock
(a) John Keats. 'The Nightingale' (d) The Power and the Glory
(b) P.B. Shelley. 'The Skylark'
Ans: (b) Graham Greene’s ‘England Made Me’ (1935)
(c) William Wordsworth. 'The Wye Valley' which was republished in 1953 as ‘The shipwrecked’
(d) Robert Browning. 'The Grammarian' does not end in any form of suicide by the protagonist.
Ans: (b) In the above lines P.B. Shelley is addressing Hence option (b) is correct.
the Skylark in his poem ‘To a Skylark’ (1820).
49. Who among the following gave a happy ending
Hence option (b) is correct.
to King Lear?
46. Match List – I with List – II according to the (a) James Quin
code given below :
(b) Nahum Tate
List–I List–II
(c) Peg Woffington
(Novel) (Major symbol)
(d) Charles Macklin
i. Dombey and son 1. fog
ii. The Return of the Native 2. train Ans: (b) Nahum Tate in ‘The History of King Lear’
iii. Bleak House 3. health (1681) altered the ending of the original ‘King Lear’ by
iv. Tess 4. mist Shakespeare.
Codes : Hence option (b) is correct.
i ii iii iv 50. Jane Austen's 'Pride and Prejudice' starts with
(a) 2 3 1 4 the famous statement: "It is a truth universally
(b) 4 2 3 1 acknowledged that a single man in possession
(c) 2 3 4 1 of a good fortune must be in want of a wife."
(d) 1 3 4 2 As we get to read the novel this statement
Ans: (a) seems to be made from the point of view of :
i. C. Dicken’s ‘Dombey and 2. train. I. the surrounding families
Son’ (1848) has the major II. Mrs Bennet
symbol of III. Mr Bennet
ii. T. Hardy’s ‘The Return of 3. health. IV. The women of Jane Austen's age and society
the Native’ (1878) has the Find out the correct combination according to
symbol of the code :
iii. C. Dicken’s ‘Bleak House’ 1. fog. (a) I, II and III are correct.
(1853) has the symbol of (b) I, II and IV are correct.
iv. T. Hardy’s 'Tess of the 4. mist. (c) II, III and IV are correct.
D'Urbervilles’ (1891) has (d) I, III and IV are correct.
the symbol of Ans: (*) Jane Austen is well known for Landgentry
Hence option (a) is correct. novels and she gives preference to marriage in state of
47. The following postmodernist novel has an love in her novels. The title 'Pride and Prejudice' which
unusual protagonist whose gender is not has been given another name "First Impression". It
revealed. So much so, that we keep wondering revolves around the domestic issues and concern for
whether that person's relationships are homo- marriage. These lines show the concern of lower middle
/hetero-sexual : class family.
(a) The French Lieutenant's Woman • NTA has dropped this question because there is no
(b) English Music proper harmony between the question and available
(c) Written on the Body answers. Therefore, marks have been given to all
(d) Enduring Love NTA candidates.

UGC NET/JRF English Paper II, December 2013 145 YCT


UGC NET/JRF Exam, June-2014
ENGLISH
Solved Paper-III
Note : This paper contains seventy five (75) (b) Una - Pride
objective type questions of two (2) marks each. All Guyon - Deceit
questions are compulsory. Duessa - Temperance
1. Whereas Sir Thomas Wyatt adapted Petrarch Orgoglio - Truth
and Petrarchanism to English sounds and (c) Una - Deceit
metres, Surrey's verse tends to look back beyond Guyon - Pride
Petrarch to the Duessa - Temperance
(a) French verse (b) Italian verse Orgoglio - Truth
(c) Spanish verse (d) Latin verse (d) Una - Temperance
Ans : (*) Wyatt and Surrey are known as 'Fathers of Guyon - Truth
the English Sonnet' for their excellent translations of Duessa - Pride
Petrarch's sonnets. Wyatt adapted Petrarch an style or Orgoglio – Deceit
Italian Model into English and Surrey gave then the Ans: (a) In Spenser’s ‘Faerie Queene’ –
rhyming meter and 14 lines division into three. Book I is centered on the virtue of Holiness in
Quatrains (4 lines) and a couplet (2 line). Thus, Redcrosse Knight; Book II on the virtue of Temperance
differentiating it from Petrarch an Octave (8 line) and in Sir Guyon; Book III on the virtue of Chastity in Sir
sestet (6 line). Britomart, a lady Knight; Book IV on the virtue of
Note– UGC has awarded equal marks to all candidates. Friendship in Sir Campbell and Sir Triamond; Book V
on the virtue of Justice in Sir Artegal and Book VI on
2. Here are some characteristics of Morality Plays: the virtue of Courtesy in Sir Calidore. Una personifies
1. They are dramatized allegories of the life of
Truth or True Church and defeats the deceitful Duessa
man. who symbolises Falsehood or False Church. Guyon
2. They depict man's temptation and sinning, his symbolises Temperance and Orgoglio symbolises
quest for salvation and his confrontation with Pride. Hence option (a) is correct.
Death.
3. Though the hero represents Mankind, the other 4. "Fop at the toilet, flatt'rer at the board
characters are by no means personifications, of Now trips a lady, a now struts a lord."
virtues, vices and death. The above lines are quoted from
(a) MacFlecknoe
4. A character known as the Vice often plays the
(b) The Rape of the Lock
role of the hero, a predecessor of the Villain-
(c) Epistle to Dr. Arbuthnot
hero in Elizabethan drama.
(d) Absalom and Achitophel
Find the correct combination according to the
code: Ans: (c) The above lines 'fop at the … lord' are taken
(a) Only 1 and 2 are correct. from Pope’s 'Epistle to Dr. Arbuthnot' (1735).
(b) Only 1 and 3 are correct. Hence option (c) is correct.
(c) Only 1 and 4 are correct. 5. Which of the following arrangements is in the
(d) Only 2 and 3 are correct. correct chronological sequence?
Ans: (a) Morality Plays are dramatized allegories of the (a) Every Man in His Humor
life of man and depict man’s temptation and sinning, The Shoemaker's Holiday
his quest for salvation and his confrontation with death. Antonio's Revenge
In these plays the hero represents mankind and other The Changeling
characters are personifications of virtues, vices and (b) The Shoemaker's Holiday
death. A character, Vice plays the role of pure villain. Every Man in his Humour
The Changeling
Hence option (a) only 1 and 2 is correct.
Antonia's Revenge
3. In Spenser's 'Faerie Queene' there are the (c) The Changeling
allegorized moral and religious virtues with Antonio's Revenge
their counterparts in the vices. Identify the Every Man in His Humour
correctly matched set : The Shoemaker's Holiday
(a) Una - Truth (d) Antonio's Revenge
Guyon - Temperance Every Man in His Humour
Duessa - Deceit The Changeling
Orgoglio - Pride The Shoemaker's Holiday
UGC NET/JRF English Paper III, June 2014 146 YCT
Ans : (a) The correct sequence in order of their Faustus' pride is evident in his overstatement that, he
performance is – has reached the end of every subject he has studied. He
B. Jonson’s ‘Every Man in His Humour’- 1598 appreciates Logic as being a tool for arguing; Medicine
T. Dekker’s ‘The Shoemaker’s Holiday’ – 1599 as being unvalued unless it allowed raising the dead
J. Marston’s ‘Antonio’s Revenge’ – 1600 and immortality; Law as being upstanding and above
T. Middleton and W. Rowley’s ‘The Changeling’- him; Divinity as useless because he feels that all
1622. humans commit sin and thus to have sins punishable by
Hence option (a) is correct. death complicates the logic of divinity. He dismisses it
6. Though Coleridge refers to "Motive-hunting of as–"What doctrine call you this? Que sera, sera" (what
a motiveless malignity", the "human villain" will be, shall be). He ends his soliloquy with the
Iago is far from "motiveless". His motives are decision to give his soul to the devil. This is how his
I. He has been disappointed of military quest for knowledge and power leads to his self-
promotion. destruction.
II. He suspects Othello of cuckolding him Hence option (c) I and IV is correct.
III. He has been in love with Desdemona 8. The centre of his plays is a proud character on
IV. He wants to become Othello. Marlowe's model, with a bold licence in speech
Find the most appropriate combination and action, full of elaborate metaphors, phrase
according to the code : tumbling after phrase, as he asserts himself in
(a) I and II are correct the French Court. Dryden unjustly described his
(b) I and III are correct style as "a dwarfish thought, dressed up in
(c) I and IV are correct gigantic words". Who is this Jacobean
(d) II and IV are correct
playwright?
Ans: (a) According to A.C. Bradley, "Iago stands (a) John Fletcher (b) John Webster
supreme among Shakespeare’s evil characters". Iago’s (c) George Chapman (d) John Marston
prime motives against Othello are– he has been
disappointed of military promotion and he suspects Ans: (c) The Jacobean playwright in question above is
Othello to have slept with his wife. (Cuckolding means George Chapman who initiated the comedy and tragi-
incest). His egoism is evident from the following comedy later made popular by Jonson and Beaumont
extracts from the play- "One Michael Cassio, a and Fetcher respectively.
Florentine, He modelled his central character on Marlowe’s
That never set a squadron in the field proud hero, with a bold licence in speech and action
But he, Sir, had the election …" [I. (i). 20-27] and full of elaborate metaphors, phrase after phrase.
"I hate the Moor (Othello) Dryden described his style as 'a dwarfish thought
And it is thought abroad that twixt my sheets dressed up in gigantic words'. Chapman was born in
Has done my office. I know not if ‘t be true ; 1559 and died in 1634. So, he belonged to Elizabethan
Yet I, for mere suspicion in that kind, and Jacobean periods.
Will do as if for surety". [I. (iii). 378-82] Elizabethan period – (1558-1603) – Reign of Queen
Note: Othello is referred to as ‘The Moor of Venice’ Elizabeth
which is also the sub-title of the play. Jacobean Period – (1603-1625) – Reign of King James
Thus his jealousy from Cassio and Othello is quite VI of Scotland as James I of England
clear. Hence option (a) I and II is correct. Chapman has retained till date his reputation as
7. In 'The Prologue' to Dr. Faustus, the chorus playwright, poet and translator of English Renaissance.
proposes that the theme should be- Hence option (c) is correct.
I. "cursed necromancy" 9. In Paradise Lost BK IX Milton writes that
II. "audacious deeds" Adam was overcome with "_____" and so ate
III. "dalliance of love" the forbidden fruit against his "better
IV. "self-conceit" knowledge".
The correct combination according to the code is (a) "female charm" (b) "exceeding love"
(a) I and II are correct (c) "faithful love" (d) "taste so divine"
(b) II and III are correct
Ans : (a) Milton writes in Paradise Lost (Bk IX) that
(c) I and IV are correct
(d) III and IV are correct Adam was overcome with female charm to eat the
forbidden fruit against his better knowledge.
Ans: (c) In the Prologue to Marlowe’s ‘The Tragical Hence option (a) is correct.
History of the Life and Death of Doctor Faustus (1592),
the chorus proposes the theme to be of – cursed 10. In which poem of Donne's is the lover's face
necromancy and self-conceit. Faustus' tale is compared reflected in the eyes of his beloved?
to that of 'Icarus' whose wings were melted by the sun (a) "The Good Morrow"
while flying too close to it. Faustus' end is hinted by the (b) "The Canonization"
idea of 'hubris' (excessive pride) leading to his downfall (c) "The Apparition"
similar to that of Icarus. (d) "A Valediction: Forbidding Mourning"
UGC NET/JRF English Paper III, June 2014 147 YCT
Ans : (a) In John Donne’s ‘The Good Morrow (1633) 13. Coleridge's "The Rime of the Ancient Mariner"
the lover’s face is reflected in the eyes of his beloved. is a poem in ______
Hence option (a) is correct. (a) 8 parts (b) 9 parts
11. Match List- I with List -II according to the code (c) 7 parts (d) 6 parts
given below : Ans: (c) Coleridge’s ‘Rime of the Ancient Mariner’,
List-I List-II published in ‘Lyrical Ballads’ (1798) with Wordsworth,
(Dramatists) (Plays) is a poem in 7 parts.
i. Thomas 1. The Provok'd Hence option (c) is correct.
Otway Husband
ii. William 2. The Recruiting 14. Scott is known for the creation of mad,
Wycherley Officer irrational witch-like women characters. From
iii. Colley 3. The Country the following list pick the odd one out :
Cibber Wife (a) Madge Wildfire (b) Meg Murdockson
iv. George 4. The Orphan, or the (c) Euphemia Deans (d) Meg Merrilees
Farquhar unhappy marriage Ans: (c) Meg Merrilees is the gipsy woman from
Codes : Scott’s 'Guy Mannering or The Astrologer' (1815).
i ii iii iv Madge Wildfire or Magdalene Murdockson is a mad-
(a) 4 3 1 2
(b) 3 2 1 2 woman and daughter of Meg Murdockson. And
(c) 4 2 3 1 Euphemia Deans is called the 'Lily of St. Leonards' and
(d) 3 1 2 4 is the sister of Jeanie Deans the first woman protagonist
and first character of Scott to come from the lower
Ans: (a) The correct matches are –
classes. Madge, Meg and Euphemia are characters from
i. Thomas Otway 4. ‘The Orphan, or the Scott’s ‘The Heart of Midlothian’ (1818), the 7th of his
unhappy marriage’ Waverly novels. So, Euphemia or Effie Deans is the
(1680) odd one because though accused of a severe crime of
ii. William 3. ‘The Country Wife’ killing her child she is not a mad-woman.
Wycherly (1675) Hence option (c) is correct.
15. Joseph Addison called him "The Miracle of the
iii. Colley Cibber 1. ‘The Provok’d present age" and Alexander Pope wrote the
with Vanbrugh Husband or , A epitaph for the monument erected in his memory.
Journey to London’ Who is he?
(1728) (a) John Locke (b) Isaac Newton
iv. George 2. ‘The Recruiting (c) Ashley Cooper (d) Christopher Wren
Farquhar Officer’ (1706). Ans: (b) The person in question here is Isaac Newton
Hence option (a) is correct. in whose memory Pope wrote an epitaph and Addison
12. "Thou wast not born for death immortal Bird." called him ‘The miracle of the present age.’
In what sense is the Bird "immortal" as Hence option (b) is correct.
compared to mortal man? 16. The play was first performed in 1773. The
I. Here man as an individual is unfairly author asked a friend "Did it make you laugh ?"
compared to a bird as a species. and getting the answer "Exceedingly" said then
II. The word "Bird" stands for the nightingale's that was all he required. He used for plot a
song. reputed experience of his own as a schoolboy
III. When considered as a species man is equally
when he lost his way and asked to be directed to
"immortal" as the "Bird".
IV. The "Bird" is "Immortal" because songs of an inn but was shown the gateway to the local
birds have given pleasure to man through the squire's house. Which play is this?
ages. (a) Sheridan's The Rivals
Find the correct combination according to the (b) Sheridan's The School for Scandal
code: (c) Goldsmith's She Stoops to Conquer
(a) Only I and III are correct (d) Goldsmith's The Good Natured Man
(b) Only IV is incorrect Ans: (c) The play in question here is Oliver
(c) Only II and IV are correct Goldsmith’s 'She Stoops to Conquer' (1773). Sheridan’s
(d) Only I and IV are incorrect ‘The Rivals’ was produced in 1775 and ‘The School for
Ans: (c) In the line 'Thou wast not born for death, Scandal’ in 1777. Goldsmith’s ‘The Good Natur’d
immortal Bird!, From Keats' Ode to a Nightingale', the Man’ was produced in 1768.
Bird stands for the nightingale’s song and is immortal
in the sense that songs of birds have given pleasure to Hence option (c) is correct.
man through the ages so it is not the physical bird but 17. What is Johnson's opinion regarding the
the melody of its song that is immortal. "Violation" of the three unities in the plays of
Hence option (c) is correct. Shakespeare?
UGC NET/JRF English Paper III, June 2014 148 YCT
I. Shakespeare should have followed the Unities. 21. Here are sentences labelled Assertion (A) and
II. Shakespeare followed the important Unity of Reason (R) :
Action satisfactorily. Assertion (A) : While referring to Charlotte
III. Shakespeare's plays suffered because they did Bronte's claim that she has excluded public interest
from her novels Graham Greene writes: 'Public
not follow the Unities.
interest in her day was surely more separate from
IV. Unity of Time and Place arise from false public life…. with us, however consciously
assumptions. unconcerned we are, it obtrudes through the cracks
The correct combination according to the code is of our stories terribly persistent like grass through
(a) I and II are correct. cement'.
(b) II and IV are correct. Reason (R): The decade of the "thirties was
(c) III and IV are correct. bristling with recurring economic and political
(d) I and III are correct. crisis like the Great Depression, Wall Street Crash,
Unemployment, rise of Hitler and Mussolini, series
Ans: (b) Shakespeare successfully broke from the of murders, invasions and tensions; writers could
classical tradition of the three unities and only not remain unaffected.
satisfactorily followed the Unity of Action in his plays In the light of (A) and (R) which of the following
and in Johnson’s opinion Unity of Time and Place arise is correct?
from false assumptions. Hence option (b) is correct. (a) Both (A) and (R) are true and (R) is the correct
18. The Tatler appeared thrice a week explanation of (A).
(a) On Tuesdays, Thursdays and Saturdays (b) Both (A) and (R) are true, but (R) is not the
correct explanation of (A).
(b) On Sundays, Tuesdays and Thursdays (c) (A) is true but (R) is false.
(c) On Mondays, Wednesdays and Fridays (d) (A) is false, but (R) is true.
(d) On Wednesdays, Thursdays and Fridays Ans: (a) Graham Greene’s observation of Charlotte
Ans : (a) Richard Steele’s ‘The Tatler’ first appeared in Bronte’s claim to exclude public interest from her
London on Tuesday April 12, 1709 and it subsequently novels is true in the light of recurring economic and
appeared thrice weekly on Tuesdays, Thursdays and political crises of the thirties.
Saturdays. The journal continued for two years. Hence option (a) Both (A) and (R) are true and (R) is
Hence option (a) is correct. the correct explanation of (A).
19. "No man is truly great, who is great only in his 22. Match the titles of the books with their authors:
List- I List- II
lifetime. The test of greatness is the page of i. "Psychology 1. John Strachey
history. Nothing can be said to be great that has and Art Today"
a distinct limit, or that borders on something ii. "Revolution in Writing" 2. W.H. Auden
evidently greater than itself. Besides, what is iii. "The Coming 3. C. Day Lewis
short-lived and pampered into mere notoriety, is Struggle for Power"
of a gross and vulgar quality in itself." This iv. "Arrow in the Blue" 4. Arthur Koestler
passage describing the quality of greatness is Codes :
taken from i ii iii iv
(a) "Of studies" by Francis Bacon (a) 3 1 2 4
(b) 4 2 3 1
(b) "The Indian Jugglers" by William Hazlitt
(c) 2 3 1 4
(c) 'Preface to Shakespeare' by Samuel Johnson (d) 1 2 4 3
(d) 'An Essay of Dramatic Poesy' by John Dryden
Ans: (c) The correct matches are –
Ans : (b) The above passage describing the quality of i. ‘Psychology and 2. W.H. Auden.
greatness is taken from – ‘The Indian Jugglers’ by Art Today’ (1935)
William Hazlilt, an essay published in ‘Table-Talk’ ii. ‘Revolution in 3. C. Day Lewis
(1821). Writing’ (1935)
Hence option (b) is correct. iii. ‘The Coming Struggle 1. John Strachey
20. In Blake's "The Human Abstract", the for Power’ (1932)
fragmented world of Experience is symbolized in iv. ‘Arrow in the Blue’ 4. Arthur Koestler.
the image of the (1952)
(a) Caterpillar (b) Fly Hence option (c) is correct.
(c) Raven (d) Fruit of Deceit 23. George Meredith's first novel was banned by
Ans: (d) In Blake’s ‘The Human Abstract’ (1794) the Mudie's Circulating Library for its supposed
fragmented world of experience is symbolized in the moral offence.
Identify the novel :
image of the- Fruit of Deceit. (a) "The Egoist"
This poem was published as a part of his collection (b) "Evan Harrington"
‘Songs of Experience’ in 1794. (c) "Diana of the Crossways"
Hence option (d) is correct. (d) "The Ordeal of Richard Feverel"
UGC NET/JRF English Paper III, June 2014 149 YCT
Ans : (*) George Meredith's first Major novel in 1859 iv ‘The Lady of 3. “On either side the
that caused much scandal but received critical Shalott’ river lie, Long fields of
appreciation was 'The Ordeal of Richard Feverel'. (1832) barley and of rye.”
Meredith had published his series of oriental fantasies All poems are by Lord Tennyson.
'The shaving of shagpat' in 1856 but received fame with
Hence option (a) is correct.
'The Ordeal.'
Note- UGC has awarded equal marks to all candidates. 25. Why are Elizabeth Barrett Browning's sonnets
called "From Sonnets from the Portuguese"?
24. Match the titles of the following poems by (a) She wrote the whole in Portugal
Tennyson with their opening lines according to (b) The sonnets were translated from the
the code given below : Portuguese.
List - I List- II (c) She presented it under the guise of a
(Titles of poems) (Opening Lines)
translation from the Portuguese language.
i. "Tithonus" 1. "Courage' he said,
and pointed towards (d) The sonnets were narrated by a Portuguese.
the land. The Ans: (c) Elizabeth Barrett Browning’s sequence of
mounting wave will Sonnets called 'Sonnets from the Portuguese' which was
roll us shoreward published in 1850 and includes 44 love sonnets, are so
soon." called because she presented it under the guise of a
ii. "The Lotos-Eaters" 2. "The woods decay, translation from the Portuguese language.
the woods decay and Hence option (c) is correct.
fall, The vapours 26. Yeats' "Sailing to Byzantium" is about
weep their burthen to (a) Irish Culture
the (b) The art and culture of Byzantium in general
ground." (c) Irish revolutionaries
iii. 'Ulysses' 3. "On either side the (d) Regenerating the art and culture that existed in
river lie
Byzantium
Long fields of barley
and of rye." Ans : (d) W.B. Yeats' 'Sailing to Byzantium' published
iv. 'The Lady of 4. "It little profits that in 1928 in ‘The Tower’ is about regenerating the art
Shalott' an idle king, and culture that existed in Byzantium.
By this still hearth, Hence option (d) is correct.
among these barren 27. "She had______ lilies in her hand
crags. Matched with And the stars in her hair were _____."
an aged wife, I mete (Rossetti's "The Blessed Damozel")
and dole Unequal (a) 7 and 3 (b) 3 and 7
laws unto a (c) 6 and 4 (d) 4 and 6
savage race."
Codes: Ans: (b) Rossetti in his 'The Blessed Damozel' (1850)
i ii iii iv wrote- “She had three lilies in her hand, And the stars
(a) 2 1 4 3 in her hair were seven.”
(b) 3 2 1 4 Hence option (b) is correct.
(c) 4 3 2 1 28. Which of the following arrangements is in the
(d) 2 4 3 1 correct chronological sequence?
Ans : (a) The correct matches are – (a) Adam Bede - Wuthering Heights - North and
i. ‘Tithonus’ 2. “The woods decay, the South - Villette
(1860) woods decay and fall. (b) Wuthering Heights - Villette - North and South
The vapours weep their - Adam Bede
burthen to the ground.” (c) Villette - North and South - Wuthering
ii. ‘The Lotus 1. “Courage he said, and Heights - Adam Bede
eaters’ (1832) pointed towards the (d) North and South - Wuthering Heights - Adam
land. The mounting Bede – Villette
wave will roll us Ans : (b) The correct chronological sequence is –
shoreward soon.” 1847 – Emily Bronte’s ‘Wuthering Heights’
iii. ‘Ulysses’ 4. “It little profits that an 1853 – Charlotte Bronte’s ‘Villette’
(1842) idle king, By this still 1855 – Mrs. Gaskell’s ‘North and South’
hearth, among these 1859 – George Eliot’s ‘Adam Bede’
barren crags. Matched
with an aged wife, I Hence option (b) is correct.
mete and dole, Unequal 29. In which of the following novels by Conrad do
laws unto a savage the Gould couple and Decoud appear as
race.” characters with Costaguana as the setting?
UGC NET/JRF English Paper III, June 2014 150 YCT
(a) Victory 33. Identify the poet in whose verse rural Ulster
(b) Under Western Eyes figures prominently
(c) Nostromo (a) Tony Harrison (b) Ted Hughes
(d) The Nigger of the Narcissus (c) Seamus Heaney (d) Louis MacNeice
Ans: (c) Costaguana is the setting and Gould couple Ans: (c) The rural Ulster figures prominently in poetry
and Decoud appear as characters in Joseph Conrad’s of Seamus Heaney.
'Nostromo' (1904). Hence option (c) is correct.
Hence option (c) is correct. 34. Match the pairs of authors and their works
30. Match the following plays with their authors according to the code given :
according to the code given below : List - I List - II
List-I List-II (Anthors) (Works)
(Plays) (Authors) i. Alexander Dumas 1. Remembrance of
i. Heartbreak 1. John Things Past
House Galsworthy ii. Honore de Balzac 2. Madame Bovary
ii. Loyalties 2. Bertolt Brecht iii. Gustav Flaubert 3. The Human Comedy
iii. In the Jungle of 3. T.S. Eliot iv. Marcel Proust 4. The Count of
Cities Monte Christo
iv. The Family 4. George Codes :
Reunion Bernard Shaw i ii iii iv
(a) 4 3 2 1
Codes:
(b) 1 2 3 4
i ii iii iv
(c) 2 1 4 3
(a) 3 4 2 1
(d) 3 4 1 2
(b) 1 2 3 4
(c) 2 1 4 3 Ans: (a) The correct matches are –
(d) 4 1 2 3 i. Alexander 4. ‘The Count of Monte
Ans : (d) The correct matches are– Dumas is the Christo’ (1845)
author of
i. 'Heartbreak 4. George Bernard
House' (1920) Shaw ii. Honore de 3. ‘The Human
was authored by Balzac is the Comedy’ (1829-47)
author of which is a series of
ii. 'Loyalties' (1922) 1. John Galsworthy interlinked novels,
was authored by novellas and stories
iii. 'In the Jungle of 2. Bertolt Brecht originally titled ‘La
Cities' (1923) Comedie Humaine’
was authored by in French
iv. 'The Family 3. T.S. Eliot iii. Gustav 2. ‘Madame Bovary’
Reunion' (1939) Flaubert is the (1856)
was authored by author of
Hence option (d) is correct. iv. Marcel Proust 1. ‘Remembrance of
31. In November 1910 in an exhibition organized by is famous for Things Past’ (1927
Roger Fry, the paintings of three painters were Fr., 1931 Eng.) also
displayed. Identify the painters : known as ‘In Search
(a) Duncan Grant, Vanessa Bell, Clive Bell of Lost Time.’ A
(b) Cezanne, Van Gogh, Gauguin literal translation of
(c) Matisse, Picasso, Braque the original French
(d) Cezanne, Van Gogh, Matisse title. ‘A la recherché
du temps perdu’
Ans: (b) In the exhibition organized by Roger Fry the
paintings of Cezanne, Van Gogh and Gauguin were Hence option (a) is correct.
displayed. 35. Which of the following statements best applies to
Hence option (b) is correct. Anna Karenina?
1. Among her most prominent qualities are her
32. Why did Phaedra, wife of Theseus, commit passionate spirit and determination to live life
suicide by hanging herself? on her own terms.
(a) Theseus hated her 2. She accepts the exile to which she has been
(b) Her stepson, Hippolytus rejected her love condemned.
(c) Hippolytus wanted to marry her 3. She is a victim of Russian patriarchal system.
(d) She was lonely and depressed 4. Anna is deeply devoted to her family and
Ans: (b) Phaedra committed suicide because her children.
stepson, Hippolytus rejected her love. (a) 1 and 2 are correct (b) 2 and 3 are correct
Hence option (b) is correct. (c) 1 and 3 are correct (d) 1, 3 and 4 are correct
UGC NET/JRF English Paper III, June 2014 151 YCT
Ans: (d) ‘Anna Karenina’ (1877) is a classic story of Ans: (a) Religious controversies in England during the
doomed love by Leo Tolstoy. She is a victim of 15th century led to the production of English prose.
Russian patriarchal system and is devoted to her family Major proponents were – Fisher, Cranmer, Tyndale,
and children. She is scorned for her relationship with Caxton, Reginald Peacock, Thomas More and Thomas
Count Vronsky and is condemned for her passionate Malory.
spirit and determination to live life on her own terms. Hence option (a) is correct.
Hence option (d) is correct. 40. Fill in the blanks with a suitable word from the
36. Match the pairs of authors and their works list below:
according to the code given : In his fiction, Ian McEwan more than often
List - I List - II suggests the______ of love
(Authors) (Works) (a) Fragility (b) Madness
i. Vladimir Nabokov 1. Germinal (c) Completeness (d) Security
ii. Italo Calvino 2. Foucault's Pendulum Ans: (a) In his fiction Ian McEwan portrays love as
iii. Umberto Eco 3. If on a Winter's "very fragile and difficult to attain and hard to keep and
Night a Traveller all the more precious for it."
iv. Emile Zola 4. Lolita Hence option (a) is correct.
Codes :
41. Match List- I with List - II according to the code
i ii iii iv
given below :
(a) 3 1 4 2
List-I List-II
(b) 4 3 2 1
(Dramatists) (Plays)
(c) 1 2 3 4 i. Arnold Wesker 1. Jumpers
(d) 2 4 1 3 ii. Harold Pinter 2. What the Butler Saw
Ans: (b) The correct matches are– iii. Joe Orton 3. The Room
i. Vladimir Nabokov 4. ‘Lolita’ (1955) iv. Tom Stoppard 4. Roots
was the author of Codes :
ii. Italo Calvino was 3. ‘If on a winter’s i ii iii iv
the author of night a Traveller’ (a) 3 2 4 1
(1979) (b) 1 2 4 3
iii. Umberto Eco was 2. ‘Foucault’s (c) 4 3 2 1
the author of Pendulum’ (1988) (d) 4 3 1 2
iv. Emile Zola was 1. ‘Germinal’ (1885) Ans: (c) The correct matches are
the author of i. Arnold 4. 'Roots' (1958) which
Hence option (b) is correct. Wesker is the is the second play of
author of the ‘Wesker Trilogy’
37. Which among the following plays by (1960), other two
Aristophanes is an attack on 'modern' education being Chicken soup
and morals as imparted and taught by the with Barley’ and ‘I’
radical intellectuals known as The Sophists? m talking about
(a) Clouds (b) Wasps Jerusalem’
(c) Acharnians (d) Knights
ii. Harold Pinter 3. 'The Room' (1957)
Ans: (a) Aristophanes' ‘Clouds’ is an attack on modern wrote which is his first play
education and morals as imparted and taught by the
radical Sophists. iii. Joe Orton’s 2. ‘What the Butler
Hence option (a) is correct. saw’ (1969) is a farce
on eroticism
38. In which novel of Virginia Woolf does a painter
iv. Tom 1. ‘Jumpers’ (1972)
in the act of painting actually figure as a
Stoppard’s satirises academic
character?
philosophy through
(a) The Voyage Out (b) The Waves
an alternative reality
(c) Jacob's Room (d) To the Lighthouse
where the moon is
Ans: (d) In V. Woolf’s ‘To the Lighthouse’ (1927) Lily conquered and this
Briscoe begins the novel as a young uncertain painter event unhinges the
attempting a portrait of Mrs. Ramsay and James. sanity of Dotty (a
Hence option (d) is correct. character) who thinks
39. Religious controversies in England particularly this event as
during the 15th century led to the promotion of revelation of the
(a) English prose once universal
(b) The British Empire human race as ‘little’
(c) Naval power and local
(d) The Missionary Movement Hence option (c) is correct.
UGC NET/JRF English Paper III, June 2014 152 YCT
42. Modern English emerged from the Ans: (b) Albert Camus in his essay ‘The Myth of
(a) South Midland dialect Sisyphus’ (1942) conveys –
(b) East Midland dialect (2) The Absurdity of Human Existence
(c) French language (3) The futility of all Human Endeavour and
(d) Northumbrian dialect (4) The concept of Existentialism.
Ans: (b) Modern English emerged from the East Hence option (b) is correct.
Midland dialect spoken around the London area. 48. In the 'Portrait of a Lady' Gilbert Osmond
Hence option (b) is correct. marries Isabel Archer because
1. Osmond wanted to get hold of Isabel's
43. Most culinary terms in English are derived from property.
(a) Exotic cooking (b) French cooking 2. He loved her
(c) Native sources (d) Arabic cooking 3. Though he did not like her moral ideas about
Ans: (b) Most culinary (cooking) terms in English are many things in life, he had hoped to win her
derived from French language. over.
Hence option (b) is correct. 4. He realized that her moral ideas were quite
deep-rooted.
44. "Blended learning" is a mode of Find the correct combination according to the
instruction/learning in which code:
(a) the learner's mother tongue and the target (a) only 1 and 2 are correct
language are blended (b) only 1, 2 and 3 are correct
(b) learning is accessed through the mother tongue (c) only 3 and 4 are correct
(c) a variety of instructional modes are integrated (d) only 1 is correct
(d) learning of a language is mediated by Ans: (*) In the 'Portrait of a Lady' Gilbert Osmond
humanistic approaches marries Ibsabel Archer because Osmond wanted to get
Ans: (c) ‘Blended learning’ is a mode of instruction in hold of Isabel's property. He did not like her moral ideas
which a variety of instructional modes are integrated. about many things in life, he had hoped to win her.
Gilbert Osmond's marriage with Isabel is the scheme of
Hence option (c) is correct.
Madame Merle who had an illegitimate daughter with
45. 'Risk-taking' is one of the traits of a good Osmond, Pansy.
(a) language learner Note– UGC has awarded equal marks to all candidates.
(b) language teacher 49. Pick out the two relevant and correct
(c) teacher of grammar rules descriptions of U.R. Ananthamurthy's
(d) printer of books and authors 'Samskara'.
Ans: (a) ‘Risk-taking’ is important in language 1. The novel is written in English
learning. 2. The novel is concerned with the progressive
Hence option (a) is correct. ideas of the times.
3. The novel is set in Malgudi
46. A teaching method advocated by Dr. Georgia 4. The novel is a satire on the representatives of a
Lozanav which is based on the principle of 'joy decadent Brahmin society.
and easiness' is called 5. Samskara is a regional novel
(a) Suggesto paedia 6. Praneschacharya does not atone for his sin.
(b) Total physical response (a) 4 and 5 are correct (b) 1 and 4 are correct
(c) The Direct Method (c) 5 and 6 are correct (d) 3 and 2 are correct
(d) The audio-lingual method Ans: (a) The novel ‘Samskara’ (1965) was originally
written in Kannada by U.R. Ananthamurthy. The
Ans: (a) ‘Suggestopedia’ is a teaching method
correct options are– (4) It is a satire on the
developed by the Bulgarian Psychotherapist Georgi representatives of a decadent Brahmin society, and (5)
Lozanav. It is based on the principle of joy and it is a regional novel.
easiness. Hence option (a) is correct.
Hence option (a) is correct. 50. Willy in Arthur Miller's play 'Death of a
47. Albert Camus, in his essay, 'The Myth of Salesman' compares Biff and Happy to the
Sisyphus' conveys : mythic characters/figures
1. The concept of Naturalism (a) Venus and Adonais
2. The Absurdity of Human Existence (b) Adonais and Hercules
3. The Futility of all Human Endeavour (c) Jupiter and Hercules
4. The concept of Existentialism (d) Venus and Hercules
(a) 1, 2 and 3 are correct Ans: (b) In Arthur Miller’s ‘Death of a Salesman’
(b) 2, 3 and 4 are correct (1949) Willy compares Biff and Happy to Adonais and
(c) 1, 2 and 4 are correct Hercules.
(d) 1, 3 and 4 are correct Hence option (b) is correct.

UGC NET/JRF English Paper III, June 2014 153 YCT


[Question Nos. 51 to 55 are based on a poem. Read leave something significant behind before departing
the poem carefully and pick out the most appropriate (from the world). So, sentence I, II, and III are
answers.] explaining the question.
A Valediction Forbidding Mourning Hence option (c) is correct.
My swirling wants, your frozen lips. 53. What does Rich imply when she says "The
The grammar turned and attacked me. grammar turned and attacked me"?
Themes, written under duress. (a) Language that has been used to hurt her.
Emptiness of the notations. (b) Her lover has beaten her.
(c) The person she is leaving is not the source of
They gave me a drug that slowed the healing of wounds. pain but something else.
I want you to see this before I leave: (d) The pain she has herself inflicted through
the experience of repetition as death language.
the failure of criticism to locate the pain the poster in the Ans: (a & d) Adrienne Rich says that 'grammar (that is
bus that said: male grammar) turned and attacked in the second line of
my bleeding is under control poem 'A valediction Forbidding Mourning', In this
A red plant in a cemetary of plastic wreaths. poem, language that has been used to hurt her (Rich)
and the pain she has herself in flicted through through
A last attempt: the language is a dialect called metaphor. language.
These images go unglossed: hair, glacier, flashlight. Thus, the correct answer is option (a & d).
When I think of a landscape I am thinking of a time. 54. How would you compare Rich's poem and
When I talk of taking a trip I mean forever. Donne's poem with the same title?
I could say : those mountains have a meaning but further (a) Rich is recreating Donne's poem
than that I could not say. (b) Rich is eulogising Donne's poem
To do something very common, in my own way. (c) Rich's poem is a scathing attack on Donne's
poem.
Adrienne Rich (d) Rich is defining Donne's concept of love
51. How does the poet suggest that the lover has not
Ans: (a) Donne was a 16th century metaphysical poet
left?
whose poem 'A valediction: Forbidding Mourning' is
(a) The words "a last attempt" indicate that she is
being recreated here by Adrienne Rich who is a 20th
trying her best to leave.
century poet.
(b) The words "before I leave" suggest that the
speaker has not left yet. Hence option (a) is correct.
(c) The speaker talks of a trip 'forever' which 55. What is the theme of the poem?
means she will never return. Identify the false statement in the list below:
(d) A drug she takes slows the healing of her It is
wounds perhaps indicating that she may be (a) about the difficulty of actually saying goodbye
able to leave sometime in future. (b) about not having the strength to leave though
one might want to.
Ans: (b) The words ‘before I leave’ in line 7 of Rich’s
(c) about the pain suffered in relationship.
poem suggest that the speaker has not left yet. (d) a Classical love poem like Donne's where the
Hence option (b) is correct. speaker dominates the addressee.
52. Why does the speaker/ lover in Rich's poem plan Ans : (d) The theme of the poem is –
to leave? (a) about the difficulty of actually saying goodbye.
I. Because her love has not been returned. (b) about not having the strength to leave though one
II. Because of the pain she has suffered in the might want to
relationship.
(c) about the pain suffered in relationship.
III. Because the lover has criticized her so much.
Option (d) is the false statement because although this
IV. Because though the pain has been located, the
poem is a recreation of the classical mode, it is a
bleeding continues.
modern poem in which the speaker has been dominated
The right combination according to the code is
and hurt by the addressee.
(a) I and II are correct
(b) I and IV are correct Hence option (d) is correct.
(c) I, II and III are correct 56. Why does Girish Karnad base his play
(d) I and III are correct Hayavadana on Thomas Mann's Transposed
Ans: (c) The speaker wants to leave because her poetry Heads?
or feelings of love were not acknowledged or returned (a) It is a mock-heroic transcription of the original
by her male counterpart. Instead, she is verbally Sanskrit tales.
attacked which is the cause of her suffering and pain. (b) It is concerned with materialism.
They did not even let her wounds to heal and kept (c) It deals with domestic strife.
criticizing her but this made her more determined to (d) It deals with ancient times.
UGC NET/JRF English Paper III, June 2014 154 YCT
Ans: (a) Girish Karnad’s ‘Hayavadana’ (1970) is based 60. Which of the following is not a play by Badal
on Thomas Mann’s ‘Transposed Heads’ which is a Sircar?
mock-heroic transcription of the original Sanskrit tales. (a) Bhooma (b) Evam Indrajeet
Hence option (a) is correct. (c) That Other History (d) Agra Bazar
57. The collected poems of A.K. Ramanujan has Ans: (d) Badal Sircar has authored –
been divided into four sections. Arrange them in ‘Bhoma’ (1979), ‘Ebang Indrajit’ (1963, trans. ‘Evam
their chronological order: Indrajeet’, 1975), Baaki Itihaash (1965, trans. ‘That
(a) The Striders - The Relations - Second Sight - Other History’)
the Black Hen ‘Agra Bazar’ (1954) is authored by Habib Tanvir.
(b) The Relations - The Striders - The Black Hen -
Hence option (d) is correct.
Second Sight
(c) Second Sight - The Relations - The Black Hen 61. Who is the protagonist of Shashi Deshpande's
- Striders That Long Silence?
(d) The Black Hen - Second Sight - The Striders- (a) Mohan (b) Jaya
The Relations (c) Rati (d) Kamat
Ans: (a) A.K. Ramanujan’s poems in correct Ans: (b) Jaya is the protagonist of Shashi Deshpande’s
chronological sequence are – ‘That Long Silence’ (1988).
1966 - The Striders Hence option (b) is correct.
1971 - The Relations 62. In Derek Walcott's Dream on Monkey
1986 - Second Sight Mountain, Makak's vision of freedom for his
1995 – The Black Hen. people is
Hence option (a) is correct. (a) through money
58. In one of her novels, Margaret Atwood (b) through violence
demonstrated the potentially 'Cannibalistic' (c) through black power
nature of human relationships. Identify the (d) through a decolonisation of the mind
novel : Ans: (d) In Derek Walcott’s ‘Dream on the Monkey
(a) Surfacing (b) Lady Oracle Mountain’ (1970) Makak’s vision of freedom for his
(c) Life Before Man (d) The Edible Woman people is through a decolonization of the mind.
Ans: (d) Margaret Atwood has demonstrated, in her Hence option (d) is correct.
1969 novel ‘The Edible Woman’, the potentially 63. Given below are two statements, one labelled as
‘Cannibalistic’ nature of human relationships.
Assertion (A) and the other as Reason (R).
Hence option (d) is correct. Assertion (A): To give a text an author is to
59. Match the characters with the novels of Amitav impose a limit on that text, to furnish it with a
Ghosh in which they appear according to the final signified, to close the writing.
code given below: Reason (R): A text is made up of multiple
List-I List-II meanings drawn from many sources, and this
(Characters) (Novels) multiplicity is focused on the reader.
i. Fakir 1. The Glass Palace
In the context of the two statements which one of
ii. Tridip 2. The Hungry Tide
iii. Rajkumar 3. The Calcutta the following is correct :
Chromosome (a) Both (A) and (R) are true and (R) is the correct
iv. Murugan 4. Shadow Lines explanation of (A).
Codes : (b) Both (A) and (R) are true and (R) is not the
i ii iii iv correct explanation of (A).
(a) 2 4 1 3 (c) (A) is true but (R) is false.
(b) 2 4 3 1 (d) (A) is false but (R) is true.
(c) 1 3 1 4 Ans: (b) Both Assertion and Reason are true but
(d) 3 2 4 1 Reason does not explain the assertion as it is talking of
Ans: (a) The correct matches are– author and reason is talking about reader.
Hence option (b) is correct.
i. Fakir is a 2. ‘The Hungry Tide’
character in (2004) 64. Given below are two statements, one labelled as
ii. Tridip is a 4. ‘The shadow lines’ Assertion (A) and the other as Reason (R).
character in (1988) Assertion (A) : Spivak sees the project of
colonialism as characterized by what Foucault had
iii. Raj Kumar is a 1. ‘The Glass Palace’
called 'epistemic violence', the imposition of a
character in (2000)
given set of beliefs over another.
iv. Murugan is a 3. ‘The Calcutta Reason (R) : Spivak suggests that participation in
character in Chromosome.’ the political process - access to citizenship,
(1995) becoming a voter - will help to mobilize the
Hence option (a) is correct. subaltern on "the long road to hegemony."
UGC NET/JRF English Paper III, June 2014 155 YCT
In the context of the two statements, which one Ans: (a) The correct matches are–
of the following is correct: i. Langston Hughes 3. ‘Don’t You
(a) Both (A) and (R) are true and (R) is the correct Want to be Free’
explanation of (A). (1939)
(b) Both (A) and (R) are true and (R) is not the
ii. Lorraine Hansberry 4. ‘A Raisin in the
correct explanation of (A).
Sun’ (1959)
(c) (A) is true but (R) is false.
(d) (A) is false but (R) is true. iii. Ed Bullins 2. ‘Clara’s Ole
Ans: (b) Both Assertion and Reason are true but reason Man’ (1969)
does not explain the assertion as it is about beliefs and iv. Amiri Baraka 1. ‘Dutchman’
reason is about hegemony literally. (1964)
Hence option (b) is correct. Hence option (a) is correct.
65. Match the following authors with their works 67. Identify the critics and their respective works:
from the given below: (a) Horace - Ars Poetica
List-I List-II Aristotle - Poetics
(Authors) (Works) Quintillian - Institutio Oratoria
i. Buchi Emecheta 1. Burger's Daughter Ben Jonson - Discoveries
ii. Ama Ata Aidoo 2. Joys of Motherhood Sidney - An Apology for Poetry
iii. Nadine Gordimer 3. Devil on the Cross Dryden - An Essay of Dramatic Poesy
iv. Ngugi Wa Thiongo 4. Our Sister Killjoy (b) Horace - Poetics
Find the correct combination according to the Aristotle - Ars Poetica
code : Quintillian - On the sublime
Codes : Longinus - Discoveries
i ii iii iv Ben Jonson - Institutio Oratoria
(a) 1 2 3 4 Sidney - An Essay of Dramatic Poesy
(b) 2 4 1 3 Dryden - An Apology for Poetry
(c) 3 1 4 2 (c) Horace - On the sublime
(d) 4 3 2 1 Aristotle - Poetics
Ans: (b) The correct matches are– Quintillian - Discoveries
Longinus - Institutio Oratoria
i. Buchi Emecheta is 2. ‘Joys of Ben Jonson - An Essay of Dramatic Poesy
the author of Motherhood’ Sidney - Ars Poetica
(1979) Dryden - An Apology for Poetry
ii. Ama Ata Aidoo is 4. 'Our Sister Killjoy' (d) Horace - Ars Poetica
the author of (1977) Aristotle - Poetics
iii. Nadine Gordimer 1. 'Burger’s Quintillian - Institutio Oratoria
authored Daughter' (1979) Longinus - On the Sublime
iv. Ngugi Wa Thiongo 3. 'Devil on the Ben Jonson - An Apology for Poetry
authored Cross' (1980, Eng. Sidney - An Essay of Dramatic Poesy
trans. 1982) Dryden – Discoveries
Ans : (a) Horace is the author of ‘Ars Poetica’ (19 BC.
Hence option (b) is correct.
circa)
66. Match the following authors with their plays Aristotle - ‘Poetics’ (335BC circa)
from the lists given below :
Quintillian - ‘Institutio Oratoria’ (95AD circa)
List - I List-II
Ben Jonson - ‘Discoveries’ or ‘Timber’ (1640)
(Authors) (Plays)
i. Langston Hughes 1. Dutchman P. Sidney – ‘An Apology for Poetry; (1595)
ii. Lorraine Hansberry 2. Clara's Ole Man J. Dryden – ‘An Essay of Dramatic Poesy’ (1668)
iii. Ed Bullins 3. Don't You want Hence option (a) is correct.
to be Free 68. Which of the following is not true of Imagist
iv. Amiri Baraka 4. A Raisin in the Sun poetry?
Find the correct combination according to the (a) The poet spreads his language across the page
code: as though language were sensation, to
Codes : reproduce the mental effect of 'image'.
i ii iii iv (b) The image is itself an instrument of vision, or
(a) 3 4 2 1 lens, as well as an expression of imagination
(b) 1 2 3 4 (c) The imagist like a scientist learns from history
(c) 2 1 4 3 and uses it, and like a scientist does not deal in
(d) 4 3 1 2 emotions.
UGC NET/JRF English Paper III, June 2014 156 YCT
(d) The new artist as scientist focuses vision Ans: (a) To Frantz Fanon the ‘Negro’ village is the
through image as against the symbol which place where people live in wretched conditions, are
resorts to reduction to simplicity. suffering terribly from apartheid (racial discrimination)
Ans: (c) 'Imagism' is the name given to a movement in and the town is crouching under moral and physical
poetry originating in 1912 and represented by Ezra degradation.
Pound, Amy Lowell and a group of American and Hence option (a) is correct.
English Poets who wrote succinct verse of dry clarity 72. Why is the 'native town' a hungry town?
and hard outline presenting an exact visual image to 1. it did not have agricultural farms
make a total poetic statement in free verse with 2. it did not have markets
absolute freedom of subject without any help of 3. the blacks were steeped in poverty
conventional forms. So it is certain that they did not 4. they were denied their fundamental rights by
base their poetry on history. the Whites.
Hence option (c) is not true. (a) 1 and 2 are correct (b) 3 and 4 are correct
(c) only 1 is correct (d) only 4 is correct
69. Who among the following is not a myth critic?
(a) Robert Graves (b) Raymond Williams Ans: (b) In this extract from Frantz Fanon’s ‘The
(c) Francis Fergusson (d) Northrop Frye Wretched of the Earth’ (1916). The lines 7-9 describe
the native town as a hungry town, that is steeped in
Ans: (b) Raymond Williams was a Marxist critic who
poverty and natives are living insignificant lives having
had decisive influence on the formation of cultural denied any rights to a life of dignity and prosperity by
studies all others are related to myth criticism. the colonizers.
Hence option (b) is correct. Hence option (b) is correct.
70. According to Northrop Frye there are four main 73. What does the term 'crouching village' indicate?
narrative genres associated with the seasonal 1. The latent aggressiveness of the blacks
cycle of spring, summer, autumn and winter. 2. The defenselessness of the people
They are comedy, ______, tragedy and irony 3. Hopelessness and despair
(satire). Which is the second one? 4. Overflowing filth
(a) Romance (b) Epic (a) 1 and 2 are correct (b) 2 and 3 are correct
(c) Fiction (d) Novel (c) only 1 is correct (d) only 2 is correct
Ans: (a) Northrop Frye associated – Ans: (c) The term 'crouching village' here indicates the
Spring - Comedy latent aggressiveness of the blacks who are bound
Summer – Romance under the rule of the colonizers but aspire to attain
Autumn- Tragedy freedom or take the place of the settler. (Last line)
Winter – Irony or Satire Hence option (c) is correct.
Hence option (a) is correct. 74. Why does the native look at the settler's town
Questions No. 71-75 are based on the following with envy?
passage: 1. it arises from a sense of desperation
Read the passage carefully and select the most 2. he has no other option in his life
3. he wants to occupy a position of power.
appropriate option.
4. he want to be the colonizer instead of the
The town belonging to the colonized people, or at
colonized.
least the native town, the negro village, the medina, the
(a) only 1 is correct (b) 3 and 4 are correct
reservation, is a place of ill fame, peopled by men of
(c) only 2 is correct (d) 1 and 4 are correct
evil repute. They are born there, it matters not where,
nor how. The native town is a hungry town, starved of Ans: (b) The native has suffered for so long and so
much that he looks at the settler enviously aspiring to
bread, of meat, of shoes, of coal, of light. The native
occupy the same power and position as the settler.
town is a crouching village, town on its knees, a town
wallowing in the mire. The look that the native turns on Hence option (b) is correct.
the settler is a look of lust, of envy…. The colonized 75. What is the settler's attitude towards the blacks?
man is an envious man. And this the settler knows very 1. the settler is not afraid
well.. It is true, for there is no native who does not 2. the settler considers the blacks to be harmless
dream at least once a day of setting himself up in the 3. the settler is contemptuous of the blacks.
settler's place. 4. the settler feels resentment because he knows
(From Frantz Fanon's The Wretched of The Earth) that his position is never safe.
71. To Frantz Fanon, the 'Negro' village is (a) only 1 is correct (b) 2 and 3 are correct
1. the worst face of apartheid (c) only 4 is correct (d) 3 and 4 are correct
2. a protected area Ans: (c) The settler is cautious of the inevitable truth
3. a place of moral and physical degradation that he will have to let the natives their freedom
4. a special village with its own amenities. someday. So he knows that his position as settler is not
(a) 1 and 3 are correct (b) 1 and 2 are correct safe forever.
(c) only 3 is correct (d) only 4 is correct Hence option (c) is correct.

UGC NET/JRF English Paper III, June 2014 157 YCT


UGC NET/JRF Exam, June-2014
ENGLISH
Solved Paper-II
Note : This paper contains fifty (50) objective 3. The separation of styles in accordance with class
type questions of two (2) marks each. All questions appears more consistently in______ than in
are compulsory medieval works of literature and art.
1. "The just man justices". What kind of (a) Ben Jonson (b) Shakespeare
foregrounding do you find in the above lines? (c) Philip Sidney (d) Edmund Spenser
(a) Syntactic (b) Semantic Ans: (b) The separation of styles in accordance with
(c) Collocation (d) None of the above class appears more consistently in Shakespeare whose
Ans: (a & b) In the line 'The just man justices' The plays were audience centric and relevant for all ages.
linguistic device used is syntactic as the morphemes In that era it was important to please the patrons who
'just' and 'justices' are in the same order. Morpheme is supported the play wrights and belonged majorily to
the smallest grammatical unit in a language. the higher classes. Hence option (b) is correct.
Foregrounding is the action of emphasizing by means of 4. "Had we but world enough, and time, This
linguistic devices like syntactic, collection etc. coyness, lady, were no crime." This statement is
In linguistics, semantics is the subfield that studies an example of
meaning. Semantics can address meaning at the levels (a) Irony (b) Paradox
of words, phrases, sentences, or larger units of (c) Hyperbole (d) Euphemism
discourse.
Ans: (a) Irony means the use of words to convey a
Note:- Hence, option (a) and (b) both will be correct. meaning that is the opposite of its literal meaning.
2. Match the items in List-I with items in List- II Paradox is a self-contradictory statement that may
according to the code given: express a possible truth.
List- I List -II Hyperbole is an overstatement.
i. Iambic 1. Two unstressed syllables Euphemism is substitution of a harsh seeming
followed by a stressed statement with a mild one.
syllable In these lines from ‘To his coy mistress’ by Andrew
ii. Anapaestic 2. A stressed is followed Marvell, the figure of speech used is irony. Hence
by two unstressed syllables. option (a).
iii. Dactylic 3. An unstressed syllable 5. A Spenserian stanza has
followed by a stressed (a) four iambic pentameters
syllable (b) six iambic pentameters
iv. Trochaic 4. A stressed syllable is (c) eight iambic pentameters
followed by an (d) ten iambic pentameters
unstressed syllable Ans: (c) A Spenserian stanza has eight iambic
Codes : pentameter (five feet) lines and a final Alexandrine
i ii iii iv (iambic hexameter or six feet) with a rhyme scheme
(a) 2 1 3 4 of a b a b b c b c c.
(b) 3 2 1 4 Hence option (c).
(c) 4 1 2 3 6. Match the items in List-I with items in List-II
(d) 3 1 2 4 according to the code given below:
Ans: (*) Note– NTA has dropped this question and List-I (Critic) List-II (Theory)
distributed equal marks to all the candidates. i. Cleanth Brooks 1. Ambiguity
i. Iambic 3. An unstressed syllable ii. William Empson 2. Paradox
followed by a stressed iii. Mark Schorer 3. Archetypal
syllable patterns in poetry
ii. Anapaestic 1. Two unstressed syllables iv. Maud Bodkin 4. Techniques as
iii. Dactylic 2. A stressed is followed by discovery
two unstressed syllables. Codes:
iv. Trochaic 4. A stressed syllable is i ii iii iv i ii iii iv
followed by an unstressed (a) 2 1 4 3 (b) 3 2 1 4
syllable (c) 1 2 3 4 (d) 2 3 4 1
UGC NET/JRF English Paper II, June 2014 158 YCT
Ans: (a) Correct matches are – iv. Eve 2. ‘Epistemology of the
i. Cleanth Brooks 2. paradox in his Kosofsky Closet’ published in
outlines the use essay Sedgwick 1991.
of reading ‘The Language of wrote
poems through Paradox’. Hence option (b) is correct.
ii. William Empson 1. ambiguity in his 9. "The greatness of a poet", Arnold says, "lies in
revolutionized ‘Seven Types of
his powerful and beautiful application of ideas to
ways of reading Ambiguity: A
a poem by Study of its Effect life." But a critic pointed out it was "not a happy
describing on English Verse’ way of putting it, as if ideas were a lotion for the
(1930). inflamed skin of suffering humanity". Who was
iii. Mark Schorer’s 4. –‘Techniques as this critic?
essay discovery’ (1947) (a) T.S. Eliot (b) F.R. Leavis
was a critical work (c) David Lodge (d) Allen Tate
on fiction. Ans: (a) In his Norton Lectures titled ‘Use of Poetry
iv. Maud Bodkin 3. ‘Archetypal and Use of Criticism’ (1932-33) T. S. Eliot criticised
wrote Patterns in Poetry’ Arnold’s criticism in the above words. Hence option
in 1934. (a) is correct.
Hence option (a) is correct. 10. Derrida's American disciples were
7. "The artist may be present in his work like God (a) Geoffrey Hartman, Paul de Man, J. Hills Miller
in creation, invisible and almighty, everywhere
(b) Gertrude Stein, Barbara Ryan, Mary Ellman
felt but nowhere seen." Henry James is talking
here about the artist's (c) Barbara Johnson, Michael Ryan, Mary Ellman.
(a) impersonality (b) absence (d) Jean Baudrillard. Gilles Deleuze, Felix Guattari
(c) presence (d) creativity Ans: (a) Derrida’s American disciples were Geoffrey
Ans: (a) In these lines Henry James says that the Hartman, Paul de Man and J. Hills Miller and many
creator remains invisible thus he is talking about his others. Hence option (a) is correct.
impersonality. Hence option (a) is correct. 11. Identify the correct group of playhouses in late
8. Match the items in List-I with items in List-II sixteenth century London from the following
according to the code given below: groups.
List-I List- II (a) Curtain, Rose, Swan, Globe, Hope
(Theorist) (Book) (b) Curtain, Rose, Swan. Globe, Sejanus
i. Michel Foucault 1. Gender Trouble (c) Hope, Curtain, Rose, Swan, Globe
ii. Judith Butler 2. Epistemology of (d) Swan, Curtain, Rose, Globe, Thames
the Closet Ans: (a & c) The correct group of playhouses in late
iii. Alan Sinfield 3. History of Sexuality sixteenth century London is option (a) and option (c)
iv. Eve Kosofsky 4. Cultural Politics- both correct. According to the UGC.
Sedgwick Queer Reading 'The Theatre' was the first London playhouse built by
Which is the correct combination according to James Burbage in 1576 in shoredich. 'The Curtain' was
the code : the second London playhouse built by Henry Laneman
Codes: in 1597.
i ii iii iv i ii iii iv The 'Hope Theatre' was one of the theatres built in and
(a) 3 1 2 4 (b) 3 1 4 2 around London, comparable to the Globe, the curtain,
(c) 4 2 1 3 (d) 4 3 1 2 and the swan.
Ans: (b) Correct matches are– 12. "Keep up your bright swords, for the dew will
i. Michel 3. 'History of Sexuality' rust them.
Foucault published in three Good Signior, you shall more command with
wrote volumes from 1976- years.
84.
Than with your weapons."
ii. Judith Butler 1. ‘Gender Trouble: The above lines are addressed by Othello to
wrote Feminism and the
(a) Roderigo and officers
Subversion of Identity
and Bodies That (b) Brabantio, Roderigo and Officers
Matter’ published in (c) The Duke and Senators
1990. (d) Montano and Cassio
iii. Alan 4. ‘Cultural Politics- Ans: (b) In the above lines, taken from Act I scene 2,
Sinfield Queer Reading’ Othello addresses Brabantio, Roderigo and Officers.
wrote published in 1994. Hence option (b) is correct.
UGC NET/JRF English Paper II, June 2014 159 YCT
13. Act V of Marlowe's ‘Edward the Second’ shows Ans: (c)
the murder of the king. Where does it take place? i. Lucy Hutchinson 4. 'Memoirs of the
(a) Westminster, a room in the palace
wrote Life of Colonel
(b) A room in Berkeley Castle
(c) A room in Killingworth Castle Hutchinson' (1806
(d) Within the Abbey of Neath Dos Thumous)
Ans: (b) Act V of Marlowe’s ‘Edward the Second’ ii. John Bunyan wrote 1. 'The Life and
takes place in a room in Berkeley Castle. Hence Death of Mr.
option (b) is correct.
Badman' (1680)
14. Identify the correctly matched set:
iii. John Evelyn wrote 2. 'Sylva: or a
(a) "The Shepheards Calender"- 1579
Tottel’s Miscellany- 1557 Discourse of Forest
Astrophel and Stella- 1591 Trees' (1662).
The Spanish Tragedie- about 1585 iv. Margaret Cavendish 3. 'Nature's Pictures'
(b) "The Shepheards Calender"- 1559 wrote (1656).
Tottel;s Miscellany-1579 Hence option (c) is correct.
Astrophel and Stella- 1585 16. "But deeds, and language, such as men do use;
The Spanish Tragedie-about-1591 And persons, such as comedy would choose,
(c) "The Shepheards Calender"-1585 When she would show an image of the time,
Tottel’s Miscellany-1591
and sport with human follies, not with crime."
Astrophel and Stella-1579
In the above lines Jonson
The Spanish Tragedie- about 1557
I. Opposes the artificiality of the romantic
(d) "The Shepheards Calender"-1579 tragic-comedy.
Tottel’s Miscellany-1591
II. Initiates the use of realism.
Astrophel and Stella- about 1585
III. Considers analysis of moral short comings
The Spanish Tragedie- about 1557 more important
Ans: (a) ‘The Shepheards Calender’ is Spenser’s IV. Encourages the use of farce with
earliest notable work published in 1579. melodrama.
‘Tottel’s Miscellany’ is a compilation of songs and Find out the correct combination according to
sonnets chiefly by Wyatt and Surrey compiled by the code:
Richard Tottel in 1557. (a) I, II and III are correct
‘Astrophel and Stella’ is a sequence of 108 sonnets
(b) I, II and IV are correct
and 11 songs by Sir P. Sidney which came out in 1591
(c) I, III and IV are correct
and from 1598 was included in his ‘The Arcadia’.
(d) II, III and IV are correct
‘The Spanish Tragedie’ by Kyd is related to the
victory of Spain over Portugal in 1580 and it was Ans: (a) In the above lines Jonson–(I) Opposes the
acted and printed about 1582-92. Hence option (a) is artificiality of the romantic tragic-comedy by
correct. commenting on the 'deeds, and language' as used by
15. Match the items in the List-I with items in List- men in representing persons of tragic character in
II according to the code given below: comedy.
List-I List-II He then–(II) Initiates the use of realism when he says
that plays should ‘show an image of the time’ or
(Authors) (Works)
society of that age. Furthermore,
i. Lucy Hutchinson 1. The Life and Death of
Mr. Badman He–(III) Considers analysis of moral short-comings
more important when he says that plays should ‘sport
ii. John Bunyan 2. Sylva: or a Discourse
with human follies’ or moral values instead of just
of Forest Trees
presenting sensational stories of crime and violence.
iii. John Evelyn 3. Natures Pictures
Jonson thus discourages the use of farce with
iv. Margaret Cavendish 4. Memoirs of the Life of melodrama. Hence option (a) is correct.
Colonel Hutchinson
Codes: 17. "And if no piece of chronicle we prove,
i ii iii iv i ii iii iv We'll build in ______ pretty roomes."
(a) 2 3 1 4 (b) 4 3 2 1 (a) lyrics (b) epics
(c) 4 1 2 3 (d) 4 2 1 3 (c) sonnets (d) stanzas
UGC NET/JRF English Paper II, June 2014 160 YCT
Ans: (c) John Donne in ‘Canonization’ wrote these 22. Which of the following works is not actually a
lines– prose essay?
‘And if no piece of chronicle we prove, (a) Essay of Dramatic Poesy
We’ll build in sonnets pretty rooms.’ (b) Essay on Man
Hence option (c) is correct. (c) An Essay Concerning Human Understanding
18. "That glory never shall his wrath or might (d) An Essay Towards a New Theory of Vision
extort from me." (Paradise Lost, Book I) Ans: (b) ‘Essay on Man’ (1734) by Alexander Pope is
What 'glory' is being referred to by Satan? a philosophical poem. Hence option (b) is correct.
(a) The courage never to submit or yield 23. Whom does Mirabell deceive into believing that
(b) To reign in Hell he loves her in 'The way of the World'?
(c) To defeat God (a) Millamant (b) Lady Wishfort
(d) To spread evil (c) Mrs. Marwood (d) Mrs. Fainall
Ans: (a) In the above lines, from Milton’s ‘Paradise Ans: (b) In Congreve’s play ‘The Way of the World’
Lost’ (Book I), Satan is referring to the courage never (1700) Mirabell deceives Lady Wishfort and proposes
to submit or yield. Hence option (a) is correct. her neice Lady Millamant for marriage, the deceit
19. It has been described as a "novel without broken to Lady Wishfort by Mrs. Marwood.
predecessors", the product of an original mind Hence option (b) is correct.
and became immediately popular. It is a 24. "Competence to age is supplementary to youth,
peculiar blend of pathos and humour, though a sorry supplement indeed, but I fear the best
the pathos is sometimes overdone to the point of that is to be had. We must ride where we
becoming offensively sentimental.
formerly walked: live better and be softer and
The novel was published in 1960. What is the shall be wise to do so than we had means to do in
name of the novel?
the good old days you speak of."
(a) Gulliver's Travels
(b) The Castle of Otranto Who speaks these words and to whom?
(c) Tristram Shandy (a) Lamb to Bridget
(d) A Tender Husband (b) Wordsworth to Dorothy
Ans: (c) Laurence Sterne’s ‘The Life and Opinions of (c) Dorothy to Bridget
Gentleman Tristram Shandy’ (1760-67, 9 volumes) (d) Lamb to Dorothy
has been described as a novel without predecessors Ans: (a) The above lines ‘Competence to age…….
and original in style and construction. In vol. VI this you speak of’ have been spoken by Lamb to his sister
book is introduced as a Shandean Book. ‘Shandy’ Bridget in his essay ‘Old China’ from his ‘Last Essays
means of obscure birth or crack-brained like Tristram of Elia’ (1833).
Shandy. Hence option (c) is correct.
Hence option (a) is correct.
20. The son of a joiner, he was apprenticed as a
printer. He remained a printer throughout his 25. 'The Prelude' although begun as early as 1799
life. He was asked to prepare a series of modern and finished in its first version in 1805, was not
letters for those who could not write for published until______.
themselves. This humble task taught him the art (a) 1815 (b) 1820
of expressing himself in letters. Who is the (c) 1830 (d) 1850
novelist? Ans: (d) Wordsworth’s ‘The Prelude’ was begun in
(a) Daniel Defoe (b) Samuel Richardson 1799 but could only be published after his death in
(c) Henry Fielding (d) Tobias Smollett 1850 by his wife.
Ans: (b) The novelist in question here is Samuel Hence option (d) is correct.
Richardson who was a son of a joiner (a sort of a
carpenter higher in status than what is now). He 26. "A rosy sanctuary will I dress With the
remained a printer throughout his life. The novel wreathed trellis of a working brain." The above
talked about here is ‘Pamela: or Virtue Rewarded’ lines are quoted from
(1740) considered also the first English novel. (a) 'Adonais' (b) 'Ode to Psyche'
Hence option (b) is correct. (c) 'Eve of St. Agnes' (d) 'Endymion'
21. "Where ignorance is Bliss Tis folly to be wise." Ans: (b) The lines ‘A rosy sanctuary ………working
who wrote the following lines? brain’ appear in John Keats' poem ‘Ode to Psyche’
(a) Pope (b) Gray written in 1819.
(c) Collins (d) Southey Hence option (b) is correct.
Ans: (b) The lines ‘Where ignorance is Bliss Tis folly 27. "Love seeketh only self to please,
to be wise’ appears in Thomas Gray’s ‘Elegy written To bind another to its delight."
in a Country Church Yard’ (1751). This selfish and possessive nature of love is
Hence option (b) is correct. illustrated in Blake's
UGC NET/JRF English Paper II, June 2014 161 YCT
(a) 'The Clod and the Pebble' 31. Which among the following novels was not
(b) 'The Sick Rose' written in 1922?
(c) 'A Poison Tree' (a) Ulysses (b) Jacob's Room
(d) 'Ah Sunflower' (c) Aaron's Rod (d) A Passage to India
Ans: (d) “A Passage to India’ by E.M. Forster was
Ans: (a) William Blake illustrated love’s selfish
published in 1924. It is a novel by E.M. Forster set
nature in the above lines in his poem ‘The Clod and
against the backdrop of the British Ray and the Indian
the Pebble’ published in 1794 collection of ‘Songs of Independence movement in the 1920's.
Innocence and of Experience.’ Hence option (a) is Hence option (d) is correct.
correct.
32. "A sudden blow: the great wings beating still
28. Who is the author of Mary and the unfinished Above the staggering girl, her thighs caressed
The Wrongs of Woman? By the dark webs, her nap caught in his bill,
(a) Mary Wollstonecraft He holds her helpless breast upon his breast."
(b) William Godwin Who is the author of the above lines?
(c) Mary Hay (a) W.B. Yeats (b) T.S. Eliot
(d) Elizabeth Inchbald (c) W.H. Auden (d) D.H. Lawrence
Ans: (a) Mary Wollstonecraft is the author of ‘Mary: Ans: (a) The above lines ‘A sudden blow ……. upon
A Fiction’ (1788) and ‘Maria: or The Wrongs of his breast’ are taken from W. B. Yeats' ‘Leda and the
Woman’ (1798, posthumous) . These two are her Swan’. Hence option (a) is correct.
fiction works. Her political treatises include ‘A 33. "Consume my heart away; sick with desire
Vindication of the Rights of Men’ (1790) and ‘A And fastened to a dying animal"
Vindication of the Rights of Woman’ (1792). The above lines are taken from:
Hence option (a) is correct. (a) "Felix Randal"
(b) "Sailing to Byzantium"
29. Identify the incorrect factor in Henry James’ (c) "Coole and the Ballylee", 1931
theory of the novel: (d) "The Second Coming"
(a) It should be sentimental Ans: (b) The above lines ‘Consume ……………
(b) It should be objective dying animal’ are taken from W. B. Yeats ‘Sailing to
(c) It should be realistic Byzantium’ published in 1928 in his collection ‘The
(d) It should be viewed as an artistic form Tower’. Hence option (b) is correct.
Ans: (a) Henry James’ theory of the Novel suggests 34. Who among the following is not a surrealist poet?
that it should be objective, realistic and should be (a) Hugh Sykes Dykes (b) David Gascoyne
viewed as an artistic form and should not be (c) Kenneth Allot (d) C. Day Lewis
sentimental. Hence option (a) is correct. Ans: (d) Dykes, Gascoyne and Allot are associated
30. Match the items in List- I with items in List-II with the Surrealist Movement while C. Day Lewis
according to the code given below: was an Archetypal poet of the 1930s.
List-I List-II Hence option (d) is correct.
(Novels) (Characters) 35. The protagonist returns with an admonition, the
i. Ulysses 1. Mrs. Moore diamond sent to him for smuggling out a packet
of diamonds as bribe.
ii. A Passage to India 2. Molly Bloom
This scene occurs in one of the novels of Graham
iii. To the Lighthouse 3. Gerald Crich Greene-Identify the novel
iv. Women in Love 4. Lily Briscoe (a) The End of the Affair
Codes: (b) The Heart of the Matter
i ii iii iv i ii iii iv (c) The Ministry of Fear
(a) 3 1 2 4 (b) 2 1 4 3 (d) Our Man in Havana
(c) 4 2 1 3 (d) 1 3 2 4 Ans: (b) The above mentioned scene occurs in
Ans: (b) Graham Greene’s novel ‘The Heart of the Matter’
(1948). Hence option (b) is correct.
i. 'Ulysses' (1922) – 2. Molly Bloom.
ii. 'A Passage to – 1. Mrs. Moore. 36. Samuel Beckett's trilogy published together in
London in 1959 under the English titles is
India' (1924)
(a) More Pricks than Kicks, Murphy, Molloy
iii. 'To the Lighthouse' – 4. Lily Briscoe. (b) Molloy, Malone Dies, The Unnamable
(1927) (c) Molloy, Murphy, Malone Dies
iv. 'Women in Love' – 3. Gerald Crich. (d) The Unnamable, More Pricks than Kicks,
(1921) Murphy
UGC NET/JRF English Paper II, June 2014 162 YCT
Ans: (*) Among the given options, there is no 40. An important poet and playwright, who in the
appropriate answer which can justify that is required. 1960s led the Black Arts Movement, in the spirit
Therefore, NTA, has dropped this question and of negritude, posited a 'Black Aesthetic' that
distributed equal marks to all the candidates. expressed a pan-African, organic and whole
'The unnamable' is a 1953 novel by Samuel Beckett. It sensibility.
is the third and final entry in Beckett's "Trilogy" of (a) Henry Louis Gates Jr. (b) Amiri Baraka
novels, which begins with 'Molloy' followed by 'Malone (c) Ishmael Reed (d) Bell Hooks
Dies.' It was originally published in French as Ans: (b) Amiri Baraka led the Black Arts Movement
L'innommable and later translated by the author into in the 1960s and posited a ‘Black Aesthetic’. Hence
English. Grove Press published the English edition in
option (b) is correct.
1959.
41. Match List -I with List -II according to the code
37. Among the following playwrights, who was given below:
awarded the Pulitzer prize in 1920?
List-I List-II
(a) Eugene O' Neill
(Authors) (Books)
(b) Sean O' Casey
i. V.S. Naipaul 1. Foe
(c) William Somerset Maugham
ii. Jean Rhys 2. Indigo or Mapping
(d) J.B. Priestly
the Waters
Ans: (a) Eugene O’ Neill was awarded the 1920
iii. Marina Warners 3. Wide Sargasso Sea
Pulitzer Prize for Drama for his play ‘Beyond the
iv. J.M. Coetzee 4. Mimic Men
Horizon’.
He is the only playwright to have been awarded Codes
four Pulitzer prizes for Drama. i ii iii iv
1922 – 'Anna Christie'. (a) 4 2 3 1
1928 – 'Strange Interlude'. (b) 4 1 2 3
1957 – 'Long Day’s Journey into Night'. (c) 4 3 2 1
Other Pulitzer for Drama winners with three (d) 1 3 4 2
awards each are– Ans: (c)
Robert E. Sherwood (1936, 1939 and 1941) and i. V. S. Naipaul – 4. Mimic Men
Edward Albee (1967, 1975 and 1994). (1967).
Pulitzer is awarded in 7 categories under Letters, ii. Jean Rhys – 3. Wide Sargasso Sea
Drama and Music: (1) Biography or Autobiography (1966).
(2) Fiction (3) General Non-fiction (4) History (5)
iii. Marina Warners – 2. Indigo or Mapping
Poetry (6) Drama (7) Music.
the Waters (1992).
Hence option (a) is correct.
iv. J. M. Coetzee – 1. Foe (1986).
38. D.H. Lawrence popularized the concept of ____ Hence option (c) is correct.
in his novels.
(a) Realism (b) Naturalism 42. Yasmine Gooneratne's 'The Pleasures of
Conquest' termed as a postcolonial novel of the
(c) Primitivism (d) Expressionism
nineties is ironically enough set in the tropical
Ans: (c) D. H. Lawrence popularized the concept of island nation of
primitivism in his novels.
(a) Sri Lanka (b) Fiji
Primitivism is a western art movement which
(c) The Caribbean (d) Amnesia
borrows visual forms from non-western or pre-historic
peoples. Ans: (d) Yasmine Gooneratne’s ‘The Pleasures of
Hence option (c) is correct. Conquest’ (1955) is ironically set in the tropical island
nation of Amnesia. Hence option (d) is correct.
39. Who among the following is not an American
modernist poet? 43. Which of the following is not an Asian-Canadian
writer?
(a) William Carlos Williams
(a) Shauna Singh Baldwin
(b) Ezra Pound
(c) William Ellery Channing, the younger (b) Himani Banerjee
(d) Marianne Moore (c) Joy Kogawa
(d) Meena Alexander
Ans: (c) William Ellery Channing, the younger was a
th
Transcendentalist poet of 19 century. Therefore he Ans: (d) Meena Alexander was raised in Allahabad
couldn’t be a modernist poet. Hence option (c) is and Sudan and works in New York City. So she is not
correct. related to Canada. Hence option (d) is correct.

UGC NET/JRF English Paper II, June 2014 163 YCT


44. Which of the following is true? 49. "Out of the gosple he tho wordes caughte
(a) 'Aurora Leigh' is a poem in nine books And this figure he added eek therto,
(b) 'Aurora Leigh' is a collection of sonnets from That if gold ruste, what shal iren do?"
the Portuguese In the Prologue the Parson is represented as a
(c) 'Aurora Leigh' is a nursery rhyme book man:
(d) 'Aurora Leigh' is "the Seeds and Fruits of 1. who loved money
English Poetry" 2. who criticized the corrupt clergy
Ans: (a) ‘Aurora Leigh’ (1856) by E. B. Browning is 3. who practiced what he preached
an epic poem in nine books. Hence option (a) is
4. who was a poor but honest clerk
correct.
Find the correct combination according to the
45. "The old order changeth yielding place to new, code :
And God fulfils himself in many ways" (a) 1, 2 and 3 are correct
In which of the following poems do these lines (b) 1, 2 and 4 are correct
appear?
(c) 2, 3 and 4 are correct
(a) 'Locksley Hall' (b) 'Two Voices'
(d) 1, 3 and 4 are correct
(c) 'Morte d' Arthur' (d) 'Ulysses'
Ans: (*) Note– NTA has dropped this question and
Ans: (c) The above lines ‘The old order ……….in distributed equal marks to all the candidates.
many ways’ has been taken from Tennyson’s ‘Morte In the 'Prologue to the Canterbury Tales' Chaucer
d’ Arthur’. Hence option (c) is correct.
represents Parson as a humble, meek, and an example of
46. George Eliot's attempt to write a historical novel what a follower of Christ should be. He cares for his
of the Italian Renaissance was not successful. flock and does everything to help them regardless of his
Which was this novel? own situation. He is everything that the Monk, the Friar
(a) Adam Bede (b) Felix Holt and the Pardoner are not.
(c) Silas Marner (d) Romola 50. Match the items in List-I with items in List-II
Ans: (d) George Eliot’s ‘Romola’ (1862) was an according to the code given below:
unsuccessful attempt of writing a historical novel. List-I List-II
Hence option (d) is correct. (Plays) (Characters)
47. In which novel, does the hero, driven by passion i. White Devil 1. Hieronimo
and revenge, add a new dimension to the concept ii. Maids Tragedy 2. Old Knowell
of suffering? iii. Every Man in 3. Vittoria Corombona
(a) Wuthering Heights (b) Jude the Obscure his Humour
(c) Mill on the Floss (d) Hard Times iv. The Spanish Tragedie 4. Aspatia
Ans: (a) In Emily Bronte’s ‘Wuthering Heights’ Codes:
Heathcliff, driven by passion and revenge adds a new i ii iii iv i ii iii iv
dimension to the concept of suffering. Hence option
(a) 4 3 1 2 (b) 2 1 3 4
(a) is correct.
(c) 3 4 2 1 (d) 4 3 2 1
48. From the following women characters in
Ans: (c) The correct matches are–
Hardy's novels choose the odd one out:
i. John Webster’s – 3. Vittoria
(a) Bathsheba Everdene (b) Eustacia Vye
(c) Elizabeth Jane (d) Lucetta ‘White Devil’ Corombona.
(1612)
Ans: (c) In Hardy’s women characters mentioned
above – Bathsheba, Eustacia and Lucetta have ii. Beaumont and – 4. Aspatia.
tarnished reputation which is not the case with Fletcher’s ‘Maids
Elizabeth Jane. Tragedy’ (1619)
Bathsheba – ‘Far from the Madding Crowd’ (1874) iii. Ben Jonson’s – 2. Old Knowell.
Eustacia Vye – ‘The Return of the Native’ (1878) ‘Every Man in his
Elizabeth Jane and Lucetta – ‘The Mayor of Humour’ (1598)
Casterbridge’ (1886) iv. Thomas Kyd’s – 1. Hieronimo.
The three women except Elizabeth Jane fall in love ‘The Spanish
with more than one man. Tragedy’ (1587)
Hence option (c) is correct. Hence option (c) is correct.

UGC NET/JRF English Paper II, June 2014 164 YCT


UGC NET/JRF Exam, December-2014
ENGLISH
Solved Paper-III
Note : This paper contains seventy five (75) Ans : (d) In Behn's 'Oroonoko or The Royal Slave'
objective type questions of two (2) marks each. Oroonoko is cut to pieces by the executioner publicly
All questions are compulsory. while he bore the pain without crying out smoking his
1. This work was a satire in Ottava rima, pipe. Hence option (d) is correct.
attacking George III and Robert Southey. 5. The narrative of this novel is a meticulous,
Identify the poem : present-tense account of a woman with a death-
(a) Dunciad wish who plots the circumstances of her own
(b) The Vision of Judgement violent murder.
(c) Childe Harold's Pilgrimage
Identify the novel.
(d) Alastor
(a) Iris Murdoch's A Fairly Honourable Defeat
Ans: (b) Byron in his 'The Vision of Judgement' (b) Muriel Spark's The Driver's Seat
(1822) attacked George III and Southey's criticism of (c) Doris Lessing's Children of Violence
him in 'A Vision of Judgement' (1821). (d) Angela Carter's The Passion of the New Eve
Hence option (b) is correct.
Ans: (b) Muriel Spark's 'The Driver's Seat' (1970) was
2. Here's a famous exchange from Arthur Conan hailed as a 'metaphysical shocker' for its narrative of a
Doyle's Silver Blaze : woman who plots her own violent murder.
'Is there any point to which you would wish to draw
Hence option (b) is correct.
my attention?'
6. The library where the "Battle of Books" takes
'To the curious incident of the dog in the night-
time.' place is _______.
'The dog did nothing in the night-time.' (a) St. James’s Library
What was Sherlock Holmes' response? (b) King's Library
(a) 'Nothing? Nothing at all? (c) Sir William's Library
Rather unbelievable.' (d) Christ Church Library
(b) 'That was the curious incident.' Ans: (b) 'The Battle of the Books' (1704) by Swift is
(c) 'Anything else, at all?' an account of the battle of Ancient and Modern Books
(d) 'That sounds rather curious, don't you think?' in the King's Library hosted in St. James’s Palace.
Ans: (b) 'That was the curious incident' is the Hence option (b) is correct.
response of Holmes to Watson in the above lines from 7. Who among Shakespeare's contemporaries did
Doyle's short story 'Silver Blaze' (1892). not write tragedies?
Hence option (b) is correct. (a) Thomas Kyd
(b) John Lyly
3. "The shrill, demented choirs of waiting shells,
(c) Christopher Marlowe
And bugles calling for them from sad shires." (d) Ben Jonson
These lines are from Wilfred Owens’s:
Ans: (a) William Shakespeare and his contemporaries
(a) "Strange Meeting"
helped to create not only a new kind of theatre but
(b) "Futility"
also a new form of language. Shakespeare's
(c) "Anthem for Doomed Youth"
contemporaries are Thomas Kyd, John Lyly,
(d) "Dulce et Decorum Est"
Christopher Marlowe and Ben Jonson, all are the
Ans: (c) The lines "The shrill …. sad shires" appear in
contemporaries of Shakespeare but only John lyly has
W. Owens’s 'Anthem for Doomed Youth' (1917).
not written any tragedy. Even though UGC has
Hence option (c) is correct.
considered option (a) as correct answer.
4. In Aphra Behn's Oroonoko, how does the titular
8. In Sophocles' Oedipus Rex the first scene finds
character die?
Oedipus
(a) He disembowels himself. (a) in conversation with a priest
(b) He is whipped to death. (b) in consultation with a general
(c) He is hanged in the public square. (c) giving audience to an ambassador
(d) He is cut to pieces slowly by the executioner. (d) in consultation with a minister
UGC NET/JRF English Paper III, December 2014 165 YCT
Ans: (b) In Sophocles 'Oedipus Rex' the first scene (a) the Newbolt Report; Sir Henry Newbolt
finds Oedipus in consultation with a general. Oedipus (b) the Wood's Despatch; Charles Wood, Lord
is told that the plague will cease only when the Halifax
murderer of Queen Jocasta's first husband, king Louis (c) the Chatham Report; Earl John Chatham
has been found and published for his deed. (d) the Landow Document; Sir George Landow
Thus, the correct answer is option (b). Ans: (a) The first major report on "The Teaching of
9. The Kite Runner, a novel by Khaled Hosseini English in England" (1921) was 'The Newbolt Report'
tells the story of ______. named after The Chairman of Board of Education, Sir
(a) Ahmed (b) Nadira Henry Newbolt.
(c) Amir (d) Amourrah Hence option (a) is correct.
Ans: (c) 'The Kite Runner' (2003) is the story of Amir 15. Who first developed the notion of 'competence'
in language studies?
who is the protagonist and narrator.
(a) Dell Hymes (b) Noam Chomsky
Hence option (c) is correct.
(c) Leech and Svartvik (d) Henry Sweet
10. Thomas Babington Macaulay, the writer of the
Ans: (b) The notion of 'competence' in language
infamous Minute of 1835, finds a mention in
studies was developed by Noam Chomsky who is a
Salman Rushdie's
renowned figure in modern linguistics for his two
(a) Midnight's Children
works – 'Syntactic Structures' (1957) and 'Aspects of
(b) Shame
the theory of Syntax' (1965).
(c) The Moor's Last Sigh
Hence option (b) is correct.
(d) Fury
16. The fruit was eaten.
Ans: (c) Rushdie mentions Macaulay in his 'The
The fruit is ripening.
Moor's Last Sigh' (1995).
Which of the following statement (s) is/are
Hence option (c) is correct.
correct?
11. The issue of privileging speech over writing was (1) English has two kinds of participle: the
taken up for discussion in Plato's: present and the past.
(a) Ion (b) Republic Book III
(2) English has three kinds of participle: the
(c) Republic Book X (d) Phaedrus
present, the past and the future.
Ans: (d) The issue of privileging speech over writing
(3) The first sentence here is an example of a
is discussed in Plato's 'Phaedrus'.
Hence option (d) is correct. verb in past participle.
12. 'The Medium is the Message' is a concept given (4) The first sentence here is an example of a
by verb in the perfect tense.
(a) Ernest Hemingway (b) Sylvia Plath (5) The second sentence here is an example of a
(c) Seymour Hersh (d) Marshal McLuhan verb in present participle.
Ans: (d) 'The Medium is the Message' is a phrase (6) The second sentence here is an example of a
coined by Marshal McLuhan meaning that the form of verb in the continuous tense.
a medium embeds itself in the message, creating a (a) 2, 4, 6 are correct (b) 1, 5, 6 are correct
symbolic relationship by which the medium (c) 1, 3, 5 are correct (d) 3, 4, 5 are correct
influences how the message is perceived. It was Ans: (c) The first sentence is an example of a verb in
introduced in his book 'Understanding Media: The
past participle and the second in present participle.
Extensions of Man' (1964).
Hence option (d) is correct. Hence statements (1), (3) & (5) and option (c) are
13. Seamus Heaney's famous poem "Digging" correct.
forms a part of his celebrated collection called 17. In 1722 the Crown awarded a certain English
(a) North (b) Death of a Naturalist merchant a patent to manufacture copper coins
(c) Field Work (d) Door into the Dark for Ireland. Jonathan Swift intervened by way
Ans: (b) Heaney's poem 'Digging' is part of his of composing a series of letters in response,
collection 'Death of a Naturalist' (1966). better known as The Drapier's Letters. Who was
Hence option (b) is correct. the merchant?
14. The first major report on The Teaching of (a) Isaac Bickerstaff
English in England was published in 1921. It is (b) William Bickerstaff
known as _______, named after the Chair, (c) William Wood
Board of Education, _______. (d) William Sacheverell
UGC NET/JRF English Paper III, December 2014 166 YCT
Ans: (c) William Wood was granted the letters patent Ans: (a)
to mint the coin for Ireland which Swift saw as I. David Garrick 2. The 18th Century
corrupt and so replied in a series of seven pamphlets belongs to
known as 'The Drapier's Letters' between 1724-25 II. John Gielgud 4. The 20th Century
under the pseudonym of M. B. Drapier to hide from belongs to
retaliation. Hence option (c) is correct. III. Henry Irving 1. The 19th Century
18. "While the world moves belongs to
In appentency on its metalled way IV. Thomas Betterton 3. The Restoration Age
Of time past and time future" belongs to
These lines are from : Hence option (a) is correct.
(a) "Little Gidding" (b) "Dry Salvages" 22. In his "Structure, Sign, and Play in the
(c) "Burnt Norton" (d) "East Coker" Discourse of the Human Sciences," Derrida is all
Ans: (c) The lines "While ………. time future" praise for the bricoleur whom Levi-Strauss sees
appear in T. S. Eliot's 'Burnt Norton' (1935) which is as a supreme methodologist, "someone who uses
the first in his 'Four Quartets' (1935-42) which include 'the means at hand'."
'East Coker' (1940), 'The Dry Salvages' (1941) and Who does Levi-Strauss contrast bricoleur with
'Little Gidding' (1942). in terms of method and approach?
Hence option (c) is correct. (a) The Botanist (b) The Anthropologist
19. The following is the stage-description of an (c) The Engineer (d) The Semiotician
opening scene of a famous modern play : Ans: (c) Strauss contrasts 'bricoleur' with The
A basement room. Two beds, flat against the back Engineer in his 1962 work – 'The Savage Mind.'
wall. A serving hatch, closed, between the beds. A Hence option (c) is correct.
door to the kitchen and lavatory, left. A door to a
23. Heinrich Boll has something to say, and not of
passage, right.
course merely something about the Germans.
Identify the play :
He says it several times. A common weakness of
(a) The Importance of Being Earnest
writers with something to say is their inability to
(b) Travesties
understand that saying it four times is not
(c) The Dumb Waiter
necessarily four times as effective as saying it
(d) Look Back in Anger
once. But to have something to say – how rare
Ans: (c) The stage description 'A basement ……
this is! - D. J. Enright, "Three New Germans".
passage, right' appears in Harold Pinter's one-act play
From a reading of the above, the reader can
'The Dumb Waiter' (1957).
deduce :
Hence option (c) is correct.
I. Enright mildly disapproves of Heinrich Boll's
20. 'Homonyms' are words that _______ saying not merely something about Germans.
(a) are pronounced differently but have the same
II. Enright is disappointed that Heinrich Boll has
meaning.
practically nothing to say about people other
(b) refer to both the male and female of the
than Germans.
human species.
III. Enright agrees that Heinrich Boll shares a
(c) are spelt similarly but have different
weakness with writers who prefer saying
meanings.
something four times to saying it once.
(d) refer to people who live in houses with
IV. Enright does not believe that saying
similar structures.
something four times will necessarily make
Ans: (c) Homonyms are words that are spelt similarly
the same effective.
but having different meanings.
The right combination, according to the code, is
Hence option (c) is correct.
(a) I and II (b) II and III
21. Match the columns : (c) III and IV (d) I and IV
Shakespearean Actors Period
Ans: (c) After reading the above lines the statements
I. David Garrick 1. The 19th century
II. John Gielgud th
2. The 18 century III and IV can be deduced in the line 3 & 6 of the
III. Henry Irving 3. The Restoration passage. Hence option (c) is correct.
IV. Thomas Betterton 4. The 20 century th 24. Michel Foucault's earlier "archaeological"
I II III IV study is found in
(a) 2 4 1 3 (a) Power/Knowledge
(b) 4 2 1 3 (b) Social Theory and Transgression
(c) 3 4 1 2 (c) The Birth of the Clinic
(d) 2 3 4 1 (d) Beyond Structuralism and Hermeneutics
UGC NET/JRF English Paper III, December 2014 167 YCT
Ans: (c) Foucault's earlier 'archaeological study' is Ans: (b) The extract is taken from Sheridan's play
found in his 'The Birth of the Clinic: An Archaeology 'The School for Scandal' (1777).
of Medical Perception' (1963 in French and 1973 in Hence option (b) is correct.
English). Hence option (c) is correct. 28. Who, from among the following, has NOT been
25. Invisible Man by Ralph Ellison is widely discussed by Simon-de-Beauvoir in "The Myth
recognized as a masterpiece. It is also one of the of Woman in Five Authors" in The Second Sex?
finest examples of (a) Montherlant (b) Lawrence
(a) science fiction (c) Stendhal (d) Kafka
(b) picaresque novel Ans: (d) Kafka has not been discussed by Simon-de-
(c) coming-of-age novel Beauvoir in 'The Second Sex' (1949).
(d) crime thriller The five authors are – Montherlant or The Bread of
Ans: (c) The 'Invisible Man' (1952) by Ralph Ellison Disgust, D. H. Lawrence or Phallic Pride, Claudel or
is a coming-of-age novel. the Hand-maiden of The Lord, Breton or Poetry and
Hence option (c) is correct. Stendhal or Romancing the Real.
26. Match the following correctly : Hence option (d) is correct.
List–I List–II 29. In a collection of essays Orhan Pamuk shares
I. Mulk Raj Anand 1. Premashram how he writes his novels, tells about his
II. Raja Rao 2. The Cat and Shakespeare friendship with his daughter, talks about his
III. Prem Chand 3. Coolie loneliness and happiness.
IV, Girish Karnad 4. Nagamandala (a) Other Colors (b) The Silent House
I II III IV (c) The Black Book (d) TheWhite Castle
(a) 3 2 4 1 Ans: (a) Orhan Pamuk's collection of essays is –
(b) 2 3 1 4 'Other Colors' (1999).
(c) 3 2 1 4 Hence option (a) is correct.
(d) 4 3 2 1 30. Two of the following plays won the Sultan
Ans : (c) Padamsee Prize for Indian plays in English :
I. Mulk Raj Anand 3. 'Coolie' (1936) I. Princes
authored II. Where There's a Will
II. Raja Rao authored 2. 'The Cat and III. Larins Sahib
Shakespeare' (1965) IV. Doongaji House
III. Prem Chand 1. 'Premashram' (1922) (a) III and IV (b) I and III
authored (c) II and III (d) I and IV
IV. Girish Karnad 4. 'Nagamandala' (1988) Ans: (a) Sultan Padamsee Award was instituted in
authored 1966 (by Bobby Padamsee) for awarding Indian play–
Hence option (c) is correct. wrights in English. Its winners are 'Princes' (1970) by
27. From which of Sheridan's plays the following Gieve Patel; 'Larins Sahib' (1970) by Gurcharan Das
extract is taken? and 'Doongaji House' (1977) by Cyrus Mistry.
Lady Sneerwell : Why truly Mrs. Clackitt has a Hence the correct combination (according to UGC) is
very pretty talent and a great option (a) III and IV.
deal of industry. [Note: three options should be correct (I), (III) & (IV)]
Snake : True, Madam, and has been 31. Who among the following is NOT an Australian
tolerably successful in her day. writer?
To my knowledge she has been (a) Morris West (b) Patrick White
the cause of six matches being (c) Thomas Keneally (d) Bill Pearson
broken off and three sons Ans: (d) Bill Pearson is an American writer. Hence
disinherited, of four forced option (d) is correct.
elopements….. 32. After Independence, Mulk Raj Anand, wrote a
Lady Sneerwell : She certainly has talents but her number of semi-autobiographical works to
manner is gross. narrate chunks of his own life through a
(a) The Rivals fictional persona. The name he gave this persona
(b) The School for Scandal is ________.
(c) St. Patrick's Day (a) Lal Singh (b) Krishan Chander
(d) The Critic (c) Puran Singh (d) Rahul Singh
UGC NET/JRF English Paper III, December 2014 168 YCT
Ans: (b) The name given by Mulk Raj Anand to his Ans: (b) In More's 'Utopia' (1516 Latin, 1551
fictional persona is – Krishan Chander. Krishan's life English) the traveller Raphael Hythloday's second
up to the age of 15 is depicted in 'Morning Face' name in Greek meant dispenser of nonsense. Hence
(1968). In 'Confessions of a Lover' Krishan's option (b) is correct.
undergraduate days at Khalsa College, Amritsar are 37. "You do not dwell in me nor I in you
depicted. 'The Bubble’ shows Krishan's life in however much I pander to your name"
England obtaining a Ph.D. degree. All these are part These lines from Geoffrey Hill's "Lachrimae"
of the seven volumes of 'The Seven Ages of Man' address
autobiographical works beginning in 1951 with 'Seven (a) Christ (b) The Devil
Summers'. (c) The poet's beloved (d) The poet's enemy
Hence option (b) is correct. Ans: (a) The lines from Geoffrey Hill's 'Lachrimae'
33. What a mockery this. (1604) are addressed to Christ.
Of history, the past and that to come ! Hence option (a) is correct.
Now do I feel how all men are deceived, 38. The author of Black Skin, White Masks is
Reading of nations and their, in faith, (a) Ngugi wa Thiong'o
Faith given to vanity and emptiness …. (b) Frantz Fanon
The prelude (c) Richard Wright
(d) Martin Luther King (Jr.)
The above extract is from
(a) Book 9 Residence in France Ans: (b) 'Black Skin, White Masks' (1952 French,
(b) Book 7 Residence in London 1967 English) is authored by Frantz Fanon. Hence
option (b) is correct.
(c) Book 3 Residence in Cambridge
(d) Book 4 Summer Vacations 39. Match the following :
Poet Bird
Ans: (a) These lines "What a mockery ………. and
I. John Keats 1. Hawk
emptiness" are from 'Book 9, Residence in France' in
II. P. B. Shelley 2. Falcon
Wordsworth's 'The Prelude' (1850).
III. G. M. Hopkins 3. Skylark
Hence option (a) is correct. IV. Ted Hughes 4. Nightingale
34. While foregrounding the marginal presence of I II III IV
women in history in A Room of One's Own, (a) 4 3 2 1
Virginia Woolf refers to ______ History of (b) 4 3 1 2
England. (c) 3 4 2 1
(a) Campbell's (b) Trevelyan's (d) 3 4 1 2
(c) Sander's (d) Carter's Ans: (a)
Ans: (b) Virginia Woolf in 'A Room of One's Own' I. Keats is associated 4. Nightingale
(1929) refers to G. M. Trevelyan's 'History of with
England' (1926) to foreground the marginal presence II. Shelley with 3. Skylark,
of women in history.
III. Hopkins with 2. Falcon
Hence option (b) is correct.
IV. Ted Hughes with 1. Hawk
35. Salome is a play written by Oscar Wilde written
Hence option (a) is correct.
in
(a) English (b) Irish 40. Who of the following has written the novel The
(c) French (d) Italian Return?
Ans: (*) Salome' is a play by Irish Playwright and (a) Bapsi Sidhwa (b) V. S. Naipaul
author Oscar Wilde. It was originally published in (c) K. S. Maniam (d) Pankaj Mishra
French in 1891. It is likely that wilde wrote the play in Ans: (c) 'The Return' (1981) is the first novel of K. S.
French because of concerns that the subject matter Maniam. Hence option (c) is correct.
would be too scandalous in English. 41. Who among the following is a well-known Neo-
Note– But UGC has considered option (*) correct Aristotelian critic?
answer. It means equal mark warded by all candidate. (a) R. P. Blackmur (b) John Crowe Ranson
36. In More's Utopia, the fictional traveller Raphael (c) R. S. Crane (d) Lionel Trilling
Hythloday's second name in Greek means
Ans: (c) R. S. Crane was the founder of the Chicago
(a) Dispenser of Justice
(b) Dispenser of Nonsense school of Critics also called Neo-Aristotelian critics
(c) Dispenser of Grace due to its strong emphasis on Aristotle's concepts of
(d) Dispenser of Mercy plot, character and genre. Hence option (c) is correct.
UGC NET/JRF English Paper III, December 2014 169 YCT
42. Assertion (A) : The act of reading a text is both 48. Here is a list of American words and word-
determinate and indeterminate. makers. Match the following :
Reason (R) : Since our reading includes both a I. H. L. Mencken 1. Babbit
sense of the unity of the narrative II. Philip Wylie 2. Yes man
III. Jack Conway 3. Bible belt
held in place at the end and the IV. Sinclair Lewis 4. Monism
different wishes and guesses made I II III IV
along the way. (a) 4 3 2 1
(a) Both (A) and (R) are true and (R) is the true (b) 3 4 1 2
explanation of (A). (c) 3 4 2 1
(b) Both (A) and (R) are true, but (R) is not the (d) 4 3 1 2
true explanation of (A). Ans: (*) The correct match of American words and
(c) (A) is true, but (R) is false. words makers.
(d) (A) is false, but (R) is true. American words Wordsmakers
Ans: (a) Both Assertion and Reason are true and 1. Babbit Taunton Massachuselfs
Reason is the true explanation of A. Thus the act of 2. Yes man Alex chemiel
reading a text is both determinate and indeterminate. 3. Bible Belt H.L. Mencken
Hence option (a) is correct. 4. Monism Baron christan von wolff.
43. Girish Karnad's Hayavadana, originally in Note– UGC has rewarded equal mark for all
Kannada, has been translated into English by candidate.
(a) U. R. Ananthamurthy 49. Which of the following is Jacques Derrida's
(b) By the playwright himself epigraph to his "Structure, Sign and Play in the
(c) G. S. Amur Discourse of the Human Sciences."
(d) A. K. Ramanujan (a) More body, hence more writing …….. Helene
Cixous.
Ans: (b) Girish Karnad himself translated his play (b) We need to interpret interpretations more
'Hayavadana' (1972) into English (in 1975). than to interpret things ……….. Michel
Hence option (b) is correct. Eyquem de Montaigne.
44. Edward Said's well-known book Orientalism (c) But unlike philosophical reflection, ….. the
was published in reflections we are dealing with here concern
(a) 1978 (b) 1968 rays whose only source is hypothetical ……..
Claude Levi-Strauss
(c) 2008 (d) 1988
(d) If Cleopatra's nose had been shorter the
Ans: (a) 'Orientalism' by Edward Said was published whole history of the world would have been
in 1978. Hence option (a) is correct. different………. Blaise Pascal.
45. "To the Memory of my Beloved, the Author Mr. Ans: (b) In his 'Structure, Sign and Play' (1966) the
William Shakespeare: And What He Hath Left epigraph given by Derrida is 'We need to interpret
Us" is an ode composed by interpretations more than to interpret things …..'. –
Michel Eyquem De Montaigne.
(a) John Milton (b) Ben Jonson
Hence option (b) is correct.
(c) Andrew Marvell (d) John Suckling
50. In Mann's Death in Venice, death of the
Ans: (b) The above ode is composed by Ben Jonson protagonist occurs
that opens Shakespeare's 'First Folio' (1623). (a) in a bar (b) in a beach
Hence option (b) is correct. (c) in a church (d) on the highway
46. Call me Ishmail Tonight is written by Ans: (b) In Thomas Mann's 'Death in Venice' (1912)
(a) A. K. Ramanujan (b) Agha Shahid Ali the protagonist dies in a beach.
(c) Saleem Peeradina (d) Nissim Ezekiel Hence option (b) is correct.
51. Two among the following poets wrote the
Ans: (b) 'Call me Ishmail Tonight' (2003) is a book of "Village" poems that address the perennial
Ghazals in English by Agha Shahid Ali. theme of rural poverty :
Hence option (b) is correct. I. Oliver Goldsmith II. William Collins
47. "All fiction for me is a kind of magic or trickery III. Samuel Johnson IV. George Crabbe
– a confidence trick." The statement has been The right combination according to the code is
made by (a) I and III (b) II and III
(c) I and IV (d) I and II
(a) Angus Wilson (b) Anthony Powell
Ans: (*) The idea of perennial philosophy originated
(c) John Fowles (d) George Orwell
with a number of Renaissance theologians who took
Ans: (a) The statement is made by Angus Wilson. inspiration from 'neo-platonism' and from the 'theory
Hence option (a). of farms'.
UGC NET/JRF English Paper III, December 2014 170 YCT
Oliver Goldsmith, William Collins, Samuel Johnson Ans: (c) Nancy Chodorow for her 'The Reproduction
and George Crabbe poets a wrote the "Village" poems of Mothering' (1978) and Carol Gilligan for 'In a
that address the perennial theme of rural poverty. Different Voice' (1982) have earned acclaim in
Note– UGC has awarded equal mark for all. feminist theory to have reproduced motherhood into
52. In which of the following works Yeats developed the theory. Hence option (c) is correct.
his theory of 'gyres'? 58. Flowers is a short play written by
(a) "A Vision" (a) Mahesh Dattani (b) Asif Currimbhoy
(b) "The Secret Rose" (c) Girish Karnad (d) Paoli Sengupta
(c) "John Sherman and Dhoya" Ans: (c) 'Flowers' (2012) is written by Girish Karnad.
(d) "The Celtic Twilight" Hence option (c).
Ans: (a) In ' A Vision' (1925) W. B. Yeats developed 59. Match the columns :
his theory of gyres. Character Novel
Hence option (a) is correct. I. Lady Dedlock 1. Vanity Fair
53. Mystery and Miracle plays in English were II. Lady Bertram 2. Wives and
based on _______. Daughters
(a) English folklore (b) English legends III. Lady Harriet 3. Mansfield Park
(c) Biblical stories (d) Anglo-Saxon myths IV. Lady Jane 4. Bleak House
Ans: (c) Mystery and Miracle plays in English were I II III IV
based on Biblical stories popular from 13th to 16th (a) 4 2 3 1
centuries in England. They were performed by York, (b) 3 2 1 4
Chester, Wakefield and Coventry cycles. (c) 4 3 2 1
Hence option (c) (d) 3 4 1 2
54. When we rewrite a piece of discourse from one Ans: (c)
script into another, it is called _______. I. Lady Dedlock is a 4. 'Bleak House'
(a) Translation (b) Transliteration character in (1853) by
(c) Transcreation (d) Transformation Dickens
Ans: (b) When a piece of discourse is rewritten from II. Lady Bertram is a 3. 'Mansfield Park'
one script into another it is transliteration. Hence character in (1814) by J.
option (b) is correct. Austen
55. "No wonder then." Explain. III. Lady Harriet is a 2. 'Wives and
(a) No wonder that the words here begin to mean. character in Daughters' (1866)
(b) No wonder that you now find the words by E. Gaskell
menacing. IV. Lady Jane is a 1. 'Vanity Fair’
(c) No wonder that the words find you menacing. character in (1848) by
(d) No wonder the words still mean and are tame. Thackeray
Ans: (*) According to U G C all four explanations are Hence option (c) is correct.
correct. 'No wonder then' can be interpreted as in all 60. "The Books You Needn't Read, the Books Made
the four statements with respect to its context. For Purposes Other Than Reading. Books Read
56. The term "womanism" was first used by Before You Open Them Since They Belong To
(a) Helene Cixous (b) Gayatri Spivak The Category of Books Read Before Being
(c) Kate Millet (d) Alice Walker Written …"
Ans: (d) The term 'womanism' was first used by Alice The above extract is taken from
Walker in 1979 in her short story – 'Coming Apart'. (a) Jorge Luis Borges's "The Libray of Babel"
Hence option (d) is correct. (b) Italo Colvino's If on a Winter's Night a
57. Two among the following critics have dealt with Traveller
the reproduction of motherhood in feminist (c) Umberto Eco's The Name of the Rose
theory : (d) Francis Bacon's "Of Studies"
I. Nancy Chodorow Ans: (b) These lines 'The books ……… being written'
II. Judith Fetterley have been taken from Italo Colvino's 'If on a Winter's
III. Catherine R. Stimpson Night a Traveller' (1979).
IV. Carol Gilligan Hence option (b) is correct.
The right combination according to the code is 61. Listed below are the titles of the novels and the
(a) I and II (b) II and IV sources to which they are aligned by readers.
(c) I and IV (d) III and IV Match them appropriately :
UGC NET/JRF English Paper III, December 2014 171 YCT
List - I List - II 64. Which of the following writers did NOT receive
I. Peter Carey's 1. Daniel Defoe's the Nobel Prize for Literature?
Jack Maggs Robinson Crusoe (a) Wole Soyinka (b) Chinua Achebe
II. J. M. Coetzee's 2. Charlotte Bronte's (c) J. M. Coetzee (d) Nadine Gordimer
Foe Jane Eyre Ans: (b) Chinua Achebe was awarded the 2007 Man
III. Jean Rhy's 3. R. M. Ballantyne's Booker International Prize but he hasn't been awarded
Wide Sargasso The Coral Island
a Nobel Prize yet. Wole Soyinka received it in 1986,
Sea
J. M. Coetzee in 2003 and Nadine Gordimer in 1991.
IV. William 4. Charles Dickens’
Golding's Lord Great Expectations Hence option (b) is correct.
of the Flies 65. The Decline and Fall of the Roman Empire by
I II III IV Edward Gibbon is a significant work in ______
(a) 4 1 3 2 volumes.
(b) 4 3 1 2 (a) 3 (b) 4
(c) 4 1 2 3 (c) 5 (d) 6
(d) 4 2 1 3 Ans: (d) 'The Decline and Fall of the Roman Empire'
Ans: (c) Correct matches are– (1776) by Gibbon is in 6 volumes.
I. Peter Carey's 4 Charles Dickens’s Hence option (d) is correct.
'Jack Maggs' . 'Great 66. The first novel written by Graham Greene is
(1997) is based Expectations' (a) Stamboul Train
on (1861) (b) England Made Me
II. J. M. Coetzee's 1 Daniel Defoe's (c) The Heart of the Matter
'Foe' (1986) is . 'Robinson Crusoe' (d) The Man Within
based on (1719)
Ans: (d) Graham Green's first novel is 'The Man
III. Jean Rhy's 2 Charlotte Bronte's
'Wide Sargasso . 'Jane Eyre' (1847) Within' (1929).
Sea' (1966) is Hence option (d) is correct.
based on 67. From among the Canterbury pilgrims, which
IV. William 3 R. M. Ballantyne's group would qualify as the 'upper class'?
Golding's 'Lord . 'The Coral Island' (a) The Pardoner, The Miller, The Nun's Priest
of the Files' (1858). (b) Franklin, Parson, Wife of Bath
(1954) is based (c) The Knight, The Squire, The Prioress
on (d) The Reeve, The Manciple, The Clerk
Hence option (c) is correct. Ans: (c) The Knight, The Squire and The Prioress
62. Identify the right chronological sequence: belong to the upper class of society of that era as
(a) The Game of Chess – Volpone – The Duchess represented in the Canterbury Tales.
of Malfi –The City Madam Hence option (c) is correct.
(b) The City Madam – The Duchess of Malfi – 68. Plagiarism is a well-known word and concept in
Volpone – A Game of Chess
academic circles. The word plagiarius in Latin,
(c) Volpone – The Duchess of Malfi – A Game of
however, meant
Chess – The City Madam
(a) a trickster, a cheat
(d) The Duchess of Malfi – Volpone – A Game of
Chess – The City Madam (b) a quack, a swindler
(c) a loafer, a lout
Ans: (c) The right sequence is - Ben Jonson's
'Volpone' (1607) – Webster's 'The Duchess of Malfi' (d) a torturer, a plunderer
(1623) – T. Middleton's ' A Game of Chess' (1624) – Ans: (*) Plagiarism is a well known word and concept
P. Massinger's 'The City Madam' (1658). in academic circles. The plagiarius in Latin, however,
Hence option (c) is correct. meant kidnapper, Plunderer, to denote stealing
63. 'Nasal tone' in speech is a distinguishing feature someone else's creative work was pioneered by the
of ______. Roman poet Martial.
(a) British English (b) Scottish English • Plagiary, a derivate of Plagiarius, was introduced
(c) Australian English (d) American English into English in 1601 by dramatist Ben Jonson
Ans: (d) Nasal tone in speech is the distinguishing during the Jacobean Era to describe someone guilty
feature of American English. of literary theft.
Hence option (d) is correct. Note– So, UGC has awarded equal mark for all.

UGC NET/JRF English Paper III, December 2014 172 YCT


69. What superstition around the Eve of St. Agnes is They looked like frightened Beads, I thought –
crucial to understanding John Keats’s famous He stirred his Velvet Head
poem? Like one in danger, Cautious,
(a) If a virgin performed the proper ritual on St. I offered him a Crumb
Agnes' Eve, she would dream of her future And he unrolled his feathers
husband. And rowed him softer home –
(b) If a virgin performed the proper ritual on St. Than Oars divide the Ocean,
Agnes' Eve, she would marry her lover. Too silver for a seam –
(c) If a married woman performed the proper Or Butterflies, off Banks of Noon
ritual on St. Agnes' Eve, she would be Leap, plashless as they swim
reunited with her husband. 72. Is "a convenient Grass" an example of
(d) If a woman performed the proper ritual on St. "transferred epithet"?
Agnes’s Eve, she would dream of her future (a) Yes, it is. The "convenience" of grass is
lover.
transferred from the bird to the poet who
Ans: (a) The superstition around Keats’s 'Eve of St. finds grass convenient of access.
Agnes' (1820) is that, if a virgin performed the proper (b) Yes, it is. The grass is not "convenient", but is
ritual on St. Agnes’ Eve, she would dream of her transferred from the bird who finds the grass
future husband. convenient of access.
Hence option (a) is correct.
(c) No. It is a regular epithet.
70. Identify the person who sets himself up as the (d) No. It is not an epithet in the strict sense.
'Knight' with a pestle rather than a sword in the
Ans: (b) Transferred Epithet is a figure of speech in
play The Knight of the Burning Pestle :
which an epithet (adjective) grammatically qualifies a
(a) Ralph (b) Tim
noun other than a person or thing it is actually
(c) George (d) Squire
describing.
Ans: (a) The Knight in Francis Beaumont's play 'The
Therefore 'a Convenient Grass' is an example of
Knight of the Burning Pestle' (1613) is Ralph.
transferred epithet as the grass is not 'convenient' but
Hence option (a) is correct.
is transferred from the bird who finds the grass
71. Works like The Earthy Paradise, Dante and his
convenient of access (line 1).
Circle, Goblin Market and Other Poems and the
Hence option (b) is correct.
Journal, The Germ are associated with _______.
(a) the Pre-Raphaelites (b) Higher Criticism 73. Which of the following is NOT an example of
(c) the Cavalier Poets (d) the Pre-Romantics kinetic imagery?
Ans: (a) The Journal, The Germ (1850), W. Morris' (a) "unrolled his feathers"
'The Earthy Paradise' (1868), D. G. Rossetti's 'Dante (b) "hopped sidewise"
and his Circle' (1874), Christina Rossetti's 'Goblin (c) "Velvet Head"
Market and Other Poems' (1862) etc are works (d) "rowed him"
associated with the Pre-Raphaelites founded in 1848 Ans: (c) Kinetic imagery should show movement as
by William Holman Hunt , John Everett Millais and in 'unrolled', 'hopped' and 'rowed' but in 'velvet head'
Dante Gabriel Rossetti, who were later joined by W. there is no movement.
M. Rossetti, J. Collinson, F. G. Stephens and Thomas Hence option (c) is correct.
Woolner to form the seven-member 'brotherhood.' 74. The poem stages an encounter between :
Hence option (a) is correct. (a) the human and the non-human
Read the following poem and answer questions (b) distrust of the non-human about the humans
(72 to 75) : (c) two old friends
A Bird came down the Walk – (d) two old enemies
He did not know I saw – Ans: (a & b) According to UGC both (a) and (b)
He bit an Angleworm in halves options are correct.
And ate the fellow, raw,
75. "Like one in danger …" Who is in danger?
And then he drank a Dew
(a) The Bird (b) The Poet
From a convenient Grass –
(c) The Angleworm (d) Frightened Beads
And then hopped sidewise to the Wall
To let a Beetle pass – Ans: (a) 'Like one in danger' (line 13) is referred to
He glanced with rapid eyes the Bird who is cautious of humans.
That hurried all around – Hence option (a) is correct.

UGC NET/JRF English Paper III, December 2014 173 YCT


UGC NET/JRF Exam, December-2014
ENGLISH
Solved Paper-II
Note : This paper contains fifty (50) objective 5. The Hind and the Panther Transvers's to the Story
type questions of two (2) marks each. All questions of the Country Mouse and the City Mouse is a
are compulsory. satire on
1. Two of the following list are "Angry Young (a) Alexander Pope (b) Jonathan Swift
Men" of the 1950's British scene. (c) John Dryden (d) Samuel Butler
I. John Osborne II. C. P. Snow Ans: (c) 'The Hind and the Panther Transvers'd to the
III. Anthony Powell IV. Kingsley Amis Story of the Country Mouse and the City Mouse'
The right combination, according to the code (1687) is a satire on – John Dryden by Charles
(a) I & II (b) II & IV Montague. Hence option (c) is correct.
(c) I & IV (d) I & III 6. Match the columns :
Ans: (c) John Osborne, Kingsley Amis, Sillitoe & C. Terms Theorists
Wilson are the chief among Angry Young Men of A. Apollonian – Dionysian 1. Mathew Arnold
1950s. Their political views were radical or anarchic. B. Fancy – Imagination 2. Friedrich Nietzsche
Hence option (c) is the correct pair. C. Hellenism – Hebraism 3. G. M. Hopkins
2. Laurence Sterne's Tristram Shandy contains D. Inscape – Instress 4. S. T. Coleridge
(a) Six volumes (b) Nine volumes A B C D
(c) Ten volumes (d) Four volumes (a) 2 4 1 3
(b) 2 4 3 1
Ans: (b) Sterne's 'The Life and Opinions of Gentleman
(c) 1 4 2 3
Tristram Shandy' (1759-67) was published in Nine
(d) 4 2 1 3
volumes. Hence option (b) is correct.
Ans: (a) Correct matches are–
3. Which of the following statement is NOT true of
A. Terms Apollonian – 2. Friedrich
Areopagitica?
Dionysian Nietzsche,
(a) It was published in 1644.
B. Fancy – Imagination 4. S. T. Coleridge,
(b) It argues for the liberty of Unlicensed Printing.
C. Hellenism – Hebraism 1. M. Arnold
(c) It pleads for British privileges regarding Free
D. Inscape – Instress 3. G. M. Hopkins.
Trade.
Hence option (a) is correct.
(d) It is speech addressed to the Parliament of
7. In King Lear who among the following speaks in
England.
the voice of Poor Tom?
Ans: (c) 'Areopagitica: A Speech of Mr. John Milton (a) Kent (b) Edgar
for the Liberty of the Unlicens'd Printing to the (c) Edmund (d) Gloucester
Parliament of England’ by Milton was published in
Ans: (b) 'In King Lear' voice of Poor Tom is spoken
1644. Through it Milton argues for the liberty of
by Edgar. Hence option (b) is correct.
Unlicensed Printing addressing the Parliament. Hence
8. In Wordsworth's Prelude the Boy of Winander is
option (c) is correct because the statement is not true.
affected by
4. Thomas Hardy's last major novel was_____. (a) Blindness (b) Deafness
(a) Tess of the D'Urbervilles (c) Muteness (d) Lameness
(b) Jude the Obscure
Ans: (c) In Wordsworth's 'The Prelude' (1805-50) The
(c) The Return of the Native
boy of Winander is affected by Muteness. Hence
(d) The Trumpet Major
option (c) is correct.
Ans: (b) Hardy's last of the major novels was 'Jude
9. Which of the following in NOT mentioned as part
the Obscure' (1896); 'Tess of the D'Urbervilles' of the London locale in The Waste Land?
(1891); 'The Return of the Native' (1878); 'The (a) St. Magnus Martyr
Trumpet Major' (1880). Due to controversy over Jude, (b) King Arthur Street
Hardy decided not to write novels again and he never (c) St. Mary Woolnoth
did. Hence option (b) is correct. (d) Lower Thames Street
UGC NET/JRF English Paper II, December 2014 174 YCT
Ans: (b) In T.S. Eliot's 'The Waste Land' (1922) he Ans: (b) Piggy in Golding's 'Lord of The Flies' (1954)
has mentioned St. Magnus Martyr, St. Mary Woolnoth maintains that 'Life is scientific'. Hence option (b) is
and Lower Thames Street but not King Arthur Street. correct.
There is a King William Street. Hence option (b) is 15. Match the authors under List–I with the titles
correct. under List–II :
10. Which of the following novels is NOT written by
Jean Rhys? List–I List-II
(a) After Leaving Mr. Mackenzie I. Claude Levi –Strauss 1. Of Grammatology
(b) Good Morning, Midnight II. Jacques Derrida 2. The Archaeology
(c) The Quiet American of Knowledge
(d) Wide Sargasso Sea III. Northrop Frye 3. Structural
Ans: (c) 'The Quiet American' (1955) is a novel by Anthropology
Graham Greene and the other three are by Rhys. IV. Michel Foucault 4. Anatomy of
Hence option (c) is correct. Criticism
'After Leaving Mr. Mackenzie' – 1930 I II II IV
'Good Morning, Midnight' – 1939 (a) 1 3 4 2
'Wide Sargasso Sea' - 1966 (b) 3 1 2 4
Hence option (c). (c) 3 1 4 2
11. The first official royal Poet Laureate in English (d) 2 1 3 4
literary history was ________. Ans: (c) Correct matches are–
(a) Ben Jonson (b) William Davenant I. Claude Levi-Strauss 3. 'Structural authored
(c) John Dryden (d) Thomas Shadwell Anthropology'
Ans: (c) First Official Royal Poet Laureate in English (1958);
literary history was – Dryden from 1668-89. Second II. Jacques Derrida 1. 'Of
was Shadwell from 1689-92 and third was Nahum authored Grammatology'
Tate from 1692-1715. Hence option (c) is correct. (1967);
12. Who does Alexander Pope refer to in the III. Northrop Frye authored 4. 'The Anatomy
following lines? of Criticism'
"Born to no pride; inheriting no strife, (1957);
Nor marrying discord in a noble wife, IV. Michel Foucault 2. 'The
Stranger to civil and religious rage, authored Archaeology of
The good man walked innoxious through his age." Knowledge'
(a) Pope's father (1969).
(b) Pope himself Hence option (c) is correct.
(c) Dr. Arbuthnot 16. How did Chaucer's Pardoner make his living?
(d) The Duke of Marlborough (a) By selling stolen cattle from the neighbourhood
Ans: (a) Alexander Pope wrote the lines in his 'Epistle ottery
to Dr. Arbuthnot' (1735) addressing his father. Hence (b) By selling indulgences to those who committed
option (a) is correct. sins
13. The Theory of Natural Selection is attributed to (c) By pardoning those who stole property or
______ . committed other crimes
(a) Arthur Schopenhauer (b) Charles Darwin (d) By assisting the Friar in Church services
(c) A. N. Whitehead (d) Aldous Huxley Ans: (b) Chaucer's 'Pardoner' in "Canterbury Tales"
Ans: (b) The theory of Natural Selection was given made his living by selling indulgences to those who
by Charles Darwin in his 'On the Origin of Species by committed sins. Hence option (b) is correct.
Means of Natural Selection' (1859). Hence option (b) 17. From among the following, identify Coleridge's
is correct. companion in a fanciful scheme to establish a
14. Which character in William Golding's Lord of the Utopian community of free love on the banks of
Flies maintains, "Life is scientific"? the Susquehaina river?
(a) Simon (b) Piggy (a) Lord Byron (b) Robert Southey
(c) Ralph (d) Jack (c) William Hazlitt (d) William Wordsworth
UGC NET/JRF English Paper II, December 2014 175 YCT
Ans: (b) Coleridge's companion in their fanciful 24. Who are Didi and Gogo?
scheme was Robert Southey. Hence option (b) is (a) They are two characters in Endgame.
correct. (b) They are nicknames, respectively, for Lucky
18. Which of the following novels by H. G. Wells is and Pozzo.
about the condition of England as Empire? (c) They are nicknames, respectively, for Vladimir
(a) The Island of Dr. Moreau and Estragon.
(b) The War of the Worlds (d) They are two characters in Breath.
(c) Tono-Bungay Ans: (c) Didi and Gogo are nicknames respectively
(d) The Invisible Man
for Vladimir and Estragon in S. Beckett's absurd play
Ans: (c) Condition of England as an Empire is
'Waiting for Godot' (1952). Hence option (c) is
depicted in H. G. Well's 'Tono-Bungay' (1909). Hence
correct.
option (c) is correct.
25. Who among the following theories talks "the
19. Joothan by Om Prakash Valmiki is ________.
(a) a collection of poems (b) a play circulation of social energy"?
(c) an autobiography (d) a novel (a) Raymond Williams
Ans: (c) 'Joothan' (1997) by O. P. Valmiki is an (b) Stephen Greenblatt
autobiography. Hence option (c) is correct. (c) Antonio Gramsci
20. Listed below are some English plays across (d) Haydon White
several centuries : Ans: (b) Stephen Greenblatt talks about the
Twelfth Night, She Stoops to Conquer, The circulation of social energy. Hence option (b) is
Importance of Being Earnest, Pygmalion and correct.
Blithe Spirit. 26. How many legends of good women could
What is common of them? Chaucer complete in his The Legend of Good
(a) All problem plays; scheming and intrigue Women?
(b) All tragedies; sin and redemption
(a) Six (b) Seven
(c) All ideologically framed; class and gender
(b) Eight (d) Nine
(d) All romantic comedies; love and laughter
Ans: (d) Chaucer completed nine stories in his 'The
Ans: (d) They are all romantic comedies full of love
and laughter. Hence option (d) is correct. Legend of Good Women' (1372-86) – Cleopatra
Thisbe, Dido, Hypsipyle, Medea, Lucrece, Ariadne,
21. Who among the following wrote a poem
comparing a lover's heart to a hand grenade? Philomela, Phyllis and Hypermnestra. Hence option
(a) John Donne (b) Abraham Cowley (d) is correct.
(c) Wilfred Owen (d) Robert Graves 27. The Round Table is a collection of essays jointly
Ans: (b) Abraham Cowley compared a lover's heart to written by ________.
a hand grenade in 'A lover neither dead nor alive'. (a) Charles Lamb and William Hazlitt
Hence option (b) is correct. (b) Charles Lamb and Leigh Hunt
22. The Uncertainty Principle is attributed to _____. (c) William Hazlitt and Leigh Hunt
(a) William James (b) John Dewey (d) William Hazlitt and Thomas de Quincey
(c) Werner Heisenberg (d) Charles Darwin Ans: (c) 'The Round Table' (1817) is jointly written
Ans: (c) Werner Heisenberg is attributed with the by – William Hazlitt and Leigh Hunt. Hence option
Uncertainty Principle. Hence option (c) is correct. (c) is correct.
23. "Jabberwocky" is a creation in _______. 28. Dylan Thomas is associated with the group ___.
(a) Edward Lear's poetry
(a) The New Apocalypse
(b) Lewis Carroll's work
(b) The Black Arts
(c) Charles Dickens's Martin Chuzzlewit
(c) The Movement
(d) Thomas Hardy's Woodlanders
(d) Deep Image Poetry
Ans: (b) 'Jabberwocky' is a creation in Lewis Carroll's
(real name C. L. Dodgeson) work 'Through the Ans: (a) Dylan Thomas is associated with the group –
Looking Glass and What Alice found there' (1871) The New Apocalypse which flourished in 1940s. G.
which is a follow up of his 'Alice's Adventures in Barker and V. Watkins were also in the group. Hence
Wonderland' (1865). Hence option (b) is correct. option (a) is correct.

UGC NET/JRF English Paper II, December 2014 176 YCT


29. Which of the following writers writes from 34. Which of the following cannot be classified as
Canada? fantasy fiction?
(a) V. S. Naipaul (b) Margaret Atwood (a) The Inheritors (William Golding)
(c) Derek Walcott (d) James Joyce (b) The Magus (John Fowles)
Ans: (b) The Canadian author in the list is Margaret (c) The Lord of the Rings (J. R. R. Tolkien)
Atwood.
(d) The History Man (Malcolm Bradbury)
V. S. Naipaul is from Trinidad.
Derek Walcott is from St. Lucia and Ans: (d) 'The History Man' (1975) by Malcolm
James Joyce is from Dublin. Bradbury is a satirical campus novel and hence is not
Hence option (b) is correct. a fantasy fiction. Hence option (d) is correct.
30. "The boast of heraldry, the pomp of power, The other three – 'The Inheritors' (1955) by Golding,
And all the beauty, all that wealth e'er gave, 'The Magus' (1966) by John Fowles and 'The Lord of
Awaits alike the inevitable hour the Rings' (1954-55) by J. R. R. Tolkien are all
The paths of glory lead but to the grave" fantasy fiction.
What is the subject of awaits?
(a) Hour 35. Philosophy of Symbolic Forms is a work
(b) The things mentioned in the first 2 lines. associated with_____. .
(c) "And all that beauty, all that wealth e'er gave" (a) Wilhelm von Humboldt
(d) Grave (b) Ernst Cassirer
Ans: (a) In the lines, the subject of awaits is 'Hour' as (c) Immanuel Kant
is evident in the 3rd line– 'Awaits alike the inevitable (d) Battista Vico
hour.' The lines are taken from Gray's 'Elegy Written Ans: (b) German Ernst Cassirer authored the
in a Country Churchyard' (1751). Hence option (a) is 'Philosophy of Symbolic Forms' (1923-9). Hence
correct. option (b) is correct.
31. "Heav'n has no rage, like love to hatred 36. Which of the following facts is NOT true of
turn'd/Nor Hell a fury, like a woman scorn'd." Spenser?
Identify the text in which the above quote occurs: (a) He is a kind of English Homer, telling stories of
(a) The Double-Dealer heroic confrontations.
(b) The Way of the World (b) He fashioned an original verse form: The
(c) The Mourning Bride Spenserian Stanza.
(d) Love for Love (c) He opposed England's break with the Roman
Ans: (c) The lines have been said by Zara in Act III Catholic Church.
Sc. 2 in the tragedy 'The Mourning Bride' (1697) by (d) He is a Christian poet.
Congreve. Hence option (c) is correct. Ans: (c) Spenser was a Protestant and supported (not
32. A Young Lady's Entrance into the World is the opposed) the break of England with the Roman
sub-title of______. Catholic Church because he condemned its
(a) Belinda (b) Cecilia corruption. Hence option (c).
(c) Evelina (d) Camilla 37. William Blake developed the ideas of "Prolifics"
Ans: (c) The sub-title of Francis Burney's 'Evelina' and "Devourers" in ________.
(1778), is – 'A Young Lady's Entrance into the (a) Jerusalem
World'. Hence option (c) is correct. (b) Milton
33. "The old order changeth, yielding place to new" (c) Marriage of Heaven and Hell
is from ______. (d) Songs of Innocence and Songs of Experience
(a) "Morte d' Arthur" (b) "Idylls of the King" Ans: (c) William Blake developed the ideas of
(c) "Paracelsus" (d) "Asolando" 'Prolifics' and 'Devourers' in his 'Marriage of Heaven
Ans: (a) The line "The old order changeth yielding and Hell' (1793). Hence option (c) is correct.
place to new" is taken from Tennyson's poem 'Morte 38. Surrealism is associated with ________.
d' Arthur', which latter known as 'The passing of (a) Ernst Cassirer (b) Tristan Tzara
Arthur'. These dying words of king Arthur are his (c) Henrik Ibsen (d) Andre Breton
realization that there will be a change in generations Ans: (d) Surrealism is associated with Andre Breton
with views different from his. whose 'Surrealist Manifesto' (1924) began the
Thus, the correct answer is option (a). movement in Paris. Hence option (d) is correct.

UGC NET/JRF English Paper II, December 2014 177 YCT


39. "And miles to go before I sleep" is a line from a Ans: (b) Tony Harrison is a working-class poet.
poem by ________. Hence option (b) is correct.
(a) Emily Dickinson 45. New Science is a work associated with______.
(b) Walt Whitman (a) Ernest Cassirer
(c) Ralph Waldo Emerson (b) Wilhelm von Humboldt
(d) Robert Frost (c) G. Battista Vico
(d) Immanuel Kant
Ans: (d) The line appears is Robert Frost's poem
Ans: (c) 'New Science' (1725) is authored by – G.
'Stopping by Woods on a Snowy Evening' written in Battista Vico. Hence option (c) is correct.
(1922) and published in 1923. Hence option (d) is 46. Identify Petrarch's sonnet sequence from among
correct. the following :
40. What common link do you find among (a) Rine Sparse (b) Astrophel and Stella
"The Disquieting Muses" by Sylvia Plath, (c) Amoretti (d) Delia
"The Starry Night" by Anne Sexton, Ans: (a) Petrarch's song book 'Il Canzoniere' also
"Mourning Picture" by Adrienne Rich, and known as 'Rine Sparse' was originally titled – 'Rerum
"Musee des Beaux Arts" by W. H. Auden? Vulgarium Fragmenta' or 'Fragments composed in
(a) They inspired paintings. Vernacular.' Hence option (a) is correct.
(b) They are confessional poems.
47. The island setting of Latmos figured in Keats's
(c) They are all inspired by paintings.
(a) Endymion (b) The Eve of St, Agnes
(d) They are all inspired by Van Gogh's paintings.
(c) Lamia (d) Hyperion
Ans: (c) They are all inspired by paintings. Hence
Ans: (a) Latmos island figures in Keats' 'Endymion'
option (c) is correct.
written in 1817 and published in 1818. Hence option
41. "All Rising to Great Place is by a ________
(a) is correct.
staire." (Francis Bacon)
(a) Murky (b) Winding 48. The Artist Hero is a theatrical creation
(c) Crooked (d) Sinister emphasized by______.
(a) W. B. Yeats (b) Charles Baudelaire
Ans: (b) The correct word is 'winding staire'. Hence
option (b) is correct. (c) Oscar Wilde (d) Andre Gide
42. In Jeremy Collier's 1698 pamphlet attacking the Ans: (*) The Artist Hero is a theatrical creation
immorality and profaneness of the English stage, emphasized by W.B. Yeats, Charles Baudelaire, Oscar
who among the following was the principal Wilde and Andre Gide, because the artist as hero
target? remains a primary focus of modern literature and art.
(a) William Congreve (b) John Dryden Note– UGC NET rewarded all candidate to mark for
(c) John Vanbrugh (d) William Wycherley his wrong option.
Ans: (c) In Jeremy Collier's 1698 pamphlet John 49. Which of the following African writers won the
Vanbrugh’s 'The Relapse’ and 'The Provok'd Wife' Nobel Prize for Literature?
were the principal targets for which Vanbrugh replied (a) Chinua Achebe (b) Nadine Gordimer
with a retort in the same year in ‘A Short (c) Ngugi wa Thiong'o (d) Bessie Head
Vindication’. Hence option (c) is correct. Ans: (b) Nobel Prize for Literature was awarded to
43. Charles Dickens's visit to the United States the South African Nadine Gordimer in 1991. Hence
produced_____. option (b) is correct.
(a) Hard Times 50. "My lute, be as thou wert when thou didst grow
(b) Nicholas Nickleby With thy green mother in some shady groove"
(c) Martin Chuzzlewit - William Drummond
(d) Oliver Twist
The above quote is an example of ________.
Ans: (c) Dickens's visit to the US produced 'Martin (a) End-stopped rhyme (b) Alliteration
Chuzzlewit' (1844). Hence option (c) is correct. (c) Run-on line (d) Tercet
44. Who among the following is a working-class Ans: (c) The meaning is not ending with the end of
poet? sentence in the first line and is carrying on into the
(a) John Betjeman (b) Tony Harrison second line. Hence it is an example of run-on line.
(c) Thom Gunn (d) Robert Graves Hence option (c) is correct.

UGC NET/JRF English Paper II, December 2014 178 YCT


UGC NET/JRF Exam, June-2015
ENGLISH
Solved Paper-III
Note : This paper contains Seventy five (75) 5. Chapter III of Oliver Twist opens with a
objective type questions of two marks each. All narratorial remark about Oliver being punished
questions are compulsory. for "the commission of the impious and profane
1. When Luigi Pirandello's Six Characters in Search offence of asking for more." What did Oliver ask
of an Author opens, the audience find the for more?
producer attempting to stage a play. What is the (a) More time to play
title of this play? (b) More food to eat
(a) "Rites of Performance" (c) More books to read
(b) "Rules of the Game" (d) More money to spend
(c) "Tonight We Stage a Play"
(d) "Modes of Acting" Ans: (b) In Chapter III of Dicken's 'Oliver Twist'
Ans: (b) 'The Rules of the Game' is a play by Luigi (1839) Oliver asks for more food to eat.
Pirandello first performed in 1918 and first published Hence option (b) is correct.
in 1919. His 'Six Characters in Search of an Author' 6. Edmund Spenser's Epithalamion is a carefully
premiered in 1921, in which the characters – father, structured poem carrying ________
mother, stepdaughter, son, boy, child are rehearsing corresponding to the _______.
the above play. Hence option (b) is correct. (a) twelve stanzas; months of the year
2. Which Canterbury pilgrim carries a brooch (b) three hundred and sixty five lines; days of the
inscribed with the Latin words meaning "Love year
Conquers All"? (c) fourteen stanzas; two week-ling bridal
(a) The Prioress (b) The Monk ceremonies
(c) The Wife of Bath (d) The Squire (d) eleven stanzas; eleventh month, November
Ans: (a) The Prioress in Chaucer's 'Canterbury Tales' Ans: (b) Spenser's 'Epithalamion' (1595) is a marriage
carries a brooch with the Latin words 'Amor Vincit song which is structured into 365 lines corresponding
Omnia' with emphasis on 'A' for Amor or love and the to the days of the year. It was published in 1595
phrase meaning 'Love Conquers All'. including Amoretti.
Hence option (a) is correct. Hence option (b) is correct.
3. In his Introduction to The Oxford Book of 7. Choose the right chronological sequence below :
Twentieth-Century English Verse (1973), Philip
(a) Victorian Period – Jacobean Period – Tudor
Larkin underlines the importance of a native
tradition with ________ seen as the major poet of Period – Restoration Period
the Modern Period. (b) Edwardian Period – Tudor Period – Jacobean
(a) William Butler Yeats (b) T. S. Eliot Period – Victorian Period
(c) Thomas Hardy (d) D. H. Lawrence (c) Tudor Period – Jacobean Period – Restoration
Ans: (c) Philip Larkin underlines the importance of Period – Edwardian Period
the native tradition with Thomas Hardy seen as the (d) Jacobean Period – Tudor Period – Restoration
major poet of Modern Period for his 'Wessex Poems'. Period – Edwardian Period
Hence option (c) is correct. Ans: (c) Tudor period is from 1485 to 1603.
4. Philip Sidney defended poetry against such Jacobean period is from 1603 to 1625.
descriptions of it as "the mother of lies" and "the Restoration period is from 1660 to 1700.
nurse of abuse." His main argument here is Edwardian period is from 1901 to 1910.
________. Hence option (c) is correct.
(a) The poet is no conjuror or illusionist and 8. "That woman's days were spent
represents a world. In ignorant good – will,
(b) The poet cannot lie because he is not claiming Her rights in argument
to tell us the truth. Until her voice grew shrill"
(c) The poet cannot speak the truth because he is
(W. B. Yeats : "Easter 1916")
not representing the real world.
(d) The poet is a philosopher for whom truth is a Who is the poet referring to?
lie, and lie truth, in an imaginary world. (a) Maud Gonne
(b) Lady Augusta Gregory
Ans: (b) Philip Sidney in his 'Apology for Poetrie'
defends poetry from the accusations made by Stephen (c) Kathleen Pilcher
Gosson in his 'School of Abuse' who described poetry (d) Constance Gore – Booth Markievicz
as 'mother of lies' and argues that – the poet cannot lie Ans: (d) W. B. Yeats in 'Easter 1916' (1921) is
because he is not claiming to tell us the truth. referring to Constance Gore – Booth Markievicz.
Hence option (b) is correct. Hence option (d) is correct.
UGC NET/JRF English Paper III, June 2015 179 YCT
9. Which of the following was replaced by (D) Arthur Hugh Clough (iv) A sensational
Communicative Language Teaching? : The Bothie of 17th century
(a) Motivational Approach Tober-na-Vuolich murder presented
(b) Situational Language Teaching through multiple
(c) Natural Language Processing dramatic
(d) Structural Approach monologues
Ans: (b & d) – Situational Language Teaching (SLT) The right matching according to the code is:
was developed by British applied linguists Harold (A) (B) (C) (D)
Palmer and A.S. Hornby. SLT bridges the gap between (a) (iii) (iv) (i) (ii)
grammar translation approaches to language teaching (b) (ii) (iv) (iii) (i)
and Communicative Language. Teaching, and is similar (c) (iii) (i) (iv) (ii)
to audiolingualism. (d) (iv) (ii) (iii) (i)
– The Structural approach is based on the assumption Ans : (c) The correct matches are–
that language teaching can best be done through (A) Christina (iii) a story of
systematic selection and grading of structures or Rossetti's pleasure-seeking
sentence pattern. The basic principle underlying the 'Goblin Laura and the
approach is language is speech, not writing.
Market' conventionally
Note– NTA didn't consider any of the given options (1862) moral Lizzie who
because (b) and (d) both are correct.
resists
10. To whom does Francis Bacon offer the following temptations.
piece of advice? (B) Matthew (i) a tale of a father
"Let him sequester himself from the Company of
Arnold's who
his Countrymen, and diet in such Places, where
'Sohrab and inadvertently
there is good company of the Nation… Let him
upon his Removes, … procure Recommendation, Rustom' destroys his son
to some person of Quality, residing in the Place, (1853) (unknowingly).
whither he removeth…" (C) Robert (iv) a sensational
(a) The Beaux (b) The Peddler Browning's 17th century
(c) The Traveller (d) The Stationer 'The Ring and murder
Ans: (c) Francis Bacon in his essay 'Of Travel' offers the Book' presented
his advice to The Traveller. (1869) through multiple
Hence option (c) is correct. dramatic
11. In his masterpiece, Of the Lawes of Ecclesiastical monologues.
Politie, Richard Hooker affirmed the Anglican (D) Arthur Hugh (ii) a gently satiric
tradition as that of a "threefold cord not quickly Clough's 'The account of an
broken." He specifically referred to the following Bothie of oxford student on
EXCEPT________. Tober-na- vacation.
(a) tradition (b) scripture Vuolich'
(c) community (d) reason (1848)
Ans: (c) In his 'Of the Lawes of Ecclesiastical Politie' Hence option (c) is correct.
(1594) Richard Hooker has specifically referred to – 13. "Beneath them sit the aged men, wise guardians
tradition, scripture and reason but not community. of the poor;
Hence option (c) is correct. Then cherish pity, lest you drive an angel
12. Match the following : from your door."
List–I List–II These concluding lines of William Blake's
(A) Christina Rossetti : (i) The tale of a Innocence poem called "Holy Thursday" allude
Goblin Market father who to a Biblical passage. Identify the passage.
inadvertently (a) The angel of the Lord encampeth round about
destroys his son those who fear Him and delivereth them. Psalms
(B) Matthew Arnold : (ii) Gently satiric 34.7
Sohrab and Rustom account of an
(b) Suffer not thy mouth to cause thy flesh to sin;
Oxford student
on vacation neither say thou before the angel, that it was an
(C) Robert Browning : (iii) Story of error. Ecclesiastes 5.6
The Ring and the Pleasure-seeking (c) And they said unto her, Thou art mad. But she
Book Laura and the constantly affirmed that it was even so. Then
conventionally said they, It is his angel. The Acts 12.15
moral Lizzie who (d) Be not forgetful to entertain strangers for
resists thereby some have entertained angels unawares.
temptations Hebrews 13.2
UGC NET/JRF English Paper III, June 2015 180 YCT
Ans: (d) The concluding lines of Blake's poem 'Holy 18. Assertion (A) : The world does not become raceless
Thursday' (1794) alludes to the Biblical passage in or will not become unracialized by assertion. The act
Hebrews 13.2– "Be not forgetful to entertain strangers of enforcing racelessness in literary discourse is
for thereby some have entertained angels unawares". itself a racial act.
Hence option (d) is correct. Reason (R) : Pouring rhetorical acid on the fingers
14. Direct Method of Language Teaching involves : of a black hand may indeed destroy the prints, but
(A) the use of Target Language only not the hand. Besides, what happens, in that violent,
(B) repetition of exercises self-serving act of erasure, to the hands, the fingers,
(C) linguistic correctness the fingerprints of the one who does the pouring? Do
(D) problem solving exercises they remain acid-free? The literature itself suggests
In relation to the above which of the following is otherwise.
correct? In the context to the statements above,
(a) (C) and (D) only (a) (A) makes complete sense in the light of (R).
(b) (A), (B) and (D) (b) (A) makes complete sense regardless of (R).
(c) (A), (B) and (C) (c) Neither (A) nor (R) makes complete sense.
(d) (A), (B), (C) and (D) (d) (R) challenges the view advanced in (A).
Ans: (*) Lambert sauveur, one of the precursors of the Ans: (a) The Reason (R) explains the Assertion (A)
direct method in language teaching in USA. The direct correctly. Thus (A) makes complete sense only in the
method of teaching, which is sometimes called the light of (R) and will give a vague sense without (R).
natural method and is often used in teaching foreign Hence option (a) is correct.
languages 'refrains from using the learners' native 19. A poet laureate said "I do not think that since
language and uses only the target language. It is directly Shakespeare there has been such a master of the
establishing an immediate and audiovisual association English language as I." Who is the poet?
between experience and expression, rules and (a) Stephen Spender (b) John Dryden
performances through the teacher body and mental (c) Alfred Lord Tennyson (d) Ted Hughes
skills without any help of the learners' mother tongue. Ans: (c) Alfred Lord Tennyson was the poet laureate
Note– NTA has dropped this question due to who said in a remark to Carlyle – 'I do not think that
discrepancies in options, therefore no option is since Shakespeare there has been such a master of the
suitable for the given question. English language as I.'
Hence option (c) is correct.
15. In which of the following works does the narrator
proclaim, "either I'm nobody, or I'm the 20. Who among the following was a contemporary of
nation"? John Milton and wrote The Worthy
(a) George Lamming's In the Castle of My Skin Communicant? It is said that his prose "can be
(b) Derek Walcott's "The Schooner Flight" read easily, when Milton's must be studied."
(c) Jamaica Kincaid's "Girl" (a) Jeremy Taylor (b) John Bunyan
(d) Kamau Braithwaite's "Nation Language" (c) Andrew Marvell (d) George Herbert
Ans: (b) Derek Walcott in his 'The Schooner Flight' Ans: (a) John Milton's contemporary and author of
(1990) proclaims – "either I'm nobody, or I'm the 'The Worthy Communicant' (1660) described above is
nation." Hence option (b) is correct. – Jeremy Taylor.
16. Like Cordelia, the Fool in King Lear is ________. Hence option (a) is correct.
(a) killed by Goneril's troops. 21. In 1668, Dryden wrote Of Dramatic Poesie : An
(b) referred to by Lear as his child. Essay which uses ________ separate characters to
(c) disliked by Regan and Cornwall. dramatise the conflicting viewpoints which new
(d) Punished for not telling the truth. theatrical activity had produced.
Ans: (b) King Lear in the play refers to the Fool as his (a) three (b) two
child like Cordelia. "Why my boy? Fool. If I gave (c) four (d) six
them all my living, I'd keep my coxcombs myself." Ans: (c) 'Of Dramatic Poesy/Poesie: An Essay' by
Hence option (b) is correct. Dryden was published in 1668. It is in form of a
17. Sindi Oberoi, the narrator hero in Arun Joshi's dialogue between 4 characters.
The Foreigner says : "My foreignness lay within Eugenius considered modern drama better than the
me and I couldn't leave myself behind wherever I ancients (C. Sackville), Crites defends the ancients
went." Identify the countries which Sindi Oberoi (Sir R. Howard), Lisideius considered French drama
went to. better than the English (Sedley) and Neander
(a) Kenya, Uganda, England, America, India favoured modern drama (Dryden himself).
(b) Kenya, Uganda, New Zealand, England, India Hence option (c) is correct.
(c) Kenya, England, Canada, India 22. Writing his most influential play, August
(d) Kenya, America, England, Australia, India Strindberg called it "My most beloved drama,
Ans: (a) Sindi Oberoi in Arun Joshi's 'The Foreigner' the child of my greatest suffering." The play is :
(1971) has been to – Kenya, Uganda, England, (a) A Dream Play (b) Miss Julie
America, India. Hence option (a) is correct. (c) The Bridal Crown (d) The Dance of Death
UGC NET/JRF English Paper III, June 2015 181 YCT
Ans: (a) August Strindberg's most influential play was 27. Which of the following statements about Thomas
– 'A Dream Play' (1907). Mann's novels is true?
Hence option (a) is correct. (A) Buddenbrooks is a family saga set in the early
23. In which essay does Virginia Woolf observe that decades of the twentieth century.
"if a writer were a free man [sic] and not a slave" (B) Aschenbach, the writer protagonist in Death in
to the conventions of the literary market-place, Venice, is preoccupied with classicism,
especially with classical ideals of male beauty.
there would be "no plot, no comedy, no tragedy,
(C) In his second winter at the sanatorium, Hans
no love interest, or catastrophe in the accepted
Castorp, protagonist of The Magic Mountain
style, and perhaps not a single button sewn on as gets lost in a blizzard during a solitary skiing
the Bond Street tailors would have it"? expedition.
(a) "How it Strikes a Contemporary" (D) Adrian Leverkuhn, the modern day Faustus in
(b) "Modern Fiction" Mann's Doctor Faustus is a musician.
(c) "The Russian Point of View" The right combination according to the code is :
(d) "Mr. Bennett and Mr. Brown" (a) Only (A) and (C) are correct
Ans: (b) The line 'if a writer were a free man and not (b) Only (B) and (D) are correct
a slave' appears in Virginia Woolf's essay 'Modern (c) (B), (C) and (D) are correct
Fiction' (1921). Hence option (b) is correct. (d) (A), (B) and (D) are correct
24. In his famous letter to Benjamin Bailey Ans: (c) Thomas Mann's 'Buddenbrooks' was
(November 22, 1817) John Keats wrote: "I am published in 1901 and relates the story of three
certain of nothing but the holiness of the Heart's generations of his family history in the 19th century
affections and the truth of Imagination – What and not the twentieth century. The other works –
the imagination seizes as Beauty must be truth." 'Death in Venice' (1912), The 'Magic Mountain'
Which of the following sentences follows this (1924) and 'Doctor Faustus' (1947) are explained
passage? correctly. Hence option (c) is correct.
(a) Now I am sensible all this is a mere 28. To whom did Raja Ram Mohan Roy write in
sophistication, however it may neighbour to any 1823 his letter seeking the introduction of English
truths, to excuse my own indolence… education in India?
(b) The Imagination may be compared to Adam's (a) Lord Amherst (b) Lord Bentinck
dream – he woke and found it true. (c) Lord Cunningham (d) Lord Hastings
(c) This however I am persuaded of, that nothing Ans: (a) Raja Ram Mohan Roy wrote a letter to Lord
besides Imagination can give us sweet Amherst in 1823 seeking introduction of English
sensations and pleasurable thoughts. education in India.
(d) My pains at last some respite shall afford, while Hence option (a) is correct.
I behold the battle Imagination maintains. 29. Listed below are the seemingly friendly
Ans: (b) In his letter to Bailey, Keats wrote: "I am characters in The Pilgrim's Progress who give
certain of nothing but the holiness of the Heart's Christian dangerous advice. Among them is one
affections and the truth of Imagination – What the who does not belong to this group. Identify this
imagination seizes as Beauty must be truth". This is odd character.
followed by – "The Imagination may be compared to (a) Mr. Worldly Wiseman (b) Evangelist
Adam's dream – he woke and found it true". Hence (c) Ignorance (d) Talkative
option (b) is correct. Ans: (b) In John Bunyan's 'Pilgrim's Progress' (1678)
25. Which of the following pair best describes the Christian flees from the city of Destruction on advice
characteristic features of Marlowe's portrait of of Evangelist. So he is the odd one among others who
Tamburlaine? give dangerous advice.
(A) ambition (B) apathy Hence option (b) is correct.
(C) cruelty (D) sympathy 30. Aristotle argued that poetry provides a/an ______
The right combination according to the code is__ outlet for the release of intense emotions.
(a) (A) and (B) (b) (A) and (D) (a) safe (b) dangerous
(c) (A) and (C) (d) (B) and (C) (c) uncertain (d) unreliable
Ans: (c) Ambition and cruelty are the characteristic Ans: (a) Aristotle in 'Poetics, argued that poetry
features of Marlowe's portrait of Tamburlaine. Hence provides a safe outlet for the release of intense
option (c) is correct. emotions of pity & fear through Catharsis.
Hence option (a) is correct.
26. Who is the author of the statement: "The
31. The direct French influence on the English
nineteenth century dislike of Realism is the rage
language during the Middle English period was
of Caliban seeing his own face in the glass"?
in the form of ________.
(a) Arthur Symons (b) Benjamin Disraeli
(a) loss of inflections.
(c) W. B. Yeats (d) Oscar Wilde (b) intake of French words into English.
Ans: (d) Oscar Wilde in his 'The Picture of Dorian (c) both the loss of inflections and intake of French
Gray' (1890) says the above statement in The Preface. words into English.
Hence option (d) is correct. (d) addition of inflections.
UGC NET/JRF English Paper III, June 2015 182 YCT
Ans: (b) During Middle English period English Ans: (d) Pope's epitaph for Newton impressively
language was influenced directly by intake of French captures- Newton's discovery that all colours are
words into English. contained in white light. Hence option (d) is correct.
Hence option (b) is correct. 36. What was the name of the experimental theatre
32. A significant development in 1662 was the group founded in 1915 by Susan Glaspell, Eugene
establishment to The Royal Society in England. O'Neil and other dramatists in order to challenge
The main purpose of the society was ________. Broadway's control over American drama?
(a) to set the rules of the royal court and (a) The Wall Street Theatre Group
governance (b) The Washington Square Players
(b) to guide and promote the development of (c) The Actor's Studio
science and scientific exploration (d) The Provincetown Players
(c) to set norms for civil society Ans: (d) In 1915 Susan Glaspell, Eugene O'Neill and
(d) to promote theatre others founded – The Provincetown Players to
Ans: (b) The main purpose of The Royal Society challenge Broadway's control over American drama.
(1662) in England was – to guide and promote the Hence option (d) is correct.
development of science and scientific exploration. 37. After his return from the land of Houyhnhnms,
Hence option (b) is correct. Gulliver refused to let his wife and children____.
33. William Cowper wrote in The Task (IV. 681-82) (a) show disrespect to English horses.
about those who "Build factories with blood, (b) ride horse-drawn carriages.
conducting trade/At the sword's point …" These (c) touch his bread, or drink out of his cup.
lines allude to : (d) communicate with him in English tongue.
(a) Turkish militant traders across Europe Ans: (c) After his return from the land of
(b) Nordic conquerors across East Asia Houyhnhnms, Gulliver refused to let his wife and
(c) West Indian slave-plantation owners and the children – touch his bread, or drink out of his cup.
East India Company 'nabobs'
Hence option (c) is correct.
(d) Exploiters of child labour in the London
suburbs 38. In which of the following volumes do you find a
charming appreciation of the Wordsworth
Ans: (c) William Cowper's 'The Task' (1785) alludes
household by Thomas de Quincey?
to – West Indian slave-plantation owners and the East
(a) The Confessions of an English Opium-Eater
India Company 'nabobs'.
Hence option (c) is correct. (b) Lives and Letters, Far Away and Long Ago
(c) Notes on My Lake Country Evenings
34. The commedia dell 'arte originated in Italy in the (d) Reminiscences of the English Lake Poets
sixteenth century. Which of the following
descriptions are the most appropriate? Ans: (d) Thomas De Quincey appreciates the Lake
(A) Tears alternating with crude laughter Poets – Wordsworth, Coleridge, Southey etc in his
(B) Comedy of the guild or by the professionals in 'Reminiscences or Recollections of the Lake Poets'
the "art" published in Tait's Magazine from 1834-40.
(C) Plautine comedy alternating with ritualistic Hence option (d) is correct.
manoeuvres 39. One of the most highly revered, scholarly, and
(D) Improvised comedy that follows a scenario passionate interpreters of English and world
rather than written dialogue literatures, he was appointed the Lord
The right combination according to the code is : Northcliffe Professor of Modern English
(a) (A) and (B) (b) (B) and (D) Literature at University College, London in 1967,
(c) (A) and (C) (d) (B) and (C) and later as King Edward VII Professor of
Ans: (b) 'Commedia dell' arte’ originating in Italy in English Literature at Cambridge in 1974, an
16th century is described as – Comedy of the guild or appointment made by the Crown at the
by the professionals in the 'art'; and improvised suggestion of the Prime Minister of the United
comedy that follows a scenario rather than written Kingdom. He was knighted by Queen Elizabeth
dialogue. Hence option (b) is correct. in 1991. Entitled to designate himself as "Sir," he
35. "Nature and Nature's Laws lay hid in Night, never did, but wrote an autobiography entitled
God said Let Newton be! And all was Light." Not Entitled in 1995. The epigraph to this book
Alexander Pope's famous couplet impressively came from Coriolanus: "He was a kind of
captures ________. nothing, titleless."
(a) Newton's confirmation of the Genesis passage Who among the following is this writer/critic?
where God ordains Light (a) F. R. Leavis (b) I. A. Richards
(b) Newton's empirical observations of (c) Frank Kermode (d) David Lodge
Philosophiae Naturalis Principia Mathematica Ans: (c) The above description is for Frank Kermode
(c) Newton's application of principles of motion to who remained titleless even after being Knighted in
account for many natural phenomena 1991 by the Queen and also wrote an autobiography
(d) Newton's discovery that all colours are titled 'Not Entitled'; in 1995. Hence option (c) is
contained in white light correct.

UGC NET/JRF English Paper III, June 2015 183 YCT


40. Which of the following provided theoretical basis Ans: (b) James Fennimore Cooper authored the series
for Audio-Lingual Method of Language Teaching? of five books of Natty Bumppo, (also called Deers-
(a) Transformational Generative Linguistics layer, Pathfinder or Hawkeye) known as Leather-
(b) Cognitive Psychology and Structural stocking tales.
Linguistics The books were – The Pioneers (1823), The Last of
(c) Behaviourist Psychology and Bloomfieldian the Mohicans (1826), The Prairie (1827), The
Structural Linguistics Pathfinder (1840) and The Deerslayer (1841).
(d) Systemic Functional Linguistics Hence option (b) is correct.
Ans: (c) Theoretical basis for Audio-Lingual Method 45. In John Dryden's Essay on Dramatic Poesy
of Language Teaching was provided by – Neander defends the English invention of
Behaviourist Psychology and Bloomfieldian _______.
Structural Linguistics. Hence option (c) is correct. (a) romantic comedy (b) action tragedy
41. Who among the following characters of The (c) tragi-comedy (d) morality plays
Cherry Orchard by Anton Chekhov dies in the Ans: (c) In Dryden's Essay on Dramatic Poesy (1668)
final scene? Neander defends the English invention of tragi-
(a) Anya (b) Firs comedy. Hence option (c) is correct.
(c) Varya (d) Lopakhin 46. Who wrote The History of Australian Literature in
Ans: (b) Firs dies in the final scene of Chekhov's 'The 1961?
Cherry Orchard' (1904). Hence option (b) is correct. (a) Randolph Stow (b) H. M. Green
42. In tracing the history of English poetry, Thomas (c) Handel Richardson (d) Francis Adam
Gray's "Progress of Poesy" invokes a major poet Ans: (b) Author of 'The History of Australian
as follows : Literature' (1961) is – H. M. Green.
"Nor second He, that rode sublime Hence option (b) is correct.
Upon the seraph-wings of Extasy, 47. Match the following :
The secrets of th' Abyss to spy." Theorist Theories
Who is "He"? (A) Bharata (i) Vakrokti
(a) William Shakespeare (b) Edmund Spenser (B) Kuntaka (ii) Riti
(c) John Milton (d) John Dryden (C) Bhamaha (iii) Dhvani
Ans: (c) In his 'Progress of Poesy' Thomas Gray (D) Anandavardhana (iv) Rasa
invokes – John Milton. The right matching according to the code is :
Hence option (c) is correct. (A) (B) (C) (D)
43. "I suffered from impaired eye-sight, depression (a) (i) (iv) (ii) (iii)
and poverty and left Oxford without a degree. (b) (ii) (iii) (i) (iv)
After a period as a teacher and my marriage to a (c) (iv) (i) (ii) (iii)
widow twice my age, I left for London, to begin (d) (iii) (ii) (iv) (i)
writing for a magazine, I produced my own Ans: (c) Theory given by–
journal." Choose the correct answer, identifying (A) Bharata is – (iv) Rasa;
the writer, the magazine and the journal. (B) Kuntaka is – (i) Vakrokti;
(a) John Milton, The Examiner's Magazine, London (C) Bhamaha is – (ii) Riti
Magazine (D) Anandavardhana is – (iii) Dhvani.
(b) Joseph Addison, The Freeholder, The Tatler Hence option (c) is correct
(c) Richard Steele, The Guardian, The Spectator 48. What is "Forster Collection"?
(d) Samuel Johnson, The Gentlemen's Magazine, (a) Memorabilia and documents related to the
The Rambler Scottish War of Independence (1296-1328)
Ans: (d) Samuel Johnson suffered from eye-sight housed in Glasgow Museum
impairment, depression and poverty. He had to leave (b) The special collection of E. M. Forster effects
Oxford without a degree due to poverty. He began housed in King's College, Cambridge
writing for 'The Gentlemen's Magazine’ in London (c) The largest collection of Charles Dickens
and produced 'The Rambler' in 1750. Hence option (d) manuscripts and proofs curated by John Forster
is correct. (d) The collection of political and military
44. Which of the American novelists is associated documents named after the liberal M. P., W. E.
with the series of five books about Natty Forster reputed for the Forster Education Act
Bumppo, an old hunter, also called Ans: (c) 'Forster Collection' is the largest collection of
Leatherstocking? Charles Dickens' manuscript and proofs curated by John
(a) Stephen Crane Forster, a close friend and adviser of Charles Dickens.
(b) James Fennimore Cooper He was an important Victorian English biographer and
(c) Herman Melville literary critic.
(d) Jack London Hence, option (c) will be correct answer.
UGC NET/JRF English Paper III, June 2015 184 YCT
49. What was remarkable about the poet F. T. 53. Who is the author of the statement "A prophet is
Marinetti's first Futurist Manifesto in Le Figaro? a Seer, not an Arbitrary Dictator"?
(a) It resounded like the monotonous beating of a (a) Salman Rushdie (b) Kahlil Gibran
big drum that filled the air with muffled shocks (c) William Blake (d) Oscar Wilde
and lingering vibration. Ans: (c) William Blake is the author of the statement-
(b) It proclaimed that someone must go on writing 'A prophet is a seer, not an Arbitrary Dictator.'
for those who were still convinced of the future Hence option (c) is correct.
for which they had taken up arms.
(c) It blasted the dead weight of "museums, 54. The word order in Modern English became
libraries, and academics," glorifying "the beauty relatively fixed because ________.
of speed." (a) it developed its inflectional system.
(d) It declared that man, the individual, is an (b) it lost its highly developed inflectional system.
infinite reservoir of possibilities; and if man can (c) it lost its derivational system of word formation.
so rearrange society by the destruction of (d) it developed its derivational system of word
oppressive disorder, then the possibilities have a formation.
future. Ans: (b) The word order in Modern English after
Ans: (c) Marinetti's 'Manifesto of Futurism' was losing its highly developed inflectional system of old
published in the newspaper 'Le Figaro' on 20th Feb. English became relatively fixed. Hence option (b) is
1909. Its remarkable feature was that it blasted the correct.
dead weight of 'museums, libraries and academics', 55. Julia Kristeva's 'intertextuality' derives from
glorifying the 'beauty of speed.' ________.
Hence option (c) is correct. (A) Noam Chomsky's deep structure
50. How would one best describe Thomas Carlyle's (B) Mikhail Bakhtin's dialogism
Sartor Resartus (1833)? (C) Jacques Derrida's difference
(a) A combination of journal, fashion-book, and (D) Ferdinand de Saussure's sign
tips for advertisers The right combination according to the code is :
(b) A lyrical novel a la Marcel Proust (a) (A) and (B) (b) (B) and (C)
(c) A combination of novel, autobiography, and (c) (C) and (D) (d) (A) and (D)
essay
Ans: (*) The term 'intertextuality' is coined by Julia
(d) A satire on sartorial fashions and foibles of
medieval Europe Kristeva in late 1960s. It describes the phenomenon of
a continual exchange and relationship building
Ans: (c) Thomas Carlyle's 'Sartor Resartus' (1833) is a
between texts. This term was popularised when she
combination of novel, autobiography and essay.
was analysing Bakhtin's concepts Dialogism and
Hence option (c) is correct.
Carnival. NTA has dropped this question because
51. An Indian English poet once remarked that his
there was no proper answer in the options.
discipline and education gave him his "outer"
whereas his Indian origin gave him "inner" form 56. Dylan Thomas's famous poem "Fern Hill," is
Reflecting a part of this claim is a famous essay named after ________.
he called ________. (a) a countryside in Austria to which he paid
(a) "Is There a Native Way of Thinking?" occasional visits.
(b) "Can the Subaltern Speak?" (b) a childhood haunt of the poet's family in
(c) "Where Do We Go from Here: Some Devonshire.
Speculations" (c) the Welsh farmhouse where the poet spent
(d) "Is There an Indian Way of Thinking?" summer holidays as a boy.
Ans: (d) A. K. Ramanujan's remarks on his outer and (d) The Welsh Anglican church to which the young
inner forms are reflected in his essay – 'Is there an poet used to be taken by his mother.
Indian Way of Thinking?' Ans: (c) 'Fern Hill' (1945) by Dylan Thomas is named
Hence option (d) is correct. after the Welsh farmhouse where the poet spent
52. In the remarkably crucial courtroom scene of summer holidays as a boy.
Their Eyes were Watching God, Janie is called Hence option (c) is correct.
upon to speak. Whose voice do we hear in the 57. "In the seventeenth century," writes T. S. Eliot in
narrative? "The Metaphysical Poets," "a dissociation of
(a) Tea Cake's voice sensibility set in, from which we have never
(b) Janie's first-person voice recovered; and this dissociation, as is natural,
(c) Pheoby's voice was aggravated by the influence of the two most
(d) The omniscient third-person voice powerful poets of the century,_____ and _____."
Ans: (d) In the courtroom scene of 'Their Eyes were (a) Ben Jonson and Abraham Cowley
Watching God' (1937) by Zora N. Hurston the (b) George Herbert and Henry Vaughan
narrative voice we hear is of the omniscient third- (c) John Donne and Andrew Marvell
person voice. Hence option (d) is correct. (d) John Milton and John Dryden
UGC NET/JRF English Paper III, June 2015 185 YCT
Ans: (d) John Milton and John Dryden are the two 63. Azizun, a courtesan from Kanpur in A Tale from
powerful poets of the 17th century whom T. S. Eliot the Year 1857: Azizun Nisa by Tripurari Sharma
says in 'The Metaphysical Poets' (1921) aggravated undergoing self-actualisation says: "Yes I must
the dissociation of sensibility. complete what I've set out to do. I'm not a mere
Hence option (d) is correct. woman." In order to make her impact by her
58. The label 'material feminism' refers to the work attitudinal shift, she________.
of those thinkers who study inequality in terms of (a) challenges the codifiers of the Shariat.
________. (b) forsakes her profession to become a soldier.
(a) gender differences. (c) becomes a political leader.
(b) class differences. (d) becomes a successful dancer.
(c) both gender and class differences. Ans: (b) In Tripurari Sharma's play 'A Tale from the
(d) female consumerism. Year 1857: Azizun Nisa’– Azizun, a courtesan
Ans: (c) 'Material Feminism' refers to the work of forsakes her profession to become a soldier in the
chiefly Rosemary Hennessy, Stevi Jackson and fight of 1857. Hence option (b) is correct.
Christine Delphy in late 1970s, who studied inequality 64. The hermeneutics of suspicion is a term coined by
in terms of both gender and class differences. Hence Paul Ricoeur ________.
option (c) is correct. (A) to designate the postcolonial tendency to see
59. Who among the following displays in her best theory and related reading manoeuvres as a
work the dual influence of feminism and magic global conspiracy.
realism? (B) to describe interpretive bids that challenge and
(a) Pat Barker (b) Muriel Spark seek to overcome compartmentalized cultural
(c) Angela Carter (d) J. K Rowling experiences.
Ans: (c) Angela Carter (1940-92) is known to have (C) who, following Marx, Nietzsche, and Freud,
displayed the dual influence of feminism and magic held that textual appearances are deceptive and
realism in her works. Hence option (c) is correct. texts do not gracefully relinquish their
60. Identify the group of British poets who evidently meanings.
draw upon new trends in literary theory (such as (D) to describe a mode of interpretation that adopts
poststructuralism) and wrote poems that reflect a distrustful attitude towards texts in order to
on themselves and the language used in/by them. elicit otherwise inaccessible meanings or
(a) Seamus Heaney, Michael Longley, Derek implications.
Mahon The right combination according to the code is :
(b) Medbh McGuckian, Denise Riley, Wendy Cope (a) (A) and (B) (b) (A) and (D)
(c) Christopher Middleton, Roy Fisher, J. H. (c) (C) and (D) (d) (B) and (C)
Prynne
Ans: (c) The term 'hermeneutics of suspicion' was
(d) Donald Davie, Charles Tomlinson, Thom Gunn
coined by Paul Ricoeur who held that textual
Ans: (c) Christopher Middleton, Roy Fisher and J. H. appearances are deceptive and texts do not gracefully
Prynne are the British poets who drew upon new relinquish their meanings, and to describe a mode of
trends in Post-structuralism and modern poetry. Hence interpretation that adopts a distrustful attitude towards
option (c) is correct.
texts in order to elicit otherwise inaccessible meanings
61. In Old English other grammatical classes also or implications. Hence option (c) is correct.
had the four cases that nouns had. Which were
65. Which of the following is not a feminist novel?
these grammatical classes?
(a) Ashapurna Debi's Subarnalata
(a) Pronouns and verbs only
(b) Pronouns and adjectives only (b) Rajam Krishnan's Lamp in the Whirlpool
(c) Definite article and verbs only (c) Chudamani Raghavan's Yamini
(d) Pronouns, adjectives and definite article (d) Bani Basu's The Enemy Within
Ans: (d) In old English Pronouns, adjectives and Ans: (d) Bani Basu's 'The Enemy Within' (2002) is
definite article had the four cases that nouns had. about the Naxalite movement in West Bengal in the
Hence option (d) is correct. 1960s and 70s against bonded labour and corrupt
62. Which of the following novels opens with the feudal officers. The other three works are feminist
description of an accident to a hot-air balloon? novels. Hence option (d) is correct.
(a) John Fowles's The Magus 66. The term 'poetic justice' was coined by________.
(b) Ian McEwan's Enduring Love (a) Samuel Taylor Coleridge
(c) James Kelman's How Late It Was, How Late (b) Thomas Rymer
(d) Irvine Welsh's Trainspotting (c) Samuel Johnson
Ans: (b) 'Enduring Love' (1997) by Ian McEwan (d) William Wordsworth
opens with an accident to a hot-air balloon. Ans: (b) The term 'poetic justice' was coined by
Hence option (b) is correct. Thomas Rymer. Hence option (b) is correct.

UGC NET/JRF English Paper III, June 2015 186 YCT


67. Which of the following novels deals with the Thus I; faltering forward,
Biafran War? Leaves around me falling,
(a) July's People Wind oozing thin through the thorn from norward
(b) Waiting for the Barbarians And the woman calling.
(c) Half of a Yellow Sun 71. What suggestion does the opening stanza give of a
(d) Arrow of God woman won or a woman lost?
Ans: (c) 'Half of a Yellow Sun' (2006) is a novel by (a) The contrast between 'now' and 'then'
the Nigerian author Chimamanda N. Adichie. It tells (b) The continuity between 'now' and 'then'
the story of the Biafran war through the characters (c) The phrase "had changed"
Olanna, Ugwu, Odenigbo, Kainene & Richard. Hence (d) The phrase "our day was fair"
option (c) is correct. Ans: (b) The continuity between 'now' and 'then'
68. Which of the following is not true in Dalit suggests the relevance of a woman won or a woman lost
aesthetics as given by Sharan Kumar Limbale? according to the passage given.
(a) The agony, assertion, resistance, anger and Hence, option (b) will be correct answer.
protest of the dalits should be expressed.
(b) Dalit anubhava (experience) should take 72. What is tantalizing about the speaker's experience
precedence over anuman (speculation). in stanza 2?
(c) Sympathy for the dalits should be generated. (a) the disappearance of the lady and the echo of
(d) Ungrammatical language, different from the the voice
standard norms of expression, should be used. (b) the indistinct voice heard by the speaker and the
absence of woman
Ans: (c) Sharan Kumar Limbale's 'Dalit Aesthetics'
does not say that (c) sympathy for the dalits should be (c) the uncertainty of the voice and the speaker's
generated. The other three options are correctly inability to see the woman
expressing his dalit aesthetics. (d) the woman disappearing before her voice is
Hence option (c). fully heard
69. Which of the following is not a critical study by Ans: (c) The first line of stanza 2 arouses expectation
William Empson? from the uncertainty of the voice and the speaker’s
(a) Seven Types of Ambiguity inability to see the woman is most intriguing.
(b) The Dyer's Hand Hence option (c) is correct.
(c) Milton's God 73. What phrase in the poem suggests the possibility
(d) Some Versions of the Pastoral of the woman as "dead"?
Ans: (b) 'The Dyer's Hand' (1962) is a prose book (a) "You had changed …. to me"
authored by W. H. Auden. The other three critical (b) "I knew you then"
works are by Empson. (c) "You being ever dissolved"
Hence option (b) is correct. (d) "Woman much missed"
70. This was a path-breaking feminist essay written Ans: (c) In line 11 the phrase – 'You being ever
in the 1970s which used hybrid terms like "sext" dissolved' suggests the woman as dead.
and "chaosmos." Identify the author. Hence option (c) is correct.
(a) Luce Irigaray (b) Helene Cixous 74. Identify the special sound effect in the line given :
(c) Julia Kristeva (d) Simon de Beauvoir Wind oozing thin through the thorn from
Ans: (b) In her 'The Laugh of the Medusa' (1975) norward…
Helene Cixous coined the hybrid terms like 'sext' and (a) Alliteration (b) Onomatopoeia
'chaosmos'. Hence option (b) is correct. (c) Assonance (d) The use of sibilants
Read the poem and answer the questions that Ans: (a) The repetition of sound in 'thin through the
follow (71 – 75) : thorn' identifies the figure of speech as alliteration.
The Voice Hence option (a) is correct.
Woman much missed, how you call to me, call to me, 75. What longing does the speaker voice?
Saying that now you are not as what you were (a) longing for reunion in the other world
When you had changed from the one who was all to me (b) longing for physical union in the present
But as first, when our day was fair (c) longing for physical union in the town where
Can it be you that I hear? Let me view you then they used to meet
Standing as when I drew near to the town (d) longing for a return to the town where they used
Where you would wait for me : yes, as I knew you then, to meet
Even to the original air-blue gown! Ans: (c) The description of town and gown in the
Or is it only the breeze, in its listlessness second para shows the poet’s (Thomas Hardy) longing
Travelling across the wet mead to me here, for physical union in the town where they used to
You being ever dissolved to wan wistlessness meet.
Heard no more again far or near? Hence option (c) is correct.

UGC NET/JRF English Paper III, June 2015 187 YCT


UGC NET/JRF Exam, June-2015
ENGLISH
Solved Paper-II
Note : This paper contains fifty (50) objective Ans: (a) Louis Mac Neice, C. D. Lewis and Stephen
type questions of two (2) marks each. All questions Spender were followers of W. H. Auden, who was the
are compulsory. leader of the Oxfordians. Hence option (a) is correct.
1. Matthew Arnold’s “touchstones” were short 5. When one line of poetry runs into the next, with
passages, even single lines of classic poetry beside no punctuation to slow the reading, it is a case of
which the lines of other poets may be placed in
________
order to detect the presence or absence of high
(a) caesura (b) consonance
poetic quality. In his “Study of Poetry” Arnold
(c) enjambment (d) hyperbole
cited “touchstones” from such non-English poets
Ans: (c) When one line of poetry runs into the next
as Homer and Dante and also from the English
poets, Shakespeare and Milton. Which English without pause or punctuation it is called enjambment.
poet did he disapprovingly call “not one of the Caesura is a pause in verse.
great classics” in the list below? Consonance is recurrence of similar sounding
(a) Chaucer (b) Sidney consonants in prosody.
(c) Spenser (d) Donne Hyperbole is over-statement.
Ans: (a) In his 'Study of Poetry' Arnold cited Hence option (c) is correct.
'touchstones' and approves Sidney, Spenser and 6. Which of the following is NOT a characteristic of
Donne as classical poets but not Chaucer whom he the Victorian Age?
considers 'not one of the great classics'. (a) The rise of a highly competitive industrial
Hence option (a) Chaucer is correct. technology
2. Samuel Pepys began his diary on _____ (b) An emphasis on strictly controlled social
(a) New Year’s Day 1660 behavior
(b) All Saint’s Day 1662 (c) A romantic focus on home and family
(c) Thanksgiving Day 1665 (d) The growth of rural traditions and movement
(d) New Year’s Day 1667 from large cities
Ans: (a) Samuel Pepys began his diary on 1st Jan. 1660 Ans: (d) Victorian Age characterized -
which is the New Year's Day and finished it in May 1. The rise of a highly competitive industrial
1669. technology - industrial revolution.
Hence option (a). 2. Emphasis on strictly controlled social behavior –
3. On which of the following authors has Peter Victorian restrain.
Ackroyd NOT written a biography? 3. A romantic focus on home and family.
(a) Charles Dickens (b) William Blake 4. Movement to (and not from) cities and genteel city
(c) T. S. Eliot (d) W. B. Yeats culture was evident.
Ans: (d) Peter Ackroyd has not written a biography of Hence option (d).
W. B. Yeats. His biographies are- Dickens (1990), 7. In The Heart of Midlothian, Walter Scott deals
Blake (1995) and T. S. Eliot (1984). with real political and personal details, but
Hence option (d) is correct.
notable among his characters is the depiction of
4. Which group of the following poets was called the ______
Auden Group because they developed a style and (a) Queen Anne (b) Queen Victoria
viewpoint similar to that of W. H. Auden? (c) Queen Caroline (d) Queen Elizabeth
(a) Louis MacNeice, C. D. Lewis, Stephen Spender
Ans: (c) One of Walter Scott's notable character in
(b) John Masefield, Edwin Muir, Norman McCaig
'The Heart of Midlothian' is Queen Caroline.
(c) MacDiarmid, G. M. Hopkins, Edwin Muir
Hence option (c) Queen Caroline is correct.
(d) W. H. Davies, Robert Bridges, John Masefield
UGC NET/JRF English Paper II, June 2015 188 YCT
8. Chaucer's first work, The Book of the Duchess is the recognition or revelation. Synecdoche is a figure
a dream poem on the death of ______ of speech in which a part is made to represent the
(a) Duchess of Malfi (b) Duchess of Lancaster whole or vice-versa.
(c) Duchess of Scotland (d) Duchess of Paris Hence option (a) is correct.
Ans: (b) Chaucer's 'The Book of the Duchess (1369) 13. What significance do we attach to the publication
is a dream poem on the death of Blanche, Duchess of of I Am an Indian in Canada?
Lancaster, wife of the Duke, John of Gaunt. (a) The title refers to the autobiography of an
Hence option (b) is correct. unknown Indian writer longing for the South
9. What was Charles Lamb’s connection with Asian countryside
India? (b) The first ever account of ethnic conflicts within
(a) He was fascinated by the Indian jugglers and Canada
trades-people in London and wrote an essay on (c) The first anthology of Native Canadian writing
them. following the Civil Rights Movement of the
(b) He was fascinated by Eastern mystical religions, 1960s
especially Buddhism (d) The first anthology of writers afflicted by class
(c) He was a clerk for thirty three years in the East and gender differences in Canada of the late
India Company 1970s
(d) He was clerk in South Sea House that prepared Ans: (c) 'I am an Indian’ (1969) published in Canada
patents and documents for British trading was the first anthology of Native Canadian writing by
companies in India Kent Gooderham following the Civil Rights
Ans: (d) Charles Lamb was a clerk in South Sea Movement of the 1960s. Hence option (c) is correct.
House (before his job in East India House for 33 yrs) 14. What is the moral of “The Nun’s Priest’s Tale”?
that prepared patents and documents for British (a) Slow and steady wins the race.
trading companies in India. (b) Greed is the root of all evil.
Hence option (d) is correct. (c) Beauty lies within.
10. Find the odd one among the Marxist critics below : (d) Never trust a flatterer.
(a) George Lukacs (b) Louis Althusser Ans: (d) The moral of Chaucer's 'The Nun's Priest's
(c) Raymond Williams (d) Northrop Frye Tale' is that never trust a flatterer.
Ans: (d) G. Lukacs, L. Althusser and R. Williams Hence option (d) is correct.
belong to Marxist school whereas Northrop Frye is 15. The author of the essay “Silly Novels by Lady
known for his ideas on Archetypes. Novelists” is ________.
Hence option (d) is correct. (a) George Eliot (b) Henry James
11. In the lines “With gold jewels cover every part, / (c) Oscar Wilde (d) Richard Steele
And hide with ornaments their want of art” Ans: (a) The author of the essay 'Silly Novels by
(Essay on Criticism), Pope rejects Lady Novelists' (1856) is – George Eliot.
(a) the ‘Follow Nature’ fallacy Hence option (a) is correct.
(b) artificiality 16. The unquenchable spirit of Robinson Crusoe
(c) aesthetic order struggling to maintain a substantial existence on
(d) poor taste a lonely island reflects_______
Ans: (b) The phrase 'hide (art) with ornaments' (a) man’s desire to return to nature
suggests artificiality in Pope's ' Essay on Criticism' (b) the author’s criticism of colonization
(1711). Hence option (b) is correct. (c) the ideal of rising bourgeoisie
12. The opposite of hyperbole is_______ (d) the aristocrat’s disdain for the harsh social
(a) meiosis (b) inversion reality
(c) anagnorisis (d) synecdoche Ans: (c) The plantations and slave trade deputed in
Ans: (a) Hyperbole is overstatement, so its opposite Robinson Crusoe represent the ideal of rising
will be 'meiosis' which means under- statement. bourgeoisie or capitalist class.
Inversion is reversal of normal order. Anagnorisis is Hence option (c) is correct.

UGC NET/JRF English Paper II, June 2015 189 YCT


17. Who is the author of the collection The Celtic 22. Match the following :
Twilight? (A) “The Function of (i) Terry Eagleton
(a) J. M. Synge (b) Sean O’Casey Criticism”
(c) W. B. Yeats (d) Lady Gregory (B) “The Function of (ii) Richard Ohmann
Ans: (c) 'The Celtic Twilight' (1893) is authored by – Criticism at the
W. B. Yeats. Hence option (c) is correct. Present Time”
(C) The Function of (iii) Matthew Arnold
18. In medieval England a____ was understood to be
Criticism: From
a trained craftsman, one who worked under a
‘The Spectator’ to
master who owned the business.
Poststrucluralism
(a) pardoner (b) summoner
(D) “The Function of (iv) T. S. Eliot
(c ) journeyman (d) manciple
English at the
Ans: (c) Pardoner – represents a person licensed to
Present Time”
sell papal pardons. Summoner – calls on people to
The right matching according to the code is :
appear in a law court. Manciple – is an officer who (A) (B) (C) (D)
buys provisions for a college or monastery or Inn of (a) (iv) (iii) (i) (ii)
court. So, Journeyman, who is a skilled worker (b) (i) (ii) (iii) (iv)
employed by a master who owned the business, (c) (iii) (iv) (i) (ii)
probably on daily wages, is correct. (d) (ii) (iii) (iv) (i)
Hence option (c) Journeyman is correct. Ans: (a) The correct matches are–
19. Christopher Marlowe’s heroes are said to be (A) 'The function of (iv) T. S. Eliot
larger than life, exaggerated both in their faults Criticism' (1923)
and in their qualities. They have a desire for is by
everything in extreme. In one of his plays the (B) 'The function of (iii) Matthew Arnold
hero wants to conquer the whole world. The Criticism at the
name of the play is ________. Present Time' (1865)
(a) The Jew of Malta (b) Doctor Faustus is by
(c) Tamburlaine the Great (d) Edward II (C) 'The function of (i) Terry Eagleton
Ans: (c) The hero in Marlowe's play 'Tamburlaine the Criticism: from
Great' (1590) desires for everything in extreme and 'The Spectator to
wants to conquer the whole world. Poststrucluralism'
Hence option (c) is correct. (1996) is by
20. With what does the speaker claim to be half in (D) 'The function of (ii) Richard Ohmann.
English at the Present
love in “Ode to a Nightingale”?
Time' (2000) is by
(a) the nightingale’s haunting melody
Hence option (a) is correct.
(b) the scented flavor of early summer
(c) the night sky and all the stars 23. Identify the TRUE statement on Thomas More’s
(d) the peace that comes with death Utopia.
(a) Utopia is divided into four parts, each dealing
Ans: (d) In his poem 'Ode to a Nightingale' (1820)
with Raphael Hythloday’s adventures in the
Keats claims to be half in love with the peace that
four suburbs of Antwerp.
comes with death. Hence option (d) is correct.
(b) Utopia is divided into two parts; the first
21. In which chapter of Poetics does Aristotle use the records a conversation between Thomas More
word ‘Catharsis’ in his definition of tragedy? and Raphael Hythloday, and the second is
(a) Chapter IV (b) Chapter VI Hythloday’s discourse on the institutions and
(c) Chapter III (d) Chapter V practices of Utopia.
Ans: (b) Catharsis in tragedy means to relieve of (c) Utopia is divided into two parts; the first is
emotions of pity and fear. In his 'Poetics' Aristotle Thomas More’s discourse on the institutions
used this word first in Chapter VI. and practices of Utopia, and the second a
Hence option (b) is correct. conversation between More and Hythloday.

UGC NET/JRF English Paper II, June 2015 190 YCT


(d) Utopia is divided into four parts, each dealing 28. Which of the following is NOT true of the
with the ordered patterns of towns and cities in Byronic hero?
Antwerp. (a) moody (b) passionate
Ans: (b) Thomas More's 'Utopia' (1516 Latin, 1551 (c) repentant (d) remorse-torn
Eng.) is divided into two parts – the first records a Ans: (c) Byronic hero was moody, passionate,
conversation between Thomas More and Raphael remorse-torn but not repentant.
Hythloday and the second is Hythloday's discourse Hence option (c) is correct.
on the institutions and practices of Utopia. Hence 29. Like many other novelists, Hardy employed
option (b) is correct. language variation (dialect and standard) with a
24. In “The Rime of the Ancient Mariner” what purpose. In this respect which of the following
disaster befalls the ship and the crew? statements is correct?
(a) The ship is caught in ice and breaks into pieces. (a) His major characters such as Tess and Jude
(b) A fierce storm batters the ship and drowns the always speak in local dialects, as per their social
crew. positions.
(c) “Slimy things with legs” attack the ship and kill (b) His major characters such as Tess and Jude
rarely speak in local dialects, in spite of their
many of the crew.
social positions.
(d) The ship is becalmed and the crew dies of thirst.
(c) His major characters such as Tess and Jude
Ans: (d) In Coleridge's 'The Rime of the Ancient
rarely speak in standard language in spite of
Mariner’ (1798, 'Lyrical Ballads'), after the storm the
their social positions.
ship is becalmed and the crew dies of thirst.
(d) His major characters such as Tess and Jude
Hence option (d) is correct.
rarely speak in a mixture of a dialect and
25. Falstaff is a character in ________. standard.
(A) Henry IV Part I
Ans: (b) Hardy's major characters such as Tess and
(B) The Merry Wives of Windsor
Jude were rarely found speaking in local dialects
(C) The Comedy of Errors inspite of their social positions.
(D) Titus Andronicus Hence option (b) is correct.
The right combination according to the code is :
30. “It used to be said,” began a famous English
(a) (A) and (B) (b) (A) and (C)
writer, “everyone had a novel in them… Just
(c) (C) and (D) (d) (A) and (D)
now, though, in 1999, you would probably be
Ans: (a) Falstaff is a comic character that appears in obliged to doubt the basic proposition: What
Shakespeare's 'Henry IV Part I' (1597) and in 'The everyone has in them, these days, is not a novel
Merry Wives of Windsor' (1602). but a memoir". Identify the source :
Hence option (a) is correct. (a) Martin Amis, Experience
26. In her essay “Professions for Women” Virginia (b) Michel Butor, Passing Time
Woolf finds an analogy between the act of writing (c) John Fowles, The French Lieutenant's Woman
and ________. (d) Julian Barnes, Flaubert's Parrot
(a) driving a motor car (b) riding a horse Ans: (a) Martin Amis in 'Experience' (2000) said the
(c) fishing (d) gardening above lines that "...What everyone has in them, these
Ans: (c) In her essay 'Professions for Women' (1931) days, is not a novel but a memoir".
V. Woolf finds an analogy between the act of writing Hence option (a) is correct.
and fishing. Hence option (c) is correct. 31. The opening sixteen lines of Paradise Lost
27. The ascension of King James I in____________ comprise :
inaugurated the Jacobean age. (a) One sentence (b) Two sentences
(a) 1600 (b) 1601 (c) Three sentences (d) Four sentences
(c) 1603 (d) 1609 Ans: (a) The first 16 lines of Milton's 'Paradise Lost'
Ans: (c) Jacobean Age begins with ascension of King (1667) comprise one sentence.
James I in 1603. Hence option (c) is correct. Hence option (a) is correct.

UGC NET/JRF English Paper II, June 2015 191 YCT


32. Who among the following poets compared human Ans: (b) Struldbruggs in Jonathan Swift's 'Gulliver's
tears to "love's wine"? Travels' (1726) are – people exempt from natural
(a) Ben Jonson (b) John Donne death. Hence option (b) is correct.
(c) Andrew Marvell (d) John Suckling 38. Margaret Atwood has tried a revisionist writing
Ans: (b) John Donne in his 'Twickenham Garden' of a crucial scene in Hamlet called "Gertrude
compares human ‘tears’ to ‘love’s wine’’. Talks Back". The scene in Atwood opens with a
Hence option (b) is correct. reference to the name of an implied listener. Who
is this implied listener?
33. Ernest Pontifex is a character in
(a) Hamlet (b) Ophelia
(a) Tono Bungay (b) The Man of Property (c) Polonius (d) Claudius
(c) The Way of All Flesh (d) Nostromo
Ans: (a) The implied listener in Margaret Atwood's
Ans: (c) Ernest Pontifex is a character in 'The Way of 'Gertrude Talks Back' is – Hamlet. It was a piece in
All Flesh' by S. Butler (1903). her short story collection 'Good Bones' published in
Hence option (c) is correct. 1992. Hence option (a) is correct.
34. In which of the following stories does Rudyard 39. Samuel Johnson wrote London in imitation of
Kipling present a newspaper editor who recounts ________.
his dealings with a couple of "loafers"? (a) Horace (b) Ovid
(a) "His Chance in Life" (c) Juvenal (d) Moschus
(b) "Thrown Away" Ans: (c) Samuel Johnson's 'London' is written in
(c) "Lispeth" imitation of – Juvenal and was published in 1738.
(d) "The Man Who Would Be King" Hence option (c) is correct.
40. Which of the following is NOT written by Buchi
Ans: (d) A newspaper editor is a character in Rudyard
Emecheta?
Kipling’s story – 'The Man Who Would Be King'
(a) The Joys of Motherhood
(1888). Hence option (d) is correct.
(b) Second-Class Citizen
35. Trying to capture the upbeat mood of 1964-65, (c) A Question of Power
the poet Thom Gunn said: "They stood for a (d) Kehinde
great optimism, barriers seemed to be coming Ans: (c) Buchi Emecheta has written – 'The Joys of
down all over, it was as if World War II had Motherhood' (1979), 'Second-Class Citizen' (1974)
finally drawn to close, there was an openness and and 'Kehinde' (1994). 'A Question of Power' (1973) is
high-spiritedness and relaxation of mood". Who written by – Bessie Head.
were 'they'? Hence option (c) is correct.
(a) The Beatles (b) The Rolling Stones 41. Samuel Johnson's use of the term "metaphysical"
(c) The New Left (d) The Arts Council folks in a piece of criticism was ________.
Ans: (a) 'They' in Thom Gunn's statement above is (a) approving (b) disapproving
(c) positive (d) accidental
used for – The Beatles.
Ans: (b) Samuel Johnson's use of the term
Hence option (a) is correct.
'metaphysical' was – disapproving in his 'Lives of
36. In Paradise Lost Milton presents the action of the
English Poets' published in 1781.
fall of man in two stages in Books ________. Hence option (b) is correct.
(a) IV and IX (b) IV and VIII
42. "I am not an angel ……. and I will not be one till
(c) III and IX (d) V and X I die: I will be myself." This is ________.
Ans: (a) In Milton's 'Paradise Lost' (1667) the Fall of (a) Maggie Tulliver in Mill on the Floss
Man is presented in two stages in Books IV and IX. (b) Aurora Leigh in the eponymous poem
Hence option (a) correct. (c) Jane Eyre in the eponymous novel
37. In Gulliver's Travels Struldbruggs are ________. (d) Betty Higdon in Our Mutual Friend
(a) people replete with abstract learning. Ans: (c) The statement 'I am not an angel ….. and I
(b) people exempt from natural death. will not be one till I die : I will be myself' is said by –
(c) people persecuted by pets and servants. 'Jane Eyre' (1847) (in the eponymous novel) by
(d) people lured by a new ideal. Charlotte Bronte. Hence option (c) is correct.

UGC NET/JRF English Paper II, June 2015 192 YCT


43. Who among the following playwrights was the Ans: (c) Central character of Derek Walcott's 'Dream
son of a gardener? on the Monkey Mountain' (1970) is – Makak who is a
(a) Harold Pinter (b) Joe Orton charcoal burner. Hence option (c) is correct.
(c) Tom Stoppard (d) Edward Bond 48. The phrase "darkness visible" (Paradise Lost,
Ans: (b) Joe Orton was the son of a gardener. 1.63) is an example of ________.
Hence option (b) is correct. (a) periphrasis (b) pun
44. "He is the very pineapple of politeness!" This (c) oxymoron (d) transposition
sentence is an example of ________. Ans: (c) The phrase 'darkness visible' is an example of
(a) paronomasia (b) spoonerism – oxymoron, which means an idea or phrase in which
(c) malapropism (d) anaphora contradictory terms appears in conjunction.
Ans: (c) The sentence 'pineapple of politeness' is an Periphrasis means use of indirect or circum-locutary
example of – Malapropism which means the mistaken speech. Transposition is to trans-locate a segment to a
use of a similar sounding word. Paronomasia means different site. Pun is a joke.
a play on words or a pun. Spoonerism means an error Hence option (c) is correct.
in speech. Anaphora is used to refer back to a word 49. What is common to writers such as Sam Selvon
used earlier in text or conversation. (The Lonely Londoners), Timothy Mo (Sour
Hence option (c) is correct. Sweet), and Hanif Kureishi (The Black Album)?
45. Ferdinand de Saussure argued that meaning is (a) All of them are brilliant writers of
generated through ________. autobiographies who tell stories and write
(a) a system of structured differences in language poetry.
(b) a system of random differences in language (b) They use Standard English with some Creole
(c) a system of structured references in language inflections peculiar to the Caribbean.
(d) a system of random references in language (c) They are diasporic writers who depict
Ans: (a) Saussure argued that meaning is generated postcolonial London very different from its
through a system of structured differences in language colonial representations.
– the sign, signifier, signified and langue, parole etc (d) They contrast the 'First Nations' with local
are ideas in semiotics by him. populations of their respective countries.
Hence option (a) is correct.
Ans: (c) Sam Selvon, Timothy Mo and Hanif
46. Identify the group known as "The Wesker Kureishi are diasporic writers who depict postcolonial
Trilogy"? London very different from its colonial represent-
(a) The Growth of the Soil, Game of Life, In the ations. Hence option (c) is correct.
Grip of Life
50. F. R. Leavis and Q. D. Leavis launched a critical
(b) Chicken Soup with Barley, Roots, I'm Talking
journal devoted to the moral centrality of English
about Jerusalem
Studies. Name the Journal.
(c) The Four Seasons, Chips with Everything,
(a) The English Historical Review
Golden City
(b) The Criterion
(d) Lunatics and Lovers, The Patriots, Dead End
(c) Scrutiny
Ans: (b) The Wesker Trilogy by – A. Wesker:
(d) The Edinburgh review
'Chicken Soup with Barley' (1958),
Ans: (c) Journal by F. R. Leavis and Q. D. Leavis is –
'Roots' (1959),
'Scrutiny' which ran from 1932-53.
'I'm Talking about Jerusalem' (1960).
'The English Historical Review' was established
Hence option (b) is correct.
in 1886 by John Dalberg – Acton.
47. Who is the central character of Derek Walcott's
'The Criterion' ran from 1922-39 by T. S. Eliot.
Dream on the Monkey Mountain?
'The Edinburgh Review' ran from 1802-1929 and
(a) Diana Guinness, one of the Mitford Sisters
was established by F. Jeffrey, S. Smith and H.
(b) Jordan, a fantasist
Brougham and A. Constable was the publisher.
(c) Makak, a charcoal burner
Hence option (c) is correct.
(d) Eva Smith, a seamstress
UGC NET/JRF English Paper II, June 2015 193 YCT
UGC NET/JRF Exam, December-2015
ENGLISH
Solved Paper-III
Note : This paper contains seventy five (75) (d) The culturally dominant global empire is the
objective type questions of two (2) marks each. All only one that really matters. We signify that
questions are compulsory. Empire with E upper case.
1. Thomas and Henrietta Bowdler's edition of Ans: (c) Hardt and Negri’s book titled ‘Empire’ (2000)
'The Family Shakespeare' gave rise to the word is praised as the communist manifesto of 21st century.
"Bowdlerize". What does it mean? According to them the present day empire does not have
(a) the expurgation of indelicate language an identifiable location or centre. Hence we ought to
(b) the modernization of archaic vocabulary differentiate this view with an upper case E in ‘Empire.’
(c) the insertion of bawdy songs Hence option (c).
(d) the expansion of female characters 5. Who among the following critics discerned in
Ans: (a) The word ‘Bowdlerize’ given by Thomas and the Shelleyan Lyric the signs "of adolescence"?
Henrietta Bowdler’s expurgated edition of ‘The Family (a) F.R. Leavis (b) T.S. Eliot
Shakespeare’ (1807) was intended to make the plays (c) Cleanth Brooks (d) I.A. Richards
more appropriate for 19th century women and children. In Ans: (b) T.S. Eliot in 1933 discerned the signs ‘of
verb form this word has come to mean the censorship or adolescence’ in the Shelleyan Lyric.
expurgation of indelicate language not only of literature Hence option (b).
but also of motion pictures and television serials.
Hence option (a). 6. Two among the following critical journals
became strongly associated with New
2. First follow _______and your judgement Criticism.
frame. By her just _______, which is still the (A) Partisan Review (B) Southern Review
same. Supply the appropriate words to fill in (C) Kenyon Review (D) Hudson Review
the blanks.
The right combination according to the code is :
(a) wit, law (b) reason, rule
(a) (A) and (B) (b) (A) and (D)
(c) nature, standard (d) sense, criterion
(c) (B) and (C) (d) (C) and (D)
Ans: (c) The above lines from Alexander Pope’s ‘An
Essay on Criticism’ (1711), lines 17-18 go as – Ans: (c) The two journals, from among the options,
strongly associated with New Criticism are – (B)
“First follow Nature, and your judgement frame,
Southern Review, which began in 1935 edited by
by her just standard, which is still the same.” Brooks and Warren; and the (C) Kenyon Review,
Hence option (c). founded by John Crowe Ransom in 1939 and served as
3. Preparation of vocabulary list for the purpose editor till 1959.
of English language teaching was carried out Hence option (c).
by _____. 7. Match the columns :
(a) Otto Jespersen (b) Noam Chomsky (A) Robert Burton (i) Urn Burial
(c) N.S. Prachu (d) Michael West (B) Richard Hooker (ii) The Unfortunate
Ans: (d) Dr. Michael P. West (1888-1973) was an Traveller
English language teacher and researcher credited to (C) Thomas Browne (iii) The Anatomy of
have produced ‘A General Service List of English Melancholy
Words’ (1953). (D) Thomas Nashe (iv) Of the Laws of
Hence option (d). Ecclesiastical
4. Michael Hardt and Antonio Negri prefer to use Politie
"Empire" rather than imperialism. According (A) (B) (C) (D)
to them : (a) (iii) (i) (ii) (iv)
(a) There is only one empire and we had better (b) (iv) (ii) (i) (iii)
recognize it. Hence the Empire with E upper (c) (iii) (iv) (i) (ii)
case. (d) (i) (iii) (iv) (ii)
(b) There may be many empires but only one is
Ans: (c) The correct matches are –
patently visible and operational. That is
denoted by Empire with E upper case. (A) Robert Burton – (iii) 'The Anatomy of
(c) The present day empire does not have an Melancholy' (1621)
identifiable location or centre. Hence we (B) Richard Hooker – (iv) ‘Of the Laws of
ought to differentiate this view of Empire Ecclesiastical
with E upper case. Politie’ (1594)

UGC NET/JRF English Paper III, December 2015 194 YCT


(C) Thomas Browne – (i) 'Hydriotaphia, or Urn Ans: (b) In Ivan Turgenev’s 'Fathers and Sons' (1862)
Burial' (1658) Bazarov is the nihilist who is a medical student who
(D) Thomas Nashe – (ii) 'The Unfortunate challenges the liberal ideas of the Kirsanov brothers and
the orthodox feelings of his parents.
Traveller, or the life
Hence option (b).
of Jack Wilton'
12. Which of these works in nineteenth-century
(1594). Russian fiction originated the type of
Hence option (c). Superfluous Man?
8. Which of the following characters in The White (a) The Diary of a Superfluous Man
Devil describes the glory of great men as: (b) A Hero of Our Own Times
"Glories, like glow worms a far off shine bright (c) Eugene Onegin
/ But looked to near have neither heat nor (d) Dead Souls
light". Ans: (c) 'Eugene Onegin' by Alexander Pushkin is a
(a) Vittoria (b) Lodovico Russian fiction originates the type of superfluous man.
(c) Flamineo (d) Cornelia It is a story of a Byronic youth who wastes his life,
allows the girl who loves him to marry another.
Ans: (c) UGC marked the answer as option (a) Vittoria
Hence, option (c) will be correct answer.
which is wrong. In the above lines from John Webster’s
play ‘The White Devil’ (1612), the speaker of the above 13. What is Gilgamesh?
lines is Flamineo. Here Webster compares a prince’s (i) a Babylonian epic poem
glories to a glow –worm’s light which despite looking (ii) a Series of gnomic verses
magnificent for some time proves hollow on close (iii) a classical play
inspection. The irony of deceptive outward appearances (iv) the story of a harsh ruler
is expressed through the character of Flamineo in Act (a) (i) and (ii) (b) (iii)
V, Sc. 1, lines 41-2 of the play. (c) (i) and (iv) (d) (ii)
Hence option (c). Ans: (c) ‘Gilgamesh’ is a Babylonian epic poem which
features the story of a harsh Sumerian ruler, the King of
9. In which of Philip Larkin's poem does he refer Uruk dating circa 2100 B.C.
to "long uneven lines" of men waiting to be Hence option (c).
enlisted for the war?
14. American Dictionary of the English Language
("Never such innocence again" concludes the was the work of ______ published in _______.
poem) (a) Merriam Webster, 1903
(a) "Mr. Bleaney" (b) "Mc MXIV" (b) H.L. Mencken, 1930
(c) "Ambulances" (d) "Sad Steps" (c) Noah Webster, 1828
Ans: (b) The above extract is from Philip Larkin’s (d) Benjamin Franklin, 1768
poem ‘Mc MXIV’ (1964). Ans: (c) Noah Webster published the American
Hence option (b). Dictionary of the English language in 1828.
10. In Franz Kafka's Metamorphosis, Gregor Hence option (c).
Samsa one morning found himself changed in 15. Which of the following texts of Amitav Ghosh
his bed to a monstrous kind of vermin. The is based on the refugee occupation of an island
most difficult thing for Samsa was: in the Sundarvans?
(a) to look at his image in the mirror (a) Sea of Poppies (b) The Hungry Tide
(b) to remember what happened the day before (c) River of Smoke (d) The Glass Palace
(c) to communicate with anyone Ans: (b) ‘The Hungry Tide’ (2004) by Amitav Ghosh is
based on the refugee occupation of an island in the
(d) to brush his teeth Sundarvans in Bay of Bengal and revolves around
Ans: (c) In Kafka’s novel ‘Metamorphosis’ (1915), the Piyali Roy, a marine biologist and Fokir, an illiterate
most difficult thing for Gregor Samsa after he wakes up fisherman.
as an insect monster is his speechlessness. He was not Hence option (b).
able to communicate with anyone which he later 16. Which of the following is described by Robert
realized that his voice had also transformed. Browning as "A Child's Story" ?
Hence option (c). (a) "Bells and Pomegranates"
11. Identify the individual who is a nihilist from (b) "Pauline"
the following : (c) "Fifine at the Fair"
(a) Pechorin in A Hero of Our Times (d) "The Pied Piper of Hamelin"
(b) Bazarov in Fathers and Sons Ans: (d) Robert Browning has described ‘The Pied
(c) Levin in Anna Karenina Piper of Hamelin’ (1888) as a child’s story.
(d) Oblomov in Oblomov Hence option (d).

UGC NET/JRF English Paper III, December 2015 195 YCT


17. Identify the New Critic who served as the (B) Aporia (ii) A term used by
cultural attaché at the American Embassy in Mikhail Bakhtin
London from 1964 to 1966 : to describe the
(a) John Crowe Ransom variety of languages
(b) Cleanth Brooks and voices within
(c) Allen Tate a novel.
(d) Robert Penn Warren (C) Intertextuality (iii) An irresolvable
Ans: (b) Cleanth Brooks served as the cultural attaché internal contradiction
at the American Embassy in London from 1964-66. or logical disjunction
Hence option (b). in a text, usually
18. "The Gilded Age" refers to a period of associated with
American history between 1870 and the first deconstructive
decades of the twentieth century. thinking.
Who among the following American writers is (D) Heteroglossia (iv) A term made famous
credited with the coining of the term? by William Empson
(a) F. Scott Fitzgerald to indicate that a
(b) Mark Twain word, phrase, or text
(c) William Dean Howells can be interpreted
(d) Theodore Dreiser in more than
Ans: (b) Mark Twain coined the term as the title of his ` one way.
book – ‘The Gilded Age: A Tale of today’ (1873) (A) (B) (C) (D)
which refers to the period of American history (a) (iv) (i) (ii) (iii)
between 1870s to 1910s. (b) (ii) (iii) (iv) (i)
Hence option (b). (c) (iv) (iii) (i) (ii)
19. The Decline and Fall of the Roman Empire in (d) (iii) (iv) (i) (ii)
six volumes was a great achievement by Ans: (c) The correct matches are –
Edward Gibbon. It was published between (A) Ambiguity (iv) A term made famous by
1776 and 1788, two significant dates that. William Empson to
(a) Signalled the end of the Napoleonic wars and indicate that a word,
the rise of Feudalism. phrase, or text can be
(b) Signalled the American Revolution and the interpreted in more
French Revolution. than one way.
(c) Covered the fall of peasantry and the rise of
bureaucracy in England.
(B) Aporia (iii) An irresolvable
(d) Suggest the period of Queen Anne's reign.
internal contradiction
Ans: (b) The two dates are significant as they signaled or logical disjunction
the American Revolution and the French Revolution. in a text, usually
Hence option (b). associated with
20. Being so caught up, so mastered by the brute deconstructive thinking.
_____ of the air, Did she put on his knowledge (C) Intertextuality (i) A term coined by
with his power, Before the _______ beak could Julia Kristeva to
let her droop. refer to the fact
Yeats, "Leda and the Swan" that texts are
Choose the right words for the blanks: constituted by a
(a) beast, shiny (b) force, animal ‘tissue of citations’.
(c) blood, indifferent (d) thrust, irate (D) Heteroglossia (ii) A term used by
Ans: (c) In Yeats’ poem ‘Leda and the Swan’ the right Mikhail Bakhtin
words for the blanks are – Blood and indifferent to describe the
Hence option (c). variety of languages
21. Match the following : and voices within
Terms Description a novel.
(A) Ambiguity (i) A term coined by Hence option (c).
Julia Kristeva to 22. Did I request thee, Maker, from my clay To
refer to the fact mould me man? Did I solicit thee From
that texts are darkness to promote me?
constituted by a Which nineteenth-century work bears these
"tissue of citations." lines from Paradise Lost as epigraph?
UGC NET/JRF English Paper III, December 2015 196 YCT
(a) Wuthering Heights (b) Frankenstein Reason (R) : It follows, if even only part of all
(c) Don Juan (d) Jude the Obscure this is true that the arts do have an important
Ans: (b) The above lines have been used by social function. [...] Arts can give greater depth to
Mary Shelley in 'Frankenstein' (1818). a society's sense of itself. [...] A country without
great art might be a powerful collection of
Hence option (b). thriving earthworms but would be a sorry society.
23. A literary researcher now faced with choosing (a) Reason (R) is perfectly aligned with
between a print text and its digital counterpart Assertion (A)
chooses the latter mostly to : (b) Assertion (A) is unrelated to Reason (R)
(a) facilitate the consultation of an exhaustive (c) Assertion (A) hardly reflects Reason (R)
bibliography (d) Reason (R), in fact, contradicts Assertion (A)
(b) avoid the expense of buying books Ans: (a) Reason is in perfect alignment with Assertion.
(c) look for specific words and phrases and lines Hence option (a).
(d) enhance his/her understanding of textual 28. Which of the following is NOT an example of
variants, if any, between the two media derivational morpheme ?
Ans: (c) A researcher chooses a digital edition of a text (a) friend - friendship
over print version for the ease of finding specific words, (b) courage - courageous
phrases and lines. (c) rely -reliable
Hence option (c). (d) climate - climactic
24. Which of the following statements on Hudibras Ans: (d) Climate and climactic (climax) have two
are true? different meanings and so are not derivational
(A) It is a novel written by Matthew Prior. morphemes.
(B) It is a satirical poem published in 3 parts. Hence option (d).
(C) Hudibras was written by Samuel Butler. 29. Which of these statements is incorrect about
(D) Hudibras discusses complex issues of justice, presentism and its basic premises?
politics and religion. (a) Hugh Grady is its principal proponent.
(a) (C) and (D) are true (b) Our knowledge of works from the past is
(b) (A) and (D) are true conditioned by and dependent upon the
(c) (B) and (C) are true ideologies of the present.
(d) (A) and (B) are true (c) presentism does not contextualize cultural
Ans: (c) ‘Hudibras’ is a satirical poem by Samuel production in the same way or make use of
the theorists that New Historicism does.
Butler published in three parts in 1663, 64 and 78
(d) Historicism itself necessarily produces an
Hence option (c).
implicit allegory of the present in its
25. The formalist critic ________ mocked the configuration of the past.
character-based criticism of _______ by posing Ans: (c) The context for the emergence of presentism
a famous question, "How many children had itself is the dominance of new historicism and cultural
Lady Macbeth"? materialism within early modern studies in English
(a) F.R. Leavis, E.K. Chambers. literary criticism.
(b) Cleanth Brooks, F.L. Lucas Hence the incorrect statement is option (c).
(c) Monroe Beardsley, Kenneth Burke 30. "Where there is leisure for fiction, there is little
(d) L.C. Knights, A.C. Bradley grief", was Samuel Johnson's criticism of a
Ans: (d) The formalist critic L.C. Knights, mocked the famous poem. Which poem was it?
character based criticism of A.C. Bradley in his famous (a) P.B. Shelley's "Adonais"
classic essay of modern criticism – ‘How many children (b) Philip Sidney's "Astrophel and Stella"
had Lady Macbeth’? (1933). (c) Thomas Gray's "Elegy Written on a Country
Hence option (d). Churchyard"
26. Which of the following pair of words does not (d) John Milton's "Lycidas"
have two different vowel glides? Ans: (d) The above line from Milton’s ‘Lycidas’ (1638)
(a) care, pure (b) write, freight was quoted by Dr. Samuel Johnson in his ‘Lives of
(c) caught, court (d) eight, ate Poets’ (1779).
Hence option (d).
Ans: (d) Eight and ate have similar vowel sounds while
others have different. 31. The story is grounded in the forbidden nature
of Aschenbach's Obsession with a young boy;
Hence option (d). its author ultimately links the obsession with
27. Assertion (A) : Arts will often work obliquely, by death, disease and esthetic disintegration.
myth or symbol. They may make their best The author of the story is :
'criticism of life' simply by being; they may best (a) Goethe (b) Mann
state by not stating. (c) Borges (d) Proust
UGC NET/JRF English Paper III, December 2015 197 YCT
Ans: (b) Aschenbach is the protagonist in Thomas 37. "How all their plays be neither right tragedies,
Mann’s short story collection ‘Death in Venice’ (1930). nor right comedies, mingling kings and clowns,
Hence option (b). not because the matter so carrieth it, but thrust
32. Which of the following novels of Joseph in the clown by head and shoulders to play a
Conrad is set in Malay? part in majestical matters".
(a) Nigger of the Narcissus What term does Philip Sidney use to
(b) Lord Jim characterize such plays and which of the
(c) Nostromo unities of Aristotle do they violate ?
(d) Heart of Darkness (a) mongrel tragicomedy; unity of action
Ans: (b) Joseph Conrad’s ‘Lord Jim’ (1900) is set in (b) mixed tragedies, unity of action
Malay. The native Malays rule the fictional Patusan (c) multi-plot drama; unity of time
country. (d) mingled yarn; unity of place
Hence option (b). Ans: (a) In his 'Defence of Poesie' (1583) Sir. Philip
33. Nuruddin Farah's Maps tells the story of_____. Sidney said –
(a) Abida (b) Abu “But, besides these gross absurdities, how all their
(c) Askar (d) Andy plays be neither right tragedies nor right comedies,
Ans: (c) ‘Maps’ (1986) is the first of the trilogy ‘Blood mingling kings and clowns, not because the matter so
in the Sun’ by Somalian writer Nuruddin Farah. It tells carrieth it, but thrust in the clown by head and shoulders
the story of Askar. ‘Maps’ was followed by ‘Gifts’ to play a part in majestical matters, with neither decency
(1993) and ‘Secrets’ (1998) in the trilogy. nor discretion; so as neither the admiration and
Hence option (c). commiseration, nor the right sportfulness, is by their
34. One of the most quoted statements on poetry mongrel tragi-comedy obtained”.
by John Keats is reproduced with blanks Hence option (a).
below. Complete the statements with correct 38. There is a large number of religious poems in
words. Old English Poetry. One of the finest is the
"If Poetry ________ as naturally as the leaves Dream of the Rood. The words 'the Rood' in the
to a tree, it ________ at all". title means:
(a) does not come; had better not come (a) the Cross (b) the Christian
(b) comes not; had better not come (c) the Infidel (d) the Cardinal
(c) come not; had better not come
(d) come not; did not come Ans: (a) The ‘Rood’ in the old English poem ‘The
Dream of the Rood’ is the crucifixion cross set above
Ans: (c) In his letter to John Taylor, 1818 Keats wrote – the entrance of a church or a large sculpture or painting
“…If poetry come not as naturally as the leaves to a of the crucifixion of Jesus.
tree, it had better not come at all….”
Hence option (a).
Hence option (c).
35. Manohar Malgonkar was a hunter, a 39. Identify from among the following the one
lieutenant colonel in the British army, and a incorrect statements on M. Anantanarayanan's
tea-planter. Silver Pilgrimage (1961) :
He also wrote a memorable novel about the (a) M. Anantanarayanan modelled this narrative
Sepoy Mutiny, especially Peshwa Baji Rao II. on the well-known picaresque novels in
What is that novel ? English.
(a) A Distant Drum (b) 'The Silver Pilgrimage' is M.
(b) A Combat of Shadows Anantanarayanan's only foray into fiction.
(c) A Bend in the Ganges (c) This novel is mainly an account of the
(d) The Devil's Wind adventures of Jayasurya, a Sri Lankan prince
Ans: (d) Manohar Malgonkar’s ‘The Devil’s Wind’ of the sixteenth century.
(1972) is a historical novel that tells the story of the (d) Among the literary texts quoted by the novel
Indian Mutiny and Peshwa Baji Rao II or Nana Saheb. are lines from Shakespeare, Donne and Rilke
Hence option (d). and classical Tamil poets.
36. Who wrote the screenplay for the film version Ans: (a) M. Anantanarayanan’s ‘The Silver
of John Fowles's novel The French Lieutenant's Pilgrimage’(1961) is not modeled on any well known
Woman? picaresque English novel but on the 7-8th century prose
(a) Harold Pinter (b) Tom Stoppard romance of Dandin’s 'Das Kumara – Charitra' and is
(c) David Mamet (d) Caryl Phillips the author's only foray into fiction which narrates the
Ans: (a) Harold Pinter wrote the screenplay in 1981 for account of the adventures of a 16th cent. Sri Lankan
John Fowle’s novel ‘The French Lieutenant’s Woman’ prince Jayasurya quoting famous lines from Donne,
(1969). Shakespeare Rilke and classical Tamil Poets.
Hence option (a). Hence option (a).

UGC NET/JRF English Paper III, December 2015 198 YCT


40. Listed below are the titles of some influential Ans: (d) The correct order of Emerson’s quote in his
books by Frank Kermode. Identify which one 1841 essay ‘Self-reliance’ is – “Misunderstood! It is a
of the titles that does NOT belong to the set. right fool's word. Is it so bad then to be misunderstood?
(a) The Sense of an Ending Pythagoras was misunderstood, and Socrates, and Jesus,
(b) Not Entitled - A Memoir and Luther, and Copernicus, and Galileo, and Newton,
and every pure and wise spirit that ever took flesh. To
(c) The Genesis of Secrecy
be great is to be misunderstood.”
(d) The Great Code: The Bible and Literature Hence option (d) e, d, c, b, a.
Ans: (d) Frank Kermode’s books are – 44. X ... Do you know it is nearly seven?
(a) 'The Sense of an Ending: Studies in the Theory of Y (irritably) Oh! it always is nearly seven?
Fiction'. – 1967 X well, I'm hungry.
(b) ‘Not Entitled - A Memoir’ – 1995 Y I never knew you when you weren't ...
c) ‘The Genesis of Secrecy’ – 1979 X What shall we do after dinner? Go to a theatre?
(d) ‘The Great Code: The Bible and Literature’ (1982) Y Oh no! I loathe listening.
is authored by Northrop Frye. X Well, let us go to the club?
Hence option (d). Y Oh no! I hate talking.
41. Identify the one erroneous statement on X Well, we might trot round to the Empire at ten?
Neoclassicism listed below : Y Oh no! I can't bear looking at things. It is so
(a) Lodovico Castelvetro and Tasso greatly silly.
influenced English writers like Milton and X Well, what shall we do?
Dryden. Y Nothing!
(b) Neoclassicism took its final form during the X It is awfully hard work doing nothing.
reign of Louis XIV (1638-1715). However, I don't mind hard work where there
(c) Boileau's L'Art Poetique influenced Pope's is no definite object of any kind.
Essay on Criticism. Identify the speakers in this dialogue:
(a) Aston (X) to Mick (Y) The Caretaker
(4) The English relation to Neoclassicism was
one of dialogue. Most literally, this dialogue (b) Algernon (X) to Jack (Y) The Importance of
Being Earnest
is effected in Addison's An Essay on
(c) Lucky (X) to Pozzo (Y) Waiting for Godot
Dramatic Poesy.
(d) Man (X) to the Woman (Y) The Waste Land
Ans: (d) ‘An Essay on Dramatic Poesy’ (1668) was
Ans: (b) In the above extract from Oscar Wilde's ‘The
authored by Dryden and not by Addison. Importance of Being Earnest’ (1985), X is Algernon
Hence option (d) is erroneous. and Y is Jack.
42. In his 'Poems of Love and War', a collection of Hence option (b).
classic Indian poems in English translation, 45. Which of these Greek plays was a source for
A.K. Ramanujan sought to revive an ancient The Winter's Tale?
_______ poetic tradition. Choose the right (a) Iphigeneia at Aulis
word. (b) Alcestis
(a) Tamil (b) Sanskrit (c) Medea
(c) Kannada (d) Pali (d) Iphigeneia at Tauris
Ans: (a) In his ‘Poems of Love and War: From the Ans: (b) The source of Shakespeare’s ‘The Winter’s
Eight Anthologies and the 'Ten Long Poems of Tale’ (1609-11 wr.) was the ancient Greek tragedy by
Classical Tamil' (1985), A. K Ramanujan sought to Euripides – ‘Alcestis’ (438BC).
revive an ancient Tamil poetic tradition. Hence option (b).
Hence option (a). 46. Sweet is the lore which nature brings;
43. Arrange the following sentences in the order in Our meddling intellect
which they appear in Emerson's "Self- Mis-shapes the beauteous forms of things:
Reliance" : We murder to dissect.
a. To be great is to be misunderstood. - Wordsworth
b. Pythagoras was misunderstood, and Socrates, Which of the following best summarises the
and Jesus, and Luther, and Copernicus, and speaker's position?
Galileo, and Newton, and every pure and (a) Nature is incomplete without a human
wise spirit that ever took flesh. witness to attest to its beauty.
c. Is it so bad then to be misunderstood! (b) Human endeavours will succeed only if the
laws of nature are taken into account.
d. It is a right fool's word.
(c) Nature yields a pleasure superior to that
e. Misunderstood! derived from intrusive human inquiry.
(a) a, e, d, c, b (b) e, a, b, c, d (d) The flaws inherent in human nature are also
(c) c, d, a, b, e (d) e, d, c, b, a evident in the natural world.
UGC NET/JRF English Paper III, December 2015 199 YCT
Ans: (c) In the above extract from Wordsworth’s ‘The Ans: (d) In Shakespeare’s time the acting company
Tables Turned’ (1798), the poet is saying that – Nature owned the rights to a theatrical script.
yields a pleasure superior to that derived from intrusive Hence option (d).
human inquiry. 51. Which of the following sentences uses more
Hence option (c). than three cohesive devices?
47. a. Jean Baudrillard tells us postmodern societies (a) At that time a person could drive for miles
are marked by simulacra. without seeing a house.
b. By simulacra he means non-representations (b) All of them could recite the poem yesterday.
of reality. (c) You can use a pencil, though not a pen, to
c. Simulacra artificially produce a mediated write your name.
word masquerading as authenticity. (d) As soon as Mohan entered the stadium the
d. It was not Jean Baudrillard but his crowd cheered.
interpreters who coined the term "simulacra". Ans: (c) Option (c) uses more than three cohesive
Which of the above statements are true? devices which are linking words.
(a) b, c and d (b) a and c Hence option (c).
(c) c and d (d) b and c 52. Match the columns :
Ans: (b) In his ‘Simulacra and Simulacrum’ (1994 Indian Text English Translator
Eng.) Jean Baudrillard tells us that post-modern (A) The love of (i) William Jones
societies are marked by simulacra which artificially Kamarupa and
produce a mediated world masquerading as authenticity. Kamalata
He talks about reality and hyper-reality in the context of (B) Ramayana (ii) Nathaniel Halhed
life in general and in relation to society. (C) Upanishads (iii) W. Franklin
The word ‘simulacra’ is derived from the Latin (D) Abhijnan (iv) T. H. Griffith
word ‘simulacrum’ which means likeness or similarity Sakuntalam
and the term was first used by Plato. (A) (B) (C) (D)
Hence option (b), (a) and (c) are correct. (a) (iv) (iii) (ii) (i)
48. Which of the following is correct as the natural (b) (iii) (iv) (ii) (i)
order of language acquisition? (c) (ii) (iv) (iii) (i)
(a) Listening - Reading - Speaking - Writing (d) (iv) (ii) (iii) (i)
(b) Writing - Reading - Listening - Speaking Ans: (b) The correct matches are–
(c) Listening - Speaking - Reading - Writing (A) 'The love of Kamarupa (iii) W. Franklin
(d) Reading - Listening - Speaking - Writing and Kamalata'.
Ans: (c) The correct order of language acquisition is – (B) 'Ramayana' (1870) (iv) T.H. Griffith
Listening → Speaking → Reading → Writing. (C) 'Upanishad' (1787) (ii) Nathaniel Halhed
Hence option (c). (D) 'Abhijnan Sakuntalam' (i) William Jones
49. Which of the following statements is NOT (1789)
TRUE regarding the poems of Derek Walcott? Hence option (b).
(a) His poem "Goats and Monkeys" has an 53. Which of the following is NOT TRUE of the
epigraph from Shakespeare's Othello New Bolt Report, "The Teaching of English in
(b) In "The Sadhu of Couva" Walcott refers to England"?
Diwali, Hanuman and the Ramayana. (a) It was commissioned in 1919.
(c) Walcott has written a poem entitled "Jean (b) It urged the teaching of the national
Rhys" literature.
(4) In "A Far Cry From Africa" Walcott depicts (c) It proposed the teaching of English Literature
his divided loyalties in the context of the at the university level.
Changuna Uprising (d) It aimed at uniting divided classes after the
Ans: (d) In Walcott’s ‘A Far Cry from Africa’ (1962) war.
he talks about the Mau-Mau uprising not the Changuna Ans: (c) The New Bolt Report commissioned in 1919,
uprising. proposed the teaching of English literature at all levels
Hence option (d). and not just at the university level.
50. In Shakespeare's time who owned the rights to Hence option (c) is not true.
a theatrical script? 54. This revenge tragedy opens with the long
(a) the playwright (s) soliloquy of the protagonist carrying the skull
(b) the patron of the acting company of his poisoned fiancé' and swearing vengeance
(c) the printer for the old Duke who has committed the vicious
(d) the acting company act. Identify the play.
UGC NET/JRF English Paper III, December 2015 200 YCT
(a) The Spanish Tragedy (a) A Doll's House
(b) The Revenger's Tragedy (b) An Enemy of the People
(c) The Duchess of Malfi (c) Hedda Gabler
(d) The Changeling (d) Pillars of Society
Ans: (b) The above description of the opening scene is Ans: (c) Ibsen’s ‘Hedda Gabler’ (1891) deals with the
from Thomas Middleton’s ‘The Revenger’s Tragedy’ perils that await the emancipated woman in a society
(1607). which is not ready to accept her.
Hence option (b). Ibsen’s ‘A Doll’s House’ (1879) raised the issue of
55. What did Anthony Trollope seek to criticize emancipation of woman but Michael Meyer’s argument
through the character Mr. Slope? is that women’s rights is not the play’s theme but rather
(a) Methodism (b) Low Churchmen “the need of every individual to find out the kind of
(c) High Church doctrine (d) Anglicanism person he or she really us”.
So, ‘A Doll’s House’ deals more with emancipation
Ans: (b) Anthony Trollope criticizes the low of an individual than of a woman in a society, as is the
Churchmen through the character of Mr. Slope, in his case with Hedda Gabler.
novel ‘Barchester Towers’ (1857).
Hence option (c).
Hence option (b).
59. "Yet it is the masculine values that prevail",
56. "To refer to symbols as 'Lacanian symbols', to observed a famous writer "Speaking cruelly",
dub self-doubt as 'Lacanian self-doubt', and to she continued, "football and sport are
call reflections in a mirror 'Lacanian 'important', the worship of fashion, the buying
reflections' is not to read the mind from a of clothes 'trivial'."
perspective informed by Lacan. Nor do
Name the author and the text.
parenthetical references to Barthes'
hermeneutic code and Foucault's analysis of (a) Mary Wollstonecraft, A Vindication of the
sexual discourse constitute an interpretation Rights of Woman
necessarily different from that of traditional (b) Audre Lorde Age, Race, Class...
humanist criticism". (c) Virginia Woolf, A Room of One's Own
The author of the passage is objecting to critics (d) Jean Rhys, After Leaving Mr. Mackenzie
who _______. Ans: (c) The above extract is taken from V. Woolf’s ‘A
(a) try to force a parallel between recent critical Room of One’s Own’ (1929).
approaches and traditional humanist Hence option (c).
criticism. 60. According to Coleridge, the "secondary
(b) decoratively apply the names and imagination" "dissolves, diffuses, _______, in
terminology of recent critical theories order to recreate..."
without employing the methodology.
Choose the right word for the blank.
(c) attempt to reduce the study of literature to a
(a) disintegrates (b) dissipates
hunt for coded messages and symbols.
(c) displaces (d) dissociates
(d) stubbornly maintain a traditional notion of
the role of criticism while refusing to Ans: (b) Coleridge in ‘Biographia Literaria’ (1817)
acknowledge new theoretical developments. discussed the elements of what writing should be to be
considered genius. On ‘Imagination’ and ‘Fancy’ he
Ans: (b) The author of the above passage is objecting to
said – "The Imagination then, I consider either as
critics who decoratively apply the names and primary, or secondary. The Primary Imagination I hold
terminology of recent critical theories without
to be the living power and prime agent of all human
employing the methodology.
perception, and as a repetition in the finite mind of the
Hence option (b). eternal act of creation in the infinite I Am. The
57. Peter Ackroyd's first novel, The Great Fire of Secondary Imagination I consider as an echo of the
London, picks up the historical echoes and former, co-existing with the conscious will, yet still as
artfully deploys a Dickens novel as an intertext. identical with the primary in the kind of its agency, and
Identify the source Dickens text. differing only in degree, and in the mode of operation. It
(a) Great Expectations (b) Little Dorrit dissolves, diffuses, dissipates, in order to recreate;...
(c) Martin Chuzzlewit (d) Old Curiosity Shop Fancy on the contrary, has no other counters to play
Ans: (b) Peter Ackroyd’s ‘The Great Fire of London’ with, but fixities and definites".
(1982) deploys Dickens' 'Little Dorrit' (1855-57) as an Hence option (b).
intertext. 61. Beginning 1996, an Indian publisher
Hence option (b). commenced the publication of a series of
58. Which of the following plays by Henrik Ibsen modern Indian novels in English translation.
deals with the perils that await the By 2003, it had published eighty novels of
emancipated woman in a society which is not repute from almost all Indian languages.
ready to accept her? Identify the publisher.
UGC NET/JRF English Paper III, December 2015 201 YCT
(a) Asia Publishing House (d) Philosophical Enquiry into Our Original
(b) Macmillan India Ideas of the Beautiful and the Sublime
(c) Jaico Ans: (b) Burke’s ‘Philosophical Enquiry into the Origin
(d) Arnold Heinemann of our Ideas of the Sublime and the Beautiful’ (1757)
Ans: (b) The publisher described above is Macmillan challenges the Neoclassical tenets of form and reason in
India. aesthetic considerations.
Hence option (b). Hence option (b).
62. William Dunbar's Lament for the makers is 67. Match the following :
about: List-A List-B
(a) kings (b) priests (1) The Grammar- (i) comprehensible
(c) poets (d) peasants Translation Method input
Ans: (c) William Dunbar’s 'Lament for the Makers' is (2) The Direct (ii) strategic use of
about the great dead poets. Method mother tongue
Hence option (c). (3) Total Physical (iii) shuns mother
63. Who among the following protagonists of Response tongue
Thomas Hardy feels his lot as akin to Job's? (4) The Natural (iv) oral input
(a) Clym Yeobright (b) Angel Clare Approach
(c) Jude (d) Troy (1) (2) (3) (4)
Ans: (c) The character Jude in Hardy’s last novel 'Jude (a) (ii) (iii) (iv) (i)
the Obscure' (1896), has similar futile notions of (b) (ii) (iv) (i) (iii)
romantic allusions as Job in the Old Testament was (c) (iv) (ii) (i) (iii)
afflicted despite his innocence. (d) (iii) (i) (ii) (iv)
Hence option (c). Ans: (a) The correct matches are –
64. Edward Brathwaite's poem "Calypso" assumes (1) The Grammar (ii) Strategic use of
that you are familiar with ______. Translation Method mother tongue
(a) the business of Calypso during the Middle (2) The Direct Method (iii) Shuns mother tongue.
Passage (3) Total Physical (iv) Oral input.
(b) the West Indian music in syncopated African response
rhythm (4) The Natural (i) Comprehensive input
(c) the folk ways and mores of Trinidadian Approach
merchants Hence option (a).
(d) the operatic performance of Banjos 68. Which of these works by Indian writers does
Ans: (b) E. Brathwaite’s ‘Calypso’ assumes that we are NOT have the Naxalite Movement as a
familiar with the West Indian music in syncopated background?
African rhythm. (a) Mother of 1084 (b) The Lives of Others
Hence option (b). (c) The Shadow Lines (d) The Lowland
65. Which of the modern plays by a British Ans: (c) Amitav Ghosh’s ‘The Shadow Lines’ (1988)
playwright actually puts Shakespeare as has the communal riots of Dhaka in 1963-4 as the
character on stage? background. It has nothing to do with the naxalite
(a) Edward Bond's Bingo movement.
(b) Harold Pinter's Mountain Language Hence option (c).
(c) Terence Rattigan's Inspector calls 69. "So when the last and dreadful hour,
(d) Joe Orton's Loot
This crumbling pageant shall devour,
Ans: (a) Edward Bond’s 'Bingo: Scenes of Money and
Death' (1973) actually put Shakespeare as an ageing The trumpet shall be heard on high,
character on stage. The dead shall live, the living die,
Hence option (a). And music shall untune the sky"
66. A famous challenge to the Neoclassical tenets of These are the closing lines of a famous poem.
form and reason in aesthetic considerations Identify the poem.
came from Edmund Burke. His work was (a) "II penseroso"
titled: (2) "Song for St. Cecilia's Day"
(a) An Enquiry into the Philosophical Origin of, (3) "The Good-Morrow"
Our Ideas of the sublime and the Beautiful (4) "Song: The Year's at the spring"
(b) Philosophical Enquiry into the Origin of Our
Ideas of the Sublime and the Beautiful Ans: (b) The above extract is from Dryden’s ‘A Song
for St Cecilia’s Day’ (1687).
(c) An Enquiry into the Philosophical Origin of
Our Ideas of the Beautiful and the Sublime Hence option (b).

UGC NET/JRF English Paper III, December 2015 202 YCT


70. This eighteenth-century English poem imitates went they turned it into England; and everybody they
Spenser in stanza form and in allegorical met they turned English. But no place could ever really
narrative: passers - by are lured by an be England, and nobody who did not look exactly like
enchanter with promises of ease, luxury, and them would ever be English, so you can imagine the
aesthetic delight, then consigned to a dungeon destruction of people and land that came from that. The
where they languish in apathy and impotence English hate each other and they hate England, and the
until the Knight of Arts and Industry dissolves reason they are so miserable now is that they have no
the spell. Identify the poem. place else to go and nobody else to feel better than.)
(a) The Vanity of Human Wishes 72. To whom is the passage directly addressed?
(b) The Seasons (a) readers (b) non-antiguans
(c) The Castle of Indolence (c) tourists (d) the English
(d) The Task Ans: (c) In line 2 the word ‘tourist’ is being directly
Ans: (c) The above described poem is James addressed.
Thomson’s ‘The Castle of Indolence’ (1748). Hence option (c).
Hence option (c). 73. The English feel extremely miserable because :
71. Which of the following statements on the (a) Their political supremacy is over
Hogarth press is FALSE? (b) They do not have anyone else to feel superior
(a) The Hogarth press was founded in 1917 by to
Leonard and Virginia Woolf (c) They have been reduced to a state of non-
(b) Its location was their home, called Hogarth entity
House (d) They have no lands to colonise
(c) The press was solely devoted to publishing
international classics in translation Ans: (b) In the last line we see the reason for the
English to feel extremely miserable as they don’t have
(d) The press published translations of Gorky,
Chekhov, Tolstoy, Dostoevsky, Rilke, Svevo anyone to feel superior to.
and others Hence option (b).
Ans: (c) The Hogarth Press was not just publishing 74. Do the British realize that colonizing countries
International (Russian) classics in translation but major was a bad practice, according to the narrator ?
English classics and books on psychoanalysis and the (a) Yes; they do
contributors include works by English authors like (b) No; they don't
Katherine Mansfield, T.S. Eliot, E.M. Forster and Ezra (c) The narrator is rather unsure they do
Pound and some amateur contemporary writers. (d) The narrator is rather unsure they don't
Hence option (c). Ans: (c) From line 7 it is clear that the narrator is
Read the below passage and answer questions 72 to unsure that the British realized their mistake in
75 that follow: colonizing countries.
THE ANTIGUA THAT I knew, the Antigua in Hence option (c).
which I grew up, is not the Antigua you, a tourist, would
75. Which of the following best describes the
see now. That Antigua no longer exists. That Antigua no
longer exists partly for the usual reason, the passing of content of the extract?
time, and partly because the bad-minded people who (a) The speaker fervently desires better
used to rule over it, the English, no longer do so. (But the understanding between the English and the
English have become such a pitiful lot these days, with colonized people in post colonial times
hardly any idea what to do with themselves now that (b) The speaker is interested in nostalgic tours of
they no longer have one quarter of the earth's human emigre antiguans to their childhood home
population bowing and scraping before them. They don't (c) The speaker whose childhood was spent in
seem to know that this empire business was all wrong Antigua reports the great change currently
and they should, at least, be wearing sackcloth and ashes evident in the pungent irony
in token penance of the wrongs committed, the (d) The speaker is making a case for the penance
irrevocableness of their bad deeds, for no natural disaster of the English, the erstwhile rulers of
imaginable could equal the harm they did. Actual death Antigua.
might have been better. And so all this fuss over empire -
what went wrong here, what went there - always makes Ans: (c) The extract clearly has a speaker whose
me quite crazy, for I can say to them what went wrong : childhood was spent in Antigua and who shows his
they should never have left their home, their precious concern about the great change currently evident in the
England, a place they loved so much, a place they had to pungent irony of years of the British regime.
leave but could never forget. And so everywhere they Hence option (c).

UGC NET/JRF English Paper III, December 2015 203 YCT


UGC NET/JRF Exam, December-2015
ENGLISH
Solved Paper-II
Note: This paper contains fifty (5) objective type (a) illustrate the essentially illogical, purposeless
questions of two (2) marks each. All questions are nature of the human condition
compulsory. (b) re-create the working of the subconscious
1. Who, among the following advanced the theory (c) mock the exaggerated dignity and wisdom of
that the mind is a tabula rasa at birth, and modern, self-professed intellectuals
acquires all ideas by experience? (d) reinforce the comic action of farcical plots
(a) John Locke (b) John Wesley Ans: (a) The characters in Samuel Beckett’s 'Waiting
(c) Isaac Watts (d) Denis Diderot for Godot' (1953) illustrate the essentially illogical,
Ans: (a) John Locke (1632-1704) in his ‘Essay purposeless nature of the human condition by using
Concerning Human Understanding’ (1689, BK II, Ch. dislocated repetitious and clichéd speech. It initiated the
vii) said that children are born as ‘blank states’ or a Theatre of the Absurd in modern drama and influenced
‘tabula rasa’ and acquire all ideas by life experiences. Harold Pinter and Tom Stoppard. Hence option (a).
Hence option (a). 7. Which of the following sixteenth-century poets
2. Which of the following authors wrote Studies in was NOT a courtier ?
the History of the Renaissance? (a) George Puttenham (b) Philip Sidney
(a) Walter Pater (b) Oscar Wilde (c) Walter Raleigh (d) Thomas Wyatt
(c) Thomas Carlyle (d) John Ruskin Ans: (a) George Puttenham was a English writer and
Ans: (a) Walter Pater wrote ‘Studies in the History of critic famed for his ‘The Arte of English Poesie’ (1589).
the Renaissance’ in 1873. He was not knighted as the other three.
Hence option (a). Hence option (a).
3. Whom does Harriet Smith finally marry in one 8. Patrick White published two novels in the
1950s giving the eras of pioneering and
of Jane Austen's novels?
exploration in Australian history an epic,
(a) Knightley (b) Darcy ironic and psychological dimension. The novels
(c) Collins (d) Mr. Martin are:
Ans: (d) Harriet Smith a character in Jane Austen’s (A) A Fringe of Leaves (B) The Tree of Man
‘Emma’ (1815), finally marries Mr. Martin. (C) Voss (D) The Aunt's Story
Hence option (d). The right combination according to the code is
4. A poet once referred to an old man as "A (a) (A) and (B) (b) (B) and (C)
tattered coat upon a stick". That is an example (c) (C) and (A) (d) (C) and (D)
of ________ Ans: (b) Patrick White is an Australian novelist whose
(a) Metonymy (b) Sarcasm fourth and fifth novels published in 1955 and 1957 are ‘
(c) Simile (d) Metaphor 'The Tree of Man' and 'Voss' resp.
Ans: (d) The line ‘A tattered coat upon a stick’ presents 'A Fringe of Leaves' – 1976
a comparison of two unlike objects without using ‘as’ 'The Aunt’s story' – 1948
or ‘like’, so it is an example of metaphor. The line is Hence option (b).
taken from the 2nd stanza of T.S. Eliot's 'Sailing to 9. In which of the following works did Bakhtin
Byzantium' published in the 1928 collection– ‘The propose his widely cited concept of the
Tower’. 'Carnivalesque'?
Hence option (d). (a) "Discourse in the novel"
5. Which of these is NOT a pastoral elegy? (b) Dialogic Imagination
(a) Lycidas (b) In Memoriam (c) Rabelais and his world
(c) Thyrsis (d) Adonais (d) "Forms of Time and of the Chronotope in the
Ans: (b) ‘In Memoriam’ (1849) by Alfred Lord Novel"
Tennyson is a lyric in memory of his friend A.H. Ans: (c) Bakhtin proposed his concept of
Hallam who had died in 1833. ‘Carnivalesque’ in his ‘Rabelais and his world’ (1965),
Hence option (b). which refers to a literary mode that subverts and
6. In Beckett's Waiting for Godot the characters liberates the assumptions of the dominant style or
often use dislocated repetitious and cliched atmosphere through humor and chaos (a carnival).
speech primarily to : Hence option (c).
UGC NET/JRF English Paper II, December 2015 204 YCT
10. Match the columns : 14. If you cannot understand an argument and
(Author) (Text) remark, "It's Greek to me", you are
(A) Sebastian Faulks (i) Amsterdam quoting______.
(B) Peter Ackroyd (ii) Changing Places (a) John Milton (b) Samuel Johnson
(C) Ian McEwan (iii) Hawksmoor (c) William Shakespeare (d) John Donne
(D) David Lodge (iv) Birdsong Ans: (c) The remark is taken from William
Codes: Shakespeare’s ‘Julius Caesar’ (1599) and spoken by
(A) (B) (C) (D) Casca when questioned by Cassius.
(a) (i) (ii) (iii) (iv) "Cassius – Did Cicero say anything? Casca – Ay, he
spoke Greek".
(b) (ii) (iii) (i) (iv)
Hence option (c).
(c) (iv) (iii) (i) (ii)
(d) (iii) (iv) (ii) (i) 15. Which of the following works did Walter Scott
compile?
Ans: (c) The correct matches are –
(a) The Lay of the Last Minstrel
(A) Sebastian Faulks – (iv) ‘Birdsong’ (1993)
(b) Marmion
(B) Peter Ackroyd – (iii) ‘Hawksmoor’ (1985)
(c) Ivanhoe
(C) Ian McEwan – (i) ‘Amsterdam’ (1998)
(d) The Minstrelsy of Scottish Border
(D) David Lodge – (ii) ‘Changing places’ (1975)
Ans: (d) 'The Minstrelsy of the Scottish Border' is a
Hence option (c).
collection of Border Ballads complied by Walter Scott
11. In New Criticism, the key term 'tension' is published in 3 volumes in 1802-03. ‘The Lay of the
associated with : Last Minstrel’ is a single composition published in
(a) Cleanth Brooks (b) John Crow Ransom 1805. ‘Marmion’ (1808) is an epic poem about the
(c) Austin Warren (d) Allen Tate Battle of Hodden and ‘Invanhoe’ (1820) is a historical
Ans: (d) The key term ‘tension’ appears in the selected novel.
essays collection ‘The Man of Letters in the Modern Hence option (d).
World’ (1955) by Allen Tate. 16. Which of the following is NOT written by Wole
Hence option (d). Soyinka?
12. While compiling what sort of book did Samuel (a) Home and Exile (b) Kongi's Harvest
Richardson conceive of the idea for his Pamela (c) The Interpreters (d) The Swamp Dwellers
or Virtue Rewarded? Ans: (a) ‘Home and Exile’ (2000) is authored by
(a) an account of the plague in London Chinua Achebe.
(b) an instruction manual for manners Hence option (a).
(c) a book of devotion 17. In the Defence of Poesy Sidney says : "Now as
(d) a book of model letters in geometry the oblique must be known as well
Ans: (d) While compiling the book that was begun as a as right and in arithmetic, the odd as well as
conduct book, 'Pamela or Virtue Rewarded' (1740) the even, so in the actions of our life who seeth
turned into a story of model letters and published as an not the filthiness of evil wanteth a great foil to
epistolary novel. perceive the beauty of virtue". Which of the
Hence option (d). following forms of poesy offers a foil that helps
us perceive the beauty of virtue ?
13. Who among the war Poets gained notoriety in
(a) Pastorals (b) Parody
1917, when disenchanted with the way the war
was being conducted he drafted his letter of (c) Comedy (d) Tragedy
"wilful defiance of the military authority" Ans: (c) Sidney in his ‘Defence of Poesy’ (1579 wr.,
which captured attention in the House of 1595 pub.) presented 'comedy' as a 'foil' to perceive the
Commons, and was forcibly admitted to the beauty of virtue.
war hospital at Craiglockhart, primarily to Hence option (c).
avoid his being court-martialled? 18. John Dryden described a major English poet as
(a) Rupert Brooke (b) Siegfried Sassoon "a rough diamond, and must first be polished
(c) Wilfred Owen (d) Isaac Rosenberg ere he shines......" Identify him :
Ans: (b) In July 1917 Siegfried Sassoon wrote the (a) Geoffrey Chaucer (b) John Gower
famous letter "I am making this statement as an act of (c) George Herbert (d) Robert Herrick
willful defiance of military authority because I believe Ans: (a) In his 'Fables, Ancient and Modern' (1700)
that the war is being deliberately prolonged by those Dryden wrote – “Chaucer, I confess, is a rough
who have the power to end it…." diamond, and must first be polished, ere he shines.”
Hence option (b). Hence option (a).
UGC NET/JRF English Paper II, December 2015 205 YCT
19. In a remarkably proleptic insight, a critic Ans: (b) The famous English architect and stage
wrote the following, anticipating Benedict designer mentioned above is Inigo Jones who also
Anderson's definition of the nation as "an designed the queens house in Greenwich in 1616 and
imagined political community". the Banqueting House in Whitehall in 1619.
"Most novels are in some sense knowable Hence option (b).
communities. It is part of a traditional method-an 23. ______may be defined as any departure from
underlying stance and approach-that a novelist the rules of pronunciation or diction, for the
offers to show people and their relationship in sake of rhyme or metre, or an unjustifiable
essentially knowable and communicable ways". departure from fact.
Name the critic and the reference : (a) Poetic license (b) Poetic justice
(a) Van Wyck Brooks, The writer in America (c) Poetic deviance (d) Poetic diction
(b) Raymond Williams, The country and the City Ans: (a) Poetic license is defined as any departure from
(c) Joseph Wood Krutch, The Modern Temper the rules of pronunciation or diction, for the sake of
(d) T.S Eliot, Notes Towards a Definition of rhyme or metre, or an unjustifiable departure from fact.
Culture Hence option (a).
Ans: (b) The above statement is written by – Raymond 24. That Humanities and the Sciences were in fact
"two cultures" was suggested by_________.
Williams in ‘The Country and the City’ (1973).
(a) Aldous Huxley in his Oxford lectures on
Hence option (b). poetry
20. "Fair is my love, and cruel as she's fair; Her (b) W.H. Auden in his Oxford lectures on poetry
brow-shades frown, although her eyes are (c) F.R. Leavis in his book, The Great Tradition
sunny". The above lines are characterized by : (d) C.P. Snow in his Rede lecture
(a) Circumlocution (b) antithesis Ans: (d) The thesis of C.P. Snow’s 'Rede Lecture' in
(c) anticlimax (d) bathos 1959 was that the intellectual life of the whole of
Ans: (b) The above extract is from Samuel Daniel’s western society was split into the titular two cultures –
poem ‘Sonnets to Delia’ and is an example of anti- the humanities and the sciences.
thesis. It is used in writing or speech when two Hence option (d).
opposites are introduced together for contrasting effect 25. Chaucer satirizes the Monk because the Monk :
as in the above case. (a) is too concerned with courtesy and matters of
Hence option (b). etiquette
21. In his "Epistle to Dr. Arbuthnot" Pope tells us (b) cheats the poor peasants by selling them false
that as a poet he had benefited from "This religious relics
saving counsel, 'keep your piece nine years' - (c) courts favour of wealthy people but spends
which enjoins on writer's patience and great no time with poor people
care before they rush to print. Whose (d) spends too much time hunting and too little
"counsel" is Pope referring to? time on religious duty
(a) Longinus's in On the Sublime Ans: (d) Chaucer satirizes the Monk because he spent
(b) Horace's in Ars Poetica too much time hunting and too little time on religious
duty. Hence option (d).
(c) Quintilian's Institutio Oratoria
26. Divided into three sections this ground-
(d) Aristotle's Poetics
breaking work published in 1953 uses as the
Ans: (b) In his ‘Epistle to Dr. Arbuthnot’ (1735) Pope frame of the spiritual and moral awakening of
is referring to the counsel of Horace who said in ‘Ars a fourteen-year-old during a Saturday night
Poetica’ that an aspiring poet should hold on to his service in a Harlem church. Identify the work.
works for nine years before publishing them. (a) Zora Neale Hurston's Their Eyes are
Hence option (b). Watching God
22. An English architect and stage-designer- (b) James Baldwin's Go Tell it on the Mountain
Beginning 1605, joined Jacobean court to (c) Toni Morrison's Song of Solomon
design masques-contributed significantly to the (d) Richard Wright's Native Son
spectacular theatre which succeeded the Ans: (b) The above story is of James Baldwin’s first
commonwealth after his death-the first major work 'Go Tell it on the Mountain' which was first
designer to use revolving screens to indicate published in 1953. Hence option (b).
scene-changes on the English stage.
27. Chartism, a political movement that took its
Indentify this artist/designer name from the People’s Charter had six points.
(a) Henry Irving (b) Inigo Jones Identify the one point on the following list that
(c) Henry Arthur Jones (d) William Inge was NOT Chartist :
UGC NET/JRF English Paper II, December 2015 206 YCT
(A) universal manhood sufferage 31. Identify the group below which is known as the
(B) equal electoral districts “Sons of Ben”
(C) comprehensive insurance scheme for labour (a) Noel Coward, E.G. Craig, William
(D) vote by secret ballot Macready, Matheson, Lang
(E) payment of MPs (b) John Dryden, the Earl of Rochester, Samuel
(F) no property qualifications for MPs Butler
(G) Annual parliaments (c) William Cartwright, Richard Corbett,
Codes: Thomas Randolph
(a) (E) (b) (G) (d) William Holman Hunt, John E. Millais, D.G.
Rossetti, William Morris
(c) (C) (d) (D)
Ans: (c) The term ‘Sons of Ben’ has been applied to the
Ans: (c) Comprehensive insurance scheme for labour is dramatists who were admittedly influenced by Jonson’s
not among the six points in the People’s Charter, written drama. The playwrights listed under this group are
in 1838, that the chartists believed were necessary to William Cartwright, Richard Corbett, Thomas
reform the electoral system and alleviate the suffering Randolph, R. Brone T. Nabbes, H. Glapthorne, T.
of the working classes. The main composer was Killigrew, Sir W. Davenant, S. Marmion, J. Mayne,
William Lovett of the London Working Men’s P. Hausted, W. Cavendish.
Association.
Hence option (c).
Hence option (c).
32. Christopher Marlowe one of the first major
28. "These beauteous forms, writers to affirm what can be identified as a
Through a long absence, have not been to me clearly homosexual sensibility. Which drama of
As is a landscape to a blind man’s eye…" his deals with it?
("Tintern Abbey Lines") (a) Edward II
Which of the following rhetorical terms best (b) The Jew of Malta
suits these lines? (c) Doctor Faustus
(a) Apostrophe (b) Litotes (d) Dido, Queen of Carthage
(c) Hyperbole (d) Catachresis Ans: (a) The theme of homosexual sensibility is evident
Ans: (*) Note– NTA has dropped this question and in many of Marlowe’s plays but it comes out clearly in
distributed equal marks to all students because there is the realistic historical character of ‘Edward II’ (1593).
no correct answer in the options. Hence option (a).
The above lines from 'Tintern Abbey' (1798) by 33. “When true silence falls we are still left with
William Wordsworth are an example of metaphor, echo but are nearer nakedness. One way of
Metaphor is a comparison between two things that are looking at speech is to say that it is a constant
otherwise unrelated. stratagem to cover nakedness.” Identify the
29. The ‘monster’ in Frankenstein is NOT playwright who underlines the significance of
responsible for the death of: silence thus.
(a) Clerval (b) Justine (a) Samuel Beckett (b) Harold Pinter
(c) Elizabeth (d) Alphonse Frankenstein (c) Luigi Pirandello (d) Joe Orton
Ans: (d) The monster in Mary Shelley’s 'Frankenstein' Ans: (b) The above statement is attributed to Harold
(1818) is directly or indirectly responsible for the deaths Pinter’s ‘Speech on Silence’ (1962).
of Clerval, Justine and Elizabeth but Alphonse dies Hence option (b).
with illness brought on with advancing age and 34. The determining features of syllabic verse is
depression from the news of the monster’s killings. neither ______nor _______but the number of
Hence option (d). syllables in a line.
30. Which of the following plays of William (a) number, numbers (c) stress, quantity
Shakespeare is NOT directly referred to in T.S. (b) sounds, silences (d) gists, piths
Eliot’s The Waste Land? Ans: (c) Syllabic verse is a poetic form having a fixed
(a) Hamlet (b) King Lear number of syllables per line in which neither stress nor
(c) Coriolanus (d) The Tempest quantity play any role. Hence option (c).
Ans: (b) In T.S. Eliot’s ‘The Waste Land’ (1922), 35. In Robert Browning’s dramatic monologue,
‘King Lear’ is not directly referred to by Eliot. The which painter does Andrea del Sarto compare
plays of Shakespeare that are mentioned in the long himself to? What does he find lacking in his
poem are – own work in comparison?
‘Hamlet’ – Line 172 (a) Fra Lippo Lippi - humour
‘Coriolanus’ – Line 417 (b) Raphael - Soul
‘The Tempest’ – Lines 48, 191, 257 (c) Leonardo da Vinci - Verisimilitude
Hence option (b). (d) Botticelli – liveliness
UGC NET/JRF English Paper II, December 2015 207 YCT
Ans: (b) Robert Browning’s ‘Andrea del Sarto’ was Ans: (b) The Ch. 10 of S. Johnson’s ‘Rasselas’ (1759)
published in the poetry collection ‘Men and Women’ in reads as – ‘Imlac’s history continued, A dissertation
1855.Andrea del Sarto compares himself to Raphael and upon poetry.’
finds his soul lacking in his painting. The above Hence option (b).
reference finds mention in the line 197 – “Ay, but the
Soul! He’s Rafael? Rub it out”! Hence option (b). 40. A new series called “New Accents” was
36. In which of the following does Robert Southey launched by Methuen in 1977. The first title to
detail the Indian superstitions as an idolatry to be published in the series was :
be suppressed by a civilizing protestant form of (a) Deconstruction : Theory and Practice
colonialism? (b) Formalism and Marxism
(a) "Thalaba" (c) Structuralism and Semiotics
(b) "The Curse of Kehama" (d) Making and Difference : Feminist Literary
(c) "Pitying the wolves" criticism
(d) "Country Horrors!" Ans: (c) The first title in the New Accents series was
Ans: (b) In his epic poem ‘The Curse of Kehama’ Terence Hawkes' 'Structuralism and Semiotics' (1977).
(1810), Robert Southey details the Indian superstitions Hence option (c).
as an idolatry to be suppressed by a civilizing protestant
form of colonialism. 41. “Humble and rustic life was generally chosen,
Hence option (b). because, in that condition, the essential
passions of the heart find a better soil in which
37. The following is the classic ending of a
celebrated novella in English : they can attain their maturity, are less under
"I kept on creeping just the same, but I looked at restraint, and speak a plainer and more
him over my shoulder. ‘I’ve got out at last", said I, emphatic language.. The language, too’ of these
“in spite of you and Jane. And I’ve pulled off men has been adopted… because such men
most of the papers, so you can’t put me back!” hourly communicate with the best objects from
Now why should that man have fainted? But he which the best part of language is originally
did, and right across my path by the wall, so derived.” Which of the following groups of the
that I had to creep over him every time!” author’s poems in the Lyrical Ballads (1800)
(a) Yellow Woman (Leslie Mormon Silko) contradict this statement in the “Preface to the
(b) The Yellow Wallpaper (Charlotte P. Gilman) Lyrical Ballads” as pointed out by S.T.
(c) Johny Panic and the Bible of Dreams (Sylvia Coleridge?
Plath) (a) "Ode on the Intimations of Immortality",
(d) Where Are You Going, Where Have You "Prelude"
Been? (Joyce C. Oates) (b) "The Tasks", "Seasons".
Ans: (b) The above extract is from Charlotte P. (c) "Michael", "Ruth", "The Brothers"
Gilman’s 'The Yellow Wallpaper' (1892).
(d) "Elegy written in a country churchyard",
Hence option (b).
"Ode on the Popular Superstitions of the
38. Harriet B. Stowe had wanted to write a work Highlands".
based on the life of an Afro-American writer
which was later published as: Ans: (c) In the Ch. XVII of ‘Biographia Literaria”
(a) Uncle Tom’s Cabin (1817), Coleridge said, "Now it is clear to me, that in
(b) Incidents in the Life of a Slave Girl the most interesting of the poems, in which the author is
(c) Cry, The Beloved Country more or less dramatic, as the Brothers, Michael, Ruth,
(d) Narrative of the Life of Frederick Douglass The Mad Mother, and others, the persons introduced are
by no means taken from low or rustic life…."
Ans: (b) Note– UGC answer key marks option (b) as
correct but the correct option is (a) Uncle Tom's Cabin Hence option (c).
(1852). It is an anti-slavery novel by American author 42. A remarkable novelist of the English
Harriet Beecher stowe. Modernist phase who wrote a short book on
The novel had a profound effect on attitudes toward what the novel is (and why it matters)
Afro-Americans and slavery in the U.S. and is said to remarked, “Oh dear, yes- the novel tells a
have "helped lay the groundwork for the Civil War". story”. Identify the novelist :
39. Samuel Johnson’s “Dissertation upon Poetry” (a) Virginia Woolf (b) James Joyce
is part of which of his following works?
(c) E.M. Forster (d) D.H. Lawrence
(a) the final section of his preface to Shakespeare
(b) a chapter of his novel Rasselas Ans: (c) E.M. Forster, in ‘Aspects of the Novel’ (1927),
(c) the epilogue of his Lives of Poets remarked “Oh dear, yes- the novel tells a story”.
(d) one of his Rambler essays Hence option (c).

UGC NET/JRF English Paper II, December 2015 208 YCT


43. What is the name of the angel, who, of those Codes :
who owed allegiance to Satan, dared to protest (A) (B) (C) (D)
against his impious doctrine and left his (a) (i) (iii) (ii) (iv)
company to return to God (Paradise Lost, (b) (iv) (ii) (iii) (i)
Book V)? (c) (iv) (iii) (i) (ii)
(a) Michael (b) Abdiel (d) (iii) (i) (ii) (iv)
(c) Uriel (d) Gabriel Ans: (c) The correct matches are –
Ans: (b) In Milton’s ‘Paradise Lost’ book V, Abdiel (A) “Motiveless – (iv) Iago (Othello,
who owed allegiance to Satan leaves his company to malignity” Act. I Sc. 3)
return to god. Lines 896-7 give the evidence – (B) “Reason in Madness” – (iii) Lear (King Lear,
“So spake the Seraph Abdiel faithful found, Act. IV Sc. 6)
Among the faithless, faithful only hee” (C) “Supp’d full of horrors” – (i) Macbeth
Hence option (b). (Macbeth, Act V,
44. Which of the following is NOT a school Sc. 5)
associated with Romantic period in English (D) “To be, or not to be” – (ii) Hamlet (Hamlet,
literature? Act III, Sc. 1)
(a) The Cockney school Hence option (c).
(b) The Fireside School 48. In Tristram Shandy the narrator’s presentation
(c) The Lake School of his life and opinions is ______
(d) The Satanic School (a) linear (b) digressive
Ans: (b) The Cockney, Lake and Satanic schools are (c) chronological (d) rounded
associated with British Romantic period but the Fireside Ans: (b) In Lawrence Sterne’s ‘Tristram Shandy’
school was associated with 19th century American Poets (1759-1767) the narrator’s presentation of his life and
from New England. opinions is – digressive. The novel’s style is marked by
Hence option (b). digression, double entendre and striking graphic
45. The idea of “new ethnicities” in post-war devices.
Britain was advanced by ________ Hence option (b).
(a) Donald Hall 49. The famous sonnet of John Milton beginning
(b) Stuart Hall “When I consider how my light is spent…..”
(c) Paul Gilroy ends with _______
(d) Hanif Kureishi (a) Before me stares a wolfish eye, Behind me
creeps a groan or sigh
Ans: (b) The idea of 'new ethnicities' in post-war
Britain was advanced by – Stuart Hall in his 1989 essay (b) They also serve who only stand and wait
titled ‘New Ethnicities’. (c) And-which is more-you’ll be a Man, my son!
Hence option (b). (d) And bless him for the sake of him that’s gone
46. Virginia Woolf’s To the Lighthouse begins in a Ans: (b) John Milton’s ‘On his Blindness’ ends with
piece of dialogue: the line – “They also serve who only stand and wait”.
“Yes, of course, if it’s fine tomorrow”, said Hence option (b).
Mrs. Ramsay. “But you’ll have to be up with 50. Her vision was of several caves. She saw herself
lark”, she added. in one, and she was also outside it, watching its
Present among the listeners of her remark entrance, for Aziz to pass in. She failed to
is______ locate him. It was the doubt that had often
visited her, but solid and attractive, like the
(a) her father
hills. “I am not-“ speech was more difficult
(b) her nephew than vision. “I am not quite sure.”
(c) her son The above extract from A Passage to India is
(d) her driver about Adela’s cave experience. Who is
Ans: (c) In V. Woolf’s ‘To the Lighthouse’ (1927) Mrs. questioning Adela?
Ramsay’s son is among the listeners of her remark. (a) Mrs. Moore (b) Mr. Mc Bryde
Hence option (c). (c) Fielding (d) Ronney Heaslop
47. Match the phrase with character : Ans: (b) The above extract is from the Ch. 24 of E.M.
(A) “motiveless malignity” (i) Macbeth Forsters’s ‘A Passage to India’ (1924), in which Adela
(B) “Reason in Madness” (ii) Hamlet is being questioned by Mr. Mc Bryde who is the
(C) “Supp’d full of horrors” (iii) Lear Superintendent of Police in Chandrapore.
(D) “To be, or not to be” (iv) Iago Hence option (b).

UGC NET/JRF English Paper II, December 2015 209 YCT


UGC NET/JRF Exam, July-2016
ENGLISH
Solved Paper-III
Note : This paper contains seventy five (75) the question of Christian Salvation. Allegory is a
objective type questions of two (2) marks each. All literary device according to which the characters and
questions are compulsory. events presented in a literary work have secondary
1. Which of W. M. Thackeray's novel's closing meaning.
sentence is this? "Which of us is happy in this 5. Who among the following translators is notable
world? Which of us has his desire? Or, having as the first translator of Bhagavad Gita into
it, is satisfied?" English?
(a) 'The History of Henry Esmond' (a) Charles Wilkins (b) Nathaniel Halhead
(b) 'Vanity Fair' (c) William Jones (d) Barbara Stoler Miller
(c) 'The Luck of Barry Lyndon' Ans : (a) Charles Wilkins (1749-1836) was an
(d) 'Pendennis' English typographer and the founding member of the
Ans : (b) "Vanity Fair : A Novel without a Hero" is a Asiatic Society. He is notable as the first translator of
novel by Thackeray, first published in 1847-48. It Bhagavad Gita into English and as the creator
satirises society in early 19th century Britain. alongside Panchanan Karmakar, of the first Bengali
2. Why does Lovewit in Ben Jonson's play 'The typeface. In 1788, Wilkins was elected a member of
Alchemist' leave his house, setting the stage for the Royal Society.
his servant Face, alongwith Subtle, a fake 6. In 'Biographia Literaria' S. T. Coleridge defines
alchemist for fleece people? the imagination as the faculty by which
(a) To visit his father who left him long ago. (a) the soul perceives the phenomenal diversity of
(b) To find out new sources of minting money. the universe
(c) Because of an epidemic of plague. (b) the soul perceives the spiritual unity of the
(d) To make a pilgrimage. universe
Ans : (c) "The Alchemist" is a comedy by English (c) the mind acquires images by its associative
playwright Ben Johnson. It was first performed in power
1610. In this, because of an outbreak of plague in (d) the mind separates images by its discriminatory
London, Lovewit temporarily flees to the country power
leaving his house under the sole charge of his butler Ans : (b) "Biographia Literaria" is an autobiography
Jeremy. in discourse by Coleridge, which he published in
3. By the end of the nineteen fifties novelists like 1817, in two volumes. It has 23 chapters. It is also
Stan Barstow, Sid Chaplin, Alan Sillitoe and known as "Biographical Sketches of My Literary Life
David Storey were routinely lumped together as and Opinions" .
representatives of "Kitchen-sink realism." Who 7. Why do the Houyhnhnms have so few words in
in 1954 wrote the article "The Kitchen Sink", their language?
calling attention to the gritty and direct realism? (a) Their wants and passions are fewer than human
(a) Martin Harrison (b) Stan Smith wants and passions, and they need fewer words
(c) David Sylvester (d) Philip Callow (b) They consider language to be morally corrupt
Ans : (c) 'Kitchen-sink realism' (or Kitchen sink and prefer to remain silent
drama) is a term coined to describe a British cultural (c) They find speech difficult because they are
movement that developed in the late 1950s and early horses
1960s, whose protagonists usually could be described (d) They prefer action to words
as "angry young men" who were disillusioned with Ans : (a) Houyhnhnms are a race of intelligent horses
modern society. The term was coined by David described in the last part of Jonathan Swift's satire
Sylvester in his article 'The Kitchen-sink' in 1954. 'Gulliver's Travels'. Gulliver's visit to land of the
4. Which of the following is not an allegorical Houyhnhnms is described in Part IV of his Travels.
character in the play Everyman? 8. Identify the title of A. D. Hope's first published
(a) Kindred (b) Strength book of poems.
(c) Christian (d) Discretion (a) 'Native Companions'
Ans : (c) 'The Somonyng of Everyman', usually (b) 'The Wandering Islands'
referred to simply as "Everyman" is a late 15th century (c) 'A Midsummer Eve's Dream'
morality play. It uses allegorical characters to examine (d) 'The Cave and the Spring'

UGC NET/JRF English Paper III, July 2016 210 YCT


Ans : (b) "The Wandering Islands" (1955) is the first (a) No (b) The
poetry collection by Australian poet A.D. Hope. It (c) Morning ! (d) Jaysus
won the Grace Leven Prize for Poetry in 1955. The Ans : (b) "Finnegan's Wake" is a work of comic
collection consists of 39 poems, most are published in fiction by Irish writer James Joyce published in 1939.
this collection for the first time. The earliest poem in It is significant for its experimental style and
the collection dates from 1943. reputation as one of the most difficult works of fiction
9. Which of the following is an incorrect in the English Language. The entire book is written in
assumption in language teaching? a largely idiosyncratic language.
(a) Learners acquire language by trying to use it in 14. Assertion (A) : In so far as we are taught how to
real situations. read, what we engage are not texts
(b) Learners' first language plays an important role but paradigms.
in learning. Reason (R) : We appropriate meaning from a
(c) Language teaching should have a focus on text according to what we need or
desire, or, in other words,
communicative activities.
according to the critical
(d) Language teaching should give importance to
assumptions or predispositions
writing rather than speech. that we bring to it.
Ans : (d) Language teaching should give importance (a) Both (A) and (R) are true and (R) is the correct
to writing rather than speech. This statement is explanation of (A).
incorrect assumption in language teaching. (b) Both (A) and (R) are true but (R) is not the
10. The Bhasmasura myth is used in R. K. Narayan's correct explanation of (A)
______. (c) (A) is true, but (R) is false
(a) 'The Man-Eater of Malgudi' (d) (A) is false, but (R) is true
(b) 'The Financial Expert' Ans : (a) While reading a text every individual brings
(c) 'The English Teacher' out meaning according to his need and desire. Hence,
(d) 'The World of Nagaraj' Reason is the correct explanation of Assertion.
Ans : (a) "The Man-Eater of Malgudi" is an Indian 15. One of the key terms in Michel Foucault's work
novel, written by R.K. Narayan in 1961. In this, R.K. is discourse. This is best described as
Narayan, has used the myth of Bhasmasur. (a) the power of persuasion in all articulations
11. During the Middle English period, many words (b) the selective language powerful people use
were borrowed from two languages : (c) conceptual frameworks which enable some
I. Celtic II. Latin mode of thought and deny or severely constrain
III. French IV. Old Norse certain others
The right combination according to the code is (d) the ability to suggest transcendental levels of
meaning in an utterance
(a) I and II (b) II and III
(c) II and IV (d) III and IV Ans : (c) Foucauldian discourse analysis is a form of
discourse analysis, focusing on power relationship in
Ans : (b) During Middle English period, words from
society, as expressed through language and practices.
Latin language and Norman French were borrowed.
Middle English refers to the varieties of the English 16. The narrators of 'Oroonoko' are
language spoken after Norman Conquest (1066) until I. a woman
the late 15th century. II. Oroonoko
III. a purported eyewitness of the events described
12. Select the right chronological sequence of the IV. Trefy
date of Bible translations. The right combination according to the code is
(a) King James Version–Tyndale–Revised Standard (a) I and IV (b) I and III
Version – Holman Christian Standard Bible (c) II and III (d) II and IV
(b) Revised Standard Version–King James Version
Ans : (b) "Oroonoko" or the Royal Slave is a short
– Tyndale – Holman Christian Standard Bible work of prose fiction by Aphra Behn (1640-1689),
(c) Tyndale–King James Version–Revised Standard published in 1688. The novel is written in a mixture of
Version – Holman Christian Standard Version first and third person.
(d) Revised Standard Version – Holman Christian
17. Which character of Henrik Ibsen speaks the
Standard Bible–King James Version – Tyndale
following lines: "The life of a normally
Ans : (c) The correct sequence is– constituted idea is generally about seventeen or
• Tyndale Bible – 1526 eighteen years, at the most twenty?"
• King James Version – 1611 (a) Nora in 'A Doll's House'
• Revised Standard Version – 1946/1952 (b) Dr. Thomas Stockman in 'An Enemy of the
• Holman Christian Standard Version – 1999 People'
13. The last word in James Joyce's 'Finnegans (c) John Rosmer in 'Rosmerscholm'
Wake' is (d) Oswald in 'Ghosts'
UGC NET/JRF English Paper III, July 2016 211 YCT
Ans : (b) "An Enemy of the People" is an 1882 play 22. Which work by Franz Kafka is also known as
by Norwegian playwright Henrik Ibsen. Ibsen wrote it 'The Man Who Disappeared'?
in response to the public outcry against his play (a) "The Castle" (b) "Metamorphosis"
"Ghosts", which at that time was considered (c) "In the Penal Colony" (d) "Amerika"
scandalous. Ans : (d) "Amerika", also known as 'The Man Who
18. In literary studies Structuralism promotes Disappeared', is the incomplete first novel of Franz
(a) new interpretations of literary works Kafka (1883-1924), written between 1911 and 1914,
(b) the view that literature is one signifying and published posthumously in 1927.
practice among others 23. Towards the end of Evelyn Waugh's 'A Handful
(c) a systematic account of literary archetypes of Dust' the protagonist Tony Last is trapped in
(d) unstable structures of systems of signification the jungle by the calculating crazy Mr. Todd
Ans : (b) Structuralism is the methodology that who forces him to read and reread the novels of
elements of human culture must be understood in a particular author. Waugh has also written a
terms of their relationship to a larger, overarching short story dealing with Tony's singular
system or structure. It works to uncover the structures experience in the jungle. Who is the novelist
that underlie all the things that humans do, think, referred to and what is the title of the short
perceive and feel. story?
(a) Rudyard Kipling, "Revisiting the Jungle"
19. P.B. Shelley's 'Julian and Maddalo' is a
(b) Joseph Conrad, "Shadows of the Dark Trees"
conversation between Julian and Count (c) Charles Dickens, "The Man Who Liked
Maddalo. Who do these two characters Dickens"
represent? (d) Henry Fielding, "Tom Jones's Journey into the
(a) Julian represents Keats and Count Maddalo, Wild"
Byron
Ans : (c) "A Handful of Dust" is a novel by British
(b) Julian represents Shelley and Count Maddalo,
writer Evelyn Waugh, first published in 1934. The
Byron
hero Tony's singular tale in the jungle was first used
(c) Julian represents Shelley and Count Maddalo,
by Waugh as the subject of an independent short
William Godwin
story, published in 1933 under the title "The Man
(d) Julian represents Mary Shelley and Count
Who Liked Dickens".
Maddalo, William Godwin
24. At the beginning of the Restoration period, there
Ans : (b) "Julian and Maddalo: A Conversation"
was a seismic shift in the social, political and
(1818-19) is a poem in 617 lines of enjambed heroic
religious attitudes of the English. Which of the
couplets by Percy B. Shelley published posthumously
following statements best describes that shift?
in 1824. The poem is inspired by conversations
(a) England shifted from an aristocratic Catholic
Shelley has with Byron in Venice in 1818.
monarchy to a parliamentary democracy
20. What is practical criticism? (b) England shifted from an atheistic oligarchy to a
(a) The close analysis of literary texts in such a deistic squirearchy
way as to bring out their political meaning (c) England shifted from a Republican Puritan
(b) A movement which wished to make literary Commonwealth to an aristocratic Anglican
criticism more relevant monarchy
(c) The close analysis of poems without taking (d) England shifted from a parliamentary
account of any external information democracy to an aristocratic Catholic tyranny
(d) The study of ambiguity Ans : (c) Restoration period begins in 1660, the year
Ans : (c) Practical Criticism is the close analysis of in which King Charles II was restored to the English
poems without taking accounts of any external throne. This was after Puritan period.
information. 25. The Grammar-Translation Method in English
21. Which of the following does not describe some of Language Teaching stresses on
the practices/beliefs of feminist literary (a) Fluency (b) Accuracy
criticism? (c) Appropriateness (d) Listening Skill
(a) Feminist criticism recuperates female writers Ans : (b) The Grammar-Translation Method in
ignored by the canon English Language Teaching stresses on the Accuracy
(b) Feminist literary critics offer a criticism of the of the Language.
construction of gender
26. "[They] then heaved out/away with a will in
(c) Feminist literary critics argue that the
their wood-wreathed ship." This line describing
traditional canon is justified
Beowulf's departure from Geatland, is typical of
(d) Feminist literary critics mostly reject the the poem's form and Old English poetic
essentialising of 'male' and 'female' technique because
Ans : (c) Feminist Literary Criticism doesn't believe I. it features alliteration
that traditional canon is justified. II. it rhymes
UGC NET/JRF English Paper III, July 2016 212 YCT
III. it features onomatopoeia 31. What part of Canada is Alice Munro most
IV. it has four strong stresses famous for depicting?
The right combination according to the code is (a) Vancouver (b) Montreal
(a) I and II (b) II and III (c) Ontario (d) Quebec
(c) I and IV (d) II and IV Ans : (c) Alice Munro (b. 1931) is a Canadian short
Ans : (c) The correct option is (c) I and IV. story writer. Munro's work has been described as
I. It features alliteration. having revolutionized the architecture of short stories,
IV. It has four strong stresses. especially in its tendency to move forward and
27. Identify the poet, translator, publisher and backward in time.
essayist who founded a press in the 1950s called 32. In John Gay's 'Beggar's Opera' what is
Writers' Workshop and provided a publishing Peachum's occupation?
outlet for Indians writing in English. I. Pimp
(a) P. Lal (b) A. K. Mehrotra II. Lawyer
(c) Vinay Dharwadkar (d) A. K. Ramanujan III. Fencer of stolen goods, and master of a gang of
Ans : (a) P. Lal (1929-2010), founded a press in the thieves
1950s for Indian English writing, 'Writers Workshop'. IV. Impeader of less powerful criminals
28. Antagonised by what he considered to be the The right combination according to the code is
provinciality of the Lake Poets, Byron wrote the (a) III & IV (b) II & III
preface to which of his works as a rebuke to (c) I & IV (d) II & IV
Wordsworth's own introduction to "The Ans : (a) "The Beggar's Opera" is a ballad opera in
Thorn"? three acts written in 1728 by John Gay with music
(a) 'The Prisoner of Chillon' arranged by Johann Christoph Pepusch. It is one of the
(b) 'Don Juan' watershed plays in Augustan drama and is the only
(c) 'Childe Harold's Pilgrimage' example of the once thriving genre of satirical ballad
(d) 'The Vision of Judgement' opera.
Ans : (b) "Don Juan" is a satiric poem by Lord Byron, 33. In the opening stanza of "Song of Myself",
in which he presents Juan not as a womanizer but as Whitman begins his spiritual awakening at the
someone easily seduced by women. Byron himself age of ____.
called it an "Epic Satire". (a) 37 (b) 15
29. Which of the following theoretical movements (c) 24 (d) 61
claimed that "the device is the only hero of Ans : (a) "Song of Myself" is a poem by Walt
literature"? Whitman that is included in his work "Leaves of
(a) Russian Formalism (b) New Criticism Grass". The poem was published in 1855. The poem is
(c) Phenomenology (d) Deconstruction written in Whitman's signature free verse style.
Ans : (a) Russian Formalism was an influential school 34. In which of the following poems does Tennyson
of literary criticism in Russia from the 1910s to the describe and condemn the spirit of aestheticism
1930s. It advocated a "scientific" method for studying whose sole religion is the worship of beauty and
poetic language, to the exclusion of traditional of knowledge for their own sake and which
psychological approaches. ignore human responsibility and obligations of
30. In Jean Francois Lyotard's works the term one's fellowmen?
"language games", sometimes also called (a) "The Princess"
"phrase regimens' denotes : (b) "The Lady of Shalott"
I. the multiplicity of communities of meaning (c) "The Palace of Art"
II. the breakdown of communities of meaning (d) "Tithonus"
III. the innumerable and incommensurable separate Ans : (c) "The Palace of Art" is a 1832 (revised 1842)
systems in which meanings are produced poem by Tennyson. In the poem a man constructs a
IV. the singular system in which meanings are palace of art for his soul with any amount of art. The
dispersed and displaced builder's soul at first likes the palace but later tires of
The right combination according to the code is it and asks for a cottage where she can purge her guilt.
(a) I and IV (b) I and III
35. Luigi Pirandello's 'Six Characters in Search of
(c) II and IV (d) II and III
an Author' deliberately blurs the border lines
Ans : (b) Jean-Francois Lyotard was a French between the world of the theatre and the world
philosopher. His interdisciplinary discourse spans
of 'real life' by carefully chiselled dialogues like :
seven topics as knowledge and communication. The
"Don't you feel the ground beneath your feet as
'language games' correctly denotes the statements I
you reflect that this 'you' which you feel today,
and III. all this present reality of yours, is destined to
UGC NET/JRF English Paper III, July 2016 213 YCT
seem a mere illusion to you tomorrow?" Who is Ans : (b) "Where Shall We Go This Summer" (1975)
the speaker? Who is it addressed to? is an intense story of a sensitive young wife born
(a) Stepdaughter to Father between the desire to abandon the boredom and
(b) Father to Stage Manager hypocrisy of her middle class.
(c) Stage Manager to Director "Cry, the Peacock" (1963) is the story of a young girl,
(d) Mother to Director Maya, obsessed by a childhood prophecy of disaster.
Ans : (b) 'Six Characters in Search of an Author' is a 40. From among the following identify the two
1921 Italian play. An absurdist meta theatrical play Indian English authors who received
about the relationship between authors, and their appreciation and encouragement from their
characters and theatre practitioners. British counterparts :
36. In a poem in memory of Major Robert Gregory, I. R. K. Narayan, Graham Greene
Lady Gregory's son, W. B. Yeats mentions an II. Nirad C. Chaudhuri, Evelyn Waugh
Irish writer who had found his inspiration "In a III. Mulk Raj Anand, E.M. Forster
most desolate stony place" that he came IV. Raja Rao, Iris Murdoch
"Towards nightfall upon a race/passionate and The right combination according to the code is
simple like his heart." Who is the writer? (a) I and II (b) II and IV
(a) J.M. Barrie (b) J.M. Synge (c) I and III (d) III and IV
(c) Isaac Bickerstaffe (d) Thomas More
Ans : (c) R. K. Narayan's (1906-2001) mentor and
Ans : (b) One of Yeats's most beautiful and most friend, Graham Greene was instrumental in getting
deeply moving poems is "In Memory of Major Robert publishers for Narayan's first four books.
Gregory" from his 1919 collection, 'The Wild Swans Mulk Raj Anand (1905-2004) was the friend of E.M.
at Coole'. Forster, whom he met while working on T. S. Eliot's
37. Jacques Derrida's work received some criticism magazine Criterion.
from analytical philosophers. Who below was a
41. Match the character with the work :
critic of Derrida?
I. Count Fosco A. 'Villette'
(a) John Searle
II. Margaret B. 'Adam Bede'
(b) Jean-Francois Lyotard
III. Lucy Snowe C. 'The Woman in White'
(c) Emmanuel Levinas
IV. Maggie Tulliver D. 'North and South'
(d) Paul de Man
Codes :
Ans : (a) John Roger Searle (b-1932) is an American I II III IV
philosopher and currently the Professor of Philosophy
(a) C D A B
at the University of California, Berkeley. In the early
(b) D C A B
1970s Searle had a brief exchange with Jacques
(c) C A D B
Derrida regarding speech act-theory. The exchange
(d) C A B D
was characterized by a degree of mutual hostility
between the philosophers, each of whom accused the Ans : (a)
other of having misunderstood his basic points. • Count Fosco is in "The Woman in White" written by
38. Who among the following bought and renovated Wilkie Collins in 1859.
the house of the Anglican poet, George Herbert, • Margaret is in "North and South" novel written by
near Salisbury, England, in 1996? Elizabeth Gaskell in 1855.
(a) Daljit Nagra (b) Vikram Seth • Lucy Snowe is in the novel "Villette" written by
(c) Amitava Kumar (d) Arundhati Roy Charlotte Bronte in 1853.
Ans : (b) Vikram Seth (b. 1952) is an Indian Novelist • Maggie Tulliver is in "Adam Bede" written by
and Poet. Seth's collections of poetry such as George Eliot in 1859.
'Mappings' and 'Beastly Tales' are notable 42. This poet was accidently killed in Burma by a
contributions to the Indian English Language. He had pistol shot in 1944. His posthumously published
bought and renovated the house of the Anglican poet collection of poems, 'Ha! Ha! Among the
George Herbert in 1996. Trumpets', is divided into three sections:
39. Which pair of novels by Anita Desai take as The first section describes a tense, waiting
their subject the suppression and oppression of England and the second the voyage to the East.
Indian women? In the third section he uncomfortably comes to
I. 'Where Shall We Go This Summer'? terms with the alien contours, the harsh light
II. 'The Zigzag Way' and the dry wastes of India as evident in poems
III. 'Cry, the Peacock' like "The Maratta Ghats", "Indian Day" and
IV. 'Baumgartner's Bombay' "Observation Post : Forward Area". Who is the
The right combination according to the code is poet?
(a) I and II (b) I and III (a) Keith Douglas (b) Sidney Keyes
(c) II and III (d) III and IV (c) David Gascoyne (d) Alun Lewis
UGC NET/JRF English Paper III, July 2016 214 YCT
Ans : (d) "Alun Lewis" (1915-1944) was a Welsh 48. In particular William Blake was influenced by
poet. His second book of poems, 'Ha ! Ha ! Among the religious writings of
the Trumpets: Poems in Transit', was published in I. Martin Luther II. Jacob Boehme
1945, and his 'Letters from India' in 1946. III. Emanuel Swedenborg IV. Confucius
43. As Adam and Eve leave Paradise, "hand in The right combination according to the code is
hand with wandring steps and slow" (Book XII, (a) I and IV (b) I and II
(c) II and III (d) III and IV
'Paradise Lost') what is their consolation?
(a) They are comforted by their love for one Ans : (c) Blake was influenced by the ideals and
another ambitions of the French and American revolutions, as
(b) They are comforted by their foreknowledge of well as by such thinkers as Jacob Boehme and
the coming of Christ as Redeemer of mankind Emanuel Swedenborg.
(c) They are comforted by God, who travels before 49. Which British King, having defeated the Viking
them in the form of a pillar of fire invaders, consciously used the English language
(d) They are comforted by the angel, who holds to create a sense of national identity and retain
each of them by the hand political control over independent countries?
(a) Alfred the Great (b) Edward the Elder
Ans : (b) Adam and Eve were comfortable by their
(c) King Arthur (d) Ethelbert of Kent
foreknowledge of the coming of Christ as Redeemer
Ans : (a) 'Alfred the Great' (849-899) was king of
of mankind.
Wessex from 871 to 899. He was the first king of the
44. In 'An Essay of Dramatic Poesy' to whom does West Saxons to style himself "King of the Anglo-
Dryden refer with the phrase "he needed not the Saxons". He is one of only two English monarchs to
spectacles of books to read Nature"? be given the epithet "the Great".
(a) Ben Jonson (b) Ovid
50. In "Politics and the English Language", George
(c) William Shakespeare (d) Geoffrey Chaucer Orwell provides a list of rules to aid in curing
Ans : (c) "An Essay of Dramatic Posey" by John the English language. What is the final rule?
Dryden was published during 1668. It was probably (a) Never use a metaphor, simile or other figure of
written during the plague year of 1666. Dryden takes speech which you are used to seeing in print
the subject that Philip Sidney had set forth in his (b) Never use a long word where a short one will
'Defence of Poesie' (1580) and attempts to justify do
drama as a legitimate form of "poetry". (c) If it is possible to cut a word out, always cut it
45. Emily Dickinson's use of "open form" or "free out
verse" is comparable to her contemporary (d) Break any of these rules sooner than say
American poet, anything outright barbarous
(a) Anne Bradstreet (b) Robert Lowell Ans : (d) "Politics and the English Language" (1946)
(c) Walt Whitman (d) Sylvia Plath is an essay by George Orwell that criticises the "ugly
Ans : (c) Walt Whitman's style introduced the motion and inaccurate" written English of his time and
of free verse, this style becoming his original mark. examines the connection between political
46. In "A Letter of the Authors" Edmund Spenser orthodoxies and the debasement of language.
writes that two characters in 'Faerie Queene' 51. In his 'Defence of Poesy' what is the "best and
represent Queen Elizabeth. Who are they? most accomplished kind of poetry" in Sidney's
I. Britomart II. Cynthia estimation?
III. Belphoebe IV. The Faerie Queene (a) Heroical, or epic poetry
The right combination according to the code is (b) Lyric poetry
(a) III and IV (b) I and IV (c) Pastoral poetry
(c) I and III (d) II and III (d) Elegiac poetry
Ans : (a) "The Faerie Queene" is an incomplete Ans : (a) "An Apology for Poetry" (or 'The Defence
of Poesy') was written by Sidney in approx 1579, and
English epic poem. The first half was published in
first published in 1595, after his death. It is generally
1590 and second in 1596. In Spenser's "Letter of the
believed that he was at least partly motivated by
Authors", he states that the entire epic poem is
Stephen Gosson, a former playwright, who dedicated
cloudily enwrapped in Allegorical devices.
his attack on the English stage, "The School of
47. Who among the following African novelists was Abuse" to Sidney in 1579.
a student of philosophy and literature in India? 52. Which writer of the Romantic period makes the
(a) Nuruddin Farah (b) Ben Okri following comment : "The poet is far from
(c) Helon Habila (d) Benjamin Kwakye dealing only with these subtle and analogical
Ans : (a) Nuruddin Farah is a Somali novelist, born in truths. Truth of every kind belongs to him,
1945. Since leaving Somalia in the 1970s he has lived provided it can bud into any kind of beauty, or
and taught in numerous countries, including the U.S., is capable of being illustrated and impressed by
England, Germany, Italy, India etc. poetic faculty"?
UGC NET/JRF English Paper III, July 2016 215 YCT
(a) Wordsworth in Preface to the 'Lyrical Ballads' Ans : (c) "Crime and Punishment" is a novel by
(b) William Hazlitt in "On the Feeling of Russian author Fyodor Dostoevsky. It was first
Immortality in Youth" published in twelve month installment during 1866.
(c) Leigh Hunt in 'What is Poetry'? The novel focuses on the mental anguish and moral
(d) Keats in one of his letters to his brother dilemmas of Rodion Raskolnikov.
Ans : (c) James Henry Leigh Hunt (1784-1859) was 57. What three Germanic tribes invaded Britons in
an English critic, essayist, poet and writer. His essay the fifth century AD, bringing with them the
"What is Poetry?" contains numerous example from roots of modern English?
several ages and languages of what in his opinion (a) The Danes, Saxons and Celts
qualified as fine poetry. His essay is inscribed in the (b) The Celts, Jutes and Saxons
times of theoretical writing on poetry that inspired (c) The Saxons, Danes and Angles
Shelley, Coleridge and Wordsworth to write their well (d) The Jutes, Angles and Saxons
known treatise. Ans : (d) The history of the English Language really
53. In his poem "Whispers of Immortality" T.S. started with the arrival of three Germanic tribes who
Eliot says that a dramatist "was much possessed invaded Britain during the 5th century A.D., the
by death/ And saw the skull beneath the skin" Angles, the Saxons and the Jutes, who crossed the
and a poet "knew the anguish of the marrow / North Sea to reach there.
The ague of the skeleton." Who are the 58. Which of the following is not a part of the series
dramatist and the poet referred to by Eliot? of poems called 'Jejuri', written by Arun
(a) Christopher Marlowe and Andrew Marvell Kolatkar?
(b) John Webster and John Donne (a) "Yeshwant Rao" (b) "Chaitanya"
(c) Seneca and Homer (c) "The Priest" (d) "An Old Man"
(d) Thomas Kyd and Henry Vaughan Ans : (d) "Jejuri" is the name of a series of poems
Ans : (b) "Whispers of Immortality" is a poem by written in 1976 and won the Commonwealth Prize in
T.S. Eliot, written between 1915 and 1918 and was 1977. The poem is made up of a secular visitor to the
published in June 1919. Eliot paid homage to his great ruins of Jejuri, a pilgrimage site in Maharashtra.
Jacobean masters, Webster and Donne in particular. 59. Bertolt Brecht's concept of alienation was a
54. Functional Communicative Approach in English rejection of the idea that realism was the only
Language Teaching is in opposition to mode of art a critique of capitalist society should
(a) Structural Approach produce. Alienation is best described as
(b) Comprehensive Approach (a) making the audience feel that they do not
(c) Translation and Grammar Method belong
(d) Functional Approach (b) distancing artistic conventions to prevent an
Ans : (a) Functional Communication Approach in emotional catharsis
English Teaching is in opposition to Structural (c) scripting unnatural behaviour on stage
Approach. (d) a rejection of capitalism or the market
55. According to Julia Kristeva, it is the eruption of Ans : (b) Bertolt Brecht (1898-1956) was a German
the ______ within the _______ that provides the poet, playwright and theatre director of the 20th
creative and innovative impulse of modern century. He wanted his audience to adopt a critical
poetic language. perspective in order to recognize social injustice and
(a) individual, tradition (b) specific, generic exploitation and to be moved to go forth from the
(c) semiotic, symbolic (d) particular, general theatre and effect change in the world outside.
Ans : (c) Julia Kristeva (1941) is a Bulgarian – French 60. Ngugi wa Thiong'o changed the medium of his
philosopher, literary critic, psychoanalyst, feminist, writing from English to _______
who has lived in France, since the mid 1960s. She is (a) Swahili (b) Yoruba
now a Professor at the University of Paris. (c) Xhosa (d) Gikuyu
56. In 'Crime and Punishment' which character Ans : (d) Ngugi wa Thiong'o (b. 1938) is a Kenyan
speaks the following words. Who/what are they writer, formerly working in English and now working
addressed to? in Gikuyu. He is founder and editor of the Gikuyu –
"I waited for you impatiently ..... all this blasted Language Journal "Mutiiri."
psychology is a double-edged weapon." 61. Which of the following ancient critics does
(a) Svidrigailov to the pistol with which he shoots Alexander Pope commend as exemplary in
himself 'Essay on Criticism'?
(b) Katherine Ivanovna to Marmeladov (a) Aristotle, Quintilian, Dryden, Dionysius, Horace
(c) Porfiry Petrovich to Raskolnikov (b) Aristotle, Longinus, Quintilian, Durfey, Dryden
(d) Raskolnikov to the Bible he finds in the prison (c) Aristotle, Horace, Dionysius, Quintilian, Longinus
cell in Siberia (d) Aristotle, Horace, Durfey, Quintilian, Longinus
UGC NET/JRF English Paper III, July 2016 216 YCT
Ans : (c) "An Essay on Criticism" is one of the first (d) They incorporate elements of popular
major poems written by the English writer Alexander contemporary culture
Pope (1688-1744). It is written in a type of rhyming Ans : (b) The plots are strikingly tight-knit is the
verse called heroic couplets. The poem first appeared incorrect statement.
in 1711. 66. Published in 1604, the first monolingual English
62. Which of the following poems by Philip Larkin Dictionary was
is best described as a self-elegy, anticipating the (a) Nathaniel Bailey's Universal Etymological
poet's death? Dictionary of the English Language
(a) "The Old Fools" (b) "Aubade" (b) Samuel Johnson's Dictionary of the English
(c) "Ambulances" (d) "Faith Healing" Language
Ans : (b) 'Aubade' was first published in 1977. An (c) Robert Cawdrey's Table Alphabetical
aubade is a morning love poem. It has been defined as (d) Thomas Blount's Glossographia
"a song or instrument composition concerning, Ans : (c) Robert Cawdrey (1538-1604) produced one
accompanying or evoking day break. But, Larkin's of the first dictionaries of the English Language, "The
poem seems to leave out the love aspect of the form Table Alphabetical" in 1604.
and focuses on death instead.
67. Which of the following statements best describe
63. In John Bunyan's 'Pilgrims Progress' what is
the narrative perspective employed in Thomas
the first obstacle encountered by Christian on
More's Utopia?
his progress?
I. First-person narration by Raphael Hythloday
(a) The Slough of Despond
II. Third-person narration by a narrator named
(b) Vanity Fair
(c) The River of Death Thomas More
(d) The Swamp of Despair III. First-person narration by narrator named
Thomas More
Ans : (a) John Bunyan's 'Pilgrims Progress' is a 1678
IV. Third-person narration by Raphael Hythloday
Christian allegory. Bunyan began his work while in
the Bedfordshire county prison for violations of the The right combination according to the code is
Conventicle Act. The first obstacle faced by Christian (a) I and III (b) II and IV
in the poem is the Slough of Despond. (c) II and III (d) I and II
64. Identify the correct chronological sequence of Ans : (a) "Utopia" is a work of fiction published in
publication of the four parts of 'The Four 1516 in Latin. The book is a frame narrative primarily
Quartets'. depicting a fictional island society and its religious,
(a) Burnt Norton – The Dry Salvages – East Coker social and political custom.
– Little Gidding 68. In the opening pages of one of Thomas Mann's
(b) Burnt Norton – Little Gidding – The Dry novels we can see space itself becoming a form
Salvages – East Coker of time : "Space, like time, engenders
(c) Burnt Norton – East Coker – The Dry Salvages forgetfulness but it does so by setting us bodily
– Little Gidding free from our surroundings and giving us back
(d) Little Gidding – Burnt Norton – The Dry our primitive unattached state." Which is the
Salvages – East Coker novel?
Ans : (c) (a) 'Doctor Faustus'
• "Burnt Norton" is the first poem of Eliot's 'Four (b) 'Death in Venice'
Quartets'. He created it while working on his play (c) 'The Confessions of Felix Krull'
'Murder in the Cathedral' and it was first published (d) 'The Magic Mountain'
in his Collected Poems 1909-1935. Ans : (d) "The Magic Mountain" is a novel by
• "East Coker" is the second poem, published in Thomas Mann, first published in Nov. 1924. It is
1940. widely considered to be one of the most influential
• "The Dry Salvages" is the third poem, published in works of 20th century German Literature.
1941. 69. Match the lines with the titles of the poems :
• "Little Gidding" is the fourth and final poem of I. The boa-constrictor's A. "Thrushes"
Eliot, published in 1942. coil/Is a fossil
65. Which of the following is not true of the novels II. My manners are tearing B. "The Jaguar"
of Charles Dickens? off heads/The allotment
(a) They deal with the problems of the discontents of death
of an urban civilization III. More coiled steel than C. "Relic"
(b) The plots are strikingly tight-knit living
(c) They share a sense of fun and determining IV. Time in the sea eats its D. "Hawk
optimism tail Roosting"
UGC NET/JRF English Paper III, July 2016 217 YCT
Codes : you figure in horror films
I II III IV in the mandatory moment
(a) A D A C between the flash of lightning
(b) B D A C and the appearance of the ghost.
(c) C D B A The light is darkish blue and you see
(d) D B C A yourself in the iris of the burning
Ans : (b) All the poems are of Ted Hughes. He was a eye. The horror is in the seeing.
children's writer too. He served as Poet Laureate from What you see is altered by the act
1984 until his death. of seeing. The mystery does not stop
70. Which one of Joseph Conrad's novels expresses there. The seer is in turn altered
the contrast between the solidarity of shipboard by what he sees. Having known this,
life and the profound underlying loneliness of stray cats jump from roof to roof.
existence thus : "loneliness impenetrable and They monitor the world from treetops
transparent, elusive and everlasting .....that and hold their weekly meetings
surrounds, envelops, clothes every human soul in the graveyard, like wandering mendicants.
from the cradle to the grave, and perhaps
beyond"? And when they walk out of the mirror
(a) 'The Heart of Darkness' of the sun and cross the crowded road
(b) 'The Nigger of the Narcissus' in a flash, for a shining moment,
(c) 'Lord Jim' they lurk in the light like a giant shadow
(d) 'Nostromo' of doubt. Ill-omens to those who cannot
Ans : (*) Note– NTA has found the characteristics see beyond what they see.
asked in the question in all the given novels of Joseph 72. The poem constructs its account of stray cats by
Conrad. Therefore, all the options are correct. way of a contrast with
According to NTA 'The Heart of Darkness', 'The (a) wild cats (b) ominous cats
Nigger of the Narcissus', 'Lord Jim' and 'Nostroneo', (c) domestic cats (d) mysterious cats
all novels express the contrast between solidarity of Ans : (c) According to the passage the answer is
shipboard life and the profound underlying loneliness 'domestic cats'.
of existence. 73. In the overall context, what do "furnished
71. John Dryden's two philosophico-religious poems interiors, morning walks/the cake and the
are cutlery" represent?
I. 'Absalom and Achitophel' (a) Ordinary life
II. 'A Layman's Faith' (b) "Domestic fictions"
III. 'Annus Mirabilis' (c) "A giant shadow of doubt"
IV. 'The Hind and the Panther' (d) Creaturely comforts
The right combination according to the code is Ans : (b) According to the passage; the answer is
(a) I and II (b) III and I "Domestic fictions".
(c) II and III (d) II and IV 74. The last two lines suggest that cats crossing the
Ans : (d) crowded road
• "Religio Laici" or "A Layman's Faith" published in (a) is an unexceptionable superstition
1682. (b) is not necessarily the ill-omen it is held out to be
• 'The Hind and the Panther: A Poem' published in (c) is an example of human obsession
1687. (d) is indicative of the homelessness of stray cats
Read the following poem and answer the Ans : (b) According to the passage, that cats crossing
questions, 72 to 75 : the crowded road is not necessarily the ill-omen it is
Stray Cats held out to be.
They are not exactly homeless. 75. From among the following select two words that
They are dissidents who have lost their faith help accentuate the enigmatic character of stray
in furnished interiors, morning walks, cats :
the cake and the cutlery. I. Doubt II. Mandatory
III. Faith IV. Mystery
When you have nine lives to live The right combination according to the code is
you learn to take things in your stride. (a) I and II (b) I and IV
You learn to stretch your body (c) II and IV (d) III and IV
at full length and yawn at domestic Ans : (b) According to the passage the correct answer
fictions. And for this reason is option (b) doubt and mystery.
UGC NET/JRF English Paper III, July 2016 218 YCT
UGC NET/JRF Exam, July-2016
ENGLISH
Solved Paper-II
Note : This paper contains fifty (50) objective (a) Grildrig (b) Glumdalclitch
type questions, each question carrying two (2) marks. (c) Splacknuck (d) Mannikin
Attempt all question. Ans : (a) In part 2 of "Gulliver's Travels" Lemuel
1. Which British University figures in William Gulliver is named Grildrig in Brobdingnag, by his
Wordsworth's 'Prelude'? caretaker whom he called Glumdalclitch, or Little
(a) Durham (b) Glasgow Nurse. Grildrig means "mannequin" in Brobdingnag.
(c) Cambridge (d) Oxford 6. Who among the following was born in India?
Ans : (c) "The Prelude" is an autobiographical poem (a) Paul Scott (b) Lawrence Durrell
in blank verse. Intended, as the introduction to (c) E.M. Forster (d) V.S. Naipaul
"Recluse", "Prelude" is a personal poem. Wordsworth Ans : (b) Lawrence George Durrell (1912-1990) was
began this in 1798 at the age of 28 and continued to a British novelist, poet, dramatist and travel writer.
work on it through his life. He never gave it a title; he Durrell was born in Jalandhar, British India and his
called it "The poem on the growth of my own mind". first school was St. Joseph's College, North Point,
It was published three months after Wordsworth's Darjeeling. His most famous work is the tetralogy
death in 1850 and his final name given to it by his "The Alexandria Quartet", particularly the first of the
widow Mary. In "Prelude" Book Sixth is named
quartet's four novels, 'Justine' (1957).
"Cambridge and The Alps" in which he talked about
Cambridge University. 7. What metaphor does Edmund Spenser employ
('Faerie Queene', Book 1, Canto 12) to frame his
2. Who is the author of 'A Woman Killed with
tale and to describe the relationship between the
Kindness'?
tale and its readers?
(a) John Marston (b) Thomas Middleton
(a) That of a caravan of lost souls, traversing a
(c) John Fletcher (d) Thomas Heywood
desert
Ans : (d) "A Woman Killed with Kindness" is an (b) That of a stagecoach, which picks up diverse
early 17th century stage play, a tragedy, acted in 1603 passengers along the way
and first published in 1607, written by Thomas (c) That of a ship filled with jolly mariners
Heywood. The plot of Heywood's play derives from
(d) That of a riderless horse, following his own
an Italian novel by Illicini, which was translated into
direction
English and published in "The Palace of Pleasure" by
William Painter (1566). Ans : (c) Lines from "Faerie Queene", Book 1, Canto
12 are:
3. In William Congreve's 'The Way of the World'
"Now strike your sailes yee Jolly Mariners
identify the speaker of the line : "One's cruelty
For we be come unto a quiet rode,"
is one's power, and when one parts with one's
cruelty, one parts with one's power." 8. Who among the following is not associated with
(a) Mirabell (b) Witwoud Russian formalism?
(c) Millamant (d) Mincing (a) Roman Jacobson
(b) George Poulet
Ans : (c) "The Way of the World" is widely regarded
(c) Boris Eichenbaum
as one of the best Restoration comedies, premiered in
(d) Victor Shklovsky
early March 1700, in the theatre in Lincoln's Inn fields
in London. The above dialogue is of Millamant in Act Ans : (b) George Poulet (1903-1950) is a pen name of
II, Scene V. He was speaking to Mirabell. Eric Arthur Blair. He was an English novelist,
essayist, journalist and critic. He was associated with
4. T.S. Eliot found spiritual support in
(a) Christianity (b) Hinduism Geneva School of Structural Linguistics.
(c) Buddhism (d) Judaism 9. Which character in Dickens keeps on hoping
Ans : (a) Note–The answer should be Hinduism but that "something will turn up"?
UGC has approved Christianity as its answer. (a) Barkis (b) Micawber
Reason–T. S. Eliot used 'Datta Dayadhvam Damyata' (c) Uriah Heep (d) Miss Havisham
as the concluding lines of "Waste Land", which means Ans : (b) Wilkins Micawber is a fictional character
'Shantih, Shantih, Shantih'. from Dicken's 1850 novel, David Copperfield. He was
5. By what name is Gulliver known in modeled on Dickens' father who kept on hoping that
Brobdingnag? "something will turn up".

UGC NET/JRF English Paper II, July 2016 219 YCT


10. What is the name of the boat that rescues 15. Who narrates Heart of Darkness?
Ishmael in Herman Melville's 'Moby Dick'? (a) Marlow
(a) Pequod (b) Rachel (b) Director of Companies
(c) Hagar (d) Sphinx (c) Kurtz
Ans : (b) "Moby Dick" or 'The Whale' is a novel by (d) An unnamed narrator
American writer Herman Melville, published in 1851, Ans : (d) "Heart of Darkness" (1899) is a novella by
during the period of the American Renaissance. Sailor Polish-British novelist Joseph Conrad, about a voyage
Ishmael tells the story of the obsessive quest of Ahab, up to Congo River into the Congo Free State, in the
heart of Africa, by the story's narrator Marlow.
captain of the whaler 'The Pequod', for revenge on
Moby Dick, the great white whale. Ishmael is rescued 16. 'The Mistakes of a Night' is the subtitle of
(a) The Conscious Lovers
by the boat Rachel.
(b) The Good Natur'd Man
11. 'Northanger Abbey' is a parody of the (c) She Stoops to Conquer
_____romance. (d) The Rivals
(a) Oriental (b) French Ans : (c) "She Stoops to Conquer" is a comedy by
(c) Gothic (d) Popular Anglo-Irish author Oliver Goldsmith that was first
Ans : (c) "Northanger Abbey" was the first of Jane performed in London in 1773. Initially the play was
Austen's novels to be completed for publication, but titled "Mistakes of a Night" and indeed, the events
published in 1817 after her novels, "Sense and within the play take place in one long night.
Sensibility" (1811) and "Pride and Prejudice" (1813). 17. Identify the first novel written by Patrick White :
It was written around 1798-99. (a) The Living and the Dead
12. Who among the following authors were greatly (b) The Tree of Man
influenced by Thomas Carlyle's writings? (c) Happy Valley
I. Charles Dickens (c) The Aunt's Story
II. Elizabeth Gaskell Ans : (c) "Happy Valley" is a 1939 novel by
III. Emily Bronte Australian author Patrick White. It won the 1941
IV. Oscar Wilde Australian Literature Society Gold Medal. Patrick
The right combination according to the code is White had dedicated the novel to artist Roy De
Maistre.
(a) I and II (b) II and III
(c) I and IV (d) I and III 18. In 'King Lear' for what reason does Kent
assume a disguise?
Ans : (a) Thomas Carlyle (1795-1881) was a Scottish (a) To continue to serve Lear, though Lear has
philosopher, satirical writer and teacher. He claims banished him
that "History is nothing but the biography of the Great (b) To spy on Edmund
Man". His 1837 book "The French Revolution: A (c) To antagonize Goneril and Regan
History" was the inspiration for Dicken's 1859 novel (d) To revenge upon Lear for banishing him
"A Tale of Two Cities". Carlyle's "Sartor Resartus" Ans : (a) Kent assumed a disguise, to continue to
(1836) is considered one of the finest works of the 19th serve Lear, though Lear has banished him.
century. 19. What is feminine rhyme?
13. Which of the following is another term to (a) A rhyme on two syllables in which the last
describe "art for art's sake"? syllable is unstressed
(a) Aestheticism (b) Didacticism (b) A rhyme on two syllables
(c) Realism (d) Neo-realism (c) A rhyme on three syllables
Ans : (a) "Art for art's sake" is the usual English (d) A poem in which every third syllable rhymes
rendering of a French slogan from the early 19th Ans : (a) A rhyme between stressed syllables
century and expresses a philosophy that the intrinsic followed by one or more unstressed syllables.
value of art and the only "true" art, is divorced from Examples– Stocking/Shocking and glamorous
any didactic, moral or utilitarian function. /amorous
20. Identify two of the following written by
14. The statement that there are "none so credulous
Christopher Fry :
as infidels" is an illustration of
I. French Without Tears
(a) Oxymoron (b) Antithesis II. The Lady's Not for Burning
(c) Paradox (d) Metonymy III. Venus Observed
Ans : (c) A paradox is a statement that is self IV. The Deep Blue Sea
contradictory because it often contains two statements The right combination according to the code is
that are both true, but in general, cannot both be true (a) II and III (b) I and III
at the same time. (c) II and IV (d) I and IV
UGC NET/JRF English Paper II, July 2016 220 YCT
Ans : (a) Ans : (c) "Possession" is a fictional story of the
• "French Without Tears" is a comic play written by relationship between two Victorian poets, based on
Terence Rattigan in 1936. Browning (II) and Christina Rossetti (III).
• "The Deep Blue Sea" is a play written by Terence 26. 'The Dark Lady of the Sonnets' is a short
Rattigan in 1952. comedy by
• "The Lady's Not for Burning" is a play written by (a) Bernard Shaw (b) W.B. Yeats
Christopher Fry in 1948. (c) J.M. Synge (d) John Osborne
• "Venus Observed" is a play written by Christopher Ans : (a) "The Dark Lady of the Sonnets" (1910) is a
Fry in 1950. short comedy composed by G. B. Shaw, in which
21. In "Tradition and Individual Talent", according William Shakespeare, intending to meet the "Dark
to T.S. Eliot, the term "Traditional" usually Lady", accidently encounters Queen Elizabeth I and
means attempts to persuade her to create the national theatre.
(a) something positive 27. John Milton's description of gold as a "precious
(b) something negative bane" (Paradise Lost, Book II) is best described as
(c) something historical (a) a dactyl (b) an oxymoron
(d) something old (c) enjambment (d) zeugma
Ans : (b) According to Eliot "Traditional" usually Ans : (b) Oxymoron is a figure of speech in which
means something negative. He has suggested this in apparently contradictory terms appear in conjunction.
one of his essays, "Tradition and Individual Talent". Here precious means valuable and bane means
distress or annoyance.
22. Which of the following is a Cavalier poet?
(a) George Herbert (b) John Donne 28. There is a play on the name of Machiavelli in the
(c) Robert Herrick (d) Andrew Marvell prologue to Christopher Marlowe's
(a) Doctor Faustus (b) The Jew of Malta
Ans : (c) Cavalier Poet is a broad description of a (c) Tamburlaine, the Great (d) Edward II
school of English poets of the 17th century, who came
Ans : (b) Machiavelli is used in the starting of the
from the classes that supported King Charles I during
play "The Jew of Malta", written in 1589 or 1590. The
the English Civil War.
plot is an original story of religious conflict, intrigue
Poets grouped themselves with the King and his
and revenge.
services, thus becoming Cavalier Poets.
29. Shakespeare famously neglects to observe
23. Which of the following is not Jacques Derrida's Aristotle's rules concerning the three dramatic
work? unities, and Samuel Johnson undertakes to
(a) Of Spirit : Heidegger and the Question defend Shakespeare from these criticisms in his
(b) The Transcendence of the Ego 'Preface to Shakespeare'. Which of the
(c) Of Grammatology Aristotelian dramatic unities does Johnson
(d) The Work of Mourning believe Shakespeare to observe most
Ans : (b) "The Transcendence of the Ego" is a successfully?
philosophical and psychological essay written by (a) Time
Jean-Paul-Sartre in 1934 and published in 1936. (b) Place
24. In 'Paradise Lost' which character narrates the (c) Action
story of the making of Eve from a rib in Adam's (d) Johnson does not feel that Aristotelian
side? dramatic unities are important
(a) Adam (b) Eve Ans : (c) In "Preface to Shakespeare" Dr Johnson
(c) Raphael (d) God wrote – "In his other works he has well enough
Ans : (a) Adam himself tells the story of making of preserved the unity of action".
Eve from his left side, lines 460-480 in Book VIII of 30. Who among the following was praised and
'Paradise Lost'. patronized as a "Ploughman Poet"?
25. A.S. Byatt's 'Possession' attempts the imitation (a) John Clare (b) George Crabbe
of the work of two Victorian poets, loosely based on (c) Robert Burns (d) Walter Scott
I. Alfred Tennyson Ans : (c) Robert Burns (1759-1796) also known as
II. Robert Browning Rabbie Burns, the Bard of Ayrshire was a Scottish
III. Christina Rossetti poet and lyricist. He is regarded as a pioneer of the
IV. William Morris Romantic Movement, and after his death he became a
The right combination according to the code is great source of inspiration to the founders of both
(a) I and II (b) II and IV liberalism and socialism, and a cultural icon in
(c) II and III (d) III and IV Scotland.

UGC NET/JRF English Paper II, July 2016 221 YCT


31. Which novel of Doris Lessing ends with a 35. 'Childe Harold's Pilgrimage' is a
projection forward in time after a devastating (a) Religious allegory (b) fairy tale
atomic war? (c) long poem (d) Utopian novel
(a) The Grass is Singing Ans : (c) "Childe Harold's Pilgrimage" is a lengthy
(b) The Golden Notebook narrative poem in four parts written by Lord Byron. It
(c) The Four-Gated City was published between 1812 and 1818 and is
(d) A Proper Marriage dedicated to Lanthe.
Ans : (c) 'The Four-Gated City' was published in 36. In Thomas More's 'Utopia' which of the
1969, written by the British Nobel Prize winner Doris following leisure pastimes is not a favourite
Lessing in five volumes of semi-autobiographical among Utopians?
series "The Children of Violence". (a) Music (b) Public lectures
32. Name the dominant meter of the following (c) Conversation (d) Dicing and cards
quatrain : Ans : (d) Utopia is a work of fiction and political
The curfew tolls and the knell of parting day, philosophy by Thomas More (1478-1535) published
The lowing herd winds slowly o'er the lea, in 1516 in Latin. The book depicted a fictional island
The plowman homeward plods his weary way,
society and its religious, social and political customs.
And leaves the world to darkness and to me.
37. Which of the following statements does not
(a) Iambic Hexameter
describe Michel Foucault's position?
(b) Trochaic Pentameter
(a) In Foucault's work sexuality is literally written
(c) Iambic Pentameter
(d) Terza Rima on the body
(b) Power operates through discourse
Ans : (c) These lines are from Thomas Gray's 'Elegy
(c) There is connection between power and
Written in a Country Churchyard'– which is an ideal
knowledge
example of heroic quatrain (four line stanza), written
(d) Where there is power, it is possible to find
in Iambic Pentameter with rhyme scheme abab.
resistance
33. Which two novels of Buchi Emecheta provide a
Ans : (a) In Michel Foucault's work sexuality is
fictionalized portrait of poor, young Nigerian
women struggling to bring up their children in literally written on the body.
London? 38. In which year did the Great Exhibition take place
I. The Slave Girl (a) 1851 (b) 1857
II. The Joys of Motherhood (c) 1861 (d) 1871
III. Second Class Citizen Ans : (a) "Great Exhibition", also referred as the
IV. In the Ditch "Crystal Palace Exhibition", took place in Hyde Park,
The right combination according to the code is London, from 1 May to 11 October 1851.
(a) I and II (b) II and III 39. When Fidessa says, "O but I fear the fickle
(c) III and IV (d) I and IV freakes ....../ Of fortune false, and oddes of
Ans : (c) All the four novels are written by Buchi armes in field" (Faerie Queene, Book I, Canto
Emecheta, a Nigerian novelist, who has published 5), this is fine example of
more than 20 books. The publishing year of the above (a) Alliteration (b) Allegory
four novels are– (c) Assonance (d) Antithesis
1- 'The Slave Girl' – 1977 Ans : (a) Alliteration is the occurrence of the same
2- 'The Joys of Motherhood' – 1979 letter or sound at the beginning of closely connected
3- 'Second Class Citizen'– 1974 words.
4- 'In the Ditch' – 1972
40. Match the work with author :
34. In John Bunyan's 'Pilgrim's Progress' who I. "The Excursion" A. S.T. Coleridge
keeps Christian's head above water in the River II. "Christabel" B. P. B. Shelley
of Death? III. "Milton" C. William Wordsworth
(a) Hopeful (b) Helpful IV. "Queen Mab" D. William Blake
(c) Faithful (d) Cheerful
I II III IV
Ans : (a) "The Pilgrim's Progress" is a 1678 Christian (a) C A B D
allegory. It is regarded as one of the most significant (b) C A D B
works of religious English Literature and has been (c) B C A D
translated into more than 200 languages. (d) B A C D
UGC NET/JRF English Paper II, July 2016 222 YCT
Ans : (b) Ans : (d) Alexander Pope (1688-1744) is best known
• "The Excursion" is a long poem published in 1814, for his translation of Homer. His translation of the
written by William Wordsworth. "Illiad" appeared between 1715 and 1720. Then, Pope
• "Christabel" is a long poem written in two parts, by translated the "Odyssey". It appeared in 1726. Pope is
Coleridge. The first part is written in 1797 and the the second most frequently quoted writer in the
second in 1800. Oxford Dictionary of Quotations, after Shakespeare.
• "Milton" is an epic poem, written by William Blake 46. Shyam Selvadurai's 'Funny Boy' is a
in 1804. (a) Picaresque novel (b) Epistolary novel
• "Queen Mab" is a philosophical poem written in (c) Diary novel (d) Coming-of-age novel
nine cantos with seventeen notes and published in Ans : (d) "Funny Boy" is a coming-of-age novel by
1813, composed by P. B. Shelley. Canadian author Shyam Selvadurai, published in
41. Which of the following phrases is not found in 1994. It won the Lambda Literary Award for gay male
Thomas Gray's "Elegy Written in a Country fiction.
Churchyard"? 47. When was the English ban on James Joyce's
(a) "Far from the madding crowd" 'Ulysses' lifted?
(b) "A youth to Fortune and Fame unknown" (a) 1924 (b) 1945
(c) "Full many a flower is born to blush unseen" (c) 1936 (d) 1962
(d) "All nature is but art, unknown to thee" Ans : (c) "Ulysses" is a modernist novel by Irish
Ans : (d) The phrase "All nature is but art, unknown writer James Joyce. It was first serialized in parts in
to thee" is taken from Alexander Pope's "Essay on the American Journal 'The Little Review' from March
Man". The poem was published in 1733-1734. 1918 to December 1920, and entirely published in
42. Robert Browning's "Rabbi Ben Ezra" is defence of 1922 in Paris. The ban levied on it was lifted in 1936.
(a) youth against old age 48. Who among the following is not an imagist?
(b) old age against youth (a) Ezra Pound (b) W.B. Yeats
(c) power against knowledge (c) Amy Lowell (d) T.E. Hulme
(d) knowledge against power Ans : (b) W. B. Yeats (1865-1939) was an Irish poet
Ans : (b) "Rabbi Ben Ezra" is a poem by Robert and one of the foremost figures of 20th century
Browning about Abraham ibn Ezra (1092-1167), one literature. He was a symbolist.
of the great poets, mathematicians and scholars of 12th 49. Thomas Carew's 'Poems' appeared in print in
century. The poem begins : 1640 and contain a variety of amorous addresses
"Grow old along with me! to and reflections on, a fictional mistress known as
The best is yet to be...." (a) Celia (b) Julia
43. In Geoffrey Chaucer's Canterbury Tales, the (c) Anne (d) Melanie
pilgrims, like the medieval society of which they Ans : (a) "Poems" by Thomas Carew, is a collection
are a part, are made up of three social groups or of lyrics, songs, pastorals, poetic dialogues, elegies
"estates". What are the three estates? and occasional poems. Many of the songs and love
(a) Nobility, church and commoners poems are addressed to a fictional character "Celia'.
(b) Royalty, nobility and peasantry 50. Match the novelists with their work :
(c) Royalists, republicans and peasants I. William Golding A. Grimus
(d) Country, city and commons II. Salman Rushdie B. Hawksmoor
Ans : (a) "The Canterbury Tales" is a collection of 24 III. Graham Swift C. Darkness Visible
stories that runs to over 17,000 lines written in Middle IV. Peter Ackroyd D. Waterland
English by Geoffrey Chaucer. The tale is mostly I II III IV I II III IV
written in verse. (a) D A C B (b) C A D B
44. Which novel of Toni Morrison tells the wrenching (c) B C A D (d) B A C D
story of a protagonist who murders her child Ans : (b) C, A, D, B
rather than to allow him/her to live as a slave? • "Hawksmoor" is written by Peter Ackroyd in 1985.
(a) Sula (b) Tar Baby • "Grimus" is the debut novel of Salman Rushdie,
(c) Song of Solomon (d) Beloved published in 1975. It is a fantasy and science fiction
Ans : (d) "Beloved" is a 1987 novel by the American novel.
writer Toni Morrison set after the American Civil War • "Darkness Visible" is a novel by William Golding
(1861-1865), it is inspired by the story of an African published in 1979.
American slave. • "Waterland" is by Graham Swift published in 1983.
45. Who among the following translated Homer? Note : William Golding is the author of the novel
(a) Thomas Gray (b) Samuel Johnson "Darkness Visible", whereas 'Darkness Visible: a
(c) Oliver Goldsmith (d) Alexander Pope Memoir of Madness' is by William Styron.

UGC NET/JRF English Paper II, July 2016 223 YCT


UGC NET/JRF Exam, January-2017
ENGLISH
Solved Paper-III
Note : This paper contains seventy five (75) objective Codes :
type questions of two (2) marks each. All A B C D
questions are compulsory. (a) II III I IV
1. Who among the following is not a diasporic (b) III I IV II
writer? (c) III IV I II
(a) Beryl Bainbridge (d) III II I IV
(b) Timothy Mo Ans. (c) :
(c) Hanif Kureishi ♦ The Egoist : (1914-1919) – Harriet Weavr & Dora
(d) Sam Selvon Marsden.
Ans. (a) : Beryl Bainbridge was an english writer from ♦ The English Review : (1923) – Ford Madox Ford.
Liverpool Herful name was Dame Beryl Margart ♦ Blast : (1914) - Wyndham Lewis & Ezra Pound.
Bainbridge, primarily known for her works of
♦ Poetry : A Magzine of Verse – Harriet Monroe.
psychological fiction often macabre tales.
5. Which statement best expresses the theme of
2. “A text is not a line of words releasing a single Coleridge’s “The Rime of the Ancient
‘theological’ meaning (the ‘message’ of the Mariner”?
Author-God) but a multi-dimensional space in
(a) To kill a living creature is immoral.
which a variety of writings, none of them
(b) People should honour and respect all living
original, blend and clash. The text is a tissue of
things.
quotations drawn from the innumerable
centres of culture.” (c) Prayer can accomplish miracles.
Which of the following best expresses the (d) True harmony is achieved only through
position stated above? cooperative effort.
(a) A text is a tissue of lies that has no referential Ans. (b) : The Rime of Ancient Mariner is the longest
and cultural validity. major poem by Coleridge, written in 1797-98 and
(b) A text is a communication from the Author- published in 1798 in the first edition of Lyrical Ballad.
God with multiple meanings. 6. “The comprehensible Output Hypothesis” was
(c) A text is a force field of ambiguity where proposed by
meanings collapse in the face of opposition. (a) Stephen Krashen (b) M.A.K. Halliday
(d) A text is a linguistic without any unity of (c) Merrill Swain (d) Gertrude Buck
meaning and is linked to multiple sources of Ans. (c): Merries Swain (1944) is Professor Emerita of
language and culture. second language education at the University of Toronto.
Ans. (d) : A text is a linguistic construct without any She developed the output hypothesis, a theory of second
unity of meaning & is linked to multiple sources of language acquisition.
language & culture. 7. In Tristram Shandy Corporal Trim’s brother
3. In William Congreve’s The Way of the World Tom describes the oppression of a black
Fairall is Lady Wishfort’s servant in a sausage shop in Lisbon that he
(a) Son (b) Son-in-law visited. This episode is inspired by a letter
Laurence Sterne received from a black man.
(c) Nephew (d) Servant
Sterne’s reply became an integral part of 18th
Ans. (b) : The way of the world is a play written by century abolitionist literature.
English playwright William Congreve. It premiered in Namethe person who wrote the aforementioned
early March 1700. letter to Sterne.
4. Match the periodical with the founder/s: (a) William Wilberforce
List-I List-II (b) Ignatius Sancho
A. The Egoist I. Wyndham Lewis and (c) William Blackstone
Ezra Pound (d) John Hawkins
B. The English Review II. Harriet Monroe Ans. (b) : The life & opinions of Tristram Shandy,
C. Blast III. Harriet Weaver and Gentleman is a novel by Laurence Sterne. It was
Dora Marsden published in nine-volumes, the first two appearing in
D. Poetry : A Magazine IV. Ford Madox Ford 1759, & seven others following over the next seven
of verse years.
UGC NET/JRF English Paper III, January 2017 224 YCT
8. In Bertolt Brecht Mother Courage and Her 13. In a writing system the minimal unit that can
Children, which song does Y vette sing to cause a difference of meaning is called
Mother Courage and Kattrin? (a) phoneme (b) grapheme
(a) “The Song of the Gret Souls of the Earth” (c) morpheme (d) jargon
(b) “The Fraternization Song” Ans. (b) : Grapheme, is the smallest meaningful
(c) “The Song of the Great Capitulation” contrastive unit in a writing system.
(d) “The Memorial Song”
14. Nnu Ego is a character in
Ans. (b) : Mother Courage & Her Children is a play (a) Chinua Achebe’s Anthills of Savannah
written in 1939 by the German dramatist & poet
(b) Chimamanda Ngozi Adichie’s Half of a
Bertolt Brecht.
Yellow Sun
9. In Gustave Flaubert’s Madame Bovary, under (c) Buchi Emecheta’s The Joys of Motherhood
what pretext does Emma go every week for her
clandestine meeting with Leon in Rouen? (d) Ben Okri’s The Famished Road
(a) Under the pretext of going to the church for Ans. (c) : The Joys of Motherhood is a novel first
weekly confession. published by Allison & Bus by in 1979.
(b) Under the pretext of meeting her blind friend 15. Match the word with definition:
who lives alone. List I List-II
(c) Under the pretext of weekly shopping A. Etymon I. Changing from one
(d) Under the pretext of taking piano lessons. language variety to
Ans. (d) : Madame Bovary is the debut novel of French another in discourse
writer Gustave Flaabert published in 1856. B. Code switching II. Rules governing the
10. Identify the two books by C.S. Lakshmi social use of language.
(Ambai) published in English translation: C. Cognate III. Etymological source of a
I. Astride the Wheel II. Going Home word
III. A Purple Sea IV. In a Forest, A Deer D. Pragmatics IV. Words with a common
The right combination according to the code is ancestor.
(a) III and II (b) I and II Codes :
(c) I and IV (d) III and IV A B C D
Ans. (d) : A Purple Sea – Short stories (1992) (a) IV I III II
In a forest, A Deer- stories. (b) III II IV I
11. Elizabeth Barrent Browning’s Sonnets from (c) III I IV II
the Portuguese is (d) IV I II III
I. a sequence of forty four Petrarchan sonnets.
Ans. (c) :
II. a rewriting of Popean didactic verse.
(A) Etymon – Etymological source of a word.
III. a depiction of a contemporary setting and
small events of ordinary life. (B) Code switching – Changing from one language
IV. a scathing criticism of the British colonial variety to another in discourse.
enterprise, (C) Lognate – Words with a common ancestor.
The right combination according to the code is (D) Pragmatics – Rules governing the social use of
(a) I and II (b) I and III language.
(c) II and IV (d) I and IV 16. What would help a reader recognize Keats’s
Ans. (b) : Elizabeth Barrett's 'Sonnets from the “To Autumn” as a poem from the Romantic
Portuguese' (1845-46) is a collection of forty-four period?
Petrachan sonnets and it describes the contemporary (a) Its logical succession of images
setting and small events of ordinary life. (b) Its concise use of couplets
Hence, option (b) will be correct answer. (c) Its lavish natural imagery
12. In The Story of My Experiments with Truth, (d) Its use of iambic pentameter
M.K.Gandhi covers the narrative of his life Ans. (c) : ‘To Autumn’ was composed on 1819 &
from early childhood through to published in 1820, in a volume of Keats’s poetry that
(a) 1925 (b) 1929 included Laima & The Eve of St. Agnes.
(c) 1921 (d) 1927 17. Which of the following is an accurate
Ans. (c) : It is an autobiography of Gandhi, covering description of ‘heteroglossia?
his life from early childhood through to 1921. It was (a) Heteroglossia makes the job of the novelist
written in weekly installment & published in his Journal easier by incorporating diversity into the
Navjivan from 1925 to 1929. novelistic structure.
UGC NET/JRF English Paper III, January 2017 225 YCT
(b) Heteroglossia functions in a novel in alliance III. The poet turns to her “to find/The Arabian
with its stylistic system incorporating desert of the human mind/Hoping if still
multiple voice inscribed in social language IV. “She is the first of lands, the warmest.”
and differentiated components of a writer’s Codes :
ideological position. (a) I and III (b) II and III
(c) Heteroglossia creats concrete (c) III and IV (d) I and IV
conceptualizations through language in
Ans. (a) :
association with the singular view of the
artistic effort resulting in the unified world of ♦ Australia is ‘without songs, architecture, history’.
the novel. ♦ The poet turns “to her to find/The Arabian desert pf
(d) Heteroglossia enters the linguistic universe of the human mind/hoping it still from deserts prophets
the novel to homogenize its multiple come”.
differences and voices in a singular vision of 22. Basic English, a simplified and fundamental
accomplished structure. framework of English, was formulated by
Ans. (b) The term "heteroglossia" refers to the qualities I. I.A. Richards II. Alastair Fowler
of a language that are extralinguistic, but common to all III. William Empson IV. C.K. Ogden
languages. This concept has been given by Mikkhail The right combination according to the code is:
Bakhtin. He gives the example of an illiterate peasant, (a) I and II (b) II and III
who speaks Church Slavonic to God, speaks to his (c) I and IV (d) I and III
family in his own peculiar dialect, sings songs in yet a Ans. (c) : The meaning of meaning: The study of the
third and attempts to emulate high-class dialect when he influence of language upon thought & of the science of
dictates petitions to the local government. Option (b) symbolism is a book by C.K. Ogden and I.A. Richards,
seems to be very close to the particular term published in 1923.
"heteroglossia", therefore, option (b) will be correct 23. “Britons will never be slaves!” –felt proud
answer. Britons in the eighteenth century. A great
18. In Ulysses Leopold Bloom works for a Dublin many Britons, though, had no qualms about
(a) bar (b) park owning slaves and profiting from them. Who
(c) newspaper (d) bank among the following British authors self-
Ans. (c) : Ulysses is a modernist novel by Trish writer consciously engaged with the issue of slavery in
James Joyce. It was first serialised in parts in the some poems?
American Journal ‘The Little Review’. I. Hannah More
19. Which pair of plays belongs to the early career II. Mary Collier
of Harold Pinter? III. Anna Seward
I. The Caretaker II. One for the Road IV. Anna Yearsley
III. Celebration IV. The Room The right combination according to the code is
The right combination according to the code is (a) I and III (b) I and IV
(a) I and III (b) II and III (c) II and III (d) III and IV
(c) I and IV (d) II and IV Ans. (b) : Anna Yearsley (1753-1806) was an english
Ans. (c) : poet & writer. She wrote A poem on the inhumanity of
the slave-Trade appeared in 1788. Her poem was
♦ The caretaker – 1960 considered by many critics to rival a similar poem
♦ One for the road– 1984 written by her ex-patron Hannah More entitled,
♦ Celebration – 2000 “Slavery: A Poem”.
20. Who among the following contemporaries of 24. Match the Novelist with the work :
John Donne wrote the following lines on his List-I List-II
death: “Here lies a king, that ruled as he A. Anita Desai I. Rich Like Us
thought fit/The universal monarch of wit”? B. Nayantra Sahgal II. The Nowhere Man
(a) George Herbert (b) Henry King C. Arun Joshi III. In Custody
(c) Thomas Carew (d) Henry Crashaw D. Kamala Markandaya IV. The Last Labyrinth
Ans. (c) :Thomas Carew (1595-1640) was an english Codes :
poet, among the ‘Caualien’ group of caroline poets. A B C D
21. In his poem “Australia” A.D. Hope says that (a) III II IV I
I. Australia is “without songs, architecture, (b) III I IV II
history”. (c) II I IV III
II. “Her five cities are like five dry rivers.” (d) III IV I II
UGC NET/JRF English Paper III, January 2017 226 YCT
Ans. (b) : (b) “In my beginning is my end”, “East Coker”
♦ Anita Desai – In custody (1984). (c) “Human kind cannot bear very much reality”,
♦ Nayantara Sahgal – Rich like vs (1985). “Burnt Norton”
(d) “I have measured out my life with coffee
♦ Arun Joshi – The Last Labyrinth (1981)
spoons,” “Love Song of J. Alfred Prufrock”
♦ Kamala Markandaya – The Nowhere Man (1972).
Ans. (b) : Baumgartner’s Bombay (1988) explores
25. Identify the right chronological sequence: German and Jewish identity in the context of a chaotic
(a) The American Pastoral –Sister Carrie–The contemporary India.
Great Gatsby – Beloved
29. In the General Prologue to The Canterbury
(b) The Great Gatsby – Sister Carrie–Beloved–
Tales which two characters are examples of
The American Pastoral
deep Christian goodness?
(c) Sister Carrie– The Great Gatsby –Beloved –
I. The Summoner II. The Parson
The American Pastoral
III. the Polughman IV. The Pardoner
(d) Sister Carrie –The Great Gatsby – The
American Pastoral-Beloved The right combination according to the code is :
(a) I and II (b) II and IV
Ans. (c) :
(c) II and III (d) I and IV
♦ Sister Carrie (1900) is a novel by Theodore Dreiser.
Ans. (c) : The Parson & the Plaughman are the
♦ The Great Gatsby (1925) by F. Scott Fitzgerald. example’s of deep Christian goodness, in the General
♦ Beloved (1987) by Tony Morrison. Prologue to The Canterbury Tales, written by Chaucer.
♦ The American Pastoral (1997) by Philip Roth. 30. Identify Falstaff’s words in Henry IV, Part I :
26. In which of the following senses did Marx and (a) “Now, Harry, what time of day is it, lad?”
Engels originally use the term “ideology” in (b) “Now, Hal, what time of day is it, lad?”
The German Ideology? (c) “Now, Harry, what time of night is it, lad?”
(a) Something that mystifies the actual material (d) “Now, Hal, what time of night is it, lad?”
conditions of society, a short of false
consciousness. Ans. (b) : Henry IV, part I is a history play by
William Shakespeare believed to have been written no
(b) The elaborate structures and institutions that
later than 1597.
mark the bourgeoise society.
(c) The concepts of base and superstructure that 31. Anna Barbauld, Laetitia Elizabeth London,
govern the economic relations of the Charlotte Smith, Mary Robinson and Felicia
(d) The fundamental class consciousness of the Hemans are
proletariat which leads to their awakening. (a) first wave feminists
Ans. (a) : The German Ideology is a set of manuscript (b) women poets of the Romantic period
written by Kar/Marx and Friedrich Engels around April (c) Victorian writers of popular fiction
or early May 1846. But the work was published for the (d) nineteenth century stage artists
first time in 1932. Ans. (b) :
27. The plot of this Coetzee novel unravels the ♦ Anna Barbauld – (1802-1838)
narrative of a poor man of colour trying to ♦ Laetitia Elizabeth London – (1802-1823)
survive in a civil-war situation, never taking ♦ Charlotte Smit – (1749-1806)
sides. Identify the novel.
♦ Mary Robinson – (1757-1800)
(a) Disgrace
♦ Felicia Hemans – (1793-1835)
(b) Age of Iron
(c) Waiting for the Barbarians 32. Ray Bradbury has titled one of his short story
(d) Life and Times of Michael K. collections- Golden Apples of the Sun-after the
last line of a W.B. Yeats poem. Which poem?
Ans. (d) : Life and Times of Michael K (1983) won the
(a) “The Death of Cuchulain”
Booker Prize for 1983. The novel is a story of a men
named Michael K. who makes an arduous journey from (b) “The Peacock”
Lape Town to his mother’s rural birth place, during an (c) “The Hour Before Dawn”
imaginary civil war during the apartheid era in the (d) “The Song of Wandering Aengus”
1970-80s. Ans. (d) : The Golden Apples of the Sun (1953) is an
28. Which of the following lines of T.S. Eliot is anthology of 22 short stories by Ray Bradbury. The
used by Anita Desai as the epigraph for her book’s title is also the title of the final story in the
novel, Baumgartner’s Bombay? collection. The words :the Golden Apples of the Sun”
(a) “I will show you fear in a handful of dust,” are from the last line of the final stanza of W.B. Yeats
The Waste Land poem “The Song of Wandering Aengus (1899)”.
UGC NET/JRF English Paper III, January 2017 227 YCT
33. Which play by Tom Stoppard set in Zurich Ans. (c) : Vanity Fair is an english novel by
during the First World War presents a William Makespeace Thackeray which follows the lives
character’s interactions with James Joyce as he of Beck Sharp and Emmy Sedley amid their friends and
was writing Ulysses, Tristran Zara during the families during and after the Maprkonic Wars.
rise of Dadaism, and Lenin leading up to the
Russian Revolution, all of whom were living in 38. Which character in Anton Chekhov’s play, The
Zurich at that time? Cherry Orchard, first suggests the selling of the
(a) After Magritte orchard?
(b) Dirty Linen (a) Trofimov (b) Yephikodov
(c) Artist Descending a Staircase (c) Lopakhin (d) Varya
(d) Travesties Ans. (c) : The Cherry Orchard is the play by Russian
Ans. (d) : Travesties is a 1974 play by Tom Stopped. Playwright Anton Chekhov written in1903, it was first
The play centres on the figure of Henry Larr an elderly published by Znaniye, and comes out as a separate
man who reminisces about Zurich in 1917 during the edition later that year in Saint Petersburg.
first World War. 39. Identify the correct chronological sequence of
34. “Most blameless is he, centered in the sphere of the founding of the following 18th century
common duties, decent not to fail in offices of English periodicals:
tenderness….” (a) Tatler–Spectator – The Gentleman’s
In these lines from “Ulysses”, what does Magazine–Rambler
Ulysses suggest about Telemachus? (b) Spectator–Tatler – The Gentleman’s
(a) He shows heroic qualities.
Magazine –Rambler
(b) He is patient and selfless.
(c) Rambler–Tatler – Spectator–The
(c) He is very much like his father.
Gentleman’s Magazine
(d) He may be too tender-hearted to be king.
(d) Tatler–Spectator – Rambler–The
Ans. (b) : Ulysses is a poem in blank verse by the
Victorian poet Alfred Lord Tennyson (1809-1892), Gentleman’s Magazine
written in 1839 and published in 1842 in his well- Ans. (a) :
received second volume of poetry. ♦ Tatler – 1709
35. In Restoration comedies the following is true ♦ Spectator – 1711
EXCEPT ♦ The Gentleman’s Magazine – 1731 (In London)
(a) the London life of hedonistic young men is ♦ Ramber – 1750
portrayed.
(b) names encapsulate traits 40. Who identified “strangled articulateness” as a
(c) unchaste women, widows and cuckolds theme in Canadian writing?
scarcely make an appearance. (a) Margaret Atwood (b) Northrop Frye
(d) the heroines seek a say in the choice of a (c) Michael Ondaatjee (d) Joy Kogawa
marriage partner. Ans. (b): Herman Northrop Frye (1912-1991) was a
Ans. (c) : In Restoration comedies, unchaste women, Canadian literary critic and literary theorist, considered
widows & cuckolds scarcely make an appearance, is the one of the most influential of the 20th century.
wrong sentence. 41. Identify the gynocritics in the following list:
36. What happens to the character Boy at the end I. Alice Jardine II. Elaine Showalter
of Luigi Pirandello’s play Six Characters in III. Sandra Gilbert IV. Kate Millett
Search of an Author?
The right combination according to the code is:
(a) He drowns in the fountain
(a) I and II (b) II and IV
(b) He is shot dead by the Father
(c) II and III (d) III and IV
(c) He leaves the stage alone
(d) He commits suicide. Ans. (c) : Gynocritics is the term coined in the seventies
by Elaine Showalter to describe a new literary project
Ans. (d) : Six Characters in search of an Author is an
Italian play by Luigi Pirandello written and first intended to construct “a female framework for the
performed in 1921. analysis of women’s literature”.
37. Which of the following adjectives will not apply 42. Identify the character who is not part of the
ti Becky Sharp, a major character in Vanity group of three protagonists in Girish Karnad’s
Fair? Hayavadana:
(a) ambitious (b) energetic (a) Padmini (b) Gautama
(c) wellborn (d) scheming (c) Kapila (d) Devadatta
UGC NET/JRF English Paper III, January 2017 228 YCT
Ans. (b): In the given options, Gautama is the character 49. Elizabeth Bishop’s poems are best remembered
who does not belong to Girish karnad's 'Hayavadana'. for their
Gautama is mentioned in Anita Desai's novel 'Cry, the (a) conversational intimacy
Peacock' (1963), while Padmini is a character in (b) intellectual tenor
Karnad's 'Hayavadana', who marries Devadutta. She is (c) astringent satire
the lady who ignites the rivalry between Devadutta and
(d) urban topography
Kapila.
Hence, option (b) will be correct answer. Ans. (a) : Elizabeth Bishop (1911-1979) was an
American poet and short story writer. She considered
43. Aurobindo Ghosh, author of ‘Savitri’, taught
for some time at Baroda College after his one of the finest poets of the 20th century.
return from England in 1893. Which subject 50. Which chilling novel of surveillance and
did he teach? entrapment had the alternative title Things as
(a) English (b) French They Are?
(c) Sanskrit (d) Bengali (a) Horace Walpole’s Castle of Otranto
Ans. (a) : In 1897, Aurobindo Ghosh joined the Baroda (b) Matther Gregory Lewis’s The Monk.
College as a French teacher. He also taught English (c) Thomas Love Peacock’s Nightmare Abbey
during this time. UGC has accepted option (a), the (d) William Godwin’s Caleb Williams
correct answer. Ans. (d) : Things as They are: or, The Adventure of
Hence, option (a) will be correct answer. Caleb Williams (1974) is a three volume novel written
44. Christopher Marlowe’s Hero and Leander can as a call to end the abuse of power by a tyrannical
be classified as a/an government.
(a) complaint (b) stichomythia
51. In “My Last Duchess” which of the following is
(c) epyllion (d) pasturelle not one of the Duchess’s misdemeanours,
Ans. (c) : Epyllion, is a narrative poem that resembles according to the Duke?
an epic poem in style, but which is notably shorter. (a) She was flattered by compliments from Fra
45. Which among the following does not belong to Pandolf.
Indo-European language family? (b) She enjoyed the sunset as much as she
(a) English (b) German enjoyed her husband’s favour.
(c) Scandinavian (d) Finnish (c) She wouldn’t listen to her husband when he
Ans. (d) : Finnish is a Finnic language spoken by the tried to correct her behaviour.
majority of the population in Finland and by ethnic (d) She was equally grateful for all acts of
Finns outside Finland. kindness, regardless of their source.
46. What, among the following, is ruled out by Ans. (c) : She wouldn’t listen to her husband when he
Longinus as a way of achieving the sublime?
tried to correct her behaviour.
(a) great thoughts
(b) immoderate emotion 52. In his essay “From Work to Text” Roland
(c) noble diction Barthes says the following about the text:
(d) dignified and elevated word arrangement I. The text is singular.
II. The text can be held in the hand.
Ans. (b): Immoderate emotion, is ruled out by Longinus
as a way of achieving the sublime. III. The text is held in language.
47. Who among the following is not a beat writer? IV. The text is a methodological field
(a) Jack Kerouac (b) Allen Ginsberg The right combination according to the code is:
(c) Robert Lowell (d) William Burroughs (a) I and III (b) II and IV
Ans. (c) : Allen Ginsberg’s Howl (1956) (c) III and IV (d) III and II
William Burroughs’s Naaked Lunch (1959) Ans. (c) : In his essay “From Work to Text” (1971),
Jack Kerouac’s On the Road (1957) Roland Barthes says that the text is held in language &
48. This was a masque written by Ben Jonson, the text is a methodological field.
staged on Twelfth Night and it was the first 53. Seamus Heaney’s “Digging” in his first volume
masque in which Prince Charles took part. of poetry, Death of a Naturalist, illustrates all
(a) Masque of Blankness the following EXCEPT
(b) The Masque of Queens (a) his preoccupation with his roots
(c) Pleasure Reconciled to Virtue (b) his obsession with Irish legend and folklore
(d) The Gypsies Metamorphed (c) his respect for the natural world of the
Ans. (c) : Pleasure Reconciled to Virtue is a Jacbean era farming community and the labour of his
masque written by Ben Jonson. First performed on ancestors
Twelfth Night, 1618. (d) his displaced vocation of digging with a pen
UGC NET/JRF English Paper III, January 2017 229 YCT
Ans. (b) : Death of a Naturalist is a collection of poems 57. In Sophocles’s play King Oedipus Laius, the
written by Seamus Heaney, who received the 1995 erstwhile ruler of Thebes, was murdered
nobel prize in literature. This collection was originally (a) at the edge of the forest on his way to Delphi
published in 1966. (b) at the edge of the forest as he returned from
Delphi
54. Here is a list of Indian writers who have
(c) at the crossroads as he returned from Delphi
translated their work into English. Match the
writer with his source language: (d) at the crossroads in his way to Delphi
List-I List-II Ans. (d) : Oedipus the King, is an Anthenian tragedy,
that was first performed around 429 Bc.
A. O.V. Vijayan I. Kannada
B. Vilas Sarang II. Malayalam 58. The quintessentially metafictional novel, If on a
Winter’s Night a Traveller by Italo Calvino has
C. Krishna Baldev Vaid III. Marathi
alternate chapters with chapter numbers and
D. Girish Karnad IV. Hindi titles. Which of the following are the titles of
Codes : the chapters in the novel?
A B C D I. Looks Down in the Gathering Shadow
(a) II IV III I II. In a Network of Lines that Enlace
(b) I III IV II III. In a Network of Linew that Interface
(c) II III IV I IV. What story there A waits its End?
(d) II III I IV The right combination according to the code is:
Ans. (c) : (a) I and II (b) I and IV
(A) O.V. Vijayan – Malayalam (c) III and IV (d) II and IV
(B) Vilas Sarang – Marathi Ans : (a)
(C) Krishna Baldev Vaid – Hindi ♦ Looks Down in the Gathering Shadow– Chapter 10
(D) Girish Karnad – Kannada ♦ In a Network of Lines that lines that Enlace –Chapter 6
55. In Book 8, Paradise Lost Adam identifies his 59. The novel Maurice by E.M. Forster appeared
posthumously in 1971. It had a homosexual
chief flaw or weakness to Raphael. What is this
theme, so Forster considered its subject matter
floaw?
too indelicate for publication during his life
(a) gluttony time. It was influenced by a writer who was a
(b) pride in his superiority to Eve socialist and open homosexual. Identify the
(c) overconfidence in his free will writer.
(d) passion for Eve (a) Oscar Wilde (b) Edward Carpenter
Ans. (d) In Book 8, 'Paradise Lost', Adam identifies his (c) W.H. Auden (d) E.F. Benson
chief flaw or weakness to Raphael and this flaw is Ans. (b) : Maurice is a tale of homosexual love in early
known as passion for Eve. Adam explains in this 20th century England. It was written in 1913-14. Forster
chapter that he is overcome with love and desire for was close friends with the poet Edward Carpenter, by
because of her physical beauty and feels literally whome he was moliuated.
weakened by her attractiveness, this proved to be a fatal 60. Who among the following has elaborated on
flaw for Adam. the “Indianisation” of English?
Hence, option (d) will be correct answer. (a) L.M.Khubchandani (b) B. Kumaravadivelu
56. Identify the correct chronological sequence of (c) B.B. Kachru (d) Rajendra Singh
the flowing early Engliosh texts: Ans. (c) "Indianisation" signifies the importance of
(a) Troilus and Criseyde–The Owl and The Indian languages, culture, economic reach, diaspora and
cuisines. B.B. Kachru has pioneered, shaped and
Nightingale –Utopia Morte d’ Arthur
defined the linguistic, socio-cultural and pedagogical
(b) Troilus and Criseyde – Morte d’ Arthur – The dimentions of cross-cultural diffusion of English. In this
Owl and the Nightingale way he has elaborated on the "Indianisation" of English.
(c) The Owl and the Nightingale – Troilus and Hence, option (c) will be correct answer.
Criseyde– Morte d’ Arthur – Utopia 61. These are four models of relating literature to
(d) The Owl and the Nightingale – Morte d’ history. Which of the following is associated
Arthur and Criseyde – Uttopia with formalism?
Ans. (c) : (a) Literary texts are universal and transcend
♦ The owl and The Nightingale – 12th or 13th century. history: the historical context of their
♦ Troilus & Criseyde – (1378-85) production and reception has no bearing on
the literary work which is aesthetically
♦ Morte d’ Arthur – (1485) autonomous, having its own laws, being a
♦ Utopia – (1516) world into itself.
UGC NET/JRF English Paper III, January 2017 230 YCT
(b) The historical context of a literary work is 65. Which of Kazuo Ishiguro’s novels are set
integral to a proper understanding of it: the mostly in Japan?
text is produced within a specific historical I. The Unconsoled
context but in its literariness it remains II. The Remains of the Day
separate from that context. III. An Artist of the Floating World
(c) Literary work can help us to understand the IV. A Pale view of Hills
time in which they are set : realist texts in The right combination according to the code is
particular provide imaginative representations (a) I and III (b) II and III
of specific historical moments events or
(c) III and IV (d) I and IV
periods.
(d) Literary texts are bound up with other Ans. (c) :
discourses and rhetorical structures: they are ♦ An Artist of the Floating World (1986) – It is set in
part of a history that is still in the process of post Klorld War II Japan, narrated by painter, Masaji
being written. Ono.
Ans. (a) : Literary texts are universal transcend history : ♦ The Unconsoled (1995) – Ryder, a famous pianist
The historical context of their production & reception arrives in Central European city to perform.
has no bearing on the literary work which is ♦ The remains of the day (1989) – Stevens, a butter,
aesthetically autonomous, having its own laws being recalls his life in form of diary.
world into itself. ♦ A Pale view of Hills (1982)- First novel. A middle
62. As Gunter Grass’s novel The Tin Drum opens age Japanese woman lives in England.
we find Oskar Matzerath 66. In the Advancement of Learning Bacon noted
(a) on the war front entertaining the soldiers as the need for more studies of
part of a band of dwarfs. I. moral knowledge
(b) in a mental hospital writing his story. II. forbidden knowledge
(c) admitted in a hospital after his fatal fall in the III. civil knowledge
wine cellar. IV. spiritual knowledge
(d) watching a ball in which the young ladies The right combination according to the code is :
ignore his presence. (a) I and III (b) I and IV
Ans. (b) : The Tin Drum is a 1959 novel by (c) II and III (d) II and IV
Gunter Grass. The novel is the first book of Grass’s Ans. (a) : The advancement of learning is a 1605 book
Danziger Trilogic. day Francis Bacon.
63. D.H. Lawrence’s 1926 novel The Plumed 67. Which among the following texts purports to
Serpent is sent in which country? be the autobiography of a mad German
(a) Egypt (b) South Africa philosopher edited by an equally fictitious
(c) Mexico (d) Peru editor?
Ans. (c): The plumed Serpent is a 1926 novel by (a) Sartos Resartus
D.H. Lawrence. Set in Mexico during the Mexican (b) The Dream of Gerontius
revolution. (c) The Professor
64. Which two writers can be described as writing (d) Felix Holf
historical novels? Ans. (a) : Sartos Resartus (meaning.The tailor
I. Sir Walter Scott II. Charlotte Bronte re-tailored) is an 1836 novel by Thomas Carlyce. The
III. Maria Edgeworth IV. Jane Austen novel purports to be a commentary on the thought &
The right combination according to the code is early life of a German Philosopher.
(a) I and II (b) II and III 68. As Sidney argues in A Defence of Poesy which
(c) I and III (d) III and IV discipline is more useful and praiseworthy
history or poetry?
Ans. (c) :
(a) History “being captivated to truth” is more
♦ Sir Walter Scott (1771-1832) was Scottish historical useful than poetry.
novelist.
(b) Poetry where man can see “virtue exalted and
♦ Charlotte Bronte (1816-1855) was english novelist & vice punished: is more useful than history.
poet. (c) History is more useful for poetry is “an
♦ Maria Edgeworth (1768-1849) was Anglo-Irish encouragement to unbridled wickedness”.
writer of adults & children’s literature. (d) History and poetry are synonymous, and so
♦ Jane Austen (1775-1817) was an english novelist. both are useful
UGC NET/JRF English Paper III, January 2017 231 YCT
Ans. (b) : Poetry where an can see “virtue exalted & On the dune-side slope, spoiling
vice punished”is more useful than history. The grass-inscribed sand. Neither of us looked-
69. In Bunyan’s Pilgrim’s Progress Christian and It seemed best to back away.
his friend faithful cause a commotion at the On the dune’s steep side
Vanity Fair for many reasons. Which of the We surveyed what we’d come for : ocean’s
following statement is not true of their Snaking blue beyond the meadow, the silvered
appearance at the fair? Blade-like wands lying down. Lovely enough
(a) They are dressed differently than the other To hold ourselves to that view.
fair-goers. But the currents of an odor wafted in and out,
(b) They speak the language of the Bible at the Until the sweep of smell grew wider, wilder.
fair. The heat compounded, and ugliness
(c) They sample every entertainment at the fair. Settled its could over us, profound as human speech,
(d) They refuse to look at the merchandise at the
Although by then we were not speaking.
fair.
72. The “We” of the opening line indicates
Ans. (c) : The Pilgrim’s Progress from this world, to (a) a group
that which is to come is a 1678 Christian allegory
(b) two persons
written by John Bunyan.
(c) the speaker and an imaginary listener
70. What does the title Morte d’ Arthur mean?
(d) an unspecified crowd
(a) Arthur mortified (b) Death of Arthur
Ans. (b) : According to the given poetry, the word "we"
(c) Castle of Arthur (d) Burial of Arthur
of the opening line indicates "two person". This poetry
Ans. (b) : Le Morte d’ Arthur (death of Arthur) is a is written by Cleopatra Mathis an American poet. These
reworking of existing tales by Sir Thomas Malory about lines tells the truth to certain stages of our lives.
the Legendary King Arthur.
Hence, option (b) will be correct answer.
71. Assertion (A) : Characters is novels are people
73. The dead animal was sighted
whose secret lives are visible or might be
(a) at the end of the trail
visible. We are people whose secret lives are
invisible. (b) on the dune’s steep side
Reason (R): Even when novels are about (c) on the dune’s sloping side
wicked people, they can solace us; they suggest (d) in the swamp undergrowth
a more manageable human race, they give us Ans. (c) : The dead animal was sighted on the dune’s
the illusion of seeing clearly and of power. sloping side.
In the light of the statement above 74. The reaction evoked in response to a glimpse of
(a) Both (A) and (R) are correct and (R) is the the dead fox is best described as
correct explanation of (A). I. evasive II. angry
(b) Both \(A) and (R) are correct but (R) is not III. bizarre IV. muted
the correct explanation of (A). The right combination according to the code is
(c) (A) is right, but (R) is wrong. (a) I and II (b) II and III
(d) (A) is wrong, but (R) is right. (c) I and IV (d) III and IV
Ans. (b) : Both (A) and (R) are correct but (R) is not the Ans. (c) : According to the passage, the reaction evoked
correct explanation of (A). in response to a glimpse of the dead fox is best
Read the following poem and answer the described as bizarre, because there are something we
questions, 72 to 75 can call variegated or mottled. Therefore, option (c) will
Dead Fox be correct answer.
We pretended to know nothing about it. 75. At the close of the poem, which of the following
I withdrew to my childhood training: stay out senses overpowers and renders the visitors
Of swampy undergrowth, choked edges. speechless?
This was around the time (a) sight (b) touch
We were too cruel to kill the mice we caught, (c) sound (d) smell
Leaving them in the Have-a-Heart trap Ans. (d) : At the close of the poem, smell overpowers
under the sun-burning bramble of rugosa. and renders the visitors speechless. Here, the poet has
But moving up the trail, we caught a glimpse used olfaction, a kind of imagery through smell.
Right at the start: the fox just over the hillock Hence, option (d) will be correct answer.

UGC NET/JRF English Paper III, January 2017 232 YCT


UGC NET/JRF Exam, January-2017
ENGLISH
Solved Paper-II
Note: This papers contains fifty (50) objective type 5. Which among the following texts can be
questions of two (2) marks each. All questions characterised as a lesbian Bilbungsroman?
are compulsory. (a) Angela Charter, The Magic Toyshop
1. Identify from the following the work Nirad C. (b) Sylvia Plath, The Bell Jar
Chaudhuri called “the finest novel in the (c) Jeanette Winterson, Oranges Are Not the
English language with an Indian theme”. Only Fruit
(a) Kim (d) Ruth Pawar Jhabvala, Heat and Dust
(b) A Passage to India Ans. (c) : Jeanette Klinterson, Oranges are not the only
(c) Train to Pakistan fruit can be characterised as a lesbian Bildurg Sroman,
(d) Private Life of an Indian Prince published in 1985. It is a coming of age story about a
Ans. (a) : ‘Kim’ a novel by Rudyard Kipling first Lesbian girl.
published in Mc Clure’s Mazine (Dec 1900-Oct 1901) 6. Identify the correct chronological sequence of
as well as Cassell’s Magzine from Jan__ to __ Nov publication:
1901. The first published in Maomillian & Co. Ltd in (a) Paradise Lost – The Advancement of
Oct 1901. The story unfolds against the backdrop of Learning–An Essay Concerning Human
“The Great Game” (Political conflict between Russia & Understanding– MacFlecknoe
Britain in Central Asia). (b) The Advancement of Learning – An Essay
Concerning Human Understanding –
2. Who is the author of the poem “The Defence of
MacFlecknoe–Paradise Lost
Lucknow” dealing with the siege of Licknow,
one of the terrible incidents of the Indian (c) The Advancement of Learning–Paradise Lost
– MacFlecknoe –An Essay Concerning
Mutiny?
Human Understanding
(a) Rudyard Kipling (b) Edward Lear
(d) Paradise Lost– MacFlecknoe – The
(c) Alfred Lord Tennyson (d) Robert Browning Advancement of Learning – An Essay
Ans. (c): The Defence of Lucknow is written by Concerning Human Understanding
Alfred L. Tennyson during 1857, depicting the incidents Ans. (c) :
of Indian Mutiny of 1857.
♦ The advancement – 1605 by Francis Bacon.
3. Who among the following theorists holds that of learning
metaphor and metonymy are the two ♦ Paradise Lost – 1667 by Milton
fundamental structures of language?
♦ Mac Flecknoce – 1682 by Dryden
(a) Ferdinand de Saussure
♦ An essay concerning – 1689 by John Lock
(b) J.L. Austin
human understanding
(c) Roman Jakobson
(d) Victor Shklovsky 7. Poe’s “The Raven” mourns the death of Poe’s
(a) lost Lenore (b) lost Abigail
Ans. (c) : Roman Jakobson (1896-1982) was a
(c) Pet animal (d) lost heritage
Russian American linguist & literary theorist. Influenced
by the work of ‘Ferdinand de Saususe’. Though his Ans. (a) : “The Raven” (1845) is a narrative poem by
decisive influence on Claude Levistrauss and Roland American writer Edgar Allan Poe.
Barthes, Jacques Lacan, Michel Foucault etc. 8. In Shakespeare’s Macbeth who was “untimely
4. From among the following, who are the ripped” from his mother’s womb?
Dashwood sisters in Jane Austen’s Sense and (a) Macbeth (b) Macduff
Sensibility? (c) Duncan (d) Malcolm
I. Elinor II. Marianne Ans. (b): Macduff was ultimately ripped from his
III. Mary IV. Amanda mother’s Womb Macbeth is a tragic play written by
Shakespeare.
The right combination according to the code is :
(a) I and III (b) I and II 9. Alexander Pope revised The Rope of the Lock
three times. In the final revision of the poem in
(c) II and III (d) III and IV
1717 he inserted a speech by
Ans. (b) : Elinor & Marianne are the Dashwood sister (a) Belinda (b) Clarissa
in Austen’s Sense & sensibility published in 1811. (c) Betty (d) Thalestris
UGC NET/JRF English Paper II, January 2017 233 YCT
Ans. (b) : “The Rape of the Lock” is a mock-heroic 14. Which of the following works Daniel Defoe
narrative poem. One of the most commonly cited offered his readers as a collection of “Strange
examples of high burlesque, it was first published in Surprising Adventures”?
anonymously in 1712. (a) Moll Flanders (b) Robinson Crusoe
10. Identify, from the following list, two plays (c) Roxana (d) Captain Singleton
written by John Webster: Ans. (b) : Robinson Crusoe (1719) is a novel, first
I. A Woman Killed with Kindness published on 25 April, 1719. The first edition credited
II. The Revenger’s Tragedy the work’s protagonist Robinson Crusoe as its author
III. The White Devil leading many readers to believe he was a real person the
IV. The Ducchess of Malfi book a travelogue of true incidents.
The right combination according the code is : 15. In Charlotte Bronte’s Jane Eyre, what does
(a) I & IV (b) II and IV Mr. Brocklehurst accuse Jane of when he visits
(c) III & IV (d) I & III Lowood School?
(a) Lazinees (b) Stealing
Ans. (c) :
(c) Lying (d) Spying
♦ The Duchess of Malfi
Ans. (c) : Jane Eyre (1847) originally published as
♦ The White Devil - By John Webster.
Jane Eyre: An Autobiography.
16. William Faulkner’s As I Lay Dying contains
♦ A woman killed with kindness (1607)
one of the shortest chapters in literary history.
- By Thomas Heywood Which of these sentences is the chapter in its
♦ A Revenger’s Tragedy (1607) entirety?
- By Thomas Middleton. (a) “For the love of God, where is my hat?”
11. Which of the following works by David Malouf (b) “My mother is a fish.”
tells the story of the Roman poet, Ovid, during (c) “Addie Bundren was dead, to begin with.”
his exile in Tomis? (d) “Apricot Jam is the worst sort of Jam.”
(a) Remembering Babylon Ans. (b) : ‘As T Lay Dying’ (1930) in the genre of
(b) The Great World Southern Gothic Faulkner said that he wrote the novel
(c) The Conversations at Curlow Creek from mid-night to 4 am over the course of 6 weeks.
(d) An Imaginary Life 17. The prelude to Middlemarch makes a reference
Ans. (d) : to the particular history of a remarkable
♦ Remembering Babylon (1993) woman,_______.
♦ The Great world (1990) (a) St. Agne (b) St. Theresa
♦ The conversations at Curlow Creek (1996) (c) St. Joan (d) St. Carmel
♦ An Imaginary life (1978) Ans. (b): Eliot spends a couple of pages on
12. In his Defence of Poesy which of the following Saint Theresa, a Catholic Saint who lived in early 1500s
works does Sidney commend as good examples in Avila, Spain.
of English Poesy? 18. “O, for a draught of vintage! That hath been
I. The Mirror of Magistrates cooled a long age in the deep-delved earth,
II. The Shepherd’s Calendar Tasting of Flora and the country green,
III. Lament for the Makers Dance, and Provencal song, and sunburnt
IV. Ballad of Scottish King mirth!”
The right combination according to the code is : The above description is an example of
(a) I & II (b) I and IV (a) Paronomasia (b) Synaesthesia
(c) I and II (d) II And III (c) Aphaeresis (d) Synecdoche
Ans. (c) : Ans. (b) : Famous lines of John Keats 'Ode to a
♦ The Mirror of Magistrate (1559) - is a collection of Nightingale' is the best example of Synaesthesia.
english poems from the Tudor period by various Synaesthesia is a figure of speech in which one sense is
authors. described using terms for another.
♦ Shephard’s Calendar (1579) – Spenser’s first major 19. The term, “poetic justice”, to designate the idea
poetic work. that the good are rewarded and the evil
13. Who among the following dismissed Ulysses as punished was devised by
“a misfire”? (a) Aristotle (b) John Dryden
(a) Virginia Woolf (b) Wyndham Lewis (c) Thomas Rhymer (d) Ben Jonson
(c) E.M. Forster (d) D.H. Lawrence Ans. (c): Thomas Rymer Coined the phrase in The
Ans. (a) : Virginia Woolf dismissed Ulysses as a Tragedies of the Last Age Considered (1678) to
Misfire. describe a work should inspire proper moral behaviour.
UGC NET/JRF English Paper II, January 2017 234 YCT
20. ______is the producer of the first complete III. Gerald Manley Hopkins
printed English Bible. IV. John Millais
(a) Jerome (b) William Tyndale The right combination according to the code is :
(c) Miles Coverdale (d) Bede (a) II and III (b) I and IV
Ans. (c): The Coverdale Bible, compiled by (c) I and III (d) II and IV
Myles Coverdale and published in 1535, was the first Ans. (b) : “The Pre-Raphaelite” Brothershood (later
complete modern english translation of the Bible. known as the Pre-Raphaelites) was a group of english
21. In The Fall of Hyperion: A Dream Keats sees a painters, poets & critics, founded in 1848 by
ladder leading upwards and is addressed by a William Holman Hunt, John Everett Millais and Dante
prophetess in the following words: “None can Gabriel Rossetti.
usurp this height …../But those to whom the 26. The seven deadly sins are sought to be
miseries of the world/Are misery, and will not portrayed in Chaucer’s Canterbury Tales.
let them rest.” Who is the prophetess? Which of the following sins is not covered by
(a) Urania (b) Moneta Chucher?
(c) Melete (d) Mneme I. Jealousy II. Envy
Ans. (b) : Keats composed The Fall of Hyperion by III. Lust IV. Homicide
reworking, expanding and personally narrating lines The right combination according to the code is
from his earlies fragmented Hyperion. Prophetess (a) I & II (b) I & III
Moneta challenged the poet to climb the stairs. (c) I & IV (d) III & IV
22. Virginia Woolf’s To the Lighthouse has a Ans. (c) : The seven deadly sins include:
tripartite structure. The three parts are named ♦ Lust – intense desire for sexual bliss.
the following EXCEPT: ♦ Envy – the desire for what other posses.
(a) The Sky (b) The Window ♦ Pride – a belief in one’s self that Nashes van extream
(c) Time Passes (d) The Lighthouse level.
Ans. (a) : To the Lighthouse (1927) centers on the ♦ Sloth – laziness.
Ramasays and their visits to the ‘Tsle of Skye’ in ♦ Wrath – intense anger with the desire to do harm.
Scotland between 19-10-1920. The three parts are:
♦ Avarice – greed
Part I - The Window
♦ Gluttony – consuming more that one needs.
Part II - Time Passes
Part III - The Lighthouse 27. Richardson’s Pamela had its origin in
(a) the real case of a woman born to lower-
23. Which novel by Patrick White is based on the
middle class parents
story of Ludwig Leichhardt, the Prussian
(b) an elementary letter-writing manual
naturalist who explored Australia in the mid-
1840s, which White’s fictional hero says when (c) the general plight of English woken
asked about navigation – “The Map? I will first (d) the suggestion of a friend to defend middle-
make it”? class values
(a) The Tree of Man (b) Voss Ans. (b) : Richardson’s Pamela Lad its origin in an
(c) Riders in the Chariot (d) The Solid Mandala elementary letter writing manual, published in 1740.
Ans. (b) : Voss (1957) is the fifth published novel of 28. The Medall, a poem written by John Dryden in
Patrick White. It is based upon the life of the 19th 1681, is sub-titled
century Prussian explorer Ludwig Leichhasdt who (a) A Satire against Sedition
disappeared whilst on an expedition into the Australian (b) A Satire against Tyranny
outback. (c) A Satire against Greed
24. Who among the following is not a character in (d) A Satire against Apostasy
William Golding’s Lord of the Files? Ans. (a) : The Medall: A satire against sedition is
(a) Ralph (b) Piggy published in 1681. The poem was written by England’s
(c) Peter (d) Jack first poet Laureate (1668), John Dryden.
Ans. (c) : Lord of the flies (1954) by nobel prize winner 29. “Full fathom five thy father lies” is an example
Golding. The book focuses on a group of British boys of
stranded on an uninhabited is land. (a) assonance (b) alliteration
25. Dante Gabriel Rossetti founded the Pre- (c) apostrophe (d) enjambment
Raphaelite Brotherhood which included Ans. (b) : Alliteration – The occurance of the same
I. Holman Hunt letter or sound at the beginning of adjacent or closely
II. Arthur Hugh Clough connected words.

UGC NET/JRF English Paper II, January 2017 235 YCT


30. What is trochee? 36. Which play by Tom Stoppard has a play within
(a) A two syllable foot of verse with two heavy the play?
stresses (a) Enter a Free Man
(b) A two syllable foot of verse in which the (b) The Real Inspector Hound
stress falls on the first syllable (c) Jumpers
(d) Night and Day
(c) Three successive heavy stresses
Ans. (b) : ‘The Real Inspector Hound’ is a short,
(d) A six line stanza in which the rhyme sounds
one-act play. The plot follows two theatre critics named
are all identical Moon & Birdboot who are watching a ludicrous of a
Ans. (b) : A two syllable foot of verse in which the country my house.
stress falls on the first syllable is known as trochee. 37. Which of the following is not true of free verse?
31. Keat’s “La Belle Dame Sans Merci” combines (a) Characterised by short, irregular lines.
two poetic forms (b) No rhyme pattern
I. Lyric II. Dramatic Monologue (c) Written in iambic pentameter
III. Ballad IV. Sonnet (d) A dependencein in the effective and more
intense use of pauses
The right combination according to the code is :
Ans. (c): Free Verse is a poetry that doesn’t rhyme or
(a) II and III (b) I and IV
have a regular rhyth.
(c) I and III (d) II and IV
38. James Thomson’long poem, The Seasons,
Ans. (c) : “La Belle Dame Sans Merci” written in 1819 revised and expanded all his life, began in the
by Johan Keats in ABCB Rhyming Scheme. first instance as a poem entitled.
32. _____narrator highlights the problem of (a) Spring (b) Summer
narrative authority. (c) Winter (d) Autumn
(a) First person (b) Self-conscious Ans. (c) : The Season is a series of four poems by
(c) Third person (d) Participant Scottish author James Jhomson. The first part, “Winter”
was published 1726 & complete poem cycle appeared in
Ans. (b) : Self- conscious narrator highlights the 1730.
problem of narrative authority. 39. Two cantos from the seventh book of The
33. Who among the following modern writers is Faerie Queene appeared posthumously. They
associated with the quote, “Only connect”? are known as
(a) D.H. Lawrence (b) Virginia Woolf (a) Mutability cantos (b) Friendship cantos
(c) James Joyce (d) E.M. Forster (c) Justice cantos (d) Courtesy cantos
Ans. (d) : E.M. Foster is associated with the quote, Ans. (a) : Facrie Queen is an english Epic poem by
Edmund Spenser Book I to III were first published in
“Only Connect”. 1590, & then re-published in 1596 together with book
34. Which of the following images does not figure IV to VI.
in Auden’s Musee des Beaux Arts”. 40. Foucault believes that the facts of history will
(a) a boy falling out of the sky protect us from
(b) children ….skating on a pond at the edge of (a) repeating mistakes (b) totalitarianism
wood (c) deconstructionism (d) historicism
(c) ranches of isolation and the busy griefs Ans. (d) : New Historicism is a form literary theory
(d) the dogs go on with their doggy life theory whose goal is to understand intellectual history
through literature, & literature through its cultural
Ans. (c) : “Musee des Beaux Arts” means Museum of context, which follows the 1950s field of history of
fine arts, first published under the little “Palais des ideas and refers to itself as a form of “Cultural Poetics”.
beauxs arts” which means ‘Palace of Fine Arts’. 41. What is the occupation of Max’s son, Lenny, in
35. Feste is a clown in Harold Pinter’s The Home Coming?
(a) Twelfth Night (a) boxer (b) butcher
(b) As You Like It (c) pimp (d) cab driver
(c) The Taming of the Shrew Ans. (c) : The Homecoming is a two-act play written in
(d) Much Ado About Nothing 1964, and was first published in 1965.
42. Which Byron poem begins in the following
Ans. (a) : Twelfth Night, or what you will is a comedy
manner: “I want a hero: an uncommon want,
by Shakespeare believed to have been written around when every year and month sends forth a new
16.01.02. life?
UGC NET/JRF English Paper II, January 2017 236 YCT
(a) Beppo Ans. (c) :
(b) Children Harold’s Pilgrimage Pip – Great expectation
(c) Don Juan Causaubon – Middlemarch
(d) The Vision of Judgement Beck Sharp – Vanity fair
Ans. (c) : ‘Don Juan’ is a satiric poem by Lord Byron, Heathcliff – Wuthering Heights
based on the legend of Don Juan which Byron reverses, 47. Samuel Johnson’s Lives of the English Poets
portraying Juan not as a womaniser but as some one combines the following except
easily seduced by women. It is a variation on the epic (a) analytical criticism (b) literary history
form. (c) personal biography (d) Socratic dialogue
43. In the second ending of John Fowles’s The Ans. (d): Lives of the English Poets is a work
French Lieutenant’s Woman Charles comprising short biographies and critical appraisals of
Smithson’s lawyer finds that Sarah has been 52 poets, most of whom lived during the 18th century.
living in the house of 48. Which two works of JM Coetzee won Booker
(a) William Morris Prize on two occasions?
(b) William Holman Hunt I. In the Heart of the country
(c) D.G. Rossetti II. Life and Times of Michael K.
(d) James Collinson III. Disgrace
Ans. (c) The French Lieutenant’s Woman is a 1969 IV. Waiting for the Barbarians
post modern historical fiction novel. It was his 3rd The right combination according to the code is :
published novel, The collector (1963) and The Magus (a) II and III (b) II and IV
(1965). (c) III and IV (d) I and III
44. In 1962 William Congreve published Incognita, Ans. (a) : Life & Times of Michael K is a 1833, won
a work of fiction which is dubbed a ‘novel’ on Booker Prize in 1983. Disgrace is published in 1999 &
its title-page. What is the sub-titlte. won the Booker Prize. The writer was also awarded the
(a) Love and Duty Reconcil’d Nobel Prize in literature four years after its publication.
(b) Beauty in Distrees 49. Who among the following Greek Philosophers
(c) Vitrue Rewarded has a bearing on the composition of Shelley’s
(d) Love in Excess Adonais”?
Ans. (a) : Incognita Or, Love & Duty Reconcil’d Novel (a) Miletus (b) Socrates
is the full title of the book written by (c) Plato (d) Aristotle
William Congreue. Ans. (c) :Adonais : An Elegy on the Death of John
45. In “Tradition and the Individual Talent” T.S Keats, is a pastoral elegy written for keats in 1821.
Eliot use the analogy of the catalyst to elucidate 50. Match the author with the work :
his theory of impersonal poetry. He cites the A. John Locke I. A short View of the
example of a filament of platinum and, in the Immorality and Profanity
poetic process this is equivalent to of the Stage
(a) the language of the poet B.William Dampier II. Two Treatises on
(b) the mind of the poet Government
(c) the soul of the poet C. Jeremy Collier III. A Short View of Tragedy
(d) the life of the poet D. Thomas Rhymer IV. Voyages
Ans. (b) : In the poetic process this is equivalent to the A B C D
mind of the poet. (a) II I IV III
46. Match the character with the work: (b) III IV I II
A. Pip I. Middlemarch (c) II IV I III
B. Causaubon II. Great Expectations (d) IV III II I
C. Beeky Sharp III. Wuthering Heights Ans. (c) :
D. Heatcliff IV. Vanity Fair John Locke – Two Trusties on Government
The right combination according to the code is: (1689)
I II III IV William Dampier – Voyages (1697)
(a) B C D A Jeremy Collier – A short view of the
(b) D A C B immorality & profanity of the
(c) B A D C stage (1698)
(d) C B A D Thomas Rhymer – A short view of Tragedy (1693)
UGC NET/JRF English Paper II, January 2017 237 YCT
UGC NET/JRF Exam, November-2017
ENGLISH
Solved Paper-III
Note: This paper contains seventy five (75) objective Ans : (a) A Handful of Dust (1934), is author’s early
type questions of two (2) marks each. satirical comic novel for which he became famous in
All questions are compulsory. the preWorld War II years.
1. This Byron work revolves around a wife whose 6. The Oxford English Dictionary was published
husband is presumed lost at sea and she takes a in twelve volumes with its current title in the
lover in his absence. Everybody behaves year:
agreeably on the husband's return. Byron's (a) 1928 (b) 1930
technical skills in verse in in display here as the (c) 1933 (d) 1915
work counterpoints the colloquial and the
Ans : (c) In 1928 it was published with the same name
formal. Identify the work:
‘The Oxford English Dictionary’ but only 10 volumes,
(a) Manfred (b) Don Juan 12 volumes appeared with the same name again in 1933
(c) Beppo (d) The Bride of Abydos with one volume supplementary.
Ans : (c) Beppo : A Venetian Story written in 1817. It 7. The Life and Opinions of Tristram Shandy,
is Byron’s first attempt at writing using the Italian Gentleman is notorious for its many
Ottava Rima metre. The poem published in 1818, it digressions across nine volumes and its failure
comparison the English & Italian morals. to deliver a complete autobiography. In which
2. Who is the author of the poem, "Our volume does Tristram Shandy finally recount
Casuarina Tree"? his birth?
(a) Sarojini Naidu (b) Toru Dutt (a) Volume III (b) Volume V
(c) Rabindranath Tagore (d) Kamala Das (c) Volume VIII (d) Volume IX
Ans : (b) The poem was published in 1881, composed Ans : (a) The Life and Opinions of Tristram Shandy,
by Toru Dutt. It is a perfect example of craftsmanship. Gentleman is a novel by Laurence Sterne. It was
3. In this Jacobean play the Black King and his published in Nine Volumes, the first two appearing in
men, representing Spain and the Jesuits, are 1759, and seven others following over the next seven
checkmated by the White Knight, Prince years.
Charles. This political satire drew crowds to 8. Miguel de Cervantes’s inimitable Don Quixote,
the Globe Theatre until the Spanish foreshadows metafictional moorings when the
ambassador protested and James I suppressed novelist.
the play. (A) says that the first chapters of the narrative are
Identify the play: recreated from the Archive of La Mancha
(a) The Wonderful Yeare (B) says that it is a faithful rendering of a Catalan
(b) A Game at Chess text in Spanish
(c) A King and No King (C) says that part of it has been translated from
(d) The Knight of the Burning Pestle the Arabic by the Moorish author Cide
Hamete Benengeli
Ans : (b) A Game at Chess is a comi satirical play by
(D) says that he is rewriting the history of a
Thomas Middleton, first staged in August 1624 by the
medieval knight altering the heroic vein with
Kings Men at the Globe Theatre, in August 1624.
a farcical mode
4. Frederic Jameson associated postmodern The right combination according to the code is :
culture with_____capitalism. (a) (A) and (B) (b) (B) and (C)
(a) Market (b) Monopoly (c) (A) and (C) (d) (B) and (D)
(c) Imperialist (d) Multinational
Ans : (c)
Ans : (d) His famous book, postmodernism, or the
♦ Says that the first chapters of the narrative are
cultural logic of late capitalism (1991), Frederic secreated from the Archive of La Mancha.
associated postmodern culture with multinational
capitalism. ♦ Says that part of it has been translated from the
Arabic by the Moorish author Cide Hamete
5. Early in Evelyn Waugh's A handful of Dust, Benegeli.
while Tony and his young son, John Andrew,
walk to the church, John tells his father a story 9. In his theory of Mimesis, Plato says that all art
he has heard from the stable manager, Ben is mimetic by nature; art is an imitation of life.
about a mule "who had drunk his company's To argue his case he gives the example of a :
rum ration" in the First World War and (a) cloud (b) chair
subsequently died. What is the mule named? (c) tree (d) river
(a) Peppermint (b) Dopey Ans : (b) In his theory of Mimesis, Plato says that all
(c) Dynamo (d) Pookey art is mimetic by nature; art is an imitation of life.
UGC NET/JRF English Paper III, November 2017 238 YCT
10. The translation of Geeta into English in 1784 14. Christina Rossetti’s “Goblin Market”, a rare
called Bhagvit-Geeta marked, in William blend of allegory and fairytale world presents
Jones’s opinion, an “event that made it possible the story of two sisters, Laura and Lizzle.
for the first time to have a reliable impression Which of the following is NOT true about the
of Indian Literature”. Who was the translator? enhanted world that the poem unravels?
(a) Charles Wilkins (b) H.J. Colebrooke (a) Laura busy fruits from the goblins in
(c) Rammohan Roy (d) Nathaniel Halhed exchange of her “golden lock” of hair and a
Ans : (a) Charles Wilkins, was an English typographer “tear more rare than pearl”
& Orientalist, and founding member of the Asiatic (b) Jeanie, a girl who ate the goblins fruits,
Society. “pined away” and “sought them by night and
11. One of the plays among the following contains day”
the characters Coll, Gib, Dan and Mak. (c) Laura, who goes to the market again, does
Identify the play: not see the goblins but hears only “shrill cry
(a) Everyman piercing the air”
(b) The Castle of Perseverance (d) Laura’s hair “grew thin grey” and she wanes
(c) The Second Shepherd’s Play like the full moon to “swift decay”
(d) The Marshals Ans : (c) Goblin Market composed in April 1859 &
Ans : (c) The Second Shepherd’s Play is a famous published in 1862 is a narrative poem by
medieval mystery play which is contained in the Chritina Rossetti.
manuscript. 15. In which of these prisons is Defoe’s character,
12. Tereza, in Milan Kundera’s novel the Moll Flanders born?
Unbearable Lightness of Being, troubled by (a) Gatehouse (b) King’s Bench
Tomas’s promiscuity, falls an easy prey to (c) Newgate (d) Ludgate
jealousy, fear and nightmares. Which of the Ans : (c) The Fortunes & Misfortunes of Moll Flanders
following are the terrible dreams she has? is first published in 1722. It purports to be the true
(A) She dreams of cats attacking her account of the life of the eponymous Moll, detailing her
(B) She dreams of wolves attacking her exploits from birth until old age.
(C) She dreams that she is dead and buried in a 16. In which poem does Judith Wright lament the
common grave where she lies with the erasure of native culture in the following lines?
corpses of strangers “The song is gone; the dance
(D) She dreams that she is dead, stripped of her Is secret with the dancers in the earth,
clothes and plagued by other naked corpses.
The ritual useless, and the tribal story
The right combination according to the
code is: Lost in an alien tale”.
(a) (A) and (C) (b) (A) and (D) (a) “The Five Senses” (b) “Legend”
(c) (B) and (C) (d) (B) and (D) (c) “Bullocky” (d) “Bora Ring”
Ans : (b) The Unbearable Lightness of Being is a 1984 Ans : (d) Bora Ring is a reflective imagist poem
novel by Milan Kundera, about two women, too men, a describes the guilt felt by early Australian settlers of the
dog & their lives in the 1968 Prague Spring period of genocide committed against the indigenous.
Czechoslovak history. 17. Years before, Winston Smith, the protagonist
13. The opening lines of Wordsworth’s of George Orwell’s dystopia, Nineteen Eighty
“Immortality Ode”: Four got an evidence of the party’s dishonesty,
“There was a time when meadow, grove and What is it?
stream. The earth, and every common sight, (a) Emmanuel Goldstein’s confession that he is a
To me did seem party operative; not an enemy of the party.
Apparelled in celestial light (b) O Brien’s diary entry hinting at the non-
The glory and freshness of a dream”, existence of Big Brother.
closely resembles Coleridge’s lines: (c) A photograph which proves that some citizen
accused of a crime was out of the country
“There was a time when earth, and sea, and
skies, The bright green vale, and the forest’s while it was committed.
dark recess, With all things, lay before mine (d) A colleague’s revelation that the Inner Party
eyes In steady loveliness” members have systematically destroyed all
Identify the Coleridge poem: historical documents and created false
(a) “Fears in Solitude” documents.
(b) “The Mad Monk” Ans : (c) A Photograph which proves that some citizen
(c) “To William Wordsworth” accused of a crime was out of the country while it was
(d) “Dejection : An Ode” committed.
Ans : (b) The Mad Monk (1800) reflects romanticism 18. The Indian Queen is:
in its gothic, exoctic and Preternatural dispositions. The (a) a heroic tragedy in rhymed couplets by John
poem is based on poem “Anselmo”: The Hermit of the Dryden
Alps” by Mary Robenson (1758-1800). (b) a long poem in free verse by Keki Daruwalla
UGC NET/JRF English Paper III, November 2017 239 YCT
(c) an autobiography of an Indian princess in 24. Whih of the following is NOT true about
exile Albert Camus’s novel, The Plague?
(d) a fictional account of the Life of Maharani (a) Dr. Rieux describes the phenomenon of
Gayatri Devi dying rats using the metaphors of disease,
Ans : (a) The Indian Queen is a play by Sir Robert especially the bubonic plague.
Howard, written in collaboration with John Dryden, his (b) Paneloux interprets the plague in his first
sister’s husband. It was first performed in 1664. sermon as a sign of the Apocalypse.
19. In J.M. Coetzee’s Disgrace David Laurie is (c) M. Michel is the first victim of the plague.
working on an opera on the life of one of the (d) Tarrou thinks that the plague symbolizes
Romantic poets. Who is the poet? human indifference
(a) Blake (b) Shelley Ans : (b) The Plague is a novel by Albert Camus,
(c) Byron (d) Coleridge published in 1947 that tells the story of a plague
Ans : (c) ‘Disgrace’ is a novel by J.M. Coetzee, sweeping the French city of Oran.
published in 1999. It won the Booker Prize. The writer 25. John Lydgate begins his Siege of Thebes with a
ws also awarded the Nobel Prize in Literature four years prologue of 176 lines in which he imagines
after its publication. himself joining Chaucer’s pilgrims in
20. Assertion (a): There is no unity or absolute Canterbury, where he speaks with the Host
source of the myth. and agrees to tell the first tale on homeward
Reason (R): The focus or the source of the journey. The story that Lydgate tells as the
myth are always shadows and virtualities pilgrims depart from Canterbury is meant to
which are elusive, unactualizable, and be companion piece to :
nonexistent in the first place. Any search for (a) The Pardoner’s Tale
the discursive unity in the myth is, therefore, (b) The Wife of Bath’s Tale
misplaced.
(c) The Knight’s Tale
In the context of the above statements :
(a) Both (a) and (R) are true and (R) is the (d) The Miller’s Tale
correct explanation of (a) Ans : (c) Siege of Thebes is a 4716 line poem written
(b) Both (a) and (R) are true but (R) is not the by John Lydgate between 1420 and 1422.
correct explanation of (a) 26. Stephen Krashen’s theory of second language
(c) (a) is true but (R) is false acquisition consists of six main hypotheses.
(d) (a) is false, but (R) is true Which of the following is NOT one of them?
Ans : (a) In the context of the above given statements, (a) The Input Hypothesis
both assertion and reason are correct because there is no (b) The Affective Filter Hypothesis
absolute source of the myth. Myths include an (c) The Monitor Hypothesis
explanation for how something came to be in the world (d) The Writing Hypothesis
and it revolves around the story of Gods and Goddesses.
It may be accepted by some people but not by everyone. Ans : (d) According to Krashen there are two
independent system of second language performance:
Hence, option (a) will be correct answer.
the acquired system and the learned system.
21. Which of the following landscapes of England
figures prominently in the poetry of Ted 27. Among Derek Walcott’s plays, which one is an
Hughes? exploration of colonial relationships through
(a) Cornish cliffs (b) Dorset moors the Robinson Crusoe story?
(c) Yorkshire moors (d) Chesil Beach (a) Pantomime
(b) Dream on Monkey Mountain
Ans : (c) ‘Chesil Beach’ is related to Mc Ewan
‘Cornish Cliffs’is a poem by John Betjeman. (c) Ti-Jean and His Brothers
(d) The Charlatan
22. The title of M.C. Chagla’s autobiography is:
(a) Memoirs of my Working Life Ans : (a) Pantomime (1978), an exploration of colonial
(b) Without Fear or Favour relationships through the Robinson Crusoe story.
(c) Roses in December 28. ‘Anti-foundationalism’ holds that:
(d) The Pen as my sword (a) Every theory poses different questions and,
Ans : (c) Mahommedali Currim Chagla (1900-1981) an therefore, what counts as ‘fact’ and ‘truth’
Indian Jurist diplomat, and Cabinet Minister. In 1973 differs in every case.
Chagla published his autobiography Chagla published (b) All truth claims can be judged true or false,
his autobiography Roses in December. usually against empirical facts.
23. Who/Which among the following gave the (c) Causal statements about the relationship
expression, “a leopard can’t change its sports,” between dependent and independent
to English language? variables can be made.
(a) The King James Bible (b) Geoffrey Chaucer (d) Truth is the foundation of all representational
(c) Shakespeare (d) The Royal Society experience.
Ans : (a) A Leopard can’t change its spots-Bloomsbury Ans : (a) Anti-foundationalism holds that every poses
interpretation A person cann’t change who & they are different questions and, therefore, what counts as ‘fact’
(their character), no matter how hard they try. and ‘truth’ differs in every case.
UGC NET/JRF English Paper III, November 2017 240 YCT
29. The interaction hypothesis is a theory of second Ans : (a) The Pilgrim’s Progress from this world to that
language acquisition which states that the which is to Come is a 1678 Christian allegory written
development of language proficiency is by John Bunyan.
promoted by face-to-face interaction and
communication. The idea is usually credited to: 34. In which play by Eugene lonesco do you find
(a) David Nunan (b) Michael Long the grotesque image of the leg of a corpse
(c) Alastair Pennycook (d) Claire Kramsch thrusting onto the stage, and which begins to
Ans : (b) The interaction hypothesis is a theory of grow larger as the play progresses in menacing
second-language acquisition which states that the manner?
development of language proficiency is promoted by (a) The Bald Sopramo
face- to- face interaction & communication. The idea (b) Amede or How to Get Rid of It
existed in the 1980s but is usually credited to (c) Exit the King
Michael Long for his 1996. The role of the linguistic (d) The Lesson
environment in second language acquisition. There are
two forms : The “strong” form and the “weak” form. Ans : (b) Amede, or How to Get Rid of It, is a play
written by Eugene Jonesco in 1954 based on his erlier
30. In Pinter’s Birthday Party Stanley is terrorised
by two visitors to a seaside boarding house. short story entitled “Oriflamme”.
Identify the two 35. Which of the following characters finds that
(A) McGrath (B) Goldberg complete happiness is elusive and that “while
(C) McCann (D) Robinson you are making the choice of life, you neglect to
The right combination according to the code is: live”?
(a) (A) and (B) (b) (B) and (C) (a) Lovelace in Samuel Richardson’s Clarissa
(c) (A) and (D) (d) (B) and (D) (b) Rasselas in Samuel Johnson’s Rasselas
Ans : (b) The Birthday Party (1957) is the second full- (c) Matthew Bramble in Tobias Smollett’s
length play Harold Pinter. Humphry Clinker
31. Match the phrase to the ode : (d) Harley in Henry Mackenzie’s The Man of
(A) beechen green (i) “Ode on a Grecian Urn” Feeling
(B) gathering swallows (ii) “Ode on Melancholy” Ans : (b) The History of Rasselas, prince of Abissinia
(C) globed peonies (iii) “Ode to a Nightingale originally titled The Prince of Abissinia: A Tale, is an
(D) green altar (iv) “To Autumn” apologue about happiness. The book’s original title was
Code : “The Choice of Life”.
(A) (B) (C) (D) 36. Arrange the following in the chronological
(a) (iii) (ii) (iv) (i) order of publication:
(b) (iv) (ii) (iii) (i) (a) In Memoriam-A Christmas Carol-Men and
(c) (iv) (iii) (ii) (i) Women-Henry Esmond
(d) (iii) (iv) (ii) (i) (b) A Christmas Carol- In Memoriam-Men and
Ans : (d) Women-Henry Esmond
♦ Beechen green – Ode to a Nightingale (c) A Christmas Carol - In Memoriam-Henry
♦ Gathering swallows – To Autumn. Esmond-Men and Women
♦ Globed peonies – Ode on Melancholy (d) In Memoriam - A Christmas Carol –Henry
♦ Green alter – Ode on a Grecian Urn Esmond-Men and Women
32. Which 19th century novelist expressed a wish to Ans : (c)
“exterminate the race” of Indians following the ♦ A Christmas Carol - (1843)
1857 Mutiny in India?
(a) William Makepeace Thackeray ♦ In Memoriam - (1850)
(b) Charles Dickens ♦ Henry Esmond - (1852)
(c) George Eliot ♦ Men & Women - (1855)
(d) Anthony Trollope 37. Which one of Alice Munro’s short stories is
Ans : (b) On the Indian “Mutiny” Dickens wrote to an about the domestic erosions of Alzheimer’s
acquantance: “I wish I were commander in Chief in disease?
India….. I should do my utmost to exterminate the Race (a) “Dear Life”
upon the stain….. (b) “Runaway”
33. The second part of Pilgrim’s Progress deals (c) “The Bear Came Over the Mountain”
with the pilgrimage of Christian’s wife,
Christiana. She has a companion and a guide (d) “Dance of the Happy Shades”
in this journey. Pick out the pair’s names from Ans : (c) Dear Life is a short story collection by
the following list. Canadian writer Alice Munro, published in 2012.
(A) Patience (B) Tenderheart 38. What work begins thus: “It befell in the days of
(C) Mrecy (D) Greatheart Uther Pendragon, when he was king of all
The right combination according to the code is : England, and so reigned, that there was a
(a) (C) and (D) (b) (B) and (C) mighty duke in Cornwall that held war against
(c) (A) and (D) (d) (B) and (D) him long time”?
UGC NET/JRF English Paper III, November 2017 241 YCT
(a) Sir Gawain and the Green Knight Ans : (c) These are lines from a poem by
(b) Le Morte D’ Arthur Stephen Spender on the death of Auden.
(c) Confessio Amantis 44. Allen Ginsberg’s “HowI”, a key work of the
(d) Piers Plowman Beat Movement, was dedicated to _____
Ans : (b) Le Morte d’ Arthur is a reworking of (a) Lucien Carr (b) Carl Solomon
existing tales by Sir Thomas Malory about the (c) Herbert Huncke (d) Jack Kerouac
Legendary King Arthur published on 1485.
Ans : (b) “Howl”, also known as “Howl for
39. _____is the subject of Asif Currimbhoy’s play, Carl Solomon” is a poem written by Allen Ginsberg in
Inquilab. 1954-1955 and published in his 1956 collection Howl
(a) The Naxalite movement and other Poems.
(b) The Freedom movement 45. In his views on the death of Cordelia in King
(c) The Non-Cooperation movement Lear, which is the ground NOT specifically
(d) The Khilafat movement cited by Samuel Johnson?
Ans : (a) Asif Currimbhoy (born 1928) is an Indian (a) It is contrary to the natural ideas of justice
playwright writer in English. Asif is India’s first (b) It is contrary to neoplatonic idea of decorum.
authentic voice in the Theatre Inquilab is published in (c) It is contrary to the hope of the reader
1970.
(d) It is contrary to the faith of chronicles.
40. Tom Stoppard’s play Rosencrantz and
Guildenstern are Dead, being metatheatrical, Ans : (b) It is contrary to neoplatonic idea of decorum.
lays bare the constructed nature of theatrical 46. Which of the following plays by David Hare is
performance. In referring to Hamlet’s end and NOT part of a trilogy of ‘state o the nation’
the Elizabethan stage conditions lacking plays?
curtains one of the characters of Stoppard’s (a) The Absence of War (b) Racing Demon
play says: “No one gets up after death-there is (c) The Power of Yes (d) Murmuring Judges
no applause-there is only silence and some
second hand clothes, and that’s death”. Who Ans : (c) The Power of Yes is a 2009 play by English
makes this statement? playwright David Hare.
(a) Rosencrantz (b) Guildenstern 47. Chimamanda Adichie’s last novel, Americanah
(c) The Player (d) Hamlet (2013) centres on the romantic and existential
Ans : (b) Rosencrantz and Guildenstern Are Dead, struggles of a young Nigerian woman studying
often referred toasjust Rosencrantz and Guildenstern, is in the United States and finding success as a
an absurdist, existenceal tragic comedy by Tom blogger. What is her blogging about?
Stoppard, first staged in 1966. (a) poverty (b) development
41. Who among the following has translated the (c) race (d) religion
classic Malayalam novel, Chemmeen? Ans : (c) Americanah is a 2013 novel by the Nigerian
(a) A.K. Ramanujan (b) Anita Nair author, for which Adichic non the 2013 National Book
(c) Nandini Nopany (d) Gita Krishnankutty Critics Circle Fiction award.
Ans : (b) Chemmeen is a Malayalam novel written by 48. Why does Father Dolan punish Stephen with
Jhakazhi Sivasankara Pillai in 1956. It tells the story of the pandybat in Joyce’s Portrait of the Artist
the relationship between Karuthamma, the daughter of a as a young Man?
Hindu fisherman, and Pareekutti, the son of a Muslim (a) Stephen is talking to another student to get
fish wholesaler. the answer to a Latin proboem.
42. Which Victorian poet is the author of the (b) Stephen is not doing his work because his
following lines? “God himself is the best poet, glasses are broken.
And the Real is His song.” (c) Stephen is looking out f the window towards
(a) Lord Tennyson the infirmary.
(b) Robert Browing (d) Stephen is lost in remembering his mother’s
(c) Mathew Arnold farewell and cannot hear Father Dolan calling
(d) Elizabeth Barrett Browning out his name.
Ans : (d) Elizabeth Barrett Browning was an English Ans : (b) A Portrait of the Artist as Young man is the
poet of the Victorian era, popular in Britain and the first novel by Irish writer James Joyce, published in
United States during her lifetime. 1916.
43. “You are your words. Your listeners see 49. Using a non-linear narrative, this American
Written on your face the poems they hear Like novel explores the psychic damage to a veteran
letters carved in a tree’s bark of world War II and shows how a measure of
The sight and sounds of solitudes endured”. healing is attained through his accepted of
These are lines from a poem by______on the Laguna myths and rituals. Identify the wor
death of______ (a) Dred (b) Beloved
(a) T.S. Eliot; Robert Frost (c) Ceremony (d) End Zone
(b) Siegfried Sassoon; Wilfred Owen
Ans : (c) Ceremony is a novel by Native American
(c) Stephen Spender; W.H. Auden
(d) Dylan Thomas; Robert Bridges writer Lestie Marmon Silko, published in March 1977.

UGC NET/JRF English Paper III, November 2017 242 YCT


50. What illusion does Lyuba Ranevsky in Anton (a) The Two Gentlemen of Verona
Chekhov’s play The Cherry Orchard have as (b) As you Like It
she looks at the orchard? (c) Twelfth Night
(a) She sees it gleaming with a bluish aura. (d) A Midsummer Night’s Dream
(b) She sees her dead mother walking through Ans : (a) The Two Gentlemen of Verona is a comedy
the orchard by Shakespeare, believed to have been written between
(c) She sees it full of ripe fruits without a trace 1589-93 and originally published in 1623.
of leaves 55. For Coleridge, our power to perceive symbols
(d) She sees her childhood friends playing in the gleaned from the world about us is related to
orchard the category of:
Ans : (b) The Cherry Orchard is the last play by (a) primary imagination (b) secondry imagination
Russian playwright Anton Chekhov. Written in 1903, it (c) fancy (d) intuition
was first published by Znaniye. Ans : (a) For Coleridge, our power to perceive symbols
51. From which source did Swift get the idea of gleaned from the world about us is related to the
writing “Verses on the Death of Dr. Swift”? category of primary imagination.
(a) In a conversation with John Gay 56. After independence, although English was not
(b) After a reading of a maxim by la an Indian language, it was accorded the status
Rochefoucauld of an:
(c) While taking a walk near Dublin’s (a) Additional language
St. James’s graveyard (b) Ancilliary language
(d) After reading Richard Burton’s Anatomy of (c) Associate language
Melancholy (d) Administrtive language
Ans : (b) Swift state- Ans : (c) Associate language.
This I say, because yor are so hardly as to tell me of 57. Which English journal announced that it was
your intentions to write maxims in oppositions to “principally intended for the use of Politick
Rochefoucauld, who is my favourite, because I found Persons who are so publick-spirited as to
my whole character in him. neglect their own Affairs to look into
52. Two of the following words were borrowed Transactions of State” but failed to live up to
from French after the Norman Conquest. this and amused readers with “accounts of
(A) mutton (B) pork Gallantry, Pleasure and Entertainment”?
(C) sheep (D) swine (a) The Spectator (b) The Tatler
The right combination according to the code is: (c) The Daily Courant (d) The Review
(a) (A) and (B) (b) (A) and (C) Ans : (b) The Tatler was a British literary and society
(c) (B) and (D) (d) (C) and (D) journal begun by Richard Stelle in 1709 and published
Ans : (a) Mutton & Pork were the names borrowed for two year.
from French. 58. The grammar-translation method of language
teaching does NOT include:
53. Which of the following is NOT true regarding
the Oresteia trilogy by Aeschylus? (a) focus on grammar rules
(a) Cassandra, cursed by Apollo predicts the (b) vocabulary memorization
death of Agamemnon, though her prophecy is (c) inductive teaching
ignored. (d) focus on written language
(b) Aegisthus’s vengeful feelings for Ans : (c) The grammar-translation method of language
Agamemnon results from their rivalry for the teaching does focus on grammar rules.
hand of Clytemnestra. 59. Who is the narrator in Kamala Markandaya’s
(c) Orestes, who has come back with the Nectar in a Sieve?
intention of murdering Clytemnestra (a) Premala (b) Saroja
unexpectedly meets her, and pretending to be (c) Rukmani (d) Mira
a stranger, tells her that Orestes is dead. Ans : (c) Nectar in a Sieve is a 1954 novel by Kamala
(d) Orestes, pursued by the Furies, flees from Markandaya’s. The novel is set in India during a period
them when they fall asleep. Then, of intense urban development and is the chronicle of the
Clytemnestra’s ghost appears to wake them marriage.
up. 60. How would a New Historicist critic interpret
Ans : (b) Aegisthus’s vengeful feelings for Agamemnon Derrida’s statement, “there is nothing outside
results from their rivalry for the hand of Clytemnestra is the text”?
not true. (a) historicist critics should restrict their
54. The first instance of female cross-dressing with attention to a culture’s literary productions,
the disconcerting nuances of a boy actor all other data is irrelevant to the critic’s task
dressing as a boy while playing the role of a (b) language conditions the way we see the
woman in the dramatic world of Shakespeare world, and there is no reality beyond the
occurs in ______. ‘prison house’ of language
UGC NET/JRF English Paper III, November 2017 243 YCT
(c) there is no meani outside of textual meaning Ans : (b) Middle march, A study of Provincial Life is a
(contrary to the mimeticist’s position) novel by the English author Grorge Eliot, first published
(d) “literature” encompasses all cultural artifacts in eight installment during 1871-72.
and all the values, power relations, and ways 66. William Blake has a rare elan to provide telling
of seeing reflected in those artifacts; there is images in arresting phrases. Match the phrases
nothing outside of the “text” broadly with the poems they belong to:
conceived (A) “mind forg’dmanacles” (i) “The Tyger”
Ans : (d) Literature encompasses all cultural artifacts (B) “eternal winter” (ii) “The Sick Rose”
and all the values, power relations, and ways of seeing (C) “fearful symmetry” (iii) “London”
reflected in those artifacts; there is nothing outside of (D) “crimson joy” (iv) “Holy Thursday”
the “text” broadly conceived. Code :
61. Pick out two Austen heronies from the (A) (B) (C) (D)
following list who are right-minded but (a) (ii) (iv) (i) (iii)
neglected in the beginning but gradually are (b) (iii) (i) (iv) (ii)
acknowledged to be correct by characters who (c) (iii) (iv) (i) (ii)
have previously looked down on them.
(d) (iv) (i) (ii) (iii)
(A) Elizabeth Bennet
(B) Fanny Price Ans : (c)
(C) Emma Woodhouse ♦ mind forg’dmanacles - London
(D) Anne Elliot ♦ eternal winter - Holy Thursday
The right combination according to the code is: ♦ fearful symmetry - The Tyger
(a) (A) and (C) (b) (B) and (D) ♦ crimson joy - The Sick Rose
(c) (C) and (D) (d) (A) and (D) 67. In the debate between the two birds in the
Ans : (b) Fanny Price is the heronies in Mansfield Park Middle English poem The Owl and the
(1814). Anne Elliot is the protagonist of Persuasion Nightingale who acts as the arbiter?
(1818). (a) Master Henry of Shrewsbury
62. The variety of English used between non-native (b) Master William of Hereford
speakers who do not share a first language is (c) Master Freeman of Stamford
called : (d) Master Nicholas of Guildford
(a) English for specific purposes Ans : (d) The Owl and the Nightingale is a twelfth-or-
(b) English for basic purposes thirteenth century middle English poem.
(c) English as a lingua Franca 68. In the first scene in which Goethe’s Faust
(d) English as a language tool appears he is dejected by the study of
Ans : (c) English as a lingua Franca Philosophy, Law, Medicine and Theology,
turns to Magic art to acquire infinite
63. Identify the story for which E.M. Froster wrote knowledge. But he fails and in desperation
the libretto for its opera version: attempts to commit suicide, but refrains at the
(a) Heart of Darkness final moment. What prevents Faust from
(b) The Man Who Would Be the King committing suicide?
(c) Billy Budd (a) The intervention of archangel Gabriel
(d) Death in Venice (b) His attendant Wagner persuades him to
Ans : (c) Billy Budd, Sailor is the final novel by revoke the decision
American writer Human Melville, first published (c) The chiming of the bells announcing Easter
postjumously in London in 1924. festivities
64. Who, among the following Prem Chand (d) Mephistopheles appears and offers to initiate
translators has NOT translated Godan? him into magic art
(a) Jai Ratan Ans : (c) The chiming of the bells announcing Easter
(b) P. Lal festivities.
(c) Gordon C. Roadarmel 69. Which novel by Joseph Conrad Presents a
(d) Christopher R. King young captain who like Coleridge’s Ancient
Mariner is haunted by the “vision of a ship
Ans : (d) Christopher R. King drifting in calm and swinging in light airs, with
65. “When Fred got into debt, it always seemed to all the crew dying slowly about her decks” and
him highly probable that somethings or other who feels “the sickness of my soul…..the weight
he did not necessarily conceive what-would of my sins…..my sense of unworthiness”?
come to pass enabling him to pay in due time”. (a) Under Western Eyes
Why is Fred Vincy in debt in Middle march? (b) The Shadow Line
(a) He takes out a large loan to enable him to (c) Victory
woo Mary Garth. (d) The Rescue
(b) He is an inveterate gambler Ans : (b) The Shadow Line is a short novel based at sea
(c) He is paying off a blackmailer by Joseph Conrad, it is one of his later works, being
(d) He runs a charity that has got into trouble written from February to December 1915.
UGC NET/JRF English Paper III, November 2017 244 YCT
70. “Our almost-instinct almost true: cover. The acid crumbs of loam, the granular
What will survive of us is love.” pink rock, its igneous veins, the sea-fans
Identify the poem by Philip Larkin that ends of dry moss, the blackish and then the graying
with the above lines: bristles on the back of his neck.
(a) “This Be the Verse” Sometimes he would whistle, sometimes
(b) “An Arundel Tomb” I would. The boring rhythm of doing
(c) “High Windows” Things over and over, carrying
(d) “Next, Please” The wood, drying
Ans : (b) “An Arundel Tomb” is a poem by Philip The dishes. Such minutiae. If’s what
Larkin, written in 1956 and published in 1964 in his Ferrying the sand, grain by grain, from their
collection The Whitsun weddings. tunnels,
71. In the epilogue to Congreve’s Way of the Shurring the leaves in their burrows. He pointed
World there is a warning: Others there are Such things out, and I would look
whose malice we’d prevent, Such, who watch
plays, with scurrilous intent To mark out who at the whorled texture of his square finger, earth
by characters are meant. under
…………………………………… The nail. Why do I remember it as sunnier
These, with false glosses feed their own ill- All he time then, although it more often
naure, And turn to libel, what was meant a rained, and more birdsong?
satire. What does this warning mean? I could hardly wait to get
(a) Critics should not be ill-natured and the hell out of there to
malicious. anywhere else. Perhaps though
(b) Critics should not look for portrait of real boredom is happier. It is for dogs or
people in the play’s characters and remember groundhogs. Now I wouldn’t be bored.
that the play is a social satire. Now I would know too much.
(c) Critics should avoid writing malicious Now I would know.
reviews, lest they be charged with libel.
73. “All those times”-the opening words of the
(d) Critics should try to identify the real-life
equivalent for each character. poem locate the speaker in:
(a) a city suburb
Ans : (b) The Way of the World is a play written by the
English playwright Klilliam Congreve. It premiered in (b) a mountain resort
early March 1700 in the theatre in Lincoln’s Inn Fields (c) a natural environment
in London. (d) a highway motel
72. Which of the following is an elegy on John Ans : (c) A natural environment
Donne’s wife, who died in 1617? 74. Which pair of words best describes the
(a) “Death, be not proud” repetitive tenor of the speaker’s unpretentious
(b) “Thou hast made me” yet oppressive life?
(c) “Holy Sonnet 17” (A) details (B) the car
(d) “At the round earth’s imagined corners” (C) the wood (D) the minutae
Ans : (c) Holy Sonnet 17 is a poet of a series of 19 The right combination according to the code is:
poems, which are most commonly referred as Divins (a) (A) and (B) (b) (A) and (D)
Meditation, Divine Sonnets or Holly Sonnet. (c) (B) and (C) (d) (C) and (D)
Read the following poem and answer questions, Ans : (b) "Details" and "the minutiae" are the words
73 to 75 : that best describe the repetitive tenor of the speaker's
Bored unpretentious yet oppressive life. So the right
Margaret Atwood combination according to the code will be (b) where
All those times I was bored "details" and "the minutae" support answer.
Out of my mind. Holding the log Hence, (b) will be correct answer.
While he sawed it. Holding 75. Which of the following approximates closely a
The string while he measured, boards thematic statement of the poem?
Distances between things, or pounded (a) Dogs or groundhogs lead a better life than
Stakes into the ground for rows and rows men or women
Of lettuces and beets, which I then (bored) (b) Irrespective of the place, the boring rhythm
Weeded. Or sat in the back of doing things over and over in human life
of the car, or sat still in boatds, cannot be escaped
sat, sat, while at the prow, stern, wheel (c) Myopia is the result if you live life in the lap
he drove, steered, paddled. It of nature
wasn’t even boredom, it was looking, (d) Knowledge cures existential boredom
looking hard and up close at the small Ans : (b) Irrespective of the place, the boring rhythm of
details. Myopia. The worn gunwales, doing things over and over in human life cannot be
the intricate twill of the seat escaped.
UGC NET/JRF English Paper III, November 2017 245 YCT
UGC NET/JRF Exam, November-2017
ENGLISH
Solved Paper-II
1. In Frances Burney's novel, Evelina, the 5. Who made the comment that, "All modern
eponymous heroine comes out in society in two American literature comes from one book by
locations. They are: Mark Twain called Huckleberry Finn"?
A. Bath B. Bristol (a) Henry James (b) William Faulkner
C. Leeds D. London (c) Jack London (d) Ernest Hemingway
The right combination according to the code is: Ans : (d) All modern American Literature comes from
(a) A & B (b) B & C one book by Mark Tawin called ‘Huckleberry Finn
(c) A & D (d) B & D Ernest Hemingway famously declared in 1935.
Ans : (d) Evelina or the History of a Young Lady’s 6. The Emblem is a poetic genre containing a
entrance into the world is a novel written by English symbolic picture with a text and a verse
Author Fanny Burney & first published in 1778. exposition popular in the early 17th century.
Who popularised this kind of poetry through
2. Which of the following lines by Shakespeare is
the work Emblems [1635]?
repeated several times in Virginia Woolf's
(a) Robert Southwell (b) Francis Quarles
novel Mrs. Dalloway?
(c) John Davies (d) Joseph Sylvester
(a) "If music be the food of love, play on"
(b) "Fear no more the heat of the sun, Nor the Ans : (b) George Wither’s Emblem Book published in
furious winter's rages" 1635, coincided with the other most famous English
book of emblems by Francis Quarles.
(c) "Those are pearls that were his eyes"
(d) "There is a tide in the affairs of man" 7. Which Byron work begins thus:
"I want a hero : an uncommon want, when
Ans : (b) Mrs. Dalloway is novel by Virginia Woolf
every year and month sends forth a new
that details a day in the life of Clasissa Dalloway, a
one......."?
fictional high-society woman in Post-First World War
(a) Beppo (b) Cain
England, published in 1925.
(c) Manfred (d) Don Juan
3. Identify the important theatres of the
Ans : (d) Don Juan is a satiric poem by Lord Byron,
Elizabethan period:
based on the Legend of Don Juan, which Byron
A. Peacock B. Globe reverses, portraying Juan not as a womaniser but as
C. Swan D. Grand someone easily seduced by women. Byron completed
The right combination according to the code 16 cantos, leaving an unfinished 17th canto before his
is: death in 1824 first published in 1819.
(a) A & B (b) B & C 8. The title of Sir Thomas Browne's famous
(c) B & D (d) A & D treatise, Religio Medici means:
Ans : (b (a) Religion of a Doctor
The Swan – Built in 1595 (b) Religion of Magician
The Globe – Built in 1599 (c) Religion of Divinity
4. In which poem does Matthew Arnold express (d) Religion of Meditation
the dilemma of: Ans : (a) Religio Medici by Sir Thomas Browne is a
"Wandering between two worlds, one dead, spiritual testament & an early psychological self-
The other powerless to be born"? portrait, published in 1643.
(a) "Self - Dependence" 9. Which among the following recent novels is a
(b) "Stanzas from the Grande Chartreuse" retelling of Sophocles's Antigone?
(c) "To a Republican Friend" (a) Kamila Shamsie, Home Fire
(d) "Dover Beach" (b) Fiona Mozley, Elmet
Ans : (b) The poem “Stanzas from the Grande (c) Zadie Smith, Swing Time
Chartreuse” is a philosophical poem of 35 stanzas. The (d) Mohsin Hamid, Exit West
poem is deeply personal, describing Matthew Arnold’s Ans : (a) Home fire is the seventh novel by
own struggle to find a faith that would give his life Kamila Shamise, reimaging Sophocles’s Antigoe in a
meaning. contemporary setting, published in 15th Aug, 2017.

UGC NET/JRF English Paper II, November 2017 246 YCT


10. Identify the two important works of Poul de Ans : (b) ‘The Heresy of Paraphrase’ is the title of a
Man from the following list: chapter in The Welt Wrought Urn, a seminal work of
A. Blindness and Insight the new criticism by Cleanth Brooks.
B. Allegories of Reading 16. Edmund Spenser's Colin Clout's Come Home
C. Theoretical Essays Again is a fine example of:
D. Criticism and Ideology (a) Carpe diem (b) Sonnet sequence
The right combination according to the code is: (c) Georgic poetry (d) Pastoral eclogue
(a) A & B (b) A & C
Ans : (d) Colin Clout’s Come Home Again a pastoral
(c) B & C (d) B & D
poem by Spenser and published in 1595.
Ans : (a)
17. In An Essay of Dramatic Poesy whom does
♦ Blindness and insight – 1983
John Dryden refer to as "the most learned and
♦ Allegories of Reading – 1979 judicious Writer which any Theater ever hea"?
11. Samuel Johnson denounced the metaphysical (a) John Webster (b) Christopher Mrlowe
poets saying, "About the beginning of the (c) Ben Jonson (d) William Shakespeare
seventeenth century appeared a race of writers
that may be termed the metaphysical poets". In Ans : (c) In Essay of Dramatic Poesy Dryden attempts
the biography of which of the following poets in to justify drama as a legitimate form of “Poetry”
his Lives of Poets did Johnson make this comparable to the epic first published in 1668.
remark? 18. This Australian poet was raised in New South
(a) John Dryden (b) Thomas Parnell Wales and grew up in rural Australian
(c) Abraham Cowley (d) Alexander Pope landscape. In 1946 she published her first book
Ans : (c) Lives of the Most Eminent English Poests is a of poems. In 1962, she became cofounder and
work by Samuel Johnson, comprising short biographics president of the wild Life Preservation Society
& critical appraisals of 52 poets. It is arranged, of Queensland and served as its president
approximately, by date of death, & published in 1779. several times thereafter. Identify the poet.
12. The terms of the contract are not disagreeable (a) Dorothy Hewett (b) Nettie Palmer
to me. (c) Judith Wright (d) Amy Witting
The above sentence contains an example of: Ans : (c) Judith Wright (1915-2000) was the author of
(a) Enumeratio (b) Litotes several collections of poetry, including. The moving
(c) Anaphora (d) Metonymy image woman to man the Gateway the two fires, birds
Ans : (b) Litotes is ironic understatement in which an etc.
affirmative is expressed by the negative of its contrary 19. Aphra Behn's Oroonoko is set in_____.
eg. I Shan’t be sorry for I shall be glad. (a) Surinam (b) Abyssinia
13. Who is the author of the following lines? (c) Egypt (d) Assyria
"To see a World in a Grain of Sand Ans : (a) Oroonoko or, the Royal Slave is a short work
And a Heaven in a Wild Flower of prose fiction published in 1688.
Hold Infinity in the palm of your hand
20. Who published the first collected edition of
And Eternity in an hour..."
Gerard Manley Hopins's poems in 1918?
(a) Thomas Gray (b) William Blake
(a) Robert Bridges (b) Coventry Patmore
(c) William Collins (d) William Cowper
(c) John Betjeman (d) Stephen Spender
Ans : (b) The lines are of the poem of Auguries of
innocence by William Blake. It is assumed to have been Ans : (a) Gerard Manley Hopkins (1844-1889) was an
written in 1803, but was not published until 1863. english poet. Two of his major themes were nature &
religion.
14. In Women in Love what is Winifred's pekinese
dog called? 21. Samuel Richardson named his heroine Pamela
(a) Bismarck (b) Looloo after one of the characters in_____.
(c) Lucky (d) Buddy (a) Edmund Spenser's Faerie Queene
Ans : (b) Women in Love (1920) is a novel by British (b) William Shakespeare's Vennus and Adonis
author D.H. Lawrence. It is a sequal to his earlier novel (c) Philp Sidney's Arcadia
The Rainbow. (d) Geoffrey Chaucer's Canterbury Tales
15. Which of the following New Critics put Ans : (c) The countess of Pembroke’s Arcadia is a long
forward the idea of the 'heresy of paraphrase'? prose work. Sidney later expand & revised his work as
(a) Allen Tate (b) Cleanth Brooks Old Arcadia & the New Arcadia. Sidney had written
(c) W.K. Wimsatt (d) Monroe C Beardsley this in the end of 16th century.
UGC NET/JRF English Paper II, November 2017 247 YCT
22. Pinter once admitted that he first became Ans : (b) The Return of the Native is Hardy’s Sixth
aware of the dramatic power of the pause from published novel. It first appeared in the magazine
seeing a popular American comedian. Which Belgravia a publication known for its sensationalism,
one? was presented in 12 monthly installments from January
(a) Bob Hope (b) W.C. Fields to December 1878.
(c) Jack Benny (d) Charlie Chaplin 27. The metrical form of Gower's Confessio
Ans : (c) Jack Benny was an American comedian, Amantis is:
vaudevillian, radio, television and film actor and (a) Iambic pentameter
violinist. (b) Anapestic trimeter
23. Charles Dickens's Bleak House is pointedly (c) Octosyllabic couplets
critical of England's: (d) Trochaic tetrameter
(a) Privy Council (b) Court of Appeal Ans : (c) Confessio Amantis means ‘The Lover’s
(c) Court of Chancery (d) military courts Confession’ is a 33,000 line Middle English poem. It
Ans : (c) Bleak House, first published as a serial was composed at the request of Richard II. It uses the
between March, 1852 & September 1823. Dickens uses confession made by an ageing lover to the Chaplain of
this case to satirise the english judicial system. Venus.
24. Which of the following is NOT true of the ideal 28. What happens to the lock of hair at the end of
state in Thomas More's Utopia? Alexander Pope's The Rape of the Lock?
(a) Personal property, money and vice are (a) It is given back to its rightful owner.
effectively abolished. (b) It is preserved in a monument.
(b) The root causes of crime, ambition and (c) It turns into a star.
political conflict, are eliminated. (d) It is presented to the poet as a token of
(c) There is only one religion guided by the gratitude.
principle of a benevolent Supreme Being. Ans : (c) The Rape of the Lock is a mock-heroic
(d) Its priesthood, which includes some women, narrative poem written by Pope. It is one of the most
is limited in number. commonly cited examples of high burlesque.
Ans : (c) Utopia is a work of fiction & soio-political 29. The Bard. The Iron Lady. The King.
satire by more (1478-1535) published in 1516 in Latin. The above are examples of:
The book is a frame narrative primarily depicting a (a) Anacoluthon (b) Aposiopesis
fictional island society & its religious, social & political (c) Asyndenton (d) Antonomasia
customs. Ans : (d) Antonomasia is the use of a proper name to
25. Which character created by Coleridge makes express a general idea “The Bard Avon” for
the following account of her harrowing William Shakespeare.
experience? 30. Which of the following novels by Margaret
"Five warriors seized me yestermorn, Atwood depicts the historical event of the
Me, even me, a maid forlorn: notorious murders committed in 1843?
They choked my cries with force and fright, (a) The Blind Assassin (b) Alias Grace
And tied me on a palfrey white" (c) Cats Eye (d) Oryx and Crake
(a) Geraldine Ans : (b) Alias Grace is a novel of historical fiction,
(b) Christabel published in 1996.
(c) Christabel's mother 31. Which of the following poems by W.B Yeats
(d) The maid who appeard in Christabel's dream repudiates the sensual world in favour of "the
Ans : (a) Christabel is long narrative poem, in two artifice of eternity"
parts. The first part was reputedly written in 1797, & the (a) "Under Ben Bulben"
second in 1800. It was published in a pamphlet in 1816, (b) "Among School Children"
alongside Kubla Khan. (c) "Sailing to Byzantium"
26. Which novel of Thomas Hardy begins with the (d) "After Long Silence"
sombre description of Egdon Heath? Ans : (c) Sailing to Byzantium is a poem first
(a) Jude the Obscure published in 1928. Yeat’s “Sailing to Byzantium”
(b) The Return of the Native describes the Metaphorical journey of a man pursuing
(c) Far from the Madding Crowd his own vision of eternal life as well as his conception
(d) Under the Greenwood Tree of paradise.
UGC NET/JRF English Paper II, November 2017 248 YCT
32. Which of the following characters in Moby Code :
Dick falls overboard and turns insane as a A B C D
result? (a) iv iii i ii
(a) Pip (b) Queequeg (b) iv ii i iii
(c) Starbuck (d) Tashtego (c) iii iv i ii
Ans : (a) Moby Dick or the Whate is a novel by (d) iv i ii iii
American writer Herman Melville, published in 1851
during the period of American Renaissance. Faulkner Ans : (b)
confessed he wished he had written it himself and ♦ Sexual Politics (1970) by Kate Millet.
Lawrence called it “One of the strangest & most ♦ A Literature of their own (1977) by Elaine Showalter.
wonderful books in the world”, and “the greatest book ♦ Thinking About Women (1968) by Mary Ellman.
of the sea ever written.”
♦ The Laugh of the Medusa (1976) by Helene Cixous.
33. Which of the following poems by Seamus
38. Which of the following historical events does
Heaney is dedicated to the Irish poet Paul
Tennyson's poem "The Charge of the Light
Muldoon? Brigade" describe?
(a) "The Loaning" (b) "The Sandpit" (a) The Battle of Hastings
(c) "A Migration" (d) "Widgeon" (b) The Wars of the Roses
Ans : (d) Widgeon by Seamus Heaney is a short price (c) The Battle of Waterloo
dedicated to fellow Irishman Paul Muldoon (b. 1951). (d) The Crimean War
34. In William Golding's Lord of the Flies which of Ans : (d) The charge of the Light Brigade by Tennyson
the following characters is put to death? was a charge of British light cavalry led by Lord
(a) Piggy (b) Ralph Cardigan against Russian forces during the battle of
(c) Simon (d) Jack Balaclava on 25 October, 1854 in the Crimean War.
Ans : (b) In William Golding's 'Lord of the Flies' Ralph 39. Northrop Frye's influential work, Anatomy of
is the character who is put to death. He never followed Criticism includes, as the subtitle indicates,
the idea of hunting, because he thought it was savage, four essays. Which of the following is NOT one
but soon learns to appreciate hunting when a boar among them?
attacked the group. (a) "Archetypal Critcism : Theory of Myths"
Hence, option (b) will be correct answer. (b) "Typological Criticim : Theory of Types"
35. In Canterbury Tales who has a red face full of (c) "Historical Criticism : Theory of Modes"
sores? (d) "Ethical Criticism : Theory of Symbols"
(a) The Summoner (b) The Shipman Ans : (b) Anatomy of Criticism : Four Essays (1957) is
(c) The Yeoman (d) The Reeve an attempt to formulate an overall view of the scope,
Ans : (a) The Canterbury Tales is a collection of 24 theory principles, and techniques of literary criticism.
stories that runs to over 17,000 lines written in Middle 40. In Robert Browing's "Andrea del Sarto", with
English by Geofrey Chaucer between 1387 & 1400. which of the following painters does Andrea
NOT compare himself with?
36. The pace of speech is called:
(a) Michelangelo (b) Leonardo da Vinci
(a) Syllable (b) Loudness
(c) Rembrandt (d) Raphael
(c) Tempo (d) Pitch
Ans : (c) Andrea del Sarto also called The Faultless
Ans : (c) Rate of speech is how fast you talk in words Painter is a poem by Robert Browning (1812-1889)
perminute (wpm) & is an important component of published in his 1855 poetry collection Men and
innovative presentatious. Rate of speech is also called Women. It is a dramatic monologue, a form poetry for
pace, or tempo. which he is famous, about the Italian Painter Andrea del
37. Match the title with the author: Sarto. The poem is in Blank verse and mainly uses
A. Sexual Politics i. Mary Ellman iambic pentameter. The poem was inspired by Andrea d
B. A Literature of ii. Elaine Showalter Agnolo, renaissance artis (1486-1530).
Their Own 41. In Jonathan Swift's Gullivers Travels Gulliver
C. Thinking About iii. Helene Cixous refers to William Dampier, the famous writer
Women of two voyages, as :
D. The Laugh of the iv. Kate Millet (a) Master (b) Brother
Medusa (c) Cousin (d) Uncle

UGC NET/JRF English Paper II, November 2017 249 YCT


Ans : (c) Gulliver’s Travels is swift’s best known full 47. Who amon the following is the author of Steps
tenth work, and a classic of english literature, published to the Temple?
in 1726. (a) John Donne (b) Richard Crashaw
42. Who among the following is NOT a character (c) George Herbert (d) Henry Vaughan
in Pride and prejudice? Ans : (b) Step to the temple Originally published in
(a) Mr. Darcy (b) Miss Bingley 1904 as part of the Cambridge english classics series.
(c) Miss Bates (d) Mr. Collins This book contains Latin, Greek 4 english poems of
Ans : (c) Pride and Prejudice is a romantic novel by Richard Crashew (c. 1614-49).
Jane Austen, first published in 1813. 48. Match the character with the work:
43. "All the world's a stage, A. Jim Dixon i. Room at the Top
And all the men and women merely players", B. Jimmy Porter ii. Hurry on Down
Occurs in Shakespeare's As You Like It. C. Joe Lampton iii. Look Back In
Which character says the line? Anger
(a) Jacques (b) Celia D. Charles Lumley iv. Lucky Jim
(c) Rosalind (d) Touchstone Code :
Ans : (a) “All the world’s a stage” is the phrase that A B C D
begins a monologue from “As You Like It”, spoken by (a) iv iii i ii
the melancholy Jaques in act II scene VII. (b) iv iii ii i
44. Which of the following rivers are mentioned in (c) iii iv i ii
Andrew Marvell's poem "To His Coy (d) iii i ii iv
Mistress"? Ans : (a) Jim Dixon – Lucky Jim, by Kingsley
(a) Thames and Rhine Aims, pub- 1954.
(b) Thames and Ganges Jimmy Porter – Look Back In Anger, by
(c) Ganges and Humber Osborne, Pub. 1956
(d) Thames and Humber Joe Lampton – Room at the Top, by
Ans : (c) “To His Coy Mistress” is a metaphysical John Braine, pub. 1957
poem written by the English author Andrew Marvell, Charles Lumley – Hurry on Down, by
published posthumously in 1681. John Wain, Pub. 1953
45. "The truth, the whole truth and nothing but 49. In the opening book of The prelude
the truth". The above is an example of: Wordswerth mentions famously that he was
(a) Ploce (b) Epizeuxis "fostered alike by____and____".
(c) Plurisignation (d) Diaeresis Pick out the right pair.
Ans : (a) Ploc- A rhetorical term for the repetition of a A. Nature
word or name often with a different sense, after the B. Fear
intervention of one or more other words eg- “A wife C. Imagination
who was a wife indeed”. D. Beauty
Epizeuxis – Is the repetition of a word or phrase in
The right combination according to the code is:
immediate succession, typically within the same
(a) A & C (b) D & B
sentence, with no other words in between or when
words or phrases are repeated for emphasis, or out of (c) D & C (d) A & D
anger or excitement eg – He was late! I tell you! Ans : (b) Wordsworth mentions famously that he was
46. Which of the following images is NOT part of “fostered alike by beauty and fear”.
W.H. Auden's poem "In Memory of W.B. 50. The title of Ngugi wa Thiong'o's Petals of
Yeats"? Blood is derived from a poem by Derek
(a) Mercury sinking in the mouth of the dying Walcott. Identify the poem.
day (a) "A Far Cry from Africa"
(b) Wolves running through evergreen forests (b) "The Swamp"
(c) Silence invading the suburbs (c) "Goats and Monkeys"
(d) Memory scattering like the beads (d) "Midsummer"
(UGC Nov. 2017 II) Ans : (b) Petals of Blood is a novel of first published in
Ans : (d) In Memory of W.B. Yeats by W.H. Auden is 1977 set Kenya just after independence the story
a modern poem in its imagery & concept. The poem is follows four character – Munira, Abdulla, Wanja, and
an elegy written to mourn the death of W.B. Yeats. Karega.

UGC NET/JRF English Paper II, November 2017 250 YCT


UGC NET/JRF Exam, July-2018
ENGLISH
Solved Paper-II
Note : This paper contains hundred (100) objective Fowles's 'French Lieutenant's Woman' and Margaret
type question of two (2) marks each. All question are Atwood's 'Handmaid's Tale'.
compulsory. Notable awards–
1. Which narrative poem by Lord Tennyson Nobel Prize in Literature (2005)
presents the story of a fisherman turned David Cohen Prize (1995)
merchant-sailor who, after a shipwreck, is Laurence Olivier Award (1996)
marooned on a desert island? Companion of Honour (2002).
(a) "Crossing the Bar" 4. The years in English literary history between
(b) "Tithonus" 1649 and 1660 are known as_______.
(c) "Enoch Arden" (a) the Neo-classical period
(d) "Maud" (b) the Commonwealth period
Ans: (c) 'Enoch Arden' is a narrative poem published (c) the Stuart period
in 1864 by Alfred Lord Tennyson (1809-1892). The (d) the Jacobean period
hero of the poem turned merchant sailor Enoch Arden Ans: (b) The year between 1649 and 1660 are known
on the offer of old captain, where on a desert island as Commonwealth period in the history of English
with two companions; both eventually die, living
literature.
Arden alone there. In this poem Tennyson presents the
The Neo-classical period (1660-1798), The
story of a fisherman turned merchant sailor who after
Commonwealth period (1649-1660), The Stuart
a shipwreck is marooned on a desert island.
period (1603-1714), The Jacobean period (1567-
2. In "Memorial Verses" Matthew Arnold pays 1625).
tribute to three great poets. Who are they?
5. In R. K. Narayan's Swami and Friends, which
(a) Goethe, Shakespeare, Wordsworth
game offers Swami the best kind of emotional
(b) Goethe, Shakespeare, Milton
release from the strains and pressures of
(c) Shakespeare, Milton, Wordsworth
disagreeable circumstances?
(d) Goethe, Wordsworth, Byron
(a) cricket (b) football
Ans: (d) Matthew Arnold (1882-1888) composed
(c) tennis (d) hockey
'Memorial Verses' in April 1850. In this poem he has
Ans: (a) Swami and Friends (1935) is the first of a
paid tribute to – Goethe, Byron and Wordsworth. First
stanza of this poem makes it clear– trilogy of novels written by R. K. Narayan (1906-
Goethe in weimer sleeps, and Greece, 2001). Swami is a good cricket bowler and pride
Long since, saw Byron's struggle cease, himself on being nicknamed “Tate” after a famous
But one such death remained to come; cricket player.
The Last poetic voice is dumb The second and third books in the trilogy are The
We stand today by Wordsworth's tomb. Bachelor of Arts (1937) and The English Teacher
3. Who among the following English playwrights (1945).
wrote screenplays on novels such as Marcel 6. William Blake expressed the importance of the
Proust's In Search of Lost Time, John Fowles's particular when he said that "To Generalize is
French Lieutenant's Woman, and Margaret to be ______. To particularize is the alone
Atwood's Handmaid's Tale? Distinction of Merit." Fill in the blank.
(a) John Arden (b) Edward Bond (a) an idiot (b) a poet
(c) Harold Pinter (d) David Hare (c) a dreamer (d) a skunk
Ans: (c) Harold Pinter (1930-2008) was a British Ans: (a) William Blake (1757-1827), in Annotations
playwrights. He directed or acted in – Radio, stage, to Sir Joshua Reynold's Discourse, Quotes – "To
television and field production of his own and other Generalize is to be an idiot. To particularize is the
works. He wrote screen plays on novels such as Alone Distinction of Merit. General Knowledge are
'Marcel Proust's' In Search of Lost Time', John those knowledge that idiot possess.

UGC NET/JRF English Paper II, July 2018 251 YCT


7. Which of the following was not a dialect of Old Ans: (a) "Ut pictura poesis" is a Latin phrase literally
English? meaning – "As is painting, so is poetry". The
(a) Irish (b) Northumbrian statement occurs most famously in Horace's – Ars
(c) Mercian (d) Kentish Poetica.
Ans: (a) Old English had four main dialects, starting 11. Who among the following is the author of
from the mid 7th century associated with particular Account of the Augustan Age in England (1759)?
Anglo-Saxon kingdom : Mercian, Northumbrian, (a) John Gay (b) William Hazlitt
Kentish and West Saxon. Hence Irish was not a (c) Oliver Goldsmith (d) Samuel Johnson
dialect of Old English. Ans: (c) Oliver Goldsmith (1728-1774) was a Irish
8. Anthony Burgess's last novel, published in 1993, novelist, playwright and poet. He is the author of
is called A Dead Man in Deptford. Who is the "Accounts of the Augustan Age in England" (1759)
central character to whom the title refers? and identifies it with the reign of Queen Anne and the
(a) Sir Walter Releigh era of Congreve, Prior and Bolingbroke.
(b) Sir Philip Sidney 12. In how many parts did Cervantes publish his
(c) Christopher Marlowe novel, Don Quixote?
(d) Earl of Southampton (a) three (b) five
(c) two (d) twelve
Ans: (c) 'A Dead Man in Deptford' is a novel by
Ans: (c) 'Don Quixote' is a Spanish novel by Miguel
Anthony Burgess published in 1993. This novel
de Cervantes, published in two volume in 1605 and
depicts the life and character of Christopher Marlowe,
1615. It basically known for his different type of
a renowned playwright of the Elizabethan era.
genres (Novel, Parody, Satire, Farce, Psychological
Marlowe is portrayed as a secretive, salitory and
Fiction).
eventually isolated person. Burgess explores his
13. Lytton Strachey's Eminent Victorians carries
sexual addiction and passion for the theatre.
biographical sketches of writers and public
9. Choose the correct chronological order : figures. Identify the list below that correctly
(a) William Caxton prints the first English book – mentions those Eminent Victorians.
William Shakespeare's First Folio – John (a) Cardinal Manning, Florence Nightingale,
Milton's Areopagitica – "Tottel's Miscellany" Thomas Arnold and General Gordon.
(Songs and Sonnets). (b) A.E.W. Mason, Sir Arthur Quiller Couch,
(b) "Tottel's Miscellany" (Songs and Sonnets) – Matthew Arnold, Robert Bridges.
William Shakespeare's First Folio – William (c) E.F. Benson, Cardinal Manning, Lord
Caxton prints the first English book – John Tennyson, Beatrice Webb.
Milton's Areopagitica. (d) George Harding, General Gordon, Robert
(c) William Caxton prints the first English book – Browning, Mrs Humphrey Ward.
"Tottel's Miscellany" (Songs and Sonnets) – Ans: (a) 'Eminent Victorians' is a book by Lytton
William Shakespeare's First Folio – John Strachey (1880-1932), first published in 1918 and
Milton's Areopagitica. consisting of biographies of four leading figure –
(d) William Shakespeare's First Folio – John Cardinal Manning (1808-1892), Florence Nightingale
Milton's Areopagitica – William Caxton prints (1820-1910), Thomas Arnold (1795-1892) and
the First English book – "Tottel's Miscellany" Charles George Gordon (1833-1855) from the
(Songs and Sonnets). Victorian era.
Ans: (c) Correct chronological order is– 14. One of the following statements about the
William Caxton (1422-1491) prints the first English eponymous saint of Dryden's "Song for St.
book – "Tottel's Miscellany" (songs and sonnet-1557) Cecilia's Day" is incorrect. Identify that
– William Shakespeare's First Folio (1609) – John statement.
(a) St. Cecilia was a Roman lady, an early
Milton's Areopagitica (1644).
Christian martyr.
10. What does the phrase ut pictura poesis from
(b) St. Cecilia was an Armenian devotee of the
Horace's Art of Poetry mean? Christian faith.
(a) "as in painting, so in poetry" (c) St. Cecilia's festival is celebrated on 22
(b) "poetry beggars pictorial description" November in England.
(c) "as in poetry, so in painting" (d) St. Cecilia was a patroness of music who was
(d) "picture above all poetry" fabled to have invented the organ.
UGC NET/JRF English Paper II, July 2018 252 YCT
Ans: (b) A song for St. Cecilia Day, composed in Ans: (a) Sir Thomas Browne (1605-1682) was an
1687 is the first of two great odes written by poet author of varied works. 'Urn Burial' Hydriotaphia, Urn
laurate John Dryden. And set the music for the annual Barial or a Discourse of the Sepulchral urns was
St. Cecilia's Day celebration held every November-22. published in 1658, it was prompted by the discovery
St. Cecilia (a Roman lady) an early Christian Martyr of ancient burial – urns near Norwich.
and patron saint of music was honoured at public 18. Identify from among the following list those that
celebration. Hence option (b) St. Cecilia was an cannot be called War Fiction.
American devotee of the Christian faith is incorrect. (1) A Modern Instance
15. Which of the statements on Michael Roberts's (2) Catch - 22
Faber Book of Modern Verse (1936) is not true? (3) The Age of Innocence
(4) The Naked and the Dead
(a) His anthology canonized modern poetry and
(a) (1) and (4) (b) (2) and (3)
poets for quite some decades.
(c) (1) and (3) (d) (2) and (4)
(b) The collection begins with the poems of Robert
Ans: (c) A Modern Instance and The Age of
Bridges.
Innocence are not war fiction. A modern Instance is a
(c) Roberts omitted the Georgian poets in his
realistic novel written by – Dean Howells in 1882
anthology.
while The Age of Innocence is a American novel by
(d) Yeats, Eliot and Pound find a place in the
American author Edith Wharton in 1920.
Faber Book of 1936.
19. Who among the following writers was not the
Ans: (b) 'The Faber of Modern Verse' an anthology one identified with The Movement of the 1950's
published in 1936 by Michael Roberts (1902-1948), England?
which did much to establish the reputation of a rising (a) Roy Fuller (b) Kingsley Amis
generation including W. H. Auden, Louis Mac Neice, (c) Donald Davie (d) Philip Larkin
William Empson and Dyln Thomson and to create a Ans: (a) The Movement of the 1950's England, Anger
lineage for the previous generation of modernist. The as a force in 1950s literature had its origins in a group
collection begins with the poems of G. M. Hopkins. known as the movement. Deeply English in outlook,
There were a lot of poets find a place in it. the movement was a gathering of poets including –
16. Who among the following proposed that the Philip Larkin, Kingsley Amis, Elizabeth Jenning,
First Gulf War had never taken place, it was Thom Gunn, John Wain, D. J. Enright and Robert
simply a hyperreal, media-generated spectacle? Conquest. Roy Fuller was not associated with the
(a) Richard Rorty movement.
(b) Jean-Francois Lyotard 20. Which of the following novels does not belong to
(c) Jean Baudrillard Nuruddin Farah's Blood in the Sun Trilogy?
(d) Umberto Eco (a) Maps (b) Knots
Ans: (c) Jean Baudrillard was a French sociologist, (c) Gifts (d) Secrets
political commentator and photographer. In his a Ans: (b) 'Blood in the Sun Trilogy' belong to
collection of three short essays – The Gulf war did not Nuruddin Farah, a Somali novelist. Titles included in
take place in the French news paper – Liberation and this trilogy are–
Maps (1986), Gifts (1992) and Secrets (1998). Knots
British paper – The Guardian proposed that the First
is not included in this trilogy.
Gulf War had never taken place, it was simply a
hyperrel, media-generated spectacle. 21. In the following series, which one has all the
poets correctly matched with their poems?
17. Sir Thomas Browne's Urn Burial was prompted
(a) Ezekiel, "Poet, Lover, Birdwatcher";
by_______ Ramanujan, "Small-scale Reflections on a
(a) the discovery of ancient burial-urns near Great House", Dutt, "Sunset at Puri",
Norwich. Mahapatra, "Our Casuarina Tree.
(b) the contemporary researches on burial rites in (b) Ezekiel, "Sunset at Puri", Ramanujan, "Small-
Norway. scale Reflections on a Great House", Dutt,
(c) the death of St. Francis of Assissi and his "Our Casuarina Tree", Mahapatra, "Poet,
burial Lover, Birdwatcher".
(d) the publication of the English Book of (c) Ezekiel, "Poet, Lover, Birdwatcher",
Common Prayer. Ramanujan, "Sunset at Puri", Dutt, "Our

UGC NET/JRF English Paper II, July 2018 253 YCT


Casuarina Tree", Mahapatra, "Small-scale 24. In the mechanical drill method of second
Reflection on a Great House". language acquisition:
(d) Ezekiel, "Poet, Lover, Bird Watcher", (1) The learner has the freedom to choose from
Ramanujan, "Small-scale Reflections on a many responses.
Great House", Dutt, "Our Casuarina Tree", (2) The learner's response in totally controlled.
Mahapatra, "Sunset at Puri". (3) Comprehension of the item by the learner is
Ans: (d) Correctly matched poems with their poets not required.
are as follows– (4) Comprehension of the item by the learner is
Ezekiel – Poet, Lover, Birdwatcher. obligatory.
Ramanujan – Small Scale Reflection on a great house. The right combination according to the code is:
Toru Dutt – Our Casuarina Tree. (a) (1) and (4) (b) (1) and (3)
Mahapatra's – Sunset at Puri. (c) (2) and (3) (d) (2) and (4)
Hence option (d) is correctly matched. Ans: (c) Option (2) and (3) both are correct. In the
22. From among the following, identify the mechanical drill method of second language
incorrect observation regarding Ferdinand de acquisition: The learner's response in totally
Saussure's seminal distinction between langue controlled and Comprehension of the item by the
and parole. learner is not required. Hence option (c) is correct.
(a) Parole is the particular language system, the
Thou wilt not wake
elements of which we learn as children, and
Till I thy fate shall overtake;
which is codified in our grammars and
Till age, or grief, or sickness must
dictionaries, whereas langue is the language-
Marry my body to that dust
occasion (what A says to B).
It so much loves; and fill in the room
(b) A language consists in the interrelationship
My heart keeps empty in thy Tomb.
between langue and parole.
Stay for me there; I will not fail
(c) Sassure made this crucial distinction in a study
called A Course in General Linguistics (1916). To meet thee in that hollow Vale.
(d) Langue is the particular language-system, the And think not much of my delay;
elements of which we learn as children, and I am already on the way.
which is codified in our grammars and 25. Which of the following readings do you find
dictionaries, whereas parole is the language- appropriate to the spirit of the lines above?
occasion (what A says to B). (a) In that interspace between the lines, the ending
Ans: (a) Parole means speech. Langue and Parole are of one and the beginning of another there is a
linguistic terms distinguished by Ferdinand De silent internal language, the poem's language-
Saussure in his 'Course in General Linguistics'. Parole within-language, tacitly signally through the
is the particular language system, the elements of deployment of rhymed space.
which we learn as children, and which is codified in (b) Ageing and dying are of course helplessly
our grammars and dictionaries, whereas langue is the passive; but here love makes them as though
language-occasion. Hence option (a) is incorrect. they were now also willing things in the
23. John Heywood wrote a farcical interlude called husband eager to join his dead wife. Through
The Four P's. simple intimate tones of their shared earthly
Who were the Four P's? life – stay for me, wait for me, I will not fail –
(a) a Palmer, a Pedlar, a Pothecary, a Packer he not only imagines her but imagines her
(b) a Printer, a Pedlar, a Pothecary, a Palmer thinking of him.
(c) a Pedlar, a Parson, a Palmer, a Pothecary (c) The lyric voice here can feel the poem
(d) a Palmer, a Paradoner, a Pothecary, a Pedlar speaking back to him – in the cold lineal stare
Ans: (d) John Heywood was an English playwright, of 'there was nothing in my belief' – even as
actor and author. 'The Four P's' is a farcical interlude his dead wife did not. It is as though the poem
involves a debate – more a competition in fact itself then demands his response, in order to be
between fourmen whose trades begin with the letter P able to move from one line to another. To
(a Palmer, a Paradoner, a Pothecary, a Pedlar). attempt that movement in keeping the poem's
Two are churchman – a Pardoner (an a Pothecary) and space alive, the lyric voice asserts, "I will not
the last is a Pedlar. Hence option (d) is correct. fail/To meet there in that hollow Vale."
UGC NET/JRF English Paper II, July 2018 254 YCT
(d) My whole nature was so penetrated with grief April 27. Old father, old artificer, stand me now
and humiliation of such considerations that, and ever in good stead."
even now, famous and caressed and happy as I (a) To the light house
am, I often forget in my dream that I have a (b) A Portrait of the Artist as a Young man
dear wife who died, leaving me alone in this (c) Maurice
world. Even that I am a man, and now I (d) Almayer's Folly
wander desolately back to that time of our Ans: (b) The very last passage is given in the James
lives when my wife and shared moments of Joyce's (1882-1941) novel 'A Portrait of the artist as a
bliss. young man'. "Welcome, O life, I go to encounter for
Ans: (b) These lines are extracted from the poem of the millionth time the reality of experience and to
Henry King's 'The Exequy'. The poem takes the form forge in the smithy of my soul the uncreated
of a lament for the speaker's dead wife. After reading conscience of my race."
we find appropriate to the spirit of the lines option Hence option (b) is correct.
(b)– 28. Francis Bacon's New Atlantis is about a utopian
Ageing and dying are of course helplessly passive; but state called ______.
here love makes them as though they were now also (a) Asgard (b) Avalon
willing things in the husband eager to join his dead (c) Bensalem (d) Baltia
wife. Through simple intimate tones of their shared Ans: (c) 'New Atlantis' is an incomplete utopian novel
earthly life – stay for me, wait for me, I will not fail – by Sir Francis Bacon (1561-1626) published in 1627.
he not only imagines her but imagines her thinking of It is about a utopian state – Bensalem.
him. Hence option (c) is correct.
26. Match the characters with the novels: 29. The 1950's saw the rise of backlash against
(A) Arthur Seaton (i) Top Girls modernism and against New Romanticism that
(B) Marlene (ii) The Golden became known as The Movement. Which of the
Notebook following little magazines came to be associated
(C) Anna Wulf (iii) The Swimming Pool with The Movement?
Library (1) Departure
(D) Beckwith (iv) Saturday Night and (2) New Verse
Sunday Morning (3) London Mercury
Code: (4) New Poems
A B C D The right combination according to the code is:
(a) (ii) (iii) (i) (iv) (a) (1) and (2) (b) (3) and (4)
(b) (iv) (i) (ii) (iii) (c) (1) and (4) (d) (2) and (4)
(c) (iii) (iv) (ii) (i) Ans: (c) Departure and New Poems magazines are
(d) (ii) (iv) (iii) (i) associated with the movement. It was against
Ans: (b) Arthur Seaton – This character appears in modernism and New Romanticism.
Alan Sillitoe's novel 'Saturday Night and Sunday Hence option (c) is correct.
Morning' (1958). 30. The error of interpreting a literary work by
Marlene – This character appears in – Caryl referring to evidence outside of itself, such as the
Churchill's play 'Top Girls' (1982). design and purpose of the author is called_____.
Anna Wulf – This character appears in Daris Lessing's (a) Affective fallacy
novel 'The Golden Notebook' (1962). (b) Intentional fallacy
William Beckwith – This character appears in – Alan (c) Authorial fallacy
Hallinghurst's novel 'The Swimming Pool Library (d) Synecdochic fallacy
(1988).
Ans: (b) The error of interpreting a literary work by
Hence option (b) is correct.
referring to evidence outside of itself, such as the
27. The very last passage of a novel is given below. design and purpose of the author is called –
Identify the novel. International fallacy. This term was given by W. K.
"Welcome, O life, I go to encounter for the Wimsatt and Monroe C Beardsley in his book 'The
millionth time the reality of experience and to Verbal Icon: Studies in the meaning of Poetry'.
forge in the smithy of my soul the uncreated Hence option (b) is correct.
conscience of my race.
UGC NET/JRF English Paper II, July 2018 255 YCT
31. A.R. Ammons parodies a famous poem in his Between this Africa and the English tongue I
"Swoggled" love?"
I'd rather (a) Derek Walcott (b) Louise Bennett
be (c) Kamau Brathwaite (d) Wole Soyinka
suckled by Ans: (a) In the following line – "Where shall I turn
an ……………….. I love", Derek Walcott raises painful
outworn pagan question of longing and belonging. These lines are
than taken from – Derek Walcott's poem 'A Far Cry from
get my horn Africa' ‘A From Cry From Africa’ (1962) explores the
wreathed in bloody history of colonial Africa in Kenya. Hence
an option (a) is correct.
old triton. 35. In the 1940's, a critic and a philosopher
Which poet, which poem? produced two influential and controversial
(a) John Keats, "On First Looking into Chapman's paper called "The Intentional Fallacy" and
Homer" "The Affective Fallacy".
(b) John Milton "On His Blindness" Identify them.
(c) William Wordsworth, "The World is Too (1) Cleanth Brooks
Much with Us" (2) Monroe C. Beardsley
(d) Elizabeth B. Browning, "How do I Love (3) William K. Wimsalt jr.
Thee…?" (4) R.P. Blackmur
Ans: (c) Archie Randolph Ammons (1926-2001) The right combination according to the code is:
parodies – William Wordsworth's (1770-1850) – The (a) (1) and (2)
World is Too Much with Us in his famous poem – (b) (2) and (4)
Swoggled. (c) (2) and (3)
Hence option (c) is correct. (d) (3) and (4)
32. Fanny Burney's Evelina carries the subtitle: Ans: (c) The term "The Affective Fallacy" and "The
(a) or a Naive Lady's Entrance into the World Intentional Fallacy" was coined by W. K. Wimsalt
(b) or a Young Lady's Entrance into the World and Monroe C. Beardsley in 1946 as a principle of
New Criticism. Hence option (c) is correct.
(c) or a Young Lady's Exit from the World
Affective Fallacy : The Affective fallacy is confusion
(d) or a Bold Lady's Entrance into the Hall
between the poem and its results (what it is and what
Ans: (b) Evelina or ‘The history of a Young Lady's
it does). It begins from the psychological effects of the
Entrance into the World’ – is a novel written by
poem and ends in impressionism and relativism.
English author Fanny Burney (1752-1840) also known Intentional Fallacy (1946) : “The intentional Fallacy
as Frances Burney or Madam d’Arblay and first is a confusion between the poem and its origin. It
published in 1778. Hence subtitle of Evelina is begins by trying to derive the standard of criticism
option (b). from the psychological causes of the poem and ends
33. What does Philip Sidney call poet-haters in his in biography and relativism.
Defence of Poesie? 36. Philip Larkin's "Sad Steps" notices "The way
(a) misogynists (b) misanthropes the moon dashes through clouds that blow
(c) misnomers (d) mysomousoi Loosely as cannot-smoke to stand apart…."
Ans: (d) ‘In Defence of Poesie’ (1595), Philip Sidney The poem alludes to:
has written - "First truly I note, not only in these (a) Coleridge's "Dejection : An Ode"
mysomousoi poet – haters but in all that kind of (b) The moonlit scenes in A Midsummer Night's
people, who seek a praise by dispraising other, that Dream
they do prodigally spend a great many wondering (c) Philip Sidney's Astrophel and Stella
words……" makes clear that Philip Sidney called poet (d) T.S. Eliot's "Morning at the Window"
haters – mysomousoi. Hence option (d) is correct. Ans: (c) Philip Larkin's poem 'Sad Steps' alludes
34. Who, among the following, raises the following Philip Sidney's (1554-1586) Astrophel and Stella
painful question of longing and belonging? (1591). ‘Sad Steps’ was completed by Philip Larkin
"Where shall I turn, divided to the vein? (1922-1985) and published in his final volume of
I who have cursed poetry, ‘High Windows’ (1974). Hence option (c) is
The drunken officer of British rule, how choose correct.

UGC NET/JRF English Paper II, July 2018 256 YCT


37. Match the following opening lines with their (a) African Psycho by Alain Mabanckou
respective titles: (b) The Chibok Girls by Helon Habila
(1) "I leant upon a (i) "Thirteen (c) The Underground Railroad by Colson
coppice gate" Blackbirds" Whitehead
(2) "A sudden blow : (ii) "Sympathy" (d) The Book of Night Women by Marlon James
the great wings Ans: (d) "I am the woman they give dead women's
beating still" clothes to" from Christine Gelineau's "Inheritance".
(3) "Among twenty (iii) "The Darkling The books name – 'The Book of Night Women' was a
snowy mountains" Thrush" line at the start of the book by the writer in 2009, the
(4) "I know what the (iv) "Leda and the novel was published by Jamaican author – Marlon
caged bird feels, Swan" James, about a woman Lilith in the 18th century.
alas…." Hence option (d) is correct.
Code 40. An English poet couldn’t help the excitement
(1) (2) (3) (4) that an historical event caused in his life-time:
(a) (iv) (iii) (ii) (i) Bliss was it in that dawn to be alive,
(b) (iii) (iv) (i) (ii) But to be young was very heaven.
(c) (ii) (i) (iii) (iv) Which poet? What "dawn"?
(d) (i) (ii) (iv) (iii) (a) W. H. Auden; the Spanish Civil War
Ans: (b) Correctly matched opening lines with their (b) Lord Tennyson; the Jubilee of Queen Victoria's
respective titles are as follows– Reign
(1) I leant upon a coppice gate – The Darkling Thrush (c) William Wordsworth; the French Revolution
by Thomas Hardy. (d) William Blake; the Industrial Revolution
(2) A Sudden blow: the great wings beating still – Ans: (c) William Wordsworth was inspired by the
Leda and the Swan by W.B. Yeats. French Revolution, and his poetic Muses took flight
(3) Among twenty snowy mountains – Thirteen spurred on by the spirit of youthful generosity and and
Blackbirds by Wallace Stevens. enthusiasm and by a vision of the future which was
(4) I know what the caged bird feels, alas – Sympathy full of hope. When this vision died poetic inspiration
by P.L.Danbar began to dry up with it. Of course this did not happen
Hence option (b) is correct. all at once. Hence option (c) is correct. This incident
occurs in famous poem written by W. Wordsworth
38. Identify the titles that were published in the
‘Prelude’ (1798) Book 11 France.
1920' s.
(1) Look, Stranger! 41. Which novel by John Banville tells the story of a
(2) The Tower group of travelers who arrive on a small island
(3) The Waste Land and stumble upon the house of Prof. Kreutznaer
(4) The Road to Wigan Pier whose relationship to a painting entitled The
Golden World by a fictional Dutch artist named
Code :
Vaublin plays a central role?
(a) (1) and (3) (b) (2) and (3)
(a) Ghosts (b) The Sea
(c) (2) and (4) (d) (3) and (4)
(c) The Ark (d) Eclipse
Ans: (b) Look Stranger! published in – 1936 by W.H.
Ans: (a) Ghosts is a novel by Irish writer John
Auden
Banville published in 1993. The novel is somewhat
The Tower published in – 1928 by W.B Yeats
unconventional and non-liner in its construction. It
The Waste Land – 1922 by T.S. Eliot
tells the story of a group of traveler who arrives on a
The Road to Wigan Pier – 1937.
small island and stumble upon the house of Prof.
The Tower and The Waste Land were published in
Kreutznaer.
1920's. Hence option (a) is correct.
Hence option (b) is correct.
42. Identify the two plays, usually paired for their
39. This novel is dedicated "To the railroad of critique of the politics of language and acts of
bones" and has as its epigraph the line, police interrogation.
"I am the woman they give dead women's (a) Earthly Powers, The Wanting Seed
clothes to" from Christine Gelineau's (b) Chicken Soup with Barley, Roots
"Inheritance". (c) Left-handed Liberty, The Hero Rises
Identify the novel. (d) One for the Road, Mountain Language
UGC NET/JRF English Paper II, July 2018 257 YCT
Ans: (d) "One for the Road" is a comedic play by Ans: (b) Anaphora is a figure of speech in which the
Willy Russel, written in 1976 and published in 1980. repetition of a word or phrase at the beginning of
"Mountain Language" is a one act play written by successive clause.In the given lines ‘every’ is repeated
Harold Pinter, first published in The Times Literary word.
Supplement on 7-13 October 1988. Both plays usually Hence option (b) is correct.
paired for their critique of the politics of language and 47. At whose behest does the Redcrosse Knight
acts of police interrogation. undertake his quest in The Faerie Queene?
Hence option (d) is correct. (a) Gloriana's (b) Una's
43. Semiotics originated mainly in the works of two (c) Duessa's (d) Prosperine's
theorists. They are: Ans: (*) Redcross Knight undertakes his quest at the
(1) Charles Sanders Peirce behest of Duessa, who acts as a woman in danger to
(2) Mikhail Bakhtin trap the Redcross Knight, because of some
(3) Ferdinand de Saussure discrepancies in the options, NTA has dropped this
(4) Valentin Voloshinov question.
The right combination according to the code is
48. In which city did John Ruskin see a paradigm
(a) (1) and (2) (b) (2) and (3)
for Victorian Britain?
(c) (1) and (3) (d) (3) and (4)
(a) Vienna (b) Venice
Ans: (c) Semiotics means – the study of sign and
(c) Rome (d) Paris
symbols and their use or interpretation. Charles
Ans: (b) John Ruskin see a paradigm for Victorian
Sanders Peirce and Ferdinand de Saussure are related
to semiotics. Britain in Venice. Hence option (b) is correct.
Hence option (c) is correct. 49. Which novel of Kazuo Ishiguro is narrated by a
44. Robert Burton's Anatomy of Melancholy was Japanese widow living in England and draws on
published in 1621 and expanded and altered in the destruction and rehabilitation of Nagasaki?
_____ subsequent editions. (a) An Artist of the Floating World
(a) two (b) four (b) The Unconsoled
(c) six (d) five (c) A Pale View of Hills
Ans: (d) 'Anatomy of Melancholy' was published in (d) When We Were Orphans
1621 and expanded and altered in five subsequent Ans: (c) A Pale View of Hills is the first novel by
editions. Nobel Prize winning author Kazuo Ishiguro. It is
Hence option (d) is correct. narrated by a Japanese widow living in England and
45. Which of the following magazines self- draws on the destruction and rehabilitation of
consciously created an identity for Vorticists, a Nagasaki.
group of painters, sculptors and writers? Hence option (c) is correct.
(a) Blast (b) The Egoist 50. Which novel opens thus:
(c) The Criterion (d) New Age "Whether I shall turn out to be hero of my own
Ans: (a) 'Blast' magazine self-consciously created an life, or whether that station will be held by
identity for Vorticists, a group of painter, sculptor and anyone else, these pages must show."
writers. Blast was the short lived literary magazine of (a) Tristram Shandy
the Vorticist Movement in Britain. Two editions were (b) Lady Audley's Secret
published the first on 2 July 1914 and published with (c) David Copperfield
a bright pink cover, referred to by Erra pound as the (d) Fitz-Boodle's Confessions
‘Great Magenta cover’d Opusculus’
Ans : (c) David Copperfield Junior is the protagonist
46. "In every cry of every Man,
after which the 1850 Charles Dickens novel David
In every Infant's cry of fear,
Copperfield was named. Opening lines of this novel is –
In every voice, in every ban…"
option (c) 'Whether I shall turn out to be hero of my
The figure of speech characterized by repetition
own life, or whether that station will be held by anyone
of words or group of words at the beginning of
else, these pages must show. David Copperfield
consecutive sentences is called
considered Charles Dickens autobiography.
(a) apostrophe
(b) anaphora Hence option (c) is correct.
(c) incremental repetition 51. Traces of Morality plays are discernible in a
(d) alliteration play like Dr. Faustus, traces such as ______.
UGC NET/JRF English Paper II, July 2018 258 YCT
(a) vernacular songs adapting secular themes Ans : (c) Gillian Wright is a translator and writer based
(b) its soliloquizing protagonist, Good and bad in New Delhi. She has translated two classic novels of
Angels and its final moral Hindi literature – Rag Darbari, A village divided by
(c) its refrains from the Corpus Christi Carol, the Rahi. So, she is not the translator of Prem Chand
complaint of Christ, the lower of mankind translation.
(d) its rhythmical prose, and the presence of 55. Which of the two novels of Jane Austen have
larger narrative rhythm in the Morality plays the spa town of Bath as a primary location?
Ans : (b) Morality plays were dramatized allegories of (i) Emma
a Christian representative life in the plot form of a quest (ii) Pride and Prejudice
for salvation, in which the crucial events are (iii) Northanger Abbey
temptations, sinning and the climatic confrontation with (iv) Persuasion
death. These characteristic of the Morality plays are The right combination according to the code is
discernible in the play. ____.
Hence option (b) is correct. (a) (i) and (iv)
52. The branch of philosophy that asks the (b) (ii) and (iii)
question, 'How do we know that we know is (c) (iii) and (iv)
____. (d) (i) and (ii)
(a) ontology Ans : (c) Jane Austen was a prolific novelist of
(b) epistemology romantic age. Her novel – Northanger Abbey and
(c) eschatology Persuasion have the spa town of Bath as a primary
(d) phenomenology location.
Ans : (b) Epistemology is the study of the nature of Hence option (c) is correct.
knowledge, justification and the rationality of belief. 56. In the communicative approach to ELT, the
This branch of philosophy asks the question, 'How do development of language learning or teaching
we know what we know'. involves a shift:
Hence option (b) is correct. (i) from form-based to a meaning-based
53. The eighteenth century practice in England of approach.
bookselling was midway between direct (ii) from an eclectic approach to a rigid method.
patronage and impersonal sales. A patron paid (iii) from teacher-centred to learner-centered
half the cost of a book before publication and classes.
half of delivery. The author of the book (iv) from broad-based competence to specific
received these payments directly. The patron's needs.
name appeared in the preface for the book The right combination according to the code is
published in this manner. ____.
This practice was known as _____. (a) (ii) and (iv)
(a) Subscription (b) (i) and (iv)
(b) Contribution (c) (ii) and (iii)
(c) Pre-publication (d) (i) and (iii)
(d) Remaindering Ans : (d) In the communicative approach to EIT, the
Ans : (a) This practice was known as subscription. development of language learning or teaching involves
Hence option (a) is correct. as shift from based to meaning based approach and
54. Oxford India has published a volume of from teacher-centred to learner centred classes.
Premchand translations in English, The Hence option (d) is correct.
Oxford India Premchand. Who among the 57. The four Moral Essays of Alexander Pope are
following is not one of the translators? addressed to carefully selected figures. Identify
(a) David rubin the correct group.
(b) Alok Rai (a) Timons, Newton, Martha Blount, Wellington
(c) Gillian Wright (b) Lord Cobham, Robert Walpole, Houghton
(d) Christopher King Hall, Chandos

UGC NET/JRF English Paper II, July 2018 259 YCT


(c) Martha Blount, Lord Cobham, Bathurst, 61. Who is the only one of Milton's
Burlington Contemporaries to be mentioned by name in
(d) William III, John Haydn Joseph Addison, Paradise Lost?
John Dennis (a) Francis Bacon
Ans : (c) Martha Blount (1690-1762), Lord Cobham, (b) Johannes Vermeer
Bathurst, Burlington is a group of four moral essay of (c) Gallileo
Alexander Pope. (d) King Charles I
Hence option (c) is correct.
Ans : (c) Gallileo was the contemporary of John Milton
58. Bertolt Brecht's Mother Courage and Her
has mentioned him as – Tuskan in his epic 'Paradise
Children presents the war-torn Europe as its
Lost'. Hence option (c) is correct.
protagonist as she follows troops with her
Paradise Lost (1667) is a epic poem by John Milton
canteen wagon.
written in Blank Verse.
What is the real name of Mother Courage?
(a) Paula Danckert 62. K.S. Maniam is a major writer of Indian
(b) Anna Fierling origin, writing in English, born and living in
Malaysis. Identify two of his novels from the
(c) Jane Vanstone
following list.
(d) Jani Lauzon
(i) The Rice Mother
Ans : (b) 'Mother Courage and Her Children' is a play
(ii) The Return
written in 1939 by the German dramatist and poet
Bertalt Brecht. The protagonist of the novel is Mother (iii) Touching Earth
Courage whose real name is – Anna Fierling, also (iv) Between Lives
known as – Canteen Anna. The right combination according to the code is
Hence option (b) is correct. ____.
59. From among the following, identify the journal (a) (i) and (iv)
that publishes articles on English language (b) (ii) and (iii)
teaching and learning. (c) (iii) and (iv)
(a) University of Toronto Quarterly (d) (ii) and (iv)
(b) Agenda
Ans : (d) 'The Return' and 'Between Lives' are the
(c) TESOL Quarterly novels of K. S. Maniam (Subramaniam Krishna).
(d) English Language Notes The Rice Mother (2002) written by Rani Manicka.
Ans : (c) TESOL Quarterly journal publishes articles on The Return (1981) written by K.S. Maniam.
English language teaching and learning. Touching Earth (2004) written by Rani Manicka.
Hence option (c) is correct. Between Lives (2003) written by K.S. Maniam.
60. Arrange the following elegies in English in 63. What did Thomas Percy collect in his Reliques?
chronological order.
(a) medieval folklore and lyrics of the Midlands
(a) "Elegy Written in a Country Churchyard" -
(b) old songs, ballads, and romances in English
"Adonais" - "Thyrsis" - "In Memoriam"
and Scots
(b) "Elegy Written in a Country Churchyard" -
(c) Highland lore, mostly oral wisdom of the
"Adonais" - "In Memoriam" - "Thyrsis"
Scots
(c) "Elegy Written in a Country Churchyard" -
"In Memoriam"- "Adonais" - "Thyrsis" (d) Romantic idylls, sonnets and odes
(d) "Adonais" - "Elegy Written in a Country Ans : (b) The Reliques of Ancient English poetry is a
Churchyard" - "In Memoriam" - "Thyrsis" collection of Ballads and popular song collected by
Ans : (b) Correct chronological order of the elegies is Bishop Thomas Percy and published in 1765. Hence
as follows– option (b) is correct.
Elegy written in a Country Churchyard – 1757 by 64. Nirad Chaudhuri's Autobiography of an
Thomas Gray. Unknown Indian concludes with an essay on
Adonais – 1821 by P.B. Shelley. the course of Indian history. But in the
In Memoriam – 1850 by Alfred Lord Tennyson. penultimate chapter Chaudhuri concludes the
Thyrsis – 1866 by Mathew Arnold. account of events in his life. How does this
Hence option (b) is correct. narrative end?
UGC NET/JRF English Paper II, July 2018 260 YCT
(a) Chaudhuri ties the knot with his childhood (a) Robert Herrick, the poet's
sweetheart and moves from Calcutta to Delhi. (b) John Hakluyt's, the collector of traveller's
(b) Chaudhuri obtains a job in the military tales
accounts department and gives it up because (c) Samuel Purchas, the London Parson's
he finds it soul-destroying. (d) Edward Purchas, the globe-trotter's
(c) Chaudhuri joins the editorial team of a Ans : (c) Purchas his Pilgrimage (1613) essentially a
Calcutta newspaper and is upset over the religious geography, was followed in 1619 by Purchas,
drudgery of a reporter's life. his pilgrim …… the Histories of man, was the work of
(d) Chaudhuri rushes to his ancestral village Samuel Purchas editor and publisher of travel literature.
Bangram on receiving the news of the death Hence option (c) is correct.
of his uncle and recalls his past life. 68. Based on the life of a thirteenth-century
Ans : (b) Nirad Chaudhuri's Autobiography of an troubadour, from among the following identify
unknown Indian ends with following lines– the work, that marked a catastrophic failure in
Chaudhuri obtains a job in the military accounts Robert Browning's poetic career, earning him
department and gives it up because he finds its soul- a reputation for impenetrable difficulty?
destroying. (a) Paracelsus
Hence option (b) is correct. (b) Sordello
65. In John Gower's Confessio Amantis, Amans, (c) The Ring and The Book
the lover makes his confession to the priest (d) Pauline
named______. Ans : (b) Sordello is a narrative poem by the English
(a) Verito poet Robert Browning. This poem marked a
(b) Genius catastrophic failure in Robert Browning's poetic career.
(c) Amor He is known for his dramatic monologue.
(d) Phoebe Hence option (b) is correct.
Ans : (b) 'Confessio Amanti's was written in middle 69. In Tristram Shandy, the Author's preface
English by John Gower. It is a long poem of 33000 _____.
lines. Name of the priest is – Genius. (a) is hawked to the highest bidder.
Hence option (b) is correct. (b) appears in-between chapters 13 and 14 in
John Gower was an English poet, a contemporary of Volume II.
William Lengland and pearl poet and a person of friend (c) is printed in italics in all editions.
of Geoffrey Chaucer. (d) appears in-between chapters 10 and 11 in
66. In Eugene Ionesco's Chairs, the absurdity is Volume I.
not so much in the banal words that are Ans : (*) The mark has seen awarded to all the
uttered as ________. candidates against the question.
(a) in the large scale use of frightening stage Tristram Shandy is a novel by Laurence Sterne.
props and lightning effects. The life and Opinions of Tristram Shandy, Gentleman
(b) in the absurdist interpretation of them by (1759) is a novel by Laurence Sterne (1713-1768).
character after character. Tristam Shandy takes six volumes to cover the chain of
events. It was written in 9 volumes.
(c) in the fact that they are spoken to an ever-
growing number of empty chairs. 70. Evelyn Waugh once complained that T.S.
(d) in the fact that they are spoken time and Eliot's Poems, 1909-1925 was "marvellously
again by members of the audience. good, but very hard to understand." The most
Ans : (c) According to Eugene Ionesco's – the absurdity pessimistic novel Waugh wrote was called
is not so much in the banal words that are uttered in the _____and he owed the title to ____.
fact that they are spoken to an ever-growing number of (a) Balck Mischief - "Sweeney among the
empty chairs. Nightingales"
Hence option (c) is correct. (b) Scoop - "Morning At the
Window"
67. A half-sentence in Purchas his Pilgrimage
(c) Prancing Nigger - Ash Wednesday
triggered off "Kubla Khan". Whose work was
(d) A Handful of Dust - The Waste Land
Purchas his Pilgrimage?
UGC NET/JRF English Paper II, July 2018 261 YCT
Ans : (d) A Handful of Dust is a novel by the British 74. Assertion (A): Our reality is linguistic, a
writer Evelyn Waugh, first published in 1934. He owed language mediated reality.
the title to – The Waste Land. Reason (R): Our perception and understanding
71. During the years 1830 to 1850, the illusion of of reality are largely constructed by the words
peace in Victorian England was broken by and other signs we use.
such incidents as ______. In the light of the statements above
(a) the Revolution in Francs and the Chartist (a) Both (A) and (R) are true and (R) is the
Movement in England correct explanation of (A).
(b) the General Strike of 1835 and the Rail (b) Bothe (A) and (R) are true, but (R) is not the
Tragedy of 1847 correct explanation of (A).
(c) the visionary libertarianism of poets and the (c) (A) is true but (R) is false.
lawless embodiment of revolution (d) (A) is false but (R) is true.
(d) the disaster of the Indian Mutiny and the Ans : (a) Both (A) and (R) are true and ® is the correct
incompetent bungling of the Crimean War explanation of (A).
Ans : (d) The disaster of the Indian Mutiny and the Hence option (a) is correct.
incompetent bungling of the Crimean War was the 75. In his book, In Theory, Aijaz Ahmed words out
incident during the year 1830 to 1850 that broken the the relations between the three entities:
illusion of peace in Victorian England. (a) Classes, Nations, Literatures
Hence option (d) is correct. (b) Regions, Nation, Languages
72. Gulliver receives the following response when (c) State, Religions, Gender
he boasts about his countrymen: (d) Literature, Print, Theory
" ..... the most pernicious race of little odious Ans : (A) In his book, 'In Theory', Aijaz Ahmed works
vermin that nature ever suffered to crawl upon out the relations between the three entities- Classes,
the face of the earth." Whose response? Nations, Literature.
(a) The King of Lilliput's Hence option (a) is correct.
(b) The King of Brobdingnag's 76. In 1660, a group of 12 people including Robert
(c) The Governor of Glubbdubrib's Boyle and Christopher Wren formed what they
(d) The first of the Houyhnhnms's he meets called the Royal Society. In 1663, it became
Ans : (b) Gulliver receives the following response- "the The Royal Society of London for Improving
most pernicious race of little odious vermin that nature Natural Knowledge. What was the society's
ever suffered to crawl upon the face of earth" by the motto?
King of Brobdingnag. (a) "In Him we trust"
Hence option (b) is correct. (b) "In the words of no one"
Gulliver's Travells or Travels into Several Remote (c) "Lighted to lighten"
Nations of the World, written in four parts by Jonathan (d) "Love conquers all"
Swift (1667-1745). Ans : (b) The motto of – The Royal Society of London
73. In the Inferno Dante, as he travels through the for improving Natural knowledge was – "In the words
various circles of the hell finds Judas who is of no one".
unable to speak. What is the reason behind Hence option (b) is correct.
this? 77. Of whom did W.B. Yeats say that "We were
(a) His tongue is transformed into a coiled snake. the last Romantics"?
(b) His head is battered and so he cannot open (a) The Pre-Raphaelite Brotherhood
his mouth. (b) The Imagiste poets
(c) Lucifer is chewing on his head. (c) His Friends in the Irish Literary Revival
(d) His tongue is pulled out and nailed on the (d) Himself and his lady love, Maud Gonne
tree of sin. Ans : (c) "We were the last Romantics" was called by
Ans : (c) The reason why Judas was unable to speak in William Butler Yeats to his friends in the Irish Literary
Dante's Inferno is that – Lucifer is chewing on his head. Revival and P.B. Shelley.
Hence option (c) is correct. Hence option (c) is correct.

UGC NET/JRF English Paper II, July 2018 262 YCT


78. Who wrote The Wandering Jew, a poem in four Ans : (d) During the reign of Norman Kings, it was
cantos and the short lyric, "The Wandering fashionable to speak French in upper class circle in
Jew's Soliloquy"? England.
(a) S.T. Coleridge Because the literary of French community as well as
(b) Lord Byron their works in literature were very impulsive that time.
(c) Thomas Gray Hence option (d) is correct.
(d) P.B. Shelley 82. Who, among the following, collaborated with
Ans : (d) 'The Wandering Jew' is a short lyric in four Purohit Swami in translating the Ten Principal
cantos was written by P.B. Shelley. Upanishads into English?
Hence option (d) is correct. (a) Christopher Fry
79. Where, according to T.S. Eliot, are we likely to (b) Aldous Huxley
find "not only the best, but the most individual (c) Lawrence Durrell
parts of a poet's work''? (d) W.B. Yeats
(a) in the poet's juvenilia or rejected drafts. Ans : (d) W. B. Yeats collaborated with Purohit Swami
(b) in the best anthologies and scrap-books. in translating the Ten Principal Upanishads into
(c) in those parts where the dead poets assert English.
their immortality. Hence option (d) is correct.
(d) in those parts where the living poets depart 83. What unique distinction does Ben Jonson's "To
from their ancestors. Penshurst" have in the English literary canon?
Ans : (c) According to T.S. Eliot, are we likely to find (a) It is the only distinguished poem in English
"not only the best, but the most individual parts of a addressed to the Lords of Penshurst.
poet's work'' in these parts where the dead poets asserts (b) It celebrates Philip Sidney's elevation to
their immortality. knighthood, Sidney being the youngest scion
Hence option (c) is correct. of the family.
80. Which of the following is true of The (c) It is one of the first English poems
Canterbury Tales? celebrating a specific place, a forerunner to
(a) Chaucer, the pilgrim, narrates Sir Thopas' Cooper's Hill and Windsor-Forest.
Tale only. (d) It is the first poem in an elegiac series that
(b) Chaucer, the pilgrim, narrates The Tale of late Elizabethan poets began on the demise of
Melibee only. the Lord of Penshurst.
(c) Chaucer, the pilgrim, narrates both Sir Ans : (c) To Penshurst is a 102 line poetic letter written
Thopas' Tale and The Tale of Melibee. in heroic couplet. It is one of the first English poems
(d) Chaucer, the pilgrim does attempt to narrate celebrating a specific place, a forerunner to Cooper's
an unnamed tale but abruptly stops due to the Hill and Windsor-Forest. It is written by Ben Johnson
intervention of the other pilgrims. (1572-1637). Ben Johnson is among the best known
Ans : (c) Option (c) is true about 'The Canterbury Tale' writers and theorist of English Renaissance literature
Chaucer, the pilgrim, narrates both sir Thopas' Tale and second in reputation only to Shakespeare.
The Tale of Melibee. Hence option (c) is correct.
Hence option (c) is correct.
84. It is well known that in many of his plays, Tom
Geofferey Chaucer considered the father of English
Stoppard has consciously drawn upon earlier,
Literature and The Canterbury Tales supposed 1st
often reputed, works. Match the following
complete English work.
Stoppard plats with earlier works whose spitit
81. During the reign of Norman Kings, it was seems to have informed them.
fashionable to speak _____ in upper-class circle
List-I List-II
in England.
(a) Norse A Rosencrantz and i Hamlet
(b) Latin Guildenstern Are
(c) Danish Dead
(d) French B Indian Ink ii A Passage to India

UGC NET/JRF English Paper II, July 2018 263 YCT


C Inspector Hound iii The Mousetrap Identify the poet.
D Travesties iv Importance of (a) Thomas Gray
Being Earnest (b) John Dryden
Code: (c) John Milton
(A) (B) (C) (D) (d) Thomas Wyatt
(a) (iii) (ii) (i) (iv) Ans : (a) Samuel Johnson has said about Thomas Gray.
(b) (i) (ii) (iv) (iii) Hence option (a) is correct.
(c) (iv) (iii) (i) (ii) 88. "Take the smoking disclaimer issue" begins
(d) (ii) (i) (iv) (iii) Vishal Bharadwaj. "Putting a disclaimer every
Ans : (*) The marks has been awarded to all the time somebody smokes on screen is not an
candidates against the question. answer. If M.F. Hussain has painted a man
Rosencrantz and Guildenstern are dead is written by with a cigar, would you have asked him to put
Tom Stoppard. The title is taken from – William the disclaimer, 'Cigarette smoking is injurious
Shakespeare tragedy 'Hamlet. to health' on the painting"?
The point Bharadwaj makes with his rhetorical
85. After discovering the truth about his heinous
question is the following:
crimes committed in the past, what does
(a) The smoking disclaimer is ineffectual
Oedipus request as his punishment?
because M.F. Hussain's painting wouldn't
(a) exile
have carried it.
(b) castration
(b) The smoking disclaimer on objects perceived
(c) decapitation
as 'art' is simply superfluous.
(d) blindness
(c) The smoking disclaimer is ineffectual
Ans : (a) After discovering the truth about his heinous because 'art' entertains but does not instruct.
crimes committed in the past, Oedipus request to exiled (d) The smoking disclaimer on screen or on an
him. Hence option (a) is correct. M.F. Hussain painting distracts us from
86. How does Women in Love open? enjoying art.
(a) Rupert Birkin, Lawrence's alter ego, is taking Ans : (b) The point Bharadwaj makes with his
a walk in the English Countryside. rhetorical question is – The smoking disclaimer on
(b) The Brangwen sisters, Ursula and Gudrun, objects perceived as 'art' is simply superfluous.
are "working and talking." Hence option (b) is correct.
(c) The wedding party gathers at shortlands, the 89. According to ________, certain verbs actually
Criches's home. 'perform' an act when they are uttered.
(d) The last lesson is in progress, "peaceful and (a) Speech Act theorists such as Austin and
still" in Ursula's classroom. Searle.
Ans : (b) Women in Love is a novel by British author (b) Russian Formalists such as Shklovsky and
D. H. Lawrence. The novel opens with The Brangwen Propp.
sisters, Ursula and Gudrun, are working and talking. (c) Language theorists such as Sapir and Whorf.
Hence option (b) is correct. (d) Cognitive linguists such as Lakoff and
87. Samuel Johnson has the following to say about Johnson.
an English poet: Ans : (a) According to Speech Act theorists such as
"These images are marked by glittering Austin and Searle, certain verbs actually 'perform' an
accumulations of ungraceful ornaments: they act when they are uttered.
strike, rather than please. The images are Hence option (a) is correct.
magnified by affectation: the language is
90. Haunted castles, strange noises, and an
laboured into harshness. The mind of the
acceptance of the supernatural with all its
writer seems to work with unnatural violence-
trappings mark_____.
'Double, double, toil and trouble'. He has a
(a) meta fiction
kind of strutting dignity, and is tall by walking
(b) fantasy fiction
on tiptoe. his art and his struggle are to visible,
and there is too little appearance of ease and (c) epistolary fiction
nature." (d) gothic fiction

UGC NET/JRF English Paper II, July 2018 264 YCT


Ans : (d) "Haunted Castles, strange noises, and an Ans : (c) Option (c) is correct according to their periods
acceptance of the supernatural with all its trappings (early, middle, late)–
mark" are the characteristic of Gothic fiction. Love's Labours Lost (1594-95), Midsummer Night's
Hence option (d) is correct. Dream (1595), As You Like It (1599-1600), Antony
and Cleopatra (1606), The Tempest (1610-11).
91. ..... sure it waits upon
Hence option (c) is correct.
Some god o' th' island. Sitting on a bank,
93. Who among the following is not a reader-
Weeping again the King my father's wrack,
response critic?
This music crept by me upon the waters,
(a) Maud Bodkin
Allaying both their fury and my passion
(b) Hans-robert Jauss
With its sweet air. Thence I gave followed it,
(c) Stanley Fish
Or it hath drawn me rather...... (d) Wolfgang Iser
Which of the following statements on this
Ans : (a) Maud Bodkin is not a reader response critic.
passage are true? She is famous for her 1934 book 'Archetypal Patterns in
(a) These lines, spoken by Edgar in King Lear, Poetry' Psychological studies of Imagination. Amy
are part of a long speech delivered on the Maud Bodkin (1875-1967) was an English classical
heath. scholar writer on mythology and literary critic not a
(b) These lines, spoken by Ferdinand in The reader response critic.
Tempest, describe Ariel's music. Hence option (a) is correct.
(c) The passage reappears in an altered and 94. Leo Tolstoy's Anna Karenina's closing lines
ironic version in T.S. Eliot's Waste Land. present.....
(d) The passage reappears verbatim in W.H. (a) a sad reflection on the unfortunate suicide of
Auden's Sea and the Mirror. Anna which should have been averted.
The correct answer according to the code is (b) the enlivening freshness of a rain which has
____. been threatening to break out.
(a) (i) and (iv) (c) Levin's affirmation that whatever happens to
(b) (ii) and (iii) him, life is not meaningless but
unquestionably meaningful.
(c) (iii) and (iv)
(d) Vronsky's lament over the death of Anna
(d) (i) and (iii)
which ends on a positive note, affirming the
Ans : (b) Option (b) is correct. The above passage is human tendency to pass over the tragic
taken from T.S. Eliot's 'Waste Land'. These lines are events with hope.
spoken by Ferdinand in the Tempest, described Ariel's Ans : (c) Leo Tolstoy's Anna Karenina's closing lines
music. Eliot has taken this reference in his Waste Land.
present " Levin's affirmation that whatever happens to
Hence option (b) is correct. him, life is not meaningless but unquestionably
92. Arrange the following plays of Shakespeare meaningful."
according to their periods (early, middle, Hence option (c) is correct.
late......) of composition. 95. Which of the following novels begins with a
(a) As You Like It, Love's Labours Lost, Antony Prologue under the title "The Storming of
and Cleopatra, The Tempest, Midsummer Seringapatam", saying :I address these lines
Night's Dream. written in India-to my relatives in England"?
(b) Antony and Cleopatra, The Tempest, (a) The Siege of Krishnapur by J.G. Farell
Midsummer Night's Dream, Love's Labours (b) The Moonstone by Wilkie Collins
Lost, As You Like It. (c) The Sign of Four by Sir Arthur Conan Doyle
(c) Love's Labours Lost, Midsummer Night's (d) The Jewel in the Crown by Paul Scott
Dream, As You Like It, Antony and Ans : (b) The Moonstone by Wilkie Collins novel
Cleopatra, The Tempest. begins with a Prologue under the title "The Storming of
(d) Midsummer Night's dream, Antony and Seringapatam", saying :I address these lines written in
Cleopatra, The Tempest. As You Like It, India-to my relatives in England"
Love's Labours Lost. Hence option (b) is correct.

UGC NET/JRF English Paper II, July 2018 265 YCT


96. In "Gerontion", T.S. Eliot sys: I am standing at the top of the mountain.
".......... has many cunning passages, contrived Both my hands are free. And the rock has added
corridors/And issues, deceives with whispering height to the mountain.
ambitions,/ Guides us by vanities." 97. Whose poetic voice is triggered right from the
What is Eliot's subject? beginning?
(a) History (a) of student's
(b) Politics (b) of teacher's
(c) State (c) of critics
(d) Religion (d) of an observer's
Ans : (a) History has many cunning passage, contrived Ans : (b) Of teacher's poetic voice is trigged right from
corridors/And issues, deceives with whispering beginning.
ambitions,/Guides us by vanities. This line appears in T. Hence option (b) is correct.
S. Eliot's famous poem 'Gerontion'. 98. The speaker brings up the story of Sisyphus
Hence option (a) is correct. specially by way of glossing _________.
Read the following poem and answer questions 97 to (a) art in life
100. (b) life in art
THE MOUNTAIN (c) endless labour
My students look at me expectantly. (d) poetic expectation
I explain to them that the life of art is a life Ans : (c) The speaker brings up the story of Sisyphus
of endless labor. Their expressions specially by way of glossing endless labour.
hardly change; they need to know Hence option (c) is correct.
a little more about endless labor. 99. In its context, the words "their fingers/tapping
So I tell them the story of Sisyphus, at the wooden desks", best represent the
how he was doomed to push student's_______.
a rock up a mountain, knowing nothing (a) lack of protest
would come of this effort (b) lack of interest
but that he would repeat it (c) show of disrespect
indefinitely. I tell them (d) show of impatience
there is joy in this, in the artist's life, Ans : (d) The words "their fingers/tapping at the
that one eludes wooden desks", best represent the student's show of
impatience.
judgment, and as I speak
Hence option (d) is correct.
I am secretly pushing a rock myself,
100. Why does the speaker say that "the rock has
slyly pushing it up the steep
added height to the mountain"?
face of a mountain. Why do i lie
(a) because the speaker is already on the rop of
to these children? They aren't listening,
the mountain.
they aren't deceived, their fingers
(b) because both the hands of the speaker are
tapping at the wooden desks- now free.
So I retract (c) because the mountain now seems largely
the myth; I tell them it occurs incomprehensible.
in hell, ad that the artist lies (d) because she feels that the immensity of the
because he is obsessed tieh attainment, problem has grown.
that he perceives the summit Ans : (d) The speaker says that "the rock has added
as that place where he will live forever, height to the mountain" because she feels that the
a place about to be immensity of the problem has grown.
transformed by his burden: with every breath, Hence option (d) is correct.

UGC NET/JRF English Paper II, July 2018 266 YCT


UGC NTA NET/JRF Exam, December-2018
ENGLISH
Solved Paper-II
1. Match the writer with the work: (a) Michael Ondaatje's The English Patient
(Writer) (Name of work) (b) A.S. Byatt's Possession
(A) George Pattenham (i) Leviathan (c) John Fowles's The French Lieutenant's Woman
(B) Thomas Sprat (ii) The Practice of (d) Graham Swift's Waterland
Piety Ans: (d) Graham Swift's 'Waterland' (1983) novel is
(C) Lewis Bayly (iii) The Art of English considered the author's premier novel. The title of the
Poesy novel refers to its setting in The Fens in East Anglia.
(D) Thomas Hobbes (iv) The History of the Waterland is concerned with the nature and
Royal Society importance of history as the primary source of 'New
Code:
Historicism'. It also fall under the category of
(a) (A)-(iii), B)-(iv). (C)-(i), (D)-(ii)
postmodern literature such as a fragmented narrative
(b) (A)-(iii), (B)-(iv), (C)-(ii), (D)-(i)
style in chronological order. The plot of the novel
(c) (A)-(iv), (B)-(iii), (C)-(ii), (D)-(i)
basically revolves around loosely interwoven themes
(d) (A)-(iii), (B)-(ii), (C)-(iv), (D)-(i)
and narrative including brother's jealousy for
Ans: (b)
plotter/narrator's wife, resulting murder, abortion the
(A) George Pattenham (iii) The Art of
girl, her subsequent inability to conceive, resulting in
English Poesy
(B) Thomas Sprat (iv) The History of the depression and kidnap of a baby.
Royal Society In 1992, the book was made into a film.
(C) Lewis Bayly (ii) The Practice of Graham Swift's Notable works–
Piety (1) Shuttlecock (1981)
(D) Thomas Hobbes (i) Leviathan (2) Waterland (1983)
2. I. Allan Sealy's, The Trotter-Nama traces the (3) Last Orders (1996)
history of the Anglo-Indian community in a Graham Swift's Notable Awards–
chronicle of seven generations of the Trotter (1) Memorial Prize (1983)
family, told by the seventh Trotter. This (2) Booker Prize (1996)
narrator is: (3) James Tait Black memorial Prize (1996).
(a) A forger of Indian miniatures 4. The en ending to denote the plural nouns as in
(b) an accountant in the Indian army (oxen children, brethren) has survived from the
(c) a quack in the Indian outback (a) Odd Middle-English pronouncing custom of
(d) a collector of rare manuscripts plurals
Ans: (a) Irwin Allan Sealy, first novel 'The Trotter (b) Anglo-Norman case of making plural nouns
Nama' was published in 1988 and tells the story of (c) Middle English hymnals and chants in English
seven generations of an Anglo-Indian family 'Trotter parishes
family'. Trotter nama is the narrative of a great house (d) Old English practice of making plural nouns
built on fortunes made from Indigo and mines Ans: (d) Most singular nouns form the plural by
gradually declining into poverty. Seventh Trotter was adding 'S'.
a longer of Indian miniatures. Example– Boat – Boats, house – houses.
Irwin Allan Sealy's awards– A singular Noun ending in S, X, Z, ch, sh makes the
(1) Sahitya Akademic Award (1991). plural by adding – es
(2) Padma Shri (2012), Example – Bus – Buses, Wish – Wishes, Pitch –
3. 'Reality is that nothing happens. How many of Pitches, Box – Boxes.
the events of history have occurred, ask A singular noun ending to a consonant Y then Y
yourselves, for this and for that reason, but for makes the plural by dropping the Y and adding – ies.
no other reason, fundamentally, than the desire
Example – Baby – Babies
to make things happen? I present to you
Diary – Diaries
History, the fabrication, the diversion, the
City – Cities.
reality-obscuring drama."
All the above rules are Old English practice of making
Which postmodern novel thus subverts the truth
claims of traditional historiography? plural nouns.

UGC NTA NET/JRF English Paper II, December 2018 267 YCT
5. What type of writing did Walter Pater define as William Godwin was an journalist, political
"the special and opportune art of the modern philosopher and novelist as well as one of the first
world"? exponents of utilitarianism and the first modern
(a) The lyric (b) Comic drama proponent of anarchism.
(c) Nonfiction prose (d) The novel Literary works–
Ans: (c) Walter Horatio Pater (1839-1894), bigot on (1) An Enquiry Concerning Political Justice (attack on
Christianity basically popular in works on political institution)
Renaissance subjects in visited Florence, Pisa and (2) Things as They Are (1794).
Ravenn. Mainly Pater was a fictional writer, English 8. In Marlow's Doctor Faustus, what books does
Essayist, literary and art critic and one of the great Valdes counsel Faustus to study in preparation
stylists. for conjuring up spirits?
Walter Pater define Non fictional prose as 'the special (i) The works of Bacon and Abanus
and opportune art of the modern world. (ii) The Hebrew Psalter and New Testament
Basically literary work is based on facts even though (iii) The works of Ovid and Homer
it may contain fictional elements. (iv) The works of Baxter and Horst
His Non fictional works– The right combination according to the code is:
(1) The Picture of Dorian Gray (1890) (a) (ii) and (iii) (b) (i) and (iv)
(2) The Age of Innocence Edith Wharton's (1920) (c) (i) and (ii) (d) (i) and (iii)
praised as a 'wonderful new volume.'
Ans: (c) The Tragical History of the life and death of
(3) The Renaissance (1873)
Doctor Faustus, commonly known as Docter Faustus
(4) Marius the Epicurean (1885).
is an Elizabethan Tragedy by Christopher Marlowe
6. Which of the following statement is true of The based on German Stories about the title character
Way of the World?
Faust. For the preparation of conjuring up spirits
(a) The Way of the World failed on stage
Faustus study The works of Bacon and Abanus as
(b) Millamant and Mirabell fail to obtain the
well as The Hebrew Psalter and New Testament.
consent of Millamant's aunt for their marriage.
Francis Bacon– A famous essayist and known as the
(c) The Way of the World was performed and
father of essay.
published in 1702.
Abanus Pietro d'Abano – An Italian philosopher,
(d) The Way of the World presents a heroine
astrologer and physician.
pretending to love an older man.
The Hebrew Psalter– The first book of the Ketuvim,
Ans: (a) William Congreve (1670-1729) was English
the third section of the Hebrew bible and consider as
playwright, written numerous plays, but vital
Old Testament.
impression of The Way of the World was, and
The New Testament– The second book of the
regarded as one of the best Restoration comedy. Now
Christian biblical canon. It discuses the teaching and
days It occasionally performed. Initially, however the
play struck many audience member as continuing the person Jesus.
immorality of the previous decades. 9. Which of the following poems is quoted as the
It's setting was London, date premiered 17000 and epigraph to A Raisin in the Sun by Lorraine
published in 1702. Restoration comedy feature can be Hansberry?
seen easily on the opposition of the wity, courtly rake (a) "The Big Sea"
and the dull witted man of business or the country (b) "I, too, Sing America"
bumpkin. (c) "The Negro Speaks of Rivers"
But the play ‘The Way of the World’ like another (d) "Harlem (A Dream Deferred)"
plays of Restoration did not found glory on the stage Ans: (d) An epigraph is an affective literary tool that
but failed on stage which attacks on aristocratic some writers utilize to focus the reader toward the
privilege. theme purpose or concerns behind the work.
7. Which of the following had the alternative title The epigraph to A Raisin in the Sun is Cangston
Things as They Are? Hughes poem "Montage of a Dream Deferred" which
(a) Mary Shelley's Frankenstein was written as a critique of Harlem life. The eleven
(b) Sir Walter Scott's Waverley lines are a hypothesis about the ramifications of white
(c) William Godwin's Caleb Williams society's actions to withhold equal opportunity from
(d) Horace Walpole's The Castle of Otranto black citizens. Hughes main point is that there could
Ans: (c) William Godwin's (1756-1836) ‘The be serious consequences when peoples frustrations
Adventures of Caled Williams’ is known alternative accumulate to a point where they must either
title of Things as They Are. Godwin wrote what he surrender to their dreams or allow circumstances to
saw as a tyrannical government. snuff out their aspirations.
UGC NTA NET/JRF English Paper II, December 2018 268 YCT
10. Adherents of the fourteenth century religious WEDDING, touches upon one of Philip Larkin's
movement associated with vernacular favourite themes in a more explicitly humorous way
preaching, translation of New Testament into than many of his most famous poems.
English, and challenges to the authority of Notes: Brunette Coleman was a pseudonym used by
priests and bishops were called the poet and writer Philip Larkin. She is mainly a
(a) Lollards (b) Levellers woman writer. The last stanza of poem.
(c) Deists (d) Agnostics Don’t read much now: the dude who lets the girl down
Ans: (a) Lollard was the popular derogatory before The hero arrives, the chap whose yellow and
nickname given to those who were without an keeps the store seem far to familiar. Get Stewed:
academic background, educated only in English, who Books are a load of crap.
were imitations of John Wycliffe's teaching and were 13. Who among the following exemplified the role of
certainly considerable energized by the translation of the "peasant poet"?
the Bible into the English language. Lolland had come (i) John Clare
th
to mean a Heretic in general by the mid 15 century.
(ii) John Keats
It was mainly a pre-protestant Christian religious
th (iii) William Cobbett
movement that existed from the mid 19 century to
(iv) Robert Burns
the English Reformation. It was started by John
The right combination according to the code is:
Wycliffe a Roman Catholic theologician. The
Lollards' demands were primarily for reform of (a) (i) and (ii) (b) (iii) and (iv)
Western Christianity. (c) (i) and (iv) (d) (ii) and (iii)
11. What is an "implied reader"? Ans: (c) John Clare and Robert Burns are name as
(a) A reader who embodies all those peasant poets. Under the influence of ideas about
predispositions necessary for a literary work to original genius, the image of the peasant poet was a
exercise its effect. pervasive part of literary culture and the standard
(b) The ideal audience envisioned by the author means for representing acceptable working class
and to whom the work of literature is literature.
supposedly addressed. John Clare Biographes Jonathan Bate, State was "the
(c) The ideal reader of a work of literature which greatest labouring class poet that England has even
is approximated over time by successive produced. No one has ever written more powerfully of
responses of generations of actual readers. nature of a rural childhood and of the alienated and
(d) The ideal "average" reader who can approach unstable self.
a work of literature with no preconceived Robert Burns also known as Rabbie Burns, the Bard
ideas about the author's life, the time of of Ayrshire, Ploughman poet and various other names
composition, etc. and epithets. He was widely regarded as the national
Ans: (a) The term "implied reader" coined by booth poet of Scotland.
in 1961. The term implied reader is a function of the 14. It was the first narrative on the life of a black
work even though it is not presented in the work. The woman slave to be published in England in 1831.
implied reader is ultimately one of the attributes of the It has profound influence on the abolition
concrete reader's reconstructed implied author. It
movement in Britain. Identify the book and its
follows that the implied reader is no less dependant on
author
the reader's individual acts of reading, understanding
(a) Harriet Jacobs – Incidents in the Life of a Slave
and reconstructing than the implied author whose
Girl
attribute it is.
Although An 'Intended reader is not fixed in the text (b) Mattie Jane Jackson – The Story of Mattie J.
but exists merely in the imagination of the author and Jackson
who can be reconstructed only with the letter's (c) Mary Prince – The History of Mary Prince
statements or extra-textual information, does not form (d) Elizabeth – Memoir of Old Elizabeth, a
a part of the work. Coloured Woman
12. Which post-War British poet ends a poem with Ans: (c) Mary Prince – The History of Mary Prince.
the line, "Get Stewed : Books are a load of Mary Prince (1788-1833) born in Bermuda to an
crap"? enslaved family of African descent. She wrote her
(a) Philip Larkin (b) Craig Raine slave narrative ‘The History of Mary Prince’, which
(c) Thom Gunn (d) Ted Hughes was published (1831) in London written on any black
Ans: (a) Philip Larkin's poetry 'A Study of Reading woman life. The first hand description of the
Habits' contained the printed lines and Larkin known brutalities of enslavement, release at a time when
as a British post war poet. It was written in August Bermuda was slave of British had a galvanizing effect
1960 and published in 1964 volume THE WHITSUN on the anti slavery movement.

UGC NTA NET/JRF English Paper II, December 2018 269 YCT
15. One of the most flexible metres, _____ is a five Who are Amaryllis and Neaera in the above
foot line. It was introduced by Geoffrey Chaucer extract from John Milton's "Lycidas"?
in the fourteenth century and has since then (a) Amaryllis is a shepherdess mentioned in
become the commonest of metres in English Shakespeare's romantic comedies, Neaera, a
poetry. minor character in Love's Labour's Lost.
Fill in the blanks (b) Both were one-time lovers of Lycidas, the dead
(a) Iambic (b) Pentameter shepherd.
(c) Trochaic (d) Hexameter (c) Both were goddesses of love and war
Ans: (b) Pentameter, a line of verse consisting of five respectively appearing in Greek pastoral
metrical feet on (in Greek and Latin verse) of two poetry.
halves each of two feet and a long syllable. (d) Amaryllis is a shepherdess mentioned in
It is a poetic meter. A poem is said to be written in a ancient pastoral poetry, notably in Virgil's
particular pentameter, which have the length of five Eclogues, Neaera, a nymph who appears in
feet. Foot is a combination of unstressed syllable and Virgil's Eclogues.
stressed syllable. Depending on the pattern of feet Ans: (d) Lycidas is a poem by John Milton, written in
pentameter can be iambic (one of three two-syllable
1637 as a pastoral elegy.
meters alongside trochaic and spondaic) or Dactylic
Herodotus is his book IX mention an Athenian
(one of two three syllable meters alongside anapestic).
councilor in salamis, "a man named Lycids".
16. This poet was of the Auden generation and was Amaryllis pointed out in lines 64-76 of Lycids. It is
only briefly a member of the Communist party. basically the character, in Roman literature, a stock
In his poem, "The Pylons", he averred that the
female character, a natural pretty young women, who
Pylons are "Bare like nude giant girls that have
was usually a shepherdess. Amaryllis is mentioned in
no secrets". This prompted the label. Pylon
classical pastoral poetry and in later work such as
Poets, for the new generation of poets who were
Thomas Comlion's "I Care Not for These Ladies"
happy to use the gas works or pistons of a
(1601) and also mention a shepherdess in an ancient
steam-engine as poetic imagery. (Name the poet)
(a) Stephen Spender pastoral poetry notably in Virgil's Eclogues:
(b) Cecil Day Lewis Neaera a nymph who appears in Virgil's Eclogues and
(c) Louis MacNeice John Milton Lycidas.
(d) Christopher Isherwood 18. Evelina was published in 1778
Ans: (a) Stephen Spender (1909-1995) full name Sir (a) posthumously
Stephen Harold Spender CBE was an English poet, (b) under a pseudonym
novelist and essayist who focused on themes of social (c) using the name Fanny Burney
Injustice and class variation struggle. He was (d) anonymously
appointed Poet Laureate Consultant in Poetry to the Ans: (d) “Evelina” or the ‘History of a Young Lady's
United States Library of Congress in 1965. Entrance into The World’ is a novel written by
Notable works– English author Fanny Burney and published in 1778.
(1) The Pylons It was published anonymously; its authorship was
(2) The Generous Days revealed by the poet George Huddesford in what
(3) Nine Experiments Burney called a 'Vile Poem'.
(4) Poems of Dedication Anonymously writers hide their real name because of
(5) Dolphins society influenced or to create secrecy for mysterious
Drama– fame. In 18th century literature was the period of male
(1) Trial of Judge chauvinist society so the ladies writers hide their name
(2) The Oedipus Triology and used pseudonym on the place of their real name.
Now over these small hills, they have built the
19. Who among the following are referred to as the
concrete
"Scottish Chaucerians"?
That trails black wire
Pylons, those pillars (i) Thomas Hoccleve
Bare like rude giant girls that have no secret. (ii) Robert Henryson
(iii) John Lydgate
17. Alas! what boots it with uncessant care
(iv) William Dunbar
To tend the homely, slighted Shepherd's trade,
And strictly meditate the thankless Muse? The right combination according to the code is:
Were it not better done, as others use, Code:
To sport with Amaryllis in the shade, (a) (iii) and (iv) (b) (i) and (ii)
Or with the tangles of Neaera's hair? (c) (ii) and (iii) (d) (ii) and (iv)

UGC NTA NET/JRF English Paper II, December 2018 270 YCT
Ans: (d) Chaucer's influence on 15th century Scottish Samuel Taylor Coleridge (1875-1912) was an English
literature began toward the beginning of the century composer of classical and musician metable
with King James I of Scotland. There were two phase composition–
of Chaucerianism, the first phase of Scottish followed (1) Ballade in D Minor (1895)
by a second phase comprising the works of Robert (2) Fife Fantasiestuke (1896)
Henryson, William Dunbar and Gavin Douglas. The (3) Two Romantic Pieces (1896)
first phase of Scottish Chaucerianism which 22. Identify the Fireside poets of the US.
purposefully and directly imitates the work of (a) Amy Lowell, Emily Dickinson, Phillis
Chaucer while preserving the Scottish author's own Wheatley
uniqueness. (b) William Cullen Bryant, H.W. Longfellow,
The second phase of Scottish Chaucerianism the Oliver Wendell Holmes
poems Robert Henryson, William Dunbar and Gavin (c) Marianne Moore, Elizabeth Bishop, Anne
Douglas represent a more academic one, which adopts Seaton
Chaucer style to the author's more individual and (d) T.S. Eliot, Wallace Stevens, William Carlos
nationalistic purpose. Williams
20. The enigmatic castle which K. attempts to reach Ans: (b) The Fireside poets also known as the
Schoolroom or Household Poets were a group of 19th
in vain in Franz Kafka's The Castle belongs to
century American poets associated with New
(a) Count Aloofwest (b) Count Strangewest
England. For their domestic themes and messages of
(c) Count Westwest (d) Count Eastwest
morality presented in conventional poetic forms
Ans: (c) Franz Kafka's (The Castle) is, in my humble deeply shaped their era until their decrease in
opinion, one of the greatest existentialist fictions ever popularity at early stage of 20th century these poets
written. It contains all of the startle emotions were very popular among readers and critics both in
existentially minded readers look for confusion, U.S. and elsewhere.
isolation, immobility, estrogenet and shadow mean of Overview, The group is typically thought to include
tragedy. The protagonist in the story K. who by all Henry Wordsworth Longfellow, William Cullen
descriptions is a simple surveyor sent to measure the Bryant, John Greenleof Whittier, James Russel Louzel
land cannot reach the count of the castle (Count and Oliver Wendell Holmes., who were popular not
Westwest). only in American poets but also rivaled that of British
Franz Kafka (1883-1924) was a German speaking poets.
Bohemian Jewish novelist and short story writer, 23. Thomas Nashe's The Unfortunate Traveller is
widely regarded as major figures of 20th century narrated by
literature. (a) Philip Foxe, an English highwayman
Notable Works– (b) Ben Lyte, a coarse Papist
(1) Die Uerwandlung (The Meta Morphosis) (1915) (c) Jack Wilton, an English page
(2) Rear Process (The Trial) (1925) (d) Peter Marston, a sworn Calvinist
(3) Das Schloss (The Castle) (1926) Ans: (c) Thomas Nashe's ‘The Unfortunate Traveller’
(4) Brief and Felice (Letters to Noted Felice) (1973) (1594) is a brutal and realistic tale of adventure
21. S.T. Coleridge's "Dejection : An Ode" opens narrated with speed and economy. The book describes
with an epigraph which is a reference to a the travels through Germany and Italy of its rouge
ballad. Identify the ballad. hero and An English page Jack Wilton, who lives by
(a) "Ballad of Sir Patrick Spence" his wits and witnesses all sort of historic events.
(b) "Ballad of the Goodly Fere" Thomas Nashe (1567-1601) was an Elizabethan
(c) "Ballad of the Gibbet" playwright, poet, satirist and a significant pamphlets.
(d) "La Belle Dame Sans Merei" He is known for his great novel The Unfortunate
Ans: (a) Samuel Taylor Coleridge's "Dejection : An Traveller, his pamphlets including Pierce Penniless
Ode" opens with an epigraph which reference to Sir and his numerous defences of the Church of England.
Patrick Spence's popular ballad "Child Ballads" (No Notable works–
58) and is of Scottish origin. It is a maritime ballad (1) Summer's Last Will And Testament (1600)
about a disaster at sea. (2) The Ishle of Dogs (1597).
Coleridge wrote in his notebook about Hutchinson 24. "The chapter on the Fall of the Rupee you may
and possible poems “Can see nothing extraordinary in omit. It is somewhat too sensational. Even these
her – a poem nothing all the virtues of the mild and metallic problems have their melodramatic
retired kind”. The original draft was titled "Letter to side." The fall of the Indian rupee in the final
Sara Hutchinson" and it became Rejection when he decades of 19th century is referred to in one of
sought to published it. Oscar Wilde's plays. Identify the play.
UGC NTA NET/JRF English Paper II, December 2018 271 YCT
(a) Lady Windermere's Fan Ans: (a) Federico Garcia Lorca's elegy "Lament for
(b) An Ideal Husband Ignacio Sanchez Mejias" on Lament for the Death of
(c) The Importance of Being Earnest Bullfighter is a long elegy divided into four parts
(d) A Woman of No Importance corresponding to four dramatic movements. It was
Ans: (c) The above lines are extracted from Oscar written to commemorate and celebrated the death of a
Wilde's play 'The Importance of Being Earnest' man who may considered the bravest and most gallant
matador of Spain. Ignacio Sanchez Mejias was also
(1895).
Federico Garcia Lorca's great friend. In this poem
Oscar Fingal O'Flahertie Wills Wilde (1854-1900)
there is complete identification between poet and
was an Irish poet and playwright. After writing in speaker.
different forms throughout the 1880's he became Federico Del Sagrado Carazon de Jesus Garcia Lorca
one of London's most popular playwright in the (1898-1936) was a Spanish poet, playwright and
early 1890s. He is best remembered for his theatre director.
epigrams and plays. His novel ‘The picture of Notable Works–
Dorian Gray’ and the circumstances of his criminal (1) Impresiones y poisajes (Impression and
conviction for homosexuality, imprisonment and Landscape)
early death at age 46. (2) Libro de poems (Book of poems 1921)
He is coined a literary movement Aestheticism. (3) Primerar Canciones (1936)
Notable Works– Select Translations–
(1) The Importance of Being Earnest (1895) (1) Poem of Deep Song
(2) Crypsy Ballads
(2) The Picture of Doeicm Gray (1890)
(3) Sonnet of Dark Love
Magzine–
27. Early African-American texts like slave
(1) The Pall Mall Gazette
narratives were often described as told to
25. Which of the following would not be invoked to narratives as their 'authors' dictated their
describe a form of new Historicist criticism? experiences. The persons who noted down these
(a) Cultural materialism experiences are
(b) Archaeology of social constructs (a) Translators (b) Slave-drivers
(c) Geneology of patriarchal discourse (c) Abolitionists (d) Amanuenses
(d) Post-structural recovery of authorial intent Ans: (d) Amanuensis–
Ans: (d) New Historicist Criticism is newly method A literary or artistic assistant in particular one who
adopted by Historicist Criticism in following rules and takes dictation on copies manuscript. Amanuensis
regulation– word originally taken from Latin Amanu which means
serves a manu (slave) at hand (writing) secretary and
(1) A very expressive act is embedded in a network
ensis means belonging to.
of material practices – Cultural Materialism.
Early 17th century : Latin from (serves) a manu (slave)
(2) That every act of unmasking, critique and at hand (writing), secretary + -ensis 'belonging to'.
opposition uses of the tools it condemns and risks
28. One of the less noticed and acknowledged
falling prey to the practice it exposes :- distinction of The Canterbury Tales is that
Archaeology of social constructs. (a) it upheld the idea that we cannot divorce poetry
(3) That literary and non-literary texts circulate from knowledge because poetry itself is an
inseparably. object of knowledge.
(4) That no discourse imaginative or archival gives (b) it alerted us to the term auctor, someone who is
access to unchanging truths nor express both 'an originator, or one who gives increase',
inalterable human nature. the best description for Chaucer himself.
(5) That a critical method and a language adequate to (c) it married domesticity to divinity, the baker's
describe culture under capitalism participate in loaf with the bread of life.
the economy they describe. (d) instead of revealing England's divisions, it
reveled in its diversity.
26. The titular figure of Federico Garcia Lorca's
Ans: (d) The Canterbury Tales considered as a first
elegy "Lament for Ignacio Sanchez Mejias" was
literary work following pentameter and express the
(a) a popular matador and writer diversity of England communism instead of Revealing
(b) a spy who helped the revolutionaries during the England's divisions.
Spanish Civil War The Canterbury Tales is a collection of 20 stories that
(c) a popular priest and poet runs to over 17000 lines written by Geoffery Chaucer.
(d) a revolutionary who was associated with Che Geoffery Chaucer was an English poet and known as
Guevara the 'Father of English Literature" and he was the first

UGC NTA NET/JRF English Paper II, December 2018 272 YCT
writer to be buried in poets corner of Westminster (a) Rudyard Kipling's "The White Man's Burden"
Abbey. (b) Flora Annie Steel's "The Garden of Fidelity"
Notable Poems– (c) J.R. Ackerley's Hindoo Holiday : An Indian
(1) Troilus and Griseyde Journal
(2) The Nun's Priest's Tale (d) Maud Diver's The English woman in India
(3) Parliment of Foules Ans: (c) J.R. Ackerley's Hindoo Holiday : An Indian
(4) The House of Fame Journa.l
Movies– 32. Identify the character, a black-eyed dwarf who
(1) A Knight's Tale "constantly revealed a few discoloured fangs
(2) The Canterbury Tales that were yet scattered in his mouth, and gave
29. Which of the following is the most accurate him the aspect of a panting dog".
description of Butler English? (a) Daniel Quilp in The Old Curiosity Shop
(a) A pidgin, also called "Kitchen English" spoken (b) Mr. Crook in Bleak House
by South Asians in Europe. (c) Mulberry Hawk in Nicholas Nickleby
(b) A minimal pidgin that emerged during colonial (d) Rigand in Little Dorrit
times in the Madras Presidency. Ans: (a) The Old Curiosity Shop is one of the two
(c) A dialect of English spoken by the descendants novels which published along with short stories in
of Anglo-Indians. Charles Dickens weekly serial ‘Master Humphrey's
(d) Any non-grammatical variety of English used Clock’ from 1840 to 1841.
by menials in Commonwealth countries. The character of a black eyed dwarf is Daniel Quilp.
Who is one of the main antagonists in the novel. Quilp
Ans: (b) Butler English, also known as Bearer
is a vicious, ill tempered and grotesque dwarf and is
English or Kitchen English is a dialect of English that
the villain of the novel.
first developed as an occupational dialect in the years Other characters of The Old Curiosity Shop – Nell
of the Madras Presidency in India. Irent, Dick Swiveller, Kit Nubbles, Garel Father.
The name derives from its origin with Butlers, the Charles John Huffam Dickens (1812-1870) was an
head servants of British colonial households and the English writer who figure out actually poverty
English that they used to communicate with their problems and life sketches of fictional character. he
masters. Butler English persisted into the second half regarded by many as the greatest novelist of the
of the 20th century beyond the Independence of India Victorian Era.
and was subject to Providian influence in its Notable Work–
phonology. (1) A Tale of Two Cities (1859)
30. In the spring of 1941, Nikos Kazantzakis (2) The Frozen Peep
embarked on one of his most ambitious projects, (3) Oliver Twist (1839)
a play known as Yangtze. What English/Greek (4) David Copper Field (Autobiography) (1849-50)
title is it now known as? (5) Great Expectations (1860-61)
(a) Brobdingnag (b) Zoroaster (6) A Christmas Carol (1843)
(c) Buddha (d) Zorba 33. "The good thing about words, "Hanif Kureishi
Ans: (c) Nikos Kazantzakis (1883-1957) was a Greek remarks in "Loose Tongues", is that their final
writer and known as a giant of Modern Greek effect is incalculable. […..] You can never know
literature. He was nominated for the Nobel Prize in what your words might turn out to mean for
Literature in nine different years. yourself or for someone else; or what the world
Yangtze is a river in China, but here Yangtze refers to they make will be like. Anything could happen.
the English Title Buddha, where writer talks about the The problem with silence is that we know
exactly what it will be like."
considerable and hypothesis story of Buddha's life and
Kureishi, in sum, suggests:
his journey.
(i) There is always some risk involved in
Notable Works– writing/speaking.
(1) Serpent and Lily (signed with the pen name Karma (ii) It is better to avoid using words than to risk
Nirvani) (1908) miscommunication.
(2) One act play title 'Comedy' (1909). (iii) Words being predictable, are always open to
31. During the Raj, the British viewed their rule in misinterpretation.
terms of a thankless duty to uplift the (iv) The unpredictable, in deed, is the strength
downtrodden and inculcate order into Oriental of words.
minds. The mission to civilize the "silent, sullen Determine the correct combination according to
peoples" of the East was a burden imposed upon the code:
them by destiny. Code:
The last observation is a fairly obvious allusion (a) (ii) and (iii) (b) (i) and (iii)
to (c) (i) and (iv) (d) (ii) and (iv)
UGC NTA NET/JRF English Paper II, December 2018 273 YCT
Ans: (c) Hanif Kureishi suggests that the writing of ….Literature….by refusing to assign a "secret",
any work focus always risk because of society an ultimate meaning, to the text (and to the
influence as well as contemporary writing pressure world as text) liberates what may be called an
and the exact words of expressing emotions as in anti-theological activity, an activity that is truly
speaking. Sometime the word contained a different revolutionary since to refuse to fix meaning is, in
meaning considered by reader. the end to refuse God and his hypostases –
Hanif Kureishi is a novelist of Pakistani, British reason, science, law."
playwright, screenwriter and filmmaker. The passage comes from which of the following
Written Works– essays?
(1) The Buddha of Suburbia (1990) (a) "The Death of the Author" by Roland Barthes
(2) The Black Album (1995) (b) "Discourse in the Novel" by Mikhail Bakhtin
(3) Gabriel's Gift (2001) (c) "What is an Author?" by Michel Foucault
(4) The Body (2003) (d) "Tradition and Individual Talent" by T.S. Eliot
Story Collections– Ans: (a) "The Death of the Author" the title is a
(1) Love On Blue Time (1997) reference to Le Morte d'Arthus an Arthurian legend
(2) Midnight All Day (1999) stories, written by Sir Thomas Malery. It's author is
(3) Collected Essays Faber and Faber (2011). French literary critic and theorist Roland Barthes
34. Arnold Wesker is associated with "kitchen-sink (1915-1990).
drama", a rather condescending title applied to Barthes argues in this essay political views, historical
the then new-wave realistic drama depicting the context, religion, ethnicity. psychology, biographical
family lives of working-class characters, on or personal attributes to distill meaning from the
stage and in broadcast plays. Two of the author's work.
following plays begin with one character doing Roland Barthes (1915-1980) French literary theorist,
the dishes in a kitchen sink. Identify the pair. linguist philosopher and Semiotician.
(i) The Kitchen Notable Works–
(ii) Chicken Soup with Barley (1) Death of the author
(iii) Roots (2) Writing Degree Zero
(iv) Menace (3) Effect of Reality
The right combination according to the code is: Movies–
Code: (1) Let the sunshine in
(a) (ii) and (iv) (b) (i) and (iv) (2) Desire In Motion
(c) (ii) and (iii) (d) (i) and (ii) (3) A Eespera
(4) Un Passatemo
Ans: (c) Arnold Wesker first of a triology play
Chicken Shop with Barly performed on stage at the 36. "Tomorrow, and tomorrow, and tomorrow,
Belgrade Theatre in Coventry in 1958. The two other ______in this petty pace from day to day,
plays of that triology – Roots and I'm Talking About To the last _______ of recorded time;
Jerusalem also premiered. The play is split into three And all our yesterdays have lighted fools
acts consisting each with two scenes. The plays are The way to dusty death. Out, out, brief candle!
"Kitchen Sink" Play/drama traces the Jewish Khan Life's but a walking shadow, a poor player,
family's downfall of their ideals in changeable world. That ______ and frets his hour upon the stage,
Wesker explores how they struggle to maintain their And then is ______ no more: it is a tale
conviction on the term of Jewish communist. Told by an idiot, full of sound and fury,
Sir Arnold Wesker was a widely known as English Signifying nothing."
Dramatist and author of 50 plays, short stories Fill in the blanks. Choose the set that carries the
containing four volumes, two volumes of essays. correct words.
Notable Works– (a) Creeps, moment, struts, seen
(1) Their Very Own and Golden City (b) Moves, syllable, frowns, heard
(2) The Wesker Trilogy (c) Walks, breath, creeps, shown
(3) The Journalists (d) Creeps, syllable, struts, heard
Awards– Ans: (d) Creeps in this pretty pace from day to day
(1) Italian Marzotto Prize To the last syllable of recorded time;
(2) Frontier Lifetime Achievement Award. That struts and frets his hour upon the stage,
35. "We know now that a text is not a line of words And then is heard no more; It is a tale.
releasing a single "theological" meaning (the 37. Given below are two statements, one labeled as
"message" of the Author-God) but a Assertion (A) and the other labeled as Reason
multidimensional space in which a variety of (R). Read the statements and choose the correct
writings, none of them original, blend and clash answer using the code given below:
UGC NTA NET/JRF English Paper II, December 2018 274 YCT
Assertion (A) : Gender studies do not see an Halo Calvino (1923-1985) was an Italian Journalist
urgent need to help us navigate the various pitfalls and writer of short stories and novels.
of racism, ethnocentrism, cultural relativism, and Notable Works–
plain ignorance that flow from using "culture" as an (1) Our Ancestors trilogy, the cosmiconics collection
explanatory tool. of short stories.
Reason (R) : Issues relating to women's rights, (2) Invisible Cities.
gender roles, sexuality and family obligations are (3) The Baron in the Trees.
centrally implicated in the so-called clash of Awards–
civilizations between Christianity or Secularism, (1) World Fantasy Award (1982)
and Islam. (2) Bagutto Prize (1959)
Code: (3) Viareggio Prize.
(a) (R) does not follow logically from (A). 40. In imitation of which classical poet did Samuel
(b) (A) is only partly addressed in (R). Johnson write his London and The Vanity of
(c) (R) is (A) and vice versa. Human Wishes?
(d) (A) and (R) are most logically related. (a) Horace (b) Tasso
Ans: (a) (R) does not follow logically from (A). (c) Juvenal (d) Homer
38. In Thomas Moore's Utopia (Book II), the reader Ans: (c) London and The Vanity of Human Wishes
is told that in this new world there are few wrote by Samuel Johnson in imitation of Roman poet
mistakes in marriage because Decimus lunius luvenalis, known in English as
(a) there is an extensive courtship period Juvenal, active in the late first and early second
preceding the actual wedding. century A.D.
(b) the family gods are invoked before finalizing Samuel Johnson known as Dr. Johnson was a poet,
the nuptials. playwright, essayist, moralist, literary critic,
(c) there is a community get together where biographer, editor and lexicographer.
prospective husbands and wives announce Notable Works–
wedding plans endorsed by elders. (1) A Dictionary of English Language (1755).
(d) prospective husbands and wives see one (2) A Journey to the Western Isles (1775).
another naked before agreeing to the match. 41. Which of the following acts were not passed
Ans: (d) Utopia (1516) in Latin, a socio political during the Victorian Era?
satire and fictional work of Thomas Moore. In Utopia, (a) The Married Women's Property Rights Act
Moore's describe a imaginative world which have not (b) The Custody Act
any earthly problem and the phenomena of each (c) The Women's Suffrage Act
aspects is heavenly. The life sketches of marriage (d) A series of Factory Acts
never satisfy wife and husband. They have to naked Ans: (c) Women's suffrage is the right of women to
one another before knowing. vote in election. Most independent countries enacted
His Works (CW– Couplet Work) women's suffrage in the interwar era, including
(1) Utopia (1516) (CW4) Canada in 1917. Britain over 30 in 1918, over 21 in
(2) Responsio ad Lutherum (1523) (CW 7) 1928, Germany, Poland in 1918 and the United States
(3) Against Erith (1532) (CW 7) in 1920 (Voting Rights Act of 1965 secured voting
(4) Apology (1533( CW 9). rights for racial minorities).
39. In which of his novels does Italo Calvino Victorian Era considered was the period of Queen
construct his narrative through a tarot pack of Victoria's reign. From 20 June 1837 until her death
cards and re-interpret the Western canon on 22 January 1901.
providing new versions of Oedipus Rex, Faust, The Married Women's Property Act - 1888.
Hamlet, Macbeth and King Lear? The Custody Act/Custody of Infants Act – 1839.
(a) The Castle of Crossed Destinies A Series of Factory Acts – 1833.
(b) The Path to the Nest of Spiders 42. The fault of Cowley and perhaps of all the
(c) Invisible Cities writers of the metaphysical race is that of
(d) Our Ancestors pursuing his thoughts to their ramifications, by
Ans: (a) The Castle of Cross Destinies (1973), written which he loses the grandeur of generality; for of
by Italian writer Italo Calvino's his inspiration turns the greatest things the parts are little; what is
the seventy-eight tarot-cards to an entirely new use little can be but pretty, and by claiming dignity
that links image and word to the purposes of narrative becomes ridiculous. Thus all the power of
: a band of travelers, trapped in an ominous castle in description is destroyed by a scrupulous
the heart of a wood find themselves struck dumb and enumeration; and the force of metaphors is lost,
is only able to converse by using the tarot cards to tell when the mind by the mention of particulars is
each other how they came to the castle. turned more upon the original than the
UGC NTA NET/JRF English Paper II, December 2018 275 YCT
secondary sense, more upon that from which the C. "Ode to a (i) "My heart aches, and
illustration is drawn than that to which it is Nightingale" a drowsy numbness
applied. pains My sense, as
(Life of Cowley, 1779) though of hemlock I
What Dr. Johnson actually faults here is: had drunk."
(a) The mind that goes astray toward the original.
(b) The force of metaphors that blunts description.
(c) The metaphysical insistence on the particular D. "Ode on (ii) "No, no, go not to
than the general. Melancholy" Lethe, neither twist
(d) The metaphysical poets' tendency to saunter Wolf's-bane, tight-
away. rooted, for its
Ans: (d) The Metaphysical Poets – The Term was poisonous wine."
coined by the Samuel Johnson to describe a loose 44. Why did Plato banish the poet from his ideal
group of 17 Century English poets, whose work was state?
characterized by the inventive use of conceits and by a (a) In representing the sensual aspects of reality,
greater emphasis on the spoken rather than lyrical the poet falls to discern the transcendent reality
quality of their verse. Metaphysical poets called behind mere appearance.
Baroque poets because of the diversity of style. The (b) Poetry deals with from, to the neglect of
metaphysical style was established by John Donne. content.
43. Match the poem with the opening lines: (c) Poetry makes an artificial distinction between
A. "Ode to Psyche" (i) "My heart aches, and a form and content.
drowsy numbness (d) The poet can never produce a completely
pains My sense, as accurate replica of the reality it seeks to
though of hemlock I represent, and (moreover) the purpose of art is
had drunk." not to describe reality but to change it.
B. "Ode on a (ii) "No, no, go not to Ans: (a) Plato banished the poet from his ideal state
Grecian Urn" Lethe, neither twist because the actuality or reality in poet's works are not
Wolf's-bane, tight- seen, poets use a lot of imaginations and ornaments to
rooted, for its decorate the words and represent beyond reality. In
poisonous wine." representing the sensuous aspects of reality, the poet
C. "Ode to a (iii) "Thou still unravish'd falls to discern the transcendent reality behind mere
Nightingale" bride of quietness, appearance.
Thou foster-child of 45. Which interpretation of Keats's "Beauty is
silence and slow time," truth, truth beauty" best represents the mimetic
D. "Ode on (iv) "O Goddess! hear these perspective?
Melancholy" tuneless numbers, (a) The line is an ironic quotation of "beauty" and
wrung By sweet "truth" as "all we know on earth" suggests that
enforcement and reality is an illusory concept and that the
remembrance dear," primary function of art is to construct a world
Code: within an aesthetic reality of its own.
(a) (A)-(iii), (B)-(iv), (C)-(ii), (D)-(i) (b) A work of literature is "beautiful" insofar as it
(b) (A)-(i), (B)-(iii), (C)-(ii), (D)-(iv) offers an accurate representation of its subject
(c) (A)-(iv), (B)-(iii), (C)-(i), (D)-(ii) matter, with fully realized characters and vivid
(d) (A)-(iv), (B)-(i), (C)-(iii), (D)-(ii) description of events.
Ans: (c) (c) The author's arbitrary imposition of order upon
A. "Ode to (iv) "O Goddess! hear the chaotic impressions of reality constitutes
Psyche" these tuneless the only "truth" in a work of art.
numbers, (d) Those aspects of reality which we perceive to
By sweet enforcement be "beautiful" are the only worthy subject
and remembrance matter of the artist, and it is the artist's job to
dear," observe closely and isolate those sublime
elements from the flux of the mundane.
B. "Ode on a (iii) "Thou still unravish'd Ans: (b) Keats's ‘Beauty is truth, truth beauty’ best
Grecian Urn" bride of quietness, interpretations. A work of literature is beautiful
Thou foster-child of insofar as it offers an accurate representation of its
silence and slow subject matter, with fully realized characters and vivid
time," description of events.
UGC NTA NET/JRF English Paper II, December 2018 276 YCT
46. What comes "After great pain" in the famous 49. In his essay "The Function of Criticism at the
Emily Dickinson poem? Present Time" (1864) Matthew Arnold
(a) A formal feeling contended that
(b) A concrete simplicity (a) Creative power should be ranked higher than
(c) The letting go critical power
(d) Substantial light (b) Critical power should be ranked higher than
creative power
Ans: (a) Emily Dickinson poem "After Great Pain" A
(c) Creative and critical powers are not
formal feeling comes.
comparable in any way
After great pain, a formal feeling comes – (d) Creative and critical powers should be ranked
The nerves sit ceremonious like Tombs – equally
The Stiff Heart questions was it He, that bore;
Ans: (a) "The Function of Criticism at the Present
And yesterday or centuries Before? Time (1864) contended that creative powers should be
47. What is peculiar about the reference in the ranked higher than critical power." It is written by
following to some poets names in the plural? Mathew Arnold.
"It is a freezing, bleak day in January, and I am Mathew Arnold (1822-1888) was an English poet and
looking for poetry. I see a few Chaucers, a few cultural critic who worked as an inspector of schools.
Shakespeares, and a hardcover, three-dollar Important Works–
History of Modern Poetry published in 1987." (1) Dover Beach (1867)
(a) Synecdochic use; names for their respective (2) The Scholar Gypsy (1853)
works. (3) Thyrsis (1866)
(b) Usually refer to biographies of the poets in (4) Cultural And Anarchy (1869)
question. (5) Literature and Dogma (1873).
(c) Unusual; awkward metaphors no longer in use. 50. Which of the following is not indebted in the
(d) Standard reference to more texts of one Gothic genre?
poet/author. (a) Tobias Smollett's Roderick Random
Ans: (a) Peculiar reference– (b) William Beckford's Vathek
(c) Ann Radcliffe's The Italian
Synecdochic use; names for their respective works.
(d) Matthew Lewis's The Monk
48. Herr God, Herr Lucifer Ans: (a) Tobias Smollett's 'Roderick Random' is a
Beware Picaresque novel published in 1748 and partially
Beware. based on his experience as a novel Surgeon's mate in
Out of the ash the Royal Navy especially during the battle of
I rise with my red hair Cartagena de Indies in 1741.
And I eat men like air. 51. Who among the ancients prescribed that poetry
Lines 4 and 5 in the above evoke: should both instruct and delight?
(a) The fairy-tale of a girl in the words (a) Plotinus (b) Longinus
(b) Christ's resurrection (c) Aristotle (d) Horace
(c) The myth of the phoenix Ans: (d) Horace prescribed that poetry should both
(d) The legend of the Lady of the Lake instruct and delight.
Ans: (c) 'Lady Lazarus' by Sylvia Plath lines 4 and 5 Horace (65 BC – 8 BC) known in the English the
talks about The myth of the phoenix. In Greek leading Roman lyric poet during the time of Augustus.
mythology, a phoenix is a long lived bird that Important Works–
cyclically regenerates or is otherwise born again. (1) Odes
Associated with the sun, a phoenix obtains new life by (2) The Art of Poetry.
arising from the ashes of its predecessor. 52. "What is honour? A word. What is that word
Sylvia Plath was an American poet, novelist and short honour? Air. A trim reckoning? Who hath it?
story writer. He that died o' Wednesday. Doth he feel it? No.
Notable Works– Doth he hear it? No. Is it insensible, then? Yes,
(1) The Bell Jar to the dead. But will it not live with the living?
(2) Plath : Poems, Ariel No. Why? Detraction will not suffer it.
(3) Lady Lazarus …..therefore. I'll none of it: Honour is a mere
(4) The Collected Poems scutcheon; and so ends my catechism."
Which character in the following Shakespeare's
Literary Movement–
dramas made this statement about honour?
(1) Confessional Poetry (Early 1960)
(a) Hotspur in King Henry IV-Part I
Awards– (b) Claudius in Hamlet, the Prince of Denmark
(1) Pulitzer Prize for Poetry (1982) (c) Hamlet, in Hamlet, the Prince of Denmark
(2) Glascock Prize (1955). (d) Falstaff in King Henry IV-Part I
UGC NTA NET/JRF English Paper II, December 2018 277 YCT
Ans: (d) Shakespeare's drama King Henry IV Part I 20th century literature. He helped to found the Abbey
statement about honour. Theatre (1966). He also known as last Romantic poet.
Shakespeare was a famous dramatist, play writer, poet Important Works–
and popular for kings man. He is a great writer of (1) When You are Old
Elizabethan Period. (2) The Cetic Twilight
Notable Works– (3) Aedh Wishes for the Clothes.
(1) Macbeth (1606) 55. In an ode, William Collins lamented the passing
(2) Hamlet (1599-1601) of a contemporary poet. The ode began with the
(3) King Lear (1605-06) line : "In yonder grave a Druid lies." Name the
(4) Othello (1603-04) poet whose passing Collins laments.
53. Match the following concepts with their (a) James Thomson
definitions: (b) Thomas Gray
(Concepts) (Definitions) (c) William Cowper
A. Collocation (i) A semantic
(d) Alexander Pope
relationship of one-
to-many Ans: (a) William Collins lamented the passing of a
B. Corpus (ii) A grid used in contemporary poet, "James Thomson". The Ode
lexical analysis began with lines "In yonder grave a Druid lies."
C. Hyponymy (iii) A combination of 56. Jonathan Swift arrived in London in 1710 and
two lexical items in confronted a rapidly changing world in the new
a grammatical Tory ministry. His reactions to this world are
pattern vividly recorded in his Journal to Stella, a series
D. Matrix (iv) A large body of of letters addressed to
texts (i) Hester Vanhomrigh
Code: (ii) Esther Johnson
(a) (A)-(i), (B)-(iii), (C)-(iv), (D)-(ii) (iii) Rebecca Dingley
(b) (A)-(iii), (B)-(i), (C)-(ii), (D)-(iv)
(iv) Lady Mary Montagu
(c) (A)-(iv), (B)-(ii), (C)-(iii), (D)-(i)
The right combination according to the code is:
(d) (A)-(iii), (B)-(iv), (C)-(i), (D)-(ii)
Code:
Ans: (d)
(a) (i) and (ii) (b) (iii) and (iv)
A. Collocation (iii) A combination of
two lexical items (c) (ii) and (iii) (d) (ii) and (iv)
in a grammatical Ans: (c) Journal to Stella a series of letters addressed
pattern to Esther Johnson and Rebecca Dingley. It consists of
B. Corpus (iv) A large body of 65 letters to his friend. Esther Johnson, whom he
texts called Stella and when he may have secretly married.
C. Hyponymy (i) A semantic Letters were written between 1710 and 1713, from
relationship of various locations in England and though clearly
one-to-many intended for Stella's eyes were sometimes addressed
D. Matrix (ii) A grid used in to her companion Rebecca Dingley.
lexical analysis 57. Match the Term with the Theorist :
54. The following lines are W.B. Yeats's metaphor (Term) (Theorist)
for an old man: (A) Negritude (i) Alice Walker
A tattered coat upon a stick, unless (B) Womanism (ii) Jurgen Habermas
Soul clap its hands and sing, and
(C) Interpellation (iii) Aime Cesaire
louder sing
(D) Public Sphere (iv) Louis Althusser
For every tatter in its mortal dress.
Here, the aged man is ______, and his "soul Code :
…… in its mortal dress," is ______. (a) (A)-(ii), (B)-(i), (C)-(iv), (D)-(iii)
(a) point, counterpoint (b) (A)-(iii), (B)-(ii), (C)-(iv), (D)-(i)
(b) vehicle, tenor (c) (A)-(i), (B)-(ii), (C)-(iv), (D)-(iii)
(c) tenor, vehicle (d) (A)-(iii), (B)-(i), (C)-(iv), (D)-(ii)
(d) analogy, analogue Ans: (d)
Ans: (c) The Aged man is tenor and his soul in its A. Negritude (iii) Aime Cesaire
mortal dress is vehicle. B. Womanism (i) Alice Walker
William Butler Yeats (1865-1939) was an Irish poet, C. Interpellation (iv) Louis Althusser
was an Irish poet and one of the foremost figures of D. Public Sphere (ii) Jurgen Habermas

UGC NTA NET/JRF English Paper II, December 2018 278 YCT
58. In this novel by Graham Greene a double agent 61. Match the Characters with the Play:
uses classic works of fiction to encode secret (Character) (Play)
information. "He put Clarissa Harlowe back in A. Dorimant (i) The Plain Dealer
the bookcase" is the first clue to his treachery. B. Lady Fidget (ii) The Man of Mode
Then he draws on War and Peace and The Way C. Malevole (iii) The Country Wife
We Live Now as matrices for secretly D. Vernish (iv) The Malcontent
transmitting information. Identify the novel. Codes :
(a) Our Man in Havona (a) (A)-(ii), (B)-(iv), (C)-(iii), (D)-(i)
(b) The Human Factor (b) (A)-(iv), (B)-(i), (C)-(iii), (D)-(ii)
(c) The Confidential Agent (c) (A)-(ii), (B)-(iii), (C)-(iv), (D)-(i)
(d) The Man Within (d) (A)-(iv), (B)-(iii), (C)-(i), (D)-(ii)
Ans: (b) The Human Factor is an espionage novel by Ans : (c)
Graham Greene, first published in 1978 and adopted (A) Dorimant – (ii) The Man of Mode
for film 1979. (B) Lady Fidget – (iii) The Country Wife
Henry Graham Greene (1904-1991) was an English (C) Malevole – (iv) The Malcontent
novelist and one of the greatest writers of the 20th (D) Vernish – (i) The Plain Dealer
century. 62. "Why don’t we have a little game? Let's pretend
Important Works– that we're human beings and that we are
(1) It's a Battlefield (1934) actually alve."
(2) Journey Without Maps (1936) This passage forms part of
(3) The Confidential Agent (1939) (a) John Osborne's Look Back in Anger
(4) A Gun for Sale (1936) (b) Harold Pinter's The Birthday Party
(5) The Human Factor (1978) (c) Samuel Beckett's Waiting for Godot
He was shortlisted in 1966 and 1967 for the Nobel (d) Agatha Christie's The Mousetrap
Prize for Literature. Ans: (a) The passage is a part of John Osborne's Look
59. The Romantic period produced a fair amount of Back in Anger. This play focuses on the life and
dramatic criticism. A notable example is "On marital struggles of an intelligent and educated but
disaffected young man of working class origin Jimmy
the Knocking at the Gate in Macbeth."
Porter and his equally competent yet impressive upper
Who is the author?
middle class wife Alison.
(a) Thomas De Quincey
John James Osborne was an English playwright
(b) William Hazlitt
screen writer and actor known for his excoriating
(c) Edmund Kean
prose and intense critical stance towards established
(d) William Charles Macready
social and political forms.
Ans: (a) ‘On the Knocking At The Gate in Macbeth’ Important Books–
is written by Thomas De Quincey. He was best known (1) A better class of person
for his confessions of an English Opium-Eater. He (2) Almost a gentleman
also inaugurated the tradition of addiction literature in (3) An Autobiography
the west. (4) West of Suez
60. Who viewed Wordsworth, Southey and 63. Nicholas Nickleby firmly established Charles
Coleridge as representatives of a "sect of Dickens as a dominant novelist of his time and
poets…. dissenters from the established systems the book as an unrivalled literary phenomenon.
in poetry and criticism" who constituted "the To celebrate the completion of the book, a
most formidable conspiracy against sound painter noted that there had been nothing
judgement in matters political"? comparable to him since the days of Samuel
(a) Ralph Vaughan (b) Francis Jeffrey Richardson. Identify the painter.
(c) Francisco Franco (d) Henry Vaughan (a) Leonard Woolf
Ans: (b) Francis Jeffrey, the author who is now before (b) Ernest Dawson
us, belongs to a sect of poets that has established itself (c) David Wilkie
in this country within these ten or twelve years. and is (d) John Cruickshank
looked upon, we believe as one of its chief champions Ans: (c) The painter is David Wilkie.
and apostles. The peculiar doctrine of this sect, it 64. I have carried the manuscript of these
would not perhaps, be very easy to explain but, that translations about with me for days, reading it
they are dissenters from the established system in in railway trains, or on the top of omnibuses and
poetry and criticism. in restaurants, and I have often had to close it
Francis Jeffrey was a literary critic and Scottish lest some stranger would see how much it moved
Judge. me. These lyrics – which are in the original, my
UGC NTA NET/JRF English Paper II, December 2018 279 YCT
….. (Indian friends) tell me, full of subtlety of 67. Which Walter Scott novel is set in France in the
rhythm, of untranslatable delicacies of colour, fifteenth century?
of metrical invention – display in their thought a (a) Redgauntlet (b) Ivanhoe
world I have dreamed of all my life long. The (c) The Antiquarry (d) Quentin
work of a supreme culture, they yet appear as Ans: (d) Quentin Durward novel is set in France in
much a growth of the common soil as the grass the 18th century written by Walter Scott. It is a
and the rushes. A tradition, where poetry and historical novel and story concerns a Scottish archer in
religion are the same thing, has passed through the service of the French King Louis XI. The novel is
the centuries gathering from learned and published in 1823.
unlearned metaphor and emotion, and carried Sir Walter Scott (1771-1832) was a historical novelist,
back again to the multitude the thought of the poet, playwright and historian.
scholar and the noble. If the civilization of Notable works–
Bengal remains unbroken, if that common mind (1) Rob Roy
which – as one divines – runs through all, is not, (2) Old Mortality
as with us, broken into a dozen minds that know (3) The Lady of the Lake
nothing of each other, something even of what is (4) The Heart of Midlothian.
most subtle in these verses will have come, in a 68. David Malouf's novel Ransom is based on
few generations, to the beggar on the roads. (a) an episode in the Trojan War
- W. B. Yeats, from Introduction to (b) a war memoir by Edmund Blunden
Rabindranath Tagore's Gitanjeli (c) a war poem by Yevgeny Yevtushenko
In this passage, Yeats praises Indian culture (d) an episode in The Mahabharata
primarily because it Ans: (a) Davis Malouf's novel Ransom is based on an
(a) has been flexible enough to survive a transition episode in the Trojan War. In Greek Mythology the
into the modern world. Trojan war was waged against the city of Troy by the
(b) embodies values and gives rise to art that can Achaeans after Paris of Troy took Helen from her
be shared by people of all classes. husband Menelaus, King of Sparta. The war is one of
(c) reflects a marvelous eclecticism in drawing the most important events in Greek mythology and
from many disparate cultures. has been narrated through many works of Greek
literature, most notably Homer's Iliad.
(d) is accessible to Westerners though it is rooted
in a different religious tradition. 69. Match the author with the title:
(Author) (Title)
Ans: (b) W.B. Yeats praises Indian culture primarily A. Alan Paton (i) Open City
because it embodies values and gives rise to art that B. Ngugi wa Thiong'o (ii) Cry, the Beloved
can be shared by people of all classes. Country
65. As a boy growing up in Squire Allworthy's C. Teju Cole (iii) A Grain of Wheat
estate, Tom gets one of the following characters D. Wole Soyinka (iv) The Interpreters
into trouble. Identify the character. Code:
(a) Nightingale (b) Partridge (a) (A)-(iii), (B)-(ii), (C)-(iv), (D)-(i)
(c) Black George (d) Blifil (b) (A)-(iii), (B)-(i), (C)-(iv), (D)-(ii)
Ans: (c) As a boy growing up in Squire Allworthy's (c) (A)-(ii), (B)-(iii), (C)-(i), (D)-(iv)
estate, Tom gets one of the following characters into (d) (A)-(i), (B)-(iii), (C)-(iv), (D)-(ii)
trouble the character is Black George. Ans: (c)
66. In his Practical Criticism I.A. Richards suggests (A) Alan Paton – (ii) Cry, the Beloved Country
(B) Ngugi wa Thiong'o – (iii) A Grain of Wheat
that there are several kinds of meanings and
(C) Teju Cole – (i) Open City
that the "total meaning" is a blend of
(D) Wole Soyinka (iv) The Interpreters.
contributory meanings which are of different
70. Braj Kachru has observed a tendency among
types. He identified four kinds of meaning, or
Indian-English speakers and writers to use
the total meaning of a word depends upon four
hybridized lexical items. One example of this is
factors. Choose the right combination as (a) Ping-pong (b) Chaywallah
proposed by Richards. (c) Jugirh (d) Lathi-charge
(a) Sound, Sense, Tone and Matter
Ans: (d) Hybridized Items:
(b) Sense, Feeling, Tone and Intention By a hybridized lexical item is meant here a lyrical
(c) Sense, Feeling, Tone and Matter item which comprises two or more elements at least
(d) Image, Feeling, Tone and Intention one of which is from a South Asian language and one
Ans: (c) The right combination of word depends upon from English. The elements of hybrid formations may
four factors– belong either to 'an open set' on to a 'closed system' in
Sense, Feeling, Tone and Matter. leris (Kachru 1983-153).

UGC NTA NET/JRF English Paper II, December 2018 280 YCT
In this the first component – the modifier is from 74. Which of the following is the most accurate
English and the head is from a South Asian Language statement by W.E.B. Du Bois's famous
(Kachru 1983-156). Consider for example, the articulation of the 'twoness' of black
following Bad Hindustani, Broken Hindustani, British Americans?
Raj, Obscene Sarker, Big Paktar. (a) "This double consciousness, this sense of
Example – Lathi Charge. always looking at one's self through the eyes of
71. "Full many a lady others, is a peculiar sensation."
I have eye'd with best regard: and many a time (b) "This sense of always looking at one's self, a
The harmony of their tongues hath into bondage peculiar sensation through the eyes of others, is
Brought my too diligent ear, for several virtues double consciousness."
Have I liked several women; never any (c) "It is a peculiar sensation, this double-
With so full soul, but some defect in her consciousness, this sense of always looking at
Did quarrel with the noblest grace she ow'd, one's self through the eyes of others."
And put it to the foil, But you, O you, (d) "Through the eyes of others, this sense of
So perfect and so peerless, are created always looking at one's self, we acquire the
Of every creature's best." double-consciousness."
This passage admiring the perfect matching of Ans: (c) The most accurate statement by W.E.B. Du
inner and outward beauty of a woman is taken
Bois's "It is a peculiar sensation, this double-
from
(a) John Webster's The Duchess of Malfi consciousness, this sense of always looking at one's
(b) Shakespeare's Tempest self through the eyes of others."
(c) Marlowe's Dr. Faustus 75. The term 'Digger' is associated with a group of
(d) Thomas Middleton's Women Beware Women agrarian communists who flourished in England
Ans: (b) Shakespeare's 'Tempest' is one of the last in 1649-50 and were led by
plays that Shakespeare wrote alone probably written (a) Laurence Clarkson
in 1610-1611. By Tempest poet admiring the perfect (b) Gerrard Winstanley
matching of inner and outward beauty of a woman. (c) John Liburne
It's another name is 'The Enchanted Island'. (d) George Fox
72. In traditional ELT methods and materials the Ans: (b) The term 'Digger' any of a group of agrarian
native speaker is elevated and idealized against communist who flourished in England in (1649-50)
stereotyped non-native speakers. This tendency and were led by Gerrard Winstanley, In April 1649
is dubbed _______ by Adrian Holiday. about so poor men assembled at St. George's Hill
(a) The near-native fallacy Surrey and began to cultivate the common land. This
(b) Native speakerism Diggers held that the English Civil War had been
(c) The non-native fallacy fought against the king and the great landowners.
(d) The native-speaker bias Now that Charles I had been executed land should be
Ans: (b) Native Speakerism is a pervasive idealogy made available for the very poor to cultivate.
within ELT, characterized by the belief that native 76. What attitude towards death would you find in
speaker teachers represent a 'Western Culture' from such poems as Tennyson's "Crossing the Bar."
which spring the ideals both of the English language Whitman's "Death Carol," and Kipling's
and of English language teaching methodology. "L'Envoi"?
73. There are helpers and harmers among fellow- (a) Despair (b) Hope
pilgrims in Christian's journey in Pilgrim's (c) Resignation (d) Protest
Progress. Who among the following is not a
Ans: (b) Tennyson's "Crossing the Bar", Whitman's
helper?
(a) Mr. Worldly Wiseman "Death Carol" and Kipling's "L'Envoi" poetry
(b) The Interpreter contained Hope. Because all three poetries are
(c) Good Will optimistic contained items as well as thoughts.
(d) The Evangelist 77. In which work does William Blake say that
Ans: (a) Mr. Worldly Wiseman is also a character of Milton was "a true poet and of devil's party
Pilgrim's Progress played by Maurice O' Callaghan, without knowing it"?
Peter Thomas. The Pilgrim's Progress from This (a) "The Marriage of Heaven and Hell"
world to That which is to come is a 1678 Christian (b) "Songs of Innocence"
allegory written by John Bunyan. Mr. Worldly (c) "London"
Wiseman a resident of a place called Carnal Policy, (d) "The Chimney Sweeper"
who persuades Christian to go out of his way to Ans: (a) The marriage of Heaven and Hell is a book
focuses salvation on the law and good deeds instead by William Blake. It is a series of texts written in
of faith and love in Jesus Christ. His real advice is imitation of biblical prophecy but expressing Blake's
from the world and not from God. own intensely personal Romantic and revolutionary
UGC NTA NET/JRF English Paper II, December 2018 281 YCT
beliefs. It opens with an introduction of a short poem (a) Annals and Antiquities, Between Sunlight and
"Rintrah roars and shakes his fires in the burden'd air." Shadows
William Blake claims that John Milton as a true poet (b) The Moor's Last Sigh, Lajja
and his epic poem Paradise Lost. War of the Devil's (c) Chronicles of a Riot Foretold, Shame
party without knowing it. Milton's "Satan" was truly (d) Out of Place, The Algebra of Infinite Justice
his Messiah, he also claims that. Ans: (b) The Moor's last sigh is the fifth novel by
78. The Norman Conquest was a significant Salman Rushdie, published in 1995. The book draws
landmark in English history. What French did on a variety of real historical figures and events,
the Normans speak and what was it known as? including the surrender of Granada by Boabdil, the
(a) They spoke Norman French (Anglo-Norman). demolition of Babri Mashid, the 1993 Bombay
Theirs was certainly not the standard French. Bombings the gangster and terrorist.
(b) They spoke standard French (of mainland Lajja is a novel in Bengali by Taslima Nasrin, a writer
France). Their French was very sweet and of Bangladesh. Nasrin dedicated the book to the
musical. people of the Indian subcontinent beginning the text
(c) They spoke a dialectal French (also called with the words, "let another name for religion be
Anglo-Frisian), somewhat closer to the humanism." Lajja is a response of Taslima Nasrin to
Parisian. anti Hindu riots that erupted in parts of Bangladesh
(d) They spoke normal French, rather distinct from soon after the demolition of Babri Masjid in India on
Anglo-Norman, another standard language. 6 December 1992.
Ans: (a) They spoke Norman French (Anglo- 82. Albert Camus borrows the following epigraph in
Norman). Theirs was certainly not the standard his novel The Plague from _____
French. Hence option (a) is correct. "It is as reasonable to represent one kind of
imprisonment by another, as it is to represent
79. Which of the following statements on
anything that really exists by that which exists
Rajmohan's Wife is not true?
not."
(a) His vivid description of the routine of Bengali
(a) Jeremy Bentham's The Principles of Morals
households reveal a lot about the nineteenth
and Legislation
century.
(b) Bankim Chandra published it soon after
] (b) Robert Burton's The Anatomy of Melancholy
serialization and was elated in seeing its first (c) Daniel Defoe's Robinson Crusoe
copy. (d) James Hogg's The Confessions of a Justified
Sinner
(c) By common consent, Rajmohan's Wife is the
first novel in English published by an Indian. Ans: (c) Albert Camus (1913-1960), borrows the
(d) The novel was serialized in 1864 in a short- epigraph to his novel 'The Plague' from Daniel
lived magazine in Calcutta. Defoe's 'Robinson Crusoe'. Mainly Albert Camus was
a French philosopher, author and journalist. His views
Ans: (b) Bankim Chandra published it soon after
contributed to the rise of the philosophy known as
serialization and was elated in seeing its first copy. Absurdism.
80. Deconstructionist critics argue that texts are Notable works-
never free from (1) The Stranger
(a) the material conditions that determine the (2) The Myth of Sisyphus
production and reception. (3) The Rebel
(b) the equivocal and ironically unstable (4) The Plague.
worldview of the author. Awards–
(c) distortions inherent in the rhetoricity of Nobel Prize (1957) at the age of 44 in 1957, the
language. second youngest recipient in history.
(d) the interpretations bestowed by the totalizing 83. Which ancient Greek writer's name is directly
critic. mentioned in Lord Byron's poem "The Isles of
Ans: (c) Deconstructionist critics believe meaning in Greece"?
literature is created during the act of reading a test. "If (a) Aeschylus (b) Sappho
language is the ground of being then the world is (c) Euripides (d) Sophocles
infinite test, that is an infinite chian of signifiers Ans: (b) Lord Byron's poem "The Isles of Greece" is
always in play. Because human beings are constituted a personal lyric and elegy. Greek writer's burning
by language, they too, are texts." Texts are never free Sappho name is directly mentioned.
from distortions inherent in the rhetrocity of language. The ISLES of Greece, the isles of Greece!
81. 1992 demolition of the disputed structure in Where burning Sappho lowed and,
Ayodhya produced two controversial literary Where grew the arts of war and peace-
responses Identify them. Where Delos rose and Phoebus sprang!
UGC NTA NET/JRF English Paper II, December 2018 282 YCT
84. What is the delicate balancing act of Andrew Which sentence best paraphrases line 5 of the
Marvell's "Horatian Ode"? passage above?
(a) Celebrating Cromwell's victories while inviting (a) Emotions can damage us, but none as severely
sympathy for the executed King. as love.
(b) Celebrating the Restoration while regretting the (b) Love trends to grab us and never let go.
frivolity of the new regime. (c) Distress comes in many forms, but none lasts
(c) Praising Roman virtues while endorsing as long as heartache.
Christian beliefs. (d) Unbidden pain afflicts us, but we are lured by
(d) Praising feminine virtues while mocking the love.
fixation on chastity. Ans: (d) The stanza is composed by John Donne
Ans: (a) Like 'To His Coy Mistress', "An Horatian poetry 'The Broken Heart'.
Ode" operates on several levels. On the surface, it is Line 5 means, unbidden pain afflicts us, but we are
conventional celebratory ode about a military and lured by love.
political hero, praising his exploits and virtues. One The speaker declares that any man who claims he has
can infer from Marvell's other laudatory poems about been in love for an hour is insane , not because love
Oliver Cromwell that the poet genuinely admired the "decays" in so short a time, but because in an hour,
lord protector : the tone of the poem is not openly love can "devour" ten men.
ironic. John Donne was an English poet and cleric in the
Andrew Marvell was an English metaphysical poet, Church of England. He is considered the preeminent
satirist and politician who sat in the house of representative of the metaphysical poets.
commons at various times between 1659 and 1678. (1) Biathanatos (1610)
During the Common wealth period he was a colleague (2) Pseudo Martyr (1610)
and friend of John Milton. (3) Ignatius His Conclave (1611)
Notable works– (4) Devotions upon Emergent Occasions (1624).
(1) The Rehearsal Transpros'd 87. _______ read Adam Bede with such pleasure
(2) Mr. Smirke : On The Divine in Mode. that she not only keenly recommended it to her
Pamphlet– relatives but also commissioned two paintings of
(1) An account of the Growth of propery. scenes from the novel.
(2) Arbitrary Government in England. (a) Queen Victoria (b) George Eliot
85. Which of the following themes was not common (c) Horace Nightingale (d) Margaret Cavendish
to the works of Cavalier poets such as Thomas Ans: (a) Adam Bede, the first novel written by
Carew, Sir John Demham, Edmund Waller, Sir George Eliot was published in 1859. It was published
John Suckling, James Shirley, Richard pseudonymously (the pen name of Mary Ann Evans)
Lovelace, and Robert Herrick? was a well published and highly respected scholar her
(a) Courtly ideals of the good life time.
(b) Carp diem The plot is founded on a story told to George Eliot by
(c) Pious devotion to religious virtues her aunt Elizabeth Evans, a Methodist preacher and
(d) Loyalty to the king the original of Dinoh Merris of the novel of a
Ans: (c) Option (c) Pious devotion to religious virtues confession of child murder, made to her by a girl in
The Cavalier poets, members of the aristocracy, wrote prison. Victorian Era period consider from 1830 to
in the 17th century and supported King Charles I who 1901. From Reign of Queen Victoria to the death of
was later executed as a result of civil war. Cavalier Victoria. Queen Victoria introduced 'Adam Bede'
poetry on the subject matter different from traditional George Eliot's novel to prince Albert immediately
poetry. Instead of talking issues like religion, after re-reading it, she marked several philosophical
philosophy and the arts, Cavalier poetry aims to passage in the margins. In May 1860 the Queen
express the joy and simple gratification of celebratory Victoria commissioned Edward Conbould to paint
things much livelier than the traditional works of their water colours of two scenes from the book.
predecessors. Most Cavalier works had allegorical 88. The "grammar bullies" you read them in places
and/or classical references. like the New York Times – and they tell you what
86. Ah, what a trifle is a heart, is correct.
If once into love's hands it come! Your must never use "hopefully, we will be going
All other griefs allow a part there on Thursday." That is incorrect and wrong
To other griefs, and ask themselves but some; and you are basically an ignorant pig if you say it.
They come to us, but us love draws; This is judgementalism. The game that is being
He swallows us and never chaws; played there is a game of social class. It has
By him as by chain'd shot, whole ranks do die; nothing do with the morality of writing and
He is the tyrant pike, our hearts the fry. speaking and thinking clearly, of which George
–John Donne, 1633 Orwell, for instance, talked so well.
UGC NTA NET/JRF English Paper II, December 2018 283 YCT
To which famous essay of Orwell does the 90. The following epitaph was written by Rudyard
author refer here? Kipling during the War of 1914-18.
(a) "Inside the Whale" HINDU SEPOY IN FRANCE
(b) "Why I Write" This man is his own country prayed we know
(c) "Reflections on Gandhi"
not to what Powers.
(d) "Politics and the English Language"
We pray Them to reward him for his bravery in
Ans: (d) "Politics and the English Language" is an
essay by George Orwell that criticized the "ugly ours.
inaccurate". Style for Orwell was never simply a "Powers" here refers to _____. "Them" to
question of aesthetic; It was always inextricably ______, and "ours" to _____.
linked to politics and to truth' All issues are political (a) the Hindus, the French, the British
issues and political itself is a mass of lies, evasions, (b) the military, the Hindu sepoys, Powers
folly hatred and schizophrenia. When the general (c) the divine, Powers, our country
atmosphere is bad, language must suffer. 'Language is (d) authorities, his compatriots, our country
a political issue and slovenly use of language to hide
unpleasant political facts. Bad English he believed, Ans: (c) "Powers" here refers to – The divine Powers.
was a vehicle for oppressive ideology, and it is no "Them" and "Ours" refers to – Our Country.
accident that politics and the English Language' was Hindu Sepoy In France – The man in his own country
written after the close of World War II. we know not to what powers. We pray them to reward
Eric Arthur Blair Better known by his pen name him for his bravery in ours.
George Orwell, was an English novelist, essayist, Rudyard Kipling (1865-1936) – Rudyard Kipling a
journalist and critic, whose work is marked by lucid English poet, short story writer and novelist chiefly
prose, awareness of social injustice, opposition to
remembered for his celebration of British imperialism
totalitarianism and outspoken support of democratic
socialism. tales and poems of British soldiers in India and his
Notable works– tales for children. Kipling received the 1907 Nobel
(1) Shooting An Elephant. Prize for Literature. He was born in Bombay, in the
(2) Why I Write. Bombay Presidency of British India was taken by his
(3) A Hanging. family to England when he was five years old. Kipling
(4) Adam Bede. is best known for his works of fiction.
(5) Politics and the English Language. Notable works–
89. Match the plays to their setting: (1) The Jungle Book (Stories collection which
(A) Krapp's Last (i) A country road; A tree. includes 'Rikki-Tikki-Tovi')
Tape
(2) Just So Stories (1902)
(B) Happy Days (ii) Bare interior; Two small
windows high up; grey (3) Kim (1901) (A tale of Adventure)
light. (4) The Man Who Would Be King (1888)
(C) Waiting for (iii) Expanse of scorched (5) The White Man's Burden (1899).
Godot grass forming a low 91. Match the following authors with the novels:
mound; blinding light. (Name of Author) (Name of Novel)
(D) Endgame (iv) A late evening in future; (A) Chitra Banerjee (i) Inheritance
white light. Divakaruni
Code:
(B) Anita Rau Badami (ii) Listening Now
(a) (A)-(ii), (B)-(iv), (C)-(iii), (D)-(i)
(b) (A)-(iii), (B)-(iv), (C)-(i), (D)-(ii) (C) Anjana Appachana (iii) Sister of My Heart
(c) (A)-(ii), (B)-(iii), (C)-(i), (D)-(iv) (D) Indira Ganesan (iv) The Hero's Walk
(d) (A)-(iv), (B)-(iii), (C)-(i), (D)-(ii) Code:
Ans: (d) Correct Matching (a) (A)-(iii), (B)-(iv), (C)-(ii), (D)-(i)
(A) Krapp's Last (iv) A late evening in future; (b) (A)-(i), (B)-(iii), (C)-(ii), (D)-(iv)
Tape white light. (c) (A)-(iv), (B)-(i), (C)-(iii), (D)-(ii)
(B) Happy Days (iii) Expanse of scorched (d) (A)-(iv), (B)-(ii), (C)-(i), (D)-(iii)
grass forming a low Ans: (a)
mound; blinding light. (A) Chitra Banerjee Divakaruni – (iii) Sister of My
(C) Waiting for (i) A country road; A tree.
Heart
Godot
(B) Anita Rau Badami – (iv) The Hero's Walk
(D) Endgame (ii) Bare interior; Two
small windows high (C) Anjana Appachana – (ii) Listening Now
up; grey light. (D) Indira Ganesan – (i) Inheritance

UGC NTA NET/JRF English Paper II, December 2018 284 YCT
92. Allen Tate once made a useful distinction confusing line because the word lying could mean two
between structure and texture. The distinction different things. Lying as in untruthful or as in lazy? I
referred to think the last line says "Now I may wither into the
(a) the main line of a narrative, argument, etc, and truth" the word 'lying' is supposed to be untruthful. So
the rhetorical, stylistic, metaphorical and other we see poet has a deep regret in his heart.
devices respectively. 95. "Search the heads of the greatest rivers in the
(b) the rhetorical, stylistic, metaphorical and other world, you shall find them but bubbles of
devices, and the main line of a narrative, water."
argument, etc, respectively. Who is the author of this line?
(c) the devices employed to enlighten objects and (a) Oscar Wilde (b) Francis Bacon
materials in a narrative, and the objects and (c) John Webster (d) R.B. Sheridan
material themselves, respectively. Ans: (c) John Webster's 'The Duchess of Malfi' has
(d) objects and materials on which a narrative been taken extracted lines.
casts light, and the devices employed to John Webster was an English Jacobean dramatist best
enlighten them respectively. known for his tragedies. 'The White Devil' and 'The
Ans: (a) Allen Tate, An American poet and critic. Duchess of Malfi' which are often regarded as
Tate was a member of the Agrarian movement at masterpieces of the early 17th century English stage.
Vanderbilt and later a New critic. Tate, primarily a His life and career overlapped William Shakespeare.
poet and critic both of literature and in the realm of
ideas where literature morals and politics exist side by 96. The title of Dylan Thomas's Deaths and
side, was one of the theorists of the Fugitive Groups, Entrances was taken from
and The Fathers could be taken as a dramatization in (a) Rudyard Kipling's "A Death-Bed"
fiction of ideas of society and tradition. According to (b) T.S. Eliot's Murder in the Cathedral
Allen Tate, structure is the main line of a narrative (c) William Shakespeare's Macbeth
argument etc. while Texture is the rhetorical, stylistic, (d) John Donne's "Death's Duell"
metaphorical and other deuces respectively. Ans: (d) Dylan Thomas's 'Deaths and Entrances' title
93. Match the character with the work: was taken from John Donne's "Death's Duell". Death's
(Characters) (Name of work) Duell is the final sermon delivered by John Donne as
(A) Rupert Birkin (i) Sons and Lovers the dean of St. Poul's Cathedral Donne received notice
(B) Lydia Lensky (ii) Kangaroo to preach the Sermon on the first Friday of Lent (12
(C) Miriam Leivers (iii) Women in Love Feb 1631) but preached the sermon on (25 Feb 1631).
(D) Richard Somers (iv) The Rainbow The Sermon was likely written out in full prior to
Code: Donne preaching it as it was subsequently prepared
(a) (A)-(iv), (B)-(i), (C)-(ii), (D)-(iii) for publication. The act of preaching exhausted Donne
(b) (A)-(ii), (B)-(iii), (C)-(iv), (D)-(i) to those he had preached to. It seemed as though he
(c) (A)-(iii), (B)-(iv), (C)-(i), (D)-(ii) had delivered his own death Sermon.
(d) (A)-(i), (B)-(ii), (C)-(iv), (D)-(iii) 97. Mango Souffle, India's first major gay themed
Ans: (c) Correct Matching– film, is an adaptation of Mahesh Dattani's play
(A) Rupert Birkin – (iii) Women in Love (a) On a Muggy Night in Mumbai
(B) Lydia Lensky – (iv) The Rainbow (b) Do the Needful
(C) Miriam Leivers – (i) Sons and Lovers
(c) Bravely Fought the Queen
(D) Richard Somers – (ii) Kangaroo.
(d) Dance like a Man
94. What tone will be best suited to the following
Ans: (a) Mango Souffle, first major gay themed film
poem?
is an adaptation of Mahesh Dattani's play 'On a
THE COMING OF WISDOM WITH AGE
Though leaves are many, the root is one; Muggy Night in Mumbai'.
Through all the lying days of my youth A playwright of world stature – Mario Relich Wasafiri
I swayed my leaves and flowers in the sun; On a Muggy Night in Mumbai is the first
Now I may wither into the truth. contemporary Indian play to openly tackle gay themes
(a) Excitement (b) Exultation love, partnership, trust and betrayal.
(c) Revulsion (d) Regret Mahesh Dattani is an Indian director, actor,
Ans: (d) "The Coming of Wisdom With Age" is playwright and writer.
written by William Butler Yeats, on the last stage of Notable works–
his life. The four lines roughly correlate with the four (1) Final Solution
quarters of life. And I think each quarter lived, he gets (2) Dance like a Man
a little bit wiser, the wisdom he paid for with time. (3) Bravely Fought the Queen
"Through all the lying days of my youth" is a (4) Thirty days in September.
UGC NTA NET/JRF English Paper II, December 2018 285 YCT
Directions: (Q. No. 98-100) Read the passage given MRS. LINDE : [Smiling wearily and smoothing her
below and answer the questions. hair.] Oh, sometime it happens Nora.
The following is an extract from a famous play. Read it NORA : So completely alone. How terrible hard that
carefully to answer questions that follow. must be for you. I have three lovely children. You can't
MAID : [From the hall doorway] Ma'am a lady to see see them now; they're out with the maid.
you – […..]
NORA : All right, let her come in. 98. Identify the play of which this section is an
[…The MAID shows in MRS. LINDE, dressed in excerpt.
travelling clothes, and shuts the door after her.] (a) The Cherry Orchard by Anton Chekhow
MRS. LINDE : [In a dispirited and somewhat hesitant (b) Wit by Margaret Edson
voice.] Hello, Nora. (c) The Importance of Being Earnest by Oscar
NORA : [Uncertain.] Hello – Wilde
MRS. LINDE : You don't recognize me. (d) A Doll's House by Henrik Ibsen
NORA : No, I don’t know – but wait, I think – Ans: (d) 'A Doll's House' is a play by Henrk Ibsen
[Exclaiming.] What! What! Is it really you? that was first performed in 1879. It is genre is realistic
MRS. LINDE : Yes, It's me. and basically modern prose drama. The original
NORA : Kristine ! To think I didn’t recognize you. But language is Norwegian. It's theme The sacrificial role
them, how could i? [More quietly.] of women : parental and filial obligation; the
How you've changed, Kristine! unreliability of appearances. The symbol of drama got
MRS. LINDE : Yes, no doubt I have. In nine – ten long enormous popularity as 'The Christmas Tree', 'New
years. Year's Day'.
NORA : It is so long since we met! Yes, It's all of that. Henrik Ibsen (1828-1906)
Oh, these last eight years have been a happy time, Henrik Ibsen considered by many to be the father of
believe me. And so now you've come in to town, too. modern drama, "father of realism."
Made the long trip in the winter. That took courage. Notable Works–
MRS. LINDE : I just got here by ship this morning. (i) A five act tragedy 'Catiline'.
NORA : To enjoy yourself over Christians, of course. (ii) Brand
Oh, how lovely! Yes enjoy ourselves, we'll do that. But (iii) Peer Gynt
take your coat off. You're not still cold? [Helping her.] (iv) An Enemy of the People
There now, let's get cozy here by the stove. No, the easy (v) Emperor and Galilean
chair there ! I'll take the rocker here. [Seizing her hands.] (vi) A Doll's House.
Yes, now you have your old look again; it was only in 99. "Not even a sense of loss to feed on" implies that
that first moment. You're a bit more pale. Kristine – and (a) Mrs. Linde is completely devoid of all feeling.
maybe a bit thinner. (b) Mrs. Linde is sentimentally attached to an
MRS. LINDE : And much, much older, Nora. irretrievable past.
NORA : Yes, perhaps a bit older, a tiny, tiny bit; not (c) Mrs. Linde's severance from her tragic pair is
much at all. [Stopping short, suddenly serious.] Oh, but total.
thoughtless me, to sit here, chattering away. Sweet, good (d) Mrs. Linde is given over to feeding on sorrow.
Kristine, can you forgive me? Ans: (c) This play is also 'A Doll's House' extract.
MRS. LINDE : What do you mean, Nora? Mrs. Linde's severance from her tragic pair is total.
NORA : [Softly] Poor Kristine, you've become a widow.
100. Which of the following descriptions best applies
MRS. LINDE : Yes, three years ago.
to the above extract?
NORA : Oh, I knew it, of course; I read it in the papers.
(a) A chance meeting between old friends which
Oh, Kristine, you must believe me; I often thought of
leaves one puzzling over the inexplicable
writing you then, but kept postponing it, and something
losses the other suffered.
always interfered.
(b) A meeting of two friends – one married, the
MRS. LINDE : Nora, dear, I understand completely.
other unmarried after a gap of years.
NORA : No, it was awful of me, Kristine, You poor
(c) The sense of loss inevitable with the passage
thing, how much you have gone through. And he left you
of time and the imperceptible dissolution of the
nothing?
conventional marriage.
MRS. LINDE : No.
(d) Friends comparing notes and counting losses
NORA : And no children?
in a meeting sudden and unanticipated.
MRS. LINDE : No.
NORA : Nothing at all, them? Ans: (a) This play is also 'A Doll's House' extract.
MRS. LINDE : Not even a sense of loss to feed on. The best applies, A chance meeting between old
NORA : [Looking incredulously at her.] But Kristine, friends which leaves one puzzling over the
how could that be? inexplicable losses the other suffered.
UGC NTA NET/JRF English Paper II, December 2018 286 YCT
UGC NTA NET/JRF Exam, June-2019
ENGLISH
Solved Paper-II
1. By which two of the following processes, according C. Shakespearean iii. Stanley Fish
to Michel Foucault, does power operate? Negotiations
(i) By right rather than technique D. Is There a Text in iv. Sandra M. Gilbert This
(ii) By normalization rather than law Class? and Susan Gubar
(iii) By control rather than punishment Choose the correct option from those given below :
(iv) By repression rather than agreement A B C D A B C D
Choose the correct option: (a) iii iv i ii (b) i ii iii iv
(a) (i) and (iii) (b) (ii) and (iii) (c) iv i ii iii (d) ii iii iv i
(c) (ii) and (iv) (d) (i) and (iv) Ans : (c) Correctly matched the books with the writer in
Ans : (b) In his essay ‘Discipline and Punish’ (1975), Michel option (c).
Foucault operates by normalization rather than law and
A The Madwoman in the iv Sandra M. Gilbert
control rather than punishment. He laid out his thoughts on how
Attic and Susan Gubar
the elite in the society dominate and control the rest of society.
B The Wretched of the i Frantz Fanon
2. Which of the following books is written by an
Earth
Englishman in universal Latin, is further added to
C Shakespearean ii Stephen
by the Flemish Peter Giles, is revised by the Dutch
Negotiations Greenblatt
Erasmus, is printed at Louvain in 1516, later at
Paris, still later at Basle, where it was illustrated by D Is There a Text in This iii Stanley Fish
two woodcuts from the hand of the German Holbein? Class?
(a) The Golden Legend (b) Confessio Amantis 6. What, in sum, is Sidney's point in the following?
(c) Utopia (d) Erwhon "Nature never set forth the earth in so rich tapestry
Ans : (c) ‘Utopia’ is a work of fiction and socio political as divers poets have done; neither with pleasant
satire by Thomas More. This book was written in Latin and is rivers, fruitful trees, sweet-smelling flowers, nor
further added to by the Flemish Peter Giles, is revised by the what so ever else may make the too-much-loved
Dutch Erasmus, is printed at Louvain in 1516. The first earth more lovely. Her world is brazen, the poets
edition contain a woodcut map of the Island of Utopia. only deliver a golden." (Philip Sidney)
3. Which edition of the Lyrical Ballads was the first (a) Works of art are superior to the natural world they
one to have the Preface by Wordsworth? represent
(a) 1798 (b) 1800 (b) Works of art can often compete with the natural
(c) 1802 (d) 1804 world represented by them
Ans : (b) ‘The Lyrical Ballads’ was a collaborative work of (c) Neither the poets nor the natural world they set
S.T. Coleridge and William Wordsworth. The first edition of forth equal nature's rich tapestry
this volume was published in 1798 with four poems of S.T. (d) The natural world is far superior to the works of
Coleridge and 19 by William Wordsworth. The second art that represent it
volume was published in 1800 in which Wordsworth included Ans : (a) In ‘Apologie for Poetrie’ (1580-81) written by Sir
‘Preface’ or ‘Preface to The Lyrical Ballads’. Philip Sidney, the text – “Nature never set forth the earth in
4. "The great English novelists are Jane Austen, so rich tapestry as divers poets have done; neither with
George Eliot, Henry James and Joseph Conrad." pleasant rivers, fruitful trees, sweet-smelling flowers, nor
Which one of the following critical texts begins with what so ever else may make the too-much-loved earth more
the above assertion? lovely. Her world is brazen, the poets only deliver a golden.”
(a) Walter Allen, The English Novel As such, the poets ‘making’ or production is superior to
(b) Terry Eagleton, The English Novel nature. So sum of this text is option (a) Works of art are
(c) F.R. Leavis, The Great Tradition superior to the natural world they represent.
(d) Ian Watt, Rise of the Novel 7. Which of the following novels by Iris Murdoch tells
Ans : (c) Critical text – “The great English novelist are Jane the story of an ageing theatre celerity who withdraws
Austen, George Eliot, Henry James and Joseph Conrad” into a life of seclusion and writes a diary/journal/ novel?
begins in – Frank Raymond Leavis’s – “The Great Tradition”. (a) The Sandcastle (b) Under the Net
This book is a literary criticism published in – 1948. (c) The Sea, the Sea (d) Flight from the Enchanter
5. Match the books with the writers: Ans : (c) ‘The Sea, the Sea’ is a novel by Iris Murdoch,
A. The Madwoman i. Frantz Fanon published in 1978. This novel is a tale of the strange
in the Attic obsessions that haunt a self-satisfied playwright and director
B. The Wretched of ii. Stephen Greenblatt as he begins to write his memoirs.
the Earth
UGC NTA NET/JRF English Paper II, June 2019 287 YCT
8. Who wrote a guide called How to Write a Doctoral 12. What is being described by Wordsworth in the
Thesis: The Humanistic Subjects, considered equal following lines from his poem, The Thorn
in standard to the American MLS Handbook or the I've measured if from side
Chicago, Manual of Style? "Tis three feet long and two feet wide.
(a) Alain Robbe-Grillet (b) Cesare Pavese (a) Fallen bought (b) A cradle
(c) Umberto Eco (d) Leo Spitzer (c) A small cot (d) An infant's grave
Ans : (c) Umberto Eco’s wise and witty guide to researching Ans : (d) Poem – ‘The Thorn’ was published in 1815 in
and writing a thesis, published in English for first time. He Poems Volume II. This poem was composed by William
published this book in 1977 entitled – How to Write a Wordsworth. It is an infant's grave ‘Tis three feet long and
Doctoral Thesis: The Humanistic Subjects. two feet wide’.
9. Which of the following propositions refers to the 13. For which one of the following reasons, in Oscar
recommendations of Charles Grant? Wilde's novel, The picture of Dorian gray, Gray
(a) The introduction of English as the medium of breaks down when he sees his finished portrait?
instruction in an Indian system of education that (a) Overwhelmed by the beauty of the portrait
included literature, art and craft (b) Overjoyed by the feeling that his beauty will be
(b) The introduction of English as the medium of known to all
instruction from lower levels in a few states as an (c) Distraught by the fact that his beauty will fade
experiment while the portrait stays beautiful
(c) The introduction of English as the medium of (d) Distraught by the badly drawn portrait
instruction in a Western system of education that Ans : (c) ‘The Picture of Dorian Gray’ is a gothic and
included literature, natural sciences and philosophical novel by Oscar Wild first published complete in
mechanical inventions. July 1890. The novel begins on a beautiful summer day in
(d) The introduction of English as the medium of Victorian Era England. Major characters of the novel are –
instruction in regional medium institutions that Dorian Gray, Basil Hallward, Sibyl Vane, Alan Campbell,
included only literature Lord Fermar etc. Gray breaks down when he sees his finished
Ans : (c) The introduction of English as the medium of portrait distraught by the fact that his beauty will fade while
instruction in a Western system of education that included the portrait stays beautiful.
literature, natural sciences and mechanical inventions is refers 14. Which of the following statements is true in terms of
to the recommendations of Charles Grant. Charles Grant distribution of metrical feet?
served as chairman of the British East India Company and as (a) Anapaestic is to Dactylic as Trochaic is to Iambic
a member of parliament. (b) Trochaic is to Anapaestic as Dactylic is to Iambic
10. In which of Anita Desai's novels does an insane wife (c) Iambic is to Trochaic as Anapaestic is to Dactylic
kill her husband? (d) Dactylic is to Trochaic as Iambic is to Anapaestic
(a) Voices in the City (b) In Custody Ans : (c) Statement (c) is true in terms of distribution of
(c) Cry, The Peacock (d) Baumgartner's Bombay
metrical feet.
Ans : (c) ‘Cry, The Peacock’ is the first novel by Anita Desai Iambic – Unstressed / Stressed
published in – 1963. In this novel Anita Desai portrays the Trochaic – Stressed / Unstressed
psychic tumult of a young and sensitive married girl Maya Anapaestic – Unstressed / Unstressed / Stressed
who is haunted by a childhood prophecy of a fatal disaster. Dactylic – Stressed / Unstressed / Unstressed.
She kills her husband Gautama.
15. Which of the following descriptions fits the unit of
11. The Sadler Commission Report (1917-1919) was verse, Dactyl?
critical of the quality of students graduating from
(a) One stressed syllable followed by three unstressed
the university and had very perceptive remarks on
syllables
English and the use of mother tongue in Indian
(b) One stressed syllable followed by two unstressed
education.
syllables
What was the commission appointed for?
(c) Two stressed syllables followed by one unstressed
(a) To examine the functioning of the Directorate of
syllable
Public Instruction in Delhi
(b) To study the problems of Calcutta University (d) Two stressed syllables followed by two unstressed
(c) To investigate and recommend teaching methods syllables
of languages generally Ans : (b) In Dactylic : a stressed syllable followed by two
(d) To evolve a three-language formula for the Indian unstressed syllables.
schools Éve wĭth hĕr / bás kĕt wăs /
Ans : (b) The Sadler Commission is known as The Calcutta Déep ĭn thĕ / bélls ănd grăss /
University Commission (1917). The Calcutta University 16. Who is the author of the short story, The Ghost of
Commission in 1917 under the chairmanship of the late Sir Firozsha Baag?
Michael Sadler. This commission was appointed for to study (a) Vikram Seth (b) V.S. Naipaul
the problems of Calcutta University. (c) Kiran Desai (d) Rohinton Mistry

UGC NTA NET/JRF English Paper II, June 2019 288 YCT
Ans : (d) ‘The Ghost of Firozsha Baag’ is a short story by 21. Which two titles from among the following deal with
Rohinton Mistry, who is an Indian born Canadian writer. He issues related to the institutionalization of English in
was awarded the Neustadt International Prize for Literature in post – independence India?
2012. i. Provocations ii. Professing Literature
17. Which one of the following paired terms is correct in iii. The Lie of the Land iv. The Muse Unchained
its explication? (a) (i) and (iv) (b) (i) and (iii)
(a) Phonology – Sound system (c) (ii) and (iii) (d) (iii) and (iv)
(b) Semiology – Ordering of Speech sounds Ans : (b) ‘Provocations’ and ‘The Lie of the Land’ are two
(c) Etymology – Sign system titles that deal with issues related to the institutionalization of
(d) Morphology – Evolution of words English in post-independence India. Hence option (b) is correct.
Ans : (a) Option (a) is correct paired. 22. Match the character with the Novel:
Phonology : the study of the elementary speech sound, it is a Character Novel
branch of linguistic that studies the sounds of human speech. A. Kate i. Great Expectations
18. Given below are two statements – one is labelled as B. Florence ii. Nicholas Nickle by
Assertion (A) the other is labelled as Reason (R) : C. Miss Havisham iii. David Copperfield
Assertion (A) : The dialects of English that have D. Agnes iv. Dombey and Son
resulted from the regional separation of English- Choose the correct option from those given below :
speaking communities have not acquired the status A B C D A B C D
of languages. (a) i iii iv ii (b) ii iv i iii
Reason (R) : The Germanic dialects that are now (c) iii i ii iv (d) iv ii iii i
Dutch, English, German, Swedish etc., have become Ans : (b) Option (b) is correctly matched the character with
distinct owing to geographical dispersion. the novel.
In the light of the above two statements choose the A. Kate – ii. Nicholas Nickle by
correct option: B. Florence – iv. Dombey and Son
(a) Both (A) and (R) are true and (R) is the correct C. Miss Havisham – i. Great Expectation
explanation of (A) D. Agnes – iii. David Copperfield
(b) Both (A) and (R) are true but (R) is not the correct Hence option (b) is correct.
explanation of (A) 23. Which of the following is the accurate description of
(c) (A) is true, but (R) is false 'dramatic irony'?
(d) (A) is false, but (R) is true (a) A character's knowledge or expectation is
Ans : (b) Both (A) and (R) are true but (R) is not the correct contradicted by what the audience knows, or by
explanation of (A). the outcome of events
19. In Eliot's Love Song of J. Alfred Prufrock who (b) An audience knows or expects something to
among the following painters is the subject of happen but the events on stage turn out to be
conversation among the perambulating women? different
(a) da Vinci (b) Raphael (c) Ironic events and expectations of actual actions
(c) Michelangelo (d) Donatello and results converge in drama and the audience
Ans : (c) ‘The Love Song of J. Alfred Prufrock’ is the first feels rewarded
professionally published poem by American born British poet (d) A dramatist's irony reinforces his actors' performance,
T.S. Eliot. It was first published in 1915. Michelangelo is the thereby fulfilling audience expectations
subject of conversation among the perambulating women. Ans : (a) 'Dramatic irony' is a literary device by which A
“In the room the women come and go character's knowledge or expectation is contradicted by what
Talking of Michelangelo.” the audience knows, or by the outcome of events. Dramatic
20. Match the Novelist with the Publisher: irony is a form of irony that is expressed through work.
A. Laurence Sterne i. Thomas Lowndes 24. Who among the following is mourned in Walt
B. Henry Fielding ii. Andrew Millar Whitman's O Captain! My Captain!?
C. Frances Burney iii. William Taylor (a) R. W. Emerson (b) John Keats
D. Daniel Defoe iv. Robert Dodsley (c) P.B. Shelley (d) Abraham Lincoln
Choose the correct option from those given below: Ans : (d) 'O Captain! My Captain' is an extended metaphor
A B C D A B C D poem written in 1865 by Walt Whitman, about the death of
(a) iii i ii iv (b) ii iv i iii American President - Abraham Lincoln. Hence option (d) is
(c) iv ii i iii (d) ii iii iv i correct.
Ans : (c) Option (c) is correctly matched the novelist with the 25. Match the following items/ideas with the writers
publisher– who first used/popularized them:
A. Laurence Sterne – iv. Robert Dodsley A. The Frontier Thesis i. Raymond Williams
B. Henry Fielding – ii. Andrew Millar B. The Lost Generation ii. Homi Bhabha
C. Frances Burney – i. Thomas Lowndes C. Third Space iii. F.J. Turner
D. Daniel Defoe – iii. William Taylor D. Structure of Feeling iv. Gertrude Stein
UGC NTA NET/JRF English Paper II, June 2019 289 YCT
Choose the correct option from those given below: 30. Which among the following clusters matches the
A B C D A B C D prose style that came to be known as 'Carlylese'?
(a) iv i ii iii (b) iii I iv ii (a) Capital letters, exclamation marks, phrases in
(c) iii iv ii i (d) i iii iv ii German
Ans : (c) Option (c) is correctly matched items/ideas with the (b) Question marks, long sentences, phrases in French
writers who first used/popularized them. (c) Frequent ellipses, Latin sayings, comic non-
sequitors
A. The Frontier Thesis iii. F.J. Turner
(d) Biblical phrases, capital letters, missing letters
B. The Lost Generation iv. Gertrude Stein
Ans : (a) The Clusters - capital letters, exclamation marks,
C. Third Space ii. Homi Bhabha
phrases in German matches the prose style that come to be
D. Structure of Feeling i. Raymond Williams known as - Carlylese.
Hence option (c) is correct. Hence option (a) is correct.
26. Considering the story of the novel, what does the 31. Which type of textual copy is concerned with an
title Dombey and Son stand for? assessment of the physical details of the books and
(a) It suggests the choice between a son and a daughter their exact relationship to the condition in which the
(b) It suggests the commercial aspect life book was planned to appear at the time of its initial
(c) It suggests the opposition between a father and a son publication?
(d) It suggests the importance of a dynasty (a) Real copy (b) Ideal copy
Ans : (a) 'Dombey and Son' is a novel by English novelist (c) Initial copy (d) Base copy
Charlse Dickens. The novel was first published in monthly Ans : (b) Ideal type of textual copy is concerned with an
parts between 1846 and 1848. This novel suggests the choice assessment of the physical details of the books and their exact
between a son and a daughter. Major character of his novel relationship to the condition in which the book was planned to
are - Mr. Paul Dombey, Fanny Dombey, Pilkins, Toodle, appear at the time of its initial publications.
Susan Nipper etc. Hence option (b) is correct.
27. Identify the two names from the following who are 32. Match the following journals with their
associated with Hermeneutics: distinguishing aims and methods of scholarship:
A. Obsidian i. Literature, History and the
i. Edmund Husserl ii. E.D. Hirsch
philosophy of history
iii. Martin Heidegger iv. Stephen Greenblatt
B. Clio ii. Literature and arts in the
Choose the correct option: African diaspora
(a) (i) and (iii) (b) (i) and (ii) C. Interventions iii. Feminist writing
(c) (ii) and (iii) (d) (ii) and (iv) D. Sings iv. Postcolonial writing
Ans : (a) 'Hermeneutics' is the theory and methodology of Choose the correct option from those given below:
interpretation especially the interpretation of Bible biblical A B C D A B C D
text, wisdom literature and philosophical text. Edmund (a) ii i iv iii (b) iv iii i ii
Husserl and Martin Heidegger are associated with (c) ii iv i iii (d) iii i iv ii
Hermeneutics. Edmund Husserl was a German philosopher Ans : (d) Option (d) is correct journals with their
and Martin Heidegger was also a German philosopher. Hence distinguishing aims and methods of scholarship.
option (a) is correct. A. Obsidian iii. Feminist writing
28. In which play, other than Julius Caesar, has B. Clio i. Literature, History and the
Shakespeare depicted the Romans better than the philosophy of history
Roman writers themselves have done? C. Interventions iv. Postcolonial writing
(a) Troilus and Cressida (b) Coriolanus D. Signs ii. Literature and arts in the
(c) Romeo and Juliet (d) Two Gentlemen of Verona African diaspora
Ans : (b) 'Coriolanus' is a tragedy by William Shakespeare 33. "He that is not with us is against us. He that is not
believe to have been written between 1605 and 1608. In this against us is with us." Who said this?
play William Shakespeare had depicted other than Julius (a) Charles Lamb (b) Francis Bacon
Caesar, the Romans better than the Roman writers themselves (c) Samuel Johnson (d) R.W. Emerson
have done. Ans : (b) The lines,, "He that is not with us is against us. He
Hence option (b) is correct. that is not against us is with us" appear in the famous essay of
Francis Bacon in 1601 entitled - "Of Unity in Religion".
29. What is 'euphuism'?
Hence option (b) Francis Bacon is correct.
(a) Eulogical and adulatory style of writing
34. Which one of the following novels of Jane Austen
(b) Discursive and hortatory style of writing
was abandoned unfinished?
(c) Pompous and affected style of writing
(a) Northanger Abbey (b) Persuasion
(d) Exalted and grand style of writing (c) The Watsons (d) Emma
Ans : (c) A euphuism is an innocuous word or expression Ans : (c) Jane Austen (1775-1817) was famous novelist of
used in place of one that may be found offensive or suggest Romantic era. 'The Watsons' is unfinished novel of Jane
something unpleasant. It is pompous and affected style of Austen. Her another novel are very famous such as - Pride
writing. and Prejudice (1813), Sense and Sensibility (1811), Northanger
Hence option (c) is correct. Abbey (1818), Emma (1816), Persuasion (1818) etc.

UGC NTA NET/JRF English Paper II, June 2019 290 YCT
35. Which of the following works is reviewed in George 39. Which artistic technique best describes the interplay
Orwell's essay, Inside the Whale? of light and shade in the following lines?
(a) Henry Miller's Tropic of Cancer "I have looked at it so long
(b) James Joyce's Ulysses I think it is part of my heart. But it flickers.
(c) D.H. Lawrence's Lady Chatterley's Lover Faces and darkness separate us over and over
A woman bends over me,
(d) Anais Nin's Delta of Venus
Searching my reaches for what she really is
Ans : (a) "Inside the Whale" is an essay in three parts written Then she turns to those liars, the candles or the moon.
by George Orwell in 1940. It is primarily a review of Henry I see her back, and reflect it faithfully."
Millers work - Tropic of Cancer. Hence option (a) is correct. (a) Collage (b) Flashback (c) Montage (d) Chiaroscuro
36. Match the works with authors: Ans : (d) The term 'Chiaroscuro' originated during the
Works Authors Renaissance as drawing on coloured paper but in literature
A. Image-Music-Text i. M.H. Abrams Chiaroscuro is the use of strong contrast between light and
B. Why Marx was ii. Raymond Williams dark, usually bold contrast affecting the whole composition.
So the lines–
Right
"I have looked at it so long
C. The Mirror and iii. Roland Barthes I think it is part of my heart. But it flickers.
the Lamp Faces and darkness separate us over and over
D. Culture and Society iv. Terry Eagleton A woman bends over me,
Choose the correct option from those given below: Searching my reaches for what she really is
A B C D A B C D Then she turns to those liars, the candles or the moon.
(a) i ii iv iii (b) iv iii ii i I see her back, and reflect if faithfully" are best example of
(c) ii i iii iv (d) iii iv i ii Chiaroscuro artistic technique.
Ans : (d) Option (d) is correctly matched the works with 40. Which of the following poets does William Hazlitt
call 'Don Quixote-like' in his essay, My First
authors–
Acquaintance with Poets?
A. Image-Music-Text iii. Roland Barthes (a) William Wordsworth (b) Samuel Taylor Coleridge
(1977) (c) William Cowper (d) Lord Byron
B. Why Marx was Right iv. Terry Eagleton Ans : (a) 'My First Acquaintance with Poets' was published in
(2011) 1823 in a short-lived but a highly significant periodical of the
C. The Mirror and the i. M.H. Abrams Romantic age by William Hazlitt. In this essay William
Lamp (1953) Hazlitt calls William Wordsworth - 'Don Quixote- like'. 'Don
D. Culture and Society ii. Raymond Williams Quixote' is a Spanish novel by Miguel De Cervantes.
(1958) 41. Match each of the following concepts/ objects with
Hence option (d) is correct. the corresponding description:
A. Farce i. Articles and objects used on
37. Who among the following established and
the stage
popularized the concept of 'Cardinal Vowels'? B. Props ii. Drama written to be read
(a) A.S. Hornby (b) E.V. Lucas rather than acted
(c) Daniel Jones (d) C.J. Dodson C. Music hall iii. Characterized by broad
Ans : (c) 'Cardinal Vowels' are a set of reference vowels used humour, wild antics,
by phoneticians in describing the sounds of language. Daniel slapsticks etc.
Jones established and popularized the concept of 'Cardinal D. Closet drama iv. Variety entertainment of
Vowels'. He wrote - "The values of cardinal vowels cannot be songs, comic turns that
learnt from written descriptions; they should be learnt by oral flourished in England through
instruction from a teacher who knows them." the late 19th century
Choose the correct option from those given below :
38. Which one of the following of Plato's beliefs/ acts A B C D A B C D
was Shelley countering by Saying that 'Poets are the (a) iv ii i iii (b) iii i iv ii
unacknowledged Legislators of Mankind'? (c) i iii ii iv (d) ii iv iii i
(a) Banishment of poets from the republic Ans : (b) Option (b) is correctly matched concepts/objects
(b) Distrust of value of poetry for mankind with the corresponding description–
(c) Preference for legislators over poets A. Farce iii Characterized by broad
(d) Description of poets as mad men . humour, wild antics,
Ans : (c) Shelley countering by saying that 'Poets are the slapsticks etc
unacknowledged legislators of mankind' in his famous critical B. Props i. Articles and objects used
work - 'In A Defence of Poetry' written in 1821 and first on the stage
C. Music hall iv Variety entertainment of
published posthumously in 1840 in essays, Letters from
songs, comic turns that
Abroad, Translation and fragments by Edward Moxon in flourished in England
London. This essay was written in response to his friend through the late 19th
Thomas Love Peacock's article 'The Four Ages of Poetry' century
published in 1820. Shelley nominated unlikely figures such as D. Closet drama ii. Drama written to be read
-Plato and Jesus in their excellent use of language to conceive rather than acted
the inconceivable, in preference for legislators over poets. Hence option (b) is correct.

UGC NTA NET/JRF English Paper II, June 2019 291 YCT
42. "To see him act is like reading Shakespeare by 46. Given below are two statements – one is labelled as
flashes of lighting." About which Shakespearean Assertion (A) and the other is labelled as Reason (R) :
actor Coleridge wrote the above line? Assertion (A) : Instances of beliefs triggering action
(a) David Garrick (b) Richard Burbage are present in social life and may give rise to
(c) John Philip Kemble (d) Edmund Kean problems in determining 'causality'.
Ans : (d) Coleridge wrote the line "To see him act is like Reason (R) : Beliefs may not be accompanied by or
give rise to logically appropriate actions, and actions
reading Shakespeare by flash of lighting" for the celebrated
may occur which are consistent with motivations
British Shakespearean stage actor - Edmund Kean, born in
and intentions, but they often, if not usually, also
England. It was in the impersonation of the great creations of
have unanticipated outcomes.
Shakespeare's genius that the varied beauty and grandeur of
In the light of the above two statements choose the
the acting of Kean were displayed in their highest form. His correct option:
main disadvantage as an actor was his small stature. (a) Both (A) and (R) are true and (R) is the correct
43. Match the critics and their works: explanation of (A)
Critics Works (b) Both (A) and (R) are true and (R) is not the correct
A. Edward Said i. The Illusions of explanation of (A)
Postmodernism (c) (A) is true, but (R) is false
B. Terry Eagleton ii. Contemporary Marxist (d) (A) is false, but (R) is true
Criticism Ans : (a) Option (a) is correct. Both (A) and (R) are true and
C. Francis Mulhern iii. Theory into Practice (R) is the correct explanation of (A).
D. K.M. Newton iv. Culture and 47. What, according to Raymond Williams, is the right
Imperialism description of the term 'Cultural Materialism'?
Choose the correct option from those given below : (a) The cultural effect that religion has in social life
A B C D A B C D (b) The political effect that matter has in social lives
(a) iv i ii iii (b) iv i iii ii (c) The material effect that culture has in wider social
(c) ii i iv iii (d) i ii iii iv life
Ans : (a) Option (a) is correctly matched the critics and their (d) The effect of social life in cultural situations of
work– uncertainty
A Edward Said iv Culture and Imperialism Ans : (c) 'Cultural Materialism' emerged as a theoretical
movement in the early 1980s along with 'new historicism'.
(1993)
The term was coined by Raymond Williams. Cultural
B Terry Eagleton i The Illusions of
materialism makes analysis based in critical theory in the
Postmodernism (1996)
tradition of the Frankfurt school. According to Raymond
C Francis ii Contemporary Marxist Williams 'Cultural Materialism' is the material effect that
Mulhern Criticism (1992) culture has in wider social life.
D K.M. Newton iii Theory into Practice 48. Which novel by J.G. Farrel describes the
(1992) experiences of a polio victim?
Hence option (a) is correct. (a) Troubles (b) The Singapore Grip
44. Which of the following plays is characterized by the (c) The Lung (d) The Hill Station
exclusivity of a single character talking to himself? Ans : (c) 'The Lung' is a novel written by James Gardon
(a) A Streetcar Named Desire (b) Equus Farrel, published in 1965. In this novel he drew upon his own
(c) The misanthrope (d) Krapp's Last Tape affliction with 'Polio' which he contracted at Oxford to
Ans : (d) 'Krapp's Last Tape' is a one act play in English by present a downbeat portrait of an irascible man confined to an
Samuel Beckett, with a cast of one man it was written for iron lung. So this novel describes the experiences of a polio
Northern Irish actor - Patrick Magee. The play was first victim.
performed as a curtain raiser to 'Endgame' from 28 October to 49. Which of the following two points were emphasised
29 November 1958 at the royal court theatre London. This by 'Wood's Dispatch of 1854'?
play is characterized by the exclusivity of a single character i. Teaching of the English language along with the
talking to himself. study of vernacular language
ii. compulsory inclusion of Christianity in the
45. Who among the following explored the shifting and
curriculum
contested power-relations, knowledge and the
iii. The gradual withdrawal of government patronage
human body? from Indian languages
(a) Louis Althusser (b) Clifford Geertz iv. The importance of female education
(c) Jacques Lacan (d) Michel Foucault (a) (i) and (iv) (b) (i) and (ii)
Ans : (d) Michel Foucault explored the shifting and contested (c) (i) and (iii) (d) (ii) and (iii)
power relations, knowledge and the human body. He was Ans : (a) Sir Charles Wood, the President of the board of
French historian and philosopher. His famous works are as control, had an important effect on spreading English learning
follows: History of Madness in the Classical Age (1961), The and female education in India. In 1854 he sent a dispatch to
Birth of the Clinic (1963), Discipline and Punish (1975), The Lord Dalhousie to suggest the teaching of English language
History of Sexuality. along with the vernacular language.

UGC NTA NET/JRF English Paper II, June 2019 292 YCT
50. From among the following, identify the two correct 55. All in the world know the beauty of the beautiful,
statements in Johnson's criticism of Shakespeare: and in doing this they have (The idea of) what
A. His Athenians are not sufficiently Greek and his ugliness is; they all know the skill of the skillful, and
kings not completely royal. in doing this they have (the idea of) what the want of
B. He sacrifices virtue to convenience and is more the skill is.
careful to please than to instruct. So it is that existence and non-existence gave birth to
C. He adheres to strict chronology and gives to one (the idea of) the other, that difficulty and ease
age or nation only its own customs and opinions. produce (the idea of) the other, that the length and
D. He sacrifices reason, property and truth to pursue shortness fashion out the one figure of the other,
even a poor and barren quibble. that (the idea of) height and lowness arise from the
contrast of one with the other, that the musical notes
Choose the correct option:
and tones become harmonious through the relation
(a) A and B (b) A and C
of one with another, and that being before and
(c) C and D (d) B and D
behind give the idea of one following another.
Ans : (d) Dr. Johnson was a lexicographer, editor, biographer Which one of the following is the correct meaning of
and literary critic. In his critical work 'Preface to Shakespeare' the ominous little phrase "the idea of' in the first
he had given an appreciative analysis about the Shakespeare, sentence of the passage?
as option (b) and (d). (a) Prior knowledge (b) Prior imagination
A. He sacrifices virtue to convenience and is more careful to (c) Prior confirmation (d) Prior rejection
please than to instruct. Ans : (a) Priar knowledge is the knowledge the learner
B. He sacrifices reason, property and truth to pursue even a already has before they meet new information. A learner's
poor and barren quibble. understanding of a text can be improved by activating their
51. Identify the stage that falls between the imaginary prior knowledge before dealing with the text, and developing
and symbolic stages according to Jacques Lacan: this habit is good learner training for them. So the correct
(a) Middle stage (b) Minor stage meaning of the ominous little phrase "the idea of" in the first
(c) Medieval stage (d) Intermediate stage sentence of the passage is option (a) Prior knowledge.
Ans : (*) The question remains cancelled and marks awarded 56. What is the meaning of Ziaudding Sardar's statement?
to all candidates appeared in the test. "Cultural studies started as a dissenting intellectual
tradition outside academia, dedicate to exposing
52. Which one of the following arrangements of poets is
power in all its cultural forms. But it has now
in the correct chronological order?
become a discipline and a part of the academic
(a) William Langland, William Dunbar, Layamon establishment and its power structure."
(b) William Langland, Layamon, William Dunbar (a) Devolution (b) Displacement
(c) Layamon, William Langland, William Dunbar (c) Institutionalization (d) Dissension
(d) William Dunbar, Layamon, William Langland Ans : (c) Ziauddin Sardar's statement - "Cultural studies
Ans : (c) Correct chronological order of the poet is option (c). started as a dissenting intellectual tradition outside academia,
Layamon (Late 12th/early 13th century) dedicate to exposing power in all its cultural forms. But it has
William Langland (1332-1386) now become a discipline and a part of the academic
William Dunbar (1459-1520) establishment and its power structure."
Hence option (c) is correct. Institutionalization– Institutionalization refers to the process
53. Which of the following aptly names the language of embedding some conception within an organization social
resulting from the contact of two mutually system or society as whole.
unintelligible language systems? 57. What is the meaning of 'langue' in Saussurean
(a) Creole (b) Dialect linguistics?
(c) Colloquial (d) Pidgin (a) Individual speech acts
(b) An organized system of differences
Ans : (d) A Pidgin is grammatically simplified means of
(c) The dialectic between thought and speech
communication that develops between two or more groups
(d) Language in the abstract sense
that do not have a language in common. Hence option (d) is
Ans : (b & d) Ferdinand de Sassure was a Swiss linguist and
correct.
semiotician. In his 'Course of General Linguistics' Ferdinand
54. Which of the following poems by Thomas Hardy de Saussure introduced term - Langue, Language and parole.
was originally titled by the century's Deathbed? According to him Langue means Language in the abstract
(a) The minute Before Meeting (b) Neutral Tones sense and an organized system of differences. Langue is a
(c) The Darkling Thrush (d) The Oxen composite of all the traits of a language, think of it as a big
Ans : (c) 'The Darkling Thrush' is one of Hardy's most container that hosts all the parts of language as we use it.
anthologized poem. The word 'Darkling' means 'in the dark' Particularly the image and words that come to mind and the
and has poetic associations with Keat's 'Ode to a Nightingale'. sound-to-symbol connection that occurs when we speak. All
The poem was written on 31 December 1900 and was first of those stared images, words-meaning and sub-meanings
called - 'By the Century's Deathbed'. comprise 'Langue'.

UGC NTA NET/JRF English Paper II, June 2019 293 YCT
58. Which of the following statements best describes (c) A pre-publications process in which work
T.S. Eliot's assertion that Shakespeare's Hamlet is submitted for publication is accompanied by
an 'artistic failure'? recommendation of other experts in the field
(a) Hamlet's emotion is not adequately objectified (d) A post-publication process in which the work is
(b) Hamlet's feelings far outweigh the release of his submitted for a professional review
emotions Ans : (b) 'Peer review' means - "A pre publication process in
(c) Hamlet's obsession should have been within which work submitted for publication is evaluated for quality
representational limits by expert in the field. Peer review is the evaluation of work
(d) Hamlet's indecisiveness slows the steady progress by one or more people with similar competences as the
of action producer of the work (Pear) It functions as a form of self
Ans : (a) 'Hamlet and his Problems' is an essay written by regulation by qualified members of a profession within the
T.S. Eliot in 1919, that offers a critical reading of Hamlet. relevant field.
The essay first appeared in Eliot's - The Sacred Wood : Essay 62. Who is referred to as 'beast' in the quote 'Kill the
on Poetry and Criticism in 1920. It was Eliot who said beast! cut his throat! Spill his blood' in William
Hamlet, far from being success. Shakespeare's masterpiece Golding's Lord of the Flies?
'Hamlet' is most certainly is an artistic failure. Eliot thought (a) Ralph (b) Piggy (c) Simon (d) Roger
that Shakespeare could not established "Objective Co- Ans : (c) After Killing yet another pig, the boys chant - "Kill
relatives" to Hamlet's sensibilities. Hamlet's emotion is not the beast! Cut his throat! Spill his blood". They say it as the
adequately objectified. way react the murdering of the pig. When Simon appears,
they see him not as a human being, but as the beast and
59. Which one of the following groups of novelists has,
violently kill him. 'Lord of the Flies' is a novel by Nobel Prize
in the given order, Captain Ahab, Hester Prynne,
winning British author - William Golding, published in 1954.
Roderick Usher and Daisy Miller as characters in
63. Who among the following analysed the naturalization
their novels?
of connotative meanings into myths?
(a) Henry James, Edgar A. Poe, Nathaniel Hawthorne,
(a) Michel Foucault (b) Roman Ingarn
Herman Melville
(c) J. Hillis Miller (d) Ronald Barthes
(b) Herman Melville, Nathaniel Hawthorne, Edgar A.
Ans : (d) Ronald Gerard Barthes was a French Literary
Poe, Henry James
theorist, philosopher, critic and semiotician. Barthes ideas
(c) Edgar A. Poe, Henry James, Nathaniel Hawthorne,
explored a diverse range of field and he influenced the
Herman Melville
development of many schools of theory including
(d) Nathaniel Hawthorne, Edgar A. Poe, Henry James,
structuralism, semiotician, social theory, design theory
Herman Melville
anthropology and post-structuralism. He analyzed the
Ans : (b) Option (b) is correct. naturalization of connotative meaning into myth.
Character Work Novelist 64. What is the name of the poetic style characterized by
Captain Ahab Moby Dick Herman short staccato rhymed lines, as shown below?
Melville What can it avayle
Hester Prynne The Scarlet Letter Nathaniel To dryve forth a snayle,
Hawthorne Or to make a sayle
Roderick Usher The Fall of the Edgar A. Poe Of a herynges tayle?
house of Usher (a) Cranmerish (b) Wolseyan (c) Chaucerian (d) Skeltonic
Essay Ans : (d) 'Skeltonic' short verses of an irregular meter much
Daisy Miller Daisy Miller Henry James used by the Tudor poet John Skelton. The Verses have two or
(Novella) three stresses arranged sometimes in falling and sometime in
60. From which Greek word does the term 'comedy' rising rhythm.
derive and what does it man? 65. What is the Priest's entreaty to Oedipus in the
(a) Comedia, largeness of heart opening scene of Oedipus Rex?
(b) Komoidia revel-song (a) To liberate Thebes from the domination of the Sphinx
(c) Commodios, commodious (b) To rid Thebes of the plague that afflicts its people
(d) Komedieon, Light foolery (c) To afford the Thebans the luxury of newer forms
Ans : (b) The term 'Comedy' is derived from Greek word of worship
'Komoidia' which means - revel song. (d) To send creon to seek advice from the oracle of
61. Which one of the following is the right definition of Delphi oracle
'peer review'? Ans : (b) Oedipus Rex, also known by its Greek title Oedipus
(a) A post-publication process in which the work is Tyrannous or Oedipus the king is an Athenian tragedy by
submitted to a panel of reviewers for ascertaining Sophocles. Sophocles's Oedipus Rex begins as a priest of
quality Apollo asks king Oedipus of Thabes to help end the plague
(b) A pre-publication process in which work that is ravaging the city. In response, Oedipus reveals that he
submitted for publication is evaluated for quality has already sent his brother in law Creon, to consult with the
by experts in the field oracle of Apollo at Delphi on the matter.

UGC NTA NET/JRF English Paper II, June 2019 294 YCT
66. While looking for publication details of a book, a Ans : (c) The following lines–
researcher may consult the book's copyright page, "If it be aught toward the general good,
which may appear Set honor in one eye and death i' th' other,
(a) just after the cover And I will look on both indifferently" are said by Brutus to
(b) usually the reverse of the title page Cassius in Act I scene II in Julius Caesar. Julius Caesar is a
(c) invariably the reverse of the title page history play and tragedy by William Shakespeare believed to
(d) just before the title page have been written in 1599.
Ans : (b) The book's copyright page appears usually the 71. Which one of the following is the source of the
reverse of the title page. passage given below?
"I have observed with growing anxiety the career of
67. Which two writers have written essays on the this word culture during the past six of seven years.
defence of poetry? We may find it natural, and significant, that during
A. Sir Philip Sidney B. P.B. Shelley a period of unparalleled destructiveness, this word
C. Dr. Mathew Arnold D. T.S. Eliot should come to have an important role...."
Choose the correct option : (a) F.R. Leavis, Mass Civilization and Minority Culture
(a) A and D (b) A and C (b) T.S. Eliot, Notes Towards the Definition of Culture
(c) C and D (d) A and B (c) Raymond Williams, Culture and Society
Ans : (d) Sir Philip Sidney and P.B. Shelley have written (d) Stuart Hall, Cultural Representations and
essays on the defence of poetry. Sidney's "The Defence of Signifying Practices
Poesy" was originally published under two different title, 'The Ans : (b) Source of the given passage is - Notes Towards the
Defence of Posie' and 'An Apologie for Poetrie'. Definition of Culture by T.S. Eliot, that originally appeared as
"A Defence of Poetry" is an essay by the English poet P.B. a series of articles in 'New England Weekly' in 1943. It was
published in book form in 1948. Eliot presents culture as an
Shelley written in 1821. This essay was written in response to
organic, shared system of beliefs that cannot be planned or
his friend - Thomas Love, Peacock's article "The Four Ages
artificially induced.
of Poetry".
72. Identify the author in whose works the character
68. Which of the following correctly describes 'black Ashenden appears many times:
humour' as a morbid and provocative treatment of (a) Dorothy Sayers (b) Daniel Defoe
(a) old age and disease (c) D.H. Lawrence (d) Somerset Maugham
(b) youth and passionate love Ans : (d) A character named 'William Ashenden' is the
(c) death and disease narrator of Maugham's novel 'Cakes and Ale', 'The Moon',
(d) childhood and accident 'Sixpence' and 'The Razar's Edge'. Ashenden is also the name
Ans : (c) 'Black Comedy' also known as black humour, dark of a character who appears in several of Maugham's short story.
comedy or gallows humour is a comic style that makes light 73. Who among the following is celebrated in John
of subject matter that is generally considered taboo, Keats's Lines on the Mermaid Tavern?
particularly subjects that are normally considered serious or (a) Jack, the Ripper (b) Bryson of the Park
painful to discuss or death and disease. (c) Jack, the Giant-Killer (d) Robin Hood
69. Read the following liens: Ans : (d) Robin Hood is celebrated in John Keats's 'Lines on
IN A STATION OF THE METRO the Mermaid Tavern'.
Souls of poets dead and gone,
The apparition of these faces in the crowd:
What Elysium ………….
Petals on a wet, black bough. ………………………….
Which of the following poetic programmes is ……………… O generous food!
illustrated by the above lines? Drest as though bold Robin Hood.
(a) The Movement (b) Naturalism 74. "The last temptation is the greatest treason
(c) Symbolism (d) Imagism To do the right deed for the wrong reason."
Ans : (d) In the above line 'In A Station of The Metro' that is (T.S. Eliot, Murder in the Cathedral)
composed by Ezra Pound, poetic programme 'Imagism' is Why is the 'temptation', 'treason' for the speaker of
illustrated. Imagism is a movement in early 20th century the lines?
English and American poetry which sought clarity of (a) It is only self-serving (b) It is not intended
expression through the use of precise image. The movement (c) It violates a norm (d) It is conspiratorial
derived in part from the aesthetic philosophy of T.E. Hulme Ans : (a) The lines - "The Last temptation ………." is spoken
and involved Ezra Pound, James Joyce, Amy Lowell and by Thomas in T.S. Eliot's play 'Murder in the Cathedral'. As
others. the Chorus, Priests and Tempters speak together about the
uncertainty of life Thomas retreats into himself to consider
70. Who says the following lines and to whom?
the Fourth Tempter's promise that he could find glory if he
"If it be aught toward the general good,
wills martyrdom for himself. When he speaks again
Set honor in one eye and death i' th' other, beginning a long speech with the above lines he has firmly
And I will look on both indifferently." committed to dying for the right reason. Thomas's are in the
(a) Octavius to Antony play is to first acknowledged that his pride is leading him
(b) Hamlet to Claudius towards "the right deed for the wrong reason" and then to rid
(c) Brutus to Cassius himself of the "self" the personality that is keeping him from
(d) Casca to Calpurnia being God's instrument.

UGC NTA NET/JRF English Paper II, June 2019 295 YCT
75. Who speaks the following lines and to whom? Ans : (b) "Culture is ordinary : that is the first fact" is the
"O, look upon me, sir, source of Raymond Williams famous book 'Resources of
And hold your hands in benediction o'er me. Hope : Culture, Democracy, Socialism'. Hence option (b) is
No, sir, you must not kneel." correct.
(a) Kent to Lear (b) Cordelia to Lear 80. Who of the following are being talked about in the
(c) Goneril to Lear (d) Regan to Kent following lines?
Ans : (b) Cordelia speaks the lines– "..... you seem to understand me,
"O, look upon me, sir, By each at once her choppy finger laying
And hold your hands in benediction o'er me. Upon her skinny lips : you should be women,
No, sir, you must not kneel" to King Ler, in the famous An yet your beards forbid me to interpret
tragedy of William Shakespeare's 'King Lear' in Act-IV, That you are so."
Scene VII (A tent in the French Camp Lear on a bed asleep). (a) The plebeians in Coriolanus
(b) The sisters in King Lear
76. Which writer applied the term 'cultural poetics' to (c) The witches in Macbeth
his own critical contribution to make literature and (d) The players in Hamlet
arts as part of social practice?
Ans : (c) These lines–
(a) Stephen Greenblatt (b) Mikhail Bakhtin "..... you seem to understand me,
(c) Jonathan Dollimore (d) Raymond Williams By each at once her choppy finger laying
Ans : (a) 'Cultural poetics' also known as the 'New Upon her skinny lips : you should be women,
Historicism' in America and 'Cultural Materialism' in Britain, An yet your beards forbid me to interpret
is a form of literary analysis whose purpose is to discover the That you are so" appears in famous tragedy of William
original ideology behind significant historical and biological Shakespeare's 'Macbeth'. These lines occurs in Act I Scene III
facts about writers, resources and the art they create. Stephen and are being talked about 'The Witches in Macbeth'.
Greenblatt applied the term 'Cultural poetics' to his own 81. Who is the author of the essay, The Rational of the
critical contribution to make literature and arts as part of Copy-Text?
social practice. (a) Freson Burns (b) W. W. Greg
77. Which one of the following correctly describes the (c) R. B. McKerrow (d) Paul Maas
meaning of Macbeth's words ' ....life is but a walking Ans : (b) 'The Rational of the Copy-Text' is an essay by
shadow'? W.W. Greg (1949).
(a) Life is just devoid of light 82. Why did T.S. Eliot assert that Virgil, not Homer, is
(b) Life is just devoid of substance the poet of Europe?
(c) Life is just devoid of spirit (a) There are some initial moral concerns in Virgil
(d) Life is just devoid of stability (b) Virgil belongs to the Roman period
Ans : (b) The Macbeth Quote from Act V, Scene V "Out, out (c) Homer was a pagan who was a renegade
(d) Virgil wrote in Latin while Homer wrote in Greek
brief Candle Life's but a walking shadow" is a famous
Shakespeare Quote on death. These lines are spoken by Ans : (a) There are some initial moral concerns in Virgil
Macbeth after hearing the news of his wife's death. In the that’s reason T.S. Eliot assert that Virgil, not Homer, is the
poet of Europe. Hence option (a) is correct.
above lines, the brief candle is related with short life span
Macbeth in the state of numbness and pain after his wife's 83. Which of the following combinations correctly
death compares life with a brief candle, a walking shadow, a defines the phonological system of Indian English in
relation to Standard English?
poor player, a tale told by an idiot. Option (b) correctly
A. Absence of aspirated consonants
describes the meaning of Macbeth's words - 'Life is just
B. Simplified vowel system
devoid of substance'.
C. Similar international pattern
78. In which of the following paired terms, the D. Presence of voiced aspirated consonants
relationship between the active and passive forms of Choose the correct option:
a sentence can best established? (a) A and B (b) B and D (c) C and A (d) B and C
(a) Deep structure – Surface structure Ans : (b) Option (b) and (d) correctly defines the
(b) Signifier – Signified phonological system of Indian English in relation to Standard
(c) Metaphor – Metonymy English - simplified vowel system and presence of voiced
(d) Syntagmatic – Paradigmatic aspirated consonants. Hence option (b) is correct.
Ans : (a) 'Deep structure and Surface structure' can be best 84. What was Gramsci's term for cultural consensus
established in the relationship between the active and passive supporting capitalism?
forms of a sentence. Deep structure and surface structure are (a) Monopoly (b) Ideology (c) Discourse (d) Hegemony
concepts used in linguistic syntax in the Chomskyan tradition Ans : (d) Antonio Gramsci was an Italian Marxist intellectual
of 'transformational generative grammar'. and Politician who can be seen as the perfect example of the
79. "Culture is ordinary : that is the first fact." Which synthesis of theoretician and politician. Hegemony was
one of the following is the source of this statement? Gramsci's term for cultural consensus supporting capitalism.
(a) The Country and the City (b) Resources of Hope Hegemony is the political, economic or military
(c) The Long Revolution (d) Keywords predominance or control of one state over other.

UGC NTA NET/JRF English Paper II, June 2019 296 YCT
85. The medieval English university organized its 89. According to the passage given, which of the
studies based on the seven liberal arts Three of following correctly captures the meaning of 'a
these, The trivium, referred to the study of cyclopaedic history of the universe'?
(a) arithmetic, geometry, music (a) The knowledge about the universe from its
(b) astronomy, music, logic beginning to its possible end
(c) Geometry, grammar, music (b) A catalogue of rivers, mountains and continents
(d) grammar, logic, rhetoric (c) Statements about the universe based on logic and
Ans : (d) The medieval English University organized its mathematics
studies based on the seven liberal arts. Three of these, The (d) A published encyclopaedia of the universe
trivium, referred to the study of - grammar, logic, rhetoric. Ans : (a) Option (a) 'The knowledge about the universe from
its beginning to its possible end' correctly capture the
86. What term used by Ferdinand de Saussure
meaning of 'a cyclopaedic history of the universe'.
corresponds to Noam Chomsky's term
'performance'? 90. According to the writer, perception is the basic
(a) Difference (b) Parole (c) Paradigm (d) Langue epistemology. Which one of the following is the other
accepted epistemology?
Ans : (b) Ferdinand de Saussure used the term - Parol in
(a) Language (b) Experience (c) Inference (d) Simile
corresponds to Noam Chomsky's term "performance". Parole
Ans : (c) 'Epistemology' is the theory of knowledge,
typically when it is translated means - speech. Saussure, on
especially with regard to its methods, validity and scope, and
the other hand, intended for it to mean both the written and
the distinction between justified believe and opinion. It is a
spoken language as experienced in everyday life.
branch of Philosophy, Language, Experience and Simile is
87. Which of the following sociologists' ideas on the accepted in epistemology whether 'Inference' is not. Inference
practice of receiving and giving gifts are used by J. is an idea or conclusion that's drawn from evidence and
Hillis Miller to reinforce her arguments in the essay, reasoning. An inference is an educated guess.
Critic as Host? 91. Given below are two statements - one is labelled as
(a) Emile Durkheim (b) Max Weber Assertion (A) and the other is labelled as Reason (R):
(c) Marcel Mauss (d) Daniel Bell Assertion (A) : Language constructs meaning.
Ans : (c) Joseph Hillis Miller is an important humanities and Reason (R) : Language structures meanings
literature scholar. He was among the pioneer in introducing depending on the speaking subjects' perception,
phenomalogy and deconstruction to Anglo-American context and auditor(s).
audience. Sociologists Marcel Mauss's ideas on the practice In the light of the above two statements choose the
of receiving and giving gifts are used by J. Hillis Miller to correct option:
reinforce her arguments in the essay - Critic as Host. (a) Both (A) and (R) are true and (R) is the correct
88. What does 'Harlem Renaissance' refer to? explanation of (A)
(a) A scientific and rational ethos, including freedom (b) Both (A) and (R) are true but (R) is not the correct
from superstition, in 18th century Europe explanation of (A)
(b) The flourishing of African American literature in (c) (A) is true, but (R) is false
(d) (A) is false, but (R) is true
the 1920s and 1930s
(c) A church system, overseen by a governing Ans : (a) Option (a) is correct. Both (A) and (R) are true and
hierarchy of four courts, championed by the (R) is the correct explanation of (A).
Language structures meaning.
English puritans
Language structures meanings depending on the speaking
(d) The revelation of Christ to the Gentiles in the
subjects perception, context and auditors.
persons of the Magi
92. Which one of the following words best describes the
Ans : (b) A period of remarkable creativity in literature,
heroes of Cervantes' Don Quixote, Mark Twain's
music, dance, painting and sculpture by African-Americans,
The Adventures of Tom Sawyer and Thomas
from the end of the First World War 1917 to the early 1930s. Mann's The Confessions of Felix Krull?
So Harlem Renaissance refers to - The flourishing of African (a) Ficelle (b) Picaro
American literature in the 1920s to 1930s. (c) Mannequin (d) Philanderer
Instruction for Q.N0. (89-90) Ans : (b) 'Picaro' is a Spanish word which means a "rogue"
Comprehension: and "Knave". The Picaresque novel the word - 'Picaresque'
It is an axiom in mental philosophy, that we can think has been derived from the word - Picaro.
of nothing which we have not perceived. When I say that we The heroes of Cervantes Don Quixote, Mark Twain's - The
can think of nothing, I mean we can imagine nothing, we can Adventure of Tom Sawyer and Thomas Mann's - The
reason of nothing, we can remember nothing, we can foresee confessions of Felix kurll are 'rogue' and 'knave'.
nothing. The most astonishing combinations of poetry, the 93. In the study of Anglo-American literatures, certain
subtlest deductions of logic and mathematics, are no other than distinguished names in critical/editorial scholarship
combinations which the intellect makes of sensations according become synonymous with famous writers and
to its own laws. A catalogue of all the thoughts of the mind, periods of literary history.
and of all their possible modifications, is a cyclopedic history Match the following names with their respective
of the universe. areas of scholarship:
UGC NTA NET/JRF English Paper II, June 2019 297 YCT
A. Edward Mendelson i. John Milton Ans : (b) The term 'University Wits' was coined by George
B. Jerome McGann ii. Ezra Pound Saintsbury. The University Wits is a phrase used to name a
C. Stanley Fish iii. W.H. Auden group of late 16th century playwrights and pamphleteers who
D. Hugh Kenner iv. Textual Scholarship were educated at the University - Oxford or Cambridge.
Choose the correct option from those given below: Prominent member of this group were - Christopher Marlow,
A B C D A B C D Robert Green and Thomas Nash from Cambridge and John
(a) ii i iv iii (b) iii iv i ii Lyly, Thomas Lodge and George Peel from Oxford. So
(c) iv iii ii i (d) iii ii iv i Thomas Nashe is correct answer, whether Thomas Hooker,
Ans : (b) Option (b) correctly matched the names with their Michael Drayton, William Harvey are not associated with
respective areas of scholarship– University Wits.
A. Edward Mendelson iii. W.H. Auden Instruction for Q.N0. (97-100)
B. Jerome McGann iv. Textual THE GROCER'S CHILDREN
Scholarship The grocer's children
C. Stanley Fish i. John Milton eat day-old bread,
D. Hugh Kenner ii. Ezra Pound moldy cakes and cheese,
Hence option (b) is correct. soft black bananas
94. Which of the following correctly list the two novels on stale shredded wheat,
figuring the writer as a public figure, as a celebrity weeviled rice, their plates
and as grist for the academic mill? heaped high with wilted
(a) Rabbit Redux and Rabbit, Run greens, bruised fruit,
(b) Rabbit is Rich and The Coup surprise treats
(c) Of the Farm and The centaur from unlabelled cans,
(d) Bech : A Book and Bech is Back tainted meat.
Ans : (d) 'Bech : A Book' is written by John Updike, The grocer's Children
published in 1965 and 'Bech is Back' is also written by John never go hungry.
Updike and published in 1982. Basic 'Bech' combines two 97. How does the poem achieve its effect?
classic titles - Bech : A Book and Bech is Back. The hero of (a) It lists a number of grocery items which do not
"Beck : A Book" is writer in trouble, as a celebrity and as have any tangible nutritive benefit
grist for the academic mill.
(b) It presents a series of inedible fare in the face of
Hence option (d) is correct.
the basic need to eat
95. Match the play with the subject matter of the play : (c) It strays away from the tongue-in-cheek beginning
A. The Doctor's Dilemma i. Flouting of stage to state the obvious
conventions (d) It posits the circumspect existence of a reasonable
B. You Never Can Tell ii. Satire on military plan to alleviate hunger
heroes
Ans : (b) The poem achieve its effect that it presents a series
C. Candida iii. Devaluation of social
of inedible fare in the face of the basic need to eat.
traditions
D. Arms and the Man iv. Mockery of 98. What is suggested by the word 'tainted' in line 11?
physicians' ignorance (a) Tinctured (b) Cooked
Choose the correct option from those given below: (c) Spoiled (d) Boiled
A B C D A B C D Ans : (c) The word 'tainted' suggest option (c) Spoiled. Taint
(a) ii iii iv i (b) iii i iv ii means - the effect of something bad or unpleasant that spoils
(c) i ii iii iv (d) iv iii i ii the quality of something.
Ans : (d) Option (d) is correctly matched the play with the 99. Whose point of view seems to have been stated in the
subject matter of the play. poem?
A The Doctor's iv. Mockery of (a) The gorcer's (b) The Children's
Dilemma physicians' ignorance (c) The narrator's (d) The poet's
B You Never Can iii. Devaluation of social Ans : (c) The narrator's point of view seems to have been
Tell traditions stated in the poem.
C Candida i. Flouting of stage 100. Which of the following words best describes the last
conventions sentence of the poem?
D Arms and the Man ii. Satire on military (a) Ironic (b) Paradoxical
heroes (c) Pathetic (d) Disdainful
Hence option (d) is correct.
Ans : (a) Last sentence of the poem–
96. Who among the following is one of the University The Grocer's children
Wits?
Never go hungry, is ironic.
(a) Thomas Hooker (b) Thomas Nashe
(c) Michael Drayton (d) William Harvey Hence option (a) is correct.

UGC NTA NET/JRF English Paper II, June 2019 298 YCT
UGC NTA NET/JRF Exam, December-2019
ENGLISH
Solved Paper-II
1. Which two concepts, developed by the French murdered Agamemnon. It details the revenge of
sociologist Pierre Bourdieu have become Agamemnon's daughter Electra and his son, Orestes.
increasingly influential in cultural studies? Eumenides : The third play, Eumenides opens at the
(1) Dissemination shrine of Apollo at Delphi, where Orestes has taken
(2) Gynesis sanctuary from the furies. At the command of the
(3) Cultural Capital Delphic oracle, Orestes journies to Athens to stand
(4) Habitus trial for his matricide.
Choose the correct option: 3. How often did Richard Steele's Tatler appear
(a) (1) and (3) (b) (2) and (3) every week and how many issues of Tatler in
(c) (3) and (4) (d) (2) and (4) total were published?
Ans: (c) Pierre Bourdieu, French sociologist who was (a) Two times a week: 171 issues
a public intellectual in the tradition of Emile Zola and (b) Once a week: 151 issues
Jean-Paul-Sartre. His concept of 'Habitus' and (c) Three times a week: 271 issues
'Cultural Capital' was influential in recent past (d) Three times a week: 261 issues
modernist Humanities and social science.
Ans: (c) The Tatler, a periodical launched in London
Cultural Capital : In the field of sociology, cultural
by the essayist Sir Richard Steele in April 1709,
capital comprises that capital formed the foundation
of social life and dictated one's position within the appearing three times weekly until January 1711; a
social order. total of 271 issues, published in London by John
Habitus : Habitus is one of Bourdieu's most Morphew and printed by John Nutt.
influential yet ambigious concept. It refers to the 4. Given below two statements Assertion (A) and
physical embodiment of cultural capital, to the deeply Reason (R):
ingrained habits, skills and dispositions that we Assertion (A) : Only actual research develops
possess due to our life experience. research skills.
2. Which combination in the following constitutes Reason (R) : Information is discrete, whereas
the trilogy Qresteia? knowledge consists of a network of connections.
(a) Agamemnon, The Persians, Eumenides In the light of the above two statements choose
(b) The Persians, The Suppliants, Agamemnon the correct options:
(c) Agamemnon, Choephoroe, Eumenides
(a) Both (A) and (R) are true and (R) is the correct
(d) Seven Against Thebes, Agamemnon, The
explanation of (A)
Suppliants
(b) Both (A) and (R) are true but (R) is not the
Ans: (c) 'Qresteia' a trilogy of tragic drama's by the
correct explanation of (A)
ancient Greek dramatist, Aeschylus. It is his last work
and the only complete trilogy of Greek dramas that (c) (A) is true, but (R) is false
has survived. (d) (A) is false, but (R) is true
The 'Qresteia' tells the story of the house of Atreus. Ans: (b) Both (A) and (R) are true but (R) is not the
Agamemnon, Choephoroe and Eumenides constitutes correct explanation of (A).
the trilogy Qresteia. 5. Following Plato, which two of the following
Agamemnon : The first play, Agamemnon, portrays statements about 'Phantasm' and 'Semblance'
the victorious return of that king from the Trojan war
are correct?
and his murder by his wife – Clytemnestra and her
(1) 'Phantasm' is an image, while 'Semblance' is
lover Aegisthus.
Choephoroe : The second play - Choephoroe takes the real object
its title from the Chorus of women servants who come (2) 'Phantasm' is the real object while 'Semblance'
to pour propitiatory offerings at the tomb of the is only a resemblance.
UGC NTA NET/JRF English Paper II, December 2019 299 YCT
(3) Phantasm unlike semblance has the same 8. The following is a list of key critical terms.
proportions as the object. Which is the right chronological order of their
(4) Semblance is 'unreal' but looks 'real' as formulation?
compared to phantasm. (a) Langue – the unconscious – difference – heresy
Choose the correct option : of paraphrase
(a) (2) and (3) (b) (3) and (4) (b) The unconscious – langue – heresy of
(c) (1) and (2) (d) (4) and (1) paraphrase – difference
Ans: (b) In the 'Republic' Socrates says that painting (c) Difference – langue – heresy of paraphrase –
(as well as tragedy) is an imitation of a phantasm or the unconscious
appearence as it appears rather than an imitation of (d) Langue – difference – the unconscious –
reality or truth and Plato is clearly drawing on the heresy of paraphrase
distinction made in the sophists between the image of
Ans: (b) The unconscious – langue – heresy of
likeness that resembles the original it imitates and the
paraphrase – difference is the right chronological
semblance (Phantasma) that only mimics its modal
order of the critical terms formulation.
without bearing any true resemblance. Plato's signs
are of two sorts, image and simulacara (phantasm and 9. Match the works with authors
sembalance) both are governed by mimesis. (A) Homi Bhabha (i) Saving the Text
6. Which of the following fictional characters is in (B) Geoffrey Hartman (ii) The Location of
the right Chronological order? Culture
(a) Uncle Toby–Man Friday–Stephen Dedalus – (C) Edward Said (iii) Desire in Language
Miss Havisham (D) Julia Kristeva (iv) Culture and
(b) Stephen Dedalus – Man Friday – Uncle Toby – Imperialism
Miss Havisham Choose the correct option :
(c) Man Friday – Uncle Toby – Miss Havisham – (a) (A)-(i), (B)-(ii), (C)-(iv), (D)-(iii)
Stephen Dedalus
(b) (A)-(ii), (B)-(i), (C)-(iv), (D)-(iii)
(d) Miss Havisham – Uncle Toby – Stephen
(c) (A)-(iv), (B)-(iii), (C)-(i), (D)-(ii)
Dedalus – Man Friday
(d) (A)-(iii), (B)-(ii), (C)-(i), (D)-(iv)
Ans: (c) Right chronological order of the fictional
characters is option (c). Ans: (b) Option (b) is correctly matched–
Man Friday : This character appears in Daniel (A) Homi Bhabha (ii) The Location of
Defoe's novel – Robinson Crusoe, published in 1719. Culture
Uncle Toby : This character appears in Laurence (B) Geoffrey Hartman (i) Saving the Text
Sterne's novel – The Life and Opinions of Tristram (C) Edward Said (iv) Culture and
Shandy, Gentleman published in 1759.
Imperialism
Miss Havisham : This character appears in Charles
(D) Julia Kristeva (iii) Desire in
Dicken's novel–Great Expectaction, published in 1861.
Language
Stephen Dedalus : This character appears in James
Joyce's novel – A Portrait of the Artist as a Young 10. In the following list, which two journals relate to
Man, published in 1916. the field of post-colonial literature?
7. Which one of the following has two heroes with (1) Kunapipi
the same name? (2) Interventions
(a) The Island of the Mighty (3) Daedalus
(b) The German Goddess (4) Clio
(c) Animal Farm Choose the correct option :
(d) Armadale (a) (1) and (3) (b) (2) and (3)
Ans: (d) 'Armadale' is a novel by Wilkie Collins, first (c) (3) and (4) (d) (1) and (2)
published in November 1864 to June 1866. The novel Ans: (d) Kunapipi : Journal of post colonial writing
has a convaluted plot about two distant cousins both and culture was a biannual arts magzine with special
name – Allan Armadale. The father of one had but not exclusive emphasis on the new literature
murdered the father of the others (the two fathers are written in English. Kunapipi and Interventions are two
also named Allan Armadale). journals relate to the field of post-colonial literature.

UGC NTA NET/JRF English Paper II, December 2019 300 YCT
11. Which of the following stylistic features Ans: (d) Robert Browning was an English poet and
characterise spoken discourse? playwright whose mastery of the dramatic monologue
(1) Greater use of explicit connectives made him one of the foremost Victorian poets.
(2) Greater dependence on non-verbal 'Asolando' is famous work of Robert Browning, which
connectives means – To disport in the open air, to amuse oneself at
(3) Greater syntactic embedding random.
(4) Greater use of fillers and repetitions 14. What is the Chronological order of the
Choose the correct option; appearance of the following periodicals?
(a) (1) and (2) (1) The Tatler
(b) (2) and (3) (2) The Spectator
(c) (3) and (4) (3) The Examiner
(d) (2) and (4) (4) The Reflector
Ans: (d) Spoken discourse is the ongoing, situated Choose the correct option :
interpretation of a speakers communicative intentions, (a) (2), (1), (4), (3)
of which the addressee's expected and actual reactions (b) (3), (2), (1), (4)
are an integral part. (c) (1), (2), (3), (4)
The context of spoken discourse the distinction (d) (4), (1), (2), (3)
between Text and Discourse. Ans: (c) The chronological order of the appearence of
Text –The connected sequence of verbal signs and the periodicals is following:
non verbal signals in terms of which discourse is co- The Tatler (1709) : The Tatler was a British literary
constructed by the participants in the act of and society journal begun by Richard Steele in –
communication. 1709.
Discourse–The hierarchically structed situated The Spectator (1711-1712) : The Spectator was a
sequence of indexical, prepositional, utterance and daily publication founded by Joseph Addison and
illocutionary acts carried out in pursuance of some Richard Steele in England lasting from (1711-1712).
communicative goal, as integrated within a given The Examiner (1808) : The Examiner was a weekly
context. paper founded by Leigh Hunt and John Hunt in –
The following stylistic features (a) Greater 1808.
dependence on non-verbal connectives (b) Greater use The Reflector (1884) : The Reflector is the student
of fillers and repetations are characterize of spoken newspaper of Mississippi State University, founded in
discourse. – 1884.
12. Of the five conditions of the Sublime, according 15. Which of the following is the proper explanation
to Longinus the most important condition is of the concept of "Freytag's Pyramid"?
(a) Vigorous treatment of passions (a) Analysis of the plot of a drama
(b) Majesty of the structure (b) Analysis of the characters of a drama
(c) A lofty cast of mind (c) Analysis of the theme of conflict between a
(d) A wide range of thoughts woman and two men in drama
Ans: (c) Cassius Longinus was a rhetorician and (d) Analysis of the different types of drama
philosophical critic. He composed a great number of Ans: (a) 'Freytag's Pyramid' is one of the oldest
works, on the 'Sublime' was written by him. He has dramatic structure, developed by Gustav Freytag. It is
given the five condition of the Sublime. Among them the analysis of the plot of a drama.
– A lofty cast of mind is the most important condition. Freytag derives his five part model from the conflict
13. Which one of the following titles of Robert of man against man, the hero and his adversary.
Browning's works means, 'to disport in the open
air, to amuse oneself at random'?
(a) Jacoseria
(b) Andrea del Sarto
(c) Abt Vogler
(d) Asolando
UGC NTA NET/JRF English Paper II, December 2019 301 YCT
16. Which two principal kinds of melancholy are Ans: (a) 'Langue and Parole' are linguistic terms
proposed by Robert Burton in Volume III of distinguished by 'Ferdinand de Saussure' in his
Anatomy of Melancholy? "Course in General Linguistic". Langue is the system.
(1) 'Love' A sign is a basic unit of language while Parole is the
(2) 'Religious' use of system to produce speech and also to produce
writing.
(3) 'Morbid'
Language involves the principle of language while
(4) 'Psychic' Parole refers to the concrete instances of the use of
The correct option is language. So, Langue is the language system and
(a) (1) and (2) Parole is the individual use.
(b) (1) and (3) 19. Which two of the following novels deal with the
(c) (2) and (4) theme of apartheid?
(d) (3) and (4) (1) Purple Hibiscus
Ans: (a) The 'Anatomy of Melancholy' was produce (2) July's People
by the English clergyman 'Robert Burton'. The work is (3) Cry, The Beloved Country
(4) The Mimic Men
divided into three sections. The first consider the
Choose the correct option:
nature symptoms and diverse causes of melancholy.
(a) (1) and (3) (b) (2) and (4)
The second section discusses cures such as exercise
(c) (2) and (3) (d) (1) and (4)
and diet, purging blood letting and potions. The third
focuses on two particular types: Love melancholy and Ans: (c) Novels, 'July's People' and 'Cry, The Beloved
Country' deal with the theme of apartheid. 'July
Religious melancholy.
People' is a 1981 novel by the South African writer
17. Which two of the following novels belong to the 'Nadine Gardiner'. It is set in a near future version of
Victorian Age in English Literature? South Africa, where Apartheid is ended through a
(1) Pendennis civil war. 'Cry, The Beloved Country' is a novel by
(2) The Way of All Flesh 'Allan Paton' published in 1948. This novel is a social
(3) The Battle of the Books protest against the structure of the society that would
later give rise to aparthied.
(4) Barchester Towers
Note–Apartheid is an Afrikaans word meaning
Choose the correct option
"separateness" or "The state of being apart"; literary –
(a) (1) and (3) "Apart hood".
(b) (2) and (4) 20. Given below are two statements Assertion (A)
(c) (3) and (4) and Reason (R):
(d) (1) and (4) Assertion (A) :The Primary component in
Ans: (d) 'Pendennis' and 'Barchester Towers' novels novelistic forms is a plot that evolves coherently
belongs to the Victorian Age in English literature. The from its beginning to an end in which all
History of Pendennis : His fortunes and Misfortune, complications are resolved.
His friends and His greatest Enemy is a novel by the Reason (R) : The novel is constituted by a
English author 'William Makepeace Thackeray'. multiplicity off divergent and contending social
voices that achieve their full significance only in
'Barchesters Towers' is the second novel in series
the process of their dialogic interaction both with
known as the "Chronicles of Barsetshire" published in
each other and with the voice of the narrator.
1857 written by Anthony Trollope.
In the light of the above two statements choose
18. Which of the following statements is correct? the correct option :
(a) Langue is the language system, and Parole, the (a) Both (A) and (R) are true and (R) is the correct
individual usage explanation of (A)
(b) Langue is the language usage, and Parole, the (b) Both (A) and (R) are true but (R) is not the
individual system correct explanation of (A)
(c) Langue is the language in abeyance, and (c) (A) is true, but (R) is false
Parole, the individual application (d) (A) is false, but (R) is true
(d) Langue is the language collective, and Parole, Ans: (b) Both (A) and (R) are true but (R) is not the
the individual deviation correct explanation of (A).

UGC NTA NET/JRF English Paper II, December 2019 302 YCT
21. Match the types of writing with their (a) Writers are likely to be mere entertainers who
descriptions : appeal to the emotions and passions of the
(A) exegesis (i) writing about audience
saints' lives (b) Texts created by poets are almost inevitably
inaccurate and defective as imitations
(B) invective (ii) detailed
(c) The best artistic texts will be both complex and
explanation of a
unified: every part of the work will be essential
passage
to it and will be linked to every other part
(C) hagiography (iii) a defence or (d) Texts should be judged on the basis of how
justification of accurately they imitate philosophical
one's actions and
Ans : (c) Aristotle's 'Critical Position' – "The best
beliefs artistic texts will be both complex and unified : Every
apology (iv) a bitterly critical part of the work will be essential to it and will be
(D) attack of something linked to every other part" is true.
Choose the correct option : 24. Which three of the following poets figure in
(a) (A)-(ii), (B)-(iv), (C)-(i), (D)-(iii) William Dunbar's Lament for the Makers?
(b) (A)-(iv), (B)-(ii), (C)-(iii), (D)-(i) (1) Geoffrey Chaucer
(2) John Gower
(c) (A)-(i), (B)-(iii), (C)-(iv), (D)-(ii)
(3) Robert Henryson
(d) (A)-(iii), (B)-(i), (C)-(ii), (D)-(iv)
(4) William Langland
Ans : (a) Option (a) is correctly matched the types of
Choose the most appropriate option:
writing with their descriptions–
(a) (1), (2) and (4) (b) (1), (2) and (3)
( Exegesi ( Detailed (c) (2), (3) and (4) (d) (1), (3) and (4)
A) s ii) explanation of a Ans: (b) William Dunbar was a Scottish maker poet
passage active in the late fifteenth and the early sixteenth
( Invectiv ( A bitterly critical century. He was closely associated with the court of
B) e iv) attack of something King James IV. In his poem 'Lament for the Makers'
( Hagiogr ( Writing about there is reference of Geoffrey Chaucer, John Gower,
C) aphy i) saint's lives Robert Henryson.
Source :
( Apology ( A defence or
He has done petuously devour
D) iii) justification of one's
The noble Chaucer, of makaris flour,
actions and beliefs
The Monk of Bery and Gower, all there
22. In which of the following works is the character
––––––––––––––––––––
'Ariel' an exclusion?
––––––––––––––––––––
(a) The Tempest
In Dumfermelyne he has done roune
(b) Paradise Lost
with Maister Robert Henrisoun;
(c) The Rape of the Lock – – – – – – – – – – – – – – – – –.
(d) The Rime of the Ancient Mariner
25. From whose work did John Milton take the
Ans: (d) The character 'Ariel' appears in 'The epigraph to his Areopagitica?
Tempest', 'Paradise Lost' and 'The Rape of the Lock' (a) Sophocles (b) Euripides
whether in 'The Rime of the Ancient Mariner' Ariel is (c) Plato (d) More
not listed. In 'The Tempest' Ariel helps Prospero Ans: (b) From Euripides's work Milton took the
reconcile with his enemies regain his throne and epigraph to his Areopagitica. Milton begins his
marry off his daughter. In John Milton's 'Paradise arguement by praising parliament's history of
Lost' Ariel is one of the fallen angels who accompany defending liberty. Areopagitica protests against and
Satan. In Alexander Pope's 'The Rape of the Lock' he calls for the repeal of the Licensing order of 1643,
is the comic protectors of the heroin Belinda. which required all books to be reviewed by the
23. Which of the following is true of Aristotle's official censor before being published.
Critical Position? 26. Which of the following movements was Arthur
Symons was referring to as 'an interesting

UGC NTA NET/JRF English Paper II, December 2019 303 YCT
disease' and 'an over-subtilizing refinement (1) Problems of Knowledge and Freedom
upon refinement'? (2) Aspects of the Theory of Syntax
(a) Celtic Revival (3) Syntactic Structures
(b) Romantic Movement (4) Knowledge of Language
(c) Decadence Choose the correct option :
(d) Feminism (a) (4), (3), (2), (1) (b) (2), (3), (4), (1)
Ans: (c) Decadent movement was the view that art is (c) (3), (2), (1), (4) (d) (1), (2), (3), (4)
totally opposed to nature in the sense both of Ans: (c) Avram Noam Chomsky was an American
biological nature and of the standard or natural norms theoretical linguist. Sometimes called "the father of
of morality and sexual behaviour. modern linguistic". The order of the publication of
In England the ideas, moods and behaviour of the books Noam Chomsky is as following– Syntactic
Decadence were manifested beginning in the 1860, in Structures (1957), Aspects of the Theory of Syntax
the poems of A.C. Swinburne and in 1890 by writers (1965), Problems of Knowledge and Freedom (1971)
such as Oscar Wild, Arthur Symons, Ernest Dowson. and Knowledge of Language (1986).
Arthur Symons referred Decadence as an interesting 30. Who among the following theorists believes that
disease and an over subtilizing refinement upon the proliferation of television images is
refinement. producing a cultural condition akin to 'historical
27. Which of the following tales in Chaucer's amnesia'?
Canterbury Tales deals with the murder of a (a) Jean Baudrillard (b) Ihab Hassan
child by Jews? (c) Frederic Jameson (d) Daniel Bell
(a) "The Monk's Tale" Ans: (c) Frederic Jameson is an American literary
(b) "The Second Nun's Tale" critic, philosopher and Marxist political theorist.
(c) "The Prioress's Tale" Frederic Jameson believes that the proliferation of
(d) "The Shipman's Tale" television images is producing a cultural condition
Ans: (c) The 'Canterbury Tales' is a collection of 24 akin to "historical amnesia".
stories, that was published in 1400. The framing 31. Who among the following are associated with
device for the collection of stories is a pilgrimage to the 'Jazz Age'?
the shrine of Thomas a Becket in Canterbury, Kent. (a) Ernest Hemingway and Scott Fitzgerald
The 30 pilgrimage who undertake the journey gather (b) Scott Fitzgerald and John Dos Passos
at the Tabard Inn in South work, across the Thomas (c) John Dos Passos and Sherwood Anderson
from London. In the Prioress's Tale describes how a (d) Ernest Hemingway and Sherwood Anderso
widow's devout young son is abducted by Jew.
Ans: (a) The 'Jazz Age' was a period in the 1920 and
28. Which two of the following statements are 1930, in which Jazz music and dance styles rapidly
applicable to feminist criticism? gained nationwide popularity in the United State.
(1) Recuperate the female writers ignored by the Ernest Hemingway and Scott Fitzgerald are associated
canon with Jazz Age.
(2) Fully endorse the social construction of gender 32. In which one of the following Middle English
(3) Valorize the traditional canon uncritically poems is Hector a character?
(4) Mostly reject the essentialising of 'male' and (a) Troilus and Cressida
'female' (b) Piers Plowman
Choose the correct option: (c) The Seafarer
(a) (1) and (2) (b) (2) and (3) (d) Beowulf
(c) (1) and (4) (d) (1) and (3) Ans: (a) Troilus and Cressida is an epic poem by
Ans: (c) Recuperate the females writers ignored by Geoffrey Chaucer which retells in middle English the
the canon and mostly rejected the essentialising of tragic story of the lovers Troilus and Cressida set
male and female statements are applicable to feminist against a backdrop of war during the Siege of Troy.
criticism. 'Hector' is a character in this poem.
29. What is the order of publication of the following
books of Noam Chomsky?
UGC NTA NET/JRF English Paper II, December 2019 304 YCT
33. In the UNESCO definition a 'Pamphlet' is an Ans: (b) 'Meatless Days' and 'Prison and Chocolate
unbound publication that is not a periodical and Cake' are autobiographical narratives. Meatless Days
contains: is a searing memoir of life in the newly created
(a) No fewer than 5 and no more than 48 pages country of Pakistan. Meatless Days is an
(b) No fewer than 10 and no more than 68 pages autobiography of Sara Suleri Goodyear. 'Prison and
(c) No fewer than 15 and no more than 64 pages Chocolate Cake' is an autobiography of Nayantara
(d) No fewer than 20 and no more than 80 pages Sehgal, an Indian writer and member of Nehru –
Ans: (a) UNESCO defines a pamphlet as 'a non Gandhi family.
periodical printed publication of at least 5 but not 37. Which two of the following works were
more than 48 pages, exclusive of the cover pages, published after 1947?
published in a particular country and made available (1) The Dark Room
to the public. (2) Mr. Sampath: A Painter of Malgudi
34. Given below are two statements – Assertion (A) (3) Seven Summers
and Reason (R) : (4) The Big Heart
Assertion (A) : Many modern British writers Choose the correct option:
infused their works with an extreme sense of (a) (1) and (2) (b) (2) and (3)
uncertainty, disillusionment and despair. (c) (3) and (4) (d) (1) and (4)
Reason (R) : The Waste Land ends in a flurry of
Ans: (b) 'Mr. Sampath: A Painter of Malgudi' and
random allusions.
'Seven Summers' are the following works were
In the light of the above two statements choose
published after 1947. 'Mr. Sampath: A Painter of
the correct option:
Malgudi' is novel by R.K. Narayan, published in
(a) Both (A) and (R) are true and (R) is the correct
1949. 'Seven Summers' is a novel by Mulkraj Anand,
explanation of (A)
published in 1951.
(b) Both (A) and (R) are true but (R) is not the
* Seven Summers first drafted when Mulkraj Anand
correct explanation of (A)
was a student at London University but not published
(c) (A) is true, but (R) is false
till 1951.
(d) (A) is false, but (R) is true
38. Who among the following has written a series of
Ans: (b) Both (A) and (R) are true but (R) is not
poems entitled,, 'Very Indian Poems in Indian
correct explanation of (A).
English'?
35. Which of the following is the correct (a) Vikram Seth
chronological order of publication of the (b) Arun Kolarkar
following poems?
(c) Nissim Ezekiel
(a) Lamia–Paradise Lost – Alastor – The Dunciad
(d) Keki N Daruwalla
(b) The Dunciad–Alastor – Lamia – Paradise Lost
Ans: (c) 'Very Indian Poems in Indian English' is
(c) Alastor–The Dunciad – Paradise Lost – Lamia
written by Nissim Ezekiel. The poem written in a very
(d) Paradise Lost–The Dunciad – Alastor – Lamia
light is an 'Indian Poem' because the poet looks at the
Ans: (d) The correct chronological order of world around him through the eyes of a typical middle
publication of the poems is as following–
class Indian.
Paradise Lost (1667) – The Dunciad (1728) – Alastor
39. Who made the remark: "Great literature is
(1816) – Lamia (1820).
simply language charged with meaning to the
36. Which two of the following are autobiographical utmost possible degree"?
narratives?
(a) Rabindranath Tagore
(1) Kanthapura
(b) Ezra Pound
(2) Meatless Days
(c) W.B. Yeats
(3) Prison and Chocolate Cake
(d) T.S. Eliot
(4) The God of Small Things
Ans: (b) 'Ezra Pound' made the remark – "Great
The correct option is:
literature is simply language charged with meaning to
(a) (1) and (2) (b) (2) and (3)
the utmost possible degree."
(c) (1) and (3) (d) (3) and (4)
UGC NTA NET/JRF English Paper II, December 2019 305 YCT
40. Which one of the following observations of 'Lost 43. Given below are two statements – Assertion (A)
Generation', a term coined by Gertude Stein is and Reason (R):
correct? Assertion (A) : Cultural Studies is simply the study
(a) German Jews who survived the Second World of culture as a discrete entity divorced from its
War and went to Israel social and political context.
(b) The American expatriates in Europe after the Reason (R) : Cultural Studies aim to understand
First World War Culture in all its complex forms and to analyse the
(c) The Irish Freedom fighters of the early Social and Political context within which it
Twentieth Century manifests itself.
(d) The Europeans living in America In the light of the above two statements choose
Ans: (b) Many prominent American writers of the the correct option:
decade following the end of World War I, (a) Both (A) and (R) are true and (R) is the correct
disillusioned by their war experiences and alienated explanation of (A)
by what they perceived as the crashness of American (b) Both (A) and (R) are true but (R) is not the
culture and its puritanical repression are often tagged correct explanation of (A)
as the 'Lost Generation'. (c) (A) is true but (R) is false
41. Which cultural analyst has combined the study (d) (A) is false but (R) is true
of different dimensions of youth culture with
Ans: (d) (A) is false but (R) is true.
commentary on developments in cultural theory
and politics? 44. Which term among the following will be
(a) Angela Mc Robbie applicable to a situation in which a character
initiates a scheme which depends for its success
(b) Donna Horraway
on the ignorance of the person against whom it
(c) Linda Hutcheon
is directed?
(d) Julia Kristeva
(a) Conflict
Ans: (a) Angela Mc Robbie is a British cultural
(b) Intrigue
theorist, feminist and commentator whose work
(c) Ally
combines the study of popular culture, contemporary
media practices and feminism through conceptions of (d) Foil
a third person reflexive gaze. She combined the study Ans: (b) The term 'Intrigue' is a situation in which a
of different dimensions of youth culture with character initiates a scheme which depends for its
commentary on development in cultural theory and success on the ignorance of the person against whom
politics. it is directed.
42. Which of the following plays by T.S. Eliot is in 45. Match the author with the story:
the correct chronological order of publication? (A) Edgar Allan Poe (i) "The Fall of
(a) Murder in the Cathedral – The Family Reunion the House of
– The Cocktail Party – The Confidential Clerk Usher"
(b) The Cocktail Party – The Confidential Clerk – (B) E.M. Forster (ii) "The
The Family Reunion – Murder in the Cathedral
Prophet's
(c) The Family Reunion – The Cocktail Party – Hair"
Murder in the Cathedral – The Confidential
(C) Katherine (iii) "The Garden
Clerk
Mansfield Party"
(d) The Confidential Clerk – Murder in the
Cathedral – The Cocktail Party – The Family (D) Salman Rushdie (iv) "The Celestial
Reunion Omnibus"
Ans : (a) Correct chronological order of publication Choose the correct option:
of T.S. Eliot's plays are as following– (a) (A)-(iii), (B)-(ii), (C)-(i), (D)-(iv)
Murder in the Cathedral (1935) – The Family Reunion (b) (A)-(iv), (B)-(iii), (C)-(i), (D)-(ii)
(1939) – The Cocktail Party (1949) – The (c) (A)-(i), (B)-(iv), (C)-(iii), (D)-(ii)
Confidential Clerk (1954). (d) (A)-(ii), (B)-(i), (C)-(iv), (D)-(iii)

UGC NTA NET/JRF English Paper II, December 2019 306 YCT
Ans: (c) Option (c) is correctly matched– Ans: (c) Richard Burbage and Will Kempe belongs to
(A) Edgar Allan Poe (i) The Fall of the House the Elizabethan period. They were actors in William
of Usher Shakespeare's plays.
(B) E.M. Forster (iv) The Celestial Omnibus 50. How many syllables are there in the word
intransigently?
(C) Katherine (iii) The Garden Party
(a) Three (b) Six
Mansfield
(c) Five (d) Four
(D) Salman Rushdie (ii) The Prophet's Hair
Ans: (c) There are five syllables in the word –
46. Which three of the following writers are Intransigently.
associated with 'kitchen sink drama'? 51. Which arrangement of D.H. Lawrence's novels
(A) Arnold Wesker is in the correct chronological sequence?
(B) John Arden (a) Kangaroo – The Plumed Serpent – Sons and
(C) Shelagh Delaney Lovers – The Rainbow
(D) John Osborne (b) Sons and Lovers – The Rainbow – Kangaroo –
Choose the most appropriate option: The Plumed Serpent
(a) (A), (B) and (D) (b) (A), (B) and (C) (c) The Rainbow – The Plumed Serpent –
(c) (B), (C) and (D) (d) (A), (C) and (D) Kangaroo – Sons and Lovers
Ans: (d) 'Kitchen sink drama' is a British cultural (d) The Rainbow – Kangaroo – The Plumed
movement that develop in the late 1950 and early Serpent – Sons and Lovers
1960, in theatre art novels, films and television play. Ans: (b) Correct arrangement of D.H. Lawrence's
Arnold Wesker, Shelagh Delaney and John Osborne novels are as following–
are associated with 'Kitchen sink drama'. Sons and Lovers – The Rainbow – Kangaroo – The
Plumed Serpent.
47. Which two of the following correctly describe
the features of Wuthering Heights? 52. Which two names from R.M. Ballantyne's Coral
(A) Flash backs and time shifts Island are repeated in William Golding's
reworking of the same text as Lord of the Flies?
(B) Oedipal obsessions
(A) Ralph (B) Roger
(C) Magic and ritual
(C) Jack (D) Simon
(D) Acute evocation of place
The correct option is:
Choose the correct option:
(a) (A) and (D) (b) (A) and (C)
(a) (A) and (C) (b) (B) and (D)
(c) (C) and (D) (d) (B) and (D)
(c) (A) and (D) (d) (C) and (D)
Ans: (b) The 'Coral Island : A Tale of the Pacific
Ans: (c) 'Wuthering Heights' is a novel by 'Emily Ocean' is a novel written by Scottish author R.M.
Bronte' published in 1847 under his pseudonym Ellis Ballantyne. It was the inspiration for William
Bell. Flash backs and time shifts, Acute evocation of Golding's dystopian novel 'Lord of the Flies' (1854).
place describes the features of Wuthering Heights. Character Ralph and Jack appears in this novel.
48. In which of the following essays did Charles 53. Which of the following set of characters in
Lamb first use the pseudonym/persona Elia? Charles Dickens' novels is in the right
(a) "My First Play" chronological order?
(b) "The Two Races of Men" (a) Mr. Bounderby – David Copperfield – Mrs.
(c) "New Year's Eve" Mann – Nathaniel Winkle
(d) "The South Sea House" (b) David Copperfield – Mr. Bounderby –
Ans: (a) Charles Lamb first use the pseudonyms 'Elia' Nathaniel Winkle – Mrs. Mann
in his essay "My First Play". (c) Nathaniel Winkle – Mrs. Mann – David
Copperfield – Mr. Bounderby
49. In the following list, which two actors belong to
the Elizabethan period? (d) Mrs. Mann – David Copperfield – Nathaniel
Winkle – Mr. Bounderby
(A) Richard Burbage (B) Will Kempe
(C) David Garrick (D) John Kemble Ans: (c) Following set of characters in Charles
Dickens' novels is in the right chronological order–
Choose the correct option:
Nathaniel Winkle – Mrs. Mann – David Copperfield –
(a) (A) and (C) (b) (B) and (D)
Mr. Bounderby.
(c) (A) and (B) (d) (C) and (D)
UGC NTA NET/JRF English Paper II, December 2019 307 YCT
54. Who said the following? "Discursive practices 57. Match the author with the text:
are not purely and simply modes of (A) Rita Kothari (i) The Queen's
manufacture of discourse. They take shape in Hinglish
technical ensembles, in institutions, in (B) Probal Dasgupta (ii) The Indianization
behavioural schemes, in types of transmission of English
and dissemination, in pedagogical forms that (C) Braj B. Kachru (iii) Translating India
both impose and maintain them"
(D) Baljinder K. Mahal (iv) The Otherness of
(a) Roland Barthes
English
(b) Michel Foucault
(a) (A)-(iv), (B)-(iii), (C)-(i), (D)-(ii)
(c) Homi K. Bhabha
(b) (A)-(iii), (B)-(iv), (C)-(ii), (D)-(i)
(d) Gayatri Chakravorty Spivak
(c) (A)-(iv), (B)-(ii), (C)-(i), (D)-(iii)
Ans: (b) Michel Foucault said the statement (d) (A)-(iii), (B)-(i), (C)-(ii), (D)-(iv)
"Discursive practices are not purely and simply modes
Ans: (b) Correctly matched author with the text is as
of manufacture of discourse. They take shape in
following–
technical ensembles, in institutions, in behavioural
(A) Rita Kothari – (iii) Translating India
schemes, in types of transmission and dissemination,
in pedagogical forms that both impose and maintain (B) Probal Dasgupta – (iv) The Otherness of
them." English
55. Match the characters with the play: (C) Braj B Kachrru – (ii) The Indianization of
(A) Donalbain (i) King Lear English
(B) Claudio (ii) Macbeth (D) Baljinder K. Mahal – (i) The Queen's
(C) Neerissa (iii) Merchant of Venice Hinglish
(D) Goneril (iv) Measure for 58. Which of the following characters in
Measure Shakespeare's Love's Labour Lost over uses
formal Latinate diction?
Choose the correct option :
(a) Holofernes (b) Dull
(a) (A)-(iv), (B)-(iii), (C)-(ii), (D)-(i)
(c) Costard (d) Moth
(b) (A)-(ii), (B)-(iv), (C)-(iii), (D)-(i)
(c) (A)-(iii), (B)-(i), (C)-(ii), (D)-(iv) Ans: (a) Holofernes a school master in Shakespeare's
(d) (A)-(i), (B)-(iv), (C)-(ii), (D)-(iii) 'Love's Labour Lost' over uses formal Latinate diction.
'Love's Labour Lost' is one of William Shakespeare's
Ans: (b) William Shakespeare's following plays are
early comedy.
correctly matched with character–
59. Match the periodicals with their
(A) Donalbain – (ii) Macbeth
writers/contributors
(B) Claudio – (iv) Measure for Measure
(A) The Rambler (i) Charles Dickens
(C) Nerissa – (iii) Merchant of Venice
(B) Macmillan's (ii) Samuel Johnson
(D) Goneril – (i) King Lear
Magazine
56. Who among the following proposed that the
(C) The Guardian (iii) David Masson
English language is 'man made', not 'woman
(D) Bentley's Miscellany (iv) Richard Steele
made'?
(a) (A)-(iii), (B)-(iv), (C)-(i), (D)-(ii)
(a) Mary Haas (b) Dorothy L. Sayers
(b) (A)-(i), (B)-(ii), (C)-(iii), (D)-(iv)
(c) Dale Spender (d) Carol Chomsky
(c) (A)-(ii), (B)-(iii), (C)-(iv), (D)-(i)
Ans: (c) 'Man Made Language' 1980 is a book by
(d) (A)-(iv), (B)-(i), (C)-(ii), (D)-(iii)
Australian feminist writer Dale Spender. She
proposed that the English language is 'man made' not Ans: (c) Correctly matched the periodicals with their
'woman made'. In it she examines numerous areas of writers/contributors is as following–
sexism as it appears in nature and in the use of the (A) The Rambler – (ii) Samuel Johnson
English language with particular focus on the way (B) Macmillan's Magazine – (iii) David Masson
men and women talk and listen differently in couples (C) The Guardian – (iv) Richard Steele
and in mixed or single sex group. (D) Bentley's Miscellany – (i) Charles Dickens

UGC NTA NET/JRF English Paper II, December 2019 308 YCT
60. Which of the following books carried the Ans: (b) 'Deconstruction' as applied in the criticism of
additional title Sermon on the Sea? literature, designates a theory and practice of reading
(a) The Religion of Man by Tagore that questions and claims to "Subvert" or
(b) Essay on the Gita by Aurobindo "Undermine" the assumption that the system of
(c) Hind Swaraj or Indian Home Rule by Gandhi language is based on grounds that are adequate to
establish the boundaries, the coherence or unity and
(d) Christ and Satyagraha by Elwin
the determinate meaning of literary text. Jacques
Ans : (c) Hind Swaraj or Indian Home Rule is a book Derrida and Paul de Man are associated with
written by Mohan Das K. Gandhi in 1909. The book deconstruction.
was banned in 1910, by British Government in India 64. What is the correct chronological order of the
as a seditious text. This book carried the additional publication of the following?
title 'Sermon on the Sea'. (A) German Grammar (Jacob Grimm)
61. Which two of the following poems can be (B) Comparative Grammar of Sanskrit, Zend,
categorized as poems belonging to the neo- Greek Latin, Lithuanian, Gothic and German
classical period of English literature. (Franz Bopp)
(A) "The Ring and the Book' (C) An Investigation into the Origin of Old Norse
(B) "The Vanity of Human Wishes" or Icelandic Language (Rasmus Rask)
(C) "Cato" (D) Concerning the Conjugation System of the
Sanskrit Language in Comparison with those
(D) "Lamia"
of the Greek, Latin, Persian and German
Choose the correct option: Languages (Franz Bopp)
(a) (A) and (B) (b) (B) and (C) (a) (A), (B), (C), (D) (b) (B), (C), (D), (A)
(c) (C) and (D) (d) (A) and (D) (c) (C), (D), (A), (B) (d) (D), (C), (B), (A)
Ans: (b) "The Vanity of Human Wishes" and "Cato" Ans: (c) The correct chronological order of the
are the following poems can be categorized as poems publication of the book is as following–
belonging to the neo-classical period of English (C) An Investigation into the Origin of Old Norse or
literature. Cato a tragedy is a play written by Joseph Icelandic Language – Rasmus Rask.
Addison in 1712 and first performed on 14 April, (D) Concerning the Conjugation System of the
1713. The Vanity of Human Wishes : The Tenth Sanskrit Language in Comparison with those of the
Satire of Juvenal imitated is a poem by the English Greek, Latin, Persian and German Languages – Franz
author Samuel Johnson. Bopp.
62. The key figures in the development of British (A) German Grammar – Jacob Grimm.
cultural studies are (B) Comparative Grammar of Sanskrit, Zend, Greek
Latin, Lithuanian, Gothic and German – Franz Bopp.
(A) Richard Hoggart
(B) Raymond Williams 65. Match the works with authors:
(A) Bodies that Matter (i) Camille Paglia
(C) Stuart Hall
(B) A World of (ii) Elaine Showalter
(D) Lawrence Grossberg
Difference
The most appropriate option is:
(C) A Literature of their (iii) Barbara Johnson
(a) (A) and (B) (b) (B) and (C)
Own
(c) (A), (B) and (C) (d) (B), (C) and (D) (D) Vamps and Tramps (iv) Judith Butler
Ans: (c) The key figures in the development of British Choose the correct option:
Cultural studies are – Richard Hoggart, Raymond (a) (A)-(i), (B)-(ii), (C)-(iii), (D)-(iv)
Williams and Lawrence Grossberg. (b) (A)-(ii), (B)-(iii), (C)-(iv), (D)-(i)
63. Which two of the following are associated with (c) (A)-(iv), (B)-(iii), (C)-(ii), (D)-(i)
Deconstruction? (d) (A)-(iii), (B)-(iv), (C)-(i), (D)-(ii)
(A) Jacques Derrida Ans: (c) Correctly matched the works with authors is
(B) Raymond Williams as following–
(C) Paul de Man (A) Bodies that matter – (iv) Judith Butler
(D) Jonathan Dolli more (B) A World of Difference – (iii) Barbara Johnson
Choose the correct option : (C) A Literature of their – (ii) Elaine Showalter
(a) (A) and (B) (b) (A) and (C) Own
(c) (A) and (D) (d) (B) and (D) (D) Vamps and Tramps – (i) Camille Paglia

UGC NTA NET/JRF English Paper II, December 2019 309 YCT
66. How many tales and pilgrims are there in 69. Match the following technological advancements
Chaucer's The Caunterbury Tales? impacting learning and teaching of language
(a) 24 pilgrims and 23 tales with their corresponding years:
(b) 23 pilgrims and 24 tales (A) Hypertext Markup (i) 2004
(c) 22 pilgrims and 24 tales Language (HTML)
(d) 24 pilgrims and 22 tales
(B) Streaming of Video on (ii) 2003
Ans: (*) Question remains cancelled and equal marks the Internet
awarded to all candidates.
'The Caunterbury Tales' is a collection of 24 stories (C) My Space. com (iii) 1991
that runs over 17000 lines written in middle English (D) Facebook (iv) 1997
by Geoffrey Chaucer. Choose the correct option:
67. Match the theorist with the text: (a) (A)-(iii), (B)-(iv), (C)-(ii), (D)-(i)
(A) John Fiske (i) Distinction (b) (A)-(ii), (B)-(iv), (C)-(i), (D)-(iii)
(B) Michel de Certeau (ii) The Postmodern (c) (A)-(iii), (B)-(ii), (C)-(iv), (D)-(i)
Condition
(d) (A)-(iv), (B)-(iii), (C)-(i), (D)-(ii)
(C) Pierre Bourdieu (iii) Reading the
Ans: (a) Correctly matched the technological
Popular
(D) Jean Francois (iv) The Practice of advancements impacting learning and teaching of
Lyotard Everyday Life language with their corresponding years is as
following–
Choose the correct option :
(a) (A)-(iii), (B)-(iv), (C)-(i), (D)-(ii) Hypertext Markup Language (HTML) – 1991
(b) (A)-(iv), (B)-(iii), (C)-(ii), (D)-(i) Streaming of Video on the Internet – 1997
(c) (A)-(ii), (B)-(i), (C)-(iv), (D)-(iii) My Space. com – 2003
(d) (A)-(ii), (B)-(iv), (C)-(iii), (D)-(i) Facebook – 2004
Ans: (a) Correctly matched the theorist with the text 70. Match the poet with the opening line of the
is as following– poem
(A) John Fiske (iii) Reading the Popular (A) Shelley (i) I cry your mercy –
(B) Michel de Certeau (iv) The Practice of pity love! aye, love!
Everyday Life (B) Coleridge (ii) The world is too
(C) Pierre Bourdieu (i) Distinction much with us
(D) Jean Francois (ii) The Postmodern (C) Keats (iii) O world, O life, O
Lyotard Condition time
68. Which of the following describes Foucault's
(D) Wordsworth (iv) When true love
views on knowledge?
burns desire is
(A) Knowledge is not metaphysical or
Love's pure flame
transcendental
(B) Knowledge is not a matter of perspective Choose the correct option:
(C) Knowledge is not pure or neutral but is always (a) (A)-(iii), (B)-(iv), (C)-(i), (D)-(ii)
from a point of view (b) (A)-(iv), (B)-(iii), (C)-(ii), (D)-(i)
(D) Knowledge is unconstrained by regimes of (c) (A)-(ii), (B)-(iii), (C)-(iv), (D)-(i)
power (d) (A)-(i), (B)-(ii), (C)-(iii), (D)-(iv)
Choose the correct option:
Ans: (a) Correctly matched poet with the opening line
(a) (A) and (D) (b) (B) and (C)
of the poem is as following–
(c) (A) and (C) (d) (B) and (D)
Ans: (c) Michel Foucault was a French philosopher (A) Shelley (iii) O world, O life, O time
and historian, one of the most influential and (B) Coleridge (iv) When true love burns desire
controversial scholar of the past World War II period. is Love's pure flame
Foucault describes his view on knowledge as (C) Keats (i) I cry your mercy – pity love!
following–
aye, love!
(A) Knowledge is not metaphysical or transcendental.
(D) Wordsworth (ii) The world is too much with
(C) Knowledge is not pure or neutral but is always
from a point of view. us

UGC NTA NET/JRF English Paper II, December 2019 310 YCT
71. In Paradise Lost Milton invokes his 'Heavenly 75. Which two aspects of cultural diffusion in the
Muse', 'Urania', at the beginning of which two Age of Globalization need to be addressed by
books? pedagogy of language in general and of English
(A) Book I (B) Book IV in particular?
(C) Book IX (D) Book VII (A) Uni directionality
Choose the correct option: (B) Multidirectionality
(C) Complex and extensive
(a) (A) and (D) (b) (B) and (C)
(D) Simplistic and abbreviated
(c) (C) and (D) (d) (B) and (D)
Choose the correct option:
Ans: (a) 'Paradise Lost' is an epic poem in blank verse
(a) (A) and (B) (b) (B) and (C)
by the 17th century English poet John Milton. In this
(c) (C) and (D) (d) (D) and (A)
epic Milton invokes 'Heavenly Muse', 'Urania' at the
Ans: (b) Multidirectionality complex and extensive
beginning of Book (I) and Book (VII).
are the aspects of cultural diffusion in the age of
72. Which among the following group of writers is Globalization need to be addressed by pedagogy of
labelled as 'University Wits'? language in general and of English in particular.
(a) Thomas Lodge, Thomas Wilson, Walter 76. Examples of poetic compounding are found in
Raleigh the work of which two modernist writers?
(b) John Fletcher, Ben Jonson, George Peele (A) Graham Greene
(c) Thomas Kyd, Francis Beaumont, John Lyly (B) James Joyce
(d) Christopher Marlowe, Robert Greene, Thomas (C) Gerard Manley Hopkins
Nashe (D) Stephen Spender
Ans: (d) Christopher Marlowe, Robert Greene, Choose the correct option:
Thomas Nashe are labelled as 'University Wits'. (a) (C) and (D) (b) (A) and (B)
73. Which of the following work by Henry Fielding (c) (B) and (C) (d) (A) and (C)
begins as a parody of Samuel Richardson's Ans: (c) Examples of poetic compounding are found
Pamela? in the work of James Joyce and Gerard Manley
(a) Tom Jones (b) Don Quixote Hopkins. James Joyce and Gerard Manley Hopkins
(c) Amelia (d) Joseph Andrews are modernist writer.
77. Who among the following prose writers of the
Ans: (d) Joseph Andrews, in full – The History of the
Romantic period authored "On Murder
Adventures of Joseph Andrews and of his friend Mr.
Considered as one of the Fine Arts"?
Abraham Adams, novel written by Henry Fielding,
(a) Charles Lamb
published in 1742. It was written as a reaction against
(b) Walter Savage Lander
Samuel Richardson's novel 'Pamela' as Virtue
(c) Thomas De Quincey
Rewarded (1740). Joseph Andrews begins as a parody
(d) Anne Radcliffe
of 'Pamela'.
Ans: (c) "On Murder Considered as one of the Fine
74. Which two of the following plays were written
Arts" is an essay by Thomas Dee Quincey first
by Thomas Heywood? published in 1827 in Blackwood's Magazine. In this
(A) Gorboduc provocative and blackly funny essay, Thomas De
(B) The Play Called the Four P.P. Quincey considers murder in a purely aesthetic light
(C) The Play of the Weather and explains how practically every philosopher over
(D) The Spanish Tragedy the past two hundred years has been murdered
Choose the correct option 'insomuch' that if a man calls himself a philosopher
(a) (A) and (B) (b) (A) and (C) and never had his life attempted rest assured there is
nothing in him.
(c) (B) and (C) (d) (C) and (D)
78. Which of the following combinations best
Ans: (c) Thomas Heywood was an English writer,
describes the typical methodology of literary
known for his plays, poems and collection of research?
proverbs. 'The play called the Four PP' and 'The play (a) Direct, empirical and quantitative
of the Weather' is written by Thomas Heywood. 'The (b) Phenomenological, speculative and abstract
Play of the Weather' is an English interlude or
(c) Textual, critical and historical
morality play from early Tudor period.
(d) Synoptic, conceptual and speculative

UGC NTA NET/JRF English Paper II, December 2019 311 YCT
Ans: (c) Textual, critical and historical combination 83. Given below are two statements – Assertion (A)
best describe the typical methodology of literary and Reason (R)
research. Assertion (A) : Dialects are the broad range of
79. Who are the co-editors of Chutneyfying English: social as well as regional varieties.
The Phenomenon of Hinglish? Reason (R) : A dialect describes variations not
(A) Jamuna Kachru only at the phonological level, but also at the levels
(B) Rita Kothari of lexis and syntax.
(C) Rupert Snell In the light of the above two statements choose
(D) Alastair Pennycook the correct option:
Choose the correct option: (a) Both (A) and (R) are true and (R) is the correct
explanation of (A).
(a) (A) and (D) (b) (A) and (B)
(c) (B) and (C) (d) (B) and (D) (b) Both (A) and (R) are true but (R) is not the
correct explanation of (A).
Ans: (c) Rita Kothari and Rupert Snell are the co-
(c) (A) is true, but (R) is false
editors of Chuntneyfying English : The Phenomenon
(d) (A) is false, but (R) is true
of Hinglish.
Ans: (b) Both (A) and (R) are true but (R) is not the
80. Which British administrator passed a resolution
correct explanation of (A).
for the 'Promotion of European literatures and
Science among the natives of India"? 84. Which of the following arrangements of prose-
(a) Lord Hastings (b) Lord Cornwallis pamphlets is in the right chronological order?
(c) Lord Bentick (d) Lord Hardinge (a) The Shortest Way with Dissenters – A Modest
Proposal – Areopagitica – Reflections on the
Ans: (c) Lord Bentick was the British administrator
Revolution in France
who passed a resolution for the 'Promotion of
European literature and science among the native of (b) A Modest Proposal – The Shortest Way with
Indian. Dissenters – Areopagitica – Reflections on the
Revolution in France
81. Which of the following plays by Ben Jonson
(c) Areopagitica – The Shortest Way with
ends with the performance of a puppet play in
Dissenters – A Modest Proposal – Reflections
imitation of Marlowe's Hero and Leander?
on the Revolution in France
(a) The Alchemist
(d) Areopagitica – Reflection on the Revolution in
(b) Volpone
France – The Shortest Way with Dissenters –
(c) Bartholomew Fair
A Modest Proposal
(d) Every Man in His Humour
Ans: (c) Correct arrangements of prose-pamphlets is
Ans: (c) 'Bartholomew Fair' is a Jacobean comedy in in the right chronological order as follows:
five acts by Ben Jonson. The play 'Bartholomew Fair
Areopagitica – The Shortest Way with Dissenters – A
ends with the performance of a puppet play in
Modest Proposal – Reflections on the Revolution in
imitation of Marlow's – 'Hero and Leander'.
France.
82. Which of the following periods of English
85. Which one of the following novels by Kingsley
Literature is also called 'Puritan Interregnum'?
Amis represents its protagonist as an 'angry
(a) The Neoclassical Period
young man'?
(b) The Caroline Age
(a) I Like it Here
(c) The Restoration
(b) Lucky Jim
(d) The Commonwealth Period
(c) The Biographer's Moustache
Ans: (d) The Interregnum was the period between the
(d) The Great Man
execution of Charlse I on 30 January 1649 and the
arrival of his son Charlse II in London on 29 May Ans: (b) Lucky Jim, best – selling novel by Kingsley
1660 which marked the start of the Restoration. Amis, published in 1954. It was Amis first novel and
During the Interregnum, England was under various won the 1955 Somerset Maugham Award for fiction.
forms of republican government. The period between The novel features the anti hero Jim Dixon, a junior
1649-1660 is known as – Commonwealth Period or faculty member at a provincial university who
Puritan Interregnum in the history of English despises the pretensions of academic life. Amis
literature. represents Jim Dixon as a "angry young man".

UGC NTA NET/JRF English Paper II, December 2019 312 YCT
86. Which of the following is a collaborative work of 90. Which one of W.M. Thackeray's novels has the
W.H. Auden and Christopher Isherwood? following as the closing sentence? "Which of us
(a) Letters from Iceland is happy in this world? Which of us has his
(b) The Dance of Death desire? or having it, is satisfied"?
(c) The Ascent of F6 (a) The Luck of Barry Lyndon
(d) The Orators (b) Pendennis
Ans: (c) The Ascent of F6, a poetic drama by W.H. (c) Vanity Fair
Auden and Christopher Isherwood, published in – (d) The History of Henry Esmond
1936 and performed in 1937. The play is a tragedy in
Two Acts. Ans: (c) 'Vanity Fair' is an English novel by William
Makepeace Thackeray, which follows the life of
87. Which two aspects of cultural diffusion in the
Becky Sharp and Amelia Sedley amid their friends
Age of Globalization need to be addressed by
pedagogy of language in general and of English and families during and after the Napoleonic Wars.
in particular? Closing sentence of the novel is–
(A) Uni directionality 'Ah! Vanitas Vanitatum! which of us is happy in this
(B) Multidirectionality world? Which of us has his desire? or having it is
(C) Complex and extensive satisfied? Come, Children, let us shut up the box and
(D) Simplistic and abbreviated the puppets, for our play is played out.
Choose the correct option: Direction : Read the following poem and answer
(a) (A) and (B) (b) (B) and (C) the questions :
(c) (C) and (D) (d) (D) and (A) HOME IS SO SAD
Ans: (*) Question remains cancelled and equal marks Home is so sad. It stays as it was left, Shaped to the
awarded to all candidates. comfort of the last to go As if to win them back. Instead,
88. What does Socrates mean when in Plato's Ion, bereft of anyone to please, it withers so, Having no heart
he says "Poets are nothing but the interpreters to put aside the theft.
of gods"? And turn again to what is started as, A joyous shot at
(a) The poets are the markers of their poems. how things ought to be. Long fallen wide. You can see
(b) The poets are acutely aware of gods in how it was : Look at the pictures and the cutlery. The
composing their poems. music in the piano stool. That Vase.
(c) The poets are divinely possessed when they
91. How was the home before it became 'sad'?
compose their poems.
(a) It was as it would be
(d) The poets first hear what gods say then put that
into words. (b) It was as it shall be
Ans: (c) According to Socrates "Poets are nothing but (c) It was as it should be
the interpreters of gods" means – The poet are (d) It was as it could be
divinely possessed when they compose their poems. Ans: (c) It was as it should be.
89. Which of the following descriptions delineate 92. Why is the 'home' 'Sad'?
Roman a Clef (Novel with key)? (a) Because it has waited in vain
(a) A novel depicting the life of an artist from
(b) Because it remains unchanged
childhood to maturity
(c) Because its joy has faded
(b) A novel using the altered names of the actual
people of the time (d) Because it is devoid of residents
(c) A novel describing historical incidents with Ans : (a) Home is sad because it has waited in vain.
fictional characters 93. There is a 'thief' in the poem. Who is that thief?
(d) A novel giving the effect of realism by (a) The time that ticks away
highlighting the social problems of the time (b) The fate that overpowers
Ans: (b) 'Roman a Clef', French for novel with a key (c) The tenant who leaves
is a novel about real life events that is overlaid with a
(d) The past that beckons
façade of fiction. This novel delineate – A novel using
the altered names of the actual people of the time. Ans : (c) In the poem 'Thief' is the tenant who leaves.

UGC NTA NET/JRF English Paper II, December 2019 313 YCT
94. Why has the 'home' 'withered'? 98. How does 'lying' affect human relationships?
(a) Because everything has to fade sooner or later (a) It makes the relationships 'human'
(b) Because it has no longer the reason to be what (b) It reduces the affinity among people
it was (c) It promotes togetherness among diverse people
(c) Because it is an organic entity in any case (d) It does not affect at all as it is merely words
(d) Because it has been betrayed categorically Ans: (b) 'Lying' affect human relationships as it
Ans: (b) Home has withered because it has no longer reduces the affinity among people.
the reason to be what it was. Direction: Read the following passage and
Direction: Read the following passage and answer the question
answer the questions: It was the best of times, it was the worst of times, it
Lying is an accursed vice. It is only our words was the age of wisdom, it was the age of foolishness, it
which bind us together and make us human. If we was the epoch of belief, it was the epoch of incredulity, it
realized the horror and weight of lying, we would see was the season of light, it was the season of Darkness, it
that it is more worthy of the stake than other crimes. I was the spring of hope, it was the winter of despair, we
find that people normally waste time quite had everything before us, we had nothing before us, we
inappropriately punishing children for innocent were all going direct to Heaven, we were all going direct
misdemeanours formenting them for thoughtless actions the other way – in short, the period was far like the
which lead nowhere and leave no trace. It seems to me present period, that some of its noisiest authorities
that the only faults which we should vigorously attack as insisted on its being received, for good or for evil, in the
soon as they arise and start to develop are lying and, a superlative degree of comparison only.
little below that, stubbornness. Those faults grow up with
99. The Age described in the above passage is best
the children. Once left the tongue acquire the habit of
described as the Age of
lying and it is astonishing how impossible it is to make it
(a) Parallelisms (b) Inconsistencies
give it up. That is why some otherwise decent men are
object slaven to it. One of my tailors is a good enough (c) Contraries (d) Anomalies
fellow, but I have never heard him once speak the truth, Ans: (c) The Age described in the above passage is
not even when it would help him, if he did so. best described as the age of contraries. These lines are
95. It is suggested in the passage that the tailor does extracted from the opening line of Charles Dickens
never speak the truth because novel – 'A Tale of Two Cities'.
(a) He cannot keep the word he gives Direction: Read the following passage from
(b) He does not know lying is a crime Antigone and answer question
(c) He thinks lying will help him Creon : And yet wert bold enough to break the law.
(d) He is a slave of his profession Antigone : Yea, for these laws were not ordained by
Ans: (a) The tailor does never speak the truth because Zeus.
he cannot keep the word he gives. And she who sits enthroned with gods
96. 'Lying' is a fault that should be punished only below.
(a) When the first lie is uttered Justice, enacted not these human laws.
(b) When it becomes convenient Nor did I deem that thou, a mortal man,
(c) When it becomes stubborn Could'st by a breath annual and override.
(d) When it begins to turn into a habit The immutable unwritten laws of Heaven.
Ans: (d) 'Lying' is a fault that should be punished 100. The three kinds of laws implicit in Antigone's
only when it begins to turn into a habit. response are:
(a) Human, unwritten, written
97. According to the author "thoughtless actions"
(a) Torment others (b) Of Gods, of Zeus, of Justice
(b) Are strictly not misdemeanours (c) Of Gods, of Justice, of Man
(c) Mean nothing and are soon forgotten (d) of Man, of Heaven, of Zeus
(d) Are punishments for children Ans : (c) The three kinds of laws implicit in
Ans: (c) According to the author "thoughtless actions" Antigone's responses are – Of Gods, of Justice, of
mean nothing and are soon forgotten. Man.

UGC NTA NET/JRF English Paper II, December 2019 314 YCT
UGC NTA NET/JRF Exam, June-2020
ENGLISH
Solved Paper-II (Shift-II)
1. Match List I with List II Ans: (d) The Education Commission, 1964-66 or The
List I List II Kothari Commission, was appointed by the
(A) John Donne (i) "The Retreat” Government of India to overhaul the Indian education
(B) Andrew Marvell (ii) "A Valediction of sector. Main feature of this commission was it tried to
Weeping" cover every field and aspect of the entire educational.
(C) George Herbert (iii) "The Garden" This commission made a distinction between the
(D) Henry Vaugham (iv) "The Collar" teaching of English as a skill and the teaching of
Choose the correct answer from the options English literature.
given below : 5. A deconstructive reading of a text shows that
(a) (A)-(iv), (B)-(iii), (C)-(ii), (D)-(i) (A) a text is to be read always in a context
(b) (A)-(iv), (B)-(i), (C)-(ii), (D)-(iii) (B) there is nothing except the text
(c) (A)-(ii), (B)-(iv), (C)-(i), (D)-(iii) (C) a text may betray itself
(d) (A)-(ii), (B)-(iii), (C)-(iv), (D)-(i) (D) a text may possess an ascertainable meaning
Ans : (d) 'A Valediction of Weeping" is a dramatic (E) there is an endless postponement of meaning
monologue from 'love poems' of John Donne. In Choose the correct answer from the options
which the poet, after careful analysis, talks about the given below:
love between him and his beloved. 'The Garden' is (a) (A), (B) and (C) only
poem by Andrew Marvell. 'The Retreat' by Henry (b) (C), (D) and (E) only
Vaughan describes a speaker's desire to escape from (c) (B), (C) and (E) only
the past where he was a younger, purer and generally (d) (B), (C) and (D) only
happier human being. 'The Collar' is a poem by Ans: (c) The philosophical term deconstruction
George Herbert, published in 1633. originates in the 1960s by Jacques Derrida, one of the
2. How does Christ respond to the Grand most well known philosopher and literary theorist. In
Inquisitor's accusations in Brothers Karamazov? deconstructive reading of a text shows that (a) There
(a) He kneels before the Grand Inquisitor is nothing except the text (b) a text may betray itself
(b) He kisses the Grand Inquisitor on his lips (c) there is an endless postponement of meaning.
(c) He begins to weep in remorse 6. Given below are two statements: One is labelled
(d) He says "Mea culpa, mia culpa, mia maxima as Assertion A and the other is labelled as
culpa" Reason R.
Ans: (b) 'The Brothers Karamazov' is a novel by Assertion (A): No piece of research will be the
Fyodar Dostoevsky, that was first published in 1880. first of its kind
The Grand Inquisitor is a poem (a story within a story) Reason (R): The reliability of progress in
inside the novel – The Brother Karamazov. Ivan knowledge is dependent on the
explains his prose poem 'The Grand Inquisitor', in honesty of the researchers.
16th century, Christ arrives apparently reborn on In the light of the above statements, choose the
earth. In Book V. Pro and Contra and Chapter V correct answer from the options given below:
Christ Kisses the Grand Inquisitor on his lips. (a) Both (A) and (R) are true and (R) is the correct
3. "Nice day again isn't it?" This sentence is an explanation of (A)
example of : (b) Both (A) and (R) are true but (R) is NOT the
(a) Code-switching (b) Multiple negation correct explanation of (A)
(c) Phatic communication (d) Nominaliation (c) (A) is true but (R) is false
Ans: (c) Phatic communication or small talk is a (d) (A) is false but (R) is true
conversation for its own sake. This term was coined Ans: (b) Both (A) and (R) are true but (R) is not the
by Bronislaw Malinowski. It is an informal type of correct explanation of (A).
discourse that does not cover any functional topics of 7. Match List I with List II
conversation or any transactions that need to be List I List II
addressed. Institutions Locations
4. Which agency among the following made a A. The Bhandarkar Oriental (i) Shimla
distinction between the teaching of English as a Research Institute
skill and the teaching of English literature? B. Indian Institute of (ii) New Delhi
(a) The university Education Commission 1948-49 Advanced Study
(b) The Secondary Education Commission, 1952-53 C. National Library of India (iii) Kolkatta
(c) Indian Universities Commission, 1902 D. Nehru Memorial (iv) Pune
(d) The Education Commission, 1964-66 Museum and Library
UGC NTA NET/JRF English Paper II, June 2020 Shift-II 315 YCT
Choose the correct answer from the options 12. Which of the following statements is correct in
given below: relation to Shakespeare's works?
(a) A-(iii), B-(iv), C-(ii), D-(i) (a) The Folio edition appeared in the sixteenth
(b) A-(iv), B-(i), C-(ii), D-(iii) century and the 'quartos' appeared in the
(c) A-(iv), B-(i), C-(iii), D-(ii) seventeenth century
(d) A-(i), B-(iv), C-(iii), D-(ii) (b) The 'quartos' appeared during his lifetime and
Ans : (c) Option (c) is correctly matched the the Folio edition appeared posthumously
institution and its locations. (c) The Folio edition appeared during his lifetime
(A) The Bhandarkar Oriental Research Institute–Pune. and the 'quartos' appeared posthumously
(B) Indian Institute of Advanced Study – Shimla (d) The 'quartos' refer to works written between
(C) National Library of India – Kolkatta 1594 and 1599, and the Folio includes works
(D) Nehru Memorial Museum and Library–New Delhi.
written between 1608 and 1613
8. Which two of the following are non-fictional
works by Peter Ackroyd? Ans: (b) The 'quartos' appeared during his life time
(A) Escape from Earth and the Folio edition appeared posthumously, this
(B) The Great Fire of London statement is correct in relation to Shakespeare's
(C) The English Ghost works. William Shakespeare was born in 1564 and
(D) English Music died in 1616. William Shakespeare's work was
Choose the correct answer from the options compiled in Folio by John Hemings and Henry
given below: Condell, after his death. The first folio was published
(a) (A) and (B) only (b) (A) and (C) only in 1623. It contains 36 plays.
(c) (B) and (C) only (d) (B) and (D) only 13. Which of the following journals deals with the
Ans: (b) Peter Ackroyd, British novelist, critic analysis of only theoretical concepts?
biographer and scholar. 'Escape from Earth' and 'The (a) Granta (b) Manoa
English Ghost' are non-fictional works by Peter (c) boundary 2 (d) Arethusa
Ackroyd. Ans: (c) 'Boundary 2' is a quarterly peer reviewed
9. Who was Milton's model when he recast the first academic journal of the 20th century modern theory,
edition (1667) of Paradise Lost in 10 books to 12 literature and culture. This journals deals with the
books of the second edition (1674)? analysis of only theoretical concepts.
(a) Lucan (b) Ovid
(c) Virgil (d) Homer 14. In which short story does the narrator witness a
Ans: (c) 'Virgil' was Milton's model when he recast consumptive young man named Mr. Shaynor
the first edition (1667) of Paradise Lost in 10 books to recreate "The Eve of St. Agnes" in a trance?
12 books of the second edition. Virgil's Aeneid is (a) E.M. Forster's "The Eternal Moment"
divided into 12 books. Books 1-6 Aenea's journey to (b) Rudyard Kipling's "Wireless"
Latium in Italy and Books 7-12, the war of Latium. (c) Somerset Maugham's "The Creative Impulse"
10. Which two of the following poems by Seamus (d) Aldous Huxley's "The Bookshop"
Heaney come under his Bog Poems? Ans: (b) 'Wireless' is a short story by Rudyard
(A) "Personal Helicon" (B) "Punishment" Kipling. It was first published in Scribner's Magazine
(C) "The Early Purges" (D) "Tollund Man" in 1902, and was later collected in Traffics and
Choose the correct answer from the options Discoveries, originally published in 1902. In this short
given below: story the narrator witnesses a consumptive young men
(a) (A) and (B) only (b) (A) and (C) only named Mr. Shaynor recreate 'The Eve of St. Agnes' in
(c) (B) and (C) only (d) (B) and (D) only a trance.
Ans: (d) Heaney's bog poems ostensibly represent his 15. Match List I with List II
human protest against violence embedded in the cruel List I List II
images of ritual killings and they establish his name as Author Autobiography/
a laureate of peace. 'Punishment' and 'Tollund Man' Memoir
come under his Bog Poems. 'Tollund Man' is a poem
A. Pablo Neruda (i) Under My Skin
in which the speaker promises a pilgrimage to Aarhus
in Danemark where a mummy of an unnamed victim B. Graham Greene (ii) Speak, Memory
was found preserved in a bog. C. Doris Lessing (iii) Memoirs
D. Vladimir Nabakov (iv) A Sort of Life
11. Who among the following refutes Plato's charge
that poets are liars, by arguing that the poet Choose the correct answer from the options
"nothing affirms, and therefore never lieth"? given below:
(a) John Dryden (b) Philip Sidney (a) A-(iv), B-(iii), C-(i), D-(ii)
(c) George Puttenham (d) Richard Hooker (b) A-(ii), B-(iv), C-(iii), D-(i)
Ans: (b) Sir Philip Sidney was an English poet, (c) A-(iii), B-(iv), C-(ii), D-(i)
courtier, scholar and soldier, who was remembered as (d) A-(iii), B-(iv), C-(i), D -(ii)
one of the most prominent figures of the Elizabethan Ans: (d) Correctly matched option Author with
age. He refutes Plato's charge that poets are liars by Autobiography/Memoir is option (d).
arguing that the poet "nothing affirms and therefore Pablo Neruda – Memoirs
never lieth", Philip Sidney stated this – "The poet Graham Greene – A Sort of Life
nothing affirms and therefore never lieth" in his well Doris Lessing – Under My Skin
known Defense of Poesie. Vladimir Nabakov – Speak Memory.
UGC NTA NET/JRF English Paper II, June 2020 Shift-II 316 YCT
16. Arrange the following in the chronological order Choose the correct answer from the options
of publication given below:
(A) Modern English Usage (a) (A)-(iii), (B)-(ii), (C)-(i), (D)-(iv)
(B) Proposals for Perfecting the English Language (b) (A)-(ii), (B)-(i), (C)-(iii), (D)-(iv)
(C) Usage and Abusage (c) (A)-(iii), (B)-(iv), (C)-(ii), (D)-(i)
(D) An American Dictionary of the English (d) (A)-(iv), (B)-(i), (C)-(iii), (D)-(ii)
Language Ans: (c) Option (c) is correctly matched with authors
Choose the correct answer from the options and their works.
given below: A. Ferdinand de Saussure (iii) A Course in
(a) (D), (B), (C), (A) (b) (B), (C), (D), (A) General Linguistics
(c) (B), (D), (A), (C) (d) (D), (C), (A), (B) (1916)
B. Edward Sapir (iv) Language
Ans: (c) Correct chronological order of publication is C. Jacques Derrida (ii) Of Grammatology
option (c) Proposal for perfecting the English (1967)
Language : Thomas Cooke, An American dictionary D. Roman Jakobson (i) Two Aspects of
of the English Language : Noah Webster in 1828. language and two
Modern English usages : Henry Watson Fowler in – types of Aphasic
1926. Disturbances
Usage and Abusage : Eric Partridge in - 1942 21. Who wrote the essay "Naipaul's India and
17. Who among the following called the 'Poetasters', Mine" (1984) as a reply to V.S. Naipaul's An
"The rhyming friends"? Area of Darkness?
(a) Lucan (b) Horace (a) A.K. Ramanujan (b) Nissim Ezekiel
(c) Pindar (d) Plato (c) Nayantara Sahgal (d) Mahesh Dattani
Ans: (*) The question remains cancelled and marks Ans: (b) Nissim Ezekiel wrote the essay 'Naipaul's
awarded to all. India and Mine' (1984) as a reply to V.S. Naipaul's –
18. Which two of the following edited the defining 'An Area of Darkness'. He was an Indian Jewish poet,
work of third wave feminism, This Bridge Called editor and art critic.
My Back: Writings by Radical Women of Color? 22. Which of the following is true in relation to
(A) Audre Lorde Edmund Spenser's Faerie Queene?
(B) Barbara Smith (a) A letter addressed to Sir Walter Raleigh was
(C) Gloria Anzaldua prefixed to the 1590 edition of the poem
(D) Cherrie Moraga (b) A letter addressed to Sir Walter Raleigh was
appended to the 1590 edition of the poem
Choose the correct answer from the options
(c) A letter addressed to Sir Walter Raleigh was
given below: prefixed to the 1596 edition of the poem
(a) (A) and (B) only (b) (C) and (D) only (d) A letter addressed to Sir Walter Raleigh was
(c) (A) and (C) only (d) (B) and (D) only appended to the 1596 edition of the poem
Ans: (b) 'This Bridge Called My Back : Writings by Ans: (b) 'The Faerie Queene' is an English epic poem
Radical Women of Color' is a feminist anthology by Edmund Spenser. 'The Faerie Queene' is divided
edited by Cherrie Moraga and Gloria E. Anzaldua first into seven books with the first six books containing 12
published in 1981 by Persephone Press. cantos each. The seventh book has only two cantos. It
19. What does the titular Setebos in Robert is addressed to Spenser's friend and court superior Sir
Browning's "Caliban upon Setebos" refer to? Walter Raleigh, the letter explains Spenser's intentions
(a) The original name of Sycorax, Caliban's mother in the poem.
(b) The brutal god in whom Caliban believes 23. In his "Self-Reliance" which two qualities does
(c) The name of the island in which Caliban lives Emerson refer to as "the Chancellors of God"?
(d) The monster whom Caliban is afraid of (A) Truth (B) Cause
Ans: (b) 'Caliban upon Setebos' is a poem written by (C) Spirit (D) Effect
the British poet Robert Browning, and published in Choose the correct answer from the options
his 1864 Dramatis Personea Collection. It deals with given below:
Caliban, a character from Shakespeare's 'The Tempest' (a) (A) and (B) only (b) (A) and (C) only
and his reflection on Setebos, the brutal god he (c) (B) and (C) only (d) (B) and (D) only
believes in. Ans: (d) 'Self Reliance' is an 1841 essay written by
American transcendentalist philosopher and essayist
20. Match List I with List II Ralph Waldo Emerson. 'Cause' and 'Effect' are two
A. Ferdinand de Saussure (i) "Two Aspects of qualities that Emerson refers to as 'the Chancellors of
language and two God'.
types of Aphasic 24. Arrange the following critical works in the
Disturbances" chronological order of publication:
B. Edward Sapir (ii) Of Grammatology (A) "Preface to Lyrical Ballads"
C. Jacques Derrida (iii) A Course in (B) A Defence of Rhyme
General Linguistics (C) "Life of Cowley"
D. Roman Jakobson (iv) Language (D) "Frontiers of Criticism"

UGC NTA NET/JRF English Paper II, June 2020 Shift-II 317 YCT
Choose the correct answer from the options (b) Both (A) and (R) are correct but (R) is NOT
given below: the correct explanation of (A)
(a) (A), (C), (B) and (D) (b) (B), (A), (C) and (D) (c) (A) is correct but (R) is not correct
(c) (B), (C), (A) and (D) (d) (C), (A), (D) and (B) (d) (A) is not correct but (R) is correct
Ans: (c) 'A Defence of Rhyme' (1503) by Samuel Ans: (a) Both (A) and (R) are correct and (R) is the
Daniel, 'Life of Cowley' (1779-1781) by Samuel correct explanation of (A).
Johnson, 'Frontiers of Criticism' is a lecture given by 28. Which one of these essays by Ezra Pound
T.S. Eliot, at the university of Minnesota in 1956. defines an Image as "that which presents an
Hence option (c) is correct in the chronological order intellectual and emotional complex in an instant
of publication. of time?
25. Which one of the following captures accurately (a) "A Retrospect" (b) "The Tradition"
the view of Frankfart School of Critical Theory? (c) "The Renaissance" (d) "How to Read"
(a) the culture industries in still in their mass Ans: (a) "A Retrospect"
audience a capacity to question and transform Pound's "A Retrospect" published in 1918 is a
(b) The culture industries engender passivity and collection of his essay on poetry. In "A Retrospect"
conformity among their mass audiences
Pound presents his beliefs about what makes good
(c) Power and culture are two distinct modes of
poetry. The essay begins with the three principles of
social articulation, separate from each other
(d) The analysis of culture should be divorced imagism. He defines "image" as an intellectual and
from politics and power relations emotional complex in on instant of time.
Ans: (b) The Frankfart School, known more 29. Which two poems in the following list are Odes
appropriately as critical theory, is a philosophical and Written in the Horatian manner?
sociological movement spread across many (A) Ben Jonson, "To the Immortal Memory and
universities around the world. The culture industries Friendship of that Noble Pair, Sir Lucius Cary
engender passivity and conformity among their mass and Sir H. Morison"
audiences captures accurately the view of Frankfart (B) Andrew Marwell, "Upon Cromwell's Return
School of Critical Theory. from Ireland"
26. Arrange the following in the chronological order (C) Alexander Pope, "Ode on Solitude"
of their publication: (D) Alfred Tennyson, "Ode on the Death of the
(A) Past and Present Duke of Wellington"
(B) Leviathan Choose the correct answer from the options
(C) Unto This Last given below:
(D) The Life of Samuel Johnson (a) (A) and (B) only (b) (B) and (C) only
Choose the correct answer from the options (c) (C) and (D) only (d) (A) and (D) only
given below: Ans: (b) The two poems – "Upon Cromwell's Return
(a) (B), (D), (A), (C) (b) (C), (D), (A), (B) from Ireland" by Andrew Marwell and Alexander
(c) (B), (A), (D), (C) (d) (C), (A), (D), (B) Pope's "Ode on Solitude" are ode that written in the
Ans: (a) The correct chronological order of the Horatian Manner. The Horatian Ode is simply a
publication is 'Leviathan' (1651), 'The Life of Samuel stanzaic form in which all stanzas are structured in the
Johnson' (1791), 'Past and Present' (1852) and 'Unto same pattern at the direction of the poet.
This Last' (1860). 'Leviathan' is a mythical creature 30. Which two of the following are the interludes in
with the form of a sea serpent in Judaism, 'The Life of John Galsworthy's The Forsyte Saga (1922)
Samuel Johnson' by James Boswell is a biography of (A) To Let
English writer Dr. Samuel Johnson, 'Past and Present' (B) Indian Summer of a Forsyte
is a book by Thomas Carlyle and 'Unto This Last' is (C) Awakening
an essay and book on economy by John Ruskin. (D) In Chancery
27. Given below are two statements : One is labelled Choose the correct answer from the options
as Assertion A and the other is labelled as given below:
Reason R. (a) (A) and (B) only (b) (A) and (C) only
Assertion (A): English today is not only the (c) (B) and (C) only (d) (B) and (D) only
language we teach but also the
subject that enables its learners to Ans: (c) 'The Forsyte Saga', sequence of three novels
become subtle and tough minded linked by two interludes by John Galsworthy as
readers. published in 1922. 'The Forsyte Saga' consisted of the
Reason (R): Students are encouraged to think novel 'The Man of Property' (1906) the interlude
and analyze the historical and "Indian Summer of a Forsyte" (1918) the novel 'In
ontological status of the texts they Chancery' (1920), the interlude "Awakening" (1920)
read, and how best to read them. and the novel 'To Let' (1921).
In the light of the above statements, choose the 31. Given below are two statements:
most appropriate answer from the options given Statement I: Cultures and cultural meanings are
below: the same the world over.
(a) Both (A) and (R) are correct and (R) is the Statement II: It is impossible to divide the world
correct explanation of (A) into exclusive cultural blocs.
UGC NTA NET/JRF English Paper II, June 2020 Shift-II 318 YCT
In the light of the above statements, choose the Choose the correct answer from the options
correct answer from the options given below: given below:
(a) Both Statement I and Statement II are true (a) (A) and (B) only (b) (B) and (C) only
(b) Both Statement I and Statement II are false (c) (B) and (D) only (d) (C) and (D) only
(c) Statement I is correct but Statement II is false
(d) Statement I is incorrect but Statement II is true Ans: (d) 'A Journal of the Plague Year' and
Ans: (d) Culture is the characteristic and knowledge 'Memories of a Cavalier' both works are historical
of a particular group of people encompassing narratives. 'A Journal of the Plague Year' (1722) is a
language, religion, social habits music and arts fictionalized account of the bubonic Plague that swept
whether cultural means relating to a particular society through London in 1665. 'Memories of a Cavalier' is
and its ideas, customs and art. So, statement (I) is also a work of historical fiction, set during the thirty
incorrect but Statement (II) is true. years war and the English Civil Wars.
32. Which two of the following novels are part of 37. Which two of the following essays have proved
Paul Auster's New York Trilogy? particularly productive in the disciplinary
(A) The Book of Illusions (B) Ghosts practices of Cultural Studies?
(C) The Locked Room (D) Winter Journal
Choose the correct answer from the options (A) Laura Mulvey, "Visual Pleasure and Narrative
given below: Cinema"
(a) (A) and (B) only (b) (A) and (C) only (B) Viktor Shklovsky, "Art as Technique"
(c) (B) and (C) only (d) (B) and (D) only (C) Sigmund Freud, "The Uncanny"
Ans: (c) The New York Trilogy is a series of novels (D) Stuart Hall, "Encoding/decoding"
by American writer Paul Auster. Originally published Choose the most appropriate answer from the
sequentially as City of Glass (1985), Ghosts (1986) options given below:
and The Locked Room (1986), it has since been (a) (A) and (B) only (b) (B) and (C) only
collected into a single volume. Trilogy is a (c) (A) and (D) only (d) (A) and (C) only
postmodern interpretation of detective and mystery
fiction exploring various philosophical themes. Ans: (c) "Visual Pleasure and Narrative Cinema" and
"Encoding/decoding" essays have proved productive
33. Who among the following theorists defines novel
as "a phenomenon multiform in style and in the disciplinary practices of Cultural Studies.
variform in speech and voice"? 'Visual Pleasure and Narrative Cinema' was written in
(a) E.M. Forster (b) Henry James 1973 and published in 1975 in the influential British
(c) Mikhail Bakhtin (d) Eric Auerbach film theory journal screen. The 'Encoding/decoding'
Ans: (c) In Epic and Novel Bakhtin argues that the model of communication was first developed by
novel flourishes on diversity, making it uniquely cultural studies in 1973.
suited to post-industrial society. Bakhtin defines novel 38. Match List I with List II
as a whole is 'a phenomenon multiform in style and List I List II
variform in speech and voice'. Concepts Theorists
34. Who is the author of the short play, The Dark (A) Competence/ (i) Noam Chomsky
Lady of the Sonnets? Performance
(a) Ben Jonson (b) George Bernard Shaw
(c) Oscar Wilde (d) Oliver Goldsmith (B) Signifier/Signified (ii) Roman Jakobson
Ans : (b) 'The Dark Lady of the Sonnets' is a 1910 (C) Metaphor/Metonymy (iii) Louis Hjelmslev
short comedy by George Bernard Shaw in which (D) Content/Expression (iv) Ferdinand de
William Shakespeare, intending to meet the Dark Saussure
Lady accidentally encounters queen Elizabeth I and Choose the correct answer from the options
attempts to pursue her to create a national theater. given below:
35. Which two of the following plays are mentioned (a) (A)-(iii), (B)-(ii), (C)-(i), (D)-(iv)
in T.S. Eliot's "Tradition and Individual (b) (A)-(ii), (B)-(iii), (C)-(i), (D)-(iv)
Talent"? (c) (A)-(iii), (B)-(iv), (C)-(ii), (D)-(i)
(A) Agamemnon (B) Antigone (d) (A)-(i), (B)-(iv), (C)-(ii), (D)-(iii)
(C) Othello (D) Dr. Faustus
Choose the correct answer from the options Ans: (d) The concept Competence/Performance is the
given below: system of linguistic knowledge possessed by native
(a) (A) and (D) only (b) (A) and (C) only speakers of a language and given by Noam Chomsky.
(c) (B) and (C) only (d) (B) and (D) only Signifier/Signified is a concept most commonly
Ans: (b) "Tradition and the Individual Talent" (1919) related to semiotics, that can be described as "The
is an essay written by poet and literary critic T.S. study of signs and symbols". Metaphor and
Eliot. This is one of the more well known work that Metonymy are two fundamental opposite poles along
Eliot produced in his critical capacity. The plays which a discourse with human language is developed.
Agamemnon and Othello are mentioned in this text. 39. Which two of the following are part of Virginia
36. Which two of the following works are Daniel Woolf's collection of autobiographical essays?
Defoe's historical narratives? (A) "A Will to Word It"
(A) History of the Rebellion
(B) Meditations on a Broomstick (B) "A Sketch of the Past"
(C) A Journal of the Plague Year (C) "A Faint Hue of the Past"
(D) Memories of a Cavalier (D) "Am I a Snob"
UGC NTA NET/JRF English Paper II, June 2020 Shift-II 319 YCT
Choose the correct answer from the options 43. Which of the following groups of words
given below: correctly states the stages of communication as
(a) (A) and (B) only (b) (A) and (C) only envisioned by Stuart Hall in his essay
(c) (B) and (C) only (d) (B) and (D) only "Encoding, Decoding"?
Ans: (d) Virginia Woolf provides insight into her (a) Production, transference, circulation, contact,
early life in her autobiographical essays, including reproduction
'Reminiscences' (1908), '22 Hyde Park Gate' (1921), (b) Production, circulation, realization, consumption,
'A Sketch of the Past' (1940) and 'Am I a Snob'. reproduction
(c) Production, circulation, distribution,
40. Which two of the following are Samuel
consumption, reproduction
Johnson's statements about metaphysical poets? (d) Production, dissemination, transference,
(A) they were singular in their thoughts consumption, reproduction
(B) they were careful in their diction
(C) they effected combination of dissimilar images Ans: (c) The 'Encoding/decoding' model of
(D) they avoided occult resemblances communication was first developed by cultural studies
scholar Stuart Hall in 1975. titled – Encoding and
Choose the most appropriate answer from the
Decoding in the Television Discourse. Hall's essay
options given below: offers a theoretical approach of production,
(a) (B) and (C) only (b) (C) and (D) only circulation, distribution, consumption and
(c) (B) and (A) only (d) (A) and (C) only reproduction of media messages.
Ans: (d) The term metaphysical poets was coined by 44. What is the content of the suitcases that Lucky
the critic Samuel Johnson to describe a loose group of carries in the second Act of Waiting for Godot?
17th century English poets whose work was (a) Books (b) Pozzo's Clothing
characterized by the inventive use of conceits and by (c) Sand (d) Tiny Skulls
greater emphasis on the spoken rather than lyrical Ans: (c) 'Lucky' is unique in a play where most of the
quality of their verse. They were singular in their characters talk incessantly. Lucky is often compared
thoughts and they effected combination of dissimilar to Vladimir as being the intellectual.
images. Sand is the content of the suitcases that Lucky carries
41. While assembling a working bibliography which in the second act of 'Waiting for Godot'. Lucky suffers
two of the following reference sources will be at the hand of Pozzo willingly and without hesitation.
particularly useful to a literary researcher? 'Waiting for Godot', published by Samuel Backet in
(A) MLA International Bibliography 1949, is a work of Absurdism that explores themes of
(B) New Princeton Encyclopedia of Poetry and Existentialist philosophy.
Poetics 45. Inductive method differs from deductive method
(C) Library of Congress Catalogue in drawing its conclusion from
(D) Reader's Guide to Periodical Literature (a) Verification (b) Particular instances
Choose the correct answer from the options (c) Applications (d) General truths
given below: Ans: (b) Inductive method– An inductive approach
(a) (A) and (B) only (b) (B) and (C) only to teaching language starts with examples and asks
(c) (A) and (D) only (d) (C) and (D) only learners to find rules.
Ans: (c) Working bibliography is a listing of possible Deductive reasoning work from the "general" to the
sources that you might use for gathering information 'specific'. This is also called a 'top-down' approach.
and writing your research paper. The two references In a conclusion, when we use deduction we reason
'MLA International Bibliography' and 'Reader's Guide from general principles to specific cases, as in
to Periodical Literature' are useful to a literary applying a mathematical theorem to a particular
researcher. problem.
42. Which two of the following plays were written Inductive method differs from deductive method in
by John Osborne? drawing its conclusion from – particular instances.
(A) Look Back in Anger (B) Loot 46. Arrange these autobiographical texts in the
(C) Funeral Games (D) Dejavu chronological order of publication:
Choose the correct answer from the options (A) Autobiography of an Unknown Indian
given below: (B) My Experiments with Truth
(a) (A) and (B) only (b) (A) and (C) only (C) Prison and Chocolate Cake
(c) (A) and (D) only (d) (B) and (C) only (D) My Story
Ans: (c) 'Look Back in Anger' and 'Dejavu' plays Choose the correct answer from the options
were written by John Osborne. 'Look Back in Anger' given below:
is a realist play published in 1956. It focuses on the (a) (D), (A), (C), (B) (b) (C), (B), (A), (D)
life and marital struggles of an intelligent and (c) (B), (A), (C), (D) (d) (B), (C), (A), (D)
educated but disaffected young man of working class Ans : (c) My Experiments with Truth or
origin Jimmy Porter and his equally competent yet Autobiography is the autobiographical work of
impassive upper middle class wife Alison. 'Dejavu' is Mohandas Karam Chandra Gandhi covering his life
a stage play by John Osborne. It was his final work for from early childhood through to 1921. It was written
the theatre, the failure of which on the stage made him in weekly installments and published in his journal
decide to give up play writing. Navjivan from 1925 to 1929.

UGC NTA NET/JRF English Paper II, June 2020 Shift-II 320 YCT
Autobiography of an Unknown Indian is the 50. Which two of the following periodicals are
autobiography of Nirad C. Chaudhuri (an Indian devoted to feminist theoretical discussion?
writer). The book relates his mental and intellectual (A) Spectrum (B) Signs
development. It is written in 1951. (C) Chrysalis (D) Transition
Prison and Chocolate Cake is written in 1954 by Choose the correct answer from the options
Nayantara Sahgal, the niece of Jawaharlal Nehru and given below
cousin of the late Indira Gandhi. Sahgal is known to (a) (B) and (C) only (b) (A) and (C) only
have insisted, 'being Nehru's niece or Madame (c) (B) and (D) only (d) (A) and (D) only
Pandit's daughter. Ans: (a) 'Chrysalis : A magazine of women's culture'
My Story (1976 in English) is an autobiographical was a feminist publication produced from 1977 to
book by Indian author and poet Kamala Das (also 1980. The self published magazine was founded by
known as Kamala Surayya or Madhavikutty. Kristen Grimstad and Sussan Rennie. 'Signs : Journal
The book, with 50 chapters, narrates thatshe had to of women in culture and society' is a peer reviewed
face racist discrimination, through the brutal and feminist academic journal. It was established in 1975
indecent relationship with her husband. by Jean W. Sacks.
47. Which two of the following features shall apply 51. The two broad divisions of reality in Plato's
to Roland Barthes's notion of a 'writerly text'? theory of reality are:
(A) In case of writerly text, the reader accepts the (a) visible and assumable
meaning without too much reading effort (b) intelligible and opinable
(B) A writerly text tends to focus attention on what (c) visible and intelligible
is written (d) intelligible and shadows
(C) A writerly text makes the reader a producer Ans: (c) Plato believed that true reality is not found
(D) A writerly text tends to be self-conscious through the sense. Phenomenon is that perception of
Choose the correct answer from the options an object which we recognize through our sense. The
given below: two broad divisions of reality in Plato's theory of
(a) (A) and (B) only (b) (B) and (C) only reality are visible and intelligible.
(c) (A) and (C) only (d) (C) and (D) only 52. Match List I with List II
Ans: (d) A writerly text makes the reader a producer List I List II
Text Author
and A writerly text tends to be self-conscious shall A. After Amnesia (i) Gauri
apply to Roland Barthes's notion of a writerly text. Viswanathan
The writerly text– By contrast, writerly text reveal B. The Indianization (ii) Harish Trivedi
those element that the readerly attempts to conceal. of English
The reader now in a position of control, takes an C. Masks of Conquest (iii) G.N. Devy
active role in the construction of meaning. D. Colonial Transactions (iv) B.B. Kachru
48. Which two of the following words are borrowed Choose the correct answer from the options
into English from Czech? given below:
(A) pistol (B) robot (a) A-(iv), B-(iii), C-(i), D-(ii)
(C) Sauna (D) coach (b) A-(ii), B-(iv), C-(i), D-(iii)
Choose the correct answer from the options (c) A-(iii), B-(iv), C-(i), D-(ii)
given below: (d) A-(iii), B-(i), C-(iv), D-(ii)
(a) (A) and (B) only (b) (A) and (C) only Ans: (c) Ganesh N. Devy is a thinker, cultural activist
(c) (B) and (C) only (d) (A) and (D) only and an institution builder best known for the people's
Ans: (a) 'Pistol' a word of uncertain origin, sometimes linguistic served of India.
said to be from German Pistole, from Czech pis'tala His first full length book in English 'After Amnesia'
"firearm". Word 'Robot' is a relative new comer to the (1992). He got Padmashri (2014) and Sahitya
English language. It was the brain child of a brilliant Akademi (1993).
Czech playwright novelist and journalist Karel Capek. B.B. Kachru was an Indian linguist. He was Jubilee
Hence option (a) is correct. professor of linguistics at the University of Illinois at
Urbana – Champaign.
49. Arrange the following plays in their In 'Colonial Transactions' a study of the literary
chronological order: exchanges between England and India through the
(A) The Tempest (B) All For Love colonial and post-colonial periods. Using materials
(C) Volpone (D) The School for Scandal from Indian and Western archives.
Choose the correct answer from the options 'Masks of Conquest' is a classic work in postcolonial
given below: studies, it describes the introduction of English studies
(a) (A), (C), (B), (D) (b) (C), (A), (B), (D) in India under British rule and its function as an
(c) (C), (B), (A), (D) (d) (A), (D), (B), (C) effective form of political control.
Ans: (b) 'Volpone' is a comedy play by English play 53. Match List I with List II
writer Ben Jonson first produced in 1605-1606. 'The List I List II
Tempest' is a dramatic romance by William Novel Character
Shakespeare first performed in 1611. 'All For Love' is A. Barnaby Rudge (i) Miss La Creevy
a 1677 heroic drama by John Dryden. 'The School for B. Little Dorrit (ii) Miss Dolly
Scandal' is written by R.B. Sheriden, first performed C. Nicholas Nickleby (iii) Mrs. Boffin
in London, 1777. D. Our Mutual Friend (iv) Mrs. Flintwinch
UGC NTA NET/JRF English Paper II, June 2020 Shift-II 321 YCT
Choose the correct answer from the options 57. Which two of the following poets defended
given below : poetry against Plato's denigration of Poetry?
(a) A-(i), B-(iii), C-(ii), D-(iv) (A) John Dryden (B) P.B. Shelley
(b) A-(iii), B-(ii), C-(iv), D-(i) (C) T.S. Eliot (D) Philip Sidney
(c) A-(ii), B-(iv), C-(i), D-(iii) Choose the most appropriate answer from the
(d) A-(iv), B-(i), C-(iii), D-(ii) options given below:
Ans: (c) 'Barnaby Rudge' is a historical novel by (a) (B) and (D) only (b) (A) and (B) only
British novelist Charles Dickens. It was the fifth of (c) (B) and (C) only (d) (C) and (A) only
Dickens' novels, it had initially been planned to
Ans: (a) P.B. Shelley and Philip Sidney defended
appear as his first but changes of publisher.
Jeremiah Flintwinch, fictional character in the novel poetry against Plato's denigration of Poetry.
'Little Dorrit' by Charles Dickens. Originally the The study of Plato on beauty must begin with one
Clennam family butler, Flintwinch becomes the warning. Readers can take this distinction between the
business partner of Mrs. Clennam. Greek and English. It always feels more scrupulous to
'Nicholas Nickleby or The Life and Adventures of argue.
Nicholas Nickleby' is a novel by Charles Dickens. On Philip Sidney states that poetry is an art of imitation,
the second morning after the departure of Nicholas for for so Aristotle termed it in his word mimesis. Sir
Yorkshire, Kete Nickleby sat in a very faded chair Philip Sidney (1554-1586) is often cited as an
raised upon a very dusty throne in Miss La Creevy's archetype of the well-rounded "Renaissance Man".
room. 58. Match List I with List II
Mrs. Boffin, the Golden Dustman – becomes a List I List II
member of the nouveaux riches when old Mr. Critics Essays
Harmon's heir is considered dead. Mrs. Henrietta A. L.C. Knights (i) "The Study of Poetry"
Boffin – is Noddy Boffin's wife, and a very motherly B. Lionel Trilling (ii) "Restoration Comedy
woman, who convinces Mr. Boffin to take in an
orphan boy called Johnny. : The Reality and the
Myth"
54. Who is the author of The Complete Plain Words? C. Matthew Arnold (iii) "Poetry for Poetry's
(a) Samuel Johnson (b) Daniel Jones
(c) Ernest Gowers (d) Michael Everson Sake"
D. A.C. Bradley (iv) "The Sense of the
Ans: (c) 'The Complete Plain Words' is a style guide
written by Sir Ernest Gowers published in 1954. The Past"
aim of the book is to help officials in their use of Choose the correct answer from the options
English as a tool of their trade. given below:
Gowers's public service career continued as chairman (a) A-(iii), B-(iv), C-(i), D-(ii)
of several Royal Commissions. (b) A-(iv), B-(i), C-(ii), D-(iii)
55. In Harold Pinter's play The Birthday Party, who (c) A-(ii), B-(iv), C-(i), D-(iii)
suggests the idea of having a birthday party? (d) A-(iv), B-(iii), C-(i), D-(ii)
(a) Meg (b) Goldberg Ans: (c) Lionel Charles Knights was an English
(c) Lulu (d) McCann literary critic, an authority on Shakespeare and his
Ans: (b) In 'The Birthday Party', Goldberg suggests period. His essay 'Restoration Comedy : The Reality
the idea of having a birthday, he is called – "Simey" and the Myth' became very popular.
and "Benny", is a Jewish gentleman who works for an 'The Sense of the Past' is written by Lionel Trilling. In
unnamed "Organization" that has employed him to sincerity and authenticity Trilling analyses the
take Stanley away from the boarding house. He is emergence of two historical value in the European
defined by his outwardly polite and suave demeanor, literature and philosophy of the early modern and
which stands in stark contrast to that of his associate moderns' periods.
McCann. Mathew Arnold was one of the foremost poets and
56. In which two of the following plays does the critics of the 19th century. Perhaps Arnold's most
blind seer Tiresias, appear? famous piece of literary criticism in his essay – "The
(A) Oedipus the King Study of Poetry".
(B) Agamemnon 'Poetry for Poetry's Sake' by Andrew Cecil Bradley,
(C) Antigone was an English literary scholar, best remembered for
(D) Oedipus at Colonus his work on Shakespeare.
Choose the correct answer from the options 59. To which of these boarding schools is Jane Eyre
given below:
(a) (A) and (B) only (b) (A) and (C) only sent by her aunt Mrs. Reed?
(c) (B) and (C) only (d) (C) and (D) only (a) Lowood School (b) Hailsham school
(c) Abbey Mount (d) Greyfriar's School
Ans: (b) Tiresias, the blind soothsayer of Thebes,
appears in both 'Oedipus the King' and 'Antigone'. Ans: (a) Lowood School (boarding schools) is Jane
In 'Oedipus the King' Tiresias tells Oedipus that he is Eyre set by her aunt Mrs. Reed.
the murderer he hunts, and Oedipus does not believe Jane Eyre is a novel by English writer Charlotte
him. Bronte, published under the pen name "Currer Bell".
In 'Antigone' Tiresias tells Creon that Creon himself is The novel revolutionized prose fiction by being the
bringing disaster upon Thebes, and Creon does not first to focus on its protagonist's moral and spiritual
believe him. development.

UGC NTA NET/JRF English Paper II, June 2020 Shift-II 322 YCT
60. Arrange the following novels in the Benedict Anderson writing was one possible route
chronological order of their publication: through which student of literature, politics and
(A) The White Tiger history and numerous other disciplines would unlearn
(B) A Tiger for Malgudi the idea that the expression and movement for
(C) A Suitable Boy national political independence was a European
(D) Heat and Dust invention.
Choose the correct answer from the options 64. Language allows us to talk about the things and
given below: events not present in immediate environment.
(a) (D), (B), (C), (A) (b) (B), (D), (C), (A) Which of the following terms describes this
(c) (B), (C), (A), (D) (d) (B), (C), (D), (A) property of language?
Ans: (a) 'Heat and Dust' (1975) is a novel by Ruth (a) Arbitrariness (b) Displacement
Prawer Jhabvala. (c) Productivity (d) Discreteness
'A Tiger for Malgudi' is a novel which published in Ans: (b) Language allows us to talk about the things
1983 by R.K. Narayan. and events not present in immediate environment
'A Suitable Boy' is a novel by Vikram Seth, published Displacement describes this property of language.
in 1993. Displacement mean – a situation in which people are
'The White Tiger' is the debut novel by Indian author forced to leave the place where they normally live.
Arvind Adiga. It was first published in 2008. Lateral displacement : This modification of the
61. Which of the following information has now prepulse level induced a lateral displacement of the
been excluded while making an entry for a book local spot.
in the 8th edition of MLA Hand book for Writers 65. Which one of the following statements by
of Research Papers? Roman Jacobson is true about metaphor and
(a) Year of publication metonymy?
(b) Place of publication (a) Metaphor is alien to the continuity disorder
(c) Name of the publisher whereas metonymy is alien to similarity
(d) Omission of subtitle disorder
(b) Metaphor is alien to the similarity disorder and
Ans: (b) Place of publication has now been excluded metonymy to the continuity disorder
while making an entry for a book in the 8th edition of (c) Metaphor is alien to both similarity disorder
MLA Hand book for Writers of Research Papers. and continuity disorder and metonymy is
For this reason, the new edition focuses on the writers common to both
strategy and individual decisions. Like earlier edition, (d) Metaphor is common to both similarity
this handbook includes information on evaluating disorder and continuity disorder but metonymy
sources, avoiding plagiarism, using quotations. is alien to both
This updated version reflects the ways in which digital Ans: (b) Metaphor is alien to the similarity and
publication has changed how writers and researchers metonymy to the continuity disorder.
document sources. Roman Jacobson concepts of Metaphor and
62. In which Act of William Congreve's The Way of Metonymy in his essay, Two Aspects of language and
the World does the Proviso scene between two types of Aphasic disturbance, Jacobson proposes
Mirabell and Millamant take place? that language has a bipolar structure.
(a) Act I (b) Act II Jakobson (Jacobson) holds that poetry is metaphoric,
(c) Act III (d) Act IV in that it focuses on signs and on the principle of
Ans: (d) In Act IV, William Congreve's 'The Way of similarity, while prose is metonymic. Analyzing the
the World' does the Proviso scene between Mirabell structure of dreams, he illustrates this conflict by
and Millamant take place. highlighting the question whether the symbols and
William Congreve was an English playwright and temporal sequences are based on contiguity.
poet of the Restoration period. Congreve is known for 66. Who among the following has coined the term
his clever, satirical dialogue. 'genderlect'?
When the play opens, Mirabell is impatiently waiting (a) Lydia Callis (b) Kate Burridge
to hear that Waitwell is married to Foible. During (c) Deborah Tannen (d) Mary Haas
Mirabell's card game with Fainall, it become clear that Ans: (c) Deborah Tannen has coined the term
the relations between the two men are strained. 'genderlect' to describe the way that the conversation
63. Who among the following drew attention to the of men and women are not right and wrong, superior
role of print languages in enabling the rise and and inferior – they are just different.
spread of nationalism? A useful way of viewing this that she uses it they are
(a) Ernest Gellner (b) Charles Jenks as different cultures. Thus, as a Japanese and French
(c) Benedict Anderson (d) Frederic Jameson person conversing would take account of each others.
Ans: (c) Benedict Anderson drew attention to the role 67. In "Advancement of learning" Francis Bacon
of print languages in enabling the rise and spread of divides poetry into three divisions:
nationalism. (a) Philosophical, religious, imaginative
The printing press is widely credited for modern (b) Epic, dramatic, lyrical
nationalism and the birth of the nation-state as the (c) Narrative, representative, allusive
primary actors in political legitimacy. (d) Odes, sonnets, eclogues

UGC NTA NET/JRF English Paper II, June 2020 Shift-II 323 YCT
Ans: (c) In "Advancement of learning" Francis Bacon Choose the correct answer from the options
divides poetry into three divisions : Narrative, given below :
representative, allusive. In Advancement of learning (a) (A)-(iii), (B)-(i), (C)-(iv), D-(ii)
(1605), had earlier proposed a new science of (b) (A)-(ii), (B)-(iv), (C)-(iii), D-(i)
observation and experiment to replace the traditional (c) (A)-(iii), (B)-(iv), (C)-(i), D-(ii)
Aristotelian science. He charted the map of (d) (A)-(ii), (B)-(iv), (C)-(i), D-(iii)
knowledge history, which depends on the human
Ans: (c) The root of 'caste' is the Latin castus which
faculty of memory, poetry, which depends on
imagination and philosophy, which depends on means 'chaste' or pure, separated. The word arrived in
reason. English through the Portuguese caste, which means –
68. Which of the following is the correct sequence of 'race' of 'lineage'.
stages in empirical research? 'Beef' has been borrowed from Anglo-Norman (such
(A) Data Collection (B) Hypothesis as 'beef' prepared food) or French.
(C) Validation (D) Findings The word 'blunder' has been borrowed from Norse.
(E) Analysis 'Flak' has been borrowed from German.
Choose the correct answer from the options 71. Joyce Cary's The Horse's Mouth, considered by
given below: many to be his masterpiece, is part of a trilogy
(a) (A), (E), (D), (B) and (C) of novel. Which two titles from the following list
(b) (B), (A), (E), (C) and (D) belong to this trilogy?
(c) (B), (C), (A), (D) and (E) (A) Aissa Saved
(d) (A), (C), (B), (E) and (D)
(B) To Be a Pilgrim
Ans: (b) An empirical hypothesis or working (C) Herself Surprised
hypothesis, comes to life when a theory is being put to
the test, using observation and experiment. (D) Charley Is My Darling
Data collection is the process of gathering and Choose the correct answer from the options
measuring information an variables of interest in an given below:
established systematic fashion. (a) (A) and (B) only (b) (B) and (C) only
Analysis provides an empirical review and synthesis (c) (C) and (D) only (d) (A) and (D) only
of published studies that have used content analysis Ans: (b) Joyce Cary's 'The Horse's Mouth' considered
methods. by many to be his masterpiece, is part of a trilogy of
Empirical Validation of a Dynamic Hypothesis – The novels. 'To Be a Pilgrim' and 'Herself Surprised' two
starting point for this research is a dynamic hypothesis titles belong to this trilogy.
– a potential explanation of how structure is causing 'The Horse's Mouth' (1944) is a novel by Anglo-Irish
observed behaviour. writer Joyce Cary, the third in his First Trilogy, whose
Empirical research is based on observation and
measurement of phenomena, as directly experienced first two books are 'Herself surprised' and 'To Be a
by the researcher. Pilgrim'.
69. Which of the following characters instruct 72. Which of the following poems by Philip Larkin
Faustus in the dark arts? ends with the line "Never such innocence
(a) Robin and Rafe again"?
(b) Cornelius and Valdes (a) "An Arundel Tomb" (b) "MCMXIV"
(c) Wagner and Bruno (c) "This Be the Verse" (d) "Aubade"
(d) Old Man and Evil Angel Ans: (b) MCMXIV (1914) is a poem written by
Ans: (b) Cornelius and Valdes instruct Faustus in the English poet Philip Larkin. It was first published in
dark arts. The play may have been entered into the the book 'The Whitsun Weddings' in 1964.
stationers Register on 18 December 1592, though the The poem, a single sentence spread over four stanzas,
records are confused and appear to indicate a conflict
over the rights to the play. begins by describing what is seemingly a photograph
Cornelius also instructs Faustus in the art of black and goes on to reflect on the momentous changes in
magic. England that would result from the First World War.
Valdes : A magician, Valdes instructs Faustus in the 'MCMXIV' is one of Philip Larkin's best loved poems,
art of black magic. completed in May 1960, the poem was published in
Students of Faustus, the three scholar come in search Larkin's 1964 volume 'The Whitsun Weddings'.
of Faustus early in the play, only to find that he has 73. Metaphor differs from simile in that
fallen into a damned art from which he may not be (a) A comparison in metaphor is usually explicit
able to be reclaimed. whereas in simile it is implicit
70. Match List I with List II (b) A comparison in metaphor is usually implicit
List I List II whereas in simile it is explicit
Word Borrowed Source Language
(A) caste (i) Norse (c) Neither metaphor nor simile is rooted in
(B) beef (ii) German comparison
(C) blunder (iii) Portuguese (d) Simile involves superimposition while
(D) flak (iv) French metaphor involves comparison
UGC NTA NET/JRF English Paper II, June 2020 Shift-II 324 YCT
Ans: (b) Metaphor differs from simile in that a Choose the correct answer from the options
comparison in metaphor is implicit whereas in simile given below:
it is explicit. (a) (A) and (B) only (b) (A) and (C) only
Metaphor is a figure of speech for rhetorical effect, (c) (B) and (C) only (d) (B) and (D) only
directly refers to one thing by mentioning another. It Ans: (a) 'On the Origin of Species' and 'A Tale of
may provide identify hidden similarities between two Two Cities' were published in the year 1859.
different ideas. 'On the Origin of Species' published on 24 November
e.g. All the world's stage 1859, is a work of scientific literature by Charles
And all the men and women merely Darwin. The book presented a body of evidence that
players. ( Metaphor) the diversity of life.
74. Arrange the following in their chronological 'A Tale of Two Cities' (1859) is an historical novel by
order of publication: Charles Dickens, set in London and Paris before and
(A) The Pisan Canto during the French Revolution. The novel tells the
(B) Ballad of Reading Goal story of French Doctor, Manette, his 18 year-long
(C) Mourn not for Adonais imprisonment.
(D) First step up Parnassus 77. Which of the following short stories by Jorge
(E) The Complaint of Troilus Luis Borges has its epigraph from The Anatomy
Choose the correct answer from the options of Melancholy?
given below: (a) "Borges and I"
(a) (E), (D), (B), (C), (A) (b) "Death and the Compass"
(b) (B), (C), (A), (E), (D) (c) "The Library of Babel"
(c) (C), (A), (B), (D), (E) (d) "The Garden of Forking Paths"
(d) (E), (D), (C), (B), (A) Ans: (c) 'The Library of Babel' is a short story by
Ans: (*) The question remains cancelled and marks Jorge Luis Borges has its epigraph from 'The
awarded to all. Anatomy of Melancholy'.
75. Given below are two statements: One is labelled Borges narrator describes how his universe consists of
as Assertion A and the other is labelled as an enormous expanse of adjacent. Though the order
Reason R. and content of the books are random and apparently
Assertion (A) : Understanding the meaning of any completely meaningless, inhabitants believe that the
cultural form would not simply book contain every possible ordering.
locate it within a specific culture. Though the east majority of the books in this universe
Reason (R) : Cultural forms are best studied in are pure gibberish. The narrator notes that the library
terms of how these fit into the must contain all useful information.
intersection between different The story repeats the theme of Borges 'The Total
cultural networks. Library' which in turn acknowledge the earlier
In the light of the above statements, choose the development of this theme.
most appropriate answer from the options given 78. Arrange in the chronological order of
below: publication:
(a) Both (A) and (R) are correct and (R) is the (A) The Unfinished Man (B) Gitanjali
correct explanation of (A) (C) Jejuri (D) The Sceptred Flute
(b) Both (A) and (R) are correct but (R) is NOT Choose the correct answer from the options
the correct explanation of (A) given below:
(c) (A) is correct but (R) is not correct (a) (B), (A), (D), (C) (b) (D), (B), (C), (A)
(d) (A) is not correct but (R) is correct (c) (B), (D), (A), (C) (d) (B), (D), (C), (A)
Ans: (a) Both (A) and (R) are correct and (R) is the Ans: (c) 'Gitanjali' (1910) is written by Rabindranath
correct explanation of (A). Tagore, he was a Bengali poet, writer, composer,
Culture is characteristics and knowledge of a philosopher and painter. It was published in English
particular group of people, encompassing language, (1912).
religion, cuisine, social habits, music and art……. The 'Gitanjali' or 'Song Offerings' is a collection of poems
culture derives from 'French' term, which in turn translated by the author Rabindranath Tagore from the
derives from the Latin 'colure' which means to tend to original Bengali. This collection won the Noble Prize
the earth and grow or cultivation and nature. for Tagore in 1913.
Culture encompasses religion, food, what we wear, 'The Sceptred Flute' (1943) is the flute song of India.
how we wear it, our language, marriage what we It is written by Indian poet and activist Sarojini Naidu
believe is right or wrong, how we sit at the table, how (1879-1949).
we behave with loved ones. 'The Unfinished Man' (1959) is written by Nissim
76. Which two of the following were published in Ezekiel. Although 'The Unfinished Man' marks a
the year 1859? positive advance over the early poems in terms of
(A) On the Origin of Species quality.
(B) A Tale of Two Cities 'Jejuri' is a series of poems written by Indian poet
(C) Alice in Wonderland Arun Kolatkar. It was first published in opinion
(D) Silas Marner literary quarterly in 1974.

UGC NTA NET/JRF English Paper II, June 2020 Shift-II 325 YCT
79. Who among the following coined the term, 83. Given below are two statements :
'aesthetics'? Statement I : The Education Commission
(a) Arthur Danto (b) Alexander Baumgarten (1964-66) recommended the
(c) Immanuel Kant (d) David Hume removal of English as a medium
Ans: (b) 'Alexander Baumgarten' coined the term of instruction at the college level.
'Aesthetics'. The first use of term aesthetic in Statement II : English in still largely the
something like its modern sense is commonly language of administration and
attributed to Alexander Baumgarten in 1735. jurisprudence in India.
Aesthetics or esthetics is a branch of philosophy that In the light of the above statements, choose the
deals with the nature of beauty and taste, as well as Correct answer from the option given below:
(a) Both Statement I and Statement II are true
philosophy of art. (b) Both Statement I and Statement II are false
It examines subjective and sensory-emotional value or (c) Statement I is correct but Statement II is false
sometimes called judgements of sentiment and taste. (d) Statement I is incorrect but Statement II is true
80. Which agency among the following was of the Ans : (d) English is still largely the language of
view that "use of English…… divides the people administration and jurisprudence in India.
into two nations, the few who govern and the The language used in courts in India has seen a
many who are governed"? transition over centuries with the shift from Urdu to
(a) The Kunzru Committee (1955) Persian and Farsi script during the Mughal period
(b) The Education Commission (1948) which continued in subordinate courts even during the
(c) The Education Commission (1964-66) British rule.
(d) The working Group (UGC) on Regional Constitution of India provides that the official
Languages (1978) language of the union shall be Hindi in the
Ans: (b) The Education Commission (1948) was of Devanagari script.
the view that "use of English ……… divides the 84. Arrange the following 19th Century magazines
people into two nations, the few who govern and the in the chronological order or their publication:
many who are governed. (A) The London Magazine (B) Quarterly Review
The Government of India appoints the Radhakrishnan (C) The Spectator (D) Edinburgh Review
Commission on 4th November 1948 under the Choose the correct answer from the options
chairmanship of Dr. Sarvapalli Radhakrishnan. given below:
It has guided the development of university education (a) (A), (D), (C), (B) (b) (B), (A), (D), (C)
in India in the post-independence period. (c) (D), (B), (A), (C) (d) (C), (D), (B), (A)
81. Which of the following short stories by Edgar Ans: (c) The Edinburgh review was founded in 1802
Allan Poe has a narrator who has a rival with jointly by Sydney Smith, Francis Jeffrey, Francis
the same name and uncanny physical Horner and Henry Brougham.
'The Quarterly Review' was a literary and political
resemblance? periodical founded in March 1809, but the well known
(a) "Hop Frog" London publishing house John Murray.
(b) "William Wilson" The London Magazine first founded in 1732 by Issac
(c) "The System of Doctor Tarr and Professor Kimber but relaunched in 1820 under John Scott's
Father" editorship.
(d) "The Imp of the Perverse" 'The Spectator' is a weekly British Magazine on
Ans: (b) 'William Wilson' (1839) is a short story by politics, culture and current affair. It was first
Edgar Allan Poe. It was first published in 1839, with a published in July 1828, thus making it the oldest
setting inspired by Poe's formative years on the weekly magazine in the world.
outskirts of London. 85. Who among the following critics is said to have
The story follows a man of a noble descent who calls developed the notion of 'interpretive
himself William Wilson because, although communities'?
denouncing his profligate past, he does not accept full (a) Terry Eagleton (b) Jane Tompkins
blame for his actions, saying that 'man was never thus (c) Roland Barthes (d) Stanley Fish
tempted before'. Ans: (d) Stanley Fish is said to have developed the
82. Mr. Pumblechook is a character in: nation of interpretive communities.
(a) Little Dorret (b) Nicholas Nickleby Fish claims that we as individuals interpret texts
(c) Hard Times (d) Great Expectations because each of us is part of an interpretive
community that gives as a particular way of reading a
Ans: (d) Mr. Pumblechook is a character in 'Great text. The idea has been very influential in reader-
Expectations', written by Charles Dickens. response criticism, though it has also been very
Dickens is justly famous for his vivid secondary controversial.
character and for their names, which are often 86. Match List I with List II
deliciously absurd, like that of Mr. Pumblechook. Mr. List I List II
Pubblechook is introduced to us as a large hard- Terms Theorists
breathing middle-aged slow man. He is the uncle of (A) Heteroglossia (i) Michel Foucault
gentle Joe Gargery, Pip's brother-in-law and father (B) Heterotopia (ii) Louis Althusser
figure. Since Mr. Pumblechook has some social clout, (C) Grand Narrative (iii) Mikhail Bakhtin
he is appropriated by Pip's sister who shares Mr. (D) Interpellation (iv) Jean-Francois
Pumblechook's aggressive self-righteousness. Lyotard

UGC NTA NET/JRF English Paper II, June 2020 Shift-II 326 YCT
Choose the correct answer from the options 89. Which of these following statements are true
given below : about Pidgin and Creole?
(a) (A)-(ii), (B)-(i), (C)-(iv), (D)-(iii) (A) Pidgin begins as Creole and eventually
(b) (A)-(iii), (B)-(ii), (C)-(iv), (D)-(i) becomes the first language of a speech
(c) (A)-(iii), (B)-(i), (C)-(iv), (D)-(ii) community
(d) (A)-(iv), (B)-(i), (C)-(iii), (D)-(ii) (B) Creole begins as Pidgin and eventually
Ans: (c) The term heteroglossia describes the becomes the first language of a speech
coexistence of distinct varieties within a single community
'language' which was introduced by the Russian
(C) Pidgin is simple but a rule governed language
literary theorist Mikhail Bakhtin.
developed for communication whereas Creole
Heterotopia is a concept elaborated by philosopher
Michel Foucault to describe certain culture, in free from grammatical rules
institutional and discursive spaces. (D) Pidgin and Creole evolve successively out of a
Grand Narrative or 'Master narrative' is a term situation where speakers of mutually
introduced by Jean Francois Lyotard, in his classic unintelligible languages develop a shared
1979 work 'The Postmodern Condition : A Report on language for communication (often based on
Knowledge'. one of those languages)
Interpellation describes the process by which (a) (A) and (B) only (b) (B) and (D) only
ideology, embodied in major social and political (c) (C) and (D) only (d) (A) and (D) only
institutions, constitutes the very nature of individual Ans: (b) Pidgin and Creole evolve successively out of
subjects. a situation where speakers of mutually unintelligible
87. Which two of the following citations conform to language develop a shared language for
the documentation format of the eighth edition communication.
of the MLA Hand book?
Pidgin is a simplified speech used for communication
(a) Baron Naomi S. "Redefining Reading: The
between people with different languages, its
impact of Digital Communication Media".
PMLA vol 128, no. 1, Jan 2013 PP. 193-200. vocabulary and grammar are limited and often drawn
(b) Adichie, Chimamanda Ngosi. "On Monday of from several languages.
Last Week", The Thing Around Your Neck. Creole language, vernacular language that developed
London Knopf, 2009. 74-94 in colonial European plantation settlements in the 17th
(c) Baron Naomi S. "Redefining Reading: The and 18th centuries as a result of contact between
impact of Digital communication Media". groups that spoke mutually unintelligible languages.
PMLA 128. 1 (2013): 193-200. 90. Who among the following held that "the people
(d) Adichie, Chimamanda Ngosi "On Monday of of Hindustan" are "a race of men lamentably
Last Week". The Thing Around Your Neck, degenerate and base, retaining but a feeble sense
Alfred A. Knopf, 2009, PP. 74-94 of moral obligation…"?
Ans: (*) The question remains cancelled and marks (a) Charles Wilkins (b) Thomas Macaulay
awarded to all. (c) Charles Grant (d) David Hare
88. Which two of the following meanings are
Ans: (c) Charles Grant, chairman of the court of
admissible for the following sentences:
"You do not know how good oysters taste" Directors condemened the people of India as "a race
(A) You do not know that oysters taste good as of men lamentably degenerate and base; retaining but
food a feeble sense of moral obligation; and sunk in misery
(B) You do not know how the oysters taste when but their vices."
cooked Read the following passage and answer the questions
(C) You do not know what the oysters taste when that follow:
they eat WHEN I'M ALONE
(D) You do not know how the good oysters taste 'When I'm alone' – the words tripped off his tongue
when they eat As though to be alone were nothing strange.
Choose the most appropriate answer from the 'When I was young', he said, 'when I was young….'
options given below: I thought of age, and loneliness, and change,
(a) (A) and (D) only (b) (B) and (C) only
I thought how strange we grow when we're alone,
(c) (B) and (D) only (d) (C) and (D) only
And how unlike the selves that met, and talk,
Ans: (a) "You do not know how good oysters taste",
And blow the candles out, and say good-night,
in this sentence, There are two (A and D) sentences
admissible– Alone….The word is life endured and known.
(A) You do not know that oysters taste good as food. It is the stillness where our spirits walk
(D) You do not know how the good oysters taste when And all but in most faith is overthrown.
they eat. SIEGFRIED SASSOON

UGC NTA NET/JRF English Paper II, June 2020 Shift-II 327 YCT
91. For the poet, 'Being alone' is a condition Read the following passage and answer questions (96-98):
conducive to I do entreat your grace to pardon me.
(a) happiness of the self I know not by what power I am made bold,
(b) becoming different from others Nor how it may concern my modesty.
(c) growing up in an unexpected way In such a presence here to plead my thoughts:
(d) thinking in a strange way But I beseech your grace that I may know
Ans: (c) For the poet 'Being alone' is a condition The worst that may be fall me in this case.
conducive to growing up in an unexpected way. If I refuse to wed Demetrius.
92. For the speaker of the words 'When I'm alone', – A Midsummer's Night Dream
being alone is 96. Who was in love with Demetrius?
(a) The normal fate of a human being all his life (a) Hippolyte (b) Helena
(b) The normal fate of a human being when he is
young (c) Thisbe (d) Hermia
(c) Not unlike being with others whom we meet Ans : (b) 'A Midsummer Night Dream' is a comedy
(d) Not strange as a person should feel alone written by William Shakespeare. Demetrius, a young
Ans: (a) For the speaker of the words 'When I'm man of Athens, initially in love with Hermia and
alone' being alone is the normal fate of a human being ultimately in love with Helena.
all his life. 97. The above lines are addressed to
93. Which two of the following statements aptly (a) Theseus (b) Egeus
captures the meaning of 'Alone' for thinking (c) Oberon (d) Plilostrate
beings? Ans: (a) The above lines – I do entreat your grace
(A) Meeting talking and bidding goodnight ………………. to wed Demetrius, are spoken by
(B) Quietude and calmness of self Hermia addressed to Theseus in Act I Scene I.
(C) Life lived and understood Hermia : I would my father looked but with my eyes.
(C) Becoming free from faith
Choose the correct answer from the options Theseus : Rather your eyes must with his judgement
given below: look.
(a) (A) and (B) only (b) (B) and (C) only Hermia – I do entreat your grace ---------.
(c) (C) and (A) only (d) (D) and (B) only 98. Who is the speaker of the above lines?
Ans: (b) The statement 'quietude and calmness of self (a) Helena (b) Thisbe
and life lived and understood' aptly captures the (c) Peasblossom (d) Hermia
meaning of 'Alone' for thinking beings. Ans: (d) The above is spoken by Hermia. See the
Read the following passage and answer questions that explanation of question no. 97.
follow: Read the following and then answer the questions
Poetry, as a mania – one of Plato's two higher forms of that follow:
"divine" mania – has, in all its species, a mere insanity He went to work in this preparatory lesson, not unlike
incidental to it, the "defect of its quality", into which it Morgiana in the Forty Thieves: looking into all the
may lapse in its moment of weakness; and the insanity vessels ranged before him, one after another, to see what
which follows a vivid poetic anthropomorphism like that they contained. Say, good M'Choakumchild. When from
of Rossetti may be noted here and there in his work, in a
forced and almost grotesque materializing of thy boiling store, thou shalt fill each jar brim full by" and
abstractions, as Dante also became at times a mere by; dost thou think that thou wilt always kill outright the
subject of the scholastic realism of the Middle Age. robber Fancy lurking within – or sometimes only maim
- Walter Pater him and distort him.
94. In the above passage poetry is described as one Dickens Hard Times
of Plato's two higher forms of 'divine' madness. 99. In the expression "…… looking into all the
Which is the other one? vessels ranged before him….", which one of the
Choose the correct option? following devices is used?
(a) Beloved (b) Love (a) Synecdoche (b) Metonymy
(c) Jealously (d) Lover (c) Metaphor (d) Simile
Ans: (b) Poetry is described as one of Plato's two Ans: (c) In the expression "…….. looking into all the
higher forms of divine madness and love. vessels ranged before him….." metaphor is used.
95. In Rossetti, the forced personifications may be: Metaphor is an implicit as implied comparison
(A) an incidental defect of poetic quality between two very dissimilar thing.
(B) examples of a madness of thought 100. 'Fancy' is opposed to which two of the
(C) an exaggerated concretization of things following?
(D) a divinely inspired poetic expression (A) Emotion (B) Reason
Choose the most appropriate answer from the (C) Fact (D) Imagination
options given below:
(a) (A) and (B) only (b) (D) and (C) only Choose the correct answer from the options
(c) (B) and (A) only (d) (C) and (A) only given below:
(a) (B) and (C) only (b) (C) and (D) only
Ans: (d) In Rossetti; the forced personifications may
be an exaggerated concretization of things and an (c) (A) and (C) only (d) (B) and (D) only
incidental defect of poetic quality. Ans: (a) Fancy is opposed to reason and fact.

UGC NTA NET/JRF English Paper II, June 2020 Shift-II 328 YCT
UGC NTA NET/JRF Exam, June-2020
ENGLISH
Solved Paper-II (Shift-I)
1. The deductive method differs from the inductive 4. Which one of the following is correct Saussure's
method in drawing its conclusions from analysis of language?
(a) Verification (b) Particular instances (a) La langue is the system of a language
(c) Applications (d) General Truths (b) Parole focuses on language as a system at a
Ans: (d) Deductive method: starts from a theory. To particular time
prove it, we use some information, to test that theory. (c) La langue is the particular instance of speech
This method usually termed as general-to-specific and writing
approach. (d) Parole is study of language over a period of
Inductive method: starts from specific to general time
(theory). We reach to some conclusion from some Ans: (a) Saussure says that language is really a
specific point to generalize. This method is also called borderland between thought and sound, where thought
as 'Bottom-Up' approach. and sound combine to provide communication.
2. Which one of the following journals publishes Spoken language includes the communication of
articles related to critical theory exclusively? concepts by means of sound-images from the speaker
(a) Salmagundi (b) Diacritics to the listener. Language must be considered as a
(c) Callaloo (d) Grand Street social phenomenon, a structure system that can be
Ans: (b) (i) Salmagundi– is a US quarterly viewed synchronically (as it exists at any particular
periodical, featuring cultural criticism, fiction and time). Saussure's theory of language, the linguistic
poetry, along with transcripts of symposia and system in each individual's brain is constructed from
interviews with prominent writers and intellectuals. experience.
(ii) Diacritics– is a quarterly peer-reviewed academic 5. Who among the following theorists particularly
journal established in 1971. This journal publishes emphasizes the social and historical dimensions
original work in around critical theory, broadly of a texts reception?
conceived. (a) Wolfgang Iser (b) Stanley Fish
(iii) Callaloo– The premier journal of literature, art (c) Hans Robert Jauss (d) Pierre Bourdieu
and culture of the African Diaspora, publishes original
work by and about writers and visual artistsof African Ans: (c) Reception theory is a version of reader
descent-worldwide. It was established in 1976 by response literary theory that emphasized each
Charles Rowell. particular reader's reception or interpretation in
(iv) Grand Street– was an American magazine which making meaning from a literary text. Hans Robert
appeared from 1981 to Fall 2004. It was described by Jauss's version of reception theory was introduced in
The New York Times as "one of the most revered late 1960s, a period of social, political and intellectual
literary magazines of the postwar era". instability in West Germany. Jauss's reception theory
focused on the reader rather than the author or text.
3. Which one of the following assumptions best
expresses the position of Post-Structuralism 6. Which one among the following is a set of the
criticism? Metaphysical Poets?
(a) Definite structures underlie empirical events (a) John Dryden, George Herbert and Alexander
(b) Language is representational Pope
(c) Apprehension of reality is a construct (b) Henry Vaughan, John Dryden and John Donne
(d) Knowledge operates according to procedures (c) John Donne, Henry Vaughan and Andrew
that are axiomatic Marvell
Ans: (c) Post-Structuralism is a late-twentieth century (d) Samuel Johnson, T.S. Eliot and Herbert
development in philosophy and literary theory, Grierson
particularly associated with the work of Jacques Ans: (c) The term 'Metaphysical poet' was coined by
Derrida. It originated as a reaction against the critic Samuel Johnson. Metaphysical poetry is
structuralism, which first emerged in Ferdinand de marked by the use of elaborate figurative languages,
Saussure's work on linguistics. By the 1950s original conceits, paradoxes and philosophical topics.
structuralism had been adapted in anthropology (Levi- The best known metaphysical poet is John Donne. He
Strauss). Psychoanalysis (Lacan) and Literary theory is followed by others such as Henry Vaughan,
(Barthes), and there were hopes that it could provide Andrew Marvell and George Herbert.
the framework for rigorous account in all areas of the 7. Which one of the following statements is true
human sciences. Post-structuralism decenters the about Aristotle's poetics?
subject. The deletion of the author leave the readers as (a) He asserted the value of poetry by integrating
the focal point of post-structural criticism. An rhetoric and imitation (mimesis)
interesting aspect of post-structuralism is the currency (b) He asserted the value of poetry by focusing on
of the term "discourse". both rhetoric and imitation (mimesis)
UGC NTA NET/JRF English Paper II, June 2020 Shift-I 329 YCT
(c) He asserted the value of poetry by giving relationship they necessarily maintain with precursor
preference to rhetoric over imitation (mimesis) poets.
(d) He asserted the value of poetry by focusing on Clinamen– Bloom defines this as "poetic misreading
imitation (mimesis) rather than rhetoric or misprision proper". He took the word 'Clinamen'
Ans: (d) In the opening sentence of the 'Poetics' from "Lucretius", who refers to swerves of atoms that
Aristotle tells us that he is going to deal with poetry makes change possible.
itself, its kind and their powers and so on. He then 11. Who among the following is known to have
turns to a discussion of imitation or representation. popularized the term 'glocalization'?
Aristotle suggests to distinguish poetry from painting,
(a) Ronald Robertson (b) Francis Fukuyama
music and dancing which are also imitation. So the
statement the value of poetry by focusing on imitation (c) John Urry (d) John Tomlinson
(mimesis) rather than rhetoric is true. Ans: (a) Glocalization is a combination of the words
8. Who said of the blank verse quoting an "globalization" and "localization". The term is used to
unnamed critic that it is "…..verse only to the describe a product or service that is developed and
eye", adding further that it "has neither the distributed globally but is also adjusted to
easiness of prose, nor the melody of numbers"? accommodate the user or consumer in a local market.
(a) John Dryden The term was popularized by the sociologist Roland
(b) Alexander Pope Robertson and coined according to him, by Japanese
(c) Samuel Taylor Coleridge economist to explain Japanese global marketing
(d) Samuel Johnson strategies.
Ans: (d) Samuel Johnson wrote Lives of the Most 12. Who among the following coined the dictum,
Eminent English Poets in 1779-81. In this, he "the medium is the message"?
comprises the quote while describing about Milton.
(a) Raymond Williams (b) Erving Goffman
Milton wrote the lines as the ending paragraph of his
Paradise Lost. Johnson describes that the variety of (c) Marshall McLuhan (d) John Fiske
pauses, so much boasted by the lovers of blank verse, Ans: (c) "The medium is the message" is a phrase
changes the measures of an English Poet to the coined by the Canadian communication theorist
periods of a disclaims; and these are only a few skilful Marshall McLuhan and introduced in his
and happy readers of Milton who enable 'Blank Understanding Media: The Extensions of Man,
Verse', said an ingenious critic, 'seems to be verse published in 1964. McLuhan proposes that a
only to the eye'. communication medium itself, not the messages it
9. Poetry according to Sir Philip Sidney is of three carries, should be the primary focus of study. He
kinds. They are : showed that artifacts as media affect any society by
(a) religious dramatic romantic their characteristics, or content. McLuhan uses the
(b) classical romantic neo-classical term 'message' to signify content and character. The
(c) philosophical imaginative narrative content of the medium is a message that can be easily
(d) religious philosophical imaginative
grasped and the character of the medium is another
Ans: (d) Sidney penned several major works of the message which can be easily overlooked.
Elizabethan era, including 'Astrophel and Stella', the
first Elizabethan sonnet cycle and Arcadia, a heroic 13. Who among the following presented the concept
prose romance. He was also known for his literary of 'multi-accentuality' of the sign, saying that
criticism, known as The Defense of Poesy. Sidney signs possess an 'inner dialectical quality' and
has described about 'Poetry' in his work 'An Apology 'evaluative accent'?
for Poetry' and describes that poetry is of three kinds. (a) Roland Barthes (b) Stuart Hall
They are: (a) Religious poetry (b) Philosophical (c) Jacques Derrida (d) Valentin Voloshinov
Poetry (c) Poetry as an imaginative treatment of life Ans : (d) Valentin Mikolaevich Voloshinov, was a
and nature. He also describe poetry as an art of Soviet/Russian linguist, whose work has been
imitation, a representing, counterfeiting or figuring influential in the field of literary theory and Marxist
forth; to speak metaphorically, a speaking picture,
theory of ideology. In Voloshnov's view, the meaning
with this end – to 'teach and delight'.
of verbal signs is the arena of continuous class-
10. In Anxiety of Influence which of the following struggle : a ruling class will try to narrow the meaning
definitions is given by Harold Bloom to explain
the term, 'clinamen'? of social signs, making them "uni-accentual", but the
(a) poetic hyperbole (b) poetic misprision clash of various class-interests in times of social
(c) poetic sublime (d) poetic supplement unrest will make clear the "multi-accentuality" of
Ans: (b) 'The Anxiety of Influence : A Theory of words. By virtue of his belief that the "struggle for
Poetry' is a 1973 book by Harold Bloom. It was the meaning" coincides with class struggle, Voloshinov's
first in a series of books that advanced a new theories have much in common with those of Italian
"revisionary" or antithetical approach to literary Communist Antonio Gramsci, who shared an interest
criticism. Bloom's central thesis is that poets are in linguistics. Voloshinov's work can also be seen to
hindered in their creative process by the ambiguous prefigure many of the concerns of post-structuralism.

UGC NTA NET/JRF English Paper II, June 2020 Shift-I 330 YCT
14. On December 11, 1823 Rammohan Roy 1. between life and death
addressed a letter to the British authority which 2. between action and emotion
pleaded for modern western education and is 3. between affirmation and confirmation
considered historically important for the 4. between doing and abstaining from doing
introduction of English education in India. Who Choose the correct answer from the options
was the letter addressed to? given below :
(a) Lord Amherst (b) Lord Minto (a) 1 and 4 only (b) 2 and 4 only
(c) Lord Macaulay (d) Lord Bentick (c) 3 and 1 only (d) 4 and 3 only
Ans: (a) Ram Mohan Roy was an ardent advocate of Ans: (a) 'To be or not to be' is the opening phrase of a
western education through the medium of English. soliloquy uttered by Prince Hamlet – in the so called
Then in 1821, Mr. H.H. Wilson, a great orientalist
"nunnery scene" of the play Hamlet, Act 3, Scene 1.
rooted the idea of establishing a Sanskrit College in
Calcutta. Ram Mohan opposed this proposal. In the Hamlet says because he is questioning the value of
later, dated 11 December 1823, Lord Amherst, he life and asking himself whether it's worthwhile
wrote "we now find that the government is hanging in these. He is extremely depressed at this
establishing a Sanskrit school under Hindu Pundits to point and fed up with everything in the world around
impart knowledge as is already current in India…..". him and he is contemplating putting an end to himself.
15. Which British administrator sought "to make 19. Who among the following linguists proposed the
everything as English as possible in a country terms 'competence and 'performance'?
which resembles England in nothing", as (a) Noah Webster (b) Steven Pinker
recorded by Sir Thomas Munro? (c) Roman Jakobson (d) Noam Chomsky
(a) Lord Bentick (b) Lord Hastings Ans: (d) A distinction introduce by Chomsky in to
(c) Lord Cornwallis (d) Lord Wellesley linguistic theory but of wider application. Competence
Ans: (c) Sir Thomas Munro, Ist Baronet was a refers to a speaker's knowledge of his language as
Scottish soldier recorded the given lines about British manifest in his ability to produce and to understand a
administrator Lord Cornwallis. theoretically infinite number of sentences most of
Lord Cornwallis was British Army general and which he may never seen or heard before.
official. In the United States and the United Kingdom Performance refers to the specific utterances including
he is best remembered as one of the leading British grammatical mistakes and non-linguistic features like
generals in the American War of Independence. In hesitations accompanying the use of language.
India, he helped to enact the Cornwallis Code and the
Permanent Settlement. 20. Which one of these statements defines the scope
of semiotics?
16. Who among the following was the first Director (a) Semiotics studies the sound systems of a
of the Central Institute of English and Foreign
Languages, Hyderabad (now EFL University)? language
(a) Prof. V.K. Gokak (b) Semiotics is a study of sign systems
(b) Prof. C.D. Narasimhaiah (c) Semiotics studies human sign system only
(c) Prof. C.J. Daswani (d) Semiotics is a study of non-human sign
(d) Prof. K. R. S. Iyengar systems only
Ans: (a) English and Foreign Languages University Ans: (b) Semiotic literary criticism, also called
known as EFLU. It is a university for English and literary semiotics, is the approach to literary criticism
Foreign language located in Hyderabad. EFLU was informed by the theory of signs or semiotics.
founded in 1958 as the Central Institute of English by Semiotics is a key tool to ensure that intended
Pandit Jawaharlal Nehru. Prof. V.K. Gokak was the meanings (of for instance a piece of communication or
first director of the central Institute of English and a new product) are unambiguously understood by the
Foreign Languages, Hyderabad. person on the receiving end. A semiotic system, in
17. Which one of the following best explains the conclusion, is necessarily made of at least three
term 'paralanguage'? distinct entities: signs, meanings and code.
(a) The ways in which people ask what they mean 21. Which according to Thomas Hobbes is the only
by the words they use 'science' God has bestowed on mankind, that
(b) The ways in which people show what they informs the structure of his monumental work
mean other than by the words they use Leviathan?
(c) The ways in which words carry meanings (a) Astronomy (b) Architecture
unintended by the speaker (c) Occult sciences (d) Geometry
(d) The ways in which the silence underlying
speech communicates wrong meanings Ans: (d) Leviathan or The Matter, Forme and Power
Ans: (b) Paralanguage is the non-lexical component of a Commonwealth Ecclesiastical and Civil is a book
of communication by speech, for example intonation, written by Thomas Hobbs and published in 1651. In
pitch and speed of speaking, hesitation noises, gesture, this books chapter (IV) 'Of Speech' Thomas Hobbs
and facial expression. has given the reference – "And therefore in geometry,
18. Which two of the following oppositions are best (which is the only science that it hath pleased God
evoked by Hamlet's utterance – "To be or not to hitherto to bestow of man kind), men begin at settling
be"? the significations of their words………"

UGC NTA NET/JRF English Paper II, June 2020 Shift-I 331 YCT
22. As mentioned in "My First Acquaintance with 26. What is the subject of Ivan's controversial essay
Poets" which poet does William Hazlitt describe in Brothers Karamazov?
as the "only person I ever knew who answered (a) Transubstantiation (b) The evils of clergy
the idea of a man of genius"? (c) The Eucharist (d) Ecclesiastical courts
(a) Coleridge (b) Wordsworth Ans: (d) "The Brothers Karamazov" is the final novel
(c) Byron (d) Shelley by Russian author Fyodor Dostoevsky. Fyodor
Ans: (a) "My First Acquaintance with Poets" is an Dostoevsky's final novel, The Brothers Karamazov,
essay written by William Hazlitt in The Liberal in has rightly earned its place among the greatest books
1823. As a painter paints a picture portraying all the of all time. In the novel when Zasima returns to his
nuances on his canvas, Hazlitt depicts all the minute cell, Fyodor is relishing his attack on Miusov, who
details of the Romantic age and gives a beautiful flatters himself as an intellectual in the enlightened,
depiction of two representative bards of the age. Hazlitt European style. The conversation turns to Ivan's
gives a comprehensive sketch of a period in the life of recent editorial on the ecclesiastical courts, the one
William Wordsworth and Samuel Taylor Coleridge. that caused such a scandal before he left Moscow to
23. Which one of the following essays holds that "As return to his father's home.
a method, realism is a complete failure"? 27. Which one of the following Sherlock Holmes
(a) Virginia Woolf, "The Mark on the Wall" stories refers to a significant event in English
(b) Oscar Wilde, "The Decay of Lying" history?
(c) D. H. Lawrence, "Why the Novel Matters" (a) "The Musgrove Ritual"
(d) Mary McCarthy, "My Confession" (b) "The Speckled Band"
Ans: (b) "The Decay of Lying – An Observation" is (c) "The Solitary Cyclist"
an essay by Oscar Wilde included in his collection of (d) "The Red-Headed League"
essays titled Intentions Annotated, published in 1891.
Ans: (a) "The Adventure of the Musgrove Ritual" is a
This is a significantly revised version of the article
that first appeared in the January 1889 issue of the short story by Arthur Conan Doyle, featuring his
Nineteenth Century. Some of the important quotes are– fictional detective Sherlock Holmes. The story was
originally published in 'The Strand Magazine" in the
→ "Life imitates Arts for more than art imitates life".
United Kingdom in May 1893, and in Harper's weekly
→ "Paradox though it may seem – and paradoxes are
in the United States on 13th May 1893. It was
always dangerous things – it is none the less true that
life imitates arts for more than art imitates life". collected in The Memoirist of Sherlock Holmes. The
Adventure of the Musgrove Ritual shares elements
24. Which of the following novels is structured into with two Edgar Allan Poe Tales : "The Gold Bug"
a poem of 999 lines preceded by a Foreword, and "The Cask of Amontillado".
followed by a Commentary and an Index?
(a) Ragtime (b) Pale Fire 28. Harold Skimpole is a character in :
(c) The Inner Side of the Wind (d) Hourglass (a) Bleak House (b) Dombey and Son
(c) Great Expectations (d) Oliver Twist
Ans: (b) "Pale Fire" is a 1962 novel by Vladimir
Nabokov. It consists of a long poem and a Ans: (a) Bleak House is a novel by Charles Dickens,
commentary on it by an insane pedant. The 999 line first published as a 20-episode serial between March
poem in heroic couplets upon which the novel is 1852 and September 1853. The novel has many
centred is the masterwork of an academic, John characters and several sub-plots and is told partly by
Shade. The Nabokov authority Brian Boyd has called the novels heroine, Esther Summerson and the partly
it "Nabokov's most perfect novel", and the critic by omniscient narrator Harold Skimpole who is a
Harold Bloom called it "The surest demonstration of friend of Jarndyce "in the habit of sponging his
its own genius". friends" (Nuttall). He is irresponsible, selfish, amoral
25. Which among the following novels includes a and without remorse.
questionnaire for the reader such as "Do you 29. Who is the author of "A Fragment" (1819), one
like the story so far? Yes () No ()"? of the earliest vampire stories in English?
(a) Mantissa by John Fowles (a) P.B. Shelley (b) Lord Byron
(b) Waterland by Graham Swift (c) Bram Stoker (d) Mary Shelley
(c) Snow White by Donald Barthelme Ans: (b) "A Fragment of a Novel" is an unfinished
(d) If on a Winter's Night a Traveller by Italo 1819 vampire horror story written by Lord Byron. The
Calvino story also known as "A Fragment" and "The Burial :
Ans: (c) "Snow White" is a post modernist novel by A Fragment", was one of the first in English to feature
author Donald Barthelme published in 1967 by a vampire theme. The main character was Augustus
Athenaeum Books. The book inverts the fairy tale of Darvell. John William Polidori based his novella 'The
the same name by highlighting the form by discussing Vampyre" (1819) originally attributed in print to Lord
the different expectations and compromises the Byron himself. The story was the result of the meeting
characters makes to survive in their world. This is that Byron had in the summer of 1816 with Percy
done through Barthelme's fragmentary rhetoric and Bysshe Shelley where a "ghost writing" contest was
discourse, by shifting perspectives from the seven proposed. This contest was also what led to the certain
"dwarves" or Snow White herself, as well as the of Frankenstein according to Shelley's 1818 Preface
wicked step-mother "Jane". to the novel.

UGC NTA NET/JRF English Paper II, June 2020 Shift-I 332 YCT
30. Lala Kanshi Ram is a character in : Ans: (b) "Mazeppa" is a narrative poem written by
(a) Arun Joshi's The Apprentice Lord Byron in 1819. It is based on a popular legend
(b) Chaman Nahal's Azadi about the early of Ivan Mazepa (1639-1709), who
(c) Anita Desai's Fire on the Mountain later becomes Hetman (military leader) of Ukraine,
(d) Kamala Markandaya's A Handful of Rice The Cultural Legacy of Mazeppa was revitalized
Ans: (b) Chaman Nahal commonly known as C. with the independence of Ukraine in 1991.
Nahal, also known as Chaman Nahal Azadi, was an
Indian born writer of English Literature. He was 35. Which one of the following statements is
widely considered as one of the best exponents of appropriately true of Harold Pinter's plays?
Indian writing in English and is known for his work, (a) Menace is in the air and it leads to bloody
Azadi, which is set on India's Independence and her violence
partition. He is also known for his depiction of Gandhi (b) Menace is in the air and it is realized through
as a complex character with human failings. the female characters
31. Which of the following poems by Philip Larkin (c) Menace is in the air, but it is not pinned down,
deals with the trauma of a rape victim who says or explained
"Even so distant, I can taste the grief"? (d) Menace is in the air and anarchy follows in a
(a) "Deceptions" (b) "Faith Healing" systematic manner
(c) "Sad Steps" (d) "Wild Oats" Ans: (c) Harold Pinter is known for his magnificent
Ans: (a) "Deceptions" is written by Larkin after use of language, thus his style of writing was named
reading about an incident of a rape of a girl. He used a after him "Pinteresque". His use of colloquial
rape victim's statement from a magazine named language, numerous cliché's, unpolished grammar and
"London Labour and the London Poor" of the 1840s illogical syntax create dialogues that reflect day-to-
as an epigraph. Larkin feels the agony suffered by the day speech. 'A comedy of menace' is a play in which
victim and consoles her. 'Deceptions' is a two stanza the laughter of the audience in some or all situations is
poem that is made up of one set of nine lines and
immediately followed by a feeling of some impending
another set of eight.
disaster. 'A Pinter Pause' is a radical device that Pinter
32. In which of the Bog poems does Seamus Heaney frequently incorporated into his plays. In Pinter script,
speak about the "perishable treasure" of a body
an eclipse is denoted by three dots and was used by
"Murdered forgotten, nameless, terrible"?
(a) "Bog Queen" (b) "Grauballe Man" the playwright to indicate slight hesitation.
(c) "Punishment" (d) "Strange Fruit" 36. To which mythological character is Faustus
Ans: (d) In "Strange Fruit" the last of Seamus compared in the Prologue of Dr. Faustus?
Heaney's bog poem, the body of a murdered bog girl (a) Perseus (b) Theseus
is also presented as a document of ancient violence (c) Icarus (d) Achilles
and human cruelity. Ans: (c) 'The Tragical History of the Life and Death
"Here is the girl's head of Doctor Faustus', is an Elizabethan tragedy by
Murdered, forgotten, nameless terrible, Christopher Marlowe, based on German stories about
Beheaded girl, out staring axe …..". the title character. The story's protagonist was
33. Which book of Paradise Lost incorporates the compared to the Greek myth of Icarus, a boy whose
speech rhythms of Adam and Eve's marital father, Daedalus, gave him wings made out of feathers
quarrel? and beeswax. Dr. Faustus can be categorized as a
(a) Book 4 (b) Book 6 hero, only he would be a Tragic Hero. Dr. Faustus has
(c) Book 7 (d) Book 9 elements of both Christian morality and classical
Ans: (d) "Paradise Lost" is an epic poem in blank tragedy but deep down the story is a tragedy in
th
verse by the 17 century English poet John Milton. Christian terms, because he gives in to temptation and
After tempted by Satan Eve eats the fruit of forbidden
is damned to hell.
tree. Eve's first fallen thought is to find Adam and to
have him eat of the forbidden fruits to so that they 37. Who makes the following speech in Samuel
might be equal. She finds him nearby. Adam realizes Beckett's Waiting for Godot?
that if she is to be doomed, then he must follow. He "Astride of a grave and a difficult birth. Down
eats the fruit. In Book IX Milton explains that their in the hole, lingeringly, the grave-digger puts on
appetite for knowledge has been fulfilled, and their the forceps."
hunger for God has been quenched. Angry and (a) Estragon (b) Lucky
confused, they continued to blame each other for (c) Vladimir (d) Pozzo
committing the sin, while neither will admit and fault. Ans: (c) "Waiting for Godot" is a play by Samuel
Their shameful and tearful argument continues for Beckett in which two characters, Vladimir (Didi) and
hours. Estragon (Gogo), engage in a variety of discussions
34. Who among the following wrote Mazeppa, a long and encounters while awaiting Godot, who never
narrative poem about a seventeenth-century arrives. It is Beckett's translation of his own original
military leader of Ukraine? French-language play. This speech "Astride of a grave
(a) William Cowper (b) Lord Byron
and a difficult birth……" is spoken by Vladimir.
(c) P.B. Shelley (d) S. T. Coleridge
UGC NTA NET/JRF English Paper II, June 2020 Shift-I 333 YCT
38. Which of the following are the major themes in Choose the correct answer from the options
William Congreve's The Way of the World? given below:
(a) jealousy and revenge (a) 1 and 2 only (b) 1 and 3 only
(b) love and intrigue (c) 1 and 4 only (d) 2 and 3 only
(c) intrigue and death Ans: (c) The Modern Language Association, the
(d) love and loyalty authority on research and writing, takes a fresh look at
Ans: (b) "The Way of the World" is a play documenting source in the eight edition of the MLA
premiered in early March 1700 in the theater in Handbook. The Future of the Book and Kiss of the
Lincoln's Inn Fields in London. It is widely regarded Spiderwoman, Translated by Thomas Colchie strictly
as one of the best Restoration comedies. The setting of follow the parameters of documentation prescribed by
play is in London. In many ways, the play can be the eighth edition of the MLA Handbook.
thought of as a competition between Mirabell and 42. A research hypothesis is :
Fainall to deceive the other by means of opposing 1. a proposition which is always true
schemes to gain control of Lady Wishfort and her 2. a provisional explanation of anything
fortune. So the basic theme is of Jealousy, Deceit and 3. a theory which will be disproved by evidence
Intrigue. The title is a flippant expression meaning the 4. a statement which is assumed to be true for
way people behave. the sake of argument
39. What game do the characters play in Act II of Choose the most appropriate answer from the
Harold Pinter's The Birthday Party? options given below:
(a) A game of chess (b) A game of cards (a) 1 and 2 only (b) 2 and 3 only
(c) Blind man's buff (d) Musical chairs (c) 2 and 4 only (d) 1 and 3 only
Ans: (c) "The Birthday Party" is the first full length Ans: (c) A hypothesis is a concept or idea that you
play by Harold Pinter, in 1957. Pinter began writing test through research and experiments. In other words,
'The Birthday Party' in the summer of 1957 while it is a prediction that can be tested by research. If you
touring in Doctor in the House. He later said : "I want to test a relationship between two or more
remember writing the big interrogation scene in a things, you need to write hypothesis before you start
dressing room in Leicester". In the play Goldberg, your experiment or data collection. So it can be said
McCann, Lulu, Meg and Stanely decided to play that 'A research hypothesis is a specific, clear and
'blind man's buff' game. testable preposition or predictive statement about the
40. The Duchess Malfi is based on : possible outcome of a scientific research study based
(a) a French romance (b) an Italian novella on a particular property of a population, such as
(c) a Geman fable (d) a Scottish chronicle presumed differences between groups on a particular
Ans: (b) "The Duchess of Malfi" is a Jacobean variable or relationship between variables'.
revenge tragedy written by John Webster in 1612- 43. Which two of the following aspects are to be
1613, published in 1623, the play is loosely based on scrupulously followed to avoid the trap of
events that occurred between 1508 and 1513, plagiarism?
surrounding Giovanna d'Aragona, 'Duchess of Malfi' 1. subjectivity
(d. 1511), whose father, Enrico d'Aragona, Marquis of 2. acknowledgement
Gerace, was an illegitimate son of Ferdinand I of 3. citation
Naples. As in the play, she secretly married Antonio 4. interpretation
Beccadelli di Bologna after the death of her first Choose the most appropriate answer from the
husband Alfonso I Piccolomini, Duke of Amalfi. options given below:
Jacobean drama continued the trend of stage violence (a) 1 and 2 only (b) 1 and 3 only
and horror set by Elizabethan tragedy, under the (c) 3 and 4 only (d) 2 and 3 only
influence of Seneca. It is based on an Italian novella. Ans: (d) To avoid plagiarism in research work it is
41. Which two of the following strictly follow the very important to acknowledge the work of others
parameters of documentation prescribed by the with proper citation in the Bibliography section of the
eighth edition of the MLA Handbook? thesis.
1. Nunberg Geoffrey, editor. The Future of the 44. Which two texts among the following are linked
Book. U of California P, 1996. to literary feminism?
2. Puig, Manuel. Kiss of the Spider Woman. 1. A Small Place
Trans. Thomas Colchie, London: Vintage, 2. The Yellow Wallpaper
1991. 3. Emma
3. Nunberg Geoffrey, ed. The Future of the Book. 4. A Room of One's Own
Berkeley: U of California P, 1996. Choose the correct answer from the options
4. Puig, Manuel, Kiss of the Spider Woman. given below:
Translated by Thomas Colchie, Vintage Books, (a) 1 and 4 only (b) 3 and 4 only
1991. (c) 2 and 4 only (d) 1 and 3 only
UGC NTA NET/JRF English Paper II, June 2020 Shift-I 334 YCT
Ans: (c) "The Yellow Wallpaper" is a short story by 47. Which two terms among the following are
Charlotte Perkins Gilman, published in 1892 in the associated with formalist criticism?
New England Magazine. It is regarded as an important 1. aura 2. actant
early work of American feminist literature for its 3. narratee 4. defamiliarization
illustration of the attitude towards mental and physical 5. foregrounding
health of women in the 19th century. Choose the correct answer from the options
"A Room of One's Own" is an extended essay by given below:
Virginia Woolf, first published in September 1929. (a) 1 and 3 only (b) 2 and 4 only
The work is based on two lectures Woolf delivered in (c) 2 and 3 only (d) 4 and 5 only
October 1928 at Mewnhan College and Girton Ans: (d) Formalist literary criticism focuses on the
College, women's constituent colleges at the text as the mayor artifact worthy of study rather than,
University of Cambridge. say, the author him or herself, the historical time
45. Who among the following feminist theorists period during which the text was written, how the text
posited a separate realm of female experience responds to gender roles or class concerns during the
captured in a style of writing different from period, or anything else that exists outside of the text's
men's? world itself. The term 'defamiliarization' and
1. Elaine Showalter 'foregrounding' are associated with formalist criticism.
2. Luce Irigaray 48. Who among the following believed that rhyme is
3. Kate Millett not an integral part of poetry?
4. Simone de Beauvoir 1. William Wordsworth 2. Horace
5. Helene Cixous 3. Samuel Daniel 4. Philip Sidney
Choose the correct answer from the options Choose the most appropriate answer from the
given below: options given below:
(a) 1, 3 and 4 only (b) 2 and 4 only (a) 1 and 3 only (b) 2 and 4 only
(c) 3, 4 and 5 only (d) 2 and 5 only (c) 1 and 4 only (d) 4 and 3 only
Ans: (d) 'Lucy Irigaray' is a Belgian born French Ans: (b) Sidney's notion of "Fore-Conceit" means that
feminist, philosopher, linguist, psycholinguist, a conception of the work must exist in the poet's mind
psychoanalyst and cultural theorist who examined the before it is written. Free from the limitations of nature
uses and misuses of language in relation to women. and independent from nature, poetry is capable of
'Helene Cixous' is a professor, French feminist "making things either better than Nature bringeth
writer, poet, playwright, philosopher, literary critics forth, or quite a new, forms such as never were in
and rhetorician. Cixous is best known for her article Nature.
"The Laugh of the Medusa" which established her as Horace believed that poetry is not mere imitation
one of the early thinkers in post-structural feminism. alone. He said that a poet 'often mingles facts with
fancy, putting on something of his own'. He didn’t
46. Which of these statements describe correctly the
like too much fancy on the part of the poet and added
basic assumption of Structuralism?
that 'fiction composed to please should be very near to
1. Structuralism is concerned with signs and
the truth'.
signification.
2. A structuralist theory considers only verbal 49. Who among the following are the two great
conventions and codes. masters of the French language that T.S. Eliot
3. Structuralism began in the works of Jacques contrasts with Dryden and Milton in "The
Derrida that influenced the 20th-century Metaphysical Poets"?
literary criticism. 1. Francois Villon
4. Structuralism challenges the long-standing 2. Jean Racine
belief that literature reflects a given reality. 3. Charles Baudelaire
5. All signs are arbitrary but without them we 4. Arthur Rimbaud
cannot comprehend reality. Choose the correct answer from the options
Choose the correct answer from the options given below :
given below: (a) 1 and 3 only (b) 1 and 4 only
(a) 1, 3 and 5 only (b) 1, 4 and 5 only (c) 2 and 3 only (d) 2 and 4 only
(c) 1, 2 and 3 only (d) 1, 2 and 5 only Ans: (c) Eliot's essay 'The Metaphysical Poets' was
Ans: (b) Structuralism is a method of interpreting and first published as a review of J.G. Gierson's edition of
analyzing such a things as language, literature and Metaphysical lyrics and poems of the 17th Century. In
society, which focuses on contrasting ideas or this essays Eliot had given the reference of two great
elements of structures and attempts to show how they master of the French language as following-
relate to the whole structure. It sought to analyze the "In French literature the great master of the
adult mind (defined as the sum total of experience seventeenth century Racine and the great master of the
from birth to present) in terms of the simplest nineteenth –Baudelaire- are in some ways more like
definable components and then to find the way in each other then they are like anyone else. The
which these components fits together in complex greatest…………Milton and Dryden, triumph with a
form. dazzling………….
UGC NTA NET/JRF English Paper II, June 2020 Shift-I 335 YCT
50. Which two terms from among the following are Ans: (c) A metalanguage is a language used to
specifically linked to the work of Pierre describe another language, often called the object
Bourdieu? language. Expressions in a metalanguage are often
1. habitus 2. consciousness distinguished from those in the object language by the
3. desire 4. distinction use of italics, quotation marks or writing on separate
Choose the correct answer from the options line. The term metalanguage or 'language about
given below: language" is used increasingly in the language
(a) 1 and 3 only (b) 1 and 4 only teaching literature. Some writers use the term to refer
(c) 2 and 4 only (d) 3 and 4 only to technical and semi-technical linguistic terminology,
such as verb complement and sentence.
Ans: (b) Pierre Bourdieu was a French sociologist,
anthropologist, philosopher and public inflectors. 53. "Hari wrote a poem on the mountains". Which
two of the following are admissible statements
Bourdieu developed a theory of the action, around the
about the above sentence?
concept of 'habitus' which exerted a considerable 1. The sentence is an example of lexical
influence in the social sciences. 'Habitus' refers to the ambiguity.
physical embodiment of cultural capitals to the deeply 2. The sentence is an example of structural
ingrained habits, skill and disposition that we passes ambiguity.
due to our life experience. In 'Distinction', Bourdieu 3. The sentence involves two deep structures.
reveals how social class determines individual tastes 4. The sentence involves two surface
in things like art food and music. structures.
51. Macaulay's Minute of 1835 sought to: Choose the correct answer from the options
1. promote European literature and science given below:
among the natives. (a) 1 and 2 only (b) 2 and 3 only
2. impart knowledge of English literature and (c) 2 and 4 only (d) 3 and 4 only
science through translated texts. Ans: (b) 'Structural or syntactic ambiguity' is the
3. encourage branches of native learning by potential of multiple interpretations for a piece of
more useful studies. written or spoken language because of the way words
4. stop expenditure on the publication of or phrases are organized. Deep structure and surface
oriental works and spend funds only on structure concepts are used in linguistics, specifically
in the study of syntax in the Chomskyan tradition of
English education.
transformational generative grammar. The deep
Choose the correct answer from the options structure of a linguistic expression is a theoretical
given below: construct that seeks to unify several related structures.
(a) 1 and 4 only (b) 2 and 4 only
54. Which two of the following events are described
(c) 1 and 3 only (d) 2 and 3 only in Samuels Pepys's Diary?
Ans: (a) On 2nd February 1835, British historian and 1. The Plague in London
politician Thomas Babington Macaulay presented his 2. The Great Fire of London
'Minute on Indian Education' that sought to establish 3. The War of Spanish Succession
the need to import English Education to Indian 4. Essex Rebellion
'native'. He promotes English literature and not Choose the correct answer from the options
Sanskrit or Arabic or Persian literature. He did not given below:
favor the use of the mother tongue as the medium of (a) 1 and 2 only (b) 1 and 3 only
education. He gave strong support to English as the (c) 2 and 3 only (d) 2 and 4 only
medium of education. Macaulay spent four years in Ans: (a) Samuel Pepys's (1633-1703) was an
India, where he devoted his efforts to reforming the administrator of the navy of England and Member of
Indian criminal code, putting the British and natives Parliament, who is most famous for the diary he kept
on an equal legal footing, and to establishing an for a decade while still a relatively young man. The
educational system based upon the British model, eventual publication of the diary revealed Pepys as an
which involved introducing Indians to European exceptionally skilled recorder of the political events of
ideas. his time and also everyday life. Pepys's record of
contemporary events such as 'The Plague in London'
52. Which two of the following statements are and 'The Great Fire of London' has become an
applicable to 'metalanguage'? important source for historians seeking an
It is : understanding of life in London during the mid 17th
1. a technical language which describes the century. On 1st January 1660 Samuel Pepys made his
properties of language. first diary entry, one that would lead to a further
2. known as a 'first-order' language. decade of recording everyday trivialities mixed with
3. a 'second-order' language that replaces important events and battles.
'first-order' language with metaphors. 55. Which two of the following inspired the rise of
4. a 'second-order' language. the periodical essay?
Choose the correct answer from the options 1. Robert Burton
given below: 2. Francois Rabelais
(a) 1 and 2 only (b) 3 and 4 only 3. Francis Bacon
(c) 1 and 4 only (d) 2 and 3 only 4. Michel de Montaigne
UGC NTA NET/JRF English Paper II, June 2020 Shift-I 336 YCT
Choose the most appropriate answer from the 58. Which two of the following are the titles of the
options given below : sections in Thomas De Quincey's "The English
(a) 3 and 1 only (b) 1 and 2 only Mail – Coach"?
(c) 3 and 4 only (d) 2 and 4 only 1. The Glory of Mobility
Ans: (c) A periodical essay is an essay published in a 2. The Vision of Sudden Death
magazine or journal in particular, an essay that 3. The Glory of Motion
appears as part of a series. The 18th century is 4. The Vision of Unexpected Truth
considered the great age of the periodical essay in Choose the correct answer from the options
English. The term "periodical essay" appears to have given below:
been first used by George Colman the Elder and (a) 1 and 2 only (b) 1 and 4 only
Bonnell Thornton in their magazine 'The Connoisseur' (c) 2 and 3 only (d) 2 and 4 only
(1754-56). Periodicals were aimed at middle class Ans: (c) "The English Mail – Coach" is an essay by
people who were literate enough and could afford to the English author Thomas De Quincey. A "Three part
buy the editions regularly. masterpiece" and 'one of his most magnificent works",
it first appeared in 1849 in Blackwood's Edinburgh
56. Which two of the following works does Walter
Magazine, in the October and December issues.
Pater regard as examples of "great art" in his
Part I – "The Glory of Motion"
essay "Style"? Part II – "The Vision of Sudden Death"
1. Iliad Part III – "Dream Fugue, Founded on the Preceding
2. The Divine Comedy Theme of Sudden Death".
3. Les Miserables
59. Which two of the following books are
4. Faust
explorations of the art of the novel by novelists?
Choose the most appropriate answer from the
1. The Brief Compass
options given below:
2. The Naive and the Sentimental Novelist
(a) 1 and 2 only (b) 1 and 4 only
3. The Visionary Company
(c) 2 and 3 only (d) 2 and 4 only 4. Testaments Betrayed
Ans: (c) Walter Horatio Pater was an English essayist, Choose the most appropriate answer from the
literary and art critic and fiction writer, regarded as options given below:
one of the great stylists. His works on Renaissance (a) 1 and 2 only (b) 1 and 3 only
subjects were popular but controversial in his times. (c) 2 and 3 only (d) 2 and 4 only
His famous 'Appreciations, with an Essay on Style' Ans: (d) "The Naïve and the Sentimental Novelist"
was published in 1895. The divide comedy and Les is written by a Turkish novelist, screen writer,
Miserables are regarded as examples of "great art" in academic and recipient of the 2006 Nobel Prize in
his essay "Style". Literature, Ferit Orhan Pamuk.
57. According to his essay "Civil Disobedience", "Testaments Betrayed" is written by Milan
what two things did Thoreau learn from the Kundera. Milan is a Czech writer who went into exile
night he spent in jail? in France in 1975, becoming a naturalized French
1. He concluded that the State is ultimately citizen in 1981. "Testaments Betrayed" is written like
weak. a novel : the same characters appears and reappear
2. He realized that captivity inspires courage. throughout the nine parts of the book, as do the
3. He realized that the neighbours are only principle themes that pre-occupy the author.
friends during good times. 60. The lives of which of the following writers have
4. He concluded that captivity brings wisdom been the subject matter of novels by Anthony
about human affairs. Burgess?
Choose the correct answer from the options 1. Milton 2. Marlowe
given below: 3. Shelley 4. Keats
(a) 1 and 2 only (b) 1 and 3 only Choose the correct answer from the options
(c) 1 and 4 only (d) 3 and 4 only given below:
Ans: (b) Resistance to Civil Government, called Civil (a) 1 and 2 only (b) 1 and 4 only
Disobedience for short, is an essay by American (c) 2 and 3 only (d) 2 and 4 only
transcendentalist Henry David Thoreau that was first Ans: (d) John Anthony Burgess Wilson, (1917-1993),
published in 1849. Thoreau expounded his anarchistic was an English writer and composer. His novel about
view of government, insisting that it an injustice of the murder of Christopher Marlowe, "A Dead Man in
government is "of such a nature that it requires Deptford" was published in 1993.
injustice to another [you should] break the law [and] In 1958, Anthony Burgess wrote the book English
let your life be a counter friction to stop. In "Civil Literature : A Survey for Students. In this book is one
Disobedience" Thoreau's basic premise is that a higher of Burgess's earliest published description of Keats.
law than civil law demands the obedience of the 61. Which two rivers are mentioned by Andrew
individual. Human law and government are Marvell at the beginning of "To His Coy
subordinate. In cases where the two are at odds with Mistress"?
one another, the individual must follow his conscience 1. The Ganges 2. Thames
and it necessary disregard human law. 3. Humber 4. The Jhelum
UGC NTA NET/JRF English Paper II, June 2020 Shift-I 337 YCT
Choose the correct answer from the options Ans: (b) "Epic theatre" is a theatrical movement
given below: arising in the early 20th century. Epic theatre
(a) 1 and 4 only (b) 1 and 2 only emphasizes the audience's perspective and reaction to
(c) 1 and 3 only (d) 2 and 3 only the piece through a variety of techniques. Bertolt
Ans: (c) "To His Coy Mistress" is a Cavalier poem Brecht's method can be summed up as a process. It
written by Andrew Marwell. It was published begins with the construction of the Fabel, which then
posthumously in 1681. It is a non-stanzaic iambic leads to initial blockings in the form of the Scenes
tetrameter poem rhyming as couplets. In this poem Arrangements. James Saunders early plays led to him
two rivers are mentioned as Ganga and Humber, being considered one of the leading British exponents
following lines makes it clear- of the Theatre of the Absurd.
"Thou by the Indian Gange's side 65. Which two characters/speakers among the
Shouldst rubies find; I by the tide following exhibit the studious abstraction of
of Humber would complain. I would" scholars?
1. Shylock 2. Hamlet
62. Which two poems in the following list are
3. Il Penseroso 4. Mosca
examples of dramatic monologue? Choose the correct answer from the options
1. Alfred Tennyson, "Ulysses" given below:
2. Philip Larkin, "Church Going" (a) 1 and 4 only (b) 2 and 3 only
3. Carol Ann Duffy, "Medusa" (c) 3 and 4 only (d) 1 and 3 only
4. Katherine Philips "A Married State" Ans: (b) 'Hamlet' was a university student at
Choose the correct answer from the options Wittenberg and has a "studious temperament",
given below: illustrated by the fact that he is eager to get back to
(a) 1 and 4 only (b) 2 and 3 only school.
(c) 3 and 4 only (d) 1 and 3 only "Il Penseroso" is "the thoughtful person" whose night
Ans: (d) "Ulysses" is written as a dramatic if filled with meditative walking in the woods and
monologue : the entire poem is spoken by a single hours of study in a "Lonely Tower".
character, who identity is revealed by his own words. 66. Match List I with List II
The lines are in blank verse, or unrhymed iambic
List I List II
pentameter, which serves to impart a fluid and natural
Terms Theorists
quality to Ulysses speech.
A. arche-ecriture I. Julia Kristeva
"Medusa" fist appeared in Carol Ann Duffy's 1999
B. cyborg II. Donna Haraway
collection 'The world's wife' "Medusa" is told in the
C. genotext III. Friedrich Schleiermacher
first person as a dramatic monologue by a woman
D. hermeneutic IV. Jacques Derrida
who is insecure and worried that her husband is
circle
cheating on her.
Choose the correct answer from the options
63. Which two of the following poems are by Robert given below:
Browning? (a) A-IV, B-II, C-I, D-III (b) A-III, B-I, C-II, D-IV
1. "Locksley Hall" (c) A-III, B-II, C-IV, D-I (d) A-IV, B-I, C-II, D-III
2. "The Pied Piper of Hamelin Ans: (a)
3. "The Lady of Shalott" • Jacques Derrida → Arche-writing (archi-ecriture in
4. "Two in the Campagna" french)
Choose the correct answer from the options • Donna J. Haraway → Cyborg manifesto
given below:
• Julia Kristeva → genotext
(a) 1 and 4 only (b) 2 and 3 only
(c) 1 and 3 only (d) 2 and 4 only • Friedrich Schleiermacher → hermeneutic circle.
Ans: (d) "The Pied Piper of Hamelin" has the 67. Match List I with List II
theme of 'agreements, once made, should be honored'.
On one level, The Pied Piper is a simple, poetic List I List II
morality tale, though many have viewed the work as a Terms Theorists
commentary on death, desire and the role of the artist A. Superreader I. Michel Foucault
in society. B. Biopower II. Mikhail Bakhtin
"Two in the Campagna" refers to an area in the C. Bricolage III. Michael Riffaterre
countryside around Rome. In this, Browning engages D. Chronotope IV. Claude Levi-Strauss
with themes of love, human limits and nature. Choose the correct answer from the options
given below:
64. Which two of the following dramatists are
(a) A-III, B-II, C-IV, D-I (b) A-III, B-I, C-IV, D-II
associated with the Epic Theatre? (c) A-IV, B-I, C-III, D-II (d) A-II, B-I, C-IV, D-III
1. Fernando Arrabal 2. Bertolt Brecht
3. Arnolt Bronnen 4. James Saunders Ans: (b)
Choose the correct answer from the options • Michael Riffaterre → Superreader
given below : • Michel Foucault → Biopower
(a) 1 and 2 only (b) 2 and 3 only • Claude Levi-Strauss → Bricolage
(c) 1 and 4 only (d) 2 and 4 only • Mikhail Bakhtin → Chronotope

UGC NTA NET/JRF English Paper II, June 2020 Shift-I 338 YCT
68. Match List I with List II Choose the correct answer from the options
List I List II given below:
Critics Text (a) A-I, B-III, C-II, D-IV (b) A-IV, B-III, C-II, D-I
A. Horace I. A Defence of Rhyme (c) A-III, B-IV, C-I, D-II (d) A-III, B-IV, C-II, D-I
B. John Dryden II. Timber: or, Discoveries Ans: (b) Paul Grice – Cooperative Principle
C. Samuel Daniel III. Ars Poetica Edward Sapir – Linguistic Relativity
Ferdinand de Saussure – Linguistic Signs
D. Ben Jonson IV. Of Dramatic Poesy
Nancy Dorian – Language Death
Choose the correct answer from the options
71. Match List I with List II
given below:
(a) A-II, B-I, C-IV, D-III (b) A-III, B-IV, C-II, D-I List I List II
(c) A-III, B-IV, C-I, D-II (d) A-II, B-IV, C-I, D-III Word Source Indian Language
Borrowed
Ans: (c)
A. Mongoose I. Tamil
• "Ars Poetica" is a poem written by Horace C. 19 B. Loot II. Malayalam
BC. C. Curry III. Hindi/Urdu
• "Of Dramatic Poesy" is a work by John Dryden in D. Betel IV. Marathi
1668. Choose the correct answer from the options
• "Timber or Discoveries" is composed by Ben given below :
Jonson in 1641. (a) A-IV, B-III, C-I, D-II (b) A-IV, B-II, C-I, D-III
• "A Defence of Rhyme" is written by Samuel (c) A-II, B-III, C-IV, D-I (d) A-II, B-I, C-IV, D-III
Daniel in 1503. Ans: (a)
69. Match List I with List II • 'Mongoose' is a Marathi word.
List I List II • 'Loot' is usually comes under Hindi Vocal or
Author Test sometimes under Urdu.
A. Michel de Certeau I. Distinction • 'Curry' is a traditional word of Tamil region.
B. John Fiske II. Reading the Romance • 'Betel' is a Malayalam word.
C. Pierre Bourdieu III. Understanding 72. Match List I with List II
Popular Culture List I List II
D. Janice Radway IV. The Practice of Essayist Essay
Everyday Life A. George Orwell I. "On the Artificial
Comedy of the Last
Choose the correct answer from the options Century"
given below : B. Michel de II. "Why I Write"
(a) A-IV, B-I, C-II, D-III (b) B-III, C-IV, D-I, A-II Montaigne
(c) A-IV, B-III, C-I, D-II (d) B-III, C-I, D-IV, A-II C. Charles Lamb III. "A Modest Proposal"
Ans: (c) D. Jonathan Swift IV. "On the Cannibals"
• "The Practice of Everyday Life" is a book by (a) A-III, B-IV, C-III, D-I (b) A-II, B-IV, C-I, D-III
Michel de Certeau that examines the ways in which (c) A-IV, B-III, C-II, D-I (d) A-II, B-III, C-I, D-IV
people individualize mass culture. It was originally Ans: (b)
published in French in 1980. • On the Artificial Comedy of the Last Century",
• "Understanding Popular Culture" is a book by John written by Charles Lamb shows that Restoration
Fiske who was an American philosopher and Comedies "are a world of themselves almost as
historian. Fiske differentiates between mass culture. much as fairy land".
The book was published in 1989. • "Why I Write" is an essay by George Orwell
• "Reading the Romance" is a book by Janice detailing his personal journey to becoming a writer.
Radway that analyzes The Romance novel genre • "A Modest Proposal" is a Juvenalian satirical essay
using reader-response criticism, first published in by Jonathan Swift.
1984. • "On the Cannibals" is an essay by Michel de
"Distinction" is a 1979 book by Pierre Bourdieu, Montaigne, describing the ceremonies of Brazil.
based upon the author's empirical research from 1963 73. Match List I with List II
until 1968. List I List II
70. Match List I with List II Author Text
A. Thomas Pynchon I. G.
List I List II
B. Howard Jacobson II. V
Linguist Concept C. Anthony Burgess III. J
A. Paul Grice I. language death D. John Berger IV. M/F
B. Edward Sapir II. linguistic signs Choose the correct answer from the options
C. Ferdinand de III. linguistic relativity given below:
Saussure (a) A-III, B-IV, C-I, D-III (b) A-II, B-III, C-IV, D-I
D. Nancy Dorian IV. cooperative principle (c) A-II, B-III, C-I, D-IV (d) A-IV, B-III, C-I, D-II
UGC NTA NET/JRF English Paper II, June 2020 Shift-I 339 YCT
Ans: (b) 77. Arrange the following critical works in their
• "V" is the debut novel of Thomas Pynchon, chronological order of publication:
published in 1963. A. "Preface to Lyrical Ballads"
• "J" is a 2014 novel by Howard Jacobson. B. "A Defence of Rhyme"
• "M/F" is a 1971 novel by Anthony Burgess. C. "Life of Cowley"
• "G" is a 1972 novel by John Berger, set in Pre-First D. "The Frontiers of Criticism"
World War Europe and its protagonist, named "G". Choose the correct answer from the options
74. Match List I with List II given below:
List I List II (a) A, C, B and D (b) B, A, C and D
Lines Poems (c) B, C, A and D (d) C, A, D and B
A. Monuments of I. "Leda and the Swan" Ans: (c) • A Defence of Rhyme (1503)
unaging intellect • Life of Cowley (1779-1781)
B. In the foul rag-and- II. "Adam's Curse" • Preface to Lyrical Ballads (1800)
bone shop of the heart • The Frontiers of Criticism (1956)
C. So mastered by the III. "Sailing to
brute blood of the air Byzantium" 78. Arrange the following in the chronological order
D. As weary-hearted as IV. "The Circus of publication:
that hollow moon Animals' Desertion" A. Aspects of the Theory of Syntax
Choose the correct answer from the options B. Course in General Linguistics
given below: C. Semiotics and the Philosophy of Language
(a) A-III, B-IV, C-II, D-I (b) A-III, B-I, C-IV, D-II D. How to Do Things with Words
(c) A-III, B-IV, C-I, D-II (d) A-II, B-I, C-IV, D-III Choose the correct answer from the options
Ans: (c) given below:
• Monuments of unaging intellect is a line of "Sailing (a) D, B, A, D (b) C, B, A, D
to Byzantium". (c) B, D, A, A (d) B, A, D, C
• In the foul rag-and-bone shop of the heart is from Ans: (c) • 'Course in General Linguistics' by Sassure,
"The Circus Animals Desertion". published in 1916.
• So mastered by the brute blood of the air is from • 'How to Do Things with Words' by J.L. Austin,
"Leda and the Swan". published in 1962.
• As weary-hearted as that hollow moon lines are • 'Aspects of the Theory of Syntax' written by Noam
taken from "Adam's Curse". Chomsky, first published in 1965.
75. Match List I with List II • 'Semiotics and the Philosophy of Language' by
List I List II Umberto Eco, originally published in 1984.
Author Work 79. Arrange the following in the chronological order
A. John Keats I. Alastor of publication:
B. William Wordsworth II. Songs of A. Advancement of Learning
Experience
B. The Origin of Species
C. P. B. Shelley III. Lamia
C. On Heroes and Hero Worship
D. William Blake IV. The Excursion
D. The Lives of the Poets
(a) A-III, B-I, C-IV, D-II (b) A-III, B-IV, C-I, D-II
(a) D, A, C, B (b) D, A, B, C
(c) A-I, B-IV, C-III, D-II (d) A-IV, B-II, C-I, D-III
(c) A, D, C, B (d) A, D, B, C
Ans: (b)
• John Keats – Lamia Ans: (c) • 'Advancement of Learning' is a 1605 book
• William Wordsworth – The Excursion by Francis Bacon.
• P. B. Shelley – Alastor • 'Lives of the Poets' is a work by Samuel Johnson,
• William Blake – Songs of Experience published in 1779.
76. Arrange the following terms in the chronological • 'On Heroes and Hero Worship' is a book by
order of emergence Thomas Carlyle, published in 1841.
A. Heresy of Paraphrase • 'The Origin of Species' by Charles Darwin is
B. Stream of Consciousness published in 1859.
C. Practical Criticism 80. Arrange the following 18th century magazines in
D. Defamiliarization the chronological order of publication:
Choose the correct answer from the options A. The Critical Review
given below: B. The Monthly Review
(a) D, B, C, A (b) B, D, A, C C. The Gentleman's Magazine
(c) B, D, C, A (d) D, C, B, A D. The Rambler
Ans: (c) • Stream of Consciousness (1890) Choose the correct answer from the options
• Defamiliarization (1917) given below:
• Practical Criticism (1929) (a) A, D, B, C (b) D, A, B, C
• Heresy of Paraphrase (1949) (c) B, A, C, D (d) C, B, D, A
UGC NTA NET/JRF English Paper II, June 2020 Shift-I 340 YCT
Ans: (d) • 'The Gentleman's Magazine' was a monthly 84. Arrange the following plays in their
magazine by Edward Cave in January 1731. chronological order:
• 'The Monthly Review' (1749-1845) was an English A. The Country Wife B. Cymbeline
C. The Spanish Tragedy D. The Rivals
periodical founded by Rolph Griffiths a
Choose the correct answer from the options
nonconformist book seller. given below:
• 'The Rambler' was published on Tuesdays and (a) B, A, C, D (b) B, C, D, A
Saturday from 1750 to 1752 and total 208 articles. (c) C, B, A, D (d) C, A, B, D
It was Johnson's most consistent and sustained Ans: (c) • 'The Spanish Tragedy– or 'Hieronimo is
work in the English Language. Mad Again' is an Elizabethan tragedy written by
• 'The Critical Review' was a British publication Thomas Kyd, between 1582 and 1592.
appearing from 1756 to 1817. • 'Cymbeline' is a play by Shakespeare set in Ancient
81. Arrange the following in the chronological order Britain, was first performed in 1623.
of publication: • 'The Country Wife' is a Restoration comedy
A. Crome Yellow written in 1675 by William Wycherley.
B. Sons and Lovers • 'The Rivals' is a comedy of manners by Richard
C. Mrs Dalloway Brinsley Sheridan, first performed on 17 January,
D. A Portrait of the Artist as a Young Man 1775.
Choose the correct answer from the options 85. Arrange the following plays in the chronological
given below: order of publication:
(a) B, A, D, D (b) A, B, D, C A. All for Love
(c) A, C, B, D (d) B, D, A, C B. Venice Preserved
C. The School for Scandal
Ans: (d) • 'Sons and Lovers (1913) D. The Country Wife
• 'A Portrait of the Artist as a Young Man' (1916) Choose the correct answer from the options
• 'Crome Yellow' (1921) given below:
• 'Mrs Dalloway' (1925) (a) B, C, A, D (b) D, A, B, C
82. Arrange the following women novelists in the (c) C, B, D, A (d) A, D, C, B
chronological order (by date of birth): Ans: (b) • 'The Country Wife' is a Restoration
A. Anne Bronte comedy written in 1675 by William Wycherley.
B. Jane Austen • 'All for Love' is a 1677 heroic drama by John
C. Ann Radcliffe Dryden.
D. Fanny Burney • 'Venice Preserved' is an English Restoration play
E. Maria Edgeworth written by Thomas Otway, in 1680s.
Choose the correct answer from the options • 'The School for Scandal' is a play by Richard
Brinsley Sheridan, first performed on 1777.
given below:
(a) B, A, D, C, E (b) C, D, B, E, A 86. Given below are two statements one is labelled
(c) D, C, E, B, A (d) A, B, C, E, D as Assertion A and the other is labelled as
Reason R.
Ans: (c) • Fanny Burney – 1752 Assertion A: Research methods are a range of
• Ann Radcliffe – 1764 tools that are used for different types of inquiry.
• Maria Edgeworth – 1768 Reason R: The tools used in research are products
• Jane Austen – 1775 • Anne Bronte - 1820 of the situations in which they are applied.
83. Arrange the following authors in the In light of the above statements choose the
chronological order of the birth: correct answer from the options given below:
(a) Both A and R are true and R is the correct
A. Oscar Wilde
explanation of A
B. William Langland (b) Both A and R are true and R is NOT the
C. Geoffrey Chaucer correct explanation of A
D. John Dryden (c) A is true but R is false
E. Alexander Pope (d) A is false but R is true
Choose the correct answer from the options Ans: (b) Research methodology is a way to
given below: systematically solve the research problem. It is the
(a) B, C, D, E, A (b) A, B, C, E, D logic behind the methods we use in the context of our
(c) B, C, D, A, E (d) C, B, A, D, E research.
Ans: (a) • William Langland – (1332) 87. Given below are two statements one is labelled
• Geoffrey Chaucer – (1343) as Assertion A and the other is labelled as
• John Dryden – (1631) Reason R.
• Alexander Pope – (1688) Assertion A: Signs are never neutral or innocent.
Reason R: In all cases signs are organized into
• Oscar Wilde – (1854) systems that convey some meaning.
UGC NTA NET/JRF English Paper II, June 2020 Shift-I 341 YCT
In light of the above statements choose the Read the passage and answer the questions that
correct answer from the options given below: follow:
(a) Both A and R are true and R is the correct Daybreak
explanation of A. At dawn she lay with her profile at that angle
(b) Both A and R are true but R is NOT the correct Which, sleeping, seems the stone face of an angel;
explanation of A. Her hair a harp the hand of a breeze follows
(c) A is true but R is false. To play, against the white cloud of the pillows.
(d) A is false but R is true. Then in a flush of rose she woke, and her eyes were
Ans: (a) Sign is a kind of motion, action or movement open,
of the hand that means something. It is an indicator or Swimming with blue through the rose flesh of dawn.
marker for something very specific, very concrete and From her dew of lips the drop of one word
in general, unambiguous in meaning. Fell, from a dawn of fountains, when she murmured
88. Given below are two statements: 'Darling' – upon my heart the song of the first bird.
Statement I: Consumption is an outcome of self- 'My dream glides in my dream' she said, 'come true.
interest and a maximization of personal pleasure. I waken from you to my dream of you.'
Statement II: There are strong correlations O, then my waking dream dared to assume
between social status and such things as housing The audacity of her sleep. Our dreams
styles, musical tastes and food preferences. Flowed into each other's arms, like streams.
In light of the above statements, choose the - Stephen Spender
correct answer from the options given below:
(a) Both Statement I and Statement II are true 91. Which among the following best describes the
(b) Both Statement I and Statement II are false lady's face as "At dawn she lay….." asleep?
(c) Statement I is correct but Statement II is false (a) Her face appears to be that of a stone
(d) Statement I is incorrect but Statement II is true sculpture's.
(b) The side-view of her face appears to be that of
Ans: (a) Both of the statements are true.
a sculpted angel's.
89. Given below are two statements: (c) Her face appears to be that of a stone-angel.
Statement I: The Orientalists in British India were
(d) The side-view of her face appears to be that of
not sympathetic towards India's ancient learning.
Statement II: William Jones thought that in angel's.
"imagination", "ratiocination" and "philosophy", Ans: (b) The side-view of her face appears to be that
Indians were by no means inferior to Europeans. of a sculpted angel's.
In light of the above statements choose the 92. Match List I with List II
correct answer from the options given below: List I List II
(a) Both Statement I and Statement II are true The Item What it is an example of
(b) Both Statement I and Statement II are false A. 'Her hair' I. player
(c) Statement I is correct but Statement II is false B. 'pillows' II. 'a harp'
(d) Statement I is incorrect but Statement II is true C. 'breeze' III. 'rose'
Ans: (d) The terms Orientalism and Oriantalist first D. cheeks IV. 'cloud'
took on a markedly political meaning when they were Choose the correct answer from the options
used to refer to those English Scholars in the late 18th
given below:
and early 19th centuries.
(a) A-I, B-II, C-IV, D-III (b) A-III, B-I, C-II, D-IV
90. Given below are two statements one is labelled (c) A-II, B-IV, C-I, D-III (d) A-IV, B-III, C-I, D-II
as Assertion A and the other is labelled as
Reason R. Ans: (c) Her Hair – A harp
Assertion A: The introduction of English in India Pillows – Cloud
was primarily for the benefit and consolidation of Breeze – Player
British power. Cheeks – Rose
Reason R: English catered to the social and 93. Match List I with List II
economic aspirations of the emerging middle class List I List II
and urban elites in India. Item What it is an example
In light of the above statements choose the of
correct answer from the options given below: A. 'Her hair a harp' I. Simile
(a) Both A and R are true and R is the correct
B. 'the hand of a II. Metaphor
explanation of A
breeze'
(b) Both A and R are true but R is NOT the correct
explanation of A C. 'seems the stone III. Oxymoron
(c) A is true but R is false face'
(d) A is false but R is true D. 'my waking dream' IV. Synecdoche
Ans: (b) The introduction of English in India was Choose the correct answer from the options
primarily for the benefit and consolidation of British given below:
power. This statement is true with the reference of (a) A-II, B-IV, C-I, D-III (b) A-IV, B-II, C-III, D-I
British colonization over Indian Sub Continent. (c) A-IV, B-III, C-II, D-I (d) A-I, B-IV, C-II, D-III
UGC NTA NET/JRF English Paper II, June 2020 Shift-I 342 YCT
Ans: (a) Her hair a harp – Metaphor For which thou whipp'st her. The usurer hangs the
The hand of a breeze – Synecdoche cozener.
Seems the stone face – Simile Through tatter'd clothes small vices do appear;
My waking dream – Oxymoron Robes and furr'd gowns hide all. Plate sin with gold,
Read the given passage and answer the questions that And the strong lance of justice hurtles breaks;
follow Arm it in rags, a pigmy's straw doth pierce it.
Logic cannot have any empirical part that is, a part in King Lear
which the universal and necessary laws of thought
should rest on grounds taken from experience; otherwise 96. Who speaks these lines and to whom?
it would not be logic, i.e. a canon for the understanding (a) Edgar to Lear (b) Goneril to Edgar
or the reason, valid for all thought, and capable of (c) Lear to Gloucester (d) Gloucester to Lear
demonstration. Natural and moral philosophy, on the Ans: (c) In the above passage, King Lear speaks these
contrary, can each have their empirical part since the lines to Gloucester.
former has to determine the laws of nature as an object of 97. In the passage the church officer is asked to
experience; the latter, the laws of the human will, so far whip his own back rather than the prostitute's
as it is affected by nature: the former however, being because:
laws according to which everything does happen; the (a) as a religious man he should punish himself for
latter, laws according to which everything ought to others' sins.
happen. Ethics, however, must also consider the (b) he at one time had lusted after her.
conditions under which what ought to happen frequently (c) men like him make them prostitutes.
does not. Immanuel Kant. (d) he does not have the authority to whip a
94. "Logic cannot have any empirical part", woman.
because: Ans: (c) Option (c) is correct.
1. laws of thought are subjective. 98. The two sentences in the lines from "Through
2. it propounds laws whose applicability can be tatter'd clothes…." to "…. straw doth pierce it"
shown. deal with two foibles, (i) vice and (ii) sin. About
3. its laws are valid for all thought. these two, the speaker says that
4. its laws are valid for everyone experience. (a) Vice afflicts all but sin afflicts only the weak.
Choose the most appropriate answer from the (b) Sin afflicts all but vice afflicts only the strong.
options given below: (c) Sin and vice are seen in both the weak and the
(a) 1 and 4 only (b) 2 and 3 only strong.
(c) 1 and 3 only (d) 2 and 4 only (d) Sin and vice are palpable in the weak and
Ans: (b) Logic cannot have any empirical part impalpable in the strong.
because it propounds laws whose applicability can be Ans: (d) Sin and vice are palpable in the weak and
shown. impalpable in the strong.
95. Based on the given passage which two of the Read the given passage and answer the questions that
following statements are correct? follow
1. For natural philosophy, nature influences The surgeon deposited it in her arms. She imprinted her
the laws. cold, white lips passionately on its forehead; passed her
2. For moral philosophy, nature is to be hands over her face; gazed wildly around; shuddered; fell
experienced. back – and died. They chafed her breast, hands, temples;
3. Natural philosophy does not describe how but the blood had stopped forever. They talked of hope
things actually do happen. and comfort. They had ben strangers too long. 'It's all
4. Moral philosophy accounts for what should over, Mrs. Thingummmy!' said the surgeon at last.
be. 99. In the expression, "passed her hands over her
Choose the correct answer from the options face", the 'face' is of:
given below: (a) the lady surgeon (b) the child
(a) 1 and 3 only (b) 2 and 4 only (c) the nurse (d) the patient
(c) 3 and 4 only (d) 1 and 4 only Ans: (d) The face is of the patient.
Ans: (d) In the above passage, the statement 'For 100. The implication of "they had been strangers too
natural philosophy, nature influences the laws' and long" is
'Moral philosophy accounts for what should be' is (a) Those who spoke of 'hope and comfort' had
correct. been strangers too long.
Read the given passage and answer the questions that (b) 'Hope' had been stranger to 'comfort' for too
follow long
And the creature run from the cur? (c) 'Hope and comfort' had been stranger to the
There thou mightst behold the great image of authority: a patient too long.
dog's obeyed in office. – (d) 'Hope and comfort' had been strangers to the
Thou rascal beadle hold thy bloody hand! surgeon, nurse and the patient too long.
Why dost thou lash that whore? Strip thine own back; Ans: (c) Hope and comfort had been stranger to the
Thou hotly lust'st to use her in that kind patient too long.
UGC NTA NET/JRF English Paper II, June 2020 Shift-I 343 YCT
UGC NTA NET/JRF Exam. Dec. 2020/June 2021
ENGLISH-II
SOLVED PAPER [ Exam Date : 01.12.2021, Shift-II

Direction- 1-3 : Read the following passage and 3. The mistake human beings make is to:
answer the questions that follow: A. detach our self from family life.
The aim of all solitude...is the same: to live more at B. are deep into buying and selling.
leisure and at one’s ease. But people do not always look C. mentally abstain from hustle and bustle.
for the right way. Often they think they have left D. are in to schools of philosophy.
business, and they have only changed it. There is Choose the correct answer from the options
scarcely less trouble in governing a family than in given below:
governing an entire state: whatever the mind is wrapped (a) A and C only (b) B and C only
up in it, it is all wrapped up in it, and domestic (c) B and D only (d) C and D only
occupations are no less importunate for being less Ans. (c) : According to the passage, the 'principal
important. Furthermore, by getting rid of the court and worriers of our life' follow us if we are deep into buying
the marketplace we do not get rid of the principal worries and selling and schools of philosophy. Montaigne
of our life....Ambition, avarice, irresolution, fear and lust emphasizes in his philosophy that rationality is no more
do not leave us when we change our country....They than a form of animal behaviour. He also explored the
often follow us even into the cloisters and schools of various aspects of human nature and life by writing.
philosophy. Neither deserts, nor rocky caves, nor hair Hence, option (c) is correct answer.
shirts, nor fasting will free us of them.
Direction - 4-6 : Read the following poem and answer
-- Michel de Montaigne, “Of solitude”
the questions that follow:
1. Which of the following best captures the theme A Prayer for Old Age
of the passage? God guard me from those thoughts men think
(a) Ruling a state is easier than managing a In the mind alone;
family. He that sings a lasting song
(b) Solitude is one condition of peace with one Thinks in a marrow-bone;
self. From all that makes a wise old man
(c) The court and the marketplace must be got rid That can be praised of all;
of. O what am I that I should not seem
(d) Try what one may, no one can ever be at ease. For the song's sake a fool?
Ans. (d) : After going through the excerpt of "On I pray--for fashion's word is out
Solitude" by Michael de Montaigne, it is found that And prayer comes round again --
'Try what one may, no one can ever be at ease' is That I may seem, though I die old,
most appropriate theme of the given passage. A foolish, passionate man.
'On Solitude' is an essay, return to it three times during - W.B.Yeats
ten years of editing and emending for publication. The 4. In the second stanza the poet thinks of :
essence of 'On Solitude' is a stoic acceptance of the (a) what all earns all others’ praise.
stupidity of society and the wisdom of living a life of (b) what all makes a wise old man.
imagination and virtue. Hence, the appropriate answer (c) what he does not want to appear.
is option (d). (d) what he thinks he is, a fool.
2. The mistake human beings make is to: Ans. (c) : After analysing the poem "A Prayer For Old
(a) abjure solitude when desirable. Age" by W.B. Yeats. The second stanza can be
(b) abstain from restraining the mind. interpreted as the poet thinks of 'What he does not
(c) abstain from the love of leisure. want to appear'.
(d) exaggerate the value of family. Here 'what he does not want to appear' means, the poet
Ans. (b) : In the above passage, the appropriate answer does not want to appear as a wise and old men rather
is (b). The mistake human beings make is to abstain he want to be seen as young and passionate man.
from restraining the mind, because people are not able Hence, option (c) is correct.
to restrain themself from worries of ambition, extreme 5. Which one of the following best captures what
desire for accumulating wealth, irresolution and lust. we infer about the poet?
And all these happen only through mind of human (a) He believes in the efficacy of prayer.
beings. So the correct answer is option (b). (b) He is a foolish young man who thinks wisely.
Note- Michel de Montaigne was a French writer (c) He is an old man wise as old are.
regarded as the originator of the Modern essay. (d) He is old but happy in not being wise.
UGC NTA NET/JRF English Paper II, 2021 (Sh.-II) 344 YCT
Ans. (d) : After analysing the poem, it can be inferred 9. ‘Is man no more than this?’ means:
about the poet that he is old but happy in not being (a) Accomodated man is well endowed.
wise. This poem highlights the influence of youthful (b) As an animal, man is a superior animal.
passion and Juxtaposes the term of fool and wise. Its all (c) Man is far more than what he seems to be.
about wanting to be seen as young and passionate and (d) Man is not as well endowed as some other
not old and wise. Hence, option (d) is correct. animals.
6. Thoughts true for all time are– Ans. (d) : In the passage, extract from Shakespeare's
(a) born of God’s care. King Lear it is found that the line 'Is man no more than
(b) felt deep inside the self. this?" Means "man is not as well endowed as some
(c) for all human hearts. other animals". King Lear delivers these lines after he
(d) imbued with logic of mind. has been driven to the end of his rope by the cruelties of
Ans. (b) : In the poem, after reading it is found that Goneril and Regan. He cries explaining that humans
thoughts true for all time are felt deep inside the self. would be no different from the animals if they did not
This poem (A Prayer For Old Age) should create a need more than the fundamental necessities of life to be
passion for life that, no matter how foolish it may happy. Thus, option (d) is correct.
appear to others, provides meaning to one's life.
W.B. Yeats chooses to be a passionate man rather than a 10. Which one of the following best captures what
reasonable man. Thus, the appropriate answer is option (b). Shakespere means?
(a) Animals unlike man are more complex.
Direction - 7-8 : Read the following extract and
answer the questions that follow: (b) Animals’ attributes are external.
The earth was made for Dombey and Son to trade in, and (c) Man can accommodate same properties.
the sun and moon were made to give them light. Rivers (d) Man just uses what animals possess.
and seas were formed to float their ships; rainbows gave Ans. (a) : When we read the passage, we find that
them promise of fair weather; winds blew for or against Shakespeare means by this passage, Animals unlike
their enterprises; stars and planets circled in their orbits, man are more complex.
to preserve inviolate a system of which they were the The moral of King Lear is the idea that a person's
centre. Common abbreviations took new meanings in his actions speak louder than words alone. Thus, an
eyes, and had sole reference to them: A. D. had no appropriate answer is option (a).
concern with anno Domini, but stood for anno 11. Match List I with List II
Dombey—and Son. List I List II
- Charles Dickens, Dombey and Son (Book) (Author)
7. The whole description is an example of:
(a) analogy. (b) aporia. A. English, August I. Shyam Selvadurai
(c) image. (d) sarcasm. B. In Custody II. Anita Desai
Ans. (d) : In the above extract we find the figure of C. Such a Long III. Rohinton Mistry
speech sarcasm: Journey
Sarcasm refers to the use of words that mean the D. Funny Boy IV. Upamanyu Chatterjee
opposite of what you really want to say, especially in Choose the correct answer from the options
order to insult someone or to show irritation or just to given below:
be funny. Thus, the appropriate answer is option (d). (a) A - I, B - II, C - IV, D - III
8. What is the 'system' of which Dombey and Son (b) A - II, B - IV, C - I, D - III
were the centre? (c) A - III, B - II, C - I, D - IV
(a) The British political system (d) A - IV, B - II, C - III, D - I
(b) The country’s commerce Ans. (d) : The appropriate match of Books with their
(c) The family business authors.
(d) The workings of nature (Book) (Author)
Ans. (b) : In the above extract it is found out that in A. English, August (1988) IV. Upamanyu Chatterjee
Dombey and Son was centre of The country's B. In Custody (1984) II. Anita Desai
commerce. 'Dombey and Son' is obviously the name of C. Such a Long Journey III. Rohinton Mistry
a commercial firm. It conveys the nation of a business (1991)
enterprise which has passed through at least two D. Funny Boy (1994) I. Shyam Selvadurai
generation and is therefore effective and reliable. Thus
12. Which of these is identified by Ariel Dorfman
an appropriate answer is option (b).
and Armand Mattelart as having been
Direction - 9-10 : Read the following extract and deployed in Walt Disney comicbooks to
answer the questions that follow:
propagate imperialist ideology?
Is man no more than this? Consider him well.
Thou ow'st the worm no silk, the beast no hide, (a) Deification (b) Impoverishment
the sheep no wool, the cat no perfume. --- Ha! (c) Infantilisation (d) Personification
here's three one's are sophisticated. Thou art the Ans. (c) : Infantilisation is identified by Ariel
thing itself; unaccommodated man is no more than Dorfman and Armand Mattelart as having been
such a poor, bare, forked animal as thou art. deployed in watt Disney comicbooks to propagate
- Shakespeare, King Lear imperialist ideology.
UGC NTA NET/JRF English Paper II, 2021 (Sh.-II) 345 YCT
First published in 1971, 'How to Read Donald Duck' by 16. Which of the following are novels by David
Ariel Dorfman and Armand Mattelart, shocked readers by Lodge?
revealing how capitalist ideology operates in our most A. The British Museum is Falling down
beloved cartoons. In this book character shows how to B. The Seven Sisters
established hegemonic idea about capital, race, gender and C. Changing Places
the relationship between developed countries and the Third D. Nice Work
E. Empire of the Sun
World. Hence, the correct option is (c).
Choose the correct answer from the options
13. Which of the following is a dead language? given below:
(a) Cantonese (b) Frisian (a) A, B and C only (b) A, C and D only
(c) Gothic (d) Yiddish (c) B, D and E only (d) C, D and E only
Ans. (c) : Gothic is a dead language. Gothic is the only Ans. (b) : In the given option, the novels of David
known member of the now extinct East Germanic branch. Lodge are 'The British Museum is Falling Down',
Gothic has been a dead language for over 400 years. It was 'Changing Places' and 'Nice Work' correct answer
last reported being spoken in the Tatar Khanate of while other options are not correct. 'The seven sisters'
Crimean. Thus, appropriate answer is option (c). by Lucinda Riley is a novel which will take you to
14. How does T.S.Eliot sum up the peculiar quality another world altogether. 'Empire of the Sun' (1984;
of Marvell’s “ Horatian Ode”? film 1987) is an autobiographical novel by English
(a) ‘a contrast of ideas, different in degree but the writer J.G. Ballard; it was shortlisted for Man Booker
same in principle’ Prize. Thus, the correct answer is option (b).
(b) ‘a tough reasonableness beneath a slight lyric 17. Choose the right chronological sequence of the
grace’ following books:
(c) ‘heterogeneity of materials compelled into A. Arundhati Roy, The God of Small Things
B. Kiran Desai, The Inheritance of Loss
unity’
C. Shashi Deshpande, That Long Silence
(d) ‘telescoping of images and multiplied D. Jhumpa Lahiri, The Namesake
associations’ Choose the correct answer from the options
Ans. (b) : T.S. Eliot sums up the peculiar quality of given below:
Marvell's "Horatian Ode", as a tough reasonableness (a) A, D, B, C (b) C, A, D, B
beneath a slight lyric grace. T.S. Eliot wrote of (c) C, A, D, B (d) D, B, C, A
Marvell's style that "It is more than a technical Ans. (c) : The right chronological sequence of the
accomplishment or the vocabulary and syntax of an books are________.
epoch. Marvell's 'Horatian Ode' provides an excellent (C) Shashi Deshpande's 'That Long Silence' (1988)
example of Cromwell's return from Ireland. Its full title (A) Arundhati Roy's 'The God of Small Things' (1997)
is 'An Horatian Ode upon Cromwell's Return from (D) Jhumpa Lahiri's 'The Namesake' (2003)
Ireland. Thus, option (b) is correct. (B) Kiran Desai's The Inheritance of Loss' (2006)
15. Match List I with List II Thus, the appropriate answer is option (c).
List I (Novel) List II (Writer) 18. Which two of the following conform to the
A. A Handful of Dust I. E.M. Forster documentation style prescribed by the eighth
edition of the MLA Handbook?
B. Brighton Rock II. Evelyn Waugh
A. Puig, Manuel. Kiss of the Spider Woman,
C. Howard's End III. D.H. Lawrence translated by Thomas Colchie, Vintage
D. The Plumed Serpent IV. Aldous Huxley Books, 1991.
E. Those Barren Leaves V. Graham Greene B. Kincaid, Jamaica. “In History.” Callaloo, vol.
Choose the correct answer from the options 24, no. 2, Spring 2001, pp. 620-26.
given below: C. Nunberg, Geoffrey, editor. The Future of the
(a) A - I, B - IV, C - II, D - III, E - V Book. U of California P, 1996.
(b) A - II, B - V, C - I, D - III, E - IV D. Wellek, Rene. A History of Modern
(c) A - III, B - I, C - V, D - II, E - IV Criticism, 1750-1950, Yale UP, 1986.
(d) A - V, B - II, C - IV, D - I, E - III Choose the correct answer from the options
Ans. (b) : given below:
List I List II (a) A and B only (b) A and C only
(Novel) (Writer) (c) B and C only (d) B and D only
A. A Handful of Dust (1934) II. Evelyn Waugh Ans. (c) : The eight edition of the MLA style book is
B. Brighton Rock (1938) V. Graham Greene intended to simplify the citation process. The basic form
of MLA Handbook is- Author's last Name, First Name.
C. Howard's End (1910) I. E.M. Forster
"Title of Article," Periodical Title Volume number.
D. The Plumed Serpent III. D.H. Lawrence issue number (Date of Publication) : Page number
(1926) range. Medium of Publication.
E. Those Barren Leaves IV. Aldous Huxley The two documentation style prescribed by the eight
(1925) edition of the MLA Handbook had option (b) and (c)
Thus, the correct answer is option (b). correct answer. Thus, the correct option is (c).
UGC NTA NET/JRF English Paper II, 2021 (Sh.-II) 346 YCT
19. Which among the following are examples of the Ans. (d) : In light of the above statements statement I is not
Künstlerroman? correct while statement II is correct. Deconstruction claims
A. The Portrait of a Lady that language is non-referential since it refers neither to the
B. David Copperfield things in the world nor to our concepts of things but only to
C. Tom Jones the play of signifiers. Post-structuralist thinker Jacques
D. A Portrait of the Artist as a Young Man Derrida is known as father of deconstruction. Derrida states
Choose the correct answer from the options that deconstruction is an "antistructuralist gesture" because
given below: structures were to be undone, decomposed, desedimented,"
(a) A and B only (b) A and C only Thus option (d) is correct.
(c) B and D only (d) C and D only 22. Who wrote a postmodern reworking of Charles
Ans. (c) : Charles Dicken's novel 'David Copperfield' Dickens’s Great Expectations without altering
the original title?
and James Joyce's novel' 'A Portrait of the Artist as a
(a) Angela Carter (b) Kathy Acker
young man' (1916) are examples of the kunstlerroman (c) Peter Carey (d) Shirley Jackson
'Kunstlerroman is a subtype of the Bildungsroman or
Ans. (b) : Kathy Acker wrote a postmodern reworking
apprenticeship novel (Artist-Novel), which represents
of Charles Dicken's 'Great Expectations' without
the development of a novelist or other artist from altering the original title Kathy Acker's Great
childhood into the stage of maturity that signalize the Expectations moves her narrator through time, gender
recognition of the protagonist's artistic destiny and and identity as it examines our era's cherished beliefs
mastery of an artistic craft. Thus, appropriate answer is about life and art.
option (c). Kathy Acker was an American experimental novelist,
20. Which of these departments did the Wood’s playwright, essayist and postmodernist. Thus,
Dispatch of 1854 recommend settingup in the appropriate answer is option (b).
universities? 23. Which character in Hamlet utters the line:
A. Arabic B. English “Something is rotten in the state of Denmark”?
C. French D. Law (a) Bernardo (b) Ghost
Choose the correct answer from the options (c) Horatio (d) Marcellus
given below: Ans. (d) : The line "Something is rotten in the state of
(a) A, B and D only (b) A, B and C only Denmark". This line is spoken by Marcellus in Act I,
(c) A, C and D only (d) B, C and D only scene IV (67), as he and Horatio debate whether or not
Ans. (a) : Wood's dispatch of 1854 recommended to follow Hamlet and the ghost into the dark night.
setting up of departments of Arabic, English and Law in Thus, appropriate answer is option (d).
the Universities. In 1854 Charles Wood, a British 24. Who is the author of “The Typology of
Liberal politician sent the "Wood's dispatch" to the Detective Fiction”?
governor general lord Dalhousie. It primarily dealt with (a) G. K. Chesterton (b) Tzvetan Todorov
educational reforms Sir Wood recommended that (c) Umberto Eco (d) Vladimir Propp
primary schools adopt Anglo-Vernacular language Ans. (b) : Tzvetan Todorov is the author of "The
and that English be the medium of education in typology of detective fiction". In 'The Typology of
colleges. This is known as wood's dispatch. "It is known Detective fiction' Tzvetan Todorov deals with the way
the genre of detective fiction works within the
as Magna Carta of Education in India".
parameters of certain rules. Tzvetan Todorov is
21. Given below are two statements Bulgarian-French literary theorist and historian of ideas
Statement I : Language is not a reliable tool of whose concerns in dozens of books ranged from fantasy in
communication, says deconstruction, but argues fiction to the moral consequences of colonialism,
in favour of a theory of sign as a self-sufficient fanaticism and the Holocaust. Thus, option (b) is correct.
union of signifier and signified. 25. Arrange the following in their chronological
Statement II : Deconstruction claims that order:
language is non-referential since it refers A. English replaces Persian as official
neither to the things in the world nor to our language of the Company
concepts of things but only to the play of B. Arrival of Charles Grant in India
C. Universities established in Calcutta,
signifiers.
Bombay and Madras
In light of the above statements, choose the D. Construction of Fort William in Calcutta
most appropriate answer from the options Choose the correct answer from the options
given below given below
(a) Both Statement I and Statement II are correct (a) B, A, D, C (b) D, A, C, B
(b) Both Statement I and Statement II are (c) D, B, A, C (d) D, B, C, A
incorrect Ans. (c) : Chronological order of landmark in Indian
(c) Statement I is correct but Statement II is Education system during British rule-
incorrect D. Construction of Fort William in Calcutta (1800-01)
(d) Statement I is incorrect but Statement II is correct B. Arrival of Charles Grant in India (1813)
UGC NTA NET/JRF English Paper II, 2021 (Sh.-II) 347 YCT
A. English replaces Persian as official language of the 30. Which of these countries does Montaigne’s
Company (1833) essay, “Of Cannibals,” focus on primarily?
C. Universities established in Calcutta, Bombay and (a) Borneo (b) Brazil
Madras (1857) (c) India (d) Japan
26. Match List I with List II Ans. (b) : "Of cannibals" is an essay, one of those in the
collection essays by Michel de Montaigne, describing
List I List II
the ceremonies of the Tupinamba people in Brazil. He
(Text) (Author) reported about how the group ceremoniously ate the
A. The Lie of the Land I. Alok Mukherjee bodies of their dead enemies as a matter of honor.
B. Masks of Conquest II. Rajeswari Sunder In "Of Cannibals", Montaigne discusses the apparent
Rajan opposition between primitive and civilized societies.
C. Rethinking English III. Gauri Viswanathan Thus, appropriate answer is option (b).
D. This Gift of English IV. Svati Joshi 31. Which two of the following are works by I. A.
Choose the correct answer from the options Richards?
given below: A. Concepts of Criticism
(a) A - I, B - III, C - IV, D - II B. Science and Poetry
(b) A - II, B - III, C - IV, D - I C. The Philosophy of Rhetoric
(c) A - III, B - IV, C - II, D - I D. English Literature in Our Time and the
(d) A - IV, B - I, C - II, D - III University
Ans. (b) : The appropriate match of texts with their Choose the correct answer from the options
authors. given below:
(a) A and B only (b) A and D only
Text Author (c) B and C only (d) B and D only
A. The Lie of the Land II. Rajeswari Sunder
Ans. (c) : 'Science and Poetry' and 'The Philosophy
(1992) Rajan of Rhetoric' are works of I.A. Richards. I.A. Richards
B. Masks of conquest (1989) III. Gauri is an English educator, literary critic and rhetorician. In
Viswanathan his book 'The Philosophy of Rhetoric, he proposes that
C. Rethinking English (1991) IV. Svati Joshi rhetoric should be 'a study of misunderstanding and its
D. This Gift of English (2009) I. Alok Mukherjee remedies and investigation in how much and in how
Thus, correct answer is option (b). many ways may good communication differ from bad.
27. What was the centre set up for studying culture Thus, option (c) is correct.
at the University of Birmingham called? 32. According to Ferdinand de Saussure, language is:
(a) Centre for Contemporary Cultural Studies A. an interlocking structure.
(b) Centre for Contemporary Studies B. a system of constant change.
(c) Centre for Culture Studies C. a system of signs.
(d) Centre for New Cultural Studies D. a self-standing formation.
Ans. (a) : The Birmingham School, better known as the Choose the correct answer from the options
given below:
Centre for Contemporary Cultural Studies, was a
(a) A and B only (b) A and C only
research center at university of Birmingham. It is (c) B and D only (d) C and D only
founded by Richard Hoggart in 1964, the school played
Ans. (b) : According to Ferdinand de Saussure,
a major role in the development of cultural studies not language is–an interlocking structure and a system of
only in Britain, but across the globe. Hence, option (a) signs. Ferdinand de Saussure's 'Course in General
is correct answer. Linguistics' (1916) lectures at the university of Geneva
28. The set of inflected forms taken by a single from 1906 to 1911. Saussure examines the relationship
word is: between speech and the evolution of language and
(a) lexeme. (b) morpheme. investigates language as a structured system of signs.
(c) phoneme. (d) sememe. Thus, the appropriate answer is option (b).
Ans. (a) : The set of inflected forms taken by a single 33. Match List I with List II
word is lexeme. A lexeme is a unite of lexical meaning List I List II
that underlies a set of words that are related through (Library/ Institute) (Location)
inflection. Thus, option (a) is correct answer. A. Connemara Public Library I. Kolkata
29. In which book of Paradise Lost does Milton B. Dhvanyaloka II. Chennai
refer to “Agra and Lahore of Great Mogul”? C. Bhandarkar Oriental Institute III. Mysore
(a) Book III (b) Book IV D. Asiatic Society IV. Pune
(c) Book VII (d) Book XI Choose the correct answer from the options
Ans. (d) : 'Book XI' in 'Paradise Lost' Milton refers to given below:
"Agra and Lahore of Great Mogul" in 1670, because the (a) A - I, B - III, C - IV, D - II
massive Lahore Fort builts. Lahore reached the peak of (b) A - II, B - III, C - IV, D - I
its architectural glory during the rule of the Mughals. (c) A - III, B - IV, C - II, D - I
Thus the correct option is (d). (d) A - IV, B - I, C - II, D - III
UGC NTA NET/JRF English Paper II, 2021 (Sh.-II) 348 YCT
Ans. (b) : The appropriate match of literary institute Choose the correct answer from the options
with their location– given below:
List I List II (a) A and C only (b) B and D only
A. Connemara Public Library II. Chennai (c) C and E only (d) D and A only
B. Dhvanyaloka III. Mysore Ans. (d) : 'Syntactic Structures' and 'Aspects of the
C. Bhandarkar Oriental IV. Pune Theory of Syntax' are books by Noam Chomsky.
Institute 'Syntactic Structures' is originally published in 1957. It
D. Asiatic Society I. Kolkata is an elaboration of his teacher Zelling Harris's model of
Thus, correct answer is option (b) (A-II, B-III, C-IV, D- Transformal Generative Grammar. 'Aspects of the
I). Theory of Syntax' is a a book on linguistics written by
34. Which two of the following essays form part of American linguist Noam Chomsky, first published in
Mikhail Bakhtin’s The Dialogic Imagination: 1965. In Aspects, Chomsky presented a deeper more
Four Essays? extensive reformulation of Transformation Generative
A. “From the History of Novelistic Discourse” grammar. Thus, appropriate answer is option (d).
B. “Discourse in the Novel” 37. Who among the following posits the tradition
C. “Romance and Novel” of great writers as an inescapable fact, and
D. “Forms of Time and the Chronotope in the takes the ambivalent position of considering it
Novel” as both a blessing and a curse?
Choose the correct answer from the options (a) Allen Tate (b) F.R. Leavis
given below: (c) Harold Bloom (d) T. S. Eliot
(a) A and B only (b) A and D only Ans. (c) : Harold Bloom posits the tradition of great
(c) B and C only (d) B and D only writers as an inescapable fact and takes ambivalent
Ans. (d) : 'Discourse in the Novel' and 'Forms of Time position of considering it as both a blessing and a curse.
and the chronotope in the Novel' are essays which form Harold Bloom (1930-2019) was American literary critic
part of Mikhail Bakhtin's 'The Dialogic Imagination. known for his innovative interpretations of literary
Other four essays given in the options does not belong history and of the creation of literature. Bloom's first
to Bakhtin. language is Yiddish and he also learns Hebrew before
• 'Discourse in the Novel' (1934), Bakhtin provides a English. Thus, the correct option is (c).
model for history of discourse and introduces the 38. Which of the following terms is used to
concept of heteroglossia. describe spurious words which are the result of
• In 'Forms of time and the chronotope in the Novel', inadvertent errors made by copyists, printers
Bakhtin analyzes the ways in which configurations and editors?
of time and space (chronotopes) have been (a) dudwords (b) ghostwords
represented in narrative literature. (c) protowords (d) pseudowords
Thus option (d) [B and D] is correct. Ans. (b) : The term 'Ghostwords' is used to describe
35. Which of the following qualify for the label spurious words which are the result of inadvertent errors
‘cultural intermediary’ in the context of a made by copyists, printers and editors. A ghost word is
commercial film? a word published in a dictionary or similarly
A. The film magazine columnist authoritative reference work, having rarely once
B. The director authoritatively published, a ghost word occasionally
C. Fan clubs may be copied widely and take a long time to be erased
D. The producer from usage. Thus, option (b) is correct.
Choose the correct answer from the options 39. Given below are two statements, one is labelled
given below: as Assertion A and the other is labelled as
(a) A and C only (b) A and B only Reason R
(c) B and C only (d) B and D only Assertion A : The implied reader shifts attention
Ans. (a) : 'Cultural intermediary' concept has been from the real reading individual to a disembodied
called for by Keith Negus, who has questioned the dimension of reception, intricately interwoven into
extent that it can bridge the gap between production and the text.
consumption in a context of what he sees as an enduring Reason R : The ‘Dear Reader,’ invoked in the
distance between these two spheres. realist novels, is a fictional representation of the
'The film magazine columnist' and 'Fan clubs' qualify distant reader.
for the label 'Cultural Intermediary' in the context of a In light of the above statements, choose the
commercial film. Thus option (a) is correct answer. most appropriate answer from the options
36. Which of the following are books by Noam given below
Chomsky? (a) A is correct but R is not correct
A. Syntactic Structures (b) A is not correct but R is correct
B. Verbal Learning and Verbal Behavior (c) Both A and R are correct and R is the correct
C. Language and Society explanation of A
D. Aspects of the Theory of Syntax (d) Both A and R are correct but R is NOT the
E. The Pragmatics of Politeness correct explanation of A
UGC NTA NET/JRF English Paper II, 2021 (Sh.-II) 349 YCT
Ans. (a) : In the given statements, Assertion (a) is Asha Kelunni better known by her stage name Revathi,
correct but Reason (R) is not correct answer because the is an Indian actress and director, known for her works
second statement is not supporting with correct reason predominantly in Tamil and Malayalam Cinema. Thus,
stated into statement I. The concepts 'implied reader' and appropriate answer is option (a).
'implied author' were introduced into literary criticism by 44. Who among the following was of the view that
Wayne C. Booth in 'The Rhetoric of Fiction' (1961), poetry was only an imitation of an imitation
Which emerged from the neo-Aristotelian school of and therefore trivial?
formalism associated with the work of R.S. Crane. Thus an (a) Aristotle (b) Phaedo
appropriate answer is option (a). (c) Plato (d) Xenocrates
40. What term did Bertolt Brecht use for his mode Ans. (c) : According to Plato "poetry was only an
of drama-writing to distinguish it from imitation of an imitation and therefore trival". Plato
traditional theatre? disapproves of poetry because it is immoral, as a
(a) epic theatre (b) kitchen-sink theatre philosopher he disapproves of it because it is based in
(c) musical theatre (d) proletarian theatre falsehood. He is of the view that philosophy is better
Ans. (a) : Bertolt Brecht's term 'epic theatre' is for his than poetry because philosopher deals with idea/truth,
mode of drama-writing to distinguish it from traditional whereas poet deals with what appears to illusion. Thus,
theatre. In the theory of 'epic theatre', Brecht believed an appropriate answer is option (c).
that theatre should appeal not to the spectator's feelings
45. Which of the following narrative cycles is
but to his reason. Thus, option (a) is correct.
referred to in Michel Foucault’ s “What is an
41. Which of the following abbreviations refers to Author?”?
a documentation style? (a) The Canterbury Tales
(a) ARIEL (b) MFS (b) The Decameron
(c) MHRA (d) PMLA
(c) The Thousand and One Nights
Ans. (c) : MHRA refers to a documentation style. 'The (d) Tuti Namah
MHRA style Guide : A Handbook for Authors, Editors
and Writers of Theses' formerly the MHRA style Book Ans. (c) : 'The thousand and one Nights' narrative
is an academic style guide published by the Modern cycles is referred to in Michel Foucault's 'What is an
Humanties Research Association (MHRA). Author'? One thousand and one Night' is a collection of
PMLA- PMLA is journal of the modern language Middle Eastern falk tales compiled in Arabic during the
Association of America. Since 1884 PMLA has Islamic golden Age. It is often known in English as the
published members' essay judged to be interest of 'Arabian Nights'.
scholars and teachers of language and literature. Thus, "What is an author?" (1969) is a lecture on literary
an appropriate answer is option (c). theory given by sociologist and historian Michel
42. Which two are the works of Ted Hughes? Foucault.
A. Wildtrack Thus, the correct option is (c).
B. Wodwo 46. Arrange the following terms in the
C. Lupercal chronological order as these appeared in
D. Jack Straw’s Castle literary theory:
Choose the correct answer from the options A. Phallogocentrism
given below: B. Locutionary act
(a) A and B only (b) A and C only C. Interpellation
(c) B and C only (d) B and D only D. Interpretive community
Ans. (c) : 'Wodwo' and 'Lupercal' both are works of Ted Choose the correct answer from the options
Hughes. 'Wodwo' published in 1967, is Ted Hughes' given below:
first book for adults since Lupercal in 1960. It consists (a) A, D, C, B (b) B, C, A, D
principally of the poems he has written in the years (c) C, B, D, A (d) D, A, B, C
since then, but it also contains five stories and one radio Ans. (b) : The appropriate chronological order of
play "From the book Jacket". literary terms are-
'Lupercal' published in 1960, is Hughes' second Locutionary act → Interpellation
collection and his follow-up to 'The Hawk in the Rain'. Phallogocentrism → Interpretive Community.
Thus, correct option is (c). • Locutionary acts are- any utterances which simply
43. Who is the author of The Truth about Me: A contain a meaningful statement about objects.
Hijra Life Story? • Interpellation is a process, in which we encounter
(a) A. Revathi (b) Bama our culture's values and internalize them.
(c) Mukta Sarvagod (d) V. Geetha
• Phallogocentrism is known as a neologism coined
Ans. (a) : 'Revathi' is the author of 'The truth about Me by Jacques Derrida' to refer to privileging of the
: A Hijra Life Story'. It is the unflinchingly courageous masculine in the construction of meaning.
and moving autobiography of Revathi, a Hijra who
fought ridicule, persecution and violence both within • Interpretive Community is a collectivity of people
her home and outside to find a life of dignity. It narrates who share strategies for interpreting, using and
the life of Doraisamy, who in light of being true to his engaging in communication about a media text or
inner feelings, had his nirvaanam done and turned into a technology.
woman called Revathi. Thus, the appropriate answer is option (b) (B, C, A, D)

UGC NTA NET/JRF English Paper II, 2021 (Sh.-II) 350 YCT
47. Match List I with List II 49. Who among the following has coined the term,
List I List II ‘ecofeminism’?
(Author) (Text) (a) Monique Wittig (b) Francoise d’Eaubonne
A. Robert Browning I. Queen Mary (c) Helene Cixous (d) Marguerite Duras
B. S.T. Coleridge II. The Second Mrs Ans. (b) : Ecofeminism, also called ecological
Tanqueray feminism, branch of feminism that examines the
C. A.W. Pinero III. Remorse connections between women and nature. Its name was
D. Alfred Tennyson IV. The Borderers coined by French feminist Francoise d'Eaubonne in
E. William Wordsworth V. Strafford 1974. Hence, option (b) is correct answer.
Choose the correct answer from the options 50. Arrange the following characters in the
given below: chronological order in which they appeared in
(a) A - II, B - IV, C - III, D - V, E - I Indian literature.
(b) A - III, B - V, C - II, D - I, E - IV A. Praneshacharya (Samskara)
(c) A - IV, B - II, C - I, D - V, E - III B. Sakuni (Mahabharata)
(d) A - V, B - III, C - II, D - I, E - IV C. Rusty (The Room on the Roof)
Ans. (d) : The correct match list of Author with their D. Gobar (Godan)
texts. Choose the correct answer from the options
List I (Author) List II (Text) given below
(a) A, B, C, D (b) B, A, C, D
A. Robert Browning V. Strafford (1837)
(c) B, C, D, A (d) B, D, C, A
B. S.T. Coleridge III. Remorse (1813)
Ans. (d) : In the given option, the correct
C. A.W. Pinero II. The Second Mrs chronological order of characters appeared in Indian
Tanqueray (1893) literature.
D. Alfred Tennyson I. Queen Mary (1876) Character and work Published
E. William Wordsworth IV. The Borderers (1842) 1. Sakuni (Mahabharata) 1. 3Century B.C.
Thus, the correct answer is option (d) 2. Gobar (Godan) 2. 1936
A - V, B - III, C - II, D - I, E - IV 3. Rusty (The Room on the Roof) 3. 1956
48. Which of the following statements are true of 4. Praneshacharya (Samskara) 4. 1965
English as used in India? Thus, the correrct answer is option (d) (B, D, C, A).
A. India is counted among the largest English- 51. In “The Life of Cowley” which two of the
speaking communities in the world. following criticisms were made by Samuel
B. No group, community or population of Johnson against a group of writers he termed
Indians claims English as its mother the ‘metaphysical poets’?
tongue. A. They made an inappropriate combination
C. More than fifty per cent of Indians speak of wit and imagination.
English fluently. B. Instead of writing poetry, they only wrote
D. English is the country’s principal language verses.
of commerce. C. They neither copied nature nor life.
E. With the growing stature of Hindi as D. They never tried to be singular in their
lingua franca, it has supplanted English as thoughts.
the link language between the central Choose the correct answer from the options
government and the states. given below:
Choose the correct answer from the options (a) A and B only (b) B and C only
given below: (c) B and D only (d) C and D only
(a) A and D only (b) B and C only Ans. (b) : Samuel Johnson denounced the Metaphysical
(c) C and E only (d) D and E only Poets saying "About the beginning of the seventeenth
Ans. (a) : Even after the independence, English is being century appeared a race of writers that may be termed
used as communicative and official language in India. the Metaphysical Poets".
Now it is used in Education sector, corporate sector, In "The life of Cowley" Samuel Johnson made criticism
judiciary and government offices as an official language. against a group of writers of metaphysical poets are-
Some features of English language in India are– 1. Instead of writing poetry, they only wrote verses.
1. After Hindi, English is the most commonly spoken 2. They neither copied nature nor life.
language in India. Thus, the appropriate answer is option (b).
2. English is used among Indians as a link language due 52. Who is the author of Radiant Textuality?
to vast diversity in Indian languages in general and in (a) Jerome McGann (b) Gerald Graff
modern Indian languages eg., Assamese, Bengali, (c) James Thorpe (d) Richard D. Altick
Kannada, Malayalam, Telugu, Marathi etc. in specific. Ans. (a) : Jerome McGann is the author of Radiant
3. India is counted among the largest English- Textuality : Literary Studies after the World Wide Web
Speaking communities in the world. (2001) This book describes and explains the
4. English is the country's principle language of fundamental changes that are now taking place in the
commerce. most traditional area of humanities theory and method
Thus, appropriate answer is option (a). scholarship and education.
UGC NTA NET/JRF English Paper II, 2021 (Sh.-II) 351 YCT
Jerome John McGann is an American academic and Choose the correct answer from the options
textual scholar whose work focuses on the history of given below:
literature and culture from the late 18th century to the (a) A - II, B - III, C - I, D - IV
present. Thus, the appropriate answer is option (a). (b) A - II, B - III, C - IV, D - I
53. Match List I with List II (c) A - III, B - I, C - IV, D - II
List I List II (d) A - III, B - IV, C - II, D - I
(Text) (Author) Ans. (b) : The appropriate match of plays with their
A. Advancement of I. Susan Sontag playwrights are:
Learning
List I List II
B. Past and Present II. Francis Bacon
C. English Traits III. Thomas Carlyle Plays Playwrights
D. Illness as Metaphor IV. R.W. Emerson A. Madmen and Specialists II. Wole Soyinka
Choose the correct answer from the options (1971)
given below: B. The Sea at Dauphin (1954) III. Derek Walcott
(a) A - I, B - III, C - IV, D - II C. The Trial of Dedan IV. Ngugi wa
(b) A - II, B - III, C - IV, D - I Kimathi (1976) Thiong'o
(c) A - III, B - IV, C - II, D - I D. An Echo in the Bone I. Dennis Scott
(d) A - IV, B - I, C - II, D - III (1986)
Ans. (b) : The appropriate match of the text with their
Thus, the correct option is (b).
author are–
List I (Text) List II (Author) 56. Who among the following says that ideology
1. Advancement of Learning II. Francis Bacon is “a representation of the imaginary
2. Past and Present III. Thomas Carlyle relationship of individuals to their real
3. English Traits IV. R.W. Emerson conditions of existence”?
4. Illness as Metaphor I. Susan Sontag (a) Fredric Jameson (b) Herbert Marcuse
Thus, correct option is (b). (c) Louis Althusser (d) Terry Eagleton
54. Given below are two statements Ans. (c) : Althusser posits a series of hypothesis's that
Statement I: The opening and closing lines of he explorers to clarify his understanding of ideology:
Waiting for Godot are spoken by Estragon.
1. "Louis Althusser says that ideology is "a
Statement II: Towards the end of the play
Waiting for Godot, Estragon echoes Pozzo’s representation of the imaginary relationship of
statement, “They give birth astride of a grave . . .” individuals to their real conditions of existence".
In light of the above statements, choose the 2. "Ideology has a material existence".
correct answer from the options given below 3. "All ideology hails on interpellates concrete
(a) Both Statement I and Statement II are false individuals as concrete subjects".
(b) Both Statement I and Statement II are true 4. "Individuals are always already subjects".
(c) Statement I is false but Statement II is true The traditional way of thinking of ideology led Marxists
(d) Statement I is true but Statement II is false to show how ideologies are false by pointing to the real
Ans. (d) : 'Waiting for Godot (1952) is a play by world hidden by ideology.
Samuel Beckett in which two characters, Vladimir Louis Pierre Althusser was one of the most influential
(Didi) and Estragon (Gago), engage in a variety of
French philosophers of the 20th century. Thus, option
discussions and encounters while awaiting the titular
Godat, who never arrives. In the play, the opening and (c) is correct.
closing lines of waiting for Godot are spoken by 57. Which of these may be said to be true of the
Estragon. Thus, we can say that statement-I is true but journal published from Bowling Green
statement II is false. University from 1969, which carried essays on
'Waiting for Godot' ends with Estragon and Vladimir Spiderman comics, rock music, and detective
learning that Godot will 'surely' come tomorrow. The films?
play ends by explaining that 'they do not move' A. It sought to highlight the importance of
suggesting they will stay in their current limbo. Thus,
popular culture.
option (d) is correct.
B. It sought to highlight the importance of
55. Match List I with List II
elite culture.
List I List II
Plays Playwrights C. It sought to buttress the canon by making
A. Madmen and I. Dennis Scott it more elitist.
Specialists D. It sought to break down the dominance of
B. The Sea at Dauphin II. Wole Soyinka ‘high’ culture.
C. The Trial of Dedan III. Derek Walcott Choose the correct answer from the options
Kimathi given below:
D. An Echo in the Bone IV. Ngugi wa (a) A and B only (b) A and D only
Thiong'o (c) B and C only (d) B and D only
UGC NTA NET/JRF English Paper II, 2021 (Sh.-II) 352 YCT
Ans. (b) : There are two statements that are option (b). Ans. (a) : Both MLA Handbook and MLA style manual
'It sought to highlight the importance of popular culture' were preceded by a slim booklet titled MLA style sheet
and 'It sought to break down the dominance of 'high' first published in 1951 and revised in 1970.
culture, said to be true of the Journal published from MLA Handbook (9th ed-2021) formerly MLA Hand-book
Bowling Green University from 1969, which carried for writers of research papers (1977-2009) establishes a
essays on Spiderman comics, rock music and detective system for documenting sources in scholarly writing. It is
films. In 'Cultural Studies', popular culture is the set published by the Modern Language Association. Thus,
of beliefs, values and practices that are widely shared. the appropriate answer is option (a).
Popular culture is a true reflection of society and elite 61. Which two of the following are the earliest
culture reflects intellectual level and greatness of colonial publishing initiatives that apply to
society very well. India?
58. Arrange the following poems by W. B. Yeats in A. Andrew Lang Colonial Book Series
the chronological order of publication. B. Murray Colonial and Home Library Series
C. Colonial Library Series by Macmillan
A. “The Wild Swans at Coole”
D. Colonial Library Series by Chatto &
B. “The Second Coming” Windus
C. “Among School Children” Choose the correct answer from the options
D. “Adam’s Curse” given below:
Choose the correct answer from the options (a) A and B only (b) A and C only
given below (c) B and C only (d) B and D only
(a) A, C, D, B (b) C, A, B, D Ans. (c) : 'Murray Colonial and Home Library Series'
(c) C, A, D, B (d) D, A, B, C and 'Colonial Library Series by Macmillan' are the
Ans. (d) : The appropriate chronological order of W.B. earliest colonial publishing initiatives that apply to
Yeats' poems with publication are- India. 'The Home and Colonial Library' was a series of
Poem Publication works published in London from 1843 to 1849,
D. Adam's Curse - 1903 comprising 49 titles by John Murray III.
A The wild Swans at coole - 1917 Macmillan's Colonial Library are typically marked
B. The second coming - 1920 "Intended for circulation only in India and the British
C. Among school children - 1928 colonies". Thus, an appropriate answer is option (c).
Thus, correct answer is option (d). 62. Which two of the following poems are by
59. Given below are two statements: Judith Wright?
Statement I : A pidgin is formed by two mutually A. “Meditation on a Bone”
unintelligible speech communities trying to B. “Imperial Adam”
communicate using the most obvious features of C. “Woman to Man”
each other’s language. D. “The Old Prison”
Statement II : Notwithstanding the number of Choose the correct answer from the options
years a pidgin is spoken, it can never become the given below:
mother tongue of a community. (a) A and B only (b) A and C only
In the light of the above statements, choose the (c) B and D only (d) C and D only
most appropriate answer from the options Ans. (d) : 'Woman to Man' (1949) and 'The Old Prison'
given below: are the poems written by Judith Wright. The poem
(a) Both Statement I and Statement II are correct. 'Woman to Man' describes the feelings of a woman
(b) Both Statement I and Statement II are from the time of sexual encounter with her husband until
incorrect. the time of labour. She expresses her feelings as a wife as
(c) Statement I is correct but Statement II is well as the holder of her future offspring in her belly.
'The Old Prison' is a poem by Judith Wright, an
incorrect.
Australian who played a crucial role in the upholding of
(d) Statement I is incorrect but Statement II is aboriginal Australian's rights and took strong decisions
correct. in protecting our environmental issues.
Ans. (c) : After going through the statement I and 63. Which Shakespearean comedy is structured as
Statement II, it is found that statement I is correct and a play within a play?
statement II not correct. Pidgin, a term used in both a (a) A Midsummer Night’s Dream
general and a technical sense for a contact language (b) Love’s Labour’s Lost
which draws on elements from two or more language. A (c) The Comedy of Errors
Pidgin is formed by two mutually unintelligible (d) The Taming of the Shrew
speeches communities trying to communicate using the Ans. (d) : 'The Taming of The Shrew' is a
most obvious features of each other's language. Thus Shakespearean comedy which is structured as a play
option (a) is correct answer. within a play because there are many plot structures.
60. The MLA Style Sheet, a compilation of There is the framing plot and the triple action play that
scholarly conventions and directives, was first included the induction : Christopher Sly and the trick
published in: played on him. The main plot depicts the courtship of
(a) 1951 (b) 1957 Petruchio and Katherina the headstrong obdurate shrew.
(c) 1962 (d) 1970 Thus, an appropriate answer is option (d).
UGC NTA NET/JRF English Paper II, 2021 (Sh.-II) 353 YCT
64. Who wrote The Labyrinth of Solitude? Ans. (b) : "Taking after the practice of the Moralities
(a) Gabriel Garcia Marquez and Interludes, Ben Jonson named his dramatic
(b) Gabriela Mistral personae" and "Chapman's. 'All Fools' and Middleton's
(c) Jorge Luis Borges 'A Trick to Catch the Old one belong to the genre of
(d) Octavio Paz comedy of humours that Jonson is said to have
Ans. (d) : Octavio Paz wrote 'The Labyrinth of pioneered" are true of the dramatic legacy of Ben
Solitude'. Octavio Paz (1914-1998) Mexican poet, Jonson. The legacy Jonson left through the publication
writer and diplomat is recognized as one of the major of his folio of works came with an extra irony attached.
Thus, the correct answer is option (b).
Latin American writers of the 20th century. He received
the Nobel prize for literature in 1990. 'The Labyrinth of 68. Arrange in the right sequence the following
stages of a child’s first language acquisition:
solitude' offered an existentialism and psychoanalytic
A. holophrastic B. babbling
interpretation of Mexican culture. Hence, option (d) is
C. telegraphic speech D. cooing
correct answer. Choose the correct answer from the options
65. Which of these characters figure in Samuel given below:
Beckett’s Waiting for Godot? (a) A, C, D, B (b) B, A, D, C
A. Estragon B. Pozzo (c) C, B, A, D (d) D, B, A, C
C. Bassanio D. Murphy Ans. (d) : The right sequence of (D, B, A, C) the stages
Choose the correct answer from the options of a child's first language acquisition are- Cooing →
given below: Babbling → Holophrastic and Telegraphic speech. First
(a) A and B only (b) A and D only language acquisition refers to the way children learn
(c) B and C only (d) C and D only their native language. Second language acquisition
Ans. (a) : 'Estragon' and 'Pozzo' are the characters that refers to the learning of another language or languages
are figured in Samuel Beckett's 'Waiting for Godot'. besides the native language.
Estragon is one of the tulo men (Often referred to as For children learning their native language, linguistic
'tramps') who are waiting for Mr. Godot. He is the first competence develops in stages from babbling to one
to appear in the play and is more docile and timid than word to two words………
his friend Vladimir. Thus, the correct option is (d).
'Pozzo' is the bold, brutal, insensitive, and overbearing 69. Which pair of linguists in the following list is
figure who intimidates Estragon and Vladimir in the associated with ‘Speech Acts’?
first act of play after he drives his slave, Lucky, in the (a) Franz Boas and Rudolf Camap
(b) J.L.Austin and John Searle
scene. Hence, option (a) is correct.
(c) Noam Chomsky and Steven Pinker
66. Words with the same pronunciation and (d) Paul Grice and Michael Devitt
different meanings are: Ans. (b) : J.L. Austin and John Searle are linguists
(a) homonyms. (b) homografts. associated with Speech Acts. John L. Austin coined the
(c) homologues. (d) homophones. term 'Speech Act' in his book, How to Do Things With
Ans. (d) : Homophones : It is word with the same Words' published in 1962. According to Austin's
pronunciation and different meanings. For example preliminary informal description the idea of an
'Flower' and 'Flour' are homophones because they are 'Illocutionary act' can be captured by emphasizing that
pronounced the same but have different meanings. 'by saying something, we do something.'
67. Which of the following are true of the dramatic John R. Searle gave an alternative to Austin's
legacy of Ben Jonson? explanation of the illocutionary act saying, a "Speech
A. Jonson’s physiological interpretation of act" is often meant to refer to exactly the same thing as
the term illocutionary act.
character and personality did not have any
precedent. 70. Which two among the following condemned the
transportation of 50000 slaves into England in 1771?
B. Taking after the practice of the Moralities
A. Samuel Johnson
and Interludes, Jonson named his dramatis B. Alexander Pope
personae aptronymically. C. Horace Walpole
C. Chapman’s All Fools and Middleton’s A D. Thomas Gray
Trick to Catch the Old One belong to the Choose the correct answer from the options
genre of Comedy of Humours that Jonson given below:
is said to have pioneered. (a) A and B only (b) A and C only
D. John Marston and Thomas Dekker (c) B and C only (d) B and D only
collaborated with Jonson in writing for a Ans. (b) : 'Samuel Johnson' and 'Horace Walpole'
children’s company of players. condemned the transportation of 50000 slaves into
Choose the correct answer from the options England in 1771. In 1718, the transportation act
given below: introduced penal transportation. People convicted of
(a) A and B only (b) B and C only capital crimes had their sentences 'commuted' to 14
(c) C and D only (d) D and A only years of life in the Americas. Convicts found guilty of
UGC NTA NET/JRF English Paper II, 2021 (Sh.-II) 354 YCT
non-capital crimes received seven year sentences. • Research is systematic because there is a definite set
Between 1718 and 1776, over 50,000 convicts were of procedures and steps which you will follow.
transported to Virginia and Maryland in the modern • It is a planned procedure, not a spontaneous one.
united states. Thus, the correct answer is option (b). • Finding answer is the end of all research.
71. Who wrote the essay “My First Acquaintance • Questions are central to research. If there is no
with Poets”? question, then the answer is no use.
(a) Charles Lamb (b) John Ruskin Thus, the correct answer is option (b).
(c) Thomas De Quincey (d) William Hazlitt
Ans. (d) : William Hazlitt wrote the essay "My First 74. Which of these are generally taken to be true of
Acquaintance with poets", which was first published in Cultural Studies?
1823 in a short-lived but a highly significant periodical A. It is politically engaged.
of the Romantic Age. In this essay, Hazlitt discuss the B. It privileges text over context.
relationship between prose, poetry and gender issues, C. It has a symbiotic relationship with
arguing that Hazlitt undermines poetry and Coleridge as Formalism.
an idealized poet for him, whom he adored in his youth. D. It studies the means of production of a text.
Hence, option (d) is correct answer. Choose the correct answer from the options
72. Match List I with List II given below:
List I List II (a) A and C only (b) A and D only
(Writer) (Book) (c) A and B only (d) B and D only
A. Homi Bhabha I. Reading the popular Ans. (b) : Cultural studies is an academic subject that
B. T S Eliot II. The Location Of studies cultural phenomena in many civilizations by
Culture combining political, economics, communication,
C. Roland Barthes III. Notes Towards the sociology, social theory, literary theory, media theory,
Definition of Culture cinema studies, cultural anthropology, philosophy, art
D. John Fiske IV. Image-Music-Text history and other disciplines. 'It is politically engaged'
Choose the correct answer from the options and 'it studies the means of production of text' are
given below: generally taken to be true of cultural studies. Thus,
(a) A - I, B - II, C - IV, D - III correct option is (b).
(b) A - II, B - III, C - IV, D - I 75. Which of these themes best sums up the
(c) A - III, B - II, C - I, D - IV preoccupation of most of Vijay Tendulkar’s
(d) A - IV, B - II, C - III, D - I plays?
Ans. (b) : The appropriate match list of writer and their (a) Dynamics of media
books are- (b) Motivations of crime
List I (Writer) List II (Book) (c) Workings of love triangles
1. Homi Bhabha II. The Location Of Culture
(d) Workings of power
2. T. S. Eliot III. Notes Towards the
Definition of Culture Ans. (d) : The best theme of Vijay Tendulkar's plays are
3. Roland Barthes IV. Image-Music-Text workings of power, theme of gender discrimination and the
4. John Fiske I. Reading the Popular courtier system of India, violence stability in the society,
Thus, the correct option is (b). man-woman relationship and institution of marriage.
73. Given below are two statements: Vijay Dhondopan Tendulkar (1928-2008) was a leading
Statement I : All research being original and Indian playwright, movie and television writer, social
uninspired, it is rare for a researcher to begin a commentator primarily in Marathi. He is best known for
project by deriving ideas from predecessors. his plays- Shantata! Court Chalu Aahe (1967),
Statement II : Studying and documenting past Ghashiram Kotwal (1972), and Sakharam Binder
work on a research topic stifles the continual (1972).
expansion of human knowledge. Thus, correct option is (d).
In the light of the above statements, choose the 76. Who among the following is associated with a
most appropriate answer from the options ‘philosophy of praxis’?
given below: (a) Antonio Gramsci (b) Georg Lukacs
(a) Both Statement I and Statement II are correct. (c) Raymond Williams (d) Stuart Hall
(b) Both Statement I and Statement II are Ans. (a) : Antonio Gramsci is associated, with
incorrect.
'Philosophy of Praxis', In the 'Prison Notebooks'
(c) Statement I is correct but Statement II is
incorrect. Antonio Gramscie proposes the distinctive notion of a
(d) Statement I is incorrect but Statement II is 'Philosophy of Praxis : The interpretation of the
correct. significance of this suggestive formulation has
Ans. (b) : When we have read the two statements, we constituted a fertile field of discussion both of
find that both are not correct answer. Gramsci's approach to philosophical questions in his
• Research is an organized and systematic way of prison writings and more broadly, the nature of Marxist
finding answers to questions. philosophy. Thus, the option (a) is correct.

UGC NTA NET/JRF English Paper II, 2021 (Sh.-II) 355 YCT
77. Which of the following did Owuor Anyumba, 80. Arrange chronologically the following texts in
Taban Lo Liyong and Ngugi wa Thiongo object terms of their years of first publication:
to in 1968? A. Edmund Spenser’s The Faerie Queene
A. the primacy of English literatures and B. Coleridge and Wordsworth’s Lyrical
cultures Ballads
B. the centrality of Africa in the Department C. Pablo Neruda’s Canto General
of English D. Charles Baudelaire’s The Flowers of Evil
C. the primacy of orature in the syllabus Choose the correct answer from the options
D. the focus on the study of the historic given below
continuity of English literature (a) A, B, C, D (b) A, B, D, C
Choose the correct answer from the options (c) B, C, A, D (d) D, A, B, C
given below: Ans. (b) : The appropriate chronological order of texts
(a) A and D only (b) A and B only in terms of their years of first publication.
(c) B and C only (d) C and D only Chronological Order of Texts 1st Publication
Ans. (a) : 'Taban Lo Liyong' is a poet and writer of Year
fiction and literary criticism from South Sudan. Post 1. Edmund Spenser’s The Faerie 1. 1596
colonial students of English in Africa to question the Queene
practices of their discipline, Liyong, Owuor Anyumba, 2. Coleridge and Wordsworth’s 2. 1798
and wa Thiongo were criticized for advocating cultural Lyrical Ballads
or even racial purity within academia. In 1968, Owuor 3. Charles Baudelaire’s The 3. 1857
Anyumba, Taban Lo Liyong and Ngugi wa Thiongo Flowers of Evil
object to 'the primacy of English literatures and cultures' 4. Pablo Neruda’s Canto General 4. 1950
and 'The focus on the study of the historic continuity of
Thus, the correct answer is option (b).
English Literature'.
Hence, option (a) is correct. 81. Which of the following does Urvashi Butalia’s
The Other Side of Silence primarily seek to do?
78. Match List I with List II A. To understand the Partition as something
List I List II more than a political divide
(Poem) (Poet) B. To foreground a personal history of the
A. "The road not taken" I. Pablo Neruda Partition
B. "Tonight I can write II. Robert frost C. To foreground the Partition as an event
the saddest Lines" more tragic than the Holocaust
C. "I hear America III. Langston D. To find and unite families separated at the
Singing" Hughes Partition
D. "I, too, sing America" IV. Walt Whitman Choose the correct answer from the options
Choose the correct answer from the options given below:
given below: (a) A and B only (b) A and D only
(a) A - I, B - II, C - III, D - IV (c) B and C only (d) C and D only
(b) A - II, B - I, C - IV, D - III Ans. (a) : Urvashi Butalia’s 'The other side of silence'
(c) A - III, B - II, C - IV, D - I (1998) primarily seek 'to understand the partition as
(d) A - IV, B - I, C - II, D - III something more than a political divide' and 'to
Ans. (b) : The appropriate match of poem with their foreground a personal history of the partition'.
poets are– 'The Other Side of Silence' is the product of more than
List I (Poem) List II (Poet) seventy interviews conducted with survivors of the
1. The road not taken II. Robert frost partition, is being used as an academic text in some
2. Tonight I can write the I. Pablo Neruda Indian University. Uravashi Butalia (born 1952) is an
saddest Lines Indian feminist writer, publisher and activist. She is
3. I hear America Singing IV. Walt Whitman known for 'women's voices from Kashmir'. Thus
4. I, too, sing America III. Langston Hughes appropriate answer is option (a)
Thus, the appropriate answer is option (b). 82. Usage in “You have hissed the mystery
79. Which of the following terms describes a novel
lectures.” is an example of:
of fashionable high life in 19th-century English
literature? (a) error of lexical choice.
(a) Brass-Spittoon (b) Diamond-Jar (b) inadvertent mistake.
(c) Golden-Spoon (d) Silver-Fork (c) metathesis.
Ans. (d) : Fashionable novels, also called 'Silver-Fork' (d) spoonerism.
novels, were a 19th century genre of English literature Ans. (d) : "You have hissed the mystery lectures" is an
that depicted the lives of the upper class and aristocracy. example of Spoonerism. A spoonerism is a speech error
The term "Silver-fork" to describe these Victorian in which the speaker switches the initial consonants of
fashionable novel was coined by William Hazlitt in an two consecutive words. If you say "bunny phone"
article on "The Dandy school" in 1827, at the genre's instead of "funny bone" 'you have hissed all my mystery
heyday. lectures' can be corrected as 'you have missed all my
Thus, the correct answer is option (d). history lectures'. Thus, correct option is (d).

UGC NTA NET/JRF English Paper II, 2021 (Sh.-II) 356 YCT
83. Which of the following is true of mass media? The Movement Poets- The Movement was a term
(a) It usually has a central, single source. coined in 1954 by J.D. Scott, literary editor of 'The
(b) It can affect a localised population only. Spectator' to describe a group of writers-
(c) It usually has multiple sources. Philip Larkin, Kingsley Amis, Donald Davie, John
(d) Its audience is in close proximity to its Wain and so on.
source. Thus, the correct option is (a).
Ans. (a) : Mass Media plays a crucial role in shaping 86. Which of the following fictional characters is
how we view the world. Intensive use of Mass Media believed to be based on the 15th-century real-
has resulted in the world to appear smaller and closer. It life character, Vlad the Impaler?
also promotes the distribution of goods and services. The (a) Count Dracula (b) Peter Quint
fundamental objectives of Mass Media are to inform, (c) Prince Manfred (d) Victor Frankenstein
educate and entertain the masses. Thus, the true feature of
Mass Media is it usually has a central, single source'. Ans. (a) : 'Count Dracula' fictional character is believed
to be based on the 15th century real-life character, Vlad
84. Who, among these, are songwriters who have
the Impaler, 'Vlad the Impaler' was a medieval prince
been awarded the Nobel Prize for Literature?
A. Elton John whose bloodthirsty acts inspired the world's most
B. Rabindranath Tagore famous vampire 'Bran Stoker's Dracula.
C. Bob Dylan 87. Who wrote the short story, “The Voter”?
D. Bob Marley (a) Chimamanda Ngozi Adichie
Choose the correct answer from the options (b) Chinua Achebe
given below: (c) Ngugi wa Thiong’o
(a) A and D only (b) A and B only (d) Wole Soyinka
(c) B and C only (d) C and D only Ans. (b) : Chinua Achebe wrote the short story "The
Ans. (c) : 'Rabindranath Tagore' and 'Bob Dylan' are Voter". 'The voter discusses the theme of corruption,
songwriters who have been awarded the Nobel Prize for loyality, guilt, power, greed and tradition. It tells also
literature. about old customs and monetary bribes colliding to rig a
The Nobel Prize in Literature 1913 was awarded to local village election, which looks at the relationship
Rabindranath Tagore (Gitanjali) 'because of his between African culture while exposing the conflict
profoundly sensitive, fresh and beautiful verse by between traditional African beliefs and the modernism
which, with consummate skill, he has made his thought, introduced by British colonialism.
expressed in his own English word, a part of the Chinua Achebe was a Nigerian novelist, poet and critic
literature of the west. who is regarded as the dominant figure of modern African
The Swedish Academy awarded Dylan in 2016, the literature. Thus, the appropriate answer is option (b).
Nobel Prize in Literature for having created new poetic
expressions within the great American song tradition. 88. In his recasting the canon of English poetry in
Thus, the correct answer is option (c). New Bearings in English Poetry which of the
85. Arrange the following groups of poets in their following pairs was downgraded by F.R.
chronological sequence in relation to English Leavis?
literary history: (a) Browning and Arnold
A. The Imagist poets (b) Milton and Shelley
B. The Cavalier poets (c) Pound and Hopkins
C. The Movement poets (d) Tennyson and Swinburne
D. The Lake poets Ans. (b) : In 'New Bearing in English Poetry' (1932) he
Choose the correct answer from the options (F.R. Levis) attacked English late Victorian poetry and
given below proclaimed the importance of the work of T.S. Eliot,
(a) B, D, A, C (b) B, D, C, A Ezra Pound and G.M. Hopkins. Emphasizing wit and
(c) D, A, B, C (d) D, B, A, C the play of intellect rather than late Romantic
Ans. (a) : The correct chronological sequence of poets sensuousness. If 'New Bearing in English Poetry'
and their relation to English literary history are:–The leavis's dismissal of most late romantic poetry (like,
Cavalier poets → The Lake poets → The Imagist poets Milton and Shelley) and his welcoming of the
→ The Movement poets. modernist. Thus, the correct option is (b).
Cavalier Poets- The cavalier poets were a group of 89. Who, in “The World as India,” argues that
writers from the 17th century in England who supported English can be the only common “unifying
Charles I during the English Civil War. language” of India?
The Lake Poets- William Wordsworth, S.T. Coleridge (a) Anthony Burgess (b) C. K. Ogden
and Robert Southey are known as Lake school poets. (c) Noam Chomsky (d) Susan Sontag
They belong to 19th century.
Ans. (d) : Susan Sontag (16 Jan 1933-28 Dec 2004)
The Imagist poets- Ezra Pound is noted as the founder was an American writer, film maker, philosopher,
of Imagism, the movement was rooted in ideas first teacher and political activist. She is best known for her
developed by English philosopher and poet T.E. Hulme. essays on modern culture. 'The World as India' is a
It started in early 20th century. lecture on Literary Translation in which she argues that
UGC NTA NET/JRF English Paper II, 2021 (Sh.-II) 357 YCT
English can be only common 'unifying language' of (c) Social oppression in the third world is a
India. In her essay, she states, "Just because it is alien, matter more of race than of class.
foreign, it can become the unifying language of a (d) The African bourgeoisie can never succeed in
permanently diverse people. The only language that all the task of nation building.
Indians might have in common not only is, it has to be, Ans. (d) : The statement which best articulates Frantz
English." Fanon’s political position will be 'The African
Thus, the correct answer is option (d). bourgeoisie can never succeed in the task of nation
90. Which two of the following are fallacious building'. Frantz Omar Fanon (20 July 1925-6 Dec
evaluations of poetry according to Matthew 1961) was a French West Indian psychiatrist and
Arnold’s “The Study of Poetry”? political philosopher from the French colony of
A. contextual estimate Martinique his works have become influential in the
B. personal estimate fields of post-colonial studies, critical theory and
C. comparative estimate Marxism. He is known for his theory that some
D. historic estimate neuroses are socially generated and for his writings on
Choose the correct answer from the options behalf of the national liberation of colonial peoples.
given below: Hence option (d) is correct.
(a) A and B only (b) B and C only 93. With which of the following movements is
(c) B and D only (d) C and D only Charles Baudelaire’s Flowers of Evil generally
Ans. (c) : 'Personal Estimate' and 'Historic Estimate' are associated?
fallacious evaluations of poetry according to Matthew A. Neo-classical
Arnold's B. Symbolist
'The Study of Poetry' Matthew Arnold (24 Dec 1822- C. Modernist
15 April 1888) was an English poet and cultural critic. D. Postmodernist
'The Study of Poetry' was perhaps Arnold's most Choose the correct answer from the options
famous piece of essay on literary criticism. given below:
'The Historic Estimate' places the historical context (a) A and D only (b) A and B only
above the value of the art itself. 'The Personal Estimate' (c) B and C only (d) C and D only
on the other hand depends on the personal taste, the Ans. (c) : Charles Baudelaire’s 'Flowers of Evil' is
likes and dislikes of the reader which affects his generally associated with 'Symbolist' and 'Modernist'
judgment of poetry. Arnold says that both these movements. 'Les Fleurs du Mal' (The Flowers of Evil)
estimates tend to be fallacious. is a volume of French poetry by Charles Baudelaire, it
Thus, the correct answer is option (c). includes nearly all of Baudelaire's poetry, written in
1840 & ending with his death in August 1867. It is his
91. Which of these are true of Raymond Williams’
most famous work in which he expresses the fleeting
Culture and Society? ephemeral, ever-changing nature of beauty in modern,
A. It critiques the idea of high culture. industrializing Paris during the 19th century. First
B. It overlooks the idea of high culture. published in 1857, it was important in the Symbolist
C. It defines culture as a way of life. and modernist movements. Hence option (c) is correct.
D. It equates culture with science.
94. Arrange the following terms in their
Choose the correct answer from the options
chronological sequence of appearance:
given below: A. dissociation of sensibility
(a) A and C only (b) A and D only B. unreliable narrator
(c) A and B only (d) B and D only C. theatre of cruelty
Ans. (a) : Raymond Williams' Culture and Society, D. egotistical sublime
criticizes the idea of high culture and defines culture as Choose the correct answer from the options
a way of life. 'Culture and Society' is a book published given below
in 1958 by Welsh progressive writer Raymond (a) B, D, A, C (b) D, A, B, C
Williams', exploring how the notion of culture (c) D, A, C, B (d) D, B, A, C
developed in Great Britain, from the 18th through the 20th Ans. (c) : The correct chronological sequence of
centuries. According to him, culture is what differentiates appearance will be- D, A, C, B
one group of society from the next. Different societies Egotistical Sublime → Dissociation of Sensibility →
have different cultures, however it is important not to Theatre of Cruelty → Unreliable Narrator
confuse the idea of culture with society. 'Egotistical Sublime' is first used by Keats in a letter to
Hence option (a) is correct. Richards Woodhouse, dated 27 oct 1818. He used this
92. Which of the following statements best phrase to describe his version of Wordsworth's
articulates Frantz Fanon’s political position? distinctive genius.
(a) Colonialism will die a natural death sans any 'Dissociation of Sensibility' phrase is used by T.S. Eliot
violent struggle against it. in the essay 'The Metaphysical Poets' (1921) to explain
(b) Peasants and social outcasts have little the change that occurred in English poetry after the
revolutionary potential in Africa. heyday of the Metaphysical poets.
UGC NTA NET/JRF English Paper II, 2021 (Sh.-II) 358 YCT
'Theatre of Cruelty' is a form of theatre generally Choose the correct answer from the options
associated with Antonin Artaud, (Between 1931 and given below
1936) which aimed to shock the senses of its audience, (a) B, C, A, D (b) B, C, D, A
sometimes using violent and confronting images that (c) C, B, A, D (d) C, D, B, A
appealed to the emotions. Ans. (d) : The correct chronological order of
'Unreliable Narrator' was first coined by Wayne C.
publication of the following journals are- C, D, B, A
Booth in his 1961 book 'Rhetoric of Fiction.
'Blackwood's Magazine → Bentley's Miscellany →
95. Which book by J.G. Ballard is about a virus
that freezes anything it comes in contact with? Cornhill Magazine → Longman's Magazine
(a) Concrete Island (b) Kingdom Come Blackwood's Magazine' was a British magazine printed
(c) The Crystal World (d) The Drowned World between 1817 and 1980. It was founded by the
Ans. (c) : 'The Crystal World' by J.G. Ballard is about a publisher William Blackwood and was originally called
virus that freezes anything it comes in contact with. The Edinburgh Monthly Magazine. 'Bentley's Miscellany'
Crystal World' is a science fiction novel, published in was an English literary magazine started by Richard
1966. Its author 'James Graham Ballard' (15 Nov. 1930- Bentley . It was published between 1836 and 1868.
19 April 2009) was an English novelist, short-story 'The Cornhill Magazine' was a monthly Victorian
writer, satirist and essayist who first associated with the magazine and literary journal which was published
New Wave of science fiction. The novel tells the story of a between 1859 and 1975.
physician trying to make his way deep into the jungle to a 'Longmans' Magazine' was first published in November
secluded leprosy treatment facility. While trying to make it 1882 by C.J. Longman.
to his destination, his chaotic path leads him to try to come 99. Who is the creator of the character, Julien
to terms with an apocalyptic phenomenon in the jungle that
Sorel?
crystallizes everything it touches.
Hence, the correct option is (c). (a) Balzac (b) Flaubert
(c) Moliere (d) Stendhal
96. In “The Function of Criticism at the Present
Time” what is proposed by Matthew Arnold as Ans. (d) : Stendhal is the creator of the character, Julien
the essence of criticism? Sorel. Julien Sorel is the protagonist of 'The Red and
(a) affirmation (b) cohesiveness The Black' by Stendhal, published in 1830. Marie-Henri
(c) disinterestedness (d) judiciousness Beyle, better known by his pen name Stendhal, was a
Ans. (c) : 'Disinterestedness' was proposed by Matthew 19th century French writer. Le Rouge et le Noir (The
Arnold as the essence of criticism in his book 'The Red and the Black) is a historical psychological novel in
Function of Criticism at the Present Time' was two volumes by Stendhal. It chronicles the attempts of a
published in 1865. Matthew Arnold was an English poet provincial young man to rise socially beyond his modest
and cultural critic according to him, a critic must upbringing through a combination of talent, hard work,
maintain a position of 'disinterestedness', i.e. keeping deception and hypocrisy.
aloof from 'the practical view of things' in order to Hence the correct answer is option (d).
know the best that is known and thought in the world 100. Match List I with List II
and in turn making this known, to create a current of
true and fresh ideas'. List I List II
Hence the correct answer is option (c). (Book) (Poet)
97. Who is the author of The Otherness of English: A. Anniversaries I. Abraham Cowley
India’s Auntie Tongue Syndrome? B. The Temple II. John Donne
(a) Binoo K. John (b) Yamuna Kachru C. The Rehearsal III. George Herbert
(c) Probal Dasgupta (d) S.K.Verma Transpros'd
Ans. (c) : Probal Dasgupta is the author of 'The D. Pindarique Odes IV. Andrew Marvell
Otherness of English' : India's Auntie Tongue Choose the correct answer from the options
Syndrome,' published in 1993. He is an Indian linguist, given below:
Esperanto speaker and activist. 'The Otherness of (a) A - I, B - IV, C - II, D - III
English' offers a unique interpretation of the content and (b) A - II, B - III, C - IV, D - I
usage of the English language in India, specifically (c) A - III, B - I, C - IV, D - II
commenting on the mode of its presence. The author (d) A - IV, B - II, C - I, D - III
presents an inter disciplinary account of the role English Ans. (b) : The correct list will be
plays in the general process of modernization. Hence
option (c) is correct. (Book) (Poet)
A. Anniversaries (1601) II. John Donne
98. Arrange the following journals in the
chronological order of publication. B. The Temple (1633) III. George Herbert
A. Longman’s Magazine C. The Rehearsal Transpros'd IV. Andrew Marvell
B. Cornhill Magazine (1672)
C. Blackwood’s Magazine D. Pindarique Odes (1656) I. Abraham Cowley
D. Bentley’s Miscellany Hence the correct option is (b).
UGC NTA NET/JRF English Paper II, 2021 (Sh.-II) 359 YCT
UGC NTA NET/JRF Exam. Dec. 2020/June 2021
ENGLISH-II
SOLVED PAPER [ Exam Date : 01.12.2021, Shift-I

Read the following passage and answer the Read the following poem and answer the
questions that follow: questions that follow:
What can be the use of a poetry that has no true No worst, there is none.
subject except the poet's own selfhood? The No worst, there is none. Pitched past pitch of grief,
traditional use of poetry in the Western world has More pangs will, schooled at forepangs, wilder wring.
been instruction through delight, where teaching has Comforter, where, where is your comforting?
meant the common truths or common deceptions of Mary, mother of us, where is your relief?
societal tradition, and where esthetic pleasure has My cries heave, herds - long; huddle in a main, a chief
meant a fulfillment of expectations founded upon past Woe, world - sorrow; on an age-old anvil wince and
joys of the same design. sing - Then lull, then leave off. Fury had shrieked 'No
But an individual psyche has its own accidents, lingering!
which it needs to call truths, and its own necessity for Let me be fell: force I must be brief."
self-recognition, which requires the pleasures of O the mind, mind has mountains; cliffs of fall
Frightful, sheer, no-man-fathomed. Hold them cheap
originality, even if those pleasures depend upon a
May who ne'er hung there. Nor does long our small
kind of lying against time, and against the Durance deal with that steep or deep. Here! creep,
achievements of the past. The use of such poetry Wretch, under a comfort serves in a whirlwind: all
demands to be seen in a deidealized way, if it is to be Life death does end and each day dies with sleep.
seen more truly. ‐ Gerard Manley Hopkins
‐ Harold Bloom, "The Use of Poetry" 4. Which of the following best describes the
1. In the context of the above which is closest to meaning of the title of the poem, 'No worst,
being true? there is none.'?
(a) There can be no use of poetry. (a) It is not worst because there is nothing.
(b) Poetry can have no true subject. (b) Nothing can be so much bad as this.
(c) Man may be the true subject of poetry. (c) No it is worst as nothing is there.
(d) The poet himself may be the subject. (d) It is very bad as no one is there.
Ans. (d) : In the context of the given excerpt from "The Ans. (b) : The meaning of the title of the poem "No
Use of Poetry" by Harold Bloom, the poet himself may worst, there is none" is best described by "Nothing can
be the subject' is closest to being true. be so much bad as this".
Harold Bloom has discussed the subject of the poetry 'No worst, there is none' is Gerard Manley Hopkins's
and use of the poetry it has no true subject. terrible sonnet describes the nature of a speaker's
2. What is meant by 'traditional use of poetry'? depression and its highs and lows.
(a) Promoting serious ideas 5. Beyond the intensity of known grief, there can
(b) Promoting doubtful pleasure be:
(c) Promoting joyous expectations (a) no grief than being experienced.
(d) Promoting social honesty in life (b) only a new pain more painful.
Ans. (c) : In the context of the given passage (c) only the twisted known pains.
(d) the griefs beyond limits of pain.
"Traditional use of poetry" means "Promoting joyous
expectations". Ans. (c) : Beyond the intensity of known grief, there
Harold Bloom described "the traditional use of poetry in can be 'only twisted known pains'.
Western World" as instruction through delight and Thus, the correct answer is option (c).
esthetic pleasure as a fulfillment of expectations 6. Which two of the following are true?
founded upon past joys. A. Not all know the intensity or depth of suffering.
B. Death does not put an end to our sufferings.
3. If 'selfhood' of a poet is the subject of poetry, C. Suffering is seen as winds that hinder comfort.
then 'originality' shall spring from: D. Suffering's intensity or depth is in the mind.
(a) some truth of untruths. Choose the correct answer from the options
(b) truth of self-recognition. given below:
(c) creating ideal pleasures. (a) A and B only (b) C and D only
(d) re-living the past joys. (c) A and D only (d) B and D only
Ans. (a) : If 'Selfhood' of a poet is the subject of poetry, Ans. (c) : According to the excerpt from the given
then 'Originality shall spring from some truth of poem, lines given (A) and (D) are absolutely true.
untruths. Not all know the intensity or depth of suffering.
Thus, the correct answer is option (a). Suffering's intensity or depth is in the mind.
UGC NTA NET/JRF English Paper II, 2021 (Sh.-I) 360 YCT
Read the following extract and answer the questions 11. In "An Apology for Poetry" Sidney discusses
that follow: the didactic function of poetry by comparing it
'Justice' was done, and the President of the to philosophy and:
Immortals , in Aeschylean phrase, had ended his (a) religion. (b) aesthetics.
sport with Tess. And the D'Urberville knights and (c) history. (d) ethics.
dames slept on in their tombs unknowing. The two Ans. (c) : In 'An Apology for Poetry' Sidney discusses
speechless gazers bent themselves down to the earth, the didactic function of poetry by comparing to
as if in prayer, and remained thus a long time, philosophy and history.
absolutely motionless; the flag continued to wave 'The Defense of Poesy' is a literary criticism by Sir
silently. As soon as they had enough strength they Philip Sidney, written about 1580 and published
arose, joined hands again, and went on. posthumously in 1595.
--Thomas Hardy, Tess of the D’Urbervilles It is considered the finest work of Elizabethan Literary
7. How did the 'sport with Tess' end? Criticism, Sidney's elegant essay suggests the literature
(a) She was hanged. is a better teacher than history or philosophy, and it
(b) She was expelled from Wintoncester. masterfully reflects Plato's infamous decision to ban
(c) The tormentor married her. poets from the state in his Republic.
(d) She died an untimely death. Sidney composed his eloquent defense of imaginative
Ans. (a) : The 'sport with Tess' ended as 'she was literature against charges of time wasting prevarication
hanged'. and allurement to vice.
This passage is the last paragraph of chapter LIX at the 12. According to Longinus which two of the
close of "Tess of the d'Urbervilles" by Thomas Hardy in following qualities apply to 'great poetry'?
which Tess is executed for murder, and the black flag at A. It must be the work of genius, an inspired
the prison indicates that execution has been done. person.
8. Who are the 'two speechless gazers'? B. It must cause a feeling of melancholy in the
(a) Reverend James Clare and Mrs. Brooks reader.
(b) 'Liza-Lu and Angel Clare C. It must employ devices of rhetoric.
(c) Tess's two parents D. It must please selectively and on special
(d) Parson Tringham and Mrs. d'Urberville occasions.
Ans. (b) : 'Two speechless gazers' are 'Liza-Lu and Choose the correct answer from the options
Angel Clare'. given below:
In the final scene, 'Liza-Lu and Angel Clare' wait (a) A and B only (b) A and C only
outside of a prison. They do not witness the execution. (c) B and D only (d) C and D only
They wait and watch the building until a black flag is Ans. (b) : The qualities to 'great poetry' according to
raised on a pole outside the prison tower to indicate that Longinus are:
execution has been done. It is must be the work of genius, an inspired person.
Read the following extract and answer the questions It must employ devices of rhetoric.
that follow: According to Longinus there are five principal sources
The solemn temples, the great globe itself, of the sublime. These sources are:
Yea, all which it inherit, shall dissolve (i) Grandeur of thought: (It is largely the gift o
And, like this insubstantial pageant faded, nature. It is the echo of a great soul).
Leave not a rack behind. We are such stuff (ii) Strong emotion: (Strong motion is an inborn gift
As dreams are made on, and our little life of a genius)
Is rounded with a sleep. (iii) The use of figures: (Figure of speech/devices of
–Shakespeare, The Tempest rhetoric)
9. '[T]his insubstantial pageant' refers to: (iv) Noble diction: (Choice of proper words)
(a) the shutdown of Globe theatre. (v) Dignified composition: (Harmonious arrangement
(b) a non-real performance. of words)
(c) the destroyed mother earth. Note:- The grandeur of thought and 'strong emotion' is
(d) enactment with support structure. inborn gifts of nature. The rest three sources are the
gifts of art.
Ans. (b) : 'This insubstantial pageant' refers to 'a non-
real performance' 13. In "The Function of Criticism" T.S.Eliot
Prospero's speech in Act-IV, Scene- I of The Tempest attacked J. Middleton Murry and similar
by Shakespeare emphasizes both the beauty of the world critics for being devotees of what he called:
he has created for himself and the sadness of the fact that (a) "the Inner Voice"
this world in many ways meaningless because it is a kind (b) "the Romantic Impulse"
of dream completely removed from anything substantial. (c) "the Symbol Hunt"
(d) "the Muse's Mystery"
10. 'We are such stuff as dreams are made on' Ans. (a) : In "The Function of Criticism" T.S. Eliot
means: attacked J Middleton Murry and similar critics for being
(a) Human life is full of imaginary colours. devotees of what he called "the inner voice." He saw
(b) Our life is a text of what happened. them as individuals whose unwillingness to accept
(c) We are a bundle of past reality external standards of literary taste and propriety could
(d) There is no substance to human life. only corrupt contemporary critical intelligence, which
Ans. (d) : 'We are such stuff as dreams are made on' in Eliot's view, required objectified measures of quality,
means 'There is no substance to human life'. This line not an enthusiastic responsiveness to what the critic
appears in Act 4 of 'The Tempest' spoken by the found pleasing on personal grounds and for no other
magician Prospero. reason.
UGC NTA NET/JRF English Paper II, 2022 (Sh.-I) 361 YCT
'Function of Criticism', by T. S. Eliot, published in Ans. (d) : In Noam Chomsky's definition of grammar,
1923, basically a response to Middleton Murry's essay 'transformation' and 'generation' are two features, drawn
'Romanticism and the Tradition'. The essay revisits from mathematics. Chomsky's theories of grammar and
Tradition and the Individual Talent, opposes the view of language are often referred to as "generative",
Murry and describes the function of literary criticism "transformational", or "Transformational-generative".
and a good critic. In a mathematical sense, "generative" simply means
14. Who among the following compared 'the mind "formally explicit". In the case of language, however,
in creation' to 'a fading coal'? the meaning of the term typically also includes the
(a) Wordsworth (b) Coleridge nation of "productivity" i.e. the capacity to produce an
(c) Shelley (d) Keats infinite number of grammatical houses and sentences
Ans. (c) : Percy Bysshe Shelley compared 'the mind in using only finite means.
creation' to 'a fading coal' in his essay "A Defence of Poetry" Chomsky's theories are "transformational" in the sense
written in 1821 and published posthumously in 1840. that they account for the syntactic and semantic
By comparing the mind of the poet to the image of a properties of sentence by means of modification of
fading coal, Shelley argues that poets do not rely on structure of a phrase in the course of its generation.
"will power" and "determination", as in the case with 18. 'Potato' is a sixteenth-century borrowing into
logic or reason; instead, they focus on beauty of English from:
everything in nature. Thus, they have no control over (a) Spanish. (b) French.
when or how inspiration will strike. (c) German. (d) Norwegian.
Shelley wrote 'A Defence of Poetry' in response to an Ans. (a) : 'Potato' is a sixteenth-century borrowing into
essay "The Four Ages of Poetry' written by his friend, English from Spanish.
Thomas Love Peacock. TAINO (language)  → Spanish (patata) →
Mid16th
English (Potato)
batata sweet potato  century

15. Who among the following considered From Spanish Patata, variant of Taino batata, in mid
paraphrase as 'a heresy'? 16th it gained its current name potato is English.
(a) Percy Lubbock (b) Edmund Wilson 19. Who among the following represents the global
(c) I.A. Richards (d) Cleanth Brooks spread of English diagrammatically as three
Ans. (d) : Cleanth Brooks considered paraphrase as 'a concentric circles?
heresy' (a) David Crystal (b) Jenny Cheshire
"The Heresy of Paraphrase" is the title of a chapter in (c) Braj B. Kachru (d) Salikoko Mufwene
The Well-Wrought Urn, a seminal work of the New Ans. (c) : Braj B. Kachru represents the global spread
Criticism by Cleanth Brooks. of English diagrammatically as three concentric circles.
Cleanth Brooks in this essay tries to put forward that Kachru's model was first introduced with the term
any reductionist attempt to transform poetic meaning to "World Englishes" in 1985, opening the door for new
a prose statement such as descriptive or thematic way of understanding the spread of English language
interpretative is to do injustice to a poem. It is one's throughout the world.
failure to recognize the poem as a poem. Kachru (1985) described the distribution of English in
The heresy is that a assuming that the meaning of a work relation to three concentric circles : the inner circle, the
of art (particularly of a poetry) can be paraphrased. outer circle and the expanding circle.
16. Empiricist linguistics is concerned with :
(a) investigation of the human mind.
(b) directly observable sense-data.
(c) reason as a determinant of enquiry.
(d) innate language knowledge.
Ans. (b) : Empiricist linguistics is concerned with
directly observable sense-data.
The term empiricism derived from the ancient Greek
word empeiria, "experience".
The meaning of the term (in philosophy) is the view that The circles signify "the type of spread the pattern of
acquisition and the functional domains in which English
all concepts originate in experience, that all concepts are language is used across cultures and languages".
about or applicable to things that can be experienced, or 20. Who among the following is the founder of the
that all rationally acceptable beliefs or propositions are Survey of English Usage (SEU)?
justifiable or knowable only through experience. (a) Randolph Quirk (b) Henry Watson Fowler
Empiricists maintain that children learn the syntax of (c) Michael Swan (d) Bryan Garner
their language by using only general learning capacities Ans. (a) : Randolph Quirk is the founder of the
through experience (their senses). Survey of English usage (SEU). The Survey of English
17. In Noam Chomsky's definition of grammar Usage (the survey) was founded in 1959 in which many
which two features are drawn from well-known linguists have spent time doing research at
mathematics ? the survey. Among them are : Valerie Adams, John
A. complexity B. abstraction Algeo, David Crystal, Sidney Greenbaum, Joe Taglicht
C. transformation D. generation and many others.
Choose the correct answer from the options The survey has produced the grammatical and syntactic
given below: annotation schemes for the ICE (International Corpus of
(a) A and B only (b) A and C only English) corpora as well as numerous software
(c) B and D only (d) C and D only packages to support the compilation of the project.
UGC NTA NET/JRF English Paper II, 2022 (Sh.-I) 362 YCT
21. Who is the author of the short story, 25. Who among the following edited The Cornhill
"Beethoven Was One-Sixteenth Black" ? Magazine?
(a) J. M. Coetzee (b) Nadine Gordimer (a) Charles Dickens
(c) Andre Brink (d) Bessie Head (b) Lewis Carroll
Ans. (b) : Nadine Gordimer is the author of the short (c) William Makepeace Thackeray
story, "Beethoven was One Sixteenth Black". (d) Anthony Trollope
In this collection of new stories, Beethoven was One- Ans. (c) : The Cornhill Magazine (1860-1975) first
sixteenth Black, Nadine Gordimer crosses the frontiers edited by William Makepeace Thackeray. It was
of politics, memory, sexuality and love with the fearless monthly Victorian magazine and literary journal, first
insight that is the hallmark of her writing. published in 1860 from London.
Nadine Gordimer was a South African writer and William Makepeace Thackeray (born 1811, Calcutta-died
political activist. She received Nobel Prize in 1991 and 1863, London) was an English novelist author and
Booker prize in 1974. illustrator. In 1860 he founded the Cornhill Magazine and
22. Arrange the following texts in the chronological became its first editor for the magazine which was published
order of publication. by British publishing company Smith, Elder & Co.
A. This Bridge Called My Back 26. Which of the following novels has its epigraph
B. Sexual Politics taken from the Katha Upanishad?
C. Gender Trouble (a) The Island of Doctor Moreau by H. G. Wells
D. The Feminine Mystique (b) The Razor’s Edge by Somerset Maugham
Choose the correct answer from the options (c) Point Counter Point by Aldous Huxley
given below (d) A Room with a View by E. M. Forster
(a) B, D, A, C (b) D, B, A, C Ans. (b) : The Razor's Edge is a 1944 novel by W.
(c) D, A, B, C (d) B, D, C, A Somerset Maugham. The novel's title comes from a
Ans. (b) : Chronological order of publication of the translation of a verse in the Katha Upanishad,
given texts paraphrased in the book's epigraph as: "The sharp edge
Order Texts Year Work by of a razor is difficult to pass over; thus the wise say the
(Books/works) (writer) path to salvation is hard".
D The Feminine 1963 Betty Friedan The novel is concerned in large part with the search for
Mystique the meaning of life and with the dichotomy between
B Sexual Politics 1970 Kate Millett materialism and spirituality. It was one of the first
A This Bridge Called 1981 Edited by : western novel to propose non-western solutions to
my Back : Writings Cherrie Moraga society's ills.
by Radical Women Gloria & 27. Who is the author of the essay, "What Isn't
of Color Anzaldua Literature?" ?
C Gender Trouble 1990 Judith Butler (a) E. D. Hirsch Jr. (b) Paul Ricoeur
Note- This Bridge Called My Back is a feminist (c) Hans-Georg Gadamer(d) Terry Eagleton
anthology (a book that contains pieces of writing or Ans. (a) : E.D. Hirsch Jr. is the author of the essay
poems, often on the same subject, by different authors) 'What Isn't Literature?'
23. Who is the author of the essay "Lear, Tolstoy Eric Donald Hirsch Jr. is an American literary critic and
and the Fool"? educator who is best known for his Cultural Literacy:
(a) Aldous Huxley (b) George Orwell What Every American Needs to Know (1987). He also
(c) Virginia Woolf (d) Somerset Maugham wrote 'The Dictionary of Cultural Literacy (1988)' and
Ans. (b) : " Lear, Tolstoy and the Fool" is an essay by was the main editor of 'A First Dictionary of Cultural
George Orwell. It was inspired by a critical essay on Literacy' (1989).
Shakespeare by Leo Tolstoy, and was first published in 28. Erich Auerbach's Mimesis (1946) ends with a
Polemic (a British Magazine of philosophy, psychology chapter on:
and Aesthetics) March 1947. (a) Virginia Woolf's To the Lighthouse.
In this essay Orwell analyzes Tolstoy's criticism of (b) Wyndham Lewis's The Apes of God.
Shakespeare's work in general and his attack on King (c) James Joyce's Ulysses.
Lear in particular. (d) George Eliot's Middlemarch.
24. Which of the following poems by Robert Ans. (a) : Erich Auerbach's Mimesis (1947) ends with a
Browning contains the lines, "Our interest's on chapter on Virginia Woolf's "To The Lighthouse".
the dangerous edge of things. /The honest thief, Mimesis : The Representation of Reality in Western
the tender murderer, / The superstitious Literature is a book of literacy exoticism and contains
atheist. ...."? twenty chapters, each one anchored to a characteristic
(a) "A Death in the Desert" passage from theological or literary work, which is then
(b) "Count Gismond" tested for tore, direction and syntax, and enfolded
(c) "Bishop Blougram's Apology" within a specific historical context.
(d) "Love Among the Ruins" Title of the last chapter of Mimesis is "The Brown
Ans. (c) : The lines, "Our interest's on the dangerous stocking" in which "To The Lighthouse" by Virginia
edge of things. /The honest thief, the tender murderer, / Woolf and "In Search of Lost Time" by Marcel Poust is
The superstitious atheist....." have been taken from discussed chiefly.
Robert Browning's poem "Bishop Blougram's 29. Which of the following clusters is associated
Apology". Bishop Blougram's Apology is a long poem, with what Julia Kristeva terms the 'Semiotic'?
published in the two volume collection Men and (a) Authority, order and patriarchy
Women (1855). (b) Displacement, slippage and condensation
UGC NTA NET/JRF English Paper II, 2022 (Sh.-I) 363 YCT
(c) Repression, control and normalcy "Old Goriot" or "Father Goriot" in French (Le Pere
(d) Logic, reason and power Goriot) is 1835 novel by French novelist and playwright
Ans. (b) : The cluster of "Displacement, Slippage and Honore de Balzac.
Condensation" is associated with what Julia Kristeva "Therese Requine" is an 1868 novel by French writer
terms the 'Semiotic'. Emile Zola.
Semiotic, also called Semiology is the study of signs 32. Which two of the following conform to liberal
and sign-using behavior. It was defined by one its humanist thought?
founders, the Swiss linguist Ferdinand de Saussure, as A. Literature transcends the limits of the age of
the study of "the life of signs within society". its origin and so is timeless.
In Kristeva's terms, the semiotic is defined as the B. Literature is untouched by the essential
matriarchal aspect of language that shows the speaker's human nature which is unchanging.
inner drives and impulses. Julia Kristeva (1941), French C. Literature is devoid of any purpose to
psychoanalyst, critic, novelist and educator, best known enhance life or promote human values.
for writing is structuralist linguistics, psychoanalysis, D. Identity is a unique essence unaffected by
semiotics and philosophical feminism. environment and society.
30. Which two works in the following list are Choose the correct answer from the options
written by Aphra Behn? given below:
A. Rover (a) A and C only (b) B and C only
B. Oroonoko (c) A and D only (d) B and D only
C. Soldier's Fortune Ans. (c) : From the following options only (a) and (d)
D. The Princess of Cleve conform to liberal humanist thought.
Choose the correct answer from the options Literature transcends the limits of the age of its
given below: origin and so is timeless.
(a) A and B only (b) B and C only Identity is a unique essence unaffected by
(c) B and D only (d) A and C only environment and society.
Ans. (a) : The Rover : or, The Banish't Cavaliers and Liberal humanism is a philosophical and literary
Oroonoko are written by English playwright Aphra Behn. movement in which man and his capabilities are the
The Rover, Comedy by Apra Behn, produced and control concern:
published in two parts in 1677 and 1681. The play Some tenets of liberal humanism are:
depicts the adventures of a small group of English Good literature is timeless, transcendent and speaks
Cavaliers during the exile of England's king Charles-II to what is constant in human nature.
in Madrid and Neples. Literary text contains its own meaning.
Oroonoko; or, The Royal Slave, novel by Apra Behn, Human nature unchanging-continuity valued over
published in 1688. The author's experiences in the innovation.
Dutch colony of Surinam in South America provided Purpose of literature to enhance life in a non-
the plot and the locale for this story of a proud, Virtuous programmatic (non propagandistic) way.
African prince who is enslaved and cruelly treated by Form and content fused organically in literature.
"Civilized" white Christians; Individuality as essence securely possessed by each
Oroonoko was adapted for the theatre by Thomas 'transcendent subject' distinct from force of society,
Southerner and performed in 1695. experience and language etc.
31. Which two of the following are works by 33. Which two of the following conform to
Gustave Flaubert? Northrop Frye's typology of literature?
A. The Temptation of Saint Anthony A. Mythos of spring: Comedy
B. Old Goriot B. Mythos of summer: Satire
C. Therese Raquin C. Mythos of autumn: Tragedy
D. Sentimental Education D. Mythos of winter: Romance
Choose the correct answer from the options Choose the correct answer from the options
given below: given below:
(a) A and C only (b) B and C only (a) A and B only (b) B and D only
(c) A and D only (d) B and D only (c) A and C only (d) B and D only
Ans. (c) : The Temptation of Saint Anthony and Ans. (c) : From the given options, (a) Mythos of spring
Sentimental Education are novels written by French : Comedy (c) Mythos of autumn : Tragedy, conform to
novelist Gustave Flaubert. The novel 'The Temptation Northrop Frye's typology of literature.
of Saint Anthony' published in 1874 as 'La Tentation de Northrop Frye's essay Archetypes Criticism : A
Saint Anthony'.
Flaubert called the subject of the narrative his "old Theory of Myths describes four radical mythoi (i.e.
infatuation." The work is notable for its imagery and its plot forms, or organizing structural principles),
depiction of spiritual torment. The novel reflects on the correspondent to the four seasons in the cycle of the
life of 4th century Christian anchorite Saint Anthony, natural world, are incorporated in the four major genres:
his decision became a hermit and temptation of Mythos of spring : Comedy
sexuality and sensuality he undergoes. Mythos of summer : Romance
Sentimental Education, novel by Flaubert published in French
in 1869 as "L' education Sentimental: histoire d' Un jeune Mythos of autumn : Tragedy
homme". The story of the protoganist, frederic Moreall, and Mythos of winger : Satire
his beloved, Madame Arnoux, is based on Flaubert's youthful
infatuation with an order married women.
UGC NTA NET/JRF English Paper II, 2022 (Sh.-I) 364 YCT
34. Which two of the following writers does A. D. Ans. (b) : Circulation of Meaning and Production of
Hope address through his poetic responses in A Meaning are highlighted in relation to specific historical
Book of Answers? moments by Stephen Greenblatt.
A. Tolstoy B. Dostoevsky Stephen Greenblatt (1943), American scholar who was
C. Mallarme D. Goethe credited with establishing New Historicism, an
Choose the correct answer from the options approach to literacy criticism.
given below: New Historicism, a critical approach developed in the
(a) A and D only (b) B and C only 1950s through the works of Michel Foucault and
(c) B and D only (d) A and C only Stephan Greenblatt and discourse analysis are cross-
disciplinary practice of critical inquiry that study
Ans. (d) : From the given option (a) & (c) is correct; literary texts and their socio-cultural functions.
Alec Derwent Hope (A.D. Hope) addresses to Tolstoy Both explain the circulation and production of meaning
and Mallarme through his poetic responses in A Book in specific historical moments and share a micro-
of Answers (1978). analytic mode of interpretation. They distrust holistic
A. D. Hope, Australian poet, is best known for his and monological explanations applied by historicism
elegies and satires. He was made a member of the Order and intellectual history.
of the British Empire in 1972 and a Companion of the 37. Who among the following belong to the
Order of Australia in 1981. Chicago School of critics?
35. Which two of the following are true according A. R. S. Crane B. E. M. W. Tillyard
to the documentation style prescribed by the C. Elder Olson D. Allen Tate
eighth edition of the MLA Handbook? Choose the correct answer from the options
A. If the title page of a book contains an imprint given below:
(a) A and C only (b) A and D only
as well as the publisher's name, omit the (c) B and C only (d) B and D only
imprint and use the publisher's name.
Ans. (a) : 'R.S. Crane' and 'Elder Olson' are belong to
B. While giving a URL copy it from the Web the Chicago School of Critics. Chicago School of
browser but omit http:// or https://. Critics, group of pluralist, essentially formalist
C. If a quotation extends to more than five lines American literary critics- including Richard McKeon,
set it off from the text as block indented an Elder Olson, Ronald Salmon Crane (R.S. Crane),
inch from the left margin. Bernard Weinberg and Norman Maclean- who exerted
D. Long titles should be abbreviated using the a significant influence on the development of American
first letter of key words typed in upper case Criticism during the second half of the 20th century.
without intervening space. It was a form of criticism of English literature begun at
Choose the correct answer from the options the University of Chicago in the 1930, which lasted
given below: until the 1950s. It was also called Neo-Aristotalianism,
(a) A and B only (b) A and C only due to its strong emphasis on Aristotle's concepts of
(c) B and C only (d) B and D only plot, character and genre.
Ans. (a) : The true statements according to the 38. Which of the following poems contains John
documentation style prescribed by the eighth edition of Donne's famous conceit bringing a parallel
between lovers and the hands of a compass?
the MLA Hand book are: (a) "Negative Love"
If the title page of a book contains an imprint as well (b) "Lovers Infinitenesse"
as the publisher's name, omit the imprint and use the (c) "A Valediction: Forbidding Mourning"
publisher's name. (d) "A Valediction: Of Weeping"
While giving a URL copy it from the Web browser Ans. (c) : "A Valediction: Forbidding Mourning"
but omit http:// or https://. contains John Donne's famous conceit bringing a
Other statements are not correct regarding eighth parallel between lovers and the hands of a compass.
edition of MLA Hand book. This poem was written for Donne's wife Anne in 1611
MLA Handbook, formerly MLA Handbook for writers or 1612 and is divided into sets of four lines, or
of Research papers, published by Modern Language quatrains. The poem concludes with the well-known
Association, which is based in USA (America). It is conceit comparing lovers to a drafting compass. Donne
widely used by scholars, journal publishers, and states that his wife is the leg that holds them steady,
academic and commercial presses. fixed at a point while he "roams". It is due to her stead
fastness that he always finds his way back.
36. Which two of the following are highlighted in
39. Arrange the following essays in the
relation to specific historical moments by
chronological order of publication.
Stephen Greenblatt? A. T. S. Eliot, "The Function of Criticism"
A. crisis of meaning B. Edgar Allan Poe, "The Philosophy of
B. circulation of meaning Composition"
C. production of meaning C. Henry James, "The Art of Fiction"
D. deferral of meaning D. Virginia Woolf, "Modern Fiction"
Choose the correct answer from the options Choose the correct answer from the options
given below: given below
(a) A and B only (b) B and C only (a) C, B, A, D (b) C, B, D, A
(c) C and D only (d) B and D only (c) B, C, D, A (d) B, C, A, D
UGC NTA NET/JRF English Paper II, 2022 (Sh.-I) 365 YCT
Ans. (c) : Chronological order of publication of given (a) Both Statement I and Statement II are true
essays with their authors– (b) Both Statement I and Statement II are false
ch. Essay/work Author Year of (c) Statement I is true but Statement II is false
order publication (d) Statement I is false but Statement II is true
B The Philosophy Edgar Allan 1846 Ans. (a) : In the question, both the given statements are
of Composition Poe
C The Art of Henry James 1884 absolutely correct.
Fiction In Waiting for Godot, it is Vladimir who questions
D Modern Fiction Virginia 1921 the Boy who comes from Godot towards the end of
Woolf the two Acts.
A The Function of T. S. Eliot 1923 In Waiting for Godot, it is Estragon who says,
Criticism "Nothing happens, no body comes, nobody goes, it's
Thus, the correct option is (c). awful".
40. Arrange the following journals in the Waiting for Godot, tragicomedy in two acts by Irish
chronological order in which they started writer Samuel Beckett, published in 1952 in French as
publication. "En attendant Godot" and first produced in 1953.
A. The Tatler B. The Examiner The play consists of conversations between Vladimir
C. The Review D. The Spectator
Choose the correct answer from the options and Estragon, who are waiting for the arrival of the
given below mysterious Godot, who continually sends word that he
(a) A, D, C, B (b) B, A, D, C will appear but who never does.
(c) C, A, B, D (d) C, A, D, B 43. Given below are two statements
Ans. (c & d) : The correct chronological order of the Statement I: New Historicism stipulates that
journals are– teleological connotations of history have to be
CABD eschewed.
(C)- The Review (1704-1713) Statement II: New Historicism neither denies
(A)- The Tatler (1709-1711) nor accepts totalizing explanations of historical
(B)- The Examiner (1710-1711) (Jonathan Swift)
(D)- The Spectator (1711-1712) events.
Note: Another journal with the same title "The In light of the above statements, choose the
Examiner" (1808-86) had been published by John Hunt correct answer from the options given below
so, with this fact option (d) - CADB order is also correct. (a) Both Statement I and Statement II are true
41. Given below are two statements, one is labelled (b) Both Statement I and Statement II are false
as Assertion A and the other is labelled as (c) Statement I is true but Statement II is false
Reason R (d) Statement I is false but Statement II is true
Assertion A : Postmodern narratives focus on Ans. (c) : From the given statements in the question,
the indeterminate and unstable nature of first statement (I) is true but second statement (II) is not
textuality and subjectivity.
Reason R : Postmodern narrative acts regard true.
narratives and characters as tentative "New Historicism stipulates that teleological
representations of writing and identity. connotations of history have to be eschewed" is
In light of the above statements, choose the absolutely correct statement regarding New historicism
correct answer from the options given below while "New Historicism neither denies nor accepts
(a) Both A and R are true and R is the correct totalizing explanations of historical events". Is not
explanation of A correct statement regards of New Historicism.
(b) Both A and R are true but R is NOT the Stephen Greenblatt, who was credited with establishing
correct explanation of A New Historicism, an approach to literacy criticism
(c) A is true but R is false
(d) A is false but R is true which treats texts as historical artifacts that emerges
Ans. (a) : In the given question both the given among particular social, intellectual and economic
statements are absolutely correct and second statement circumstances. New historicist looks at literature in a
is the correct explanation of first statement. wider historical context, examining both how the
Postmodern narratives focus on the indeterminate and writer's times affected the work and how the work
unstable nature of textuality and subjectivity because reflects the writer's time, in turn recognizing that current
postmodern narratives acts regard narrative and characters cultural contexts color that critic's conclusions.
as tentative representations of writing and identity. 44. What was poor Yorick in Hamlet?
Thus, the appropriate answer is option (a). (a) Jester (b) Actor
42. Given below are two statements (c) Soldier (d) Gravedigger
Statement I: In Waiting for Godot, it is Ans. (a) : Poor Yorick in the play Hamlet was a Jester.
Vladimir who questions the Boy who comes In Act 5, Scene I of Hamlet, Hamlet is talking to a grave
from Godot towards the end of the two Acts.
digger. The Grave Digger picks up a skull and says it
Statement II: In Waiting for Godot, it is
Estragon who says, "Nothing happens, nobody has been there for 23 years and it belonged to Yorick,
comes, nobody goes, it's awful." the court jester and childhood friend of Hamlet's.
In light of the above statements, choose the The phrase alas poor Yorick refers to the brevity of
correct answer from the options given below human life.
UGC NTA NET/JRF English Paper II, 2022 (Sh.-I) 366 YCT
45. Which of the following words refers to a sound Ans. (b) : Jean Baudrillard argued the Gulf War was
that is associated with a particular meaning? not really a war, but rather an atrocity which
(a) phoneme (b) phonic substance masqueraded as a war. Jean Baudrillard published three
(c) phonestheme (d) phonemoid essays in the French daily liberation before, during and
Ans. (c) : Phonestheme refers to a sound that is after the Gulf War between January and March in 1991
associated with a particular meaning. A phonestheme is which he sequentially titled.
a particular sound or sound sequence that (at least in a 1. 'The Gulf War will not take place'
general way) suggest a certain meaning. The Adjective 2. 'The Gulf War : Is it really taking place?'
form is phonesthemic. 3. 'The Gulf War did not take place.'
For example: Thus, the appropriate answer is option (b).
In words like glimmer, glitter and glisten, the initial gl 50. What 19th-century philosophical term of
phonestheme is associated with vision or light. Russian origin did Friedrich Nietzsche use to
Words related in this fashion are called phonestheme describe the disintegration of traditional
groups or phonestheme clusters. morality in western society?
Note–The concept (Phonestheme) was proposed in (a) absolutism (b) cynicism
1930 by British Linguist J. R. Firth. (c) nihilism (d) anarchism
46. Who among the following Dickens characters Ans. (c) : Nihilism was the philosophical term of
appears as a ghost? Russian origin which Friedrich Nietzsche use to
(a) Daniel Quilp (b) Dora Spenlow describe the disintegration of traditional morality in
western society. Nihilism mean the rejection of all
(c) Esther Summerson (d) Jacob Marley religious and moral principles, in the belief that life is
Ans. (d) : Jacob Marley, fictional character, the meaningless. According to Nietzsche, this state of
deceased business partner of Ebenezer Scrooge in 'A nihilism-the idea that life has no meaning or value-
Christmas Carol' (1843) by Charles Dickens, appears as cannot be avoided, we must go through it, frightening
a ghost. and lonely as that will be. Nihilism most often
Thus, the appropriate answer is option (d). associated with friedrich who argued that its corrosive
47. Virginia Woolf's Orlando opens in 1588 and effects would eventually destroy all moral, religious and
Orlando, a sixteen-year-old boy, writes a poem metaphysical convictions and precipitate the greatest
called: crisis in human history.
(a) "The Evergreen Tree"(b) "The Poison Tree" Hence, the correct answer is option (c).
(c) "The Oak Tree" (d) "The Magic Tree" 51. Which of the following statements is true of the
Ans. (c) : Virginia Woolf's Orlando opens in 1588 and working group set up by the University Grants
Orlando, a sixteen-years-old boy, writes a poem called Commission in 1978 to study the medium of
"The Oak Tree". Orlando, novel by Virgina Woolf, instruction in higher education?
published in 1928. The fanciful biographical novel pays (a) It recommended a quicker switchover of the
homage to the family of Woolf's friend Vita Sackville- medium of instruction from English to Indian
West from the time of her ancestor Thomas Sackville languages.
(1536-1608) to the family's country estate at Knole. (b) It observed that English need not be displaced
as the medium of instruction although
48. What function of English is exemplified by the regional languages were quite ready to take
use of English in India as a "link language" over its functions.
along with a first language by stable bilinguals (c) It argued that English as a highly developed
in well-defined social contexts? language was best suited for India’s
(a) complementary (b) supplementary industrial and scientific progress.
(c) auxiliary (d) equative (d) It asserted that English had the potential to
Ans. (a) : 'Complementary' use of English in India as a further polarize an already‐divided nation
"Link language" along with a first language by stable along socio‐economic and intellectual fault
bilinguals in well-defined social contexts. lines.
English has become a link language in the real sense of Ans. (c) : The statement which is true regarding the
the term. In India it's slowly becoming the language of working group set up by the University Grants
communication for the classes and the masses in various Commission in 1978 is 'it argued that English as a
corporate offices, MNCS, colleges, schools etc. slowly highly developed language was best suited for India
but surely people have started to get the hang of it. industrial and scientific progress'.
There are four distinct function of English in India on The University Grant Commission of India is a statutory
the basis of the social needs of people. body set up by the department of Higher Education,
1. Complementary ministry of education, Government of India in
2. Supplementary accordance to the UGC Act 1956. The objectives of
3. Auxiliary UGC is coordination determination and maintenance of
4. Equative standards of university education.
Hence, option (a) is appropriate answer. Hence, option (c) is correct answer.
49. Who among the following is said to have 52. In comparative philology and sometimes in
believed that the Persian Gulf War (1990 - 91) modern phonology, what is the term used to
never happened? refer to the deletion of a vowel within a word?
(a) Zygmunt Bauman (b) Jean Baudrillard (a) Aphaeresis (b) Equi-deletion
(c) Jacques Derrida (d) Jurgen Habermas (c) Paradigm (d) Syncope
UGC NTA NET/JRF English Paper II, 2022 (Sh.-I) 367 YCT
Ans. (d) : In Comparative Philology and sometimes in (a) A and C only (b) B and D only
modern phonology, 'Syncope' is the term used to refer (c) B and D only (d) C and D only
to the deletion of a vowel within a word. In phonology, Ans. (a) : 'New Grub Street' and 'The Odd Women' are
syncope is the less of one or more sounds from the the works of George Gissing. He was an English novelist,
interior of a word, especially the loss of an unstressed who published 23 novels between 1880 and 1903.
vowel. It is found in both Synchronic and Diachronic In novels 'New Grub Street' considered by some critics
analyses of languages. Its opposite, whereby sounds are to be his only great book. Though he rejected Zola's
added, is epenthesis. Syncope is mainly used in poetry, Theory of Naturalism, his ironic, agnostic and
when poets desperately wants to avoid as single syllable pessimistic fictions came to be respected for their
from a word to harmonize the meter in each line. similarity to contemporary developments in French
However, syncope can be found in drama and in prose realist fiction.
as well. Syncope is also used in informal speech. 'The Odd Women' (1893) is a powerful study of female
Hence, the correct answer is (d). frustration. He did not lack human sympathies, but his
53. Which of the following statements best obvious contempt for so many of his character reflects
describes Terry Eagleton's views on literature? and artistic limitation.
(a) It is involved in the reproduction of the Thus, correct answer is option (a).
dominant social order. 56. Which of the following are novels by Ian
(b) Its raison d'être is to reflect social reality McEwan?
directly. A. Atonement
(c) Its primary purpose is to produce beauty and B. The Man with Two Left Feet
pleasure. C. The Child in Time
(d) It is closely allied to religion in its D. The Rachel Papers
significance and seriousness. Choose the correct answer from the options
Ans. (a) : Terry Eagleton's 'Views on Literature' that given below:
describes it is involved in the reproduction of the (a) A and C only (b) B and D only
dominant social order. He argues that to claim that (c) C and D only (d) C and D only
literature is a special kind of language presupposes the Ans. (a) : 'Atonement' and 'The Child in Time' are
existence of a normal or ordinary language. novels by Ian McEwan.
Terry Eagleton (1943) is an English literary theorist, 'Atonement' is a 2001 British Metafiction novel by Ian
critic and public intellectual. Eagleton has published McEwan, set in three time periods 1935 England,
over forty books, but remains best known for 'Literary second World War England and France, and present day
Theory : An Introduction (1983). which has sold over England. It covers an upper-class girl's half innocent
750,000 copies. Thus correct option is (a). mistake that ruins lives, her adulthood in the shadow of
54. Which of the following are true of that mistake and a reflection on the nature of writing.
'Performance', as used in linguistic theory? 'The Child in Time' is a British television film directed
A. It is analogous to the Saussurean concept of by Julian Farino, adaptation of the 1987 novel of the
langue. same name by Ian McEwan. The film premiered on
B. It refers to the specific utterances of
individual native speakers in actual BBC one on Sunday 24 September 2017 and stars
situations. Benedict Cumberbatch. Hence, option (a) is correct.
C. It is an innate grammar that suggests 57. Which of the following are poems by Nissim
humans' universal ability to use language. Ezekiel that make fun of Indians' use of
D. It includes hesitations and unfinished English?
structures arising out of psychological A. "Goodbye Party for Miss Pushpa T.S."
difficulties acting upon the speaker. B. "Philosophy"
Choose the correct answer from the options C. "Very Indian Poem in Indian English"
given below: D. "Jewish Wedding in Bombay"
(a) A and B only (b) B and C only E. "Poet, Lover, Birdwatcher"
(c) A and C only (d) B and D only Choose the correct answer from the options
Ans. (d) : Term 'Competence and Performance' were given below:
proposed by Noam Chomsky in 'Aspects of the Theory (a) A and C only (b) B and D only
of Syntax', when he stressed the need for a Generative (c) A, B and D only (d) A, C and E only
Grammars that mirrors a speaker's competence and the Ans. (a) : 'Goodbye Party for Miss Pushpa T.S.' and
creative aspect of linguistic ability. Performance refers 'Very Indian Poem in Indian English' are the poems by
to the specific utterances of individual native speakers Nissim Ezekiel that make fun of Indians' use of English.
in actual situations. It includes hesitations and Nissin Ezekiel was an Indian Jewish poet, actor,
unfinished structures arising out of psychological playwright, editor and art critic, He was foundational
difficulties acting upon the speaker. Thus, the correct figure in postcolonial India's literary history,
option is (d). specifically for Indian poetry in English, 'Goodbye
55. Which among the following are the works of Party for Miss Pushpa T.S.' is a satirical poem, in which he
George Gissing? satirizes the way in which Indians speak in English
A. New Grub Street B. Agnes Grey through this poem. 'Very Indian Poem in Indian English' is
C. The Odd Women D. Mary Barton also written in the form of a satire, based on the reality of
Choose the correct answer from the options the Indian English language that the Indians use.
given below: Hence, the correct answer is (a)
UGC NTA NET/JRF English Paper II, 2022 (Sh.-I) 368 YCT
58. Match List I with List II Ans. (d) : The appropriate match of lines with its poet
List I List II is as followed–
(Author) (Text) List I (First line) List II (Poet)
A. Sean O' Casey I. I'm Talking About A. "Courage!" he said, and Alfred Tennyson
pointed toward the land...
Jerusalem B. I am poor brother Lippo, by Robert Browning
B. Dylan Thomas II. The Winslow Boy your leave!
C. Terence Rattigan III. Juno and the Paycock C. I caught this morning G.M.Hopkins
D. Arnold Wesker IV. In the Shadow of the morning's minion...
Glen D. Look in my face; my name is D.G. Rossetti
E. J.M. Synge V. Under Milk Wood Might‐have‐been...
Choose the correct answer from the options E. The sea is calm tonight... Matthew Arnold
given below: Thus, option (d) is correct answer.
(a) A-II, B-I, C-III, D-V, E-IV 60. Match List I with List II
(b) A-V, B-IV, C-II, D-I, E-III List I (Character) List II (Novel)
A. Winston Smith I. Sons and Lovers
(c) A-III, B-V, C-II, D-I, E-IV B. Paul Morel II. Ulysses
(d) A-IV, B-II, C-III, D-V, E-I C. 'Whiskey priest' III. Nineteen Eighty-four
Ans. (c) : The appropriate match of authors and their D. Leopold Bloom IV. Decline and Fall
works are- E. Paul Pennyfeather V. The Power and the
A. Sean O’Casey III. Juno and the Paycock Glory
B. Dylan Thomas V. Under Milk Wood Choose the correct answer from the options
C. Terence Rattigan II. The Winslow Boy given below:
(a) A-III, B-I, C-V, D-II, E-IV
D. Arnold Wesker I. I'm Talking About Jerusalem (b) A-I, B-V, C-II, D-III, E-IV
E. J.M. Synge IV. In the Shadow of the Glen (c) A-IV, B-III, C-V, D-II, E-I
Juno and the Paycock is a play of Sean O' Casey, (d) A-V, B-I, C-IV, D-II, E-III
highly regarded and often performed in Ireland. It was Ans. (a) : The appropriate match of Characters with
first staged at the abbey theatre in Dublin in 1924. their Novels
'Under Milk Wood' is a 1954 radio drama a by List I (Character) List II (Novel)
welsh poet Dylan Thomas, commissioned by the A. Winston Smith III. Nineteen Eighty-four
BBC and later adapted for the stage. B. Paul Morel I. Sons and Lovers
'The Winslow Boy' is an English play from 1946 C. 'Whiskey priest' V. The Power and the Glory
D. Leopold Bloom II. Ulysses
by Terence Rattiagan based on an incident involving E. Paul Pennyfeather IV. Decline and Fall
George Archer shee in the Edwardian era. Thus, option (a) is correct answer.
I'm talking about Jerusalem (1960) is the 61. Arrange the following language areas of the
concluding, play in the trilogy which comprises human brain in the order in which they involve
'Chicken Soup with Barley' and 'Roots' by Arnold in hearing, understanding and saying a word:
Wesker. Thus, the correct option is (c). A. arcuate fasciculus
59. Match List I with List II B. anterior speech cortex
List I List II C. motor cortex
D. posterior speech cortex
(First line) (Poet) Choose the correct answer from the options
A. "Courage!" he I. G.M.Hopkins given below:
said, and pointed (a) A, C, D, B (b) B, A, D, C
toward the land... (c) C, B, A, D (d) D, A, B, C
B. I am poor brother II. Alfred Tennyson Ans. (d) : The correct sequence of the language areas of
Lippo, by your the human brain in the order in which they involve in
leave! hearing, understanding and saying a word is–
C. I caught this III. D.G. Rossetti Posterior speech cortex → Arcuate Fasciculus →
morning morning's Anterior speech cortex → Motor cortex.
Posterior speech cortex : Wernicke area, region of the
minion... brain that contains motor neurons involved in the
D. Look in my face; IV. Matthew Arnold comprehension of speech.
my name is Might- Arcuate fasciculus : Its key function is to connect
have-been... Broca's and Wernick's areas, which are involved in
E. The sea is calm V. Robert Browning producing and understanding language.
tonight... Anterior speech cortex: Broca's area, or the Broca area
Choose the correct answer from the options is a region in the frontal lobe of the dominant
hemisphere, usually the left, of the brain with function
given below: linked to speech production.
(a) A-II, B-III, C-I, D-V, E-IV Motor cortex: The motor cortex is the region of the
(b) A-IV, B-V, C-I, D-III, E-II cerebral cortex involved in the planning, controling and
(c) A-III, B-IV, C-V, D-I, E-II execution of voluntary movements.
(d) A-II, B-V, C-I, D-III, E-IV Thus, option (d) is appropriate answer.

UGC NTA NET/JRF English Paper II, 2022 (Sh.-I) 369 YCT
62. Choose the right chronological sequence of the Ans. (b) : In the light of above statement the most
publication of the following books: appropriate answer is option (b). Both statement I and
A. Margaret Atwood, The Handmaid's Tale statement II are incorrect because Literary Researches
B. Alice Walker, The Color Purple also have specific materials for investigation and
C. Doris Lessing, The Golden Notebook scientific tools for researching a conclusion and
D. Toni Morrison, The Bluest Eye formulating a theory like scientific researches.
Choose the correct answer from the options Scientific researches are studies that should be
given below: systematically planned before performing them.
(a) A, B, D, C (b) B, A, D, C Research is the systematic investigation into study of
(c) C, D, B, A (d) D, C, B, A materials and sources in order to establish facts and
Ans. (c) : The right chronological sequence of the sources.
publication of the following books is– Literary research consists of critical interpretations of
Writer Book Publishing an author's work to not to the exclusion of biography
Date and editing of texts. "Literary Research" means finding
Doris Lessing The Golden Notebook 1962 something new within a literary work (theory, term etc).
Hence the correct answer in option b.
Toni Morrison The Bluest Eye 1970
Alice Walker The Colour Purple 1982 65. Given below are two statements:
MargaretAtwood The Hand Maid's Tale 1985 Statement I: Linguists, being primarily
interested in the scientific study of language,
Hence option (c) is correct answer. approach language dispassionately.
63. Arrange the following terms in the Statement II: Linguists are necessarily
chronological order of their use in literary polyglots who bring their own biases to
theory: language study.
1. gynesis 2. scriptible In the light of the above statements, choose the
3. negritude 4. paratext most appropriate answer from the options
Choose the correct answer from the options given below:
given below: (a) Both Statement I and Statement II are correct.
(a) D, A, C, B (b) A, B, C, D (b) Both Statement I and Statement II are incorrect.
(c) C, A, D, B (d) B, C, D, A (c) Statement I is correct but Statement II is incorrect.
Ans. (*F) : The correct chronological order of literary (d) Statement I is incorrect but Statement II is correct.
theory is (3→2→4→1). Ans. (c) : From the given two statements, option (c)
Negritude- Term coined in the 1930s by Afro- statement (I) is correct but statement (II) is incorrect.
Martiniquan French poet and politician Aime Fernand 'Linguists' is one who studies language, linguists study
Cesaire, Senegalese poet and politician Leopold every aspects of language, including vocabulary,
Senghor and Leon Damas of French Guiana. grammar, the sound of language and how words evolve
R. Barthes used the terms Lisible (readerly) and over times. Linguist being primarily interested in the
Scriptible (Writerly) to distinguish, respectively, scientific study of language, approach language
between texts that are straight forward and demand no dispassionately. Statement (II) is incorrect because all
special effort to understand and those whose meaning is linguists are not necessarily polyglots who bring their
not immediately evident and demand some effort on the own biases to language study. Hence the correct answer
part of the reader. He used the term Scriptible in his is option (c).
book S/Z (1970). 66. Which of the following are features of the
Paratext is a term coined by the French literary critic 'Theatre of the Absurd' ?
Gerard Genette. The term appears for the first time in A. emphasis on the central role of God in the
Genette's Introduction a 'l', architexte' (1979). universe
Gynesis- Gynesis a term coined by the American B. presentation of futile actions devoid of any
feminist theorist Alice Jardine in her book 'Gynesis : goal
Configurations of Woman and Modernity' (1985). C. portrayal of situations that point to the
64. Given below are two statements: meaningfulness of life
Statement I: Unlike scientific research, literary D. lacking in conflicts and dramatic tensions
research does not have specific materials for E. presenting players in a stasis or drift without
investigation and scientific tools for reaching a definite roles
conclusion and formulating a theory. Choose the correct answer from the options
Statement II: Literary research consists of given below:
critical interpretations of an author’s work to (a) A, C and E only (b) B, D and E only
the exclusion of biography and editing of texts. (c) A, B and D only (d) B, C and D only
In the light of the above statements, choose the Ans. (b) : The term 'Theatre of Absurd' is coined by
most appropriate answer from the options 'Martin Esslin' in his book of the same name 'Theatre of
given below: Absurd'. Such theatre seeks to represent the absurdity of
(a) Both Statement I and Statement II are correct. human existence in a meaningless universe by bizarre or
(b) Both Statement I and Statement II are fantastic means. Important features of the 'Theatre of
incorrect. the Absurd' are
(c) Statement I is correct but Statement II is (1) Presentation of futile actions devoid of any goal.
incorrect. (2) Lacking in conflicts and dramatic tensions.
(d) Statement I is incorrect but Statement II is (3) Presenting players in a stasis or drift without definite roles.
correct. Hence correct answer is option (b) include (B, D, E only).
UGC NTA NET/JRF English Paper II, 2022 (Sh.-I) 370 YCT
67. Who, among the following, is known to have Ans. (b) : Pierre de Ronsard (poet) wrote a poem that
used elements from the Yakshagana tradition served to inspire W.B. Yeats to write his own poem
in his theatre? "When You Are Old". Pierre de Ronsard was a French
(a) Badal Sircar (b) Girish Karnad poet or as his own generation in France called him "a
(c) Mohan Rakesh (d) Mahesh Dattani prince of poets". Major theme in, "When You Are Old"
Ans. (b) : Girish Karnad was an Indian actor, film are love, rejection and time. It is bittersweet poem that
director Kannada writer, worked is south Indian cinema reveals the complexities of love.
and Bollywood. He is known to have used elements Thus, the correct option is (b).
form the ' Yakshagana Tradition' in his theatre.
Yakshagana, dance-drama of South India, associated 71. Which of these does Meenakshi Mukherjee
most strongly with the state of Karnataka. It is propose as the possible target readership of
performed in open air by all male groups. It emerged in early Indian English novel?
the 16th century with its roots in Sanskrit Literature. He A. A pan-Indian readership
was a recipient of the 1998 'Jnanpith Award. He was B. A localized Indian readership
awarded 'Padma Shri' and 'Padma Bhushan' by the C. A British readership
Government of India and own four film fare awards. D. The colonial administrator in India
Hence the correct answer is option (b). Choose the correct answer from the options
68. Match List I with List II given below:
List I (Writer) List II (Book) (a) A and B only (b) B and C only
A.Bankimchandra I. Untouchable (c) C and D only (d) A and D only
Chatterjee Ans. (c) : Meenakshi Mukharjee proposes 'A British
B. Mulk Raj Anand II. Rajmohan's Wife readership' and 'The Colonial administrator in
C. Panchkouree Khan III. Stories from India' as the possible target readership of early Indian
Indian Christian Life English novel.
D.Kamala IV. The Revelations of Meenakshi Mukharjee's, 'The Anxiety of Indianness:
Sathianadhan an Orderly Our Novels in English' first published in the Economic
Choose the correct answer from the options and Political Weekly (1993) and then included in the
given below: book, The Perishable Empire : Essay on Indian
(a) A ‐ II, B ‐ I, C ‐ IV, D ‐ III Writing in English (2000), the essay aptly sums up an
(b) A ‐ I, B ‐ III, C ‐ II, D ‐ IV important and intriguing question related to cultural
(c) A ‐ III, B ‐ IV, C ‐ II, D ‐ I identity and post-colonial make-over.
(d) A ‐ IV, B ‐ I, C ‐ II, D ‐ III In this book, Meenakshi asks and describes about Post
Ans. (a) : The appropriate match of writer with their Colonial English novels written by Indian authors.
Book Thus, option (c) is appropriate answer.
List I (Writer) List II (Book) 72. Match List I with List II
A. Bankimchandra II. Rajmohan's Wife (1864) List I (Poet) List II (Language)
Chatterjee A. Charles Baudelaire I. French
B. Mulk Raj Anand I. Untouchable (1935) B. Heinrich Heine II. German
C. Panchkouree Khan IV. The Revelations of an C. Sylvia Path III. English
Orderly (1849) D. Jose Marti IV. Spanish
D. Kamala Sathianadhan III. Stories from Indian Choose the correct answer from the options
Christian Life given below:
Thus, option (a) is appropriate answer. (a) A ‐ I, B ‐ II, C ‐ III, D ‐ IV
69. Which of these UK universities saw the first (b) A ‐ II, B ‐ III, C ‐ IV, D ‐ I
institutional incorporation of Cultural Studies? (c) A ‐ III, B ‐ IV, C ‐ I, D ‐ II
(a) Bath Spa University
(b) Oxford University (d) A ‐ IV, B ‐ I, C ‐ II, D ‐ III
(c) University of Birmingham Ans. (a) : The appropriate match of poets with their
(d) Cambridge University language- A‐I, B‐II, C‐III, D‐IV
Ans. (c) : University of Birmingham in UK university List I List II
saw the first institutional incorporation of cultural (Poet) (Language)
studies. Dennis Dworkin writes that "a critical moment" A. Charles Baudelaire I. French
in the beginning of cultural studies as a field was when B. Heinrich Heine II. German
Richard Hoggart used the term in 1964 in founding the C. Sylvia Path III. English
Centre for Contemporary Cultural Studies (CCCS) D. Jose Marti IV. Spanish
at the University of Birmingham.
Incorporation refers to the free borrowing and Thus, appropriate answer is option (a)
modification of cultural elements and occurs when 73. Which of these constitute the preoccupations of
people of different cultures maintain contact as well as the protagonist of Upamanyu Chatterjee's
political and social self-determination. Thus, option (c) English, August?
is appropriate answer. A. Marijuana B. Magic
70. Which of these poets wrote a poem that served C. Monotheism D. Marcus Aurelius
to inspire W B Yeats to write his own poem, Choose the correct answer from the options
"When You Are Old"? given below:
(a) Francois Villon (b) Pierre de Ronsard (a) A and B only (b) A and D only
(c) Edmund Spencer (d) Heinrich Heine (c) B and C only (d) B and D only
UGC NTA NET/JRF English Paper II, 2022 (Sh.-I) 371 YCT
Ans. (b) : 'Marijuana' and 'Marcus Aurelius' Ans. (c) : Pierre Bourdieu played the most significant
constitute the preoccupations of the protagonist of role in mobilizing concept of cultural Intermediaries...
Upamanyu Chatterjee's English, August. cultural intermediaries may be defined, descriptively, by
Upamanyu Chatterjee's work 'English, August' is a their occupations, which are largely located in the
novel presents the people of rural India and struggle of a cultural industries including the media. Bourdieu, a
civil servant who is posted in a rural area. The social critique of the judgment of Taste (1984), drawing
protagonist of the novel, Agartya, is a civil servant and on social survey data and qualitative vignettes from
he encounters the people of Madna on his first posting.
Although he joins India Civil Service to satisfy his France in the 1960 and early 1970s Bourdieu analyses
father's wish yet he finds himself disinterested in bizarre patterns in the Consumption Practices and Preferences
ways of government and administration. He is of socio-occupational group. In doing so, he suggests
directionless and finds solace in marijuana, how "taste" is a contested terrain on which battles for
masturbation and the Meditations of Marcus legitimacy and distinction are fought.
Aurelius. Thus, the correct option is (b). 78. From whom does Gayatri Chakravorty Spivak
74. What was the name of the journal published borrow the term 'subaltern'?
from Bowling Green University beginning (a) Karl Marx (b) Friedrich Engels
1969, which carried essays on amusement (c) Louis Althusser (d) Antonio Gramsci
parks, comics and detective films? Ans. (d) : Gayatri Chakravorty Spivak borrow the term
(a) Journal of Mass Culture 'subaltern' from 'Antonio Gramsci'.
(b) Journal of Popular Culture
(c) Journal of Public Culture She used the term Subaltern in her essay "Can the
(d) Journal of Culture Studies Subaltern Speak?" shows earliest political
Ans. (b) : In 1969 the 'Department of Popular Culture' historiography shifted the voice of the Subaltern groups
at Bowling Green University launched the "Journal of (women, tribal peoples, third world). "Subaltern"
Popular Culture". The Journal carried essays on meaning "of inferior rank" is a term adopted by
Spiderman comics, rock music, amusement parks, the Antonio Gramsci, an Italina Marxist and communist
detective movies and other forms of popular culture. who was imprisoned for a long time by Mussolini's
Hence the appropriate answer is option (b). police (from 1926) until his death at age 46, to refer to
75. Which of these best describes Shyam those working class people in Soviet Union who are
Selvadurai's novel, Funny Boy? subject to the hegemony of the ruling classes.
(a) Novel of manners (b) Sentimental novel 79. Which of the following does the Subaltern
(c) Coming-of-age novel (d) Picaresque novel Studies project contend?
Ans. (c) : Funny Boy is a coming of-age novel by Sri- A. Traditional historiography celebrated the
Lankan-Canadian author Shyam Selvadurai published role of the subalterns.
in 1994. Funny Boy is constructed in the form of six B. Traditional history of India's freedom
poignant stories about a boy coming to age within a movement celebrates the contribution of
wealthy Tamil family in Colombo. He explores his select icons.
sexual identity and encounters the Sinhala-Tamil C. Traditional historiography highlights the
tensions leading up to the 1983 riots. Hence, the correct
answer is option (c). dominant strands of India's freedom
struggle.
76. Which of these questions would Cultural
Studies be most interested in asking? D. Subaltern Studies historiography highlights
A. Who decides what is to be produced? the dominant strands of India's freedom
B. Who can afford the artifact? struggle.
C. How is the artifact marketed? Choose the correct answer from the options
D. What is the register of speech in the given below:
artifact? (a) A and B only (b) B and C only
Choose the correct answer from the options (c) A and C only (d) B and D only
given below: Ans. (b) : Subaltern Studies project contends –
(a) A, B and C only (b) A, C and D only • Traditional history of India's freedom movement
(c) B, C and D only (d) A, B and D only celebrates the contribution of select icons.
Ans. (a) : Cultural studies seeks to understand how • Traditional historiography highlights the dominant
meaning is generated, disseminated, contested, bound strands of India's freedom struggle.
up with systems of power and control, and produced The Subaltern Studies began in the early 1980s as a
from the social, political and economic spheres within a critique of the existing historiography which was
particular social formation or conjuncture. accused by its initiators for ignoring the voice of the
Hence, it can be inferred that Cultural Studies would be
most interested in asking the questions 'who decides people. The writers associated with the project promised
what is to produced?', 'who can afford the artifact?' to offer a completely new kind of history in the field of
and 'How is the artifact marketed?' Thus, the correct Indian studies. Encompassing six volumes, edited by
answer is (a). Ramajit Guha, the Subaltern studies made effort to
77. Who, among the following, played the most explore the consciousness and actions of the oppressed
significant role in mobilizing the concept of group in the Indian society. Subaltern studies began as a
'cultural intermediaries'? revisionist historiography of peasant movements in
(a) Luis Althusser (b) Karl Marx colonial India.
(c) Pierre Bourdieu (d) Jurgen Habermas Hence, option (b) is appropriate answer.
UGC NTA NET/JRF English Paper II, 2022 (Sh.-I) 372 YCT
80. Which of these did the Wood's Despatch (1854) a place of magic and wonder. He cites authors ranging
seek to propagate? from Antonio Pigafetta, a Florentine navigator who
A. Impart Western knowledge to Indians went with Ferdinand Magellan on the first voyage
B. Restrict access to English learning in India around the world, to the 19th century missionaries who
C. Educate British officers in Sanskrit and told tales of cities of gold, magical creatures and
Persian mythical riches.
D. Create a class of public servants Hence the correct answer is option (d).
Choose the correct answer from the options 83. Which of these constitutes the only extant
given below: trilogy from ancient Greek tragedy?
(a) A and B only (b) B and C only (a) King Oedipus, Oedipus at Colonus, Antigone
(c) A and C only (d) A and D only (b) Agamemnon, Choephori, The Eumenides
Ans. (d) : The Wood's Despatch (1854) sought to (c) Agamemnon, Orestes, The Eumenides
propagate to impart Western knowledge to Indians (d) King Oedipus, Orestes, Antigone
and to create a class of public servants in India. Ans. (b) : Aeschylus wrote nearly nineties plays only
Wood's letter played an important role in the spread of seven have survived to the modern era. Agamemnon is
English learning and female education in India. The aim the first of a trilogy, The Oresteia. The other two parts
of Government's policy was the promotion of western of it are The Libation-Bearers (also known as
education. In his dispatch, he emphasised on the Choephori) and The Eumenides.
education of art, Science, Philosophy and literature of Agamemnon depicts the assassination of the title
Europe. One of most favorable steps taken was to create character by his wife Clytemnestra and her lover. The
an English class among Indian people to be used as Libation-Bearers continues the story with the return of
workforce in the company administration. Agamemnon's son, Orestes, who kills his mother and
Hence option (d) is the most appropriate answer. averages his father. In The Eumenides, Orestes is
81. Arrange the following periodicals in the pursued by the Furies in punishment for his matricide.
chronological order in which they started 84. Match List I with List II
publication: List I List II
A. The Spectator B. The Tatler (Author) (Form)
C. The Rambler D. The Critical Review A. Pindar I. Epinicia
Choose the correct answer from the options B. Menander II. Old Comedy
given below: C. Sappho III. Lyric poetry
(a) A, B, C, D (b) B, A, C, D D. Aristophanes IV. New Comedy
(c) B, C, D, A (d) A, D, B, C Choose the correct answer from the options
Ans. (b) : The correct chronological order of the given below:
periodicals is B, A, C, D. (a) A ‐ I, B ‐ III, C ‐ II, D ‐ IV
The Tatler (1709-1711) by Richard Steele published
(b) A ‐ I, B ‐ IV, C ‐ III, D ‐ II
three times in a weak (Tuesday, Thursday and Saturday).
The Spectator (1711-1712) by Joseph Addison and (c) A ‐ II, B ‐ I, C ‐ IV, D ‐ III
Richard Steele a daily publication to enliven morality (d) A ‐ III, B ‐ IV, C ‐ I, D ‐ II
with wit and to temper wit with morality. Ans. (b) : Correct match list is-
The Rambler (1750-1752) by Samuel Johnson, only on Author Form
Tuesday and Saturday to provide intellectual profit and Pindar Epinicia
literary delight to the readers. Menander New comedy
The Critical Review was a British publication Sappho Lyric Poetry
appearing from 1756 to1817. It was first edited by Aristophanes Old Comedy
Tobias Smollett from 1756 to 1763. Pindar- The greatest lyric poet of ancient Greece and
Contributors included Samuel Johnson, David Hume the master of epinicia (lyric ode), choral aches
John Hunter and Oliver Goldsmith. celebrating victories achieved in the Pythian, Olympic
82. Who, among these, does Gabriel Garcia and other games.
Marquez name right in the beginning of his Menander- Athenian dramatist whom ancient critics
Nobel Prize address? considered the supreme poet of Greek New Comedy.
A. Ferdinand Magellan Sappho- is known for her lyric poetry. In ancient time
Sappho was widely regarded as one of the greatest lyric
B. Christopher Columbus
poets and was given names such as the "Tenth Muse"
C. Marco Polo and "The Poetess".
D. Antonio Pigafetta Aristophanes- was a comedy writer of ancient Athens
Choose the correct answer from the options and a poet of old Attic Comedy.
given below: 85. Which of these does the book How to read
(a) A and B only (b) B and C only Donald Duck identify as an important element
(c) C and D only (d) A and D only in its analysis of imperialist ideology in the
Ans. (d) : In 1982, Garcia Marquez was awarded the Walt Disney comicbook?
Nobel Prize in Literature for his numerous novels and (a) impoverished royalty
short stories. In his acceptance speech, entitled "The (b) noble savage
Solitude of Latin America", he begins his speech by (c) scientific magic
examining the globle fascination with Latin America as (d) heartless civilisation
UGC NTA NET/JRF English Paper II, 2022 (Sh.-I) 373 YCT
Ans. (b) : Noble Savage- is a literary stock character 88. With which of these is Ngugi wa Thiongo
who embodies the concept of the indigene, outsider, generally associated?
wild human, who has not been corrupted by civilization. (a) Decolonising the State
The chapter from Noble Savage focuses on the way in (b) Decolonising the Mind
which characters from Disney Comics and specifically (c) Decolonising the Body
Donald Duck, reflected Disney's capitalist and (d) Decolonising the Polity
imperialist ideologies in their dealing with figures from Ans. (b) : Ngugi wa Thiongo generally associated with
"virgin territories of U.S. including Latin America. 'Decolonising the Mind".
86. Match List I with List II Decolonising the Mind- is the politics of language in
List I (Play) List II (Playwright) African Literature, by the Kenyan Novelist and Post
A. Bertolt Brecht I. Cat on a Hot Tin Roof colonial theorist Ngugi wa Thiong'o is a collection of
B.Tennessee Williams II. Life of Galileo essays about language and its constructive role in
C. Vaclav Havel III. Miss Julie national culture history and identity. Ngugi dedicated
D. August Strindberg IV. Temptation Decolonising the Mind to all those who over the years
Choose the correct answer from the options have maintained the dignity of the literature, culture,
given below: philosophy carried by African languages.
(a) A ‐ I, B ‐ II, C ‐ IV, D ‐ III 89. Match List I with List II
(b) A ‐ III, B ‐ II, C ‐ I, D ‐ IV List I List II
(c) A ‐ IV, B ‐ III, C ‐ I, D ‐ II (Novel) (Author)
(d) A ‐ II, B ‐ I, C ‐ IV, D ‐ III A. Don Quixote I. Machado de Assis
Ans. (d) : The correct match of play with their B. The Sorrows of II. Honore de Balzac
playwrights are- Young Werther
Author Playwright C. Lost Illusions III. Goethe
Bertolt Brecht Life of Galileo D. Epitaph of a Small IV. Miguel de
Tennessee Williams Cat on a Hot Tin Roof Winner Cervantes
Vaclav Havel Temptation Choose the correct answer from the options
August Strindberg Miss Julie given below:
Life of Galileo- is a play by the 20th century German (a) A ‐ I, B ‐ III, C ‐ II, D ‐ IV
dramatist Bertolt Brecht with incidental music by
Hamns Eisler. (b) A ‐ II, B ‐ I, C ‐ III, D ‐ IV
Cat on a Hot Tin Roof- is a Pulitzer-Prize winning (c) A ‐ IV, B ‐ III, C ‐ II, D ‐ I
play by Tennessee Williams, an adaptation of his 1952 (d) A ‐ III, B ‐ I, C ‐ IV, D ‐ II
short story "Three Players of a Summer Game". Ans. (c) : Novel Author
Temptation- is a Faustian play written by Czech Don Quixote Miguel de Cervantes
playwright Vaclav Havel in 1985. The Faust legend had The Sorrows of Young Goethe
a significant impact on Havel before he began writing Werther
Temptation. Lost Illusions Honore de Balzac
Miss Julie- is a naturalistic play written in 1888 by Epitaph of a Small Winner Machado de Assis
August Strindberg. It is set on Midsummer's Eve and Thus are the correct match of novel and their author is (C).
the following morning. Don Quixote- by Miguel de Cervantes in two parts. It
Thus, the correct option is (d) described realistically what befalls on an aging knight
87. Which of these plays by Girish Karnad shares Don Quixote who, bemused by reading chivalric
its theme with Thomas Mann's The Transposed romances, sets out on his old horse Rocinante with his
Heads? pragmatic squire, Sancho Panza, to seek adventure.
(a) Hayavadana (b) Yayati The Sorrows of Young Werther- is an epistolary
(c) Nagamandala (d) Tale Danda novel by Johann Wolfgang von Goethe. The most of
Ans. (a) : Hayavadana- is a 1971 Indian Kannada The Sorrows of Young Werther, a story about a
language two-act play written by Girish Karnad. The young man's extreme response to unrequited love.
plot is based on Kathasaritsagara and Thomas Mann's Lost Illusions- is a serial novel written by French writer
retelling of Transposed Heads. Hayavadana presents Honore de Balzac between 1837 and 1843. It consists of
the story of two friends Devadatta and Kapila and their three part starting in provincial France, thereafter
love interest in Padmini. moving to Paris and finally returning to the provinces.
Nagamandala- is derived from a Sanskrit word
Epitaph of a Small Winner- by Machado de Assis
'Nagam' which means a beautiful and charming woman
by Girish Karnad, based on a rural setting and centered published in 1881, the novel has a unique style of short,
around the life of Rani. errotic chapters shifting in tone and style. The novel is
Yayati- by Girish Karnad's first play, was written in narrated by the dead protagonist Bras cubas, who tells
1960, based on an episode in Mahabharata, where his own life story from beyond the grave, noting his
Yayati, one of the ancestors of the Pandavas, is given mistakes and failed romances.
the curse of Pre-mature old age by his father-in-law 90. Charles Lamb used the pseudonym Elia for
Shukracharya, who is increased by Yayati's infidelity. writing in which of the following periodicals?
Tale Danda- by Girish Karnad, an eminent person in (a) London Magazine
Kannada. The play is about the rise of the radical (b) The Edinburgs Review
protest and reform movement. (c) The Quarterly Review
Hence, option (a) is correct answer. (d) Athenaeum
UGC NTA NET/JRF English Paper II, 2022 (Sh.-I) 374 YCT
Ans. (a) : Lamb used the pseudonym Elia for writing in Choose the correct answer from the options
London Magazine periodical. given below:
London Magazine- "Elia" being pen name, Lamb used (a) A-I , B-III , C-IV , D-II
as contributor to the London Magazine. Lamb's (b) A -III , B-IV , C-II , D-I
collected essays under the title "Essays of Elia", was (c) A -II , B-III , C-IV , D-I
published in 1823. (d) A -IV , B-I , C-II , D-III
The Edinburgh Review- is the title of four distinct Ans. (c) : Correct matched option is (c) (A -II , B-III ,
intellectual and cultural magazines. C-IV , D-I)
The Quarterly Review- was a literary and political Essay Essayist Year of
periodical founded in 1809 by London publishing house publication
John Murray. A. "The Tory Fox- II. Joseph Addison
Athenaeum- was a British literary magazine published Hunter"
in London, England from 1828 to 1921. B. "What I Believe" III. E.M. Forster 1939
Thus, the correct option is (a). C. "The Death of the IV. Virginia Woolf 1942
91. In "Mr Bennett and Mrs Brown" Virginia Moth"
Woolf: D. "Of Ambition" I. Francis Bacon 1612
(a) responds to E.M Forster's remarks on "Of Ambition" is an argumentative essay by Francis
character in fiction. Bacon, in which he argues about the benefits and
(b) criticises book buying preferences of the outbursts of being ambitious.
educated English class.
(c) analyses the state of modern fiction by 94. Arrange the following characters in their
contrasting two generations of writers. chronological sequence of appearance:
(d) presents modernity as a stable and coherent A. Mirabell
project uniting all artists. B. Shylock
Ans. (c) : In "Mr. Bennett and Mrs. Brown" Virginia C. Jimmy Porter
Woolf analyses the state of modern fiction by D. Sir Epicure Mammon
contrasting two generations of writers. The essay is Choose the correct answer from the options
framed as a response to an essay by novelist Arnold given below
Bennett in which he declares that the current generation (a) D, B, A, C (b) B, D, A, C
of Georgian authors- D.H. Lawrence, James Joyce, T.S. (c) D, B, C, A (d) B, D, C, A
Eliot-have failed as writers because they have not Ans. (b) : The appropriate chronological sequence of
created real, convincing characters. But Woolf characters are-
challenges Bennett's concept of reality. Shylock → Sir Epicure Mammon → Mirabell → Jimmy
Thus, appropriate answer is (c). Porter.
92. In "Politics and the English Language" which 'Shylock' is character of William Shakespeare" play
two of the following 'tricks' are mentioned by 'The Merchant of Venice' published in 1600.
George Orwell as 'bad habits' of English use? 'Sir Epicure Mammon' is character of Ben Jonson's
A. obsolete words B. pretentious diction comedy 'The Alchemist', first performed in 1610 by the
C. dying metaphors D. false modifiers King's Men.
Choose the correct answer from the options 'Mirabell' is character of William Congreve's play 'The
given below: Way of the World', published in 1700.
(a) A and B only (b) B and C only 'Jimmy Porter' is a character of John Osborne's play
(c) B and D only (d) C and D only 'Look Back in Anger' formed in 1956, published in
Ans. (b) : Pretentious diction and Dying Metaphors. 1957.
"Politics and the English Language" is an essay by Hence, the appropriate answer is (b).
George Orwell that criticised the 'ugly and inaccurate' 95. Which two of the following concepts are
written English of his time and examines the connection deployed in the work of Frederic Jameson?
between political orthodoxies and the debasement of A. Pastiche B. Hyperreal
language. In the remedy of six rules, Orwell said that C. Schizophrenia D. Habitus
never use a metaphor, simile or other figure of speech Choose the correct answer from the options
which you are used to seeing in print. He described such given below:
phrases "ring the changes" "Achilles heel, Swan song" (a) A and B only (b) C and D only
as dying metaphors and argued that they were used (c) B and D only (d) A and C only
without knowing what was truly being said. Ans. (d) : Pastiche and Schizophrenia concepts are
Never use a long word where a short one will do. deployed in the work of Frederic Jameson. Fredric
Never use a foreign phrase, a scientific word or a jargon Jameson is an American literary critic philosopher and
word if you can think of an everyday English equivalent.
Thus, the correct option is (b). Marxist political theorist. He is best known for his
analysis of contemporary cultural trends, particularly
93. Match List I with List II his analysis of postmodernity and capitalism. He is
List I (Essay) List II (Essayist) known for his books- 'The Cultural Logic of Late
A. "The Tory Fox- I. Francis Bacon Capitalism' (1991) and 'The Political Unconscious'
Hunter" (1981). According to Jameson, parody has, in the
B. "What I Believe" II. Joseph Addison postmodern age, been replaced by Pastiche. Pastiche is
C. "The Death of the III. E.M.Forster like parody, the imitation of a peculiar or unique,
Moth" idiosyncratic style, the wearing of a linguistic mask,
D. "Of Ambition" IV. Virginia Woolf speech in a dead language.
UGC NTA NET/JRF English Paper II, 2022 (Sh.-I) 375 YCT
Schizophrenia is the psychological response to the Choose the correct answer from the options
conditions of postmodernity. given below:
Thus, the correct answer is (d). (a) A and B only (b) C and D only
96. Arrange the following plays in their (c) B and D only (d) A and C only
chronological sequence: Ans. (c) : The speaker might mean when he addresses
A. Serjeant Musgrave's Dance 'Time' in a Shakespearean sonnet and declares that "I
B. The Playboy of the Western World will be true, despite thy scythe and thee", 'with time
C. Look Back in Anger comes change' and 'Time removes human life'. The line
D. Man and Superman "I will be true, despite they scythe and thee" is taken
Choose the correct answer from the options from 'Sonnet 123' by William Shakespeare, depicts the
given below revenges of time and how he sees himself, his love and
(a) D, B, A, C (b) B, D, A, C his character as beyond them. In the line, the speaker
(c) D, B, C, A (d) B, D, C, A says that no matter what happens in the world, he's
Ans. (c) : The appropriate chronological sequence of going to be true to himself and to his love.
plays are- 99. Which two of the following stage directions are
Man and Superman→The Playboy of the Western from Harold Pinter's The Birthday Party?
World→Look Back in Anger→Serjeant Musgrave's Dance. A. The living‐room of a house on a seaside town.
'Man and superman' is a four act drama written by G.B. B. A garbage pail on the ground next to the
Shaw in 1903. The series has written in response of a call porch steps.
for Show to write a play based on the Don Juan theme. C. A light shows from upstairs bedroom, lower
'The Playboy of the Western World' comedy in three floor windows being dark.
acts by J.M. Synge, published and produced in 1907. It D. He hangs the drum around his neck, taps it
gently with the sticks, then marches round
is a masterpiece of the Irish literary renaissance. the table, beating it regularly.
'Look Back in Anger' (1957) is a realist play written Choose the correct answer from the options
by John Osborne. It focuses on the life and marital given below:
struggles of an intelligent and educated but disaffected (a) A and D only (b) C and D only
young man of working class. The protagonist of this (c) B abd C only (d) A and C only
play is 'Jimmy Porter'. 'Serjeant Musgrave's Dance' Ans. (a) : 'The living-room of a house on a seaside
(1959) is regarded as John Arden's first important play. town' and 'He hangs the drum around his neck, taps it
Hence, option (c) is correct answer. gently with the sticks, then marches round the table,
97. Match List I with List II beating it regularly' are the two stage directions from
List I List II Harold Pinter's The Birthday Party.
(Text) (Author) The Birthday Party, drama in three acts by Harold
A. Modernity at Large I. J. Urry Pinter, produced in 1958 and published in 1959. Pinter's
B. The Tourist Gaze II. E.W.Said first full-length play established his trademark "comedy
C. Culture and III. C.L.R. James of menace", in which a character is suddenly threatened
Imperialism by the vague horrors at large in the outside world.
D. The Black Jacobins IV. A. Appadurai The Birthday Party was first staged at the Lyric
Hammersmith.
Choose the correct answer from the options
given below: 100. Arrange the following lines of poetry in their
chronological sequence:
(a) A‐IV , B‐III , C‐II , D‐I A. "An aged man is but a paltry thing."
(b) A‐III , B‐IV , C‐II , D‐I B. "The world is too much with us."
(c) A‐II , B‐III , C‐IV , D‐I C. "Daddy, I have had to kill you."
(d) A‐IV , B‐I , C‐II , D‐III D. "After great pain, a formal feeling comes"
Ans. (d) : Correct match of the given text and Authors Choose the correct answer from the options
are as : (A‐IV , B‐I , C‐II , D‐III) given below:
Text Author Year (a) B, D, A, C (b) D, B, A, C
A. Modernity at Large (IV). Arjun Appadurai 1996 (c) D, B, C, A (d) B, D, C, A
(A. Appadurai) Ans. (a) : Lines of poetry in their Chronological
B. The Tourist Gaze (I). John Urry 1990 sequence will be as (B, D, A, C).
(J. Urry) 1807- "The world is too much with us".
C. Culture and (II). Edward Wadie 1993 1862- "After great pain, a format feeling comes"
Imperialism (E.W.Said) 1928- "An aged man is but a paltry thing".
1965- "Daddy, I have had to kill you".
D. The Black Jacobins (III). (Cyril Lionel Robert) 1938 The line......." is taken from "The World Is Too Much
(C.L.R. James) with Us" (1807) by the English Romantic poet William
98. What might the speaker mean when he Wordsworth.
addresses 'Time' in a Shakespearean sonnet The line 'The World is Too Much With Us', "After great
and declares that "I will be true, despite thy pain, a formal...." is taken from Emily Dickinson's
scythe and thee"? poem "After great pain, a formal feeling comes" (1862).
A. Time preserves human life. The line "An aged man is....." is taken from "Sailing to
B. With time comes change. Byzantium" 1928, a poem by William Butler Yeats.
C. Time creates opportunities. The line "Daddy, have had......" is taken from "Daddy",
D. Time removes human life. published in 1965, by Sylvia Plath.
UGC NTA NET/JRF English Paper II, 2022 (Sh.-I) 376 YCT
UGC NTA NET/JRF Exam. Dec. 2021/June 2022
ENGLISH-II
SOLVED PAPER [Exam Date : 13.10.2022, Shift-II

1. Given below are two statements: 'Trainspotting' is his first novel which was published in
Statement I: In My Remembered Village 1993 and was his bestselling novel. It revolves around
(1976), M.N. Srinivas highlights the various residents of Leith, Edinburgh, who are
ethnographical details of a village, Rampura, somewhere engaged in destructive activities that are
near Mysore. effectively addictions like heroin.
Statement II: It also emphasises on the 'Filth' 1998 is his novel which deals with racism, drug
importance of economic freedom of the masses abuse, sexism, alcohol abuse, among other problems
required for their overall upliftment. faced by the Scottish working class.
In light of the above statement choose the most 'Beside The Ocean of Time' (1994) is a novel by
appropriate answer from the options given 'George Mackay Brown' while 'Mavis Belfrage' is a
below.
novel by 'Alasdair Gray'.
(a) Both Statement I and Statement II are correct
Hence, option (d) is correct.
(b) Both Statement I and Statement II are
incorrect 3. Which of the following are representative texts
(c) Statement I is correct but Statement II is of "Gynocriticism"?
incorrect A. Patricial Meyer Spacks' The Female
(d) Statement I is incorrect but Statement II is Imagination
correct B. Mary Ellman's Thinking About Women.
Ans. (c) : In the above given two statements, statement C. Sandra Gilbert and Susan Gubar's The
I is correct and statement II is incorrect. MadWoman in the Attic
'My Remembered Village' 1976 is a work by M.N. D. Ellen Moer's Literary Women
Srinivas and it highlights the ethnographical details of E. Kate Millett's Sexual Politics
Rampura, near Mysore, but it does not pay attention to Choose the correct answer from the options
economic freedom of the masses, infact it emphasises given below:
the day-to-day social relations between members of
(a) A and B only (b) A, B and C only
diverse castes living in a small village community in
India. (c) B, D and E only (d) A, C and D only
Hence option (c) is correct. Ans. (d) : Gynocriticism is the study of women's
2. Which of the following are novels of Irvine writing. The term gynocritics was coined by Elaine
Welsh? Showalter in 1979 to refer to a form of feminist literary
A. The Acid House criticism that is concerned with women as writers.
B. Trainspotting 'The Female Imagination', 'The Madwomen in the Attic'
C. Beside The Ocean of Time and 'Literary Women' are all representative texts of
D. Filth Gynocriticism.
E. Mavis Belfrage Patricial Meyer Spacks 'The Female Imagination'
Choose the correct answer from the options (1975), surveys a number of female writers with the aim
given below: of identifying how a woman's creative voice differs
(a) B, D and E only (b) A, B and C only from a man's. This non-fiction book was a nominee for
(c) B, C and D only (d) A, B and D only the National Book Award. Sandra Gilbert and Susan
Ans. (d) : Irvine Welsh is a Scottish novelist, Gubar's 'The Madwoman in The Attic' (1979), attempts
playwright and short story writer. He was born in to define a 'distinctively' female literary tradition. It
Edinburgh in 1958. presents an analysis of a trope found in 19th century
Out of all the options, 'The Acid House' 'Trainspotting' literature.
and Filth are his works. Ellen Moer's 'Literary Women' (1976) is one of the
'The Acid House' was published in 1994 and is an pioneering works of feminist criticism that separates
unsettling, shocking and very funny collection of 22 women from the mainstream of literary history.
short stories. Hence option (d) is correct.

UGC NTA NET/JRF English Paper II, 2022 (Sh.-II) 377 YCT
4. "Inane gaudiness" is a phrase used in Choose the correct answer from the options
connection with Neo-classicism by_______ given below:
(a) William Wordsworth (b) S.T. Coleridge (a) A-III, B-II, C-IV, D-I
(c) Matthew Arnold (d) T.S. Eliot (b) A-IV, B-II, C-I, D-III
Ans. (a) : 'Inane gaudiness' is a phrase used in (c) A-II, B-III, C-I, D-IV
connection with Neo-classicism by William (d) A-I, B-IV, C-II, D-III
Wordsworth. Wordsworth was against the gaudiness Ans. (d) :
and inane phraseology of the 18th century poets which Writers (List-I) Works (List-II) Year
brought forward his own theory of language of poetry.
According to him, the language of poetry should be the A. Cornelia 1. Between the 1908
real language of men without any artificiality, and by Sorabji Twilights
men he meant the rustic folks and simple people. He B. Krupabai 4. Saguna, A Story of 1895
believes that poetry should be the real language of men Satthianadhan Native Christian
in a state of vivid sensation and a poet should use Life
language, which is without ornamentation, coarseness C. Raj Lakshmi 2. The Hindu Wife 1876
and oddities. Debi
Hence, option (a) is correct. D. Kamala 3. Nector in a Sieve 1954
5. Which of the following is applicable to "New Markandaya
Criticism"? Hence, option (d) is correct.
A. It draws considerably from the works of I.A. 7. The following statement is written by which of
Richards and the critical essays of T.S. Eliot. the authors given below:-
B. Some of its concepts are pre-empted by F.R. "Could fulfillment ever be felt as deeply as
Leavis. loss?"
C. It distinguishes between literary and scientific (a) Arundhati Roy (b) Kiran Desai
usage of language. (c) Shashi Tharoor (d) Rohinton Mistry
D. It encourages an extensive exploration of the
contextual and autobiographical background of Ans. (b) : "Could fulfillment ever be felt as deeply as
a literary production. loss" is a quote by Kiran Desai in her second novel,
'The Inheritance of Loss' which was first published in
E. It vouches for a historical analysis of a text.
2006 and won the Booker Prize for that year. The novel
Choose the correct answer from the options deals with the conflict of culture, on the global level and
given below: on personal level it deals with the internal conflict of
(a) A, B and C only (b) B, E and D only identity. In the novel, Desai tries to capture the pair and
(c) A, C and D only (d) B and D only dilemma of an immigrant.
Ans. (a) : New Criticism examines the relationship Hence, option (b) is correct.
between a text's ideas and its form, between what a text 8. Who among the following refers to "high
says and the way it say it. I.A. Richards has been called seriousness" as a quality of a great poet and
the father of New Criticism as he was one of the first to quotes John Milton to prove the same?
study literary interpretation as a kind of science. New A. T.S. Eliot
Criticism draws considerably from the works of I.A.
B. Ezra Pound
Richards and the critical essays of T.S. Eliot and some
of its concepts are pre-empted by F.R. Leavis. It C. Matthew Arnold
distinguishes between literacy and scientific usage of D. I. A. Richards
language. New Criticism emphasises on close reading, E. G. M. Hopkins
particularly of poetry, to discover how a work of Choose the correct answer from the options
literature functioned as a self contained, self-referential given below:
aesthetic object. (a) A and By only (b) B and C only
Hence, option (a) is correct. (c) D and E only (d) C only
6. Match List I with List II Ans. (d) : Matthew Arnold refers to 'high seriousness'
List-I List-II as a quality of a great poet and quotes John Milton to
A. Cornelia Sorabji I. Between the prove the same. According to Arnold, the best poetry is
Twilights born of sincerity of feelings and emotions. High
seriousness means the grand style which is in poetry
B. Krupabai II. The Hindu Wife where a serious subject is treated in a simple and intense
Satthianadhan manner. Arnold believes that though Chaucer is one of
C. Raj Lakshmi Debi III. Nector in a Sieve the greatest classics of English poetry, he lacks the high
D. Kamala IV. Saguna, A Story seriousness that is found in the works of Shakespeare
Markandaya of Native and Milton.
Christian Life Hence, option (d) is correct.

UGC NTA NET/JRF English Paper II, 2022 (Sh.-II) 378 YCT
9. The concept of "eugenics" finds its illustration 11. Marge Piercy's novel 'Woman on the Edge of
predominantly in the writings of: Time' (1976) is based on.
A. John Osborne (a) Shulamith Firestone's The Dialectic of Sex
B. George Bernard Shaw (b) Margaret Atwood's The Handmaid's Tale
C. Eugene O'Neill (c) Lewis Caroll's Alice in Wonderland
D. Harold Pinter (d) Donna Haraway's A Cyborg Manifesto
E. Arthur Miller Ans. (a) : Marge Piercy's novel 'Woman on the Edge of
Choose the correct answer from the options Time' (1976) is based on Shulamith Firestone's 'The
Dialectic of Sex' (1970).
given below:
'Woman on the Edge of Time' is the moving story of
(a) A and B only (b) A and C only
Connie Ramos, a 37 year old Mexican-American,
(c) B and C only (d) D and E only unfairly incarcerated in a mental hospital, whose
Ans. (c) : The concept of "eugenics" finds its survival instinct is greatly tested. The novel is
illustration predominantly in the writings of 'George considered as a classic of utopian speculative science
Bernard Shaw' and 'Eugene O' Neill'. fiction as well as a feminist classic.
Eugenics literally means 'good creation' and the term Hence, option (a) is correct.
was first coined by British scholar 'Sir Francis Galton' 12. "The Elegy Written in a Country Churchyard"
in 1883 in his book 'Inquiries into Human Faculty and is written in:
Its Development'. (a) Quatrains of ten syllable lines
G.B. Shaw expressed an interest in eugenics, especially (b) Octave and Sestet
in preventing the deterioration of civilization. (c) Heroic couplet
G.B. Shaw was an Irish playwright and the recipient of (d) Alexandrines
the 1925 Nobel Prize in Literature, who promoted Ans. (a) : 'The Elegy Written in a Country Churchyard'
Eugenics and also advocated for a far-reaching eugenics is written in quatrains of ten syllable lines. 'Elegy
program. Written in a Country Churchyard' is a poem by 'Thomas
Eugene O' Neill was an American playwright and Nobel Gray', which was published in 1751. It is a meditative
laureate, who both adopts and reconfigure ideas of poem written in iambic pentameter quatrains and is one
heredity and eugenics in strange interlude in order to of the best known elegies as it mourns the death not of
explore the influence of the past on the present and the great or famous people but of common men. The
speaker of this poem sees a country churchyard at
power and limits of visibility.
sunset, which impels him to meditate on the nature of
Hence, option (c) is correct. human mortality.
10. Dev Virahsawmy's 'Toufann' is an adaptation The poem is written in four-line stanzas, or quatrains.
of Shakespeare's play______. Each quatrain follows the rhyme scheme 'abab'. The
(a) Hamlet (b) Macbeth meter of each line is iambic pentameter, meaning five
(c) The Twelfth Night (d) The Tempest iambs (an iamb is a two-syllable foot where an
Ans. (d) : Dev Virahsawmy's 'Toufann' is an adaptation unstressed syllable is followed by a stressed syllable)
per line.
of Shakespeare's 'The Tempest'
Hence, option (a) is correct.
The imprint of Shakespeare on Dev Virahsawmy is
greater than his imprint on all other Mauritian writers, 13. Which of these is related exclusively with
to the extent that Virahswamy's engagement extends Jacques Lacan?
beyond influence, borrowing, blueprinting or A. Real
bardolatry. B. Id
In most writers, the use of Shakespeare is referential but C. Symbolic
in his case it is far more sophisticated. D. Ego
E. Imaginary
'Toufann : A Mauritian fantasy' was published in 1999
and was originally written in 'Creole' and was later Choose the correct answer from the options
given below:
translated into English by Nisha and Michael Walling.
(a) A and B only (b) B and D only
Its original title was 'Ennfanteziant Swaak', which
(c) C and E only (d) A and E only
means 'a fantasy in three acts'.
Unlike so many adaptations of Shakespeare 'Toufann' is Ans. (*) : Jacques Lacan was a French Psychoanalyst,
not only a postcolonial adaptation, it's an effort to whose teachings and writing explore the significance of
Freud's discovery of the unconscious both within the
recreate, rewrite the play in an independent atmosphere
theory and practice of analysis itself as well as in
of 21st century times which gives the sense of
connection with a wide range of other disciplines. Out
'transcreation' to the play. of the options given, real, symbolic and imaginary are
Hence, option (d) is correct. related exclusively with Lacan. In Lacan's opinion, the
UGC NTA NET/JRF English Paper II, 2022 (Sh.-II) 379 YCT
human psyche is patterned according to three orders: 16. Match List I with List II
'imaginary order', 'symbolic order' and the 'real order' List I List II
the Imaginary is derived from perceptual and fantasied A. Yasmine I. Post Card from
mental process, and the symbolic is derived from Gunaratne Kashmir
culture and through language.
B. Imtiyaz Dharkar II. Advice to Women
Note- The correct combination will be- option (A), (C)
and (E) which is not given. Hence, both option (c) and C. Agha Shahid Ali III. Post Card from
(d) are correct. God
14. The following statement is given by which of D. Eunice de Souza IV. This Language,
this Woman
the below mentioned critics?
"I was indignant; I demanded an explanation. Choose the correct answer from the options
Nothing happened. I burst apart." given below:
(a) A-I, B-IV, C-II, D-III
(a) Gayatri Chakravorty Spivak
(b) A-IV, B-III, C-I, D-II
(b) Homi K Bhabha
(c) A-I, B-II, C-III, D-IV
(c) Stuart Hall
(d) A-IV, B-III, C-II, D-I
(d) Frantz Fanon
Ans. (b) : The correct match will be-
Ans. (d) : The statement' I was indignant, I demanded
List-I (Writer) List-II (Works) Year
an explanation nothing happened. In burst apart was
given by 'Frantz Fanon'. Frantz Fanon was French A. Yasmine IV. This Language, 1971
psychiatrist, who is best known for the classic analysis Gunaratne this Woman
of colonialism and decolonization, 'The wreched of the B. Imtiyaz III. Post Card from 1994
Earth' (1961), which was written by 'Editions Maspero' Dharkar God
Fanon perceived colonialism as a form of domination C. Agha Shahid I. Post Card from 1991
whose necessary goal for success was the reordering of Ali Kashmir
the world of indigenous people. D. Eunice de II. Advice to 1994
Hence, option (d) is correct. Souza Women
15. Which of the following statements are true in Hence option (b) is correct.
the context of Henry Fielding's Tom Jones 17. Which of the following are correct:
(1749). A. Roddy Doyle- The Barrytown triology
A. Tom Jones is comic and moralistic. B. Abraham Stoker- Confessions of an Irish Rebel
B. Tom and Jones are the main characters of Tom C. C. S. Lewis- The Chronicles of Narnia
Jones D. Brendan Behan- Dracula
C. Tom is caught poaching in neighbor's game E. Eoin Colfer- Artemis Fowl
preserve. Choose the correct answer from the options
D. The History of Tom Jones is the full title of given below:
Tom Jones (a) A, C and E only (b) A, B and D only
E. All of the above (c) A, C and D only (d) C, D and E only
Choose the correct answer from the options Ans. (a) : 'The Barrytown Triology' is an Irish comedy-
given below: drama by Roddy Doyle which was later adapted as a
(a) E only (b) A, C and D only film in 1996. The triology consists of novels-
(c) A, B, C and D only (d) B, C and D only 'The Commitments' (1987), 'The Snapper' (1990), 'The
Van' (1991), 'The Guts' (2013) and 'Paddy Clarke Ha Ha
Ans. (b) : The statements that are true in the context of Ha' (a spin off-1993) along with 2 short stories 'Two
Henry Fielding's 'Tom Jones' (1749) are- Pints' (2012) and 'Two More Pints (2014)'
• Tom Jones is comic and moralistic. 'The Chronicles of Narnia' is a series of high fantasy
• Tom is caught poaching in neighbour's game novels by British author C.S. Lewis. These 7 novels are-
preserve. 'The Lion, the Witch and the Wardrobe' (1950), 'Prince
• 'The History of Tom Jones' is the full title of Tom Caspian : The Return to Narnia' (1951), 'The Voyage of
Jones. 'Tom Jones' is considered as one of the first the Dawn Trader' (1952), 'The Magician's Nephew'
(1955), & 'The Last Battle' (1956)
prose works describable as a novel and the novel is
divided into 18 smaller books. 'Artemis Fowl' is an Irish young adult fantasy novel
series by an Irish author Eoin Colfer. The novels in the
The main theme of the novel is the contrast between series are - 'Artemis Fowl' (2001), 'The Arctic Incident'
Tom Jones' good nature, flawed but eventually (2002), 'The Eternity Code' (2003), 'The Opal
corrected by his love for virtuous Sophia Western, and Deception' (2005), 'The Lost Colony' (2006), 'The Time
his half-brother Blifil's hypocrisy. Paradox' (2008), 'The Atlantis Complex' (2010), 'The
Hence, option (b) is correct. Last Guardian' (2012).
UGC NTA NET/JRF English Paper II, 2022 (Sh.-II) 380 YCT
"The Confessions of an Irish Rebel" is a book by Irish 20. According to Gerard Genette, which of the
writer 'Brendan Behan' and "Dracula" is a novel by following are types of narrators?
'Bram Stoker' who is also an Irish writer. A. Extradiegetic
Hence, option (a) is correct. B. Heterodiegetic
C. Intradiegetic
18. Sir David Lyndsay's Satire of the Three Estatis
D. Homodiegetic
is both
E. Interdiegetic
(a) Political and religious Choose the correct answer from the options
(b) Political and secular given below:
(c) Immoral and secular (a) B and D only (b) A and B only
(d) Immoral and religious (c) C and D only (d) A and E only
Ans. (a) : (Ane Satyre of the Thrie Estaits') 'Satire of Ans. (a, b & c) : Gerard Genette was a French critic
the Three Estates' is a satirical morality play in Middle who argued for the autonomous nature of the literary
Scots written by Scottish poet David Lyndsay. The play text according to Genette, a narrator can be of any type-
is both political and Religious. The complete play was Homodiegetic, Heterodiegetic, Intradiegetic,
first performed outside in the playing field at Cupar, Extradiegetic and Autodiegctic. The Heterodiegetic
narrator is one who does not participate in the story.
Fife in June 1552 during the midsummer holiday, where
The Extradiegetic narrator is above the story. When
the action took place under the castle Hill. The full text characters in the story narrated by him or her, then
was first printed in 1602. The Satire is an attack on the She/he becomes a homodiegetic intradiegetic narrator.
three Estates represented in the Parliament of Scotland- When a character narrates her/his own tale they may be
the Clergy, lords and burgh representatives, symbolised described as autodiegetic narrators.
by the characters Spiritualitie, Temporalitie, and There is no such thing as Interdiegetic narrator.
merchant. The Clergy come in for the strongest Note- The correct combination of options should be
criticism. The work portrays the social tensions present (A), (B), (C) and (D) but as there is no such option, all
at this pivotal moment in Scottish history. three options (a), (b), (c) are considered correct.
Hence, option (a) is correct. 21. Which of the following authors have been
19. Given below are two statements: correctly matched with their works?
Statement I : Roland Barthes believes that the A. Malcom Bradbury- The History of Man
author is "the epitome and culmination of B. William Golding-Rites of Passage
C. Seamus Heaney- Darkness visible
capitalist ideology".
D. Brian Friel- Dancing at Lughnasa
Statement II: Walter Benjamin, in his 'The
E. Molly Keane- The Norman Conquests
Storyteller', observes that every real story has
Choose the correct answer from the options
something useful in it. given below:
In light of the above statements, choose the (a) A, B and E only (b) B, C and E only
correct answer from the options given below (c) C, D and E only (d) A, B and D only
(a) Both Statement I and Statement II are true Ans. (d) : 'The History Man' is Malcolm Bradbury's
(b) Both Statement I and Statement II are false masterpiece, and one of the most influential novels of
(c) Statement I is true but Statement II is false the 1970s. It is a definitive campus novel in which
(d) Statement I is false but Statement II is true Bradbury brilliantly satirizes a world of academic
power struggles as his anti-hero seduces his away
Ans. (a) : Both given statements are true as Roland
around campus.
Barthes believes that the author is the epitome and
'Rites of Passage' (1980) by William Golding is an
culmination of capitalist ideology. Roland Barthes was account of a voyage by sea to Australia.
a French essayist, who argued that seemingly innocent 'Dancing at Lughnasa' is 1990 play by Brian Friel. It is
objects and images carry a literal denotative meaning a memory play told from the perspective of an adult
and a non-literal-connotative meaning. Along with this, Michael. It is set in the fictional Irish village of
'The Storyteller : Tales out of Loneliness' is a book by Ballybeg, Ireland.
Walter Benjamin in which he observes that every real 'Darkness Visible' is book by William Styron and 'The
story has something useful in it. The book is a beautiful Norman Conquest' is a book by More Morris.
collection of short stories. His stories revel in the erode Hence option (d) is correct.
tensions of city life, cross the threshold between rational 22. Which of the following is NOT a method of
and hallucinatory realms, celebrate the importance of data analysis?
games and delve into the peculiar relationship between (a) Qualitative method
gambling and fortune-telling, and explode the themes (b) Quantitative method
that defined Benjamin. (c) Mixed method
Hence option (a) is correct. (d) Subjective method
UGC NTA NET/JRF English Paper II, 2022 (Sh.-II) 381 YCT
Ans. (d) : Out of the given options, subjective method British politician, on February 2nd circulated 'Minute on
is not a method of data analysis. Qualitative, Education' - a treatise that offered definitive reasons for
Quantitative and mixed are methods of data analysis. why the East India Company and the British
Quantitative data analysis involves working with Government should spend money on the prevision of
numerical variables including statistics percentages, English language education, as well the promotion of
calculations, measurements and other data. It typically European learning, especially the sciences, in India.
include working with algorithms, mathematical analysis Hence, option (b) is correct.
tools and software to manipulate data and uncover 25. Given below are two statements:
insights that reveal the business value. Statement I : The best poetry will be found to
Qualitative data describes information that is typically have a power of forming, sustaining and
nonnumerical. It involves working with unique delighting us, as nothing else can.
identifiers, such as labels & properties and categorical Statement II : No man has ever been a great
variables, such as stats, percentages and measurement. poet without being, at the same time, a great
Mixed research method is a method that combines and critic.
integrates qualitative and quantitative research methods In light of the above statements, choose the
in a single research study. correct answer from the options given below
Hence option (d) is correct. (a) Both Statement I and Statement II are true
23. Which of the following are true in the context (b) Both Statement I and Statement II are false
of "Phenomenology". (c) Statement I is true but Statement II is false
A. It is a form methodological idealism" which (d) Statement I is false but Statement II is true
seeks to explore 'human consciousness'.
Ans. (c) : In the above given statements, statement I is
B. As a philosophical method, it was developed by
true and II is false.
Edmund Husserl.
Statement (I) The best poetry will be found to have a
C. Martin Heidegger is one of its leading
power of forming sustaining and delighting as, as
philosophers.
nothing else can; is a statement by Matthew Arnold in
D. Martin Heidegger's approach is "essentialist". 'The Study of Poetry'. According to Arnold, more and
E. Text is considered purely as an embodiment of more mankind will discover that we have to turn to
the authors consciousness. poetry to interpret life for us, to console us, to sustain
Choose the correct answer from the options us. To Arnold Poetry itself was the criticism of life.
given below: Statement (II), 'No man has ever been a great poet
(a) A, B, C and D only (b) B, C and D only without being, at the same time, a great critic' is false as
(c) A, B, C and E only (d) C, D and E only the correct statement was said by 'Samuel Taylor
Ans. (c) : Phenomenology is the study of structures of Coleridge' that, 'No man was ever a great poet, without
consciousness as experienced from the first person point being at the same time a profound philosopher.
of view. It is further concerned with our distorted Hence option (c) is correct.
understanding of the world. It is a form of 26. Which of the following works has Santiago as
methodological idealism which seeks to explore 'human its protagonist?
consciousness'. It was developed by 'Edmund Husserl'
(a) Mansfield Park
who defined it as 'the science of the essence of
consciousness'. 'Martin Heidegger' is one of its reading (b) The Mayor of Casterbridge
philosophers. According to him, Phenomenology is the (c) The Old Man and the Sea
way of access to what is to become the theme of (d) The Lord of the Rings
anthology. In this method, text is considered purely as Ans. (c) : Santiago was the protagonist of 'The Old Man
an embodiment of the authors consciousness. and the Sea'
The correct combination of options will be- (A), (B), 'The Old Man and the Sea' is a short novel by Ernest
(C) and (E). Hemingway, which was published in 1952. It won the
Hence option (c) is correct. Pulitzer Prize for fiction in 1953. It is the story of an
24. When did T.B. Macaulay present his "Minutes epic struggle between an old, seasoned fisherman and
on Education" advocating English education in the greatest catch of his life. The story centers on an
India? aging Santiago who engages in an epic battle to catch a
(a) 12th March, 1835 (b) 2nd February, 1835 giant marlin.
(c) 22nd January, 1835 (d) 5th May, 1835 In the novel, Hemingway is making the point that being
Ans. (b) : T.B. Macaulay present his 'Minutes on determined and never giving up indicated what kind of
Education' advocating English education in India on 2nd human being one is. This is a universal theme.
February, 1835. On this day in 1835, Lord Macaulay Reflecting on the old man's experience is an inspiration
successfully westernised education in India, English for living one's own life and dealing with its problems.
was made the official language for the government and In this novella the sea as a symbol for the whole of
courts and was adopted as the official modicum of nature.
instruction. Pressing his debate to westernise India. The Hence option (c) is correct.
UGC NTA NET/JRF English Paper II, 2022 (Sh.-II) 382 YCT
27. Given below are two statements: Ans. (c) : The correct match will be-
Statement I : Dr. Johnson had an inclination List-I (Works) List-II Year
toward the Tory political ideology. (Authors)
Statement II : Dr. Johnson strongly believed in
A. The Lions II. W. Somerset 1911
transcendental scepticism.
Skin Maugham
In light of the above statements, choose the
correct answer from the options given below B. The Man IV. Evelyn Waugh 1933
(a) Both Statement I and Statement II are true who liked
(b) Both Statement I and Statement II are false Dickens
(c) Statement I is true but Statement II is false C. Rip Van I. Washington 1819
(d) Statement I is false but Statement II is true Winkle Irving
Ans. (a) : Both the above given sentences are correct. D. The Bride III. Stephen Crane 1898
Politically, Johnson was a conservative and A Tory is a comes to
person who holds a political philosophy known as Yellow Sky
Toryism, based on a British version of traditionalism Hence, option (c) is correct.
and conservatism, which upholds the supremacy of 30. Gabriel Garcia Marquez's postscript to The
social order as it has evolved in the English culture General in His Labyrinth (1991) about Simon
throughout history. To Johnson's mind wisdom, not Bolivar's last and terrible journey to the
political rhetoric, was the key : in important matters, Carribean coast of Nueva Granada in 1830 is
Johnson said, "a wise Tory and a wise whig, I believe, one of the finest examples of______.
will agree".
(a) reading archive and using its contents.
According to statement (II), Dr. Johnson strongly
believed in transcendental scepticism. Johnson's counter (b) processing archival information for
sceptical impulses towards conclusiveness and general theoretical judgment
assertion in argument existed in a vital, dialogic tension, (c) irrelevance of archive for an artist
often expressed as irony, with his understanding of (d) a novelist getting misguided by the archive.
thinking as an open ended, ongoing process where Ans. (a) : Gabriel Garcia Marquez is a Colombian
conclusions are at best partial and provisional. novelist, who was one of the best known Latin
Hence option (a) is correct. American writers in history. He won a Nobel Prize for
28. Who defined 'Hamartia' as 'tragic flaw'? Literature, mostly for his masterpiece of magic realism.
(a) Aristotle (b) Dr. Samuel Johnson this most political novel. 'The General in this
(c) Mathhew Arnold (d) A.C. Bradley Labyrinth', is the tragic story of General Simon Bolivar
who tried to unite a continent.
Ans. (a) : The term 'Hamartia' was introduced by
Aristotle in the 'Poetics'. Marquez's postscript to 'The General in this Labyrinth'
Hamartia is a fatal flaw leading to the downfall of a is one of the finest examples of reading archive and
tragic hero or heroine. It is also called tragic few and it using its contents. Postscript is generally a note a
is an inherent defect or shortcoming in the hero o fa paragraph added below the signature in a setter or at the
tragedy, who is in other respects a superior being end of a book, as an afterthought or to give
favoured by fortune. supplementary information.
The best example of Hamartia occurs in Sophocles Hence option (a) is correct.
'Oedipus the king'. In this Greek drama, Oedipus 31. Who among the following coined the term
unknowingly kills his own father and marries his "The Movement"?
mother. (a) F.W. Bateson (b) F.R. Leavis
Hence option (a) is correct. (c) J.D. Scott (d) I.A. Richards
29. Match List-I with List-II Ans. (c) : The term 'The Movement' was coined by J.D.
List-I List-II Scott in 1954. Scott was the literary edition of the
A. "The Lion's Skin" I. Washington Irving spectator, was is also the author of the story collection
Moonflower, Nightshade, All the House of the day and
B. "The Man who II. W. Somerset
liked Dickens" Maugham the poetry collection Mask for Mask. The term 'The
Movement' was used to describe a group of writers
C. "Rip Van Winkle" III. Stephen Crane including Philip Larkin, Kingsley Amis, Arnold Davie,
D. "The Bride comes IV. Evelyn Waugh D.J. Enright, John Wain, Elizabeth Jennings, Thomas
to Yellow Sky" and Robert Conquest. The poets in the group rejected
Choose the correct answer from the options modernism, avant-garde experimentation, romanticism
given below: and the metaphorical fireworks of poets such as Dylan
(a) A-I, B-II, C-IV, D-III Thomas. This verse was ironical, down to earth,
(b) A-III, B-IV, C-I, D-II unsentimental and rooted in a nostalgic idea of English
(c) A-II, B-IV, C-I, D-III identity.
(d) A-IV, B-I, C-III, D-II Hence, option (c) is correct.

UGC NTA NET/JRF English Paper II, 2022 (Sh.-II) 383 YCT
32. What are the points of convergence between Choose the correct answer from the options
Derrida and Barthes? given below:
A. Both believe in the endless play in language and (a) A-IV, B-III, C-II, D-I
literary texts (b) A-I, B-II, C-III, D-IV
B. Both conclude that the meaning of a text is not (c) A-IV, B-I, C-III, D-II
final. (d) A-IV, B-I, C-II, D-III
C. Both construe that language has retrospective
Ans. (d) : The correct match will be-
power.
D. They believe that no meaning is reliable. List-I (writer) List-II (work) year
E. They believe that "the relationship between A. Siddhant IV. The Lost Flamingoes 2009
words, meaning and texts are intrinsic to Dhanvant of Bombay
meaning rather than the word itself". Shanghvi
Choose the correct answer from the options B. Advaita Kala I. Almost Single 2007
given below: C. Amrit Shetty II. Love Over Coffee 2010
(a) A, B, D and E only (b) A, B, C and D only
D. Anuja Chauhan III. The Zoya Factor 2008
(c) B, C and D only (d) C, D and E only
Ans. (a) : Jacques Derrida was an Algerian French Hence option (d) is correct.
Philosopher who is most celebrated as the principal 35. Which of the following are NOT by
exponent of 'Deconstruction', a philosophical approach Rabindranath Tagore?
which he utilized in numerous texts. A. Visarjan
Roland Barthes was a French essayist who is the best B. Chandalika
known for his 1957 essay collection 'Mythologies' and C. Muktadhara
1967 essay 'The Death of the Author'. D. Parineeta
Derrida and Barthes both are most closely associated E. Punarnava
with the emergence of post structuralism.
Choose the correct answer from the options
The point of convergence between Derrida and Barthes given below:
are that they both believe in the endless play in
(a) A and B only (b) B and C only
language and literary texts, both conclude that the
meaning is reliable and also the belief that the (c) D and E only (d) A and C only
relationship between words, meanings and texts are Ans. (c) : Out of the options given, 'Parineeta' and
intrinsic to meaning rather than the word itself. 'Punarnava' are not works of Rabindranath Tagore.
Hence option (a) is correct. 'Parineeta' is a Bengali language novel written by Sarat
33. Who among the following is the author of The Chandra Chattopadhyay in 1914 which is set in
Steele Glass? Calcutta, India. The novel is a story about a poor
(a) The Earl of Surrey (b) Thomas Sackville teenage-girl, Lalita. Who is an orphan and lives with the
(c) George Gascoigne (d) Edmund Spenser family of her uncle Gurucharan.
Ans. (c) : 'The Steele Glass' is a poem by 'George 'Punarnava' is a novel by Hazari Prasad Dwivedi,
Gascoigne' published in 1576. It is a stave in verse Visarjan, Chandalika and Muktadhara are all works of
poem written in hexameter which reveals abuses and Rabindranath Tagore.
controlling personalities of both men and women in a Hence, option (c) is correct.
relationship. The speaker is a third party source that 36. Which among the following are correct?
speaks in both a critical and uplifting tone. A. George Lamming-Barbados
The poet uses hexameter which shows the common B. Ben Okri- Nigeria
theme of poets from his era's fondness for avid C. Fred D Aguiar-Australia
Gascoigne explores the controlling aspects parallel to
D. Wilson Harris -Guyana
the bible, and it can be drawn that this poem was written
from his own experiences in his past marriage. E. Zulfikar Ghose- India
Hence option (c) is correct. Choose the correct answer from the options
given below:
34. Match List I with List II
(a) A, C and D only (b) B, C and D only
List I List II
(c) C, D and E only (d) A, B and D only
A. Siddhant Dhanvant I. Almost Single
Ans. (d) : George Lamming was a Barbadian novelist,
Shanghvi
essayist and poet, who was born on 8th June 1927.
B. Advaita Kala II. Love Over Coffee Ben Okri was a Nigerian British poet, novelist, essayist
C. Amrit Shetty III. The Zoya factor and playwright, who was born on 15th March 1959.
D. Anuja Chauhan IV. The Lost Flamingoes Wilson Harris was a Guyanese writer, who was born on
of Bombay 24th March 1921.
UGC NTA NET/JRF English Paper II, 2022 (Sh.-II) 384 YCT
Fred D' Aguiar was not an Australian, but a British progresses, Browning quickly sets the tone for the rest
poet, novelist and playwright, who was born on 2nd of the poem. The poem recreates conflict that burns in
February 1960. the speaker after Cristina's loss has left him term
Zulfikar Ghose was not an Indian but a Pakistani- between love and scorn.
American novelist, poet and essayist who was born on Hence option (c) is correct.
13th March 1935. 39. Match List I with List II
Hence, option (d) is correct. List-I List-II
37. Which of the following statements are true A. Stephen Spender I. Cargoes
about Reader Response Criticism?
B. W.H. Auden II. Consider
A. It challenges the notion that the meaning is
located within the text. C. John Masefield III. Adlestrop
B. It refutes that the author is the originator of D. Edward Thomas IV. The Pylons
meaning. Choose the correct answer from the options
C. It sees the reader as a source of meaning. given below:
D. It treats the text as self sufficient. (a) A-I, B-II, C-III, D-IV
E. It appreciates the texts that are historical. (b) A-IV, B-II, C-I, D-III
Choose the correct answer from the options (c) A-II, B-I, C-IV, D-III
given below: (d) A-III, B-IV, C-I, D-II
(a) A and B only (b) A and C only Ans. (b) : The correct match will be-
(c) A, B and C only (d) B, D and E only
List-II (Author) List-II (Works) Year
Ans. (c) : Reader Response Criticism is a school of
literary theory that focuses on the reader and their A. Stephen spender IV. The Pylons 1933
experience of a literary work, in contrast to other B. W.H. Auden II. Consider 1930
schools and theories that focus attention mainly on the C. John Masefield I. Cargoes 1903
author or the content and form of the work. Reader- D. Edward Thomas III. Adlestrop 1917
Response Criticism challenges the notion that the
meaning is located within the text and refutes that the Hence option (b) is correct.
author is the originator of the meaning. It was the reader 40. Which of the following texts coins the slogan
as a source of meaning. "Vande Matram"?
Reader-Response Theory recognises the reader as an (a) Anandmath (b) Mrinalini
active agent who imparts 'near existence' to the work (c) Durgeshnandini (d) Kapalkundala
and completes its meaning through interpretation. Ans. (a) : The text which coins the slogan 'Vande
It argues that literature should be viewed as a Matram' is 'Anandmath'.
performing art in which each reader creates their own, 'Anandmath' is a novel by Bankim Chandra Chatterjee
possibly unique, text-related performance. It stands in which was first published in 1882. the Bengali novel
total opposition to the theories of formalism and New- 'Anandmath' was set against the backdrop of the 18th
Criticism. century Sannyasi Rebellion. This works plot was
Note- The correct pair of options placed in option (c) inspired by the Indians fight for independence from the
but NTA has marked all options (a), (b) & (c) as British.
correct.
Hence option (a) is correct.
38. The predominant emotion running through the
41. Who first translated the Bhagavad Gita into
poem "Cristina" by Robert Browning is that of
English?
(a) Sadness (b) Aggression
(a) H T Colebrook (b) Alexander Duff
(c) Love (d) None of the above
(c) Charles Wilkins (d) Sir William Jones
Ans. (c) : Robert Browing was an English poet and
playwright of the Victorian era. He was noted for his Ans. (c) : 'Bhagavad Gita' was translated into English
mastery of dramatic monologue and psychological by Charles Wilkins.
portraiture. 'The Bhagavad Gita' is a 700 verse Hindu scripture that
The poem 'Cristina' was published in Browning's is part of the Epic Mahabharata and was composed by
collection of poems 'Dramatic Lyrics' which was first an ancient sage named Ved Vyasa.
published in 1842. In 1785, 'The Bhagavad Gita' was first translated into
The predominant emotion running through the poem is English by Charles Wilkins and published as "
of love, as between the little and first two lines of the Bhagavad Gita or Dialogues of Krishna and Arjoon" by
poem, it was easy to figure out the general plot of the the British East India company with an introduction by
poem. In the poem, the speaker was once involved with Lord Warren Hastings, the first British Governor-
a woman named Christina, she self him and now he is General of India.
dealing with a broken near. As the first stanza Hence, option (c) is correct.
UGC NTA NET/JRF English Paper II, 2022 (Sh.-II) 385 YCT
42. Given below are two statements: In the novel, the author uses magic realism as a tool to
Statement I : The term "Negative Capability" draw the reader in and uses it as a representation of the
was coined by John Keats. Columbian culture which strongly influences the culture
Statement II : While analysing the term of the people living in Mocondo. 'Midnight Children'
"Dissociation of sensibility", T.S. Eliot (1981) is a novel by Salman Rushdie that deals with
proclaims that Hamlet is an artistic failure. India's transition from British colonialism to
In light of the above statements, choose the Independence. Rushdie uses the sequence of events
correct answer from the options given below style in magic realism in midnight children with fantasy
(a) Both Statement I and Statement II are true blended in real life.
(b) Both Statement I and Statement II are false 'Beloved' (1987) is a novel by Toni Morrison, which
(c) Statement I is true but Statement II is false uses the magical realist technique to talk about the
(d) Statement I is false but Statement II is true cruelty of slavery, to reinterpret the official history of
white slave owners and put an alternative history from
Ans. (c) : In the above given statements, statement (I) is
the perspective of the slaves.
correct and statement (II) is incorrect. Statement (I) is
correct as the term 'Negative Capability' was coined by Hence, option (b) is correct.
John Keats in 1817. It is a theory about the artists access 44. Given below are two statements:
to truth without the pressure and framework of logic on Statement I: Jerome K. Jerome's Three Men in
science. a Boat (1989) is humorous and journalistic in
Statement (II) is incorrect as T.S. Eliot proclaimed that form.
Hamlet is an artistic failure in his essay 'Hamlet and His Statement II: It is about three young men and
Problems'. In the essay, Eliot notoriously deems their dog on a holiday.
Shakespeare's most famous tragedy an 'artistic failure', In light of the above statements, choose the
maintaining that the play represents a primary problem correct answer from the options given below
and that it contains certain weaknesses as a whole. (a) Both Statement I and Statement II are true
While, Dissociation of Sensibility is a phrase used by (b) Both Statement I and Statement II are false
Eliot in the essay 'The Metaphysical Poets' (1921) to
(c) Statement I is true but Statement II is false
explain the change that occured in English poetry after
the heyday of the Metaphysical poets. (d) Statement I is false but Statement II is true
Hence, option (c) is correct. Ans. (a) : Both the above given statements are correct.
Jerome K. Jerome was an English novelist and
43. Choose the novels that use 'magic realism' as a
tool of narration: playwright whose humour won him wide following. His
work 'There Men in a Boat' (1889) is one of the most
A. the Shadow Lines
popular English travelogues. It is humorous and
B. One Hundred Years of Solitude journalist in form. It is about three young men and their
C. Midnight's Children dog on a holiday.
D. Beloved The story is about a boat trip that J. takes with his
E. Kanthapura friends George and William Samuel Harris. It is a two
Choose the correct answer from the options week boating holiday on the Thames from Kingston
given below: upon Thames to Oxford and back to Kingston.
(a) A, B and C only Hence, option (a) is correct.
(b) B, C and D only 45. Uttararamacharita by_______ is based on
(c) C, D and E only Valmiki's Ramayana.
(d) B, C, D and E only (a) Bhasa (b) Bhavabhuti
Ans. (b) : 'Magic realism' is one of the most unique (c) Bharavi (d) Kalidasa
literary movements of the last century. It is a genre of Ans. (b) : 'Uttararamacharita' is a Sanskrit play in seven
literature that depicts the real world as having an
acts in the Nataka style by Bhavabhuti which is based
undercurrent of magic or fantasy. It is a part of the
on Valmiki's Ramayana. It is a celebrated play on the
realism genre of fiction. Magical realism portrays
life of Rama after his return to Ayodhya. The play
fantastical events in an otherwise realistic tone. It brings
occupies a very high place in Sanskrit dramatic
fables, folk tales and myths into contemporary social
relevance. literature and is based on the will known story of
Uttara-kand of Ramayan, with certain changes
Out of the given options, the novels which use 'Magic
introduced by the poet.
realism' as a tool of narration are- 'One Hundred Years
of Solitude', 'Midnight Children', and 'Beloved'. Hence, option (b) is correct.
'One Hundred Years of Solitude'(1967) is a novel by 46. Which among the following are true about the
Gabriel Garcia Marquez that tells the multi-generational figures of speech?
story of the Buendia family, whose patriarch, Josh A. Figures based on sound- Paronomasia
Arcadis Buendia, founded the fictitious town of B. Figures based on construction- Zeugma
Mocondo. C. Figures based on Imagination- Irony
UGC NTA NET/JRF English Paper II, 2022 (Sh.-II) 386 YCT
D. Figures based on Association- Chiasmus Community'. It was developed from his 1995 essay
E. Figures based on indirectness- Euphemism entitled Bowling Alone: America's Declining Social
Choose the correct answer from the options Capital. It mode a claim that out to the quick of
given below: American voluntary: Americans just are not doing
(a) A, B and E only (b) A, B and C only things together anymore.
Hence, option (d) is correct.
(c) B, C and D only (d) C, D and E only
49. Match List I with List II
Ans. (a) : Paronomasia is a figure of speech based on
sound. They can be defined as a phrase intentionally List-I List-II
used to exploit the confusion between words having A. "Bricolage" I. Martin Heidegger
similar sound but different meanings. B. "Dasein" II. Sigmund Freud
Zeugma is a based on construction. It is a literary term C. "Parapraxes" III. Levi-Strauss
for using one word to modify two other words, in two D. "Polyphony" IV. Mikhail Bakhtin
different ways.
Choose the correct answer from the options
Euphemism is based on indirectness. It is used to given below:
express a mild, indirect or vague term to substitute for a (a) A-IV, B-I, C-II, D-III
harsh, blunt on offensive term.
(b) A-II, B-I, C-III, D-IV
Irony has nothing to do with imagination. It is a
(c) A-I, B-II, C-III, D-IV
rhetorical device that is used to express an intended
(d) A-III, B-I, C-II, D-IV
meaning by using language that conveys the opposite
meaning when taken literally. Ans. (d) : The correct match will be-
Chiasmus is not about association. It is a two part List-I List-II
sentence or phrase, where the second part is a mission A. Bricolage III. Levi-Strauss
image of the first. B. Dasein I. Martin Heidegger
Hence option (a) is correct. C. Parapraxes II. Sigmund Freud
47. The Voyage and Travail of Sir John D. Polyphony IV. Mikhail Bakhtin
Mandeville was written in______ Bricolage is the skill of using whatever is at hand and
(a) Queens English recombining them to create something new. Levi-
(b) Northumbrian dialect Strauss used this team in 'The Savage Mind' (1962).
(c) Midland dialect Dasein is German word for 'existence'. Heidegger
(d) Cockney uses the expression Dasein to refer to the experience
Ans. (c) : The 'Voyage and Travail of Sir John of being that is peculiar to human beings.
Mandeville' was originated in French about 1356-57 Parapraxis or a freudian slip is a verbal or memory
and was soon translated into many languages, and the mistake linked to the unconscious mind. It can also
English version appeared around 1375. The worked be called slip of the tongue. Freud claimed that
purports to tell the story of John Mandeville, a knight unconscious process reveal themselves through
from St. Albans in the south of England, who set off an paraphraxes.
a journey to the Holy Land and on to Asia and Africa in Polyphone according to Bakhtin, is a plurality of
1332. It was completed by an anonymous author. independent and unmerged voices and consciousness,
a genuine polyphony of fully valid voices.
The work was written in Midland dialect. The Midland
Hence option (d) is correct.
Dialect consists of the west midlands and the East
Midlands. The Midland Dialect is similar to what is 50. Which of the following are applicable to the
typically thought of as General American English. term 'Carnival'?
A. It became important through the work of the
Hence option (c) is correct.
Russian theorist Mikhail Bakhtin.
48. Who has used the term "bowling alone" to B. It means the way in which popular humour
describe the erosion of community ties in te subverts official authority in classical, medieval
United States? and renaissance texts and culture.
(a) Frank Furedi (b) Zygmunt Bauman C. It overturns the established hierarchy and sets
(c) Robert Bellah (d) Robert Putnam up a popular and democratic counter-culture.
Ans. (d) : The term 'bowling alone' was used by Robert D. It brings out the serous elements in literature.
Putnam to describe the erosion of community ties in the E. It is used as a critical tool for interpretation of
united states. Putnam first published it as and essay and poetry.
its purpose was to emphasize the positive effects that Choose the correct answer from the options
civic engagement can have on a community. Later in given below:
2000, he published a nonfiction book named, 'Bowling (a) A, B and C only (b) A and B only
Alone: The Collapse and Revival of American (c) B and C only (d) A and C only
UGC NTA NET/JRF English Paper II, 2022 (Sh.-II) 387 YCT
Ans. (a) : 'Carnival or Carnivalesque' is a literary genre Ans. (b) : The correct match will be-
which challenges authority, traditions and rules. Mikhail List-I (Book) List-II Author
Bakhtin, a Russian linguist and literary critic, used this
(Year)
term to characterize writing that depicts the de-
stabilization or reversal of power structures, as happens A. The Empire III. 1989 Bill Ashcroft,
in the traditional form of Carnival. Carnival means the writes Back Helen Tiffin,
way in which popular humour subverts officially Gareth
authority in classical, medieval and renaissance text and Griffiths
culture. It overturns to established hierarchy and sets up B. Nation and I. 1990 Homi K.
a popular and democratic counter-culture. Narration Bhabha
Hence option (a) is correct.
C. Culture and II. 1993 Edward Said
51. Who among the following considers a text as a
Imperialism
"site of struggle between authority and popular
culture"? D. The Twice IV. 1971 Meenakshi
(a) Roland Barthes (b) Northrop Frye Born Fiction Mukherjee
(c) Mikhail Bakhtin (d) Michel Foucault Hence, option (b) is correct.
Ans. (c) : Mikhail Bakhtin considers a text as a 'site of 54. Given below are two statements:
struggle between authority and popular culture'. Statement I : The Indian English novelists
Bakhtin was a Russian philosopher, literary critic and witnessed a warm reception of their writings in
scholar of the 20th century. It is especially known for Europe and they proliferated well in India
his work on the Russian writer Fyodor Dostoyevsky,
during the 1930's and 1940's.
Problemy Tvorchestva Dostoyevskogo, which he
published under his own name. Statement II : The two world wars added an
His major achievements include the formulation of an overall sense of gloom and civilisational crisis
innovative and radical philosophy of language as well across the globe especially in Europe.
as a comprehensive theory of the novel. In light of the above statements, choose the
Hence, option (c) is correct. correct answer from the options given below
52. Emblems by Francis Quarles is: (a) Both Statement I and Statement II are true
(a) A commentary on Homer's Illiad (b) Both Statement I and Statement II are false
(b) A commentary on a volume of Biblical (c) Statement I is true but Statement II is false
illustrations (d) Statement I is false but Statement II is true
(c) A commentary on Virgil's Aeneid Ans. (a) : Both the above given statements are true. The
(d) A commentary on Thomas Moore's Utopia Indian English novelists witnessed a warm reception of
Ans. (b) : Francis Quarles was an English poet most their writings in Europe, and they proliferated well in
famous for his emblem book entitled 'Emblems'. India during the 1930's and 1940's. R.K. Narayan, Mulk
'Emblems' is a commentary on a volume of Biblical Raj Anand and Raja Rao contributed to the growth and
illustration. It was originally published in 1635, with
popularity of Indian English fiction is the 1930s. In
grotesque illustrations engraved by William Marshall
and others. Each 'emblem' consists of a paraphrase from some cases, it is also associated with the works of
a passage of Scripture, expressed in ornate and members of the Indian Diaspora who subsequently
metaphorical language, followed by passages from the compose works in English. Also, the two world wars
'Christian Fathers', and concluding with an epigram of added an overall sense of gloom and civilization crises
four lines. In the 19th century a new edition of the across the globe, especially in Europe. World wars
Emblems was published, embellished with new made a magnificent impact on society. The impact
illustrations by Bennett and Roars. developed a new approach of art, literature, philosophy
Hence, option (b) is correct. and religion.
53. Match List with List II Hence, option (a) is correct.
List I List II 55. Given below are two statements:
A. The Emprie Writes Back I. 1990 Statement I : The theatre was at a greater
B. Nation and Narration II. 1993 popularity in Eighteenth century England.
C. Culture and Imperialism III. 1989 Statement II: Theatre had court patronage in
D. The Twice Born Fiction IV. 1971 and around London.
Choose the correct answer from the options In light of the above statements, choose the
given below: correct answer from the options given below
(a) A-I, B-II, C-III, D-IV (a) Both Statement I and Statement II are true
(b) A-III, B-I, C-II, D-IV (b) Both Statement I and Statement II are false
(c) A-IV, B-III, C-II, D-I (c) Statement I is true but Statement II is false
(d) A-II, B-III, C-I, D-IV (d) Statement I is false but Statement II is true
UGC NTA NET/JRF English Paper II, 2022 (Sh.-II) 388 YCT
Ans. (d) : In the above given statements, Statement I is Choose the correct answer from the options
false and Statement II is true. given below:
Statement II is true as the Theatres had court patronage (a) A-I, B-IV, C-III, D-II
in and around London. (b) A-II, B-I, C-IV, D-III
In Theatre, patronage is the support, encouragements, (c) A-IV, B-III, C-II, D-I
privilege or financial aid that an organisation or (d) A-III, B-IV, C-I, D-II
individual bestows to another. Ans. (b) : The correct match will be-
Statement I is false as the cheater was not at a greater List-I (Chts.) List-II (Works) Author
popularity in eighteenth century English. The ban on A. Doll II. The Ben Jonson
theatre in 1774 was part of a larger program of Common Alchemist
economic dissociation from Distain to promote B. Malvolio I. Twelfth William
American production and trade while hosting Britains. Night Shakespeare
Hence option (d) is correct. C. Mortimer IV. Edward II Christopher
56. Match List I with List II Marlowe
List I List II D. Bosola III. The Duchess John Webster
A. Karl Marx I. Madness and of Malfi
civilization Hence option (b) is correct.
B. Levi-Strauss II. Being and 58. Who among the following is exclusively
Nothingness associated with diary writing in English?
(a) Samuel Pepys (b) Samuel Johnson
C. Michel Foucault III. The German
(c) Ben Jonson (d) Samuel Richardson
Ideology
Ans. (a) : 'Samuel Pepys' is exclusively associated with
D. Jean Paul Sartre IV. The Elementary diary writhing in English.
Structures of Samuel Pepys was an English diarist and naval
Kinship administrator who was celebrated for his 'Diary', which
Choose the correct answer from the options was first published in 1825.
given below: His 'Diary' gives a fascinating picture of the official and
(a) A-III, B-II, C-I, D-IV upper class life of Restoration London from 1st January
(b) A-IV, B-III, C-I, D-II 1660 to 31st May 1669. The 'Diary' effuse a richly
(c) A-II, B-III, C-I, D-IV detailed account of some of the most turbulent events of
the Nation's history, including the coronation of the
(d) A-I, B-II, C-III, D-IV king Charles II. The Great Plague and the Great Fire of
Ans. (*) : The correct match will be- London.
List-I (Author) List-II (Book) Hence, option (a) is correct.
A. Karl Marx III. The German 59. Brij V. Lal's Rama's Banishment deals with the
Ideology (1846) history of Fiji of a time period of ______.
B. Levi-Strauss IV. The Elementary (a) 50 years (b) 100 years
Structures of Kinship (c) 150 years (d) 200 years
(1949) Ans. (c) : Brij V. Lal's 'Rama's Banishment deals with
the history of fiji of a time period of 150 years. Brij V.
C. Michel Foucault I. Madness and Lal was an Indo-Fijian historian who wrote about the
civilization (1961) Pacific region and the Indian indenture system.
D. Jean Paul Sartre II. Being and 'Rama's Banishment : A Centenary Tribute to the
Nothingness (1943) Fiji Indians, 1879-1979' was published in 1979. In his
Note- The correct match will be- A-III, B-IV, C-I, D- work, Lal analyzes how the Indo-Fijians have been
II, which is not given in any option. scapegoated by indigenous politicians representing
them as a controlling force in the country, while Indo-
NTA has dropped this question.
Fijian land leases have been revoked on racial grounds
57. Match List with List II and democratic representatives ousted by force.
List I List II Hence, option (c) is correct.
A. Doll Common I. Twelfth Night 60. Women of Palestine (1982) and Indian Women
B. Malvolio II. The Alchemist in Struggle (1980) deal with:
(a) women's religious and familial issues
C. Mortimer III. The Duchess of
(b) female militancy and political involvement
Malfi
(c) women's struggle at their work place
D. Bosola IV. Edward II (d) women's reproductive rights
UGC NTA NET/JRF English Paper II, 2022 (Sh.-II) 389 YCT
Ans. (b) : 'Women of Palestine' (1982) and 'Indian Ans. (b) : The correct statements among all are B, C
Women in Struggle' (1980) deal with female militancy and D only. Dramatic Monologue was fully exposited
and political involvement. (used in large numbers) by the nineteenth century poets
'We Shall Return : Women of Palestine' is a 1982 book eg; Tennyson's "Ulysses" (1842), Robert Browing's My
by 'Ingela Bendt' and Jim Downing' which describes the Last Duchess, 'Porphyria's Lovers' etc. Other writers
daily life of Palestinian women in Lebanese refugee from 20th century also used this poetic form greatly.
camps and determines how exile and revolution have Some of term are : His (Hilda Doolittle), Amy Lowell,
altered their traditional role in the family. Robert Frost, EA. Robinson, Ezra Pound, Robert
'We will Smash this Privon : Indian Women in struggle' Lowell. The best known modern instance is T.S. Eliot's
is a 1980 book by 'Gail Omvedt', who was an American "The Love Song of J. Alfred Prufrock" (1915). Robert
born Indian sociologist and human rights activist. Langbaum's 'The Poetry of Experience' outlines a
Hence option (b) is correct. discussion on dramatic monologue.
61. Identify the Postcolonial critics who used the Dramatic Monologue is a literary from where the writer
ideas of Lacan, Foucault and Derrida while takes on the voice of a character and speaks through
critiquing 'Euro-centrism'? them. Although it is used in theater and prose yet the
A. Homi Bhabha term most presently used in poetry; it is a stanza form.
B. Gayatri Chakravarty Spivak Hence, the correct answer is option (b).
C. Abdul Jan Mohammad 63. Which one is correctly matched?
D. Edward Said (a) Heathcliff ↔ Mansfield Park
E. Amie Cesaire (b) Maggie Tulliver ↔ The Mill on the Floss
Choose the correct answer from the options (c) Josiah Bounderby ↔Wuthering Height
given below:
(d) Fanny Price ↔ Hard Times
(a) A, B, C and D only (b) A, B and C only
Ans. (b) :
(c) B, C, D and E only (d) C, D and E only
Ans. (a) : The Postcolonial critics who used the ideas Character Work Writer
of Lacan, Foucault and Derrida while critiquing 'Euro- Heathclif Wuthering Heights Emily Bronte
centrism' were - Homi Bhabha, Gayatri Chakravarty (1847)
Spivak, Abdul Jan Mohammad and Edward Said. Maggie The Mill on the George Eliot
'Eurocentrism' is the discourse that emphasizes Floss (1860)
European concerns, culture and values at the expense of Josiah Hard Times(1854) Charles
others. It uses Europe as a cultural rather that a Bounderby Dickens
cartographical expression, which incorporates the so-
called 'West', the Western Europe and North America. It Fanny Price Mansfield Park Jane Austen
investigates the relation between humans and the (1812)
natural world in literature, and deals with how The protagonist of "The Mill on the Floss", Maggie
environmental issues, cultural issues concerning the Tulliver is a cleaver and impetuses child. This novel
environment and attitudes towards nature and presented is about the relationship between a brother and sister,
and analyzed. Tom and Maggie Tulliver, who live in a mill. Being
Hence option (a) is correct. the most autobiographical novel of George Eliot, it
62. Identify the correct ones among the following: fosses on young woman's (maggie) struggle for
growth and independence against the restraints of
A. The dramatic monologue ensures the
reciprocal dialogue of the narrator. small country life. Hence, the correct answer is
option (b).
B. The nineteenth century poets fully exploited the
poetic form of dramatic monologue. 64. The aboriginal Australian poet, Oodgeroo
C. The Poetry of Experience by Robert Langbaum Noonuccal, in her poem "We are going"
outlines a discussion on dramatic monologue. examines the impact of colonial conquest on
D. The linguistic pragmatics make the narcissistic aboriginal Australians with reference to:
speaker of dramatic monologue speak A. Loss of 'bora ground'
exclusively. B. Loss of aboriginal identity
E. The speaker and the listener in the dramatic C. Loss of new education
monologues of Robert Browning share the D. Loss of primitive culture
same pedestal of communication. E. None of the above
Choose the correct answer from the options Choose the correct answer from the options
given below: given below:
(a) A, B and C only (b) B, C and D only (a) A, B and D only (b) C, D and E only
(c) C, D and E only (d) B, D and E only (c) A, B and C only (d) A, C and D only
UGC NTA NET/JRF English Paper II, 2022 (Sh.-II) 390 YCT
Ans. (a) : The aboriginal Australian poet, Oodgeroo Ans. (c) : 'Bel Patra' (1987) is a short story by the
Noonuccal, in her poem "We are going" (1964) Booker Prize winner Geetanjali Shree. It was her first
examines the impact of colonial conquest on aboriginal story, published in literary megazin "Hans".
Australians with reference of Loos of "Bara ground", Khalijagaj Hamar Shahar Us Baras (1998), Mai (2000)
"Loss of aboriginal identity" and "Loss of primitive are novels written by Shree 'Mai' was shortlisted for
culture". It is a powerful poem about the struggles of Crossword Booker Award in 2001 will her novel 'Ret
Aboriginal Australians in the face of British Samadhi' (Tomb of Sand [English translation by Dasiy
Colonialism. By this poem, Oodgeroo Noonuccal rockwell ] ) (2018) won International Booker Prize in
highlights the struggle of Aboriginal Australian tribes as 2022.
they attempt to preserve their land, culture, and history. Hence, the correct answer is option (c).
Hence, the correct answer is option (a).
68. Which of these books are written by Nirad C
65. Gaiutra Bahadur is a ________ writer. Chaudhuri?
(a) Mauritian (b) Fijian A. The Continent of Circe
(c) Trinidadian (d) Guyanese B. Principal Upanishads
Ans. (d) : Gaiutra Bahadur is a Guyanese American C. A Passage to England
writer best known for 'Coolie Women: The Odyssey of D. Our New Rulers
Indenture,' which was shortlisted for the Orwell Prize in E. The Autobiography of an Unknown Indian
2014. An essayist, critic and journalist, Gaiutra Bahadur
was born in Guyana and emigrated as a child (about six Choose the correct answer from the options
years old) to Jersey City, New Jersey, where she given below:
currently lives and teaches writing and Journalism as an (a) B, C and D only (b) C, D and E only
associate professor in the Department of Arts, Culture (c) A, C and E only (d) A, B and D only
and Media at Rutgers University in Newark. Ans. (c) : Among the given books, "The Continent of
Hence, the correct answer is option (d). Circe" (1965), A Passage to England (1959), and "The
66. Identify the film/films that were based on Autobiography of an Unknown Indian" (1951) are
Booker winning novels:- written by Nirad C. Chaudhuri. He was a Bengali
A. Life of Pi (Indian) author and scholar who was opposed to the
B. The Remains of the Day withdrawal of British colonial rule from Indian
Subcontinent and Subsequent rejection of Western
C. Milkman
culture in independent India.
D. The Sellout
'The Continent of Circe' deals with the Indian Society
E. The Inheritance of Loss
from a Socio-psychological perspective, commenting on
Choose the correct answer from the options Hindu society from prehistory to modern times.
given below:
"A Passage to England" was an account of brief, five
(a) B and E only (b) A and B only weeks first visit to England. "The Autobiography of an
(c) C and D only (d) B and D only Unknown Indian" is an autobiography of Nirad C.
Ans. (b) : The films that were based on Booker winning Chaudhuri. "The Principal Upanishads" (1953) is a
novels are "Life of Pi" and "The Remains of the Day" book written by Sarvepalli Radhakrishnan.
with same name. Yann Martel's "Life of Pi" (2001) tells Hence, the correct answer is option (c).
the magical story of young Indian Pi Patel, who
survives harrowing shipwreck and months in a lifeboat 69. Hermann Hesse's Siddhartha was originally
with a large Bengal tiger named Richard Parker. written in________.
Martel's novel was adopted as a 2012 film directed by (a) French (b) Russian
AngLee. (c) English (d) German
Kazuo Ishiguro's "The Remains of the Day" (1989) won Ans. (d) : Hermann Hesse's "Siddhartha" (in German,
the Booker Prize in 1989. It was adopted as a film with Siddhartha : Eine Indische Dichtung) was originally
the same name in 1993 starring Anthomy Hopkins and written in German and published in 1922. It was
Emma Thompson. published in the United States in 1951 and became
Anna Burns' Milkman (2018), Paul Beatty's the sellout influential during 1960s. It deals with the search of
(2015), Kiran Desai's "The Inheritance of Loss" (2006) realization by a young man Siddharatha.
are Booker winning novels but they have not been Hence, the correct answer is option (d).
adopted in Films. 70. Given below are two statements:
Hence, the correct answer is option (b). Statement I: According to Michel Foucault, the
67. Which among the following is a short story by French Revolution created grounds for the
the Booker Prize winner Geetanjali Shree: birth of 'the Clinic'.
(a) Khali Jagah Statement II: Foucault mentions that the
(b) Hamara Shadar Us Baras doctors started caring for the body of the
(c) Bel Patra patients the way priests cared for the soul of
(d) Mai the sinners.
UGC NTA NET/JRF English Paper II, 2022 (Sh.-II) 391 YCT
In light of the above statements, choose the Introduced by W.K. Wimsatt, and Monroe C. Beardsley
correct answer from the options given below in "The Verbal Icon" (1954), the approach was a
(a) Both Statement I and Statement II are true reaction to the popular belief that to know what the
(b) Both Statement I and Statement II are false author intended - what he had in mind at the time of
writing - was to know the correct interpretation of the
(c) Statement I is true but Statement II is false
work. In brief, 'Intentional Fallacy' is an error of a critic
(d) Statement I is false but Statement II is true while evaluating an author's work on the basis of
Ans. (a) : The given statements regarding French historicity or his biographical details instead of his
Revolution are absolutely correct according to Michel intention or the text of poetry.
Foucault. Hence, the correct answer is option (a).
Foucault's 'The Birth of the Clinic : An Archaeology of 73. Which of the following statements are true
Medical Perception' (French, Naissance de la clinigue : about Ethnographic Research Method?
Une archeology due regard medical), (1963). presents A. It enables the exploration of the consumption
the development of the clinic, the teaching hospital, as a of literature within defined social and historical
medical institution, identities and describes the concept settings.
of "the medical gare", and epistemic re-organisation of B. Janice A. Radway's Reading the Romance
the research structure of medicine in the production of (1984) is one of the first examples of the use of
medical knowledge, at the end of 18th century. ethnographic research methodology.
In the last of 18th century, when the French (1789-1799) C. It is perceived as an act of uncovering an
and American (1775-1783) revolutions took place, at objective reality.
that time scientific discourse established medical use D. Reading National Geographic (1993) by
and importance of clinic. Catherine A. Lutz and Jane L. Collins makes
The French Revolution created grounds for the birth of effective use of ethnographic research method
the clinic. The Doctors started caring for the body of the to find out the role of the magazine National
patients the way priests cured for the soul of the sinners. Geographic in moulding Americans awareness
Hence, the correct answer is option (a). of the world beyond the United States.
71. Who among the following has used lines from E. It focuses more on objective data than
the Shakespearean play Othello to critique subjective interpretation of the materials.
racism in one of his poems? Choose the correct answer from the options
(a) Edward Braithwaite (b) Franz Fanon given below:
(c) Derek Alton Walcott (d) Ngugi wa Thong'o (a) A, B and E only
Ans. (c) : Derek Alton Walcott has used lines from the (b) A and C only
Shakespearean play 'Othello' to Critique racism in his (c) A, B and C only
poem 'Goats and Monkeys' that justifies a black man in (d) A, B, C and D only
a world where everyone looks down on him. This poem Ans. (d) : Ethnography is a research method used to
portrays many notions of racism, sex, savagery and learn about the lives of other. It helps us to understand
jealousy. Derek Walcott believes that even today, how and why people behave differently in various
modern society has not changed significantly enough to societies or culture. When we wish to understand a
accept others as Desdemona's lover. The poem begins particular social phenomenon ethnographic research can
with a quotation from "Othello" where Iago warns be a useful tool.
Brabant senator of vanice, that has been betrayed as It enables the exploration of consumption of literature
Othello, the moor is making love to Brabantio's fair and within defined social and historical settings. Janice to
beautiful daughter Desdemona. Radway's "Reading the Romance" 1984 is one of the
Hence the correct answer is option (c). first examples of the use of ethnographic research
72. Who among the following is associated with the methodology. It is perceived as an act of uncovering an
term 'Intentional Fallacy' in literary criticism? objective reality.
(a) W.K. Wimsatt (b) Rolland Barthes 'Reading National Geographic' (1993) by Catherine A.
(c) J. Hillis Miller (d) John Keats Lutz and Jane L. Collins makes effective use of
Ans. (a) : International fallacy, (A false idea that many ethnographic research method to find out the role of the
people believe is true) term used in 20th century in new megrim National geographic in molding American
criticism to describe the problem inherent in trying to awareness of the world beyond the United States.
judge a work of art by assuming the intent or purpose of Hence, the statements A, B, C & D are absolutely
the artist who created it. correct.

UGC NTA NET/JRF English Paper II, 2022 (Sh.-II) 392 YCT
74. Identify the correct one from the following: Choose the correct answer from the options
A. Bharti Mukherjee's novels display split in the given below:
diasporic subjects. (a) A-I, B-IV, C-III, D-II
B. Rohinton Mistry belongs to the community of (b) A-III, B-I, C-IV, D-II
Parsies that fled to India from Persia to escape (c) A-IV, B-III, C-II, D-I
Islamic persecution. (d) A-II, B-III, C-IV, D-I
C. Sujata Bhat recalls home as a nostalgic
Ans. (b) :
memory and longs intensely for it.
D. Farrukh Dhondy connects Delhi and New York A "There is no art to find III. Macbeth
to discuss the diasporic experiences. mind's construction in the
E. Sharat Chandra in his poem, "In the Third face".
Country". Wishes to die in India B "Time out of joint". I. Hamlet
Choose the correct answer from the options C "The better part of valour IV Twelfth
given below: is discretion". . Night
(a) A, B and D only (b) A, B and C only D "My Kingdom for a II Richard III
(c) A, D and E only (d) B, C and E only horse".
Ans. (b) : Indian born American novelist and short Macbeth, Hamlet, Twelfth Night and Richard III are
story writer, Bharati Mukharjee's novels display split in written by Shakespeare All of them are published in
the diasporic subjects. Her novels 'The Tiger's Daughter' First Folio in 1623 except "Hamlet" which was
(1972), Wife (1975) are based on diasporic subjects." published in quarto edition 1603.
Indian born Canadian writer, Rohinton Mistry, belongs Hence, the correct answer is option (b).
to the community of Parsies that fled to India from
Persia to escape Islamic persecution. He is also an 77. Hudibras of Samuel Butler reflects on the
Indian diasporic writer. Even though he is settled in revolt against:
Canada still we can see the reflection of Mumbai (his (a) Puritanism (b) Hellenism
birth place) in all his writings. Some of his novels are (c) Humanism (d) Anglicanism
"Such a Long Journey" (1991). A Fine Balance (1995), Ans. (a) : Samuel Butler's satirical poem "Hudibras"
Family Matters (2002). was published in three parts in 1663, 1664 and 1678.
Born in 1956 in Pune (India), Sujata Bhatt emigrated Butler saw fanaticism, pretentiousness, pedantry, and
with her family to the United states in 1968. She recalls hypocrisy in militant Puritanism which inspired this
home as a nostalgic memory and longs intensely for it. work. It reflects on the revolt against Puritanism.
Statement 'D' and 'E' are not correct regarding Farrukh Hence, the correct answer is option (a).
Dhondy and Sarat Chandra respectively. 78. In Kannada literature, 'Vachana Movement'
Hence, the correct answer is option (b). addressed
75. Who among the following was a major (a) progressive values (b) vedic themes
advocate of Oral history? (c) vedantic thoughts (d) upanishadic ideas
(a) Sylvia Plath (b) Esther Greenwood Ans. (a) : In Kannada literature, "Vachana Movement"
(c) Paul Thompson (d) Carolyn Steedman addressed progressive values. It is social reform
Ans. (c) : Paul Thompson regarded as pioneer in social movement of Karnataka which started in twelfth
science research, particularly due to the development of century in kalyana of Kalaburage division. It was
life stories and oral history within sociology and social launched to wage a war against untouchability.
history, was a major advocate of oral history. Born in Hence, the correct answer is option (a).
1935, Thompson founded the ''Oral History Society"
and the journal ''Oral History". 79. Derek Alton Walcott in the poem "The Sea is
History" makes a parallel between
Sylvia Plath (Pseudonym - Victoria Lucas) as an
American poet, novelist and short story writer. She was (a) Evolution of mankind and nature
well known for her 'Confessional Poetries.' Some (b) Evolution of human history and religion
notable works are- "Daddy", "The Bell Jar", "Three (c) Evolution of Paganism and Christianity
Women" etc. (d) Evolution of Christianity and slavery of the
Hence, the correct answer is option (c). Blacks
76. Match List I with List II Ans. (d) : Derek Alton Walcott, West Indian poet and
List I List II playwright, noted for works that explore the Caribbean
cultural experience in the poem. "The Sea is History"
A. "There is no art to find I. Hamlet
makes a parallel between evolution of Christianity and
mind's construction in
slavery of the Blacks, published in The Paris Review in
the face".
1978, and in "The Star-Apple kingdom" in 1979, "The
B. "Time out of joint". II. Richard III Sea is History" deals with the idea of the Bible as a
C. "The better part of III. Macbeth historical document and the journey of African slaves to
valour is discretion". the Caribbean has parallels with the journey of the
D. "My Kingdom for a IV. Twelfth Night Israelites to the promised Land.
horse". Hence, the correct answer is option (d).
UGC NTA NET/JRF English Paper II, 2022 (Sh.-II) 393 YCT
80. Which of the following is NOT a kind of 'sign' The above passage considered to be the death-
as suggested by Charles Sanders Pierce? knell of the neo-classical criticism is attributed
(a) Icon (b) Index to______
(c) Symbol (d) Visual (a) John Dryden (b) Alexender Pope
Ans. (d) : The American philosopher, logician, and (c) Samuel Johnson (d) Joseph Addison
scientist, Charles Sanders coined the term "Semiotics" Ans. (c) : The given passage considered to be the death-
and introduced several of its fundamental concepts. knell of the New-classical Criticism is attributed to
Peirce declared that anything can be a sign-words, Samuel Johnson. In the periodical "The Rambler"
images odors, objects etc. Johnson wrote these lines regarding neo-classical
criticism.
"The Rambler" (1750-1752), in short is of fundamental
importance in any estimate of Johnson's approach to
literature itself though shot through with mournful
humor, it was written to instruct and chasten.
Hence, the correct answer is option (c).
83. Identify the correct pairs.
A. Kalidasa- Amoghvarsha
B. Bhavabhuti- Uttararamcharita
C. Bhasa-Urubhanga
The semiotic fried:
D. Rajashekhara- Rajatarangini
• A representamen, the form of the sign.
E. Somadeva- Kathasaritsagara
• An interpretant, the sense made of the sign in the
Choose the correct answer from the options
mind of the observer; and
given below:
• An object that to which the sign refers. further
(a) A, C and D only (b) C, D and E only
classification of the semiotic triad has been given in
the figure. None of them contains "Visual" as a kind (c) B, C and E only (d) B, D and E only
of sign suggested by charter Sanders Pierce. Ans. (c) : The correct pairs of the given writers and
Hence, the correct answer is option (d). their work are B, C and E only. Amoghvarsha is not
81. Which of the following works is set in the written by Kalidas.
backdrop of the religious persecution in Uttararamcharita is a sanskrit play in seven acts in the
Mexico? Nataka style by Bhavabhuti.
(a) The Power and the Glory Urubhanga is a Sanskrit play written by Bhasa in the 2nd
(b) For Whom the Bell Tolls or 3rd century CE. based on the well-known epic, the
(c) In Our Time Mahabharata, by Vyasa. It focuses on the story of the
(d) All of the above character during and after his fight with "Bhim".
Ans. (a) : Graham Greene's "The Power and the Glory" Rajtarangani (River of Kings) (1148) is a historical
(1940) is set in the backdrop of the religious persecution chronicle of early India written by the Kashmiri ..........
in Mexico. It was also published as the Labyrinthine "Kathana" not by Rajashekhara.
ways; adopted as the film "The Fugitive (1947). It has a The " Kathasaritsagara" is a famous 11 century
more directly Catholic theme: The desperate collection of India legends, fairy tales and .... tales as
wanderings of a priest who is hunted down in rural retold in Sanskrit by the Shaivite Somadeva.
Mexico at a time when the church is outlawed there. Hence, the correct answer is (c).
Despite the constant threat of death at the hands of a 84. Abt Vogler is authored by
revolutionary government the weak and alcoholic priest (a) Matthew Arnold (b) Robert Browning
attempts to fulfill his priestly duties. Hence, the correct
(c) A.L. Tennyson (d) None of the above
answer is option (a).
Ans. (b) : Abt Vogler (1864) is authored by English
82. "It ought to be the first endeavour of a writer
poet and playwright and one of the proponents of
to distinguish nature from custom, or that
dramatic monologues Robert Browning.
which is established because it is right from
that which is right only because it is Many of Browning's dramatic monologues, including
established; that he may neither violate "Abt Vogler" are written in the voice of a real historical
essential principles by a desire of novelty, nor figure. Vogler was a composer and musical innovator;
debar himself from the attainment of beauties in this poem Browning imagines him as he ages
within his view by a needless fear of breaking becoming more frail, and meditating the meaning and
rules which no literary dictator had authority value of his life.
to enact". Hence, the correct answer is option (b).
UGC NTA NET/JRF English Paper II, 2022 (Sh.-II) 394 YCT
85. Which of the following statements hold true 87. Which of the following are the premises of Post
with respect to Alexander Pope's "Essay on colonial Criticism?
Criticism"? A. It rejects the claims of universalism made in
A. It is "an inquiry into the nature and value of the canonical Western literature.
poetry".
B. It foregrounds the questions of cultural
B. It presents "a series of generalizations about
good taste". difference and diversity as represented in
C. It explores the challenges of impartial and just literary texts.
criticism. C. It acts on the principles of peaceful co-
D. It underlines the traits of "the good critic". existence.
E. it critically reflects on Plato's rejection of D. It celebrates "hybridity" and "Cultural
poetry. polyvalency".
Choose the correct answer from the options E. It resists any attempt at homogenization based
given below: on race, class and nationality.
(a) A and B only (b) B and C only Choose the correct answer from the options
(c) A, B, C and E only (d) B, C and D Only given below:
Ans. (d) : Alexander Pope a translator, poet, wit, and (a) A, B and D only (b) A, B, D and E only
satirist (1688-London) wrote "An Essay on Criticism", a
(c) B, C and D only (d) C and D only
didactic poem in heroic couplets in 1711 when he was
about 22 years old. Ans. (b) : Post Colonial criticism frequently focuses on
The first of the poem's three sections arguments that relationship between colonizers and colonized people in
"good taste" derives from nature and that critics should literary texts. It also analyses construction of cultural
imitate the ancient rules established by classical writers. identity, particularly the concept of "hybridity".
In Second part pope stressed the importance of Post Colonial Criticism encompasses rejection of the
onomatopoeia in prosody and suggested that the claims of universalism made in the colonial Western
movement of second and meter should represent the literature. It foregrounds the questions of cultural
actions they carry. difference and diversity as represented in literary text. It
The final section discusses the characteristics of a "good celebrates "hybridity" and "cultural polyvalency". It
critic" and concludes with a short history of literary
criticism and a catalogue of famous critics. resists any attempt at homogenization based on race,
Hence, the statements B, C and D are absolutely correct class and nationality. Hence, the correct answer is
in the reference of Pope's "An Essay on Criticism". So, option (b).
the correct answer is option (d). 88. Find the chronological order of publication of
86. Match List I with List II the given works:
List I List II A. Mythologies
A. Acharnians I. Government by women B. Of Grammatology
B. Clouds II. Attack on parties involved C. Culture and Society
in war D. Blindness and Insight
C. Lysistrata III. Criticism of the new 'spirit E. The Location of Culture
of philosophical inquiry' Choose the correct answer from the options
D. Wasps IV. An attack on demagogues given below
Choose the correct answer from the options (a) A, B, C, D, E (b) A, C, B, D, E
given below: (c) D, B, C, A, E (d) B, C, D, A, E
(a) A-I, B-III, C-II, D-IV Ans. (b) : Chronological order of publication of the
(b) A-II, B-IV, C-I, D- III given works is Roland Barthes "Mythologies" (1957),
(c) A-III, B-I, C-IV, D-II Raymond Williams "Culture and Society" (1958),
(d) A-II, B-III, C-I, D-IV Jacques Derrida's Of Grammatology (1967), Paul de
Ans. (d) : Man's "Blindness and Insight" (1971), and Homi K.
A. Acharnians II. Attack on parties Bhabha's "The Location of Culture" (1994).
involved in war "Mythologies" (1957) provides readings of
B. Clouds III. Criticism of the new contemporary cultural icons and has become a standard
'spirit of philosophical text in the academic field of cultural studies.
inquiry' "Culture and Society" (1958) explores how the notion
C. Lysistrata I. Government by women of culture developed in great Britain from the eighteenth
D. Wasps IV. An attack on through the twentieth centuries.
demagogues In "Of Grammatology" French philosopher Jacques
Hence, the correct answer is option (d). Derrida originates the idea of deconstruction.
UGC NTA NET/JRF English Paper II, 2022 (Sh.-II) 395 YCT
De Man examines several critics in "Blindness and Ans. (a) : Derrida's statements regarding aesthetic are
Insight" and discovers a gap between their statements absolutely correct. Derrida mentions that an aesthetic
about the nature of literature and the results of their discourse always involves values and interests,
practical criticism in their writings. independent from "a pure and neutral, aesthetic realm."
"The Location of Culture" deals with the conceptual He believes that the outside influence in assessment of
imperative and the political consistency of a aesthetics always enters as and when philosophers and
postcolonial intellectual projects. historians point out the element of "truth". Hence, the
Hence, the correct answer is option (b). A, C, B, D, E. correct answer is option (a).
89. Match List with List II
Read the following passage and answer the
List I List II questions that follow:-
A. Perumal I. Adivasi Will Not Such is the matter of imaginative or artistic
Murugan Dance literature- this transcript not of mere fact, but of
B. Hansda II. The Weave of My fact in its infinite variety, as modified by human
Sowvendra Life preference in all its infinitely varied forms. It will
Shekhar be good literary art not because it is brilliant or
C. Baby Kamble III. One Part Woman sober, or rich, or impulsive, or severe, but just in
D. Urmila Pawar IV. The Prisons We proportion as its representation of that sense, that
Broke soul fact, is true, verse being only one department
Choose the correct answer from the options of such literature, and imaginative prose, it may be
given below: thought, being the special art of the modern world.
(a) A-I, B-II, C-III, D-IV That imaginative prose should be the special and
(b) A-III, B-I, C-IV, D-II opportune art of the modern world results from
(c) A-III, B-I, C-II, D-IV two important facts about the later: first, the
(d) A-IV, B-II, C-III, D-I chaotic variety and complexity of its interests,
Ans. (b) : The correct match of the authors with their making the intellectual issue, the really master
works is: A-III, B-I, C-IV, D-II currents of the present time incalculable- a
A. Perumal Murugan III. One Part Woman condition of mind little susceptible of the restraint
(2010) proper to verse form, so that the most
B. Hansda I. Adivasi Will Not characteristic verse of the nineteenth century has
Sowvendra Dance (2015) been lawless verse; and secondly, an all pervading
Shekhar naturalism, a curiosity about everything whatever
C. Baby Kamble IV. The Prisons We as it really is, involving a certain humility of
Broke (2009) attitude, cognate to what must, after all be the less
D. Urmila Pawar II. The Weave of My ambitious form of literature. And prose thus
Life (2008) asserting itself as the special and privileged
Hence, the correct answer is option (b). artistic faculty of the present day, will be, however
90. Given below are two statements: critics may try to narrow its scope, as varied in its
excellence as humanity itself reflecting on the
Statement I : Derrida mentions that an
aesthetic discourse always involves values and facts of its latest experience- an instrument of
interests, independent from "a pure and many stops, meditative, observant, descriptive,
neutral aesthetic realm". eloquent, analytic, plaintive, fervid.
Statement II : Derrida believed that the outside 91. Artistic literature is the representation of:
influence in assessment of aesthetics always (a) facts enhanced by creative illusions
enters as and when philosophers and historians (b) facts transformed by human predilection in an
point out the element of "truth". array of forms
In light of the above statements, choose the (c) facts arranged by political reflection
correct answer from the options given below
(d) complex and natural instincts of a poet
(a) Both Statement I and Statement II are correct
(b) Both Statement I and Statement II are Ans. (b) : While discussing about style in literature,
incorrect Walter Pater writes in "Appreciations with an Essay on
(c) Statement I is correct but Statement II is Style" that artistic literature is the representation of facts
incorrect transformed by human predilection (liking) in an array
(d) Statement I is incorrect but Statement II is of forms.
correct Hence, the correct answer is option (b).
UGC NTA NET/JRF English Paper II, 2022 (Sh.-II) 396 YCT
92. Which of the following is closest to what the Of midnight vision, gathering up thy skirts;
author means by "less ambitious form of By night star-veiling and by day
literature"? Darkening the light and blotting out the sun;
(a) literature responsive to heightened state of Go thou, my incense, upward from this hearth,
human perception And ask the gods to pardon this clear flame.
(b) imaginative literature Henry David Thoreau
(c) artistic literature 96. The poem deals with_______.
(d) poetry without form and diction (a) fire in the forest (b) fire in the city
Ans. (a) : "Less ambitious form literature" in the given (c) fire on the ship (d) fire in the village
excepts by Pater closely means "literature responsive to Ans. (d) : Henry David thoreau's poem "Light-Winged
heightened state of human perception" Smoke" deals with "fire in the village". As the mention
93. In the above passage, Walter Horatio Pater's of lark circling their nest; circling the hamlets where
they live, indicates that bigger area has been burnt in
statement, "Imaginative prose should be
fire.
special" implies;
Hence the correct answer is option (d).
(a) abstract language
97. Why does the poet seek pardon from the Gods?
(b) environmental crisis
(a) The singing birds disrupt the peace of the
(c) intellectual complexities
forest.
(d) metaphorical functions (b) The earth is full of peace and tranquility.
Ans. (c) : The statement from the above passage (c) the speaker admits sin of setting the fire.
"Imaginative prose should be special" implies (d) The narrator is over enthusiastic.
"intellectual complexities".
Ans. (c) : The poet Henry David Thoreau seeks pardon
94. According to the author, prose should be: from the Gods because some how he finds himself
(a) Socio-Political Guilty of burning down the hamlet (a large area of
(b) Subjective forest, hamlet or animal habitant etc.)
(c) as varied as human experience Hence, the correct answer is option (c).
(d) as visual as other art forms 98. In the first line of the poem, "Icarian bird"
Ans. (c) : According to the author, prose should be as connotes_______.
varied as human experience. As mentioned in the (a) short of ambition (b) pride
passage, prose should be of infinite variety, as modified (c) destruction (d) waxen wing
by human preference in all its infinitely varied form; as Ans. (*) : The question has been dropped by NTA due
human experiences differently in different to ambiguity in the given options.
circumstances. 99. Which figure of speech is implicit in "light
Hence, the correct answer is option (c). winged smoke!"?
95. Which of the these expressions closely (a) Apostrophe (b) Simile
represent the meaning of 'fervid'? (c) Oxymoron (d) Hyperbole
(a) feeling nostalgic Ans. (a) : Light-winged smoke ! Icarian bird, ----"
(b) portraying feelings that are too strong contains a figure of speech 'Apostrophe'. Apostrophe is
(c) riding strong feelings a figure of speech in which a speaker directly addresses
someone in which a speaker directly addresses someone
(d) expressing humility of attitude
or something that is not present or cannot respond in
Ans. (b) : "Fervid" means "believing or feeling reality. It can be a absent, dead, or imaginary person or
something too strongly; portraying feeling that are too an inanimate object (stars, ocean).
strong". Here, "light-winged smoke" has been addressed in the
Hence, the correct answer is option (b). poem.
Direction (96-100) Read the following poem Hence, the correct answer is option (a).
and answer the questions that follow: 100. The word "Lark" in the third line
SMOKE means_______
Light-winged Smoke! Icarian bird, (a) laugh (b) escape
Melting thy pinions in thy upward flight; (c) giggle (d) skylark
Lark without-song, and the messenger of dawn, Ans. (d) : The word "lark" mentioned in the third line
Circling above the hamlets as thy nest; of the poem means "Skylark", a bird.
Or else, departing dream, and shadowy form Hence, the correct answer is option (d).

UGC NTA NET/JRF English Paper II, 2022 (Sh.-II) 397 YCT
UGC NTA NET/JRF Exam. Dec. 2021/June 2022
ENGLISH-II
SOLVED PAPER [ Exam Date : 13.10.2022, Shift-I

1. Who is the author of A Literature of their Own 3. Subramani's Fantasy Eaters (1988) is a
(a) Sandra Gilbert and Susan Gubar (a) novel
(b) Elaine Showalter (b) collection of short stories
(c) Virginia Woolf (c) collection or essays
(d) Sylvia Plath (d) poem
Ans. (b) : 'A Literature of Their Own' is written by an Ans. (b) : Subramani's 'Fantasy Eaters' (1988) is a
American literary critic and feminist. The term collection of short stories (nine short stories), the works
Gynocriticism was introduced by her. Her 'A Literature focus on the ways in which transplanted Indian
of Their Own' talks about the female literary tradition traditions both nurture and suffocate the Island
which she analyses as an evolution through three phases immigrants and their descendants.
which are: Hence, option (b) is correct answer.
(i) Imitation of the modes of the dominant tradition and
4. Who is the author of the poem "The House of
internalization of the artistic and social values.
Fame"?
(ii) Protest against these standards and values and a call
(a) William Langland (b) Geoffrey Chaucer
for autonomy.
(iii) Self-discovery, turning inward free from some of (c) Thomas Moore (d) Philip Sidney
the dependency of opposition. Ans. (b) : 'The House of Fame' is an autobiographical
Hence, Elaine Showalter is correct answer. dream allegory in three books, written by Geoffrey
Chaucer. This work comes under Italian Period of
2. Match List-I with List-II :
Geoffrey Chaucer. In Book (I), the poet dreams about a
List-I List-II temple of Venus where he finds inscriptions of Aeneid.
(A) Gender and Nation (I) Catherine In book 2, the poet carried away by the eagle to the
Belsey 'House of Fame,' while book 3 talks about the poet's
(B) Greek (II) K.J. Dover difficulty in climbing on a high rock on which the house
Homosexuality is situated.
(C) The Subject of (III) W.E.B. Du Thus, option (b) will be correct answer.
Tragedy Bois 5. Which of the following denote the three phases
(D) The Souls of Black (IV) Nira of literary feminism according to Elaine
Folk Yuval- Showalter?
Davis A. Feminine
Choose the correct answer from the options B. Gynic
given below: C. Womanish
(a) (A)-(I), (B)-(IV), (C)-(III), (D)-(II) D. Feminist
(b) (A)-(III), (B)-(II), (C)-(IV), (D)-(I) E. Female
(c) (A)-(II), (B)-(IV), (C)-(III), (D)-(I) Choose the most appropriate answer from the
(d) (A)-(IV), (B)-(II), (C)-(I), (D)-(III) options given below:
Ans. (d) : In the given options, 'The Souls of Black (a) A, B and C only (b) A, D and E only
Folk' is written by W.E.B. Du Bois. It is about the (c) A, B and E only (d) B, C and D only
legacy of racism the black people were facing in the
Ans. (b) : The three phases of literary feminism
form of physical abuse, paternalism and economic
disenfranchisement. according to Elaine Showalter are: "feminine" (women
writers imitate men), the "feminist" (women advocated
'The Subject of Tragedy' moves around the
minority rights), and the "female" (the focus is now on
differential identities of man and woman. In this book,
women's texts as opposed to merely uncovering
Catherine Belsey has depicted the specific period in
which man was the subject to which woman was misogyny in men's texts). Thus, option (b) is correct
related, while the 'Greek Homosexuality' is based on answer.
homosexuality in ancient Greece, written by the 6. Who among the following has authored The
classical scholar Kenneth Dover. 'Gender and Nation' is Revenger's Tragadie?
written by Nira Yuval-Davis. (a) Cyril Tourneur (b) John Webster
Thus, option (d) will be correct answer. (c) John Flethcher (d) Thomas Heywood
UGC NTA NET/JRF English Paper II, 2022 (Sh.-I) 398 YCT
Ans. (a) : 'The Revenger's Tragedy' was written by 10. Given below are two statements : One is
Cyril Tourneur. Its central character, Vindice, is intent labelled as Assertion A and the other is labelled
upon avenging his mistress poisoned by the lecherous as Reason R.
old Duke. Tourneur was an English soldier, diplomat Assertion (A): Roland Barthes describes two
and dramatist, while John Webster was an English basic categories of text as "the readerly" and
Jacobean dramatist best known for his tragedies. 'The ''the Writerly".
White Devil,' 'The Duchess of Malfi'. Thomas Heywood Reason (R) : Language is the window through
is best known for his masterpiece 'A Woman Killed which one sees the world.
With Kindness'. John Fletcher was a Jacobean-
In the light of the above statements, choose the
playwright who collaborated on writing plays with
Francis Beaumont and with Shakespeare on three plays, most appropriate answer from the options
Thus, option (a) will be correct answer. given below:
(a) Both (A) and (R) are correct and (R) is the
7. The "Ancient and Modern Quarrel" in
correct explanation of (A)
Western Literary Criticism appears during
(a) 100 BC (b) Both (A) and (R) are correct but (R) is not the
correct explanation of (A)
(b) Fifth Century CE
(c) Sixteenth Century CE (c) (A) is correct but (R) is not correct
(d) Twentieth Century CE (d) (A) is not correct but (R) ia correct
Ans. (c & d) : 'The Ancient and Modern Quarrel' in Ans. (b) : Roland Barthes has used the term "the
western literary criticism appears during sixteenth and readerly" and "the writerly" to distinguish between texts
early 17th century and later, it reappeared in 20th that are straight forward and demand no special effort to
century. It was an essential feature of the European understand and those whose meaning is not
Renaissance. One of the key episodes in the quarrel's immediately evident and demand some effort on the
development was the so-called quarrel of the part of the reader, but here, reason is not the correct
inscriptions. explanation of assertion.
Thus, (c) and (d) will be correct answer. Thus, option (b) will be correct answer.
8. Find the correct explanation of the term 11. Identify the one that has not been paired
"Aporia": correctly.
(a) It denotes a speaker's or character's (a) Criticism is Wilderness - Geoffrey Hartman
deliberation on an irresolvable question (b) madness and Civilization- Roland Barthes
(b) It is an address to something inanimate (c) Poetry and Repression - Harold Bloom
(c) It is applied to a work of art fraught with (d) Strangers to Ourselves - Julia Kristeva
inherent tension
(d) It refers to a from of denial of existence Ans. (b) : In the given options, 'Madness and
Civilization' has not been paired. It was written by a
Ans. (*) : In the terminology of deconstruction, it is French philosopher, literary critic and historian Mitchel
favoured as a term for an insoluble contradiction
Foucault. The full title of this book is 'Madness and
between two possible meanings of a given text or
Civilization: A History of insanity in the Age of
passage. "Aporia" is a rhetorical figure in which the
speaker appears perplexed by indecision over some Reason.
question or choice. 'Criticism in the Wilderness' is written by an American
Hamlet's soliloquy "to be or not to be.....' is the best- literary critic Geoffrey Hartman, while 'Poetry and
known extended example. Repression' is, written by the American literary critic
Note- NTA did not consider any of the answer given in Harold Bloom. 'Strangers to Ourselves', written by Julia
the option. Kristeva is concerned with the notion of the 'stranger",
the foreigner, outsider or alien in a country and society.
9. Canadian Multiculturalism Act was passed in
the year: Thus, option (b) will be correct answer.
(a) 1958 (b) 1968 12. Given below are two statements : One is
(c) 1978 (d) 1988 labelled as Assertion A and the other is labelled
Ans. (d) : 'Canadian Multiculturalism Act' was passed as Reason R.
in the year 1988. It affirms the policy of the Assertion (A) : In so far as we are taught how
Government of Canada to ensure that every Canadian to read, what we engage are not texts but
must receive equal treatment by the government which paradigms.
respects and celebrates diversity. It recognizes Canada's Reason (R) : We appropriate meaning from a
multicultural heritage and this heritage must be text according to what we need or desire. or. in
protected. Besides it, it preserves equality rights, other words, according to the critical
minorities rights and aboriginal rights. assumptions of the predispositions that we
Thus, option (d) is correct answer. bring to it.
UGC NTA NET/JRF English Paper II, 2022 (Sh.-I) 399 YCT
In the light of the above statements, choose the 15. Match List-I with List-II
most appropriate answer from the options List-I List-II
given below: (A) O' Henry (I) The Last
(a) Both (A) and (R) are correct ant (R) is the Suttee
correct explanation of (A) (B) Rudyard Kipling (II) Beauty
(b) Both (A) and (R) are correct but (R) is not the (C) Oscar Wilde (III) At Verona
correct explanation of (A) (D) Ralph Waldo (IV) Hard to
(c) (A) is correct but (R) is not correct Emerson Forget
(d) (A) is not correct but (R) is correct Choose the correct answer from the options
Ans. (a) : In the given options, assertion and reason given below:
both are correct because New Criticism examines the (a) (A)-(III), (B)-(I), (C)-(IV), (D)-(II)
relationship between a text's ideas and its form, between (b) (A)-(II), (B)-(IV), (C)-(III), (D)-(I)
what a text says and the way it says it. (c) (A)-(IV), (B)-(I), (C)-(III), (D)-(II)
Thus, option (a) will be correct answer. (d) (A)-(I), (B)-(III), (C)-(II), (D)-(IV)
13. 'The Deserted Village' by Oliver Goldsmith Ans. (c) : 'The Last Suttee' by Rudyard Kipling is a
poem, moves around the death of a Rajput king, while
(a) Critiques the rural institutions 'At Verona' is a poem by Oscar Wilde. 'Beauty' is a
(b) Voices revolt of the individual man against poem by Ralph Waldo Emerson and O' Henry wrote
institutions 'Hard to Forget'.
(c) Reflects upon different views on the human Thus, option (c) will be correct answer.
soul 16. "A man can be destroyed but not defeated".
(d) Advocates urbanism over rural backwardness Which of the following texts glorifies this as its
Ans. (b) : 'The Deserted Village (1770), by Oliver predominant theme?
Goldsmith is a poem which deplores the growth of (a) The Old Man and the Sea
trade, the demand for luxuries, and the mercantile spirit (b) War and Peace
which have ruthlessly depopulated such villages and (c) A Farewell to Arms
driven' a bald peasantry, their country's pride' to (d) For whom the Bell Tolls
emigration. James Boswell ascertained that the last four
Ans. (a) : "A man can be destroyed but not defeated",
lines were contributed by Samuel Johnson, because the,
this line has been taken from Ernest Hemingway's "The
poem appears to lament a personal loss.
Old Man and the Sea" (1952), a parable-novella about
Thus, option (b) "Voices revolt of the individual man man's struggle against nature, while 'War and Peace' is
against institution" is very near to the answer. written by Russian author Leo Tolstoy. A Farewell to
14. Which among the following are appropriate Arms' (1929), a novel by Ernest Hemingway is the story
about Latin American Literature? of a love affair between an American lieutenant and an
A. There has been racial coherence and unity in English nurse during the war on the Italian front
literary representations of Latin America demonstrated a similar laconic and understated style.
B. The Latin American Literary tradition draws For whom the Bell Tolls' (1940) by Hemingway was
analogy between plant growth and human published just after the end of the Spanish Civil war.
movement Thus, option (a) will be correct answer.
C. Dzul Poot's stories depict the geography of the 17. Identify the correct combination among the
Chilam Balam Towns following:
D. Quechua had a wider popularity and presence A. Demons - Novel
across different nations. B. Landlady - Vovella
E. The Latin American Literature is unitary and C. The Crocodile - Short-Story
refrains from intextuality of any kind D. A Writer's Diary - Essay
Choose the most appropriate answer from the E. Mary Stuart - Translation
options given below: Choose the most appropriate answer from the
(a) A, D and E only (b) C, B and E only options given below :
(c) B, C and E only (d) B, C and D only (a) A, B and E only (b) C, D and E only
Ans. (d) : Latin-American Literature is literature of the (c) B, C and E only (d) B, C and D only
Spanish-speaking countries of the western Hemisphere. Ans. (*) : 'Demons' is a novel by Russian prose writer
The literary period began with the explorations in the Fedor Mikhailovich Dostoevsky. Sometimes this novel
1400s. It is capable of depicting social and political has been called 'The Possessed' or 'The Devils'. 'The
truth of humanistic ideology that posits itself as Landlady' is a short horror story by Roald Dahl, while
universal. It is a space that mirrors violent realties, 'The Crocodile' is a short story by Fyodor Dostoyevsky
power abuses and concrete administrations of justice that was published in 1865. 'Mary Stuart' is a verse play
and power through narrative imagery. by Friedrich Schiller, it is about Mary, Queen of Scots.
Thus, option (d) will be correct answer. There is no match at all in the given options.

UGC NTA NET/JRF English Paper II, 2022 (Sh.-I) 400 YCT
18. Identify the novels that were published in the Choose the correct answer from the options
1980s and the 1990s given below:
A. Red Earth and Pouring Rain (a) A and B only (b) B and E only
B. The Circle of Reason (c) C and D only (d) A, B and D only
C. The Ghosts of Vasu Master Ans. (c) : Aristotle has used the phrase "language with
pleasurable accessories" in his definition of tragedy in
D. Miguel Street 'Poetics'. 'Melody' and 'Spectacle' are simply pleasurable
E. The Siege of Babylon accessories but melody is more important to the tragedy
Choose the correct answer from the options than spectacle. Here, happiness is not pleasure, nor is it
given below : virtue. According to Aristotle happiness cannot be
(a) B, C D and E only (b) A, B and C only achieved until the end of one's life. The phrase, given in
(c) B, A and D only (d) A, C and E only the question, moves around a language full of rhythm
and harmony and a language superadded with song.
Ans. (b) : 'Red Earth and Pouring Rain' (1995) is a Thus, option (c) will be correct answer.
novel by Vikram Chandra, won the 1996 21. Should poets bicycly-pump the human heart or
Commonwealth Writers' Prize for best first book. 'The squash it flat?
Circle of Reason' (1986) is a novel written by Amitav Man's love is of man's life a thing apart:
Ghosh, while 'The Ghost of Vasu Master' (1994) is a Girls aren't like that.
novel by Githa Hariharan, an Indian writer and editor. The above line are written by:
'Miguel Street' is a collection of short stories by V.S. (a) Philip Larkin (b) Kingsley Amis
Naipaul and 'The Siege of Babylon' is a tragedy by (c) Donald Davie (d) John Wain
Samuel Pordage. Ans. (b) : The given lines are written by Kingsley
Thus, option (b) is correct answer. Amis, who raised in a small community called Norbury.
19. Which of the following observations are true Thus, option (b) will be correct answer.
about Roland Barthes' contributions to literary 22. Given below are two statements:
theory? Statement I : "...... he who discovers no God
A. He rejected the model for structural analysis of Whatever, how shall he discover Heroes, the
narratives Visible Temples of God" is a statement by
B. He perceived "meaning" as an effect of various Thomas Carlyle.
interconnections among linguistic codes Statement II : "It is not that men are ill fed, but
that they have no pleasure in the work by
C. He identified the various codes found in the which they make their bread, and therefore
process of structuration look to wealth as the only means of pleasure" is
D. He played a significant role in the development a statement made by John Ruskin.
of 'semiolgy' In the light of the above statements, choose the
E. He questioned the concept of literary criticism correct answer from the options given below:
as an act of uncovering some hidden truth (a) Both Statement I and Statement II are true
intended by the "Author" (b) Both Statement I and Statement II are false
Choose the correct answer from the options (c) Statement I is true but Statement II is false
given below: (d) Statement I is false but Statement II is true
(a) A, B and C only Ans. (a) : In the given options, statement II has been
(b) C, D and E only taken from 'Unto This Last' by John Ruskin. Here, both
statements are correct, hence, option (a) will be correct
(c) B, C, D and E only answer.
(d) A, B, C, D and E only
23. Match List-I with List-II
Ans. (c) : Roland Barthes, a French literary critic, List-I List-II
essayist and cultural theorist is well-known for his (A) "Faces along the (I) Wilfred
reaction to the trend of existentialist philosophy. He has bar/cling to their Owen
contributed semiotics (Literary semiotics, semiotics of average day".
photography, comics semiotics) and narratology (B) "The awful (II) T.S. Eliot
linguistics.' daring of a
Hence, option (c) will be correct answer. moments
20. What does Aristotle mean by the phrase surrender".
"language with pleasurable accessories" in his (C) "Bent double, like (III) Allen
definition of tragedy? old beggars under Ginsberg
sacks".
A. A language full of pompous vocabulary
(D) "I saw the best (IV) W.H.
B. An embellished language minds of my Anden
C. A language full of rhythm and harmony generation
D. A language superdded with song destroyed by
E. Emotive language madness".

UGC NTA NET/JRF English Paper II, 2022 (Sh.-I) 401 YCT
Choose the correct answer from the options D. Choriambus : It is a foot of verse
given below: consisting of two
(a) (A)-(IV), (B)-(II), (C)-(I), (D)-(III) stressed syllables
(b) (A)-(III), (B)-(I), (C)-(IV), (D)-(II) enclosing two
(c) (A)-(I), (B)-(IV), (C)-(II), (D)-(III) unstressed syllables
(d) (A)-(II), (B)-(IV), (C)-(I), (D)-(III) E. Trochaic : It has a stressed
syllable followed by
Ans. (a) : The line "faces along the bar/cling to their an unstressed one.
average day" has been taken from 'September 1, 1939',
Choose the most appropriate answer from the
a poem by W.H. Anden. It was written during the
options given below:
World War II. In the option (b), the line "the awful
(a) A, B and C only (b) B, C and E only
during of a moments surrender" comes from the section
(c) B, C and D only (d) C, D and E only
'What The Thunder Said' which is the part of 'The
Waste Land' by T.S. Eliot. Here, awful daring" means Ans. (*) : In the given options, there is no correct match
making a momentary decision that may change the at all for the right answer, that's why NTA has dropped
trajectory of one's life. The line "bent double, like old this question.
beggars under sacks" has been mentioned by a war poet Spondee is a metrical foot consisting of two long
Wilfred Owen in the poetry 'Dulce Et Decorum Est'. In syllables, as determined by syllable weight in classical
this line, the poet compares soldiers to old beggars who meters, or two stressed syllables in modern meters,
while. Pyrrhic is a metrical foot consisting of two short
are "bent double". Instead of being dressed smartly and
or unaccented syllables.
proudly in their uniform, they are pictured "under
sacks". Amphimacer is a trisyllabic foot consisting of a short
syllable between two long syllables in quantitative verse
The line "I saw the best minds of my generation or of an unstressed syllable between two stressed
destroyed by madness" has been taken from the poetry syllables in accentual verse.
'Howl' by beat generation writer Allen Ginsberg. Choriambus is a four-syllable metrical foot with the first
Hence, option (a) will be correct answer. and last syllables stressed. The choriambic metre is
24. J Hillis Miller, one of the leading exponents of found in Greek choruses and was favoured in Latin by
deconstruction, makes a deconstructionist Horace, but is very rarely used in English as the basis
reading of which of the following poems of P.B. for whole lines:
Shelley? 'Trochaic' (adjective form) or Trochee is a metrical foot
(a) "The Triumph of Life" consisting of a stressed syllable followed by an
(b) "Ode to the West Wind" unstressed one, but in Latin and Ancient Greek poetic
(c) "Revolt of Islam" metre, a trochee is a heavy syllable followed by a light
one. In this respect, a trochee is the reverse of an iamb.
(d) "The Witch of Atlas"
26. Identify the poems termed as "pastoral
Ans. (a) : J. Hillis Miller, one of the leading exponents
elegies":
of deconstruction makes a deconstructionist reading of
A. Lycidas
P.B. Shelley's 'The Triumph of Life' by saying that the
business of deconstructionism is not to deconstruct the B. In Memory of W.B. Yeats
text but to show how the text has deconstructed itself, C. Adonais
because deconstruction is not a dismantling of the D. Thyrsis
structure of a text but a demonstration that it has already E. In Memoriam
dismantled itself. In 1986, J. Hillis Miller was asked to Choose the most appropriate answer from the
give the Presidential address at the annual meeting of Option given below:
the Modern Language Association. Here, Miller (a) A, B and C only (b) B, C and E only
proclaimed the 'triumph of theory', which has (c) A, C and D only (d) C, D and E only
preoccupied English studies for the last twenty-five Ans. (c) : Pastoral writing spans the genres of poetry,
years at least. drama, and prose fiction. Despite its extreme
Hence, option (a) will be correct answer. artificiality, it is often capable of oblique or overt social
criticism, in which rural harmony is contrasted with the
25. Identify the correct combination among the
corruptions of court or city. English pastoral writing
following :
derives from classical sources (the Idylls of Theocritus).
A. Spondee : It consists of three Lycidas (1637) is a pastoral elegy by John Milton, it
stressed syllables laments the death of Edward king who drowned while
B. Pyrrhic : It consists of two crossing from Chester Bay to Dublin when his ship
unstressed syllables struck a rock and sank in calm water.
C. Amphimacer : It is a metrical foot Dr. Samuel Johnson condemned it by saying "easy,
of three syllables vulgar, and therefore disgusting".

UGC NTA NET/JRF English Paper II, 2022 (Sh.-I) 402 YCT
'Adonais' (1821) is an elegy on the death of John Keats, Margaret Atwood is a Canadian poet and novelist. Her
by P.B. Shelley. It is composed in 55 Spenserian novel 'The Blind Assassin' (2000; Booker Prize) is the
stanzas, inspired partly by the Greek elegies of Bion and story of two sisters, whose relationship is revealed within
Moschus. Shelley strongly identified himself with keats' complex layers of parallel narratives. 'Alias Grace' (1996)
sufferings, and in his preface he attacks the Tory revisits the ambiguous history of a 16 year-old Canadian
reviewers with pen 'dipped in consuming fire'. housemaid, convicted of the murder of her employer and
'Thyrsis' a monody commemorates the Matthew his housekeeper in 1843. 'The Testament' (2019) is a
Arnold's friend, Arthur Hugh Clough who died at sequel to The Handmaid's Tale (1985). 'The Handmaid's
Florence, 1861. The poem is a pastoral elegy lamenting Tale' (1985) is a dystopia set in the imaginary Republic
Clough as Thyrsis. of Gilead, where failing fertility results in the sexual
'In Memory of W.B Yeats' is an elegy written by W.H. enslavement of women for breeding purposes. Hence,
Auden, while 'In Memoriam' is written in memory of option (b) will be correct answer.
Arthur Henry Hallam by Tennyson between 1833 and 29. When was Haruki Murkami's Men Without
1850 and published anonymously in the later year. Women published?
Hence, option (c) will be correct answer. (a) 2017 (b) 2018
27. Given below are two statements: (c) 2014 (d) 2019
Statement I : Comparative Literature is a Ans. (c) : 'Men Without Women' is a collection of short
study of different clutures, nations and genres, stories by Japanese author Haruki Murakami and it was
and it explores the inherent relationship
published in 2014. The story revolves around an
between literature and other forms of cultural
unnamed narrator who receives a phone call in the
exploration.
middle of the night who tells him that his former lover
Statement II : In the study of literature
has committed suicide.
and culture, the importance of methodology is
secondary. Thus, option (c) will be correct answer.
In the light of the above statements, choose the 30. In which year Miles Coverdale translated The
correct answer from the options given below: Old Testament of The Bible?
(a) Both Statement I and Statement II are true (a) 1533 (b) 1534
(b) Both Statement I and Statement II are false (c) 1535 (d) 1536
(c) Statement I is true but Statement II is false Ans. (c) : Miles Coverdale revised and completed
(d) Statement I is false but Statement II is true Tyndale's work, producing the first complete printed
Ans. (c) : In the given options, statement I is related to English Bible in 1535. This became the basis of the
comparative literature in which literary works and English Church's first official Bible, the Great Bible
traditions of more than one nation or language are (1539-40) later revised as the Bishops' Bible (1568).
studied. By contrast with nationally or linguistically The principal Bible of English religion and culture since
defined disciplines such as 'English literature, the 17th century is the King James Bible (KJB) or
comparative literature ranges freely across frontiers in Authorized version (1611). Before the Reformation,
search of cross-cultural influences and correspondences. English writers and readers knew the Bible from the
Distinguished practitioners have included Rene Wellek Vulgate. The earliest complete English translation was
and George Steiner, while in the study of literature and the Lollard or Wycliffe Bible.
culture, the importance of methodology is not secondary. Thus, option (c) will be correct answer.
Hence, option (c) will be correct answer. 31. Given below are two statements : One is
28. Choose the right chronological sequence of labelled Assertion A and the other is labelled as
publication of the following novels by Margaret Reason R.
Atwood. Assertion (A) : The Marxists represent
A. Lady Oracle Marxism as a scientific account of social
B. The Blind Assassin change.
C. The Handmaid's Tale Reason (R) : The Marxist ideology believes that
D. The Testaments culture is a mirror of social life and the artist is
E. Alias Grace an engineer of the human soul educating the
Choose the most appropriate answer from the working classes.
options given below: In the light of the above statements, choose the
(a) A, B, C, D, E (b) A, C, E, B, D most appropriate answer from the options
(c) E, A, D, B, C (d) C, A, D, E, B given below:
Ans. (b) : The correct chronological sequence of (a) Both (A) and (R) are correct and (R) is the
publication of the given novels by Margaret Atwood correct explanation of (A)
are: (b) Both (A) adn (R) are correct but (R) is not the
(i) Lady Oracle (1976) (ii) The Blind Assassin (2000) correct explanation of (A)
(iii) The Handmaid's Tale (1985), (iv) The Testaments (c) (A) is correct but (R) is not correct
(2019) (v) Alias Grace (1996). (d) (A) is not correct but (R) is correct
UGC NTA NET/JRF English Paper II, 2022 (Sh.-I) 403 YCT
Ans. (a) : In the given options, assertion (A) and reason Ans. (c) : The concept of "Post-truth" which is very
(R) are the correct factual details regarding Marxist close to defactualization has been given by Hannah
literary Criticism. Marxism as a sociological approach Arendt, a historian, philosopher, and social scientist
denies the existence of gender biasness, differences or who is well-known for "defactualization". "Post-truth"
other type of differences such as caste issues or other term refers to the 21st century widespread
kind of diversity. In fact, they simply assert that society
is divided into two parts which is called the dominant documentation and concern about disputes over public
class or rich class or financially powerful class and the truth claims. It is believed to have made its maiden
second one is called the poor class or crushed class. appearance in a 1992 essay, pertaining to the Iran-
They say that if the monetary or financial conditions are Contra Scandal and Persian Gulf-war garnered
differing, these are the basic structure of society. widespread popularity in the form of "post-truth
Neither Marx nor Engels bequeathed a critical or politics". It gives post-truth relies on absolute lies.
aesthetic theory, but they tended to disparage socialist Hence, option (c) will be correct answer.
writers of a propagandist type, and suggested that art is
not tied directly to phases of economic development but 34. Match List-I with List-II
has a certain autonomy. List-I List-II
Marx's materialist view of history, with its emphasis on (A) To the Light (I) 1913
economic issues such as ownership of the means of house
production and the central importance of the conflict (B) Sons and Lovers (II) 1927
between social classes, as well as his dialectical
methodology, adapted from Hegel, have been especially (C) Finnegans Wake (III) 1939
influential. Hence, option (A) is correct answer. (D) The Waste Land (IV) 1922
32. Which of the following are Plato's main Choose the correct answer from the options
objections against poetry? given below:
A. The poet is an imitator. (a) (A)-(I), (B)-(II), (C)-(IV), (D)-(III)
B. The poet is incapable of bravery. (b) (A)-(IV), (B)-(II), (C)-(I), (D)-(III)
C. The poet, by fueling passions and emotions, (c) (A)-(III), (B)-(IV), (C)-(I), (D)-(II)
weakens the reasoning capacity of the citizens.
(d) (A)-(II), (B)-(I), (C)-(III), (D)-(IV)
D. The poet is less responsible.
E. The poet has no knowledge of the world. Ans. (d) : 'The Waste land' (1922) is a poem by T.S.
Choose the correct answer from the options Eliot, first published in Criterion and a few days later in
given below: The Dial. It consists of five sections, 'The Burial of the
(a) A, C and E only (b) A and B only Dead', 'A Game of Chess', 'The fire Sermon,' 'Death by
(c) B and D only (d) E and D only Water', and 'What the Thunder said'. It was seriously
Ans. (a) : Plato criticizes poetry as a whole and praised by I.A. Richards as "a perfect emotive
prescribes to banish the poets from the 'Republic' description of a state of mind which is probably
because it shows the suffering of virtuous people and inevitable for a while to all meditative people".
prosperity of wicked men. He further says that the poet 'Sons and Lovers' (1913) is an autobiographical
imitates without knowing the reality and shows his lack Bildungsroman novel, known for incestuous
of purpose and knowledge. According to Plato, poetry relationship. 'Finnegan's Wake' (1939) is a wondrously
evokes passion and emotion, therefore it is deceptive in perplexing work of James Joyce. It is written in a
nature, and the poet has no knowledge of the world
unique style which makes abundant use of puns and
because he does not depict the stark realities of world in
his poetry, rather he takes shelter in imagination and portmanteau words.
fanciful notions. 'To the Lighthouse' (1927) is a novel by Virginia Woolf
Hence, option (A) will be correct answer. and it is about the Ramsay family.
33. Given below are two statements: Hence, option (d) will be correct answer.
Statement I : Hannah Arendt's 35. The Writer and the World by V.S. Naipaul is a
"defctualization" is very close to the concept of (a) novel (b) Travelogue
"Post-truth". (c) collection fo essays (d) non-fiction
Statement II : Post-truth relies on absolute lies.
Ans. (c & d) : 'The Writer and the World' by V.S.
In the light of the above statements, choose the
Naipaul is a collection of essays on post-colonial India
most appropriate answer from the options
given below: Sir V.S. Naipaul is a novelist, travel-writer, born in
(a) Both Statement I and Statement II are correct Trinidad, got Booker Prize for 'In a free State' (1971)
(b) Bothe Statement I and Statement II are and was awarded the Nobel Prize in 2001. His major
incorrect works are: 'The Mystic Masseur' (1957), 'The Suffrage
(c) Statement I is correct but Statement II is of Elvira' (1958), The Mimic Men (1967), 'Guerrillas'
incorrect (1975), 'A Bend in the River (1979) and 'The Enigma of
(d) Statement I is incorrect but Statement II is Arrival' (1987).
correct Hence, option (c) and (d) will be correct answer.
UGC NTA NET/JRF English Paper II, 2022 (Sh.-I) 404 YCT
36. Which among the following is an incomplete experiences from a hunter to a worker on Mokamesh
poem by P.B. Shelley? Ghat to a healer. Kailash Sankhala's 'Wild Beauty' is
(a) "The Triumph of Life" focused on wild life description and Kailash is well-
(b) "Ode to the West-wind" known as The Tiger Man of India. 'Nights and Days:
(c) "Queen Mab" My Book of India's Wildlife' is written by M. Krishnan.
(d) "The Daemon of the World" Thus, option (a) will be correct answer.
Ans. (a) : 'The Triumph of Life' is an unfinished 39. A.L. Tennyson in the following lines:
visionary poem by P.B. Shelley, written in the bay "Yet I doubt not through the ages one
Lerici in summer 1822 and composed in Terza Rima. increasing purpose runs.
The poem is strongly influenced by Dante's 'Inferno'. And the thoughts of men are widen'd with the
'Ode to the West Wind' by Shelley, was written in 1819 process of the suns"
and published in 1820. The ode is a passionate (a) Reflects upon secularism
invocation to the spirit of the West Wind, both destroyer (b) Reflects upon evolutionary faith
and preserver.' 'Queen Mab' is a visionary and (c) Reflects upon utilitarianism
ideological poem by Shelley, published privately in (d) Reflects upon materialism
1813 when he was 21. The poem is in nine cantos using Ans. (b) : Through these lines of 'Locksley Hall',
didactic and descriptive blank verse greatly indebted to Tennyson reflects upon evolutionary faith. It is a
Milton and to Robert Southey's 'Thalaba'. monologue spoken by a disappointed lover, revisiting
'Alastor' or 'The Spirit of Solitude,' a visionary poem the desolate moorland home by the sea where he had
was written in 1815 and published in 1816. been brought up by an unsympathetic uncle, and where
Thus, option (a) will be correct answer. he fell in love with his cousin Amy. The narrator scorns
37. Which writer does not belong to the Angry the modern world of steamship and railway, and ends
Young Men Movement? with an ambiguous acceptance of 'the ringing grooves
(a) John Osborne (b) Kingsley Amis of change'. Thus, option (b) will be correct answer.
(c) Seamus Heaney (d) Philip Larkin 40. Which of these statements are true in the
context of Neuro-Linguistic programming?
Ans. (c & D) : The Angry Young Men Movement was
a journalistic catchphrase loosely applied to a number of A. 'Neuro' in NLP means that our behaviour is
British playwrights and novelists from the mid-1950s, determined by our sensory experiences
including Kingsley Amis, John Osborne, Alan Sillitoe B. Grammatical Knowledge is a matter of practice
and Colin Wilson whose political views were radical or C. NLP was developed by Richard Bandler and
anarchic, and who described various forms of social John Grinder in the 1970s
alienation. It is sometimes said to derive from the title D. Neuro covers "invisible thoughts and visible
of a work by the Irish writer Leslie Paul, 'Angry Young physiological reactions"
Man' (1951). E. All of the above
Thus, option (c) and (d) will be correct answer. Choose the most appropriate answer from the
options given below:
38. Match List-I with List-II
(a) E only (b) A, C and D only
List-I List-II
(c) A and D only (d) B, C and D only
(A) Salim Ali (I) Wild Beauty
Ans. (b) : Neuro-linguistic programming is a
(B) Jim Corbett (II) The Fall of pseudoscientific approach to communication, personal
a Sparrow development and psychotherapy, that first appeared in
(C) Kailash Sankhala (III) Night and Richard Bandler and John Grinder's 1975 book 'The
Days: My Structure of Magic I'. In the given options, (A), (C) and
Book of (D) are true.
Indian Thus, option (b) will be correct answer.
Wildlife 41. Match List-I with List-II
(D) M. Krishnan (IV) My India List-I List-II
Choose the correct answer from the options (A) "Willing to (I) Irony
given below: wound, and yet
(a) (A)-(II), (B)-(IV), (C)-(I), (D)-(III) afraid to strike."
(b) (A)-(III), (B)-(II), (C)-(IV), (D)-(I) (B) "It is a truth (II) Simile
(c) (A)-(I), (B)-(III), (C)-(IV), (D)-(II) universally
(d) (A)-(II), (B)-(III), (C)-(IV), (D)-(I) acknowledged
Ans. (a) : 'The Fall of a Sparrow' is written by Salim that a single man
Moizuddin Ali, an Indian ornithologist and naturalist, in possession of
sometimes referred to as the "birdsman of India". good fortune must
'My India' is written by Edward James "Jim" Corbett. It be in want of a
is an autobiography and a collection of various wife."

UGC NTA NET/JRF English Paper II, 2022 (Sh.-I) 405 YCT
(C) "Thou bride still (III) Antithesis David Lodge's 'Changing Places' (1975) comes under
unravished of the category of campus novel. 'The History Man' (1975)
quietness. Thou by Malcolm Bradbury is a campus novel. 'The Masters'
foster child of is the fifth novel in C.P. Snow's series 'Strangers and
silence and slow Brothers'.
time." Hence, option (b) will be correct answer.
(D) "And ice, mast- (IV) Assonance 43. Anglo-Irish relations in the 20th Century have
high, come been represented in which of the following
floating by, as novels?
green as A. Elizabeth Bowen - The Last September
emerald." B. May Sinclair - The Divine Fire
Choose the correct answer from the options C. JG Farrell - Troubles
given below: D. J G Farrell - The Siege of Krishnapur
(a) (A)-(II), (B)-(III), (C)-(I), (D)-(IV) E. Jeffery Farnol - Black Bartlemy's
(b) (A)-(III), (B)-(I), (C)-(IV), (D)-(II) Treasure
(c) (A)-(III), (B)-(II), (C)-(IV), (D)-(I) Choose the most appropriate answer from the
(d) (A)-(I), (B)-(IV), (C)-(II), (D)-(III) options given below:
Ans. (b) : "Thou still unravished bride of quietness....... (a) A, B and C only (b) A and C only
this line comes under the figure of speech called (c) D, E and B only (d) B, C and A only
Assonance because there is repetition of similar vowel Ans. (b) : 'The Last September' (1929) is a novel by the
sounds in the stressed syllables of neighbouring words. Anglo-Irish writer Elizabeth Bowen. It is based on
It is distinct from full rhyme in that while the vowel Anglo-Irish relations in the 20th century.
sounds correspond, the consonants do not. J.G. Farrell's (Frederic William)' Troubles' (1970) is a
"Willing to would, and yet afraid to strike", this line is novel, set in Ireland, in the decaying Majestic Hotel,
the example of antithesis. Antithesis accentuates the just after the First World War, against a background of
idea of contrasted clauses or phrases. Literally, it means Sinn Feien violence, while 'The Siege of Krishnapur'
the opposite, it is a device in which two contradictory (1973, Booker Prize) deals with the events of the Indian
viewpoints are put together in a sentence to achieve the Mutiny, in a characteristically ironic and comic vein,
opposite effect. The line "It is a truth universally while Black Bartlemv's Treasure' is written by Jeffery
acknowledged that a single man in Farnol.
possession.................. is the example of irony, a figure Hence, option (b) will be correct answer.
of speech in which there is a contradiction of 44. Which among the following is appropriate
expectation between what is said and what is really about Ngugi Wa Thiongo's article "Literature
meant. The line "And ice, mast high ..........is the in schools"?
example of simile. A. The article discusses the relevance and
Thus, option (b) will be correct answer. adequacy of the present education system.
42. Match List-I with List-II B. It advocates teaching of European texts and
List-I List-II literature to the students of the third world
(A) Malcolm (I) Masters countries
Bradbury C. It reflects negatively upon the literature taught
(B) David Lodge (II) Lucky Jim to the Kenyan students in National Schools
D. It argues that cultural imperialism distorts
(C) Kingsley Amis (III) The History
people's vision of history
Man
E. It observes that European teachers are better
(D) CP Snow (IV) Changing equipped to teach literature to the Kenyan
places students
Choose the correct answer from the options Choose the most appropriate answer from the
given below: options given below:
(a) (A)-(I), (B)-(II), (C)-(III), (D)-(IV) (a) A, B and C only (b) C, D and E only
(b) (A)-(III), (B)-(IV), (C)-(II), (D)-(I) (c) B, C and D only (d) A, C and D only
(c) (A)-(II), (B)-(III), (C)-(IV), (D)-(I) Ans. (d) : Ngugi Wa Thiong'o changed his name from
(d) (A)-(IV), (B)-(III), (C)-(I), (D)-(II) James Ngugi when he stopped writing in English and
Ans. (b) : In the given options, "Lucky Jim' (1954) by began publishing in Gikuyu. Both decisions were
Kingsley Amis whose hero, lower-middle-class lecturer motivated by his belief that writing in the language of
Jim Dixon, with his subversive attitudes (anti- the colonizer alienated Africans from their own culture.
establishment, anti-pretension, anti-arts and crafts) was Through his article "Literature in Schools", he had
hailed as an 'Angry Young Man'. depicted the relevance, adequacy, shortcomings of
UGC NTA NET/JRF English Paper II, 2022 (Sh.-I) 406 YCT
present education system. He had talked about the D. complex construction of power
failure, cultural integrity and educational enlightenment. E. identity politics
Actually, he was born in Kenya, where a State of Choose the most appropriate answer from the
Emergency was declared in 1952 because of the Mau options givne below:
Mau Mau uprising. (a) B, D and E only (b) A, C and E only
Thus, option (d) will be correct answer. (c) B, D and C only (d) C, D and E only
45. Given below are two statements: Ans. (a) : Kimberle Crenshaw's term "intersectionality
Statement I : According to W.H. Auden, The is widely used for racial justice, complex construction
Importance of Being Earnest is the purest of power and identity politics. 'Intersectionality is a
example in English Literature of a 'Verbal qualitative analytic framework flourished in the late 20th
Opera'. century that identifies how interlocking systems of
Statement II : Oscar Wilde possessed profound power affect those who are most marginalized in
insight into the range of the arts that in a society.
combined form make theatre performance Hence, option (a) will be correct answer.
possible. 49. Which of these graphic narratives depict
In the light of the above statements, choose the political crisis?
correct answer from the options given below: A. Amruta Patil - Kari
(a) Both Statement I and Statement II are true B. Joe Sacco - Footnotes in Gaza
(b) Both Statement I and Statement II are false C. Art Spiegelman - Maus
(c) Statement I is true but Statement II is false D. Sarnath Banerjee - Corridor
(d) Statement I is false but Statement II is true E. Phoebe Gloeckner - The Diary of a Teenage
Ans. (a) : 'The importance of Being Earnest': A Trivial Girl
Comedy for Serious People' is a play by Oscar Wilde Choose the most appropriate answer from the
and according to W.H. Auden it is the purest example in options given below:
English Literature of a 'Verbal Opera'. (a) A, B and C only (b) C, D and E only
Here, both statement are true, hence option (a) will be (c) B and C only (d) B, C and D only
correct answer.
Ans. (c) : 'Footnotes in Gaza' is a graphic narrative
46. Gaiutra Bahadur wrote an autobiographic based on bloody incidents between Israelis and
novel titled Palestinians in Gaza during the Suez Crisis. According
(a) An Era of Darkness (b) Dauka Puran to united Nations figures quoted in the book, Israeli
(c) The Coolie Woman (d) Rama's Banishment forces killed 275 Palestinians in Khan Yunis on in
Ans. (c) : Gaiutra Bahadur wrote an autobiographic November, 1956.
novel titled 'Coolie Woman: The Odyssey of Indenture'. 'Maus' is also a graphic narrative by an American
The story is about Sujaria, the great grandmother of the Cartoonist Art Spiegelman about the experiences as a
author, while 'An Era of Darkness: The British Empire Polish Jew and Holocaust survivor. Here Jews are
in India'. An Era of Darkness' came along, there was no depicted as mice, wehrmacht soldiers as cats, Poles as
single work that clearly and unambiguously catalogued pigs, Americans as dogs, the British as fish, the French
all the harm done to India under British rule. The as frogs, the Swedes as reindeer and the Romani as
publication of Subramani's 'Dauka Puran' is an gypsy moths. Hence, option (c) will be correct answer.
important event in the literary and cultural history of the
Indo-Fijan community in particular, and of Fiji in 50. Match the following works of Edward
general. Hence option (c) will be correct answer. Braithwaite according to their year of
publication:
47. Stuart Hall belongs to _______ schools of
cultural studies. (A) Rights of Passage (I) 1973
(a) Oxford (b) Cambridge (B) Islands (II) 1969
(c) Birmingham (d) American (C) Masks (III) 1967
Ans. (c) : Stuart Hall belongs to Birmingham schools of (D) The Arrivants (IV) 1968
cultural studies. The Centre for Contemporary Cultural Choose the correct answer from the options
studies was a research centre at the University of given below:
Birmingham, England. It was founded in 1964 by Stuart (a) (A)-(II), (B)-(I), (C)-(III), (D)-(IV)
Hall and Richard Hoggart.
(b) (A)-(I), (B)-(III), (C)-(IV), (D)-(II)
Hence, option (c) will be correct answer.
(c) (A)-(IV), (B)-(II), (C)-(I), (D)-(III)
48. Kimberle Crenshaw's term "intersectionality" (d) (A)-(III), (B)-(IV), (C)-(II), (D)-(I)
is widely used for
Ans. (*) : Rights of Passage (1967) is a notable work of
A. academic deterioration Kamau Brathwaite while 'Rights of Passage' is a novel
B. racial justice based on science-fiction, written by Alexi Panshin and
C. peace formation published in 1968. Another work 'Rights of Passage' is
UGC NTA NET/JRF English Paper II, 2022 (Sh.-I) 407 YCT
written by William Golding in 1980. It is an account of Choose the most appropriate answer from the
a voyage by sea to Australia 'Islands (1969) is written options given below:
by Kamau Brathwaite, a Barbadian poet, while 'Masks' (a) A, B and C only (b) A, C and E only
was published in (1968). 'The Arrivants: A New World (c) C, D and E only (d) B, C and D only
Trilogy' by Kamau Brathwaite, comprising three earlier
Ans. (d) : Tractarian Movement (Oxford Movement) is
volumes: 'Rights of Passage, 'Masks' and 'Islands'.
a movement of thought and doctrine within the church
Note : NTA has dropped this question. of England, centred at Oxford, which began with the
51. Who is the author of the essay "Three Assize Sermon on National Apostasy preached by John
Women's Texts and a Critique of Imperialism" Keble in 1833. The movement aimed to defend the
(1985)? Church of England as a divine institution with an
(a) Ania Loomba independent spiritual status, and to revive the High
(b) Meenakshi Mukherjee Church traditions of the 17th century. It criticized the
(c) Susan Meyer tendencies of the Anglican Church to accept the
(d) Gayatri Chakravorty Spivak authority of the state and to tolerate a range of
Ans. (d) : "Three Women's Texts and a Critique of theological positions. Keble's sermon inspired John
Imperialism" (1985) is written by Gayatri Chakravorty Henry Newman, Richard Hurrell Froude, and others to
Spivak. There are noticeably, many images of mirroring launch their series 'Tracts for the Times' in 1833 (which
in the text. gave the Tractarian movement its name)
There are two facts in this book she wanted to convey, Hence, option (d) will be correct answer.
first: The role of literature in the production of cultural 54. Given below are two statements: One is
representation should not be ignored and the second is labelled as Assertion A and the other is labelled
that It should not be possible to read nineteenth century as Reason R.
British literature without remembering that imperialism. Assertion (A) : In 19th century, Charlotte
Hence, option (d) will be correct answer. Perkins Gilman asserted economic
52. Which among the following are true about independence over voting rights.
Harold Pinter? Reason (R) : The representation of women as
A. Harold Pinter was born in the year 1925. power-seekers was not socially acceptable in
B. He was influenced by Samuel Beckett and the 19th century America.
Theatre of the Absurd. In the light of the above statements, choose the
C. The Caretaker and The Alchemist are his most appropriate answer from the options
famous plays. given below:
D. Stanley is a character in The Birthday Party. (a) Both (A) and (R) are correct and (R) is the
E. Betraval is story of a married couple. correct explanation of (A)
Choose the correct answer from the options (b) Both (A) and (R) are correct but (R) is not the
given below: correc explanation of (A)
(a) A, D and E only (b) B, D and E only (c) (A) is correct but (R) is not correct
(c) B, C and D only (d) A, C and D only (d) (A) is not correct but (R) is corrects
Ans. (b) : Harold Pinter (1930-2008) was a poet and Ans. (a) : Charlotte Perkins Gilman, a writer and
playwright, born in East London. Pinter was highly feminist asserted economic independence over voting
influenced by Samuel Beckett and the Theatre of rights. Her experience is recorded in her most famous
Absurd. He was awarded the Nobel Prize for Literature short story, 'The Yellow Wallpaper' (1892). She
in 2005. 'Betrayal' (1978; film, 1982) is an ironic advocated for the necessity of women's economic
tragedy which ends in beginning and traces with a independence in her works 'Women and Economics'
reversed chronology, the development of a love affair (1898), 'Concerning Children' (1900), and 'The Home:
between a man and his best friend's wife. Its Work and Influence' (1903). 19th century American
Hence, option (b) will be correct answer. women were facing suppression and were not allowed
53. Which among the following is true about the to raise their voice against it. Her autobiography, 'The
Tractarian Movement? Living of Charlotte Perkins Gilman', was published
posthumously in 1935.
A. It was widespread across the world
B. The other leaders of the movement were Paul Hence, option (a) will be correct answer.
Newman and R.H. Fraude 55. What is the correct sequence of the following
C. The movement began with a sermon by John feminist texts?
Keble in 1833 A. Sexual Politics
D. Pusey gave the movement cohesion, fame and a B. A World of Difference
name C. The Female Imagination
E. The ideal of the Christian Church was praised D. Thinking About Women
by Oxford Convocation E. A Room of One's Own
UGC NTA NET/JRF English Paper II, 2022 (Sh.-I) 408 YCT
Choose the correct answer from the options Reason (R) : Lacan was expelled from the
given below: International Psychoanalytical Association in
(a) (A), (B), (C), (D), (E) 1959.
(b) (E), (B), (D), (A), (C) In the light of the above statements, choose the
(c) (E), (D), (A), (C), (B) most appropriate answer from the options
(d) (B), (C), (D), (E), (A) given below:
Ans. (c) : The correct sequence of the following texts (a) Both (A) and (R) are true and (R) is the
will be: 'A Room of One's Own' (1929), 'Thinking correct explanation of (A)
About Women' (1968), 'Sexual Politics' (1970), 'The (b) Both (A) and (R) are true but (R) is not
Female Imagination' and 'A World of Difference' correct explanation of (A)
(1990). (c) (A) is true but (R) is false
'A Room of One's Own' (1929) by Virginia Woolf is a (d) (A) is false but (R) is true
classic of the feminist movement where she demands Ans. (b) : Jacques Lacan, a French psychoanalyst
that "a woman must have money and a room of her own reformulated Freudian models of the unconscious and
if she is to write fiction". underlined the fundamental role played by language in
'Sexual Politics' (1970) by Kate Millett argues that "sex the formation of human identity. For Lacan, humans are
has a frequently neglected political aspect". defined and driven principally by desire Lacan's
'Thinking About Women' (1968) by Mary Ellmann, rethinking of identity in terms of lack and instability has
talks about the evolution of femininity. had a substantial impact on disciplines across the
Hence, option (c) will be correct answer. humanities and social sciences, including political
56. Match List-I with List-II science, film studies and queer theory. He was expelled
from the international Psychoanalytical Association in
List-I List-II
1959.
(A) Donald Davie (I) Against Thus, option (b) will be correct answer.
Romanticis
m 58. The heroic couplet is a pair of
(B) Philip Larkin (II) Hurry on (a) Twelve-syllable lines that rhyme
Down (b) Ten-syllable lines that rhyme
(C) Kingsley Amis (III) The Shires (c) Eight-syllable lines that do not rhyme
(d) Eight-syllable lines that rhyme
(D) John Wain (IV) The North
Ship Ans. (b) : The heroic couplet is a pair of 'ten-syllable
Choose the correct answer from the options lines that rhyme'. It is a traditional form, commonly
given below: used in epic and narrative poetry. It may be called, a
rhyming pair of lines in iambic pentameter. It was first
(a) (A)-(IV), (B)-(I), (C)-(II), (D)-(III)
used by Geoffrey Chaucer in the 'Legend of 'Good
(b) (A)-(I), (B)-(III), (C)-(IV), (D)-(II)
Women' and was perfected by John Dryden and
(c) (A)-(II), (B)-(I), (C)-(III), (D)-(IV) Alexander Pope. Thus, option (b) will be correct
(d) (A)-(III), (B)-(IV), (C)-(I), (D)-(II) answer.
Ans. (d) : Donald Davie was an English Movement 59. Consumerism is a major theme in which of the
poet and critic. His poem 'The Shires' is well known for follwing works?
speculative, erudite and philosophical opinion A. Lovalties
manifesting a mind that (in his own phrase) moves most
B. Saint Joan of Stockyards
easily and happily among abstractions'.
C. Death of a salesman
'The North Ship' (1945), the poems were, by Larkin's
own account, much influenced by W.B. Yeats. 'Against D. Candida
Romanticism' by Kingsley Amis starts with the line "a E. Waiting for Godot
traveller who walks a temperate zone, woods devoid of Choose the most appropriate answer from the
beasts............... options given below:
Amis was a member of 'The Movement', a group of (a) A and B only (b) D and A only
British Poets (Thom Gunn, Philip Larkin, John (c) B and C only (d) A and E only
Betjeman, Elizabeth Jennings, Wendy Cope). 'Hurry on Ans. (c) : 'Saint Joan of the Stockyards' is a play written
Down' (1953) was the best novel written by John Wain. by the German modernist playwright Bertolt Brecht.
Thus, option (d) will be correct answer. Consumerism is a major theme of this play. 'Death of a
57. Given below are two statements : One is Salesman' (1949) is a play written by Arthur Miller. The
labelled as Assertion A and the other is labelled play contains a variety of themes, such as the American
as Reason R. dream, the anatomy of truth and infidelity but
Assertion (A) : Jacques Lacan was radically somewhere it moves around consumerism.
critical of the esistium psychoanalytical theory. Hence, option (c) will be correct answer.
UGC NTA NET/JRF English Paper II, 2022 (Sh.-I) 409 YCT
60. Match List-I with List-II Ans. (*) : This particular statement was given by
List-I List-II Coleridge, examining Wordsworth's theory of poetic
(A) Hamlet (I) 1606 diction. The most remarkable part of 'Biographia
Literaria' lies in Coleridge's Criticism of Wordsworth's
(B) Macbeth (II) 1599
theory of poetry and poetic diction. While critically
(C) Julius Caesar (III) 1604 analyzing Wordsworth's theory Coleridge has offered
(D) Othello (IV) 1600 his own views on the choice of rustic, themes and
Choose the correct answer from the options characters as well as the language of poetry.
given below: Note- NTA has dropped this question.
(a) (A)-(III), (B)-(IV), (C)-(II), (D)-(I) 63. Who among the following were poet Laureates
(b) (A)-(I), (B)-(II), (C)-(IV), (D)-(III) of England?
(c) (A)-(II), (B)-(I), (C)-(III), (D)-(IV) (A) Alfred Austin
(d) (A)-(IV), (B)-(I), (C)-(II), (D)-(III) (B) Robert Bridges
Ans. (*) : 'Hamlet', a tragedy written by Shakespeare in (C) Watts-Dunton
1601. It was registered as lately acted' in July 1602. (D) Oscar Wilde
Three different early versions of the play are extant: the Choose the correct answer from the options
first quarto (1603); the second quarto (1604); and the given below:
first folio (1623). 'Macbeth' by Shakespeare, probably (a) (A) and (C) only
written and first Performed at the Globe in 1606, but not (b) (A) and (D) only
Printed until the first folio (1623). Julius Caesar, a (c) (B), (C) and (D) only
Roman tragedy by Shakespeare, probably written and (d) (A), (B) and (C) only
performed in 1599, not printed until the first folio Ans. (*) : In the given options, Alfred Austin and Robert
(1623). Its major source is Thomas North's translation Bridges are the person who got poet laureateship. Alfred
of Plutarch's 'Lives'. Austin was appointed Poet Laureate in 1896 after the
Othello, the Moor of Venice', a tragedy by Shakespeare death of Tennyson, while Robert Bridges, an English
was written between 1602 and 1604 and it was poet was poet Laureate from 1913 to 1930.
performed before James I at Whitehall. It was first Note- NTA has dropped this question.
printed in quarto in 1622, and again in a different
64. Which of the following critics is associated with
version in the folio of 1623. The story is taken from
the term "contrapuntal reading"?
Cinzio, which Shakespeare probably read in Italian,
(a) Mikhail Bakhtin (b) Edward Said
possibly also in French.
(c) Roland Barthes (d) Jacques Derrida
Note : NTA has dropped this question.
Ans. (b) : "Contrapuntal Reading" is associated with
61. Philip Sidney's Arcadia was influenced by Edward Said. It is used in interpreting colonial texts,
(a) The Spanish Romance of Montemayor considering the perspectives of both the colonizer and
(b) The Italian Paintings of Veronese the colonized. This approach is not only helpful but also
(c) The Arthurian Legends necessary in making important connections in a novel.
(d) The Metaphysical Poetry This term is mentioned in his classic book, 'Culture and
Ans. (a) : Philip Sidney's 'The Arcadia' was influenced Imperialism'. According to Said some of the most
important works of the western literary canon rested
by 'The Spanish Romance of Montemayor'. It is a prose
upon a submerged and unacknowledged foundation of
romance. It exists in two versions: the first, completed
colonialism.
by 1581, it is known as the Old Arcadia; the second
Hence, option (b) will be correct answer.
version, now known as the New Arcadia.
65. Who among the following is not a recipient of
Another 'Arcadia' is a series of verse eclogues
the Nobel Prize for Literature?
connected by prose narrative, published in 1504 by
(a) Winston Churchill (b) T.S. Eliot
Sannazar, occupied with the loves, laments and other
(c) W.H. Auden (d) Madam Curie
doings of various shepherds in Arcadia, while Arcadia
is a district in the central Peloponnese, through which Ans. (c & d) : Sir Winston Leonard Spencer Churchill
Virgil's 'Ecologues' became the traditional location of got Nobel Prize in 1953 for his literary achievement,
the idealized world of pastoral. while T.S. Eliot in 1948, got Nobel Prize for literature.
W.H. Auden who was recommended for a Nobel Prize,
Hence, option (a) will be correct answer.
but he did not win this award, while Madam Curie got
62. Who said, "There is, there can be and there Nobel Prize for Physics in 1903 and in 1911 for
ought to be the difference between the language chemistry. She is the only woman to win the award in
of prose and metrical composition"? two different fields but she could not get Nobel Prize
(a) John Dryden (b) William Wordsworth for Literature.
(c) S.T. Coleridge (d) T.S. Eliot Hence, option (c) & (d) will be correct answer.
UGC NTA NET/JRF English Paper II, 2022 (Sh.-I) 410 YCT
66. Which of the following concepts are associated The semantic code points to any element in a text that
with Bhartrhari's theory of 'Sphota'? suggests a particular, often additional meaning by the
A. Rasa way of connotation.
B. Alankara The symbolic code is "deeper" structural principle that
C. Dhvani organizes semantic meanings.
D. Vakrokti Thus, option (d) will be correct answer.
E. Shabda Brahman 69. Who among the following critics approriates
Choose the most appropriate answer from the the following statement by Karl Marx?
options given below: "They cannot represent themselves : they must
(a) A and E only (b) C and E only be represented".
(c) D and B only (d) A and C only (a) Ruth Vanita
Ans. (b) : Bhartrhari's theory of "Sphota" entails a kind (b) Kamla Bhasin
of mental perception which is described as a moment of (c) Flaria Agnes
recognition, an instantaneous flash, whereby the hearer (d) Chandra Talpade Mohanty
is made conscious, though hearing sounds, of the latent Ans. (d) : Chandra Talpade Mohanty a postcolonial and
meaning unit already present in his consciousness. transnational feminist approved the statement given by
"Dhvani" and "Shabda Brahman" are associated with Karl Marx on peasant consciousness. Chandra Talpade
Bhartrhari's theory of "Sphota". The word Dhvani is Mohanty is well-known for "Under Western Eyes:
used for both-the suggestive word and the suggestive Feminist Scholarship and Colonial Discourses.
meaning. The poetic language can never be possible
Hence, option (d) will be correct answer.
without Dhvani, while "Shabda Brahman" means
transcendental sound or sound vibration. 70. Which of the following is not true about
Hence, option (b) will be correct answer. "Lyrical Ballads"?
(a) It is a manifesto fo Romantic poetry
67. Which of the following theorists indentifies
"metaphor" as two fundamental structures of (b) It turns English Poetry away from the social
language? and intellectual sophistication of the
(a) Ferdinand de Sassure (b) Roland Barthes seventeenth and the eighteenth centrury
poetry
(c) J.L. Austin (d) Roman Jacobson
(c) It takes poetry out of the confines of reason
Ans. (d) : Roman Jakobson indentified "metonymy" and intellect to the unravished and unspoilt
and "metaphor" as two fundamental structures of beauties of nature
language. In his essay Two Aspects of Language and
(d) It is very particular about the form and
Two Types of Aphasic Disturbances' Jakobson proposes
that language has a bipolar structure, oscillating structure of a poem
between the poles of metaphor and metonymy. He Ans. (d) : 'Lyrical Ballads', with a few other poems, by
further says that poetry is metaphoric and it focuses on Wordsworth and Coleridge is a collection of poems.
signs while prose is metonymic. The first edition appeared in 1798, the second with new
Hence, option (d) will be correct answer. poems and a preface (known as the 1800 edition)
January 1801 and a third in 1802. The book is a
68. According to Roland Barthes, which of the
landmark of English Romanticism and is often seen as
following "Codes" are common to all
narratives? the beginning of a new literary epoch. 'Lyrical Ballads'
is not very particular about the form and structure of a
A. Synthetic code
poem.
B. Proairctic code
Thus, option (d) will be correct answer.
C. Semic Code
D. Hermencutic code 71. Tolkappiyam is a book of grammar and poetics
written in the _____ language.
E. Symbolic code
(a) Telugu (b) Tamil
Choose the correct answer from the options
given below: (c) Kannada (d) Malayalam
(a) A and B only (b) B and C only Ans. (b) : 'Tolkappiyam' is a book of grammar and
(c) A, B and C only (d) B, C, D and E only poetics written in the Tamil Language. It is a unique
work on grammar and poetics.
Ans. (d) : Roland Barthes argues that every narrative is
interwoven with multiple codes. The Hermeneutic Code Thus, option (b) will be correct answer.
(HER) refers to any element in a story that is not 72. "It is significant that the productive capacities
explained and, therefore, exists as an enigma for the of this Third Space have a colonial or post
reader. colonial provenance". The above lines have
The proairetic code refers to the other major structuring been written by
principle that builds interest or suspense on the part of a (a) Salman Rushdie (b) Edward Said
reader or viewer. (c) Benedict Anderson (d) Homi K. Bhabha

UGC NTA NET/JRF English Paper II, 2022 (Sh.-I) 411 YCT
Ans. (d) : These given lines have been written by an "Surprised by Sin' was written by Stanley Fish, here
Indian-British scholar and critical theorist Homi K. Fish suggested that the subject of Milton's masterpiece
Bhabha. It is somewhere focused on identity and is in fact the reader, who is forced to undergo spiritual
community. Third space theory suggests that every self-examination when led by Milton down the path
person is a hybrid of their unique set of affinities. taken by Adam and Eve and Satan, while 'The Way
Edward Soja's theory of "Thirdspace" sees three urban Women Write' is written by Mary P. Hiatt.
spaces: First space, Second space and Third space. Hence, option (c) will be correct answer.
Thus, option (d) will be correct answer. 75. Which of these are not forms of flash fiction?
73. Which of the collections of poems are not A. Drabble
written by Meena Alexander? B. Postcard fiction
A. River and Bride C. Novelette
B. Articulate Silence D. Short Story
C. Raw Silk E. Nanofiction
D. Stone Roots Choose the most appropriate answer from the
E. A Time to Change options given belwo:
Choose the correct answer from the options (a) A and E only (b) A, B and E only
given below: (c) D and E only (d) C, D and E only
(a) A and B only (b) B and D only Ans. (*) : Flash fiction is a fictional work of extreme
(c) C and E only (d) B and E only brevity and has its roots going back to prehistory fables
Ans. (a & d) : 'Raw (2004) Silk' by Meena Alexander, and parables. In flash fiction, there comes "drabble"
talks about the countries, foreign and familiar, places also known also known as microfiction.
where the heart and spirit live. Her poetry collections NTA has dropped this question because there is
include 'Stone Roots' (1980), 'House of a Thousand something wrong in the options.
Doors' (1988) 'The Garden' (1992) and her new volume 76. Identify the combinations(s) that belong to the
of poem is 'River and Bridge' (1995) which is genre of sci-fi/speculative fiction.
mentioned in the option, while 'A Time to change' was A. Vandana Singh - The Woman Who
the first significant book of postcolonial poetry in Thought She was
English by Nissim Ezekiel. 'Articulate Silences' is Planet
written by King-Kok Cheung, is a subtle response to
B. Tehmina Durrani - Blasphemy
issues relating to cultural diversity.
C. Salman Rushdie - The Satanic
74. Match List-I with List-II
Verses
List-I List-II
D. Priya Sarukkai - Generation 14
(A) The Poetics of (I) Stanley Fish Chabria
Prose
E. Gautam Bhatia - The wall
(B) Problems of (II) Tzvetan
Choose the most appropriate answer from the
Dostoevsky's Todrov options given below:
Poetics
(a) A, B and D only (b) A, D and E only
(C) Surprised by Sin (III) Mikhail (c) A, C and D only (d) D, A and C only
Bakhtin
Ans. (b) : 'The Woman Who Thought She Was Planet'
(D) The Way Women (IV) Mary Hiatt by Vandana Singh is a work based on science-fiction. It
Write is a story of a woman who tells her husband of her
Choose the correct answer from the options curious discovery.
given below: Priya Sarukkai Chabria's 'Generation 14' is based on
(a) (A)-(IV), (B)-(II), (C)-(I), (D)-(III) ambitious and inventive science-fiction about cloning
(b) (A)-(III), (B)-(IV), (C)-(II), (D)-(I) and control. Gautam Bhatia's 'The Wall', comes under
(c) (A)-(II), (B)-(IV), (C)-(III), (D)-(I) the genre of science-fiction. 'The Wall', begins with a
(d) (A)-(II), (B)-(IV), (C)-(III), (D)-(I) map of a circular city called Sumer enclosed by an
Ans. (c) : 'The Poetics of Prose' is written by a impassable, miles-high wall-the ultimate gated
Bulgarian-French historian, philosopher and community.
structuralist literary Critic, Tzvetan Todorov. 'Problems Hence, option (b) will be correct answer.
of Dostoevsky's Poetics' is a book by a Russian literary 77. Beside Being a playwright, who among the
theorist Mikhail Bakhtin. Through this work, Bakhtin following has translated Homer?
introduces a number of key, concepts, such as (a) Ben Johnson (b) Thomas Dekker
polyphony and carnivalisation. (c) Thomas Heywood (d) George Chapman

UGC NTA NET/JRF English Paper II, 2022 (Sh.-I) 412 YCT
Ans. (d) : Besides being a playwright, George Chapman A. The Golden deals with Johor travelling
has translated Homer. The first of his Homeric Noteboodk to Rohonda
translations, 'Seven Books of the Iliad's of Homer', B. The Good is about a doomed love
appeared in 1598 with a dedication to the earl of Essex Terrorist affair
complaining of poverty; twelve books of the 'Iliad', C. Shikasta is about a planet, which is
dedicated to Prince Henry, appeared in 1609; the cut-off due to the advanced
complete 'Iliad' and 'Odyssey' were published together influence of civilization
in 1616 as 'The whole works of Homer, Prince of Poets'. D. Alfred and explores the life of her
Hence, option (d) will be correct answer. Emily parents
78. Which of the following is not a part of Amitav E. The Grass is Draws from her
Ghosh's Ibis Trilogy? Singing experiences in Africa
(a) Sea of Poppies Choose the correct answer from the options
(b) River of Smoke given below:
(c) Flood of Fire (a) B, A and D only (b) A, B and E only
(d) The Calcutta Chromosome (c) C, D and E only (d) E, B and A only
Ans. (d) : 'The Calcutta Chromosome' is a novel by Ans. (c) : Doris Lessing's 'Shikasta', 'Alfred and Emily'
Amitav Ghosh. This book deals with myth, nihilism, and 'The Grass is Singing' are matched correctly with
their respective themes.
science, Philosophy and superstition, and it is based on
the life of the Sir Ronald Ross, who did a breakthrough 'Shikasta' is a 1979 science fiction novel and is the
account of Canopus' work on Earth during a difficult
research on malaria in 1989, while Ibis trilogy is written
period. Canopus became interested in the planet called
by Amitav Ghosh, consisting of the novels 'Sea of Rohanda and he plans to bring the planet into its empire
Poppies' (2008), 'River of Smoke (2011), and 'Flood of by speeding human evolution. 'Alfred and Emily' is a
Fire' (2015). 2008 book in hybrid form, which is based on the lives
Hence, option (d) will be correct answer. of Lessing's parents. 'The Grass is Singing' is a 1950
79. What is meant by "Corporate author" in novel, which is drawn from her experience in Africa. It
research? follows the life of an emotionally immature woman and
(a) A writer who belongs to a corporate company hasty marriage to an unsuccessful farmer.
(b) A writer who writes on matters of corporate 'The Golden Notebook' is a 1962 novel, and is a
affairs powerful account of a woman searching for her
personal, political and professional identity amid the
(c) A work produced by an institution, an trauma of emotional refection and sexual betrayal.
association or a government agency
'The Good Terrorist' is a 1985 political novel, which
(d) A renowned author follows Alice Mellings, a woman who transforms her
Ans. (c) : When referencing a publication by a home into a headquarters for a group of radicals who
corporate author in the body of our paper, we need to plan to join the IRA.
include both the name of the corporate 82. Which of the following Articles of the Indian
body/organization and the page numbers we drew the Constitution made the provision for use of
information from, if the name of the English, alongside Hindi, for official purposes,
corporation/organization is quite long, it is often better for fifteen years?
to put the name in the sentence leading up to the (a) Article 351 (b) Article 344
citation. (c) Article 343 (d) Article 348
Hence, option (c) will be correct answer. Ans. (c) : 'Article 343' of the Indian constitution made
80. Who among the following attached himself to the provision for use of English alongside Hind for
the Earl of Nottingham's Theatrical company? official purposes, for fifteen years.
(a) William Shakespeare Article 343 is about the official language of the Union.
(b) Christopher Marlowe The 2nd part of the article states, 'for a period of 15 years
from the commencement of this constitution, the
(c) Geoge Peele English Language shall continue to be used for all the
(d) Ben Johnson official purposes of the union for which it was being
Ans. (b) : Christopher Marlowe, attached himself to the used immediately before such commencement'.
Earl of Nottingham's theatrical company. He was Hence option (c) is correct.
athestic and probably homosexual and was probably 83. Under the Net (1954) is written by
employed as a government spy. Admiral's Men, also (a) John Fowles (b) Iris Mudroch
called Lord Admiral's Men, a theatrical company in (c) Edmund Goose (d) William Cooper
Elizabethan and Jacobean England.
Ans. (b) : 'Under The Net' is a 1954 novel by Iris
Hence, option (b) will be correct answer. Murdoch. Murdoch was an Irish and British novelist
81. Which of these are correct combination of the and philosopher who is best known for her novels about
works by Diris Lessing and their respective good and evil, sexual relationships, morality and the
themes? power of the unconscious.
UGC NTA NET/JRF English Paper II, 2022 (Sh.-I) 413 YCT
'Under The Net' is her first novel. It follows the life of Ans. (*) : 'Women Beware Women' is a play by
an aspiring writer Jake Donaghue as he stumbles from Thomas Middleton and was published in 1657. it is a
place to place through Europe in search of illusory Jacobean tragedy which revolves around family
ideals. The novel falls in the genre of the Picaresque, a betrayals, romantic infidelities and false friendships.
comedic form in which a clever, lower class protagonist It is a play about a righteous questioning of presumed
makes his way up in the world using his wits. The novel morality, where sexuality and class trump traditional
uses its protagonists wandering to show how conflicts gender roles, where love cats itself and tastes delicious.
and resolutions arise unpredictably in the creative life. Note- None of the options are correct.
Hence option (b) is correct. Therefore, NTA has dropped this question.
84. Who among the following in the article. 87. Identify the correct ones among the following:
"Fleshly School of Poetry", attacked the Pre- A. Arun Kolatkar uses colloquial speech in his
Raphaelites, especially D.G. Rossetti? poems
(a) Robert Browning B. Kolatkar envisions abstract qualities to paint a
(b) William Holeman Hunt picture of life of his own kind
(c) Robert Buchanan C. The poem Boatride by Kolatkar talks about a
(d) Christina Rossetti ride in the Ganges
D. Private Poems in Public Garden is composed
Ans. (c) : In the article, 'Fleshly school of Poetry', by Dilip Chitre
Robert Buchanan attacked the Pre-Raphaelites; The
E. 'In Ethiopia' is a poem on Africa by Arun
Fleshly school of poetry is a fierce attack on the Pre-
Kolatkar
Raphaelite school. Written in 1871, the essay was first
Choose the most appropriate answer from the
published in The contemporary Review under the
options given below:
Pseudonym 'Thomas Maitland'. Principally, 'Maitland'
(a) A, B and C only (b) B, D and E only
focuses on the art of and poetry of Dante Gabriel
Rossetti. (c) A, B and D only (d) C, B and E only
The Pre-Raphaelite was an artistic movement founded Ans. (c) : Arun Kolatkar uses colloquial speech in his
in 1848 by the poet and painter Rossetti and the painters poems. Colloquial language and expressions could be
John Everett Millais and William Holman Hunt. things like informal words, phrases and slang words
which is used by writers to create a sense of community
Buchanan attacked them as he believed that Pre-
and society.
Raphaelite art was excessively 'sensual' implying 'that
the body is greater than the soul'. Arun Kolatkar was an Indian poet who wrote in both
Marathi and English. Kolatkar envisions abstract
Hence option (c) is correct. qualities to paint a picture of life of his own kind. His
85. Find out the correct sequence of the poems found humour in everyday matters.
publications of the following books Private Poems in Public Garden is composed by Dilip
(a) India : An Area of Darkness: India : A Chitre, who was one of the foremost Indian poets and
Wounded Civilization : India : A Million critics to emerge in the post Independence India.
Mutinies Now: A House for Mr. Biswas Hence option (c) is correct.
(b) A House for Mr. biswas: India : A Million 88. Given below are two statements:
Mutinies Now: India : An Area of Darkness: Statement I : The teaching of non-native
India: A Wounded Civilization literature to the students of English Language
(c) A House for Mr. Biswas: India : An Area of Teaching is arid.
Darkness: India : A Wounded Civilization: Statement II : The negative responses in ELT
India: A Million Mutinies Now classroom can create an interesting classroom
(d) India : A Wounded Civilization: A House for situation.
Mr. Biswas: India: An Area of Darkness: In the light of the above statements, choose the
India: A Million Mutinies Now most appropriate answer from the options
Ans. (c) : All the given books are written by V.S. given below:
Naipaul, who was a Trinidadian-born British writer of (a) Both Statement I and Statement II are correct
works of fiction and non-fiction in English. (b) Both Statement I and Statement II are
The correct sequence will be- incorrect
A House for Mr. Biswas - 1961 (c) Statement I is correct but Statement II is
India : An Area of Darkness- 1964 incorrect
India : A wounded civilization - 1976 (d) Statement I is incorrect but Statement II is
correct
India : A Million Mutinies Now- 1990
Hence option (c) is correct. Ans. (a) : Both the given statements are correct. The
teaching of non-nature literature to the students of
86. The book 'Women Beware Women' was English Language Teaching is arid because it has no
published in the year relativeness with non-native literature to the
(a) 1612 (b) 1620 students/learners of English Language. So, the first
(c) 1621 (d) 1622 statement is absolutely correct.
UGC NTA NET/JRF English Paper II, 2022 (Sh.-I) 414 YCT
The negative responses in ELT (English Language "The Yankee and the Yogi" (1981) is authored by B.G.
Teaching) classroom can create an interesting classroom Siddhartha.
situation as it gives ample opportunity to the teacher to Nude therapy is a work of saros cowasiee.
explore and elaborate. ELT (English Language Hence, the correct match is option (b).
Teaching) is teaching of English language to people Read the following passage, and answer the questions
whose first language is not English. that follow : (91-95)
Hence, the correct answer is option (a). However, faced with this world of faithful and
89. Identify the correct combinations: complicated objects, the child can only identify
(A) Dadaism - Tristan Tzara himself as owner, as user, never as creator, he does
not invent the world, he uses it: there are, prepared
(B) Super Realism - Guillanne
for him actions without adventure, without wonder,
Apollinaire without joy. He is turned into a little stay-at-homw
(C) Surrealism - Filipo Marinetti householder who does not even have to invent the
(D) Futurism - Andre Breton mainsprings of adult causality, they are supplied to
(E) Nihilism - Ivan Sergeyevich him ready-made, he has only to heklp himself, he is
Turgenev never allowed to discover anything from start to
Choose the correct answer from the options finish. The merest set of blocks, provided it is not too
given below: refined, implies a very different learning of the world,
then, the child does not in any way create meaningful
(a) (A), (C) and (E) (b) (A), (B), and (E)
objects, it matters little to him whether they have an
(c) (B), (C) and (D) (d) (C), (D) and (E) adult name, the actions he performs are not those of a
Ans. (b) : Dadaism is a movement in art and literature user but htose of a dimiurge. He creates forms which
based on deliberate irrationality and negation of walk, which roll, he creates life, not property, objects
traditional artistic values. Tristan Tzara was a Romanian now act by themselves, they are no longer an inert
and French poet, who was one of the founders and and complicated material in the plam of his hand.
central figures of the movement. Roland Barthes "Toys" (Excerpt from Mythologies)
Super-Realiom is a type of 20th century art and literature 91. Which of the following is a correct
in which unusual or impassible things are shown interpretation?
happening. Apollinaire is considered one of the (a) The child claims the object as his property
foremost poets of the early 20th century as well as one (b) The objects that the child holds are obscure
of the most impassioned defenders of cubism and a and useless
forefather of surrealism. (c) The child cannot understand the design of the
The philosophical idea of existential rihilirm is that life object
has no purpose or value, and that man's existence is (d) In touching the object, the child creates
insignificant. Nihilism was first popularized by the dynamic forms of life
novelist Ivan Turgenev. Ans. (d) : The correct interpretation of the given
90. Identity the correct pairs: excerpt from Roland Barthes' Mythologies, "Toys", is
(A) The Boyfriend - Juliette 'the child creates dynamic forms of life when he touches
Banerjee the object. He has only to use the readymade world, not
(B) Nude Therapy - Margaret to invent it. He thinks himself as the owner, the user of
Chatterjee the stuffs around him but never thinks that he is the
creator of the things.
(C) The Other woman - Dina Mehta
and Other Stories Hence, the correct answer is option (d).
(D) The Yankee and - B.G. Siddarth 92. In the context of the above passage, which is
the Yogi the closest to being true:
(a) Children actively learn while playing
(E) Prejudice of Ages - Vera Sharma
(b) Children are objects for toy makers
Choose the correct answer from the options (c) Toys affect the cognitive abilities of the
given below: children
(a) (A), (B) and (C) (b) (A), (C) and (D) (d) Children recreate meaning from the toys
(c) (B), (C) and (E) (d) (C), (D) and (E)
Ans. (c) : The correct statement from the given options
Ans. (b) : The correct pair of the books and their author is option (c)- Toys affect the cognitive abilities of the
is A,C,D, The Boyfriend is authored by Juliette children. With the help of toys, children tries to know
Banerjee. Born in Kolkata, she lives with her family in the world. Whatever they get, they are in purely
Sydney Australia since 1987. her other works are readymade form, they don't have to invent or create any
"Flower from a Book", "The Gap-Toothed Banister-A thing instead they have to learn from the articles around
Tale of Anglo-India" etc. "The other woman and other them.
stories" is written by Dina Mehta, published in 1981. Hence, the correct answer is option (c).
UGC NTA NET/JRF English Paper II, 2022 (Sh.-I) 415 YCT
93. The adult causality is about Likewise their yearly frame: the Frinton flok
(a) Sensual and sexual knowledge of the world Who put him up for summer holidays,
(b) Cognitive and logical structure of the world And Christmas at his sister's house in Stoke.
(c) Nihilistic recreation of the world But if he stood and watched the frigid wind
(d) Linguistic structure of the objects Tousling the clouds, lay on the fusty bed Telling
Ans. (b) : The adult causality is about cognitive and himself that this was home, and grinned,
logical structure of the world. And shivered, without shaking off the dread
For a child, it is never a matter of concern to think about That how we live measures our own nature,
adults happening; what is the reason behind any deed or And at his age having no more to show
what is cause and effect of any occurrence taking place Than one hired box should make him pretty sure
around. When the aforesaid is being understood, it is He warranted no better, I don't know.
nothing but a matter of cognitive and logical structure. Philip Larkin
Hence, the correct answer is option (b). 96. The poem, "Mr, Bleaney", is written in a
94. The word "demiurge" connotes _______ form.
(a) cognitive inactiveness of the children (a) Satirical (b) Lyrical
(b) their sensational realisation of the objects (c) Dramatic (d) Philosophical
(c) their creative abilities Ans. (c) : Philip Larkin's poem "Mr. Bleaney" is written
(d) their sudden discovery in dramatic form. Written in 1955 and published in the
Ans. (c) : The word "demiurge" connotes from the give 1964 volume "The Whitsun Weddings", it deals with
excerpt- "Children's creative ability". The actions a loneliness, deprivation, and the fear of wasting one's
child performs are not a action of user or for' anything life.
very significant but it is unknowingly a creation. Hence, Hence, the correct answer is option (c).
the correct answer is option (c). 97. In the third line 'They' refers to
95. The world of objects makes the child (a) Workers (b) Owners
(a) Imaginative (b) Inventor (c) Master (d) Manufacturers
(c) Actant (d) Creator Ans. (*) : None of the given options/words (noun) is
Ans. (c & d) : The world of objects makes the child appropriate for "they" given in third line.
Actant, and creator. A child works on the objects NTA has dropped this question.
around him, acts on and creates something with the help 98. Mr. Bleaney was the ______ of the house.
of objects. (a) Owner (b) Tenant
Hence, the correct answer is option (c & d). (c) Master (d) Possessor
Note:- NTA has considered both the option as correct Ans. (b) : Mr. Bleaney was the tenant (living in rented
answer. room) of the house. The poem's speaker rents a digy
Read the following poem, and answer the questions room and discovers that the previous tenant, Mr.
that follow (96-100): Bleaney, lived there for many years, seemingly trapped
Mr Bleaney in a solitary, dull existence.
This was Mr Bleaney's room. He stayed Hence, the correct answer is option (b).
The whole time he was at the Bodies. till 99. According to the speaker Mr. Bleaney was
They moved him,' Flowered curtains, thin and (a) a humorous person
frayed, (b) a social and fun-loving person
Fall to within five inches of the sill, (c) a hard working person
Whose windo shows a strip of building land. (d) a sad and dull person
Tussocky, littered, 'Mr Bleaney took Ans. (d) : According to the speaker Mr. Bleaney was a
My bit of garden properly in hand.' sad and dull person. The landlady explains about Mr.
Bed, upright chair, sixty-watt bulb, no hook Bleany as a solitary and dull. Hence, the correct answer
Behind the door, no room for books or bags- is option (d).
'I'll take it, So it happens that I lie 100. The poem "Mr. Bleaney" deals with the
Where Mr Bleaney lay, and stub my fags portrayal of his _________.
On the same saucer-souvenir, and try (a) richness (b) extravagance
Stuffing my ears with cotton-wool, to drown (c) luxuriousness (d) ordinariness
The jabbering set he egged her on to buy. Ans. (d) : The poem "Mr. Bleaney" deals with portrayal
I know his habits - what time he came down. of his ordinariness as his life was very monotonous;
His preference for sauce to gravy, why there was no newness in his life.
He kept on plugging at the four aways- Hence, the correct answer is option (d).
UGC NTA NET/JRF English Paper II, 2022 (Sh.-I) 416 YCT
NTA UGC NET/JRF Exam. Dec. 2022
ENGLISH- II
SOLVED PAPER [01 March, 2023 Shift- I]

1. ‘‘When nature prompted and no law denied understand the hardships of being a woman that society
promiscuous use of concubine and bride; Then wants her to be. In this poem she portrays men
Israel’s monarch after Heaven’s own heart, His dominance over women in all matters of life.
vigorous warmth did variously impart to wives Throughout the world, fathers are considered to be the
and slaves.’’ head of their family. However, only a handful of people
truly understand the vital role of father plays in the
From which poem are these lines taken?
family. Hence, option (b) is the correct answer.
(a) Absalom and Achitophel : A Poem
4. Which of the following poems is not written by
(b) MacFlecknoe
Sylvia Plath?
(c) A Song for St. Cecilia’s Day
(a) Lady Lazarus (b) Ariel
(d) Alexander’s Feast
(c) Daddy (d) To Ariel
Ans.(a): Above these lines have been taken from a
Ans.(d): The poem To Ariel is not written by Sylvia
poem Absalom and Achitophel composed by John
Plath, while three other poems Lady Lazarus, Ariel and
Dryden. Dryden’s famous Restoration satire Absalom
Daddy are written by Sylvia Plath. Lady Lazarus is a
and Achitophel (1681) is one of the key seventeenth-
poem by Sylvia Plath, originally included in Ariel,
century texts that demand a political reading. John which was published in 1965. Poem Daddy published
Dryden was a brilliant satirist in the Neo-classical posthumously in (1965) the collection Ariel. One of
period. His Absalom and Achitophel is regarded as not Plath’s most famous poems, Daddy was completed
simply a satire but a poem as Dryden himself calls it a during a brief prolific period of writing before her
poem. The central theme is : temptation, sin, fall and suicide in February, 1963.
punishment. Hence, option (a) is the correct answer.
Hence, option (d) is the correct answer.
2. Which of the following poems is written by
5. Robert Burns was born in.
Oliver Goldsmith?
(a) Scotland (b) England
(a) A Deserted Village (b) A Deserted Villa
(c) Ireland (d) America
(c) The Deserted Village (d) A Deserted City
Ans.(a): Robert Burns was born in Scotland, Alloway
Ans.(c): The Deserted Village is written by Oliver on January 25, 1759. He was the first child of William
Goldsmith, published in 1770. It is a work of social and Agnes Burne’s seven children. His father, a tenant
commentary and condemns rural depopulation and the farmer Robert Burns was educated at home. Burns also
pursuit of excessive wealth. The poem is written in attended one year of mathematics schooling in 1765. He
heroic couplets, and describes the decline of a village attended an adventure school established by his father
and the emigration of many of its residents to America. and John Murdock. His father died in bankruptcy in
The Deserted Village is a pastoral elegy by Goldsmith. 1784. At the age of fifteen, Burns fell in love and,
The central image of this 430 line poem is titular village shortly thereafter he wrote his first poem. His famous
of Auburn, the declining boyhood home of the narrator. works are – Halloween (1556) The Banks O’ Doon,
Hence, option (c) is the correct answer. John Anderson, My Jo, To a Louse, Holy Willie’s
3. Tribute to Papa is a book of poems written by. Prayer, A Red Red Rose etc.
(a) Kamala Das (b) Mamta Kalia Hence, option (a) is the correct answer.
(c) Suniti Namjoshi (d) Meena Alexander 6. Who among the following characters personifies
Ans.(b): Tribute to Papa is a book of poems written by necessity in P.B. Shelley’s Prometheus Unbound?
Indian famous poet Mamta Kalia. It is a challenging (a) Prometheus (b) Demogorgon
poem in which Mamta tries to compel the readers to (c) Jove (d) Jeus
NTA UGC NET/JRF English Paper II, (Dec 2022) Shift-I 417 YCT
Ans.(b): Demogorgon is a character who personifies is generally considered to be the greatest Indian literary
necessity in P.B. Shelley’s Prometheus Unbound. The work of any period. Abhijnanasakuntalam makes use of
Demogorgon in Prometheus Unbound is a spirit of the the following four languages : Sanskrit, Shauraseni,
underworld, associated with the realm of night or the Maharashtri and Magdhi. In Sanskrit it's known as The
world of the dead in Pagan Mythology. He represents an Recognition of Shakuntala. Shakuntala is an Indian play
eternal force in nature which cannot change the course that had numerous translations throughout the time
of history but is privy to some secrets of destiny and because it is in the Sanskrit language. Love is the main
fate. When confronted by Panthea and Asia, who have theme of Shakuntalam. Abhijnanasakuntalam is a
been led to the underworld by a dream, the drama that contains the narrative of well-built and true
Demogorgon reveals that Jupiter, although he reigns love of Indian king Dushyanta and Shakuntala.
supreme over earth and heaven, is still subject to the Hence, option (d) is correct answer.
spirit of love, whom ‘‘everything in the universe is
10. Choose from the following options the correct
subject to’’.
combination of playwrights who contributed to
7. Who among the following, after watching the the movement called ‘‘Kitchen Sink Drama’’.
performance of William Shakespeare’s play. A (a) John Osborne, Arnold Wesker, Shelagh
Midsummer Night’s Dream, observed that ‘‘it Delaney and John Arden
is the most insipid, ridiculous play I ever saw in
(b) John Osborne, Arnold Wesker, Harold Pinter
my life.’’
and Shelagh Delaney
(a) John Evelyn (b) Samuel Pepys
(c) John Osborne, Arnold Wesker, Antonin
(c) John Dryden (d) Robert Greene
Artaud and John Arden
Ans.(b): Samuel Pepys, commented after watching the (d) John Osborne, Harold Pinter, Shelagh
performance of William Shakespeare's play A Delaney and John Arden
Midsummer Night’s Dream that it is the most insipid
ridiculous play that ever saw in my life. Although Ans.(a): In the given options, option (a) John Osborne,
Samuel Pepys admits that it had ‘‘some good dancing Arnold Wesker, Shelagh Delaney and John Arden is the
and some handsome women, which all my pleasure’’. correct combination of playwrights who contributed to
the movement called Kitchen Sinks Drama.
Hence option (b) is correct answer.
• Kitchen Sink Drama is the name given to plays that
8. Name the playwright who wrote the play
depict the daily struggles of ordinary working-class
Epicoene, or the Silent Women?
people.
(a) William Congreve (b) Thomas Kyd
• Drama in this category often deal with social issues,
(c) Ben Jonson (d) Thomas Farquhar
such as poor living conditions, lack of employment,
Ans.(c): Ben Jonson, (Benjamin Jonson) a playwright, poverty, and turbulent relationships.
lyric poet and literary critic wrote the play Epicoene or
Silent Woman. Epicoene, or the Silent Woman,
• Kitchen Sink Drama was a British cultural
commonly referred to now simply as Epicene, is a movement in art and literature.
comedy by early modern English playwright Ben • John Osborne, Arnold Wesker and Shelagh Delaney
Jonson. It was originally performed in 1609 by The are some of the prominent writers.
Blackfriars' children but did not become popular until Hence option (a) is the correct answer.
many year, later in the 1660s after the restoration of the
English monarchy when Charles II assumed the crown. 11. Choose from the following options the correct
combination of the plays which made
Hence, option (c) is the correct answer.
significant use of expressionistic techniques.
9. Abhijnanasakuntalam makes use of the (a) The Hairy Ape, Machinal, All My Sons,
following four languages. Waiting for Lefty
(a) Sanskrit, Shauraseni, Tamil and Maharshtri (b) The Hairy Ape, The Crucible, Rapid Transit,
(b) Sanskrit, Shauraseni, Pali and Oriya The Adding Machine
(c) Sanskrit, Brajabuli, Maharashtri and Magdhi (c) The Hairy Ape, Rapid Transit, All My Sons,
(d) Sanskrit, Shauraseni, Maharashtri and Magdhi The Adding Machine
Ans.(d): Abhijnanasakuntalam is a drama by famous (d) The Hairy Ape, The Emperor Jones, Rapid
th
Indian author Kalidas, written about 5 century CE that Transit, The Adding Machine
NTA UGC NET/JRF English Paper II, (Dec 2022) Shift-I 418 YCT
Ans.(d): Combination of the plays : The Hairy Ape, The 14. Who wrote the popular instruction manual for
Emperor Jones, Rapid Transit, The Adding Machine is fishermen titled The Compleat Angler, or The
the correct combination which made significant use of Contemplative Man’s Recreation?
expressionistic techniques. Expressionism in literature (a) Issac Walton (b) Jeremy Taylor
arose as a reaction against materialism complacent (c) Richard Baxter (d) Thomas Hobbes
bourgeois prosperity rapid mechanization and Ans.(a): Isaac Walton wrote the popular instruction
urbanization. The two main principles are therefore : manual for fishermen titled The Compleat Angler, or
there are no laws and laws should not be imposed by The Contemplative Man's Recreation, first published in
anyone, more is painted from the feeling (the child) than 1653. A much enlarged edition appeared in 1655 and
from the ration (intelligence) in short, within the last edition supervised by the author, published in
expressionism the artist tries to shape his feelings his 1676, included additional material by Charles Cotton.
experiences by distorting/simplifying reality. This literary and Nature classic was created by a
Hence, the correct answer is option (d). Condoner with a passion for rustic life. As satisfying a
primer on fishing as any anger could wish, it celebrates
12. Apart from Bertolt Brecht others who the art and spirit of fishing with verse song and folklore,
influenced Epic theatre are. moral reflection, and timeless wisdom.
(a) Erwin Piscator and Antonin Artaud Hence, option (a) is the correct answer.
(b) Martin Esslin and Max Reinhardt 15. What did Matthew Arnold imply by the term
(c) Erwin Piscator and Max Reinhardt ‘‘Hebraism’’ in his Culture and Anarchy?
(d) Constantin Stanislavski and Max Reinhardt (a) Moral education (b) Intellectual autonomy
Ans.(c): Apart from Bertolt Brecht others who have (c) Rational outlook (d) Pragmatic attitude
influenced epic theatre are ‘Erwin Piscator and Max Ans.(a): Matthew Arnold implied Moral education by
Reinhardt’. Epic theatre is now most often associated the term ‘‘Hebraism’’ in his work Culture and Anarchy.
with the dramatic theory and practice evolved by the Hebraism is the lexical item usage or trait characteristic
playwright-director Bertolt Brecht in Germany from the of the Hebrew language. By successive extension, it is
1920s onward. Erwin Piscator is a theatrical producer often applied to the Jewish people, their faith, national
and director famed for his ingenious expressionistic ideology or culture. Finally, the word Hebraism
staging techniques. He was the originator of the epic describes a quality, character, nature, or method of
theatre style later developed by the German playwright thought or system of religion attributed to the Hebrew
Bertolt Brecht. Max Reinhardt, an eclectic as a people.
director, broke with those who favoured realism and Culture and Anarchy : An Essay in Political and Social
influenced Epic theatre and tried his hand at Symbolic Criticism is a series of periodical essays by Matthew
drama. Arnold, first, published in Cornhill Magazine (1867-68)
and collected as a book in 1869. The preface was added
Hence, the correct answer is option (c).
in 1869.
13. The theological treatise Ecclesiastical Polity Hence, option (a) is correct answer.
was written by________.
16. The Journal Scrutiny was founded in 1932 by.
(a) Richard Hakluyt (b) Francis Bacon
(a) I.A. Richards (b) F.R. Leavis
(c) Raphael Holinshed (d) Richard Hooker
(c) Cleanth Brooks (d) John Crowe Ransom
Ans.(d): The theological treatise Ecclesiastical Polity
Ans.(b): F.R. Leavis (Frank Raymond Leavis) is the
was written by Richard Hooker, who began to write his founder of the Journal Scrutiny in 1932. Scrutiny was a
major work of the laws of Ecclesiastical Polity, a quarterly journal of criticism that published until 1953
critique of the puritans and their attacks on the church and is regarded by many as his greatest contribution to
of England and particularly the book of common prayer. English letters. Always expressing his opinions with
Richard Hooker sets out to expand the Anglican severity, Leavis believed that literature should be
philosophy of government, in both civil and spiritual closely related to criticism of life, therefore a literary
matters. The essence of Anglicanism lays in the state critic's duty to assess works according to the authors
establishment of the supreme authority of the monarch. and the moral position of society moral position.
Hence, the correct answer is option (d). Hence, option (b) is correct answer.

NTA UGC NET/JRF English Paper II, (Dec 2022) Shift-I 419 YCT
17. Samuel Pepys’ claim to fame rests on his. 20. Which of the following novels is written by
(a) Biography (b) Autobiography Patrick White?
(c) Diary (d) Speculative Fiction (a) Cry, the Beloved Country
(b) The Vivisector
Ans.(c): Samuel Pepys (1633-1703) was an English
(c) The Handmaid’s Tale
diarist, and naval administrator, celebrated for his Diary
(first published in 1825) which gives a fascinating (d) The Stone Angel
picture of the official and upper-class life of restoration Ans.(b): Among the given novels, The Vivisector is
London from January 1, 1660 to May 31, 1669. Pepys' written by Patrick White, published in 1970.
Diary is one of the most important pieces of literature in • The novel enlightens the lifelong creative journey of
the history of England because it tells descriptive fictional artists/painter Hurtle Duffield.
information about the coronation of King Charles II,
• Cry, the Beloved Country (1948) is a novel by
detailed crucial events in history and outlined how South African writer Alan Paton.
th
people lived in mid-17 century England.
• The Handmaid’s Tale (1985) is a futuristic
18. Dr. Primrose is a character in. dystopian novel by Canadian author Margaret
(a) Nicholas Nickleby Atwood.
(b) Adam Bede • The Stone Angel (1964) is a novel by Canadian
(c) The Vicar of Wakefield writer Margaret Laurence.
(d) Joseph Andrews Hence, the correct answer is option (b).
Ans.(c): Dr. Primrose is a famous character in Anglo- 21. Which among the following is NOT written by
Irish writer Oliver Goldsmith's The Vicar of Wakefield Kamala Markandaya?
(1766). Dr. Primrose, is a rich man who lives in quiet (a) Some Inner Fury
neighbourhood with his family. He and his wife, (b) The Nowhere Man
Deborah, have two daughters, Olivia and Sophia, as (c) A Time to be Happy
well as four sons George, Bill, Moses and Dick. (d) The Golden Honeycomb
Hence, option (c) is correct answer. Ans.(c): A Time to be Happy (1958) is a novel by
19. ‘‘So the baby was carried in a small deal box, Nayantara Sahgal.
under an ancient woman’s shawl, to the • Some Inner Fury (1955), The Nowhere Man (1972),
churchyard that night, and buried by lantern- and The Goldan Honeycomb (1977) are authored by
light, at the cost of a shilling, and a pint of beer Kamala Markandaya.
to the sexton, in that shabby corner of God’s • Kamala Markandaya (pseudonym : Kamala
allotment where. He lets the nettles grow and Purnaiya; married name : Kamala Taylor) is an
where all unbaptized infants, notorious Indian novelist whose works concerns the struggle
drunkard, suicides and others of the of contemporary Indians with conflicting Eastern
conjecturally damned are laid.’’ and Western values.
From which novel is this excerpt taken? Hence, the correct answer is option (c).
(a) Wuthering Heights by Emily Bronte 22. Who among the following were revaluing ‘the
(b) Tess by Thomas Hardy masses’ as sources and subjects of literature?
(c) Great Expectation by Charles Dickens (a) Richard Hoggart and Raymond Williams
(d) Mill on the Floss by George Eliot (b) Penny Summerfield and Gillian Rose
Ans.(b): These above lines have been taken from ‘Tess’ (c) Rachel Alsop and Catherine Belsey
written by Thomas Hardy in 1979. A pure woman is (d) Jon Cook and Marilyn Deegan
faithfully presented in this novel, it initially appeared in Ans.(a): Richard Hoggart and Raymond Williams
censored and serialised version, published by the British are well known for Marxist criticism. According to
illustrated newspaper. Tess, attractive and innocent, is them, the ‘masses’ represents the group of common
seduced by dissolute Alec’ D'Urberville and secretly people who are not financially strong; they are just the
bears a child Sorrow, who dies in infancy. Later part of population having no significance.
working as a dairymaid, she meets and marries Angel • On the other hand there are some people who are
Clare, an idealistic gentleman who rejects Tess after affluent and considered to be the part of society in
listening of her past on their wedding night. reality.
Hence, option (b) is correct answer. Hence, the correct answer is option (a).
NTA UGC NET/JRF English Paper II, (Dec 2022) Shift-I 420 YCT
23. Who among the following theorists has written Ans.(b): Roland Barthes’ Image-Music-Text is a
on narrative empathy? collection of selected essays, published in 1977.
(a) Michael Ryan (b) Suzanne Keane Image-Music-Text brings together major essays by
(c) Toni Morrison (d) Sara Ahmed Barthes on the structural analysis of narrative and on
Ans.(b): Among the given theorists, only Suzanne issues in literary theory, on the semiotics of photograph
Keane has written on narrative empathy. and film, on the practice of music and voice.
• Narrative empathy is the feeling and perspective- Hence, the correct answer is option (b).
taking induced reading, viewing, hearing, or 27. Who among the following distinguished the
imagining narratives of another’s situation and ‘constatives’ from the ‘performatives’?
condition. (a) John Austin (b) Judith Butler
Hence, the correct answer is option (b). (c) Stanley Fish (d) Mary Louise Pratt
24. The science that systematically studies the Ans.(a): John Austin distinguished the ‘constative’
function of signs is known as. from the ‘performatives’.
(a) Semantics (b) Scriptoria
• According to the Austin, 'performative' is the term
(c) Sismography (d) Semiology
that ‘‘indicates that the issuing of the utterance is
Ans.(d): The science that systematically studies the the performing of an action – it is not normally
function of signs, is known as Semiology. thought of as just saying something.’’
• Semiotics, also called semiology, is the study of • In his How to Do Things With Words, Austin
signs and sign-using behaviour. pointed out that ‘‘it has come to be commonly help
• The Swiss linguist and one of the founders of that many utterances which look like statements
semiotics, Ferdinand de Saussure defined it as the are either not intended at all, or only intended in
study of ‘‘the life of signs within society’’. part, to record or impact straightforward
• Semantics is the study of the meaning of words, information about the facts’’.
phrases and sentences. Hence, the correct answer is option (a).
• Scriptoria (Scriptorium) is a room set apart for the 28. Who among the following coined the phrase
writing or copying of manuscripts. ‘soft capitalism’?
Hence, the correct answer is option (d). (a) Louis Althusser (b) Slavoj Zizek
25. Who among the following first framed a theory (c) Nigel Thrift (d) Terry Eagleton
of general hermeneutics? Ans.(c): The phrase 'soft capitalism' was coined by
(a) Wilhelm Dithey Nigel Thrift in his book The Rise of Soft Capitalism
(b) Ferdinand de Saussure (1998). It is a form of capitalism animated by the
intense and widespread circulation of theories of
(c) Friedrich Schleiermacher
capitalism.
(d) E.D. Hirsch
Hence, the correct answer is option (c).
Ans. (c) : Friedrich Schleiermacher first framed a
theory of general hermeneutics. 29. To whom is the term ‘thick description’
attribute?
• Hermeneutics is the branch of knowledge that deals
(a) John Storey (b) Clifford Geertz
with interpretation, especially of the Bible or
literary texts. (c) Deleuze and Guattari (d) James Clifford

• Modern hermeneutics includes both verbal and non- Ans.(b): The term ‘‘thick description’’ attributed to
verbal communication as well as Semiotics Clifford Geertz.
presuppositions, and pre-understandings. • Thick description is the term that Geertz used to
Hence, the correct answer is option (c). describe ethnography in one of the most famous
26. Roland Barthes’ Image-Music-Text was and influential anthropology texts in the second
published in. half of the twentieth century The Interpretation of
Culture (1973).
(a) 1968 (b) 1977
(c) 1979 (d) 1969 Hence, the correct answer is option (b).

NTA UGC NET/JRF English Paper II, (Dec 2022) Shift-I 421 YCT
30. The subject matter of Cultural Studies as it 34. Identify the work which has NOT been
emerged as a ‘proto-discipline’ in the 1960s, authored by the famous cultural critic
was predominantly exploration of________. Raymond Williams.
(a) High culture (b) Popular culture (a) The Country and the City
(c) Subaltern history (d) Contemporary politics (b) Cultural and Society
Ans.(b): The subject matter of cultural studies as it (c) Modern Tragedy
emerged as a ‘proto-discipline’ in the 1960s, was (d) The Making of the English Working Class
predominantly exploration of popular culture.
Ans.(d): Among the given options, The Making of the
31. What does the abbreviated term CCCS stand
English Working Class has been written by E.P.
for in the context of cultural studies?
Thompson, while The Country and the City (1973) has
(a) Consortium of Cooperative Culture and
been written by Raymond Williams. It analyses images
Society
of the country and the city in English literature since the
(b) Conference on Contemporary Culture and th
16 century. Culture and Society by Raymond
Society
Williams, explores how the notion of culture developed
(c) Centre for Contemporary Cultural Studies in Britain. Modern Tragedy (1966) is written by
(d) Conclave for Collective Consciousness and Raymond Williams. It is based on ideas and ideologies
Socialisation which have influenced the production and analysis of
Ans.(c): The abbreviated term CCCS stands for centre tragedy.
for contemporary cultural studies. This foundation was Thus, option (d) will be correct answer.
laid by Richard Hoggart. It is given another name
‘Birmingham School’. The initial goal was to challenge 35. Second Language is.
the cultural elitism of literary theory as well as the (a) The language second in importance in terms
positivism of British sociology creating an approach of the use of languages by a learner.
that had three components. (b) The language used by a second generation
Thus, option (c) will be correct answer. learner.
32. According to Stuart Hall, the concept (c) Any language other than the learner’s native
of______played a seminal role in Cultural language or mother longue.
Studies. (d) The language which is the learner’s native
(a) Interpellation (b) Dispositive language but not the mother tongue.
(c) Hegemony (d) Society Ans.(c): ‘Second language’ is any language other than
Ans.(c): According to Stuart Hall, the concept of the learner’s native language or mother tongue. For
‘hegemony’ played a seminal role in cultural studies. example : English remained his second language for the
According to Hall, the problem of hegemony calls for rest of his life, because his upbringing was done in the
an account of cultural and group formation as distinct environment of Hindi.
from their political and ideological construction. The
Thus, option (c) will be correct answer.
concept ‘hegemony’ was given by Antonio Gramsci.
Thus, option (c) will be correct answer. 36. When a learner’s second language learning
system seems to freeze or get stuck at some
33. The book Political Shakespeare : Essays in
more or less deviant stage, the phenomenon is
Cultural Materialism was jointly edited by.
known as.
(a) Gilles Deleuze and Felix Guattari
(a) Freeze
(b) Alau Sinfield and Jonathan Dollimore
(b) Fossilization
(c) Bill Ashscrof and Helen Tiffin
(c) Language Learning Blockade
(d) Theodor Adorno and Max Horkheimer
(d) Language Scaffolding
Ans.(b): The book Political Shakespeare : Essays in
Cultural Materialism was jointly edited by Alan Ans.(b): When a learner’s second language learning
Sinfield and Jonathan Dollimore. system seems to freeze or get stuck at some more or less
The second edition of this book includes all the essays deviant stage, that phenomenon is known as
of the first edition and two new chapters in this edition, ‘Fossilization’. It is the process in which incorrect
Jonathan Dollimore discusses current critical language becomes a habit and cannot easily be
approaches to questions of gender and sexuality. corrected.
Thus, option (b) will be correct answer. Thus, option (b) will be correct answer.

NTA UGC NET/JRF English Paper II, (Dec 2022) Shift-I 422 YCT
37. Integrative motivation refers to. Ans.(c): Among the given options, Shakespeare’s
(a) Language learning for immediate goals. Macbeth was translated into Bengali and directed by
(b) Language learning for practical goals. Girish Chandra Ghosh. Ghosh’s Bengali translation of
(c) Language learning for personal growth and Macbeth obviously embodies a forward looking
cultural enrichment. negotiation with western influence, rather than a
(d) None of these traditionalist disquiet, but it can hardly avoid the tangle
Ans.(c): Integrative motivation refers to language of tradition and modernity.
learning for personal growth and cultural enrichment. It Thus, option (c) will be correct answer.
is relevant to learning foreign languages. People who
immigrate to new countries are some examples of 41. Which of the following works of Browning are
people who may want to identify with the community pure dramas?
around them. A. Strafford
Thus, option (c) will be correct answer.
B. The Last Ride Together
38. In 1835, which of the following languages was
C. A Blot in the ’Scutcheon
replaced by English as the official language of
the East India Company? D. Pippa Passes
(a) Sanskrit (b) Persian E. Porphyria’s Lover
(c) Arabic (d) Hindi Choose the correct answer from the options
Ans.(b): In 1835, Persian language was replaced by given below:
English as the official language of the East India (a) A and C (b) B and E
Company. The British East India Company was
responsible for introducing English in the subcontinent (c) C and D (d) B and C
th
when they established their operations in the 17 Ans.(a): Among the given works of Browning,
century. Strafford and A Blot in the Scutcheon are pure dramas.
Thus, option (b) will be correct answer. Strafford, portrays the downfall and execution of Lord
39. Choose the correct option from the following Strafford, while A Blot in the Scutcheon is a tragedy in
options. blank verse written by Browning.
(a) The ‘‘Magna Carta’’ of Indian Education in Thus, option (a) will be correct answer.
the colonial period refers to ‘‘The Missionary
Clause 1698’’. 42. Who among the following poets have lived in
(b) The ‘‘Magna Carta’’ of Indian Education in Australia?
the colonial period refers to ‘‘Indian A. Judith Wright
Education Commission in 1882’’.
B. Yusef Komunyakaa
(c) The ‘‘Magna Carta’’ of Indian Education in
the colonial period refers to ‘‘Macaulay’s C. Thomas Kinsella
Minute 1835’’. D. T.S. Eliot
(d) The ‘‘Magna Carta’’ of Indian Education in Choose the correct answer from the options
the colonial period refers to ‘‘Wood’s given below:
Education Despatch of 1854’’.
(a) A and B (b) A, B and C
Ans.(d): The Magna Carta of Indian education in the
colonial period refers to Wood’s Education Despatch of (c) A, B and D (d) A and E
1854. It was the first declaration of British education Ans.(b): Among the given poets, Judith Wright, Yusef
policy for educating the masses at all levels.
Komunyakaa and Kath Walker are from Australia,
Thus, option (d) will be correct answer. while Thomas Kinsella was an Irish poet, editor and
40. Which of the following plays of William publisher. T.S. Eliot was an American poet, essayist,
Shakespeare was translated into Bengali and playwright and literary critic. Kath Walker’s another
directed by Girish Chandra Ghosh.
name was Oodgeroo Noonuccal.
(a) The Tempest (b) Hamlet
(c) Macbeth (d) King Lear Hence, option (b) will be correct answer.

NTA UGC NET/JRF English Paper II, (Dec 2022) Shift-I 423 YCT
43. In An Essay on Criticism, Pope. D. A Touch of Brightness
A. Analyses the causes of faulty criticism and E. Bravely Fought the Queen
praises the great critics of the past. Choose the correct answer from the options
B. Analyses the causes of faulty criticism and given below:
characteristics the good critic. (a) A and B (b) B and D
C. Analyses the structure of a good essay and (c) C and E (d) B and E
praises the great critics of the past. Ans.(b): Among the given options, Fire and the Rain
D. Analyses the structure of a good essay and and A Touch of Brightness are not written by Mahesh
suggest how such an essay could be Dattani, while rest of the works are written by Mahesh
converted into good criticism. Dattani. The Fire and the Rain is Karnad’s transcreation
in English published in 1998, while A Touch of
E. Analyses the merits of the poetry of
Brightness by Partap Sharma centres around Rukmini, a
Wordsworth and praises the great critics
girl sold to a brothel in Mumbai.
of the past.
Thus, option (b) will be correct answer.
Choose the correct answer from the options
given below: 46. Name the dramas which fall within the
category of the Theatre of the Absurd.
(a) A and B (b) A, B and C
A. The Birthday Party
(c) C, D and E (d) A and D
B. Endgame
Ans.(a): In An Essay on Criticism, Pope analyses the C. Mrs. Warren’s Profession
causes of faculty criticism and praises the great critics D. The Rhinoceros
of the past. He criticized the causes of faulty criticism
E. Riders to the Sea
and characteristics of good critic. He seeks to introduce
Choose the correct answer from the options
and demonstrate the ideals of poetry and teaches critics
given below:
how to avoid doing harm to poetry.
(a) A, B and E (b) B, C and D
Thus, option (a) will be correct answer.
(c) A, B and D (d) C, D and E
44. Which of the following are plays written by
Ans.(c): Among the given options, the play The
Harold Pinter? Birthday Party (1959) by Harold Pinter, Endgame by
A. Family voices Samuel Beckett and Rhinoceros (1959) by Eugene
B. A Moon for the Misbegotten Lonesco come under the category of the Theatre of the
Absurd. This genre was a Post-World War II
C. The Room
designation for particular plays of absurdist fiction
D. No Man’s Land written by a number of primarily European playwrights
E. Krapp’s Last Tape in the late 1950s.
Choose the correct answer from the options Thus, option (c) will be correct answer.
given below: 47. Which of the playwrights have been correctly
(a) A, B and E (b) B, C and D matched with their works?
(c) A, C and D (d) C, D and E A. William Wycherly – The Rivals
Ans.(c): Among the given plays, Family Voices, The B. Ben Jonson – Volpone, or the Fox
Room and No Man’s Land are written by Harold Pinter, C. William Congreve – The Country Wife
while A Moon for the Misbegotten was the last play of D. Aphra Behn – The Dutch Lover
Eugene O’Neill. Krapp’s Last Tape (1958) is a one-act E. Richard Sheridan – A School for Scandal
play in English by Samuel Beckett. The Room by Pinter Choose the correct answer from the options
is considered by critics, the earliest example of given below:
‘Comedy of Menace’. (a) C, D and E (b) B, C and D
Thus, option (c) will be correct answer. (c) A, C and E (d) B, D and E
45. Which of the following have NOT been written Ans.(d): Among the given options Volpone is a comedy
by Mahesh Dattani? play by English playwright Ben Jonson, while The Dutch
Lover is a play by Aphra Behn. The School for Scandal,
A. Dance Like a Man
comes under the category of Comedy of Manners. Rest of
B. Fire and the Rain the options are mismatched.
C. On A Muggy Night in Mumbai Thus, option (d) will be correct answer.

NTA UGC NET/JRF English Paper II, (Dec 2022) Shift-I 424 YCT
48. Choose the plays written by Lord Byron. Ans.(c): Confessions of an English Opium-Eater is an
A. Sardanapalus autobiographical account by Thomas De Quincey. It is
B. Hellas about his laudanum addiction and its effect on his life,
C. Cain while Autobiography and Suspiria De Profoundis are
written by De Quincey. Suspiria is a collection of
D. The Two Foscari : An Historical Tragedy
essays in the form of prose poems. Hudibras is a
E. The Cenci
vigorous satirical poem, written in a mock-heroic style
Choose the correct answer from the options by Samuel Butler.
given below:
Thus, option (c) will be correct answer.
(a) A, B and C (b) B, C and E
51. Which of the following works have been
(c) A, B and D (d) A, C and D
written by Thomas Carlyle?
Ans.(d): Sardanapalus is a historical play by Lord
A. Of Heroes and Hero-Worship
Byron. The fall of the Assyrian monarchy is the subject
matter of this play, while Cain is also a dramatic work B. The French Revolution
by Byron, published in 1821. The Two Foscari : An C. Of Human Bondage
Historical Tragedy is a verse play in five acts by Byron. D. The Hour and the Man
Hellas is a verse drama by P.B. Shelley written in 1821, E. Hudibras
while The Cenci is also a verse drama in five acts by
Choose the correct answer from the options
P.B. Shelley.
given below:
Thus, option (d) will be correct answer.
(a) A and B (b) A and C
49. John Bunyan authored the following.
(c) A, and D (d) A and E
A. The Pilgrim’s Progress
Ans.(a): Of Heroes and Hero-Worship and The French
B. Grace Abounding
Revolution are written by Thomas Carlyle, while Of
C. Short View Human Bondage (1915) is a novel by W. Somerset
D. The Holy War Maugham. The Hour and the Man is a historical
E. Thoughts on Education romance, written by Harriet Martineau and the last one
Choose the correct answer from the options Hudibras is a satirical poem, written in a mock-heroic
given below: style by Samuel Butler.
(a) A, B and C (b) A, C and D Thus option (a) will be correct answer.
(c) A, D and E (d) A, B and D 52. Which of the following books are written by
Ans.(d): The Pilgrim’s Progress, Grace Abounding and Julia Kristeva?
The Holy War are authored by John Bunyan, while A. Desire in Language : A Semiotic Approach
Short View and Thoughts on Education are not written to Literature and Art
by John Bunyan. The Pilgrim’s Progress tells the story
B. Illuminations
of a Christian and his journey from the city of
destruction to the celestial city. C. Syntax and Semantics
Grace Abounding is a Puritan Spiritual autobiography. D. La revolution du language poetique
Thus, option (d) will be correct answer. E. The Madwoman in the Attic
50. Which of the following works have been Choose the correct answer from the options
authored by Thomas De Quincey? given below:
A. Confessions of An English Opium Eater (a) A and C (b) B and A
B. The French Revolution (c) A and D (d) B and E
C. Hudibras Ans.(c): Desire in Language : A Semiotic Approach to
D. Autobiography Literature and Art and La Revolution du Language
E. Suspiria De Profoundis Poetique are written by Julia Kristeva, a Bulgarian –
Choose the correct answer from the options French Philosopher, literary critic, semiotician and
given below: feminist. The Madwoman in the Attic is a feminist work
by Sandra Gilbert and Susan Gubar.
(a) A, B and C (b) A, C and D
(c) A, D and E (d) A, B and D Thus, option (c) will be correct answer.

NTA UGC NET/JRF English Paper II, (Dec 2022) Shift-I 425 YCT
53. To which of the following theories New 55. Identify the correct pairs.
Historicism is indebted? A. Gabriel Garcia Marquez – The Feast of the
A. Marxism Goat
B. Formalism B. Jorge Luis Borges – The Autumm of the
C. Reader-response Theory Patriarch
D. Existentialism C. Salman Rushdie – The Enchantress of
E. Hermeneutics Florence
Choose the correct answer from the options D. E.L. Doctorow – Ragtime
given below:
E. A.S. Byatt – Possession
(a) A and C (b) B and A
Choose the correct answer from the options
(c) A and E (d) B and C
given below:
Ans.(a): The theories of New Historicism is indebted to (a) A, B and C (b) A, B and E
Marxism and Reader-Response theory. Marxism is a
social, economic and political philosophy that analyses (c) B, C and D (d) C, D and E
the impact of ruling class on the laborers, leading to Ans.(d): Mario Vargas Llosa – The Feast of Goat
uneven distribution of wealth and privileges in the (2000)
society.
• Gabriel Garcia Marquez – The Autumn of the
Reader-Response Theory is based on the assumption Patriarch (1875)
that a literary work takes place in the mutual
relationship between the reader and the text. • Salman Rushdie – The Enchantress of Florence
(2008)
Hence, option (a) is the correct answer.
• E.L. Doctorow – Ragtime (1975)
54. Which of the following statements are true
about cyberpunk? • A. S. Byatt – Possession (1990)
A. It is a kind of science fiction. Hence, option (d) is the correct answer.
B. It uses postmodernist techniques and 56. Which of the following books are written by
posthumanist themes.
Aravind Adiga?
C. Events in this novel usually take place
A. The Blue Bedspread
within the virtual reality.
D. It is a kind of fiction written using online B. Between the Assassinations
platforms. C. The House of the Blue Mangoes
E. The first cyberpunk was written by D. Last Man in Tower
Thomas Sterne. E. The White Tiger
Choose the most appropriate answer from the
Choose the correct answer from the options
options given below:
given below:
(a) A, B and D (b) A, C and D
(a) A, B and C (b) B, D and E
(c) A, B and C (d) A, C and E
(c) B, C and D (d) A, C and D
Ans.(c): Cyberpunk is a subgenre of science fiction in a
dystopian futuristic setting that tends to focus on a Ans.(b): Between the Assassinations, Last Man in
‘‘combination of low life and high tech (Technology)’’. Tower and The White Tiger are the books written by
It uses postmodernist techniques and Posthumanist Aravind Adiga.
themes. Events in this novel usually take place within Aravind Adiga (23 October, 1974) is an Indian writer
the virtual reality. and journalist. His debut novel The White Tiger, won
Much of cyberpunk is rooted in the New Wave science the 2008 Man Booker Prize. Adiga’s second book,
fiction movement of the 1960s and 1970s, when writers Between the Assassinations, was released in India in
like Philip K. Dick, Michael Moorcock, Rozer Zelazny,
2008 and in the US and UK in mid 2009. His third
John Bruner, J.G. Ballard etc. examined the impact of
book, Last Man in Tower, was published in UK in 2011.
drug culture, technology and the sexual revolution while
avoiding the Utopian tendencies of earlier science while, The Blue Bedspread by Raj Kamal Jha and The
fiction. House of Blue Mangoes written by David Davidar.
Hence, option (c) is the correct answer. Hence, option (b) is the correct answer.

NTA UGC NET/JRF English Paper II, (Dec 2022) Shift-I 426 YCT
57. Which among the following are correct? D. R.P. Blackmur – In Search of the New
A. JM Coetzee – South Africa Criticism
B. Margaret Atwood – Canada E. Allen Tate – The New Apologists for
C. Philip Roth – Australia Poetry
D. Orthan Pamuk – Turkey Choose the correct answer from the options
E. Graham Swift – New Zealand given below:
Choose the correct answer from the options (a) A, B and C (b) A, B and D
given below: (c) B, C and E (d) B, D and E
(a) A, B and C (b) A, B and D Ans.(a): J.C. Ransom – Criticism, Inc.
(c) B, C and E (d) B, D and E William Empson – Seven Types of Ambiguity.
Ans.(b): J. M. Coetzee – South Africa C. Brookes and R.P. Warren – Understanding Poetry.
Margaret Atwood – Canada
Cleanth Brooks – In Search of the New Criticism.
Philip Roth – America
Murray Krieger – The New Apologists for Poetry.
Orhan Pamuk – Turkey
Hence, option (a) is the correct answer.
Graham Swift – Britain
Hence, option (b) is the correct answer. 60. Thomas Rymer coined the term ‘poetic justice’
to imply the following.
58. Which among the following is correct in the
context of R.K. Narayan? A. The distribution of earthly rewards and
A. His The Guide is an open-ended novel. punishments in proportion to the virtue or
vice of the various characters
B. Most of the characters of his novels are
from elite section of society. B. Literary work governed by decorum and
C. He wrote a short story titled ‘‘The Marty’s morality
Corner.’’ C. Literary work guided by random ways
D. He received the Booker’s prize for his things often work out in the actual world
novel The Vendor of Sweets. D. The metaphysical nature of poetic
E. Malgudi is a real-life city in Karnataka. experience
Choose the correct options given below: E. The justification of poetry to be an integral
(a) A, B and C (b) A and C part of the Ideal republic
(c) A, C and D (d) A, C and E Choose the correct answer from the options
Ans.(b): R.K. Narayan was an Indian writer and given below:
novelist known for his work set in the fictional South (a) A and B (b) B and C
Indian town of Malgudi. He was a leading author of (c) C and D (d) D and E
early Indian literature in English along with Mulk Raj
Ans.(a): The term ‘‘Poetic Justice’’ was coined by the
Anand and Raja Rao. His The Guide is an open ended th
English literary critic Thomas Rymer in the 17
novel. He wrote a short story titled The Martyr’s
Corner. century, when it was believed that a work of literature
should uphold moral principles and instruct the reader
Hence, option (b) is the correct answer.
in correct moral behaviour. He wined the term to imply
59. Identify the correct pairs:
the distribution of earthly rewards and punishments in
A. J. C. Ransom – Criticism, Inc. proportion to the virtue or vice of the various characters
B. William Empson – Seven Types of and for the literary work governed by decorum and
Ambiguity morality.
C. C. Brooks and R.P. Warren – Hence, option (a) is the correct answer.
Understanding Poetry
NTA UGC NET/JRF English Paper II, (Dec 2022) Shift-I 427 YCT
61. Which among the following are written by D. Methodologies are concerned with the
Roland Barthes. perspectives one brings to bear on one’s
A. Allegories of Reading work.
E. Methods are concerned with the
B. Mythologies
perspectives one brings to bear one’s work.
C. The Pleasure of the Text Choose the correct option:
D. Some Versions of Pastoral (a) A, D and E (b) B, C and E
E. What is an Author? (c) A and C (d) B and C
Choose the correct options given below: Ans.(d): In the context of methods and methodologies
(a) B and C (b) A and C in literary research the following are true :
(c) A, C and D (d) A, C and E • Methods are concerned with how one conducts a
given piece of research.
Ans.(a): Roland Gerard Barthes and a French literary
theorist, essayist, philosopher, critic and semiotician. • Methodologies are concerned with the perspectives
His works engaged in the analysis of a sign systems, one brings to bear on one’s work.
mainly derived from western popular culture. Hence, option (d) is the correct answer.
Mythologies and The Pleasure of the Text are the works 64. According to the English Subject Centre
written by Roland Barthes while other works are written report, skills needed for postgraduate work in
by– English include:
• Allegories of Reading – Paul de Man. A. Searching skills in libraries
• Some Versions of Pastoral – William Empson B. Editorial skills
C. Bibliographic skills
• What is an Author? – Michel Foucault.
D. Peer management skills
Hence, option (a) is the correct answer.
E. IT skills
62. Which of the following are correctly matched. Choose the correct option:
A. Jonathan Culler – Culture and Society (a) A, C and D (b) B, C, D and E
B. Raymond Williams – Literary Theory : A (c) A, B and D (d) A, B, C and E
Very Short Introduction Ans.(d): According to the English subject centre report,
C. Terry Eagleton – Criticism and Ideology skills needed for postgraduate work in English
D. Walter Benjamin – Illuminations including–
E. Stanley Fish – The Implied Reader • Searching skills in libraries
Choose the correct option: • Editorial skills
(a) B, C and E (b) C and D • Bibliographic skills
(c) A, C and D (d) B, C and D • IT skills
Ans.(b): Terry Eagleton – Criticism and Ideology. Hence, option (d) is the correct answer.
Walter Benjamin – Illuminations 65. Which of the following are possible ways of
turning a topic into an argument?
These two matches are correct.
A. An argument for or against an existing
Other works are: critic (or critical position) in relation to the
Raymond Williams – Culture and Society author or group of works one is studying.
Jonathan Culler – Literary Theory : A Very Short B. An argument about the value of a new
Introduction. theoretical approach to a text or set of
texts.
Wolfgand Iser – The Implied Reader.
C. An argument about some historical or
Hence, option (b) is the correct. literary-historical aspect of literature.
63. Which among the following are true in the D. An argument showing how a particular
context of methods and methodologies in theme or concept is not all related to a
literary research? group of texts.
A. Both are identical in nature. E. An argument about the significance of only
well-known author/work.
B. Methods are concerned with how one
conducts a given piece of research. Choose the correct option:
C. Methodologies are concerned with how one (a) A, B and C (b) B, C, D and E
conducts a given piece of research. (c) A, C and D (d) A, B, C and D

NTA UGC NET/JRF English Paper II, (Dec 2022) Shift-I 428 YCT
Ans.(a): The possible ways of turning a topic into an Ans.(c): The correct matches are:
argument are :
List-I List-II
• An argument for or against an existing critic (or
A. Walt Whitman III. Beat! Beat! Drums!
critical position) in relation to the author or group of
works one is studying. B. A.D. Hope IV. Australia
• An argument about the value of a new theoretical C. Derek Walcott II. A Far Cry From Africa
approach to a text or set of texts. D. Allen Ginsberg I. Howl
• An argument about some historical or literary – 68. Match List-I with List-II.
historical aspects of literature.
List-I List-II
Hence, option (a) is the correct answer.
A. Come what come may, Time I. Othello
66. Match List-I with List-II.
and the hour runs through
List-I List-II the roughest day.
A. Sri I. The Old Playhouse and B. When sorrows come, they II. King Lear
Aurobindo Other Poems come not single spies, But
in battalions!
B. Kamala Das II. Calcutta :A Long Poem
C. I am a man more sinned III. Macbeth
C. P. Lal III. A Sheaf Gleaned in against than sinning.
French Fields
D. But I will wear my heart IV. Hamlet
D. Toru Dutt IV. Savitri upon my sleeve, For daws
Choose the correct answer from the options to peck at. I am not what I
am
given below:
(a) A-III, B-II, C-I, D-IV Choose the correct answer from the options
given below:
(b) A-IV, B-I, C-II, D-III
(a) A-I, B-III, C-II, D-I
(c) A-II, B-III, C-IV, D-I
(b) A-II, B-III, C-I, D-IV
(d) A-II, B-III, C-I, D-IV
(c) A-III, B-IV, C-II, D-I
Ans.(b): The correct matches are: (d) A-I, B-II, C-IV, D-III
List-I List-II Ans.(c): The correct matches are:
A. Sri Aurobindo IV. Savitri List-I List-II
B. Kamala Das I. The Old Playhouse and other A. Come what come may, Time and III. Macbeth
Poems the hour runs through the roughest
C. P. Lal II. Calcutta : A Long Poem day.

D. Toru Dutt III. A Sheaf Gleaned in French B. When sorrows come, they come not IV. Hamlet
single spies, But in battalions!
Fields
C. I am a man more sinned against II. King Lear
67. Match List-I with List-II.
than sinning.
List-I List-II
D. But I will wear my heart upon my I. Othello
A. Walt Whitman I. Howl sleeve, for daws to peck at. I am not
B. A.D. Hope II. A Far Cry From Africa
what I am.

C. Derek Walcott III. Beat! Beat! Drums! 69. Match List-I with List-II.
List-I List-II
D. Allen Ginsberg IV. Australia
A. You can’t eat the I. Man and
Choose the correct answer from the options
orange and throw the Superman
given below: peel away - man is not
(a) A-II, B-III, C-I, D-IV a piece of fruit.
(b) A-I, B-II, C-III, D-IV B. Liberty means II. A Doll’s House
(c) A-III, B-IV, C-II, D-I responsibility. That is
why most men dread it.
(d) A-III, B-II, C-I, D-IV
NTA UGC NET/JRF English Paper II, (Dec 2022) Shift-I 429 YCT
C. Nobody thinks, nobody III. Death of a Ans.(c): The correct match of List-I with List-II is :
cares. No beliefs, no Salesman (A)-(III), B-(I), C-(IV), D-(II)
convictions and no
enthusiasm. Just List-I List-II
another Sunday A. Anthropology I. Claude Levi-Strauss
evening.
B. Postmodern Geography II. Edward Soja
D. Our home has been IV. Look Back in
nothing but a Anger C. Diaspora Space III. Avtar Brah
playroom. It have been
D. Dasien IV. Martin Heidegger
your doll-wife, just as
at home I was papa’s Hence, option (c) is the correct answer.
doll-child; and here the
71. Match List-I with List-II.
children have been my
dolls List-I List-II
Choose the correct answer from the options A. The Poetics of Prose III. Wolfgang Iser
given below: B. Structuralist Poetics I. Tzvetan Todorov
(a) A-I, B-III, C-II, D-I
C. The Implied Reader IV. Stanley Fish
(b) A-II, B-III, C-IV, D-I
D. Is There a Text in II. Jonathan Culler
(c) A-I, B-IV, C-II, D-III
This Class?
(d) A-III, B-I, C-IV, D-II
Choose the correct answer from the options
Ans.(d): The correct matches are: given below:
List-I List-II (a) A-II, B-III, C-I, D-IV
A. You can’t eat the orange and III. Death of a (b) A-II, B-IV, C-I, D-III
throw the peel away, a man is Salesman (c) A-III, B-IV, C-II, D-I
not a piece of fruit. (d) A-III, B-II, C-I, D-IV
B. Liberty means responsibility. I. Man and Ans.(b): The correct match of List-I with List-Ii is :
That is why most men dread it. Superman (A)-(II), B-(IV), C-(I), D-(III)
C. Nobody thinks, nobody cares. IV. Look Back List-I List-II
No beliefs, no convictions and in Anger.
no enthusiasm. Just another A. The Poetics of Prose II. Tzvetan Todorov
Sunday evening. B. Structuralist Poetics IV. Jonathan Culler
D. Our home has been nothing but II. A Doll’s C. The Implied Reader I. Wolfgang Iser
a playroom. I have been your House
doll-wife, just as at home I was D. Is there a text in this class? III. Stanley Fish
Papa’s doll-child; and here the Hence, option (b) is the correct answer.
children have been dolls
72. Match List-I with List-II.
70. Match List-I with List-II.
List-I List-II
List-I List-II
A. Plato I. Rhetoric
A. Anthropology I. Edward Soja
B. Aristotle II. Symposium
B. Postmodern II. Martin Heidegger
C. P.B. Shelley III. Apology for
Geography
Poetry
C. Diaspora Space III. Claude Levi-Strauss
D. Philip Sidney IV. Defence of
D. Dasien IV. Avtar Brah Poetry
Choose the correct answer from the options Choose the correct answer from the options
given below: given below:
(a) A-III, B-II, C-I, D-IV (a) A-I, B-II, C-III, D-IV
(b) A-IV, B-I, C-II, D-III (b) A-III, B-II, C-IV, D-I
(c) A-III, B-I, C-IV, D-II (c) A-IV, B-III, C-II, D-I
(d) A-II, B-III, C-I, D-IV (d) A-II, B-I, C-IV, D-III
NTA UGC NET/JRF English Paper II, (Dec 2022) Shift-I 430 YCT
Ans.(d): The correct match of List-I with List-II is : (a) A-II, B-IV, C-I, D-III
(A)-(II), B-(I), C-(IV), D-(III) (b) A-II, B-I, C-IV, D-III
List-I List-II (c) A-IV, B-III, C-II, D-I
(d) A-IV, B-I, C-II, D-III
A. Plato II. Symposium
Ans.(d): The correct match of List-I with List-II is :
B. Aristotle I. Rhetoric
(A)-(IV), B-(I), C-(II), D-(III)
C. P.B. Shelley IV. Defence of Poetry
List-I List-II
D. Philip Sidney III. Apology for Poetry
A. Negative Capability IV. John Keats
Hence, option (d) is the correct answer.
B. Sweetness and Light I. Matthew Arnold
73. Match List-I with List-II.
C. Esemplastic II. Samuel Taylor Coleridge
List-I List-II
A. Bertrand I. The Verbal Icon D. Dissociation of III. T.S. Eliot
Russell Sensibility
B. Thomas II. The Well Wrought Urn Hence, option (d) is the correct answer.
Stearns Eliot
75. Match List-I with List-II.
C. W.K. Wimsatt III. History of Western
Philosophy List-I List-II

D. Cleanth Brooks IV. The Sacred Wood A. Munira I. Things Fall Apart

Choose the correct answer from the options B. Nnu Ego II. Petals of Blood
given below: C. Ikemefuna III. July’s People
(a) A-III, B-IV, C-I, D-II
D. Maureen IV. The Joys of Motherhood
(b) A-III, B-I, C-IV, D-II
Choose the correct answer from the options
(c) A-III, B-II, C-II, D-IV
given below:
(d) A-III, B-I, C-II, D-IV
(a) A-II, B-IV, C-I, D-III
Ans.(a): The correct match of List-I with List-Ii is : (b) A-III, B-II, C-IV, D-I
(A)-(III), B-(IV), C-(I), D-(II)
(c) A-IV, B-III, C-II, D-I
List-I List-II (d) A-III, B-IV, C-I, D-II
A. Bertrand Russell III. History of Western Ans.(a): The correct match of List-I with List-Ii is :
Philosophy
(A)-(II), B-(IV), C-(I), D-(III)
B. Thomas Stearns Eliot IV. The Sacred Wood
List-I List-II
C. W.K. Wimsatt I. The Verbal Icon
A. Munira II. Petals of Blood
D. Cleanth Brooks II. The Well Wrought
Urn. B. Nnu Ego IV. The Joys of Motherhood

Hence, option (a) is the correct answer. C. Ikemefuna I. Things Fall Apart

74. Match List-II with List-II. D. Maureen III. July’s People


List-I List-II Hence, option (a) is the correct answer.
A. ‘‘Negative I. Matthew Anrold 76. Choose the correct chronological sequence in
Capability’’ which the following texts were written.
B. ‘‘Sweetness II. Samuel Taylor Coleridge A. Lycidas
and Light’’ B. Hero and Leander
C. ‘‘Esemplastic’’ III. T.S. Eliot C. Masque of Comus
D. ‘‘Dissociation IV. John Keats D. Paradise Lost
of Sensibility’’ E. The Waste Land
Choose the correct answer from the options (a) A, B, D, E, C (b) B, C, A, D, E
given below: (c) B, A, E, C, D (d) B, E, D, C, A
NTA UGC NET/JRF English Paper II, (Dec 2022) Shift-I 431 YCT
Ans.(b): The correct chronological sequence of the 79. Find the chronological order of publication of
given texts according to their written year is : B, C, A, Charles Dickens' novels.
D, E. A. Oliver Twist
• Hero and Leander (1598)
B. Dombey and Sons
• Masque of Comus (1634)
C. Pickwick Papers
• Lycidas (1637)
D. Bleak House
• Paradise Lost (1667)
E. David Copperfield
• The Waste Land (1922)
Choose the correct answer from the options
Hence, option (b) is the correct answer.
given below :
77. Arrange the following poets in accordance with
(a) A, D, C, B, E (b) D, E, B, C, A
their years of birth.
A. Rudyard Kipling B. Robert Browning (c) B, D, C, A, E (d) C, A, B, D, E
C. John Masefield D. A.E. Housman Ans. (*): The correct chronological order of publication
E. John Donne of Charles Dickens' novels is : C, A, B, E, D.
Choose the correct answer from the options • Pickwick Papers (April 1836 to November, 1837)
given below:
• Oliver Twist (February, 1837 to April, 1839)
(a) E, A, B, D, C (b) E, B, A, C, D
• Dombey and Sons (October 1846 to April, 1848)
(c) E, B, A, D, C (d) A, D, B, C, E
• Bleak House (March 1852 to September, 1853)
Ans.(*): Note : NTA has dropped this question and
distributed equal marks to all the candidates. • David Copperfield (May 1849 to November, 1850)
The correct chronological sequence of the poets Dickens’ novels were initially serialised in weekly and
according to their years of birth is – E, B, D, A, C. monthly magazines, then reprinted in standard book
• John Donne (1572) formats.
• Robert Browning (1812) Note : NTA has dropped this question.
• A.E. Housman (1859) 80. Arrange the works in the chronological order
• Rudyard Kipling (1865) of the staging/publication of the following
• John Masefield (1878) plays.
78. Arrange the works in chronological sequence. A. A Woman Killed with Kindness
A. Rajmohan’s Wife B. John Bull’s Other Island
B. A Bend in the Ganges C. The Double Dealer
C. Kanthapura
D. The Shoemaker’s Holiday
D. Untouchable
E. The Conscious Lovers
E. Distant Drum
Choose the correct answer from the options
Choose the correct answer from the options
given below: given below:
(a) A, B, C, D, E (b) A, C, D, E, B (a) B, D, C, A, and E (b) D, A, C, E and B
(c) A, D, C, B, E (d) A, E, D, C, B (c) C, D, A, B and E (d) E, B, D, C and A
Ans.(c): The chronological sequence of the given works Ans.(b): The chronological order of the following plays
is : A, D, C, B, E. is : D, A, C, E and B.
• Rajmohan’s Wife (1864) by Bankim Chandra • The Shoemaker’s Holiday (1660), Thomas Dekker.
Chatterjee.
• A Woman Killed with Kindness (1607), Thomas
• Untouchable (1935) by Mulk Raj Anand.
Heywood.
• Kanthapura (1938) by Raja Rao.
• The Double Dealer (1694) by W. Congreve.
• A Bend in the Ganges (1964) by Manohar
Malgnokar. • The Conscious Lovers (1722) by Richard Steele.
• Distant Drum (1974) by Manohar Malgonkar. • John Bull’s Other Island (1904) by G.B. Shaw.
Hence, option (c) is the correct answer. Hence, option (b) is the correct answer.

NTA UGC NET/JRF English Paper II, (Dec 2022) Shift-I 432 YCT
81. Arrange the works in chronological sequence. Choose the correct answer from the options
A. The Theatre of Revolt by Robert Brustein given below:
B. The Theatre of the Absurd by Martin Esslin (a) E, B, D, C, A (b) E, A, C, B, D
C. The Playwright as Thinker by Eric Bentley (c) B, E, D, A, C (d) B, E, A, D, C
D. Modern American Drama by C.W.E. Bigsby
Ans.(d): The correct chronological sequence of the
E. Modern Drama in Theory and Practice by
L.N. Styan given theories is: B, E, A, D, C.
Choose the correct answer from the options • Psychoanlaysis by Sigmund Freud.
given below: • New Criticism by John Crowe Ransom.
(a) A, D, C, D and E (b) E, C, D, A and B
• Structuralism by Wilhelm Wundt.
(c) B, D, A, E and C (d) C, B, A, E and D
• Orientalism by Edward Said.
Ans.(d): The correct chronological sequence of the
given works is : C, B, A, E and D. • Ecocriticism by William Rueckert.
• The Playwright as Thinker by Eric Bentley (1946). Hence, option (d) is the correct answer.
• The Theatre of the Absurd by Martin Esslin (1961). 84. Arrange the works in the chronological
• The Theatre of Revolt by Robert Brustein (1964). sequence.
• Modern Drama in Theory and Practice by J.L. A. Matthew Arnold’s Culture and Anarchy
Styan (1981).
B. Thomas Browne’s The Anatomy of
• Modern American Drama by C.W.E. Bigsby
(2000). Melancholy
Hence, option (d) is the correct answer. C. Thomas Hobbes’ Leviathan
82. Choose the correct chronological sequence in D. Walter Pater’s Studies in the History of the
which the following texts were published. Renaissance
A. Madness and Civilization E. P.B. Shelley’s Defense of Poesie
B. The Archaeology of Knowledge Choose the correct answer from the options
C. The Language of the Self : The Function of given below:
Language in Psychoanalysis
(a) B, C, E, A, D (b) A, B, C, D, E
D. The Birth of the Clinic
E. Culture and Anarchy (c) C, D, E, A, B (d) D, C, B, A, E
Choose the correct answer from the options Ans.(a): The chronological sequence of given works
given below: and their writer is : B, C, E, A, D.
(a) E, B, D, C, A (b) E, A, C, B, D • Thomas Browne’s The Anatomy of Melancholy.
(c) E, B, D, A, C (d) C, A, B, D, E
• Thomas Hobbes’ Leviathan.
Ans.(b): The correct chronological sequence of the
given published work is : • P.B. Shelley’s Defense of Poesie.
• Culture and Anarchy (1869) by Matthew Arnold. • Matthew Arnold’s Culture and Anarchy.
• Madness and Civilization (1961) by Michel • Walter Pater’s Studies in the History of the
Foucault. Renaissance.
• The Language of the Self : The function of language Hence, option (a) is the correct answer.
in Psychoanalysis (1968) by Jacques Lacan.
• The Archaeology of Knowledge (1969) by Michel 85. Arrange the works in chronological sequence.
Foucault. A. Structuralist Poetics
• The Birth of the Clinic (1973) by Michel Foucault. B. Course in General Linguistics
Hence, option (b) is the correct answer. C. The Pursuit of Signs
83. Choose the correct chronological sequence in D. The Pleasure of the Text
which the following theories appeared.
E. The Implied Reader
A. Structuralism
B. Psychoanlyasis Choose the correct answer from the options
C. Ecocriticism given below:
D. Orientalism (a) A, C, D, E, B (b) B, D, E, A, C
E. New Criticism (c) C, D, A, B, E (d) D, E, A, C, B

NTA UGC NET/JRF English Paper II, (Dec 2022) Shift-I 433 YCT
Ans.(b): The correct chronological order of the given (a) Both Statement I and Statement II are true.
works is: B, D, E, A, C. (b) Both Statement I and Statement II are false.
• Course in General Linguistics (1916) by Ferdinand (c) Statement I is true but Statement II is false.
de Saussure. (d) Statement I is false but Statement II is true.
• The Pleasure of the Text (1973) by Roland Barthes. Ans.(a): In the light of the given statements, both the
• The Implied Reader (1974) by Wolfgang Iser. statement I and statement II are absolutely correct.
• Structuralist Poetics (1975) by Jonathan Culler. Hence, option (a) is the correct answer.
• The Pursuit of Signs (1981) by Jonathan culler. 89. Given below are two statements. One is labelled
Hence, option (b) is the correct answer. as Assertion A and the other is labelled as
86. Given below are two statements: Reason R.
Statement I : The Book The Theatre of Revolt Assertion (A) : Co-operative learning activities
was written by Robert Brustein are those in which students must work together
in order to complete a task or solve a problem.
Statement II : The Book The Theatre of Revolt
is written in the context of French Revolution. Reason (R) : These techniques are used to
identify a weak learner and to separate him/her
In the light of the statements given below from the rest of the members of the group for
choose the correct answer given below: taking special care of him/her.
(a) Both Statement I and Statement II are true
In the light of the above statements, choose the
(b) Both Statement I and Statement II are false most appropriate answer from the option given
(c) Statement I is true and Statement II is false below:
(d) Statement I is false and Statement II is true (a) Both (A) and (R) are correct and (R) is the
Ans.(c): In the light of the given statements, statement I correct explanation of (A).
is true and statement II is false. (b) Both (A) and (R) are correct and (R) is not
• The Theatre of Revolt was written by Robert the correct explanation of (A).
Brustein. (c) (A) is correct but (R) is not correct.
• In this book Brustein argues that the roots of the (d) (A) is not correct but (R) is correct.
Modern theatre may be found in the soil of rebellion Ans.(c): In the light of the given statements, statement
cultivated by eight outstanding playwrights : Ibsen, (A) is correct but (R) is not correct. The assertion is
Strindberg, Chekhov, Shaw, Brecht, Pirandello, correct but the reason is false.
O’Neill and Jean Genet.
• In Cooperative Learning, students work together in
Hence, option (c) is the correct answer. small groups on a structured activity.
87. Given below are two statements: Hence, option (c) is the correct answer.
Statement I : Constantin Stanislavski 90. Given below are two statements. One is labelled
collaborated with Anton Chekhov to stage as Assertion A and the other is labelled as
dramas on stage. Reason R.
Statement II : The book My Life in Art was Assertion (A) : In second language learning, in
written by Constantin Stanislvaski. the same classroom setting, some students
In the light of the statements given above progress rapidly through the initial stages of
choose the correct answer given below: learning a new language while others struggle
(a) Both Statement I and Statement II are true. making very slow progress.
(b) Both Statement I and Statement II are false. Reason (R) : Some learners never achieve a
(c) Statement I is true but Statement II is false. native-like command of a second language.
(d) Statement I is false but Statement II is true. In the light of the above statements, choose the
most appropriate answer from the option given
Ans.(b): In the light of the given statements, both the
below:
statement I and statement II are absolutely correct.
(a) Both (A) and (R) are correct and (R) is the
Hence, option (b) is the correct answer.
correct explanation of (A).
88. Given below are two statements. (b) Both (A) and (R) are correct and (R) is not
Statement I : Human bodies, like those of other the correct explanation of (A).
living organisms, are only ‘sexed’ from a (c) (A) is correct but (R) is not correct.
particular narrow perspective.
(d) (A) is not correct but (R) is correct.
Statement II : Most of the reproductions that
we undertake in our lifetimes has nothing to do Ans.(b): In the light of the given statements, both the
with ‘sex’. Assertion and Reason are correct but the Reason is not
In the light of the above statements, choose the the correct explanation of Assertion.
correct answer given below: Hence, option (b) is the correct answer.

NTA UGC NET/JRF English Paper II, (Dec 2022) Shift-I 434 YCT
Read the following poem and answer the question: Ans.(c): According to the given poem, the poet says
Talking in Bed that while lying in bed he and his companion pass time
Talking in bed ought to be easiest. silently.
Lying together there goes back so far. Hence, option (c) is the correct answer.
An emblem of two people being honest. 94. The poet and his companion are.
Yet more and more time passes silently. (a) In a hotel in the middle of a town.
Outside, the wind’s incomplete unrest (b) In a room of a hotel on the margin of the
Builds and disperses clouds about the sky. town.

Ana dark towns heap up on the horizon. (c) In the corridor of a hotel far away from the
towns.
None of this cares for us. Nothing shows why
(d) In a place away from the towns.
At this unique distance from isolation
Ans.(d): According to the given poem Talking in Bed
It becomes still more difficult to find
by Philip Larkin, the poet and his companions are in a
Words at once true and kind.
place away from the towns.
Or not untrue and not unkind.
Hence, option (d) is the correct answer.
Philip Larkin
95. The poet says that while lying in bed with one’s
91. Which of the following statement is true? companion it is difficult to find words which
(a) The poet says that talking in bed is very easy. are.
(b) The poet says that talking in bed is not very (a) At once honest and caring
easy. (b) At once true and unkind
(c) The poet says that talking in bed should be (c) At once pure and impure
easy but it is not.
(d) At once honest and touching
(d) The poet says that talking in bed can never be
Ans.(a): According to the given poem, Talking in Bed
easy.
by Philip Larkin, the poet says that while lying in bed
Ans.(c): According to the given poem Talking in Bed with one’s companion it is difficult to find words which
by Philip Larkin, the poet says that talking in bed are at once honest and caring.
should be easy but it is not.
Hence, option (a) is the correct answer.
Hence, option (c) is the correct answer.
Read the following passage and answer the questions
92. The poet says that when two people are lying that follows:
together, they look like.
Poetry in its use of language continually distorts and
(a) Two pure human beings. denies the structure of reality to exalt the structure
(b) Two hypocrites. of the self. By means of rhyme, assonance or
(c) Two innocent fellows. alliteration it couples together words which have no
(d) None of these. rational connection, that is, no nexus through the
world of external reality. It breaks the word up into
Ans.(a): According to the given stanzas of the poem by lines of arbitrary length, cutting across their logical
Philip Larkin, the poet says that when two people are construction. It breaks down their associations,
lying together, they look like two pure human beings. derived from the world of external reality, by means
Hence, option (a) is the correct answer. of inversion and every variety of artificial stressing
93. The poet says that while lying in bed he and his and counterpoint. Thus the world of external reality
companion pass time. recedes and the world of instinct, the affective
emotional linkage behind the words, becomes the
(a) By talking between themselves.
world of reality....... In the novel, too, the subjective
(b) By observing the trees outside the window.
elements are valued for themselves, and rise to view,
(c) Silently. but in a different way. The novel blots out external
(d) By playing cards. reality by substituting a more or less consistent
NTA UGC NET/JRF English Paper II, (Dec 2022) Shift-I 435 YCT
mock reality which has sufficient ‘stuff’ to stand (a) The implication is that the reality of fiction
between the reader and reality. This means that in has no existence independent of the words,
the novel the emotional associations attach not to and our emotional responses are directed by
words but to the moving current of mock reality the words.
symbolised by the words. This is why rhythm, (b) The implication is that the reality of fiction is
‘preciousness’, and style are alien to the novel: why not dependent on the words, and our affective
the novel translate so well : why novels are not states are not triggered by the words.
composed of words. They are composed of scenes, (c) The mock reality subverts the external reality
actions, stuff, people, just as play are. so obtrusively that readers become conscious
96. The above passage, Christopher Caudwell’s of the writer’s strategy.
statement. ‘‘Poetry in its use of language (d) The novel is different from poetry in the
continually distorts and denies the structure of sense that it is metonymic in its mode of
reality to exalt the structure of the self’’ linear progression, while poetry is metaphoric
implies: relying on subject-privileging.
(a) The pragmatic function of poetry that reflects
Ans.(a): Caudwell’s statement clearly shows that the
the social reality through expressive
reality of fiction has no existence independent of the
language.
words, and our emotional responses are directed by the
(b) The capacity of poetry to draw attention to words. It means that we cannot stay in the world of
itself as an aesthetic object or artifact. imagination for a long time, we have to face the stark
(c) Poetry exalts the ‘‘structure of the self’’ by reality of life.
privileging the notion of the ‘egotistical Thus, option (a) will be correct answer.
sublime’.
99. What do you understand by ‘‘mock reality’’ in
(d) The mimetic function of poetry that alludes to
context of the usage in the above passage?
the world of external reality in simple, clear
(a) The reality contrived into existence by
language.
novelists through strategic use of words.
Ans.(b): The capacity of poetry to draw attention to
(b) The reality evoked through figurative devices.
itself as an aesthetic object or artefact. Somewhere, it is
closely associated to the principles of Wordsworth and (c) The quasi reality effected through the use of
aesthetic beauty with the help of poetry, we can poetic devices.
perceive all that abstract things which are impossible to (d) The reality which is approximate to the
experience with being poetic. external reality.
Thus, option (b) will be correct answer. Ans.(a): According to the passage, ‘mock-reality’
97. What does the word ‘‘assonance’’ mean? means something that is not genuine, sham, counterfeit
or forgery. It is a kind of reality hard to believe or we
(a) Repetition of identical or similar consonants.
can say it pretention.
(b) Repetition of identical or similar vowels.
Thus, option (a) will be correct answer.
(c) Repetition of identical of similar phrases.
100. If rhythm, ‘preciousness’ and style are alien to
(d) Repetition of identical or similar clauses.
the novel, in which genre are they distinctive
Ans.(b): ‘Assonance’ is used as a figure of speech, features?
where we can see the repetition of identical or similar
(a) Drama (b) Poetry
vowels. The repetition created by assonance enhances
(c) Prose (d) Non-fiction
our writing with rhythm, mood and emphasis.
Thus, option (b) will be correct answer. Ans.(b): If rhythm, ‘preciousness’ and style are alien to
the novel, that will come in the genre of poetry.
98. What does Caudwell imply by the statement,
Alongwith these things voice, diction, imagery,
‘‘The novel blots out external reality by
symbolism, syntax, meter, allegory and structure are the
substituting a more or less consistent mock
basic elements of poetry.
reality which has sufficient ‘stuff’ to stand
Thus, option (b) will be correct answer.
between the reader and reality’’?
NTA UGC NET/JRF English Paper II, (Dec 2022) Shift-I 436 YCT
NTA UGC NET/JRF Exam. Dec. 2022
ENGLISH- II
SOLVED PAPER [01 March, 2023 Shift- II]

1. "What needs my Shakespeare for his honoured Ans. (c) : The Princess : A Medley by Tennyson is a
bones. narrative poem, published in 1847. It is a serio-comic
The labour of an age in piled stones? blank verse narrative poem. It tells the story of a heroic
Or that his hallowed reliques should be hid princess who tries to find her place but finds it hard.
Under a star-ypointing pyramid?" Hence, option (c) will be correct answer.
These lines are written by 5. Which of the following works is NOT written
(a) Ben Jonson by P.B. Shelley?
(b) John Milton (a) The Mask of Anarchy
(c) Robert Browning (b) Queen Mab : A Philosophical Poem
(d) William Wordsworth (c) The Vision of Judgement
Ans. (b) : These given lines are taken from On (d) The Revolt of Islam
Shakespeare, composed by John Milton. The poem is Ans. (c) : Among the given options, option (c) The
basically an epitaph where Milton has tried to honour Vision of Judgement is not written by P.B. Shelley. It is
Shakespeare and his literary achievements. This verse is a satirical poem in Ottava Rima by Lord Byron. It was
written in the form of heroic couplet. the critical rebuttal to Robert Southey's poem A Vision
Hence, option (b) will be correct answer. of Judgement which gives a optimistic perspective of
the death of king George III, while The Mask of
2. Who among the following praised Chaucer's
Anarchy, Queen Mab : A Philosophical Poem, and The
translation of Roman de La rose?
Revolt of Islam are written by Shelley.
(a) Eustache Deschamps
Hence, option (c) will be correct answer.
(b) Boccaccio
6. In whose poem the readers meet Aunt
(c) Jean de Meun
Jennifer's tigers?
(d) Guillaume de Lorris
(a) Thom Gunn
Ans. (a) : Eustache Deschamps, a leading French author (b) Kamau Brathwaite
praised Chaucer's translation Roman de la Rose. It is
(c) Roy Fisher
considered as a medieval poem written in old French.
(d) Adrienne Rich
Past of the story was translated from its original old
French into middle English as The Romaunt of the Rose, Ans. (d) : In Adrienne Rich's poem, the readers meet
which had a great influence on English literature. Aunt Jennifer's tigers. Aunt Jennifer's Tigers (1951) is a
poem by an American poet Adrienne Rich. The poem is
Hence, option (a) will be correct answer.
based on marriage, gender and power, the poem
3. For the Unfallen is a book of poems written by describes the terrified Aunt Jennifer's fear-filled
(a) Ted Hughes (b) Sylvia Plath existence in a marriage full of ordeals, in which she is
(c) Geoffrey Hill (d) A.E. Housman ruled over by her husband.
Ans. (c) : For the Unfallen (1959) is a collection of Hence, option (d) will be correct answer.
poems by Geoffrey Hill. It contains 29 poems which are 7. Name the playwright who has written Larins
: Genesis, God's Little Mountain, The Bidden Guest, In Sahib.
Memory of Jane Fraser Holy Thursday, The Turtle (a) Gieve Patel (b) Dina Mehta
Dove, Solomon's Mine, The Troublesome Reign, (c) Gurcharan Das (d) Pratap Sharma
Armodeus etc.
Ans. (c) : Larins Sahib is a historical play set in the
Hence, option (c) will be correct answer. confused period after the death of Ranjit Singh when
4. "The Princess : A Medley" by Tennyson is the British first arrived in Punjab. Through this works,
(a) a lyric Gurcharan Das has tried to depict the crisis that
(b) an elegy overtook the state, when East India Company routed the
(c) a narrative poem Sikhs, seven years after the death of Maharaja.
(d) a dramatic monologue Hence, option (c) will be correct answer.

NTA UGC NET/JRF English Paper II, (Dec 2022) Shift-II 437 YCT
8. Who among the following has composed the Ans. (d) : Sentimental comedy was prevalent in the
lyrical drama Hellas? 18th. century, denoting plays in which middle-class
(a) Lord Byron (b) P.B. Shelley protagonists triumphantly overcome a series of moral
(c) William Wordsworth (d) John Keats trials. Such comedy aimed at producing tears rather than
laughter. The best example of sentimental comedy is
Ans. (b) : Hellas is a lyrical drama by P.B. Shelley. It The Conscious Lovers (1722) by Richard Steele which
was inspired by the Greek insurrection against Turkey deals with the trials and tribulations of its penniless
in the 1820s, and the action is seen from the Turkish heroine Indiana. Oliver Goldsmith's She Stoops to
sultan Mahmud's point of view. The first chapter looks Conquer (1773) and R.B. Sheridan's The Rivals are
at the sources of Hellas and how three major writers written in this genre. Thus, option (d) will be correct
Aeschylus, Milton, and Calderon influenced Shelley in answer.
his works. 12. "I recognize that its heroine is a little prig and
Hence, option (b) will be correct answer. its hero a pompous ass, but I do not care."
9. Name the playwright who composed the play A About which novel of Jane Austen is this
Woman Killed with Kindness. statement made by Somerset Maugham?
(a) Pride and Prejudice (b) Northanger Abbey
(a) Francis Beaumont
(c) Sense and Sensibility (d) Mansfield Park
(b) Beaumont and Fletcher
Ans. (d) : This particular statement by Somerset
(c) Thomas Kyd
Maugham is a comment on Jane Austen's Mansfield
(d) Thomas Heywood Park. It focuses on the life of Fanny Price and her
Ans. (d) : Thomas Heywood wrote the play A Woman experience living at Mansfield Park, where she has been
Killed with Kindness in the early seventeenth century. sent due to the financial problems of her family. The
The plot of the play is derived from an Italian novel by themes of the novel are the development and collapse of
Illumine, which was translated into English and social relationships, slavery and morality
published in The Palace of Pleasure. Hence, option (d) will be correct answer.
Hence, option (d) will be correct answer. 13. In which year was R.L. Stevenson's Treasure
Island published?
10. Name the celebrated actor who played the
(a) 1893 (b) 1886
leading role in the first production of John
(c) 1883 (d) 1896
Osborne's The Entertainer (1957).
Ans. (c) : Treasure Island is an adventure novel by R.L.
(a) Peter Brook (b) Laurence Olivier
Stevenson. It was published in 1883. It was originally
(c) Al Pacino (d) Robert De Niro titled The Sea Cook : A Story for Boys, telling a story of
Ans. (b) : Laurence Olivier was the celebrated actor buccaneers and buried gold. The plot of the novel is set
who played the leading role in the first production of in the mid – 18th century when an old sailor who
John Osborne's The Entertainer (1957). The Entertainer identifies himself as the captain.
is a British kitchen sink drama film directed by Tony Hence, option (c) will be correct answer.
Richardson. 14. Which of the following novels is NOT written
Hence, option (b) will be correct answer. by Amitav Ghosh?
11. Which of the following statements holds true in (a) Sea of Poppies (b) Flood of Fire
regard to "Sentimental Comedy"? (c) Gun Island (d) English, August
(a) It is a dramatic composition which satirises Ans. (d) : Among the given options, English, August:
the manners and affectations of a class. An Indian Story is a novel by Indian author Upamanyu
Chatterjee, while Sea of Poppies (2008) is a novel by
(b) It is a dramatic composition that focuses on
Amitav Ghosh which was shortlisted for the Man
characters, each of them representing a type Booker Prize in 2008. Flood of Fire and Gun Island are
personality. written by Amitav Ghosh.
(c) It is a dramatic composition that depicts how Hence, option (D) will be correct answer.
seriously young people take love, and how 15. Which among the following is NOT an
foolishly it makes them behave. American Slave narrative?
(d) It is a species of dramatic composition in the (a) The Interesting Narrative of the Life of
virtues of private life are exhibited, rather Oloudah Equiano
than the vices exposed; and the distresses (b) Life and Adventure of Henry Bibb
rather than the faults of mankind make our (c) Narrative of Soloman Northup
interest in the piece. (d) Songs of Enchantment
NTA UGC NET/JRF English Paper II, (Dec 2022) Shift-II 438 YCT
Ans. (d) : Soloman Northup was an American Ans. (a) : Among the given options, Ernest Laclau can
abolitionist and the author of Twelve Years a Slave be considered as the Post-Marxist critic, while Antonio
which is a kind of slave-narrative; The Life and Gramsci was an Italian Marxist philosopher. Theodor
Adventures of Henry Bibb is among the most W. Adorno was a German philosopher, sociologist and
remarkable slave narratives. The Interesting Narrative composer; while Walter Benjamin was a German
of the life of Olaudah Equiano is the autobiography of Jewish philosopher, cultural critic and essayist.
Olaudah Equiano, based on slave-narrative; while Songs Hence, option (a) will be correct answer.
of Enchantment is based on quests and struggle for 20. Jonathan Bate's The Song of the Earth was
equanimity. published in
Hence, option (d) will be correct answer. (a) 2001 (b) 1991
16. Who among the following theorists has written (c) 2000 (d) 1999
on affective economies? Ans. (c) : Jonathan Bate's The Song of the Earth was
(a) Franco Moretti (b) Bruno Latour published in 2000. It is based on our growing alienation
(c) Sara Ahmed (d) Nigel Thrift from Nature. He traces the distinctions among nature,
Ans. (c) : Sara Ahmed has written on affective culture and environment and shows how their
economies. In this theory, emotions circulate between appearance was in the literature of the eighteenth
bodies, signs and objects, mediating the relationship century.
between the psychic (individuals), the material (bodies Thus, option (c) will be correct answer.
and objects) and the social (communities) and binding 21. Who among the following is NOT a member of
subjects together. the Frankfurt School?
Hence, option (c) will be correct answer. (a) Louis Althusser (b) Max Horkheimer
17. Who among the following is NOT a Yale critic? (c) Theodor Adorno (d) Herbert Marcuse
(a) Geoffrey Hartman (b) Paul de Man Ans. (a) : Louis Althusser is not a member of the
(c) J. Hillis Miller (d) Roland Barthes Frankfurt school instead he was a French Philosopher
and belongs to western marxism, structural marxism
Ans. (d) : Yale Critic is a shorthand way of referring to
and Neo-spinozism.
a moment in the 1970s, when the work of Jacques
The Frankfurt School was a group of scholars known
Derrida was taken up and experimented with by four
for developing critical theory and popularizing the
prominent literary critics in the department of English at
dialectical method of learning by interrogating the
Yale : Paul de Man, J. Hillis Miller, Geoffrey
contradictions of society. Among the given options,
Hartmann and Harold Bloom; while Roland Barthes is a
Max Horkheimer, Theodor Adorno and Herbert
French literary theorist known for Structuralism. Marcuse belong to the Frankfurt School.
Hence, option (D) will be correct answer. Thus, option (a) will be correct answer.
18. 'Demythologizing' is a term associated with the 22. Which of the following methods is the oldest for
works of teaching English language?
(a) Claude Levi Strauss (a) The Bilingual Method
(b) Ferdinand de Saussure (b) The Grammar Translation Method
(c) Rudolph Bultmann (c) The Direct Method
(d) Friedrich Schleirmacher (d) The Situation Method
Ans. (c) : 'Demythologizing' is a term associated with Ans. : (b) The oldest method for teaching English
the works of Rudolph Bultmann. He claimed that the language is the Grammar-Translation Method (classical
biblical three-storeyed universe, belief in angels etc was method). The method has two main goals to enable
incredible in the modern world and that the Gospel students to read and translate literature written in the
message could be freed from these stumbling blocks. source language.
Though he insisted that he was interpreting rather than • The most relevant principles of this method are:
eliminating myth, his slogan came to be attacked to • It emphasis on grammatical explanation and
various reductionist interpretations of Christianity. translation of a language pattern.
Hence, option (c) will be correct answer. • In this method the rule is often memorized and
19. Who among the following is known as a post- subsequently cited to explain a similar situation.
Marxist thinker? • The mother tongue becomes the medium of
(a) Ernest Laclau (b) Antonio Gramsci instruction of teaching.
(c) Theodor Adorno (d) Walter Benjamin Thus, option (b) will be correct answer.

NTA UGC NET/JRF English Paper II, (Dec 2022) Shift-II 439 YCT
23. In the area of theory and research known as Ans. (b) : The Indian Education Commission under the
Second Language Acquisition (SLA), the chairmanship of Dr. D.S. Kothari submitted its report
classroom is considered on 29th June 1966. The aim of this commission was
(a) an ideal model for research. promoting regional languages, as well as international
languages preferably English.
(b) a site of no use
Hence, option (b) will be correct answer.
(c) a site that always produces language learning
27. Who was the Chairman of the University
blockade.
Education Commission of 1948?
(d) an experimental laboratory. (a) Babu Rajendra Prasad
Ans. (d) : In the area of theory and research known as (b) D.S. Kothari
Second Language Acquisition (SLA), the classroom is (c) Maulana Abul Kalam Azad
considered as an experimental laboratory. It helps (d) S. Radhakrishnan
students how to be more observant and inquisitive with Ans. (d) : Dr. S. Radhakrishnan was the Chairman of
the help of experiments. They learn how to ask the University Education Commission of 1948. The
questions and find new ways how they can answer a commission made a number of significant
question or solve problems. recommendations on various aspects of higher
Thus, option (d) will be correct answer. education and submitted its report in August, 1949.
Thus, option (d) will be correct answer.
24. The arrival of corpus linguistics has revitalized
28. Which of the following is the first newspaper of
(a) the writing of observation-based grammar. India?
(b) the writing that does not care for grammar. (a) Hicky's Bengal Gazette
(c) the use of long sentences in newspaper (b) Jhones' Calcutta Gazette
reporting. (c) William's Indian Gazette
(d) the quality of newspaper reporting. (d) Salisbury's Madras Gazette
Ans. (a) : The arrival of corpus linguistics has Ans. (a) : Hicky's Bengal Gazette is the first newspaper
revitalized the writing of observation - based grammar. of India. It was founded in Calcutta, the capital of
The Corpus linguistics has generated a number of British India at that time, by Irishman James Augustus
research methods which attempt to trace a path from Hicky in 1779.
data to theory. Thus, option (a) will be correct answer.
Thus, option (a) will be correct answer. 29. "The great object of the British Government
ought to be the promotion of European
25. Which of the following is one of the DON'Ts of literature and science among the natives of
writing a dissertation or thesis : India, all funds appropriated for the purpose of
(a) Compiling a bibliography as soon as the work education would be best employed on English
is started. education alone." Who made the comment
(b) Avoid jargon wherever possible. given above?
(c) It should be kept in mind that a dissertation or (a) Lord Macaulay (b) Sir Charles Wood
a thesis should take the form of an argument (c) Lord William Bentinck (d) Arthur Mayhew
in which the writer must attempt to convince Ans. (c) : The comment is the part of Macaulay's
the reader of his or her case. Minutes for Education in India. Lord Macaulay came to
India in June 10, 1834, as the law member of the
(d) A researcher should not bother about the use Governor General's Executive council and was
of the proper scholarly conventions from the appointed as the president of the committee of public
very beginning. instruction, but these words are uttered by Lord William
Ans. (d) : Among the given options, option (d), a Bentinck who was the governor at the time and was
researcher should not bother about the use of the proper responsible for the education system in India.
scholarly conventions from the very beginning is very Thus, option (c) will be correct answer.
close to the answer otherwise he/she will not be able to 30. Todd Kachru in "Three Circles of English"
reveal the unknown facts during research work. observes that English speaking countries are
Thus, option (d) will be correct answer. separated into three groups. Choose the correct
answer from the options given below :
26. In which year, did the 'Indian Education (a) Central Circle, Middle Circle and Peripheral
Commission' (The sixth commission in the Circle
history of Indian Education) under the (b) Primary Circle, Secondary Circle and Tertiary
chairmanship of Dr. D.S. Kothari submit its Circle
report? (c) Inner Circle, Outer Circle and Expanding
(a) 1956 (b) 1966 Circle
(c) 1976 (d) 1986 (d) Inner Circle, Middle Circle and Outer Circle
NTA UGC NET/JRF English Paper II, (Dec 2022) Shift-II 440 YCT
Ans. : (c) Todd Kachru's Three Circles of English 33. Which of the following is true in the context of
observes that English speaking countries are separated New Criticism?
into three groups : the Inner Circle, the Outer Circle, (a) If follows the tradition of Historical Criticism.
and the Expanding Circle. These circles represent the (b) The main law of New Criticism is that it
patterns of acquisition and the functional domains in should be subjective analysis.
which English is used across cultures and languages. (c) The distinctive procedure for a New Critic is
• Inner circle represents native English speakers: USA, explication.
UK, Canada, Australia, New Zealand. (d) The distinction between literary genres does
play an essential role in New Criticism.
• Outer circle represents the countries which are
colonized by Britain and English language has a fair Ans. (c) : New- Criticism insisted on the intrinsic value
of a work of art and focused attention on the individual
share in official works (in offices) as: India, Pakistan,
work alone as an independent unit of meaning. It is a
Bangladesh, Kenya, Sri Lanka, Singapore etc..
style of criticism that emphasizes the close reading of
• Expanding circle represents the countries which use texts as a self-contained piece of work, capable of
their native language for official purpose but they learn producing independent meaning, without the
and use English language for corporate relates works accompaniment of any philosophical, historical or
/business and for foreign communication. They are: biographical context surrounding the text. The
China, Japan, Nepal, North & South Korea, Israel, distinctive procedure for a new critic is explication.
Taiwan etc.. Thus, option (c) will be correct answer.
Thus, option (c) will be correct answer. 34. The pamphlet The Power of Love (1643),
31. Which one of the following is false about V.S. proclaiming the importance of brotherhood as
Naipaul? a means of achieving a radical change in social
(a) He was awarded the Nobel Prize in Literature relationships was written by _________.
in 2001. (a) George Saintsbury (b) William Walwyn
(b) He won the Booker Prize for his novel A (c) F.R. Leavis (d) Gerrard Winstanley
House for Mr. Biswas. Ans. (b) : The pamphlet The Power of Love (1643),
(c) He contributed his stories to the Trinidad proclaiming the importance of brotherhood as a means
Guardian. of achieving a radical change in social relationships was
written by William Walwyn. It appeared in London in
(d) He won the Somerset Maugham Award for
September 1643.
his Miguel Street.
Thus, option (b) will be correct answer.
Ans. (b) : Among the given options, option (b) is false
35. Francis Bacon's The Advancement of Learning
because V.S. Naipaul won the Booker Prize in 1971 for attempeted to draw a distinction between two
his novel In a Free State, he got the Nobel Prize in kinds of 'truth'. Which are these?
literature in 2001 and won the Somerset Maugham (a) Theological Truth and Scientific Truth
Award for work Miguel Street. (b) Theological Truth and Aesthetic Truth
Thus, option (b) will be correct answer. (c) Aesthetic Truth and Objective Truth
32. Which one of the following is false? (d) Metaphysical Truth and Aesthetic Truth
(a) An ideal literary researcher must be an Ans. (a) : Francis Bacon's The Advancement of
insatiable reader. Learning attempted to draw a distinction between two
(b) An ideal literary researcher should not cast kinds of truth, first is theological truth and another is
himself back into another age. scientific truth. According to him, the study of nature
(c) An ideal literary researcher should came to be less about changing traditional attitudes and
comprehend the current attitude or the artistic beliefs and more about stimulating the economy.
assumptions. Thus, option (a) will be correct answer.
(d) An ideal literary researcher must have a vivid 36. "I shall be ambitious to have it said of me, that
sense of history. I have brought Philosophy out of Closets and
Libraries, Schools and Colleges, to dwell in
Ans. (b) : Among the given options, option (B) is not
Clubs and Assemblies, at Tea-Tables, and in
true that an ideal literary researcher should not cast
Coffee-Houses."
himself back into another age, because it is obvious that
To whom do you attribute this famous
a researcher has to go and peep into the past age for
statement?
comparative study.
(a) Dr. Samuel Johnson (b) Joseph Addison
Thus, option (b) will be correct answer. (c) Charles Lamb (d) Alexander Pope
NTA UGC NET/JRF English Paper II, (Dec 2022) Shift-II 441 YCT
Ans. (b) : This particular statement is attributed to Ans. (c) : Raymond Williams coined the notion
Joseph Addison. It is all about the trends going on in "structure of feeling" in the 1970s to facilitate a
18th and early 19th century. The importance was given to historical understanding of affective elements of
rationality rather than emotions in this age. consciousness and relationships. Since then, the need to
Thus, option (b) will be correct answer. understand emotions, moods and atmospheres as
37. Dr. Johnson's Dictionary of the English historical and social phenomena has only become more
Language was published in ________. acute in an era of social networking.
(a) 1751 (b) 1753 Thus, option (c) will be correct answer.
(c) 1755 (d) 1757 41. "A Valediction Forbidding Mourning" is
Ans. (c) : Dr. Johnson's Dictionary of the English written by
th
Language was published on 15 . April 1755 until the A. John Donne
completion of the Oxford English Dictionary. 173 years B. John Milton
later, it was viewed as the pre-eminent English C. Adrienne Rich
dictionary. D. Sylvia Plath
Thus, option (c) will be correct answer. E. Robert Frost
38. An Essay on the Principles of Human Action Choose the correct answer from the options
was written by _______. given below :
(a) Charles Lamb (a) A and C only. (b) A and B only.
(b) Jean Jacques Rousseau (c) D and E only (d) B and C only
(c) William Godwin Ans. (a) : A Valediction Forbidding Mourning is a
(d) William Hazlitt poem by Adrienne Rich where she talks about
Ans. (d) : An Essay on the Principles of Human Action expressing herself through the frozen language of
was written by William Hazlitt. This work has been others; with the same title, John Donne wrote and used
selected by scholars as being culturally important and is one of his famous conceits to depict the steadfast nature
part of the knowledge base of civilization. of his love. It was written for Donne's wife Anne.
Thus, option (d) will be correct answer. Thus, option (a) will be correct answer.
39. Which of the following is false about Frederick 42. Which of the following two poems are linked
Douglass? with each other in terms of form?
(a) Douglass's autobiography belongs to the A. "The Last Ride Together"
tradition of fugitive-slave narrative popular in B. "Ulysses"
the North before the Civil War. C. "Upon Appleton Houses: To my Lord Fairfax"
(b) He provides a first-person account of his life D. "To Penshurst"
spent in slavery. E. "The Waste Land"
(c) He was famous as an orator, dedicated to a Choose the correct answer from the options
black liberation movement. given below :
(d) He wrote Up From Slavery. (a) A and E only (b) A and B only
Ans. (d) : Frederick Douglas was an American social (c) A and D only (d) C and D only
reformer, abolitionist, orator and writer. He worked Ans. (d) : Upon Appleton House is composed by
tirelessly to make sure that emancipation would be one Andrew Marvell. It has been analyzed into six sections,
of outcomes of war. He recruited African - American the poem is written in 97 stanzas, each of eight lines
men to fight in the U.S. Army, including two of his own that are octosyllabic in iambic tetrameters forming
sons. After escaping from slavery in Maryland, he couplets.
became a national leader of the abolitionist movement To Penshurst by Ben Jonson talks about the ancestral
in Massachusetts and New York. estate Penshurst place, which comprises the building,
Thus, option (d) will be correct answer. gardens and forests, belonging to Sir Philip Sidney in
40. The phrase "structure of feeling" is attributed the same way as Ben Jonson did in Upon Appleton
to __________. Houses: To my Lord Fairfax.
(a) Lauren Berlant Hence, option (d) is correct.
(b) Terry Eagleton 43. Which of the following poems are written by
(c) Raymond Williams Alexander Pope?
(d) Eve Kosofsky Sedgwick A. The Dunciad
NTA UGC NET/JRF English Paper II, (Dec 2022) Shift-II 442 YCT
B. Moral Essays An Experiment with Truth is a play by Asif Currimbhoy
C. Grongar Hill on the life and assassination of Mahatma Gandhi and
D. Cooper's Hill Angkor is a TV play by Asif Currimbhoy.
E. Absalom and Achitophel Thus, option (b) will be correct answer.
Choose the correct answer from the options 46. Which of the following are the plays written by
given below : Robert Greene?
(a) A and C only (b) B and E only A. The Famous Chronicle of King Edward the
(c) A and B only (d) C and D only First
Ans. (c) : The Dunciad is a landmark, mock-heroic B. Alphonsus
narrative poem by Alexander Pope, published in three C. A Moon for he Misbegotten
different versions at different times from 1728 to 1743; D. The Old Wives' Tale
Moral Essays is a series of four poems on ethical E. King of Aragon
subjects by Alexander Pope while Grongar Hill is Choose the correct answer from the options
located in the Welsh Country of Carmarthenshire and given below :
was the subject of loco-descriptive poem by John Dyer. (a) B and D only (b) A and E only
Absalom and Achitophel is a satirical poem by John (c) B and E only (d) C and E only
Dryden.
Ans. (c) : Alphonsus, King of Aragon (1588-91) is a
Hence, option (c) will be correct answer. play by Robert Greene, while A Moon for the
44. Identify the plays originally written by Vijay Misbegotten is a play by Eugene O'Neill. The Old
Tendulkar from the following : Wives' Tale and King Edward the First are written by
A. The Cyclist and His Fifth Woman George Peele.
B. Scandal in Fairyland Thus, option (c) will be correct answer.
C. The Vultures and Encounter in Umbugland 47. Which of the following two plays were written
D. Sakharam Binder by W.B. Yeats?
E. Fire and the Rain A. The Land of heart's Desire
Choose the correct answer from the options B. Time and the Conways
given below : C. The Silver Tassie
(a) B, C and E only (b) A, B and C only D. The Countess Cathleen
(c) A, C and D only (d) A, B and D only E. The Plough and the Stars
Ans. (c) : The Fire and the Rain is a play by Girish (a) C and D only (b) A and D only
Karnad, based on the myth of Yavakri ; while Sakharam (c) A and E only (d) B and E only
Binder is a play by Vijay Tendulkar written in Marathi. Ans. (b) : The Land of Heart's Desire is a play by Irish
The Cyclist, His Fifth Woman, The Vultures and poet and dramatist W.B. Yeats, while The Silver Tassie
Encounter in Umbugland are written by Vijay is a four-act Expressionist play about the first World
Tendulkar. War, written between 1927 and 1928 by Sean O'Casey.
Thus, option (C) will be correct answer. The Countess Cathleen is a verse drama by W.B. Yeats
45. Identify the plays written by Asif Currimbhoy in blank verse, while The Plough and the Stars is a four-
from the following : act play by Sean O'Casey.
A. princes Hence, option (b) will be correct answer.
B. The Captives 48. Which among the following are true in the
C. An Experiment with Truth context of Chinua Achebe?
D. Angkor A. He wrote Arrow of God and Things Fall Apart.
E. Dance Like a Man B. His "Novelist as Teacher" is a seminal essay in
Choose the correct answer from the options the context of African Literature.
given below : C. The name of the tribe he depicted in Things
(a) A, B and D only (b) B, C and D only Fall Apart is lgbo
(c) A, B and C only (d) A, B and E only D. He is a Kenyan born American literature.
Ans. (b) : Asif Currimbhoy, an Indian playwright wrote E. He wrote the essay "An Abolition of English
The Captives, An Experiment with Truth and Angkor. Department."
The Captives is a social play that depicts the relation Choose the correct answer from the options
between the Indian Muslim and Pakistani Muslim while given below:
NTA UGC NET/JRF English Paper II, (Dec 2022) Shift-II 443 YCT
(a) A, C and D only (b) A, B and E only great figures of their history while The French
(c) A, B and C only (d) B, C and D only Revolution is a narrative history in three volume,
Ans. (c) : Arrow of God and Things fall Apart are the written by Carlyle ; Of Human Bondage (1915) is a
novel by W. Somerset Maugham and The Hour and the
novels by Nigerian author Chinua Achebe, Things Fall
Man is a collection of mystery story by Robert Barr.
Apart is the debut novel of Achebe, first published in
Hudibras is a vigorous satirical poem written in mock
1958. Achebe's The Novelist as Teacher shows how to
heroic style by Samuel Butler and published in three
build confidence in his people that makes them as proud
parts in 1663. 64 and 1678.
of their culture as any other. The name of the tribe
More than one work is not written by Thomas Carlyle
Achebe depicted in Things fall Apart is Igbo.
hence, NTA has dropped this question.
Thus, option (c) will be correct answer.
51. Which of the following works have NOT been
49. Identify the correct ones among the following : authored by John Stuart Mill?
A. The apologie for poetrie was written by Sir A. Subjection of women
Philip Sidney. B. Thoughts on Parliamentary reform
B. Sir Philip Sidney wrote the Apologie for C. Past and Present
Poetrie as a counterblast to Stephen Gosson's D. Explorations
The School of Abuse. E. On Liberty
C. Stephen Gosson wrote the school of abuse in Choose the correct answer from the options
the euphuistic style. given below :
D. Sidney's style was characterised by neoclassical (a) A and B only (b) A and E only
restraint. (c) C and D only (d) B and E only
E. Sidney and Gosson wrote their critical treatise Ans. (c) : Among the given Options, Past and Present
in the eighteenth century. (1843) is written by Thomas Carlyle; while
Choose the correct answer from the options Explorations is not written by John Stuart. The
given below : Subjection of Women is written by John Stuart Mill,
(a) A, B and C only (b) A, C and D only published in 1869. It was ahead of its time in boldly
(c) A, D and E only (d) A, C and E only championing feminism. Thoughts on parliamentary
Reform is associated with John Stuart Mill, and his book
Ans. (a) : Sir Philip Sidney's An Apology for Poetry
On Liberty 1859 considered as the most popular work
also known as The Defence of Poetry was written in
of Mill, which shows the ethical system of utilitarianism
1579-80. It was a response to Stephen Gosson's School
to society and state.
of Abuse." It is about the role of society. For Sidney,
Thus, option (c) will be correct answer.
poetry is not merely a part of civilization, it is a
civilized and civilizing art form. To Sidney, poetry is an 52. Which among the following are true in the
context of literary research?
art of imitation for specific purpose, it is imitated to
teach and delight. Poetry is simply a superior means of A. It is devoted to the enlightenment of criticism
communication and its value depends on what is B. It seeks to illuminate the work of art as it really
is
communicated.
C. It has no connection with the proffered
Thus, option (a) will be correct answer.
information
50. Which of the following works have NOT been D. It tries to see the writer as s/he really was
written by Thomas Carlyle? Choose the correct answer from the options
A. Of heroes and hero-worship given below :
B. The French Revolution (a) A, C and D only (b) B, C and D only
C. Of human bondage (c) A, B and D only (d) A, B and C only
D. The hour and the man Ans. (c) : Statement A, B, and D are absolutely correct
E. Hudibras in the context of literary research.
Choose the correct answer from the options • Literary research is devoted to the enlightenment of
given below : criticism ; it seeks to illuminate the work of art as it
(a) A and B only (b) A and C only really is ; it tries to see the writer as she/he really was ;
(c) A and D only (d) A and E only It has connection with the proffered information.
Ans. (*) : Of Heroes and Hero-Worship, written by • Acquisition of information within a specific literary
Thomas Carlyle is based on the high regard, entirely work is referred to as literary research.
proper in his view, that ordinary people have for the Hence, the correct answer is option (c).

NTA UGC NET/JRF English Paper II, (Dec 2022) Shift-II 444 YCT
53. Which among the following are false in the 55. Which among the following are written by
context of autographical research? Mikhail Bakhtin?
A. These can be accepted on face value. A. White Mythology
B. These are usually idealized. B. Freudianism : A Marxist Critique
C. These are coloured by compelling motive of the C. The ideology of the aesthetics
desire for self-justification. D. Rabelais and His world
D. These are embroidered through the sheer E. Morphology of the Folktale
exuberance of the artistic imagination.
Choose the correct answer from the options
Choose the correct answer from the options given below :
given below :
(a) A and C only (b) B and D only
(a) A, C and D only (b) B, C and D only
(c) A and E only (d) B and C only
(c) A, B and D only (d) A, B, C and D only
Ans. (*) : From the given Books, Freudianism : A
Ans. (b) : The false/incorrect statement in reference to
Marxist Critique and Rabelais and His World are
autobiographical research is B, C and D.
written by Mikhail Bakhtin.
• Autobiographical researches are not idealized. Such
• Freudianism : A Marxist Critique (1976) is co-
kind of researchers are not embroidered through the
sheer exuberance of the artistic imagination instead authored by Valentine Valoshinov and translated by I.R.
these kind of researches are completely based on the Titunik. Rabelais and His World (1965) is a literary
facts of science. criticism by Bakhtin which points on a close reading of
novels of 16th century French writer Francois Rabelais.
• These can be accepted on face value.
other works and their writers are:
• Hence, the correct answer is option (b).
• White Mythology (1971), Derrida ; The Ideology of the
54. Which of the following are the key attributes
Aesthetics (1990), Terry Eagleton ; Morphology of the
and skills required in the context of tools and
Folktale (1928), Vladimir Propp.
techniques for literary research?
A. Having an overview of the main online and None of the given option has the correct combination
printed sources relevant to the research. hence, NTA has dropped the question.
B. Not participating in any online information 56. Which of the following are correctly matched?
networks as others may copy the work. A. John Keble- On the Healing Power of Poetry
C. Getting to know a range of available online B.Carl G. Jung- Writing and Difference
sources, and being able to evaluate these C. Jacques Derrida- Modern Man in Search of a
sources comparatively. Soul
D. Using online and printed sources to identify D. Harold Bloom-The Anxiety of influence
and locate material archives. E. Kate Millett- Jacques Lacan : A Feminist
E. Peer management technique. Introduction
Choose the correct answer from the options Choose the correct answer from the options
given below : given below :
(a) A, B and C only (b) B, C and E only (a) B and C only (b) A and D only
(c) A, C and D only (d) A, B, C and D only (c) A and E only (d) B and D only
Ans. (c) : The key attributes and skills required in the
Ans. (*) : Only (D) is correctly matched.
context of tools and techniques for literacy research are:
• The Anxiety of Influence (1973) written by Harold
• One should have an overview of the main online and
Bloom is correctly matched.
printed sources relevant to the research.
• Writing and Difference (1967) is a work by Jacques
• One should get to know a range of available online
Derrida.
sources, and become able to evaluate these sources
comparatively. • Modern Man in Search of a Soul (1933) is a book of
• One should use online and printed sources to identify psychological essays written by Swiss psychologist Carl
and locate material archives. Jung.
• No participation in any online information network • Jacques Lacan : A feminist Introduction (1990) is a
for the sake of the work being copied is never book of feministic view written by Elizabeth Grosz.
appreciated and it's not an attribute of literary research. None of the given options has the correct combination
Hence, the correct answer is option (c). hence, NTA has dropped the question.

NTA UGC NET/JRF English Paper II, (Dec 2022) Shift-II 445 YCT
57. Gilles Deleuze and Felix Guattari's Anti- perspectives in writing history : the global and the
Oedipus : Capitalism and Schizophrenia fuses planetary.
two theoretical paradigms. They are Hence, the correct answer is option (c).
A. Marxism 59. Which of the following two books have their
B. Poststructuralism roots in Foucauldian thoughts on sexuality?
C. Psychoanalysis A. Metahistory
D. Feminism B. Gender Trouble : Feminism and the
Choose the correct answer from the options Subversion of Identity
given below : C. The Role of the Reader
(a) A and C only (b) A and B only D. Epistemology of the Closet
(c) A and D only (d) B and C only E. Sexual Politics
Ans. (d) : Gilles Deleuze and Felix Guattari's Anti Choose the correct answer from the options
Oedipus: Capitalism and Schizophrenia fuses two given below :
theoretical paradigm they are Poststructuralism and (a) B and C only (b) A and D only
Psychoanalysis. (c) A and C only (d) B and D only
Published in 1972, Anti- Oedipus is authored by French
Ans. (d) : Gender Trouble : Feminism and the
philosopher Gilles Deleuze and psychoanalyst Felix Subversion of Identity (1990, second edition 1999) by
Guattari. Judith Butler and Epistemology of the Closet (1990) by
• In volume one of Anti - Oedipus (1972), they drew on
Eve Kosofsky Sedgwick have their roots in Foucauldian
Lacanian (Jacques Lacan, French Psychoanalyst) ideas thoughts on sexuality.
to argue that traditional psychoanalytic conceptions of
• Epistemology of the Closet deals with the question of
the structure of personality are used to supress and what makes up human sexuality.
control human desire and indirectly to perpetuate the
• Michel Foucault expresses his thought in his book The
capitalist system.
History of Sexuality, in which the author examiner the
Hence, the correct answer is option (d). emergence of "sexuality" as a discursive object and
58. Which of the two following books are written separate sphere of life and argues that the notion that
by Dipesh Chakrabarty? every individual has a sexuality is a relatively recent
A. Provincializing Europe : postcolonial thought development in western societies.
and historical difference Hence, the correct answer is option (d).
B. The Subaltern Studies Reader 60. Which of the following are written by George
C. Identity and Violence Peele?
D. The climate of history in a planetary age A. The Famous Chronicle of King Edward the
E. In other worlds first
Choose the correct answer from the options B. A Moon for the Misbegotten
given below : C. The Arraignment of Paris
(a) A and C only (b) A and B only D. The Scottish History of James the Fourth
(c) A and D only (d) B and E only E. The Old Wives' Tale
Ans. (c) : Provincializing Europe : Postcolonial Choose the correct answer from the options
Thought and Historical Difference (2000), and The given below :
Climate of History in a planetary Age (2021) are the (a) B, C and D only
works written by Indian historian who has also made his (b) A, C and E only
contribution to postcolonial theory and subaltern (c) A, C and D only
studies, Dipesh Chakrabarthy. (d) C, D and E only
• Provincializing Europe explores how post-colonial
Ans. (b) : The Famous Chronicle of King Edward the
thinking impacts on the social science. It is set in the First (1593), The Arraignment of Paris (1584), and The
intersection between subaltern studies and postcolonial Old Wives' Tale (1595) are written by English poet,
theory. dramatist and translator George Peele.
• Dipesh Chakrabarty's The Climate of History in a • A Moon for the Misbegotten (1947) is written by
Planetary Age argues that we must combine two American playwright Eugene O'Neil.

NTA UGC NET/JRF English Paper II, (Dec 2022) Shift-II 446 YCT
• The Scottish Historie of James the Fourth (1598) is Ans. (b) : The novels The Grapes of Wrath (1939), East
written by Robert Greene. of Eden (1952), and Cannery Row (1945) are written by
• One of the intellectuals of the Elizabethan theatrical American writer and 1962 Nobel Prize (in Literature)
community, George Peele associated with Robert winner John Steinbeck.
Greene and other writers known as "University wits". • Novel The Naked and the Dead (1948) is written by
Hence, the correct answer is option (b). Norman Mailer.
61. Which among the following are true in the • To Kill a Mockingbird is a novel by Harper Lee.
context of Gabriel Garcia Marquez? • So, the correct combination of Steinbeck's novels is :
A. He was a Caribbean novelist, short-story B, C, E.
writer, screenwriter, and journalist. Hence, the correct answer is option (b).
B. He was affectionately known as Gabo or 63. Which of the following are the leading
Gabito throughout Latin America. characters in the novels written by Mulk Raj
C. He received the Nobel Prize in Literature in Anand?
1982. A. Munoo B. Bakha
D. Love in the Time of Cholera is a memoir C. Sampath D. Gangu
written by Marquez. E. Dopidi
E. His novel One Hundred Years of Solitude was (a) B, C and D only (b) A, C and E only
published in 1967. (c) C, D and E only (d) A, B and D only
Choose the correct answer from the options Ans. (d) : The leading characters in the novels written
given below : by Mulk Raj Anand from the given options are :
(a) A, C and D only (b) B, C and D only Munoo, Coolie (1936); Bakha, Untouchable (1935);
(c) B, C and E only (d) A, C and E only Gangu, Two Leaves and A Bud (1937).
Ans. (c) : Gabriel Garcia Marquez is a Colombian • The correct combination of Mulk Raj Anand's
novelist and one of the greatest writers of the 20th characters is A, B, and D.
century • Hence, the correct answer is option (d).
• He was affectionately known as Gabo or Gabito 64. Identify the correct pairs :
throughout Latin America. A. Ewan McEwan-Amsterdam
• He was awarded the Nobel Prize for Literature in B. Ialo Calvino- If on a Winter's Night A
1982, Mostly for his masterpiece Cien anos de Soledad Traveller
(1967) in English, One Hundred Years of solitude C. Amitav Ghosh- The Circle of Reason
(1970). D. D.M. Thomas- Everest Hotel
• He was the fourth Latin American to be honoured E. Doris Lessing - The Testaments
with the Nobel Prize preceded by Gabriela Mistral Choose the correct answer from the options
(1945), Pablo Neruda (1971), and Miguel Angel given below :
Asturias (1967). So, the correct combination for the (a) A, B and C only (b) A, B and E only
question is : B, C, E. (c) B, C and D only (d) C, D and E only
Hence, the correct answer is option (b). Ans. (a) : The correct pairs of writer and their works is
62. Which of the following are the novels written "A", "B", and "C".
by John Steinbeck? • Amsterdam is a novel written by Ian Mc Ewan is
A. The Naked and the Dead 1998.
B. The Grapes of wrath • If on a Winter's Night A Traveller (1979) is written by
C. East of Eden Italo Calvino.
D. To kill a Mockingbird • The Circle of Reason (1986) is a novel written by
E. Cannery row Amitav Ghosh
Choose the correct answer from the options • The Testaments is written by Margaret Atwood in
given below : 2019. It is sequel to the Handmaid's Tale.
(a) A, C and D only (b) B, C and E only • The Everest Hotel is authored by Allan Sealy in 1998.
(c) C, D and E only (d) A, D and E only • Hence, the correct answer is option (a).

NTA UGC NET/JRF English Paper II, (Dec 2022) Shift-II 447 YCT
65. Identify the correct pairs : C. Nissim Ezekiel II. The Unfinished Man
A. Aristotle- Rhetoric (1959)
B. Quintilian- Oratorical Institutions D. Imtiaz Dharker I. Purdah and other
C. C. Brooks and R.P. Warren - Understanding Poems. (1988)
Fiction Hence, the correct answer is option (c).
D. Allen Tate- The Verbal Icon 67. Match List I with List II
E. Harold Bloom- The great tradition List-I List-II
Choose the correct answer from the options A. Emily Dickinson I. Woman to Man
given below : B. Kath Walker II. Banking potatoes
(a) A, B and C only (b) A, B and D only C. Judith Wright III. Because I could not
(c) B, C and E only (d) B, D and E only Stop for Death
Ans. (a) : The correct pairs of books and their writers is D. Yusef komunyakaa IV. We are Going
A, B, and C. Choose the correct answer from the options
• Aristotle's Rhetoric is an ancient Greek treatise on the given below :
art of persuasion, dating from the 4th century BCE. (a) A-II, B-III, C-I, D-IV
• Latin teacher and writer, Quintilian's work on rhetoric, (b) A-I, B-II, C-III, D-IV
Institution Oratorio, is a major contribution to (c) A-III, B-II, C-I, D-IV
educational theory and literary criticism. (d) A-III, B-IV, C-I, D-II
• Understanding Fiction (1943) is a collection of short Ans. (d) : The correct match of the writers with their
stories and literary criticism authored by Cleanth works is (A)-III, (B)-IV, (C)-I, (D)-II.
Brooks and edited by Cleanth Brooks and Robert Penn • Emily Dickinson's Because I could not Stop for
Warren. Death, published posthumously in 1890, is an
• The Verbal Icon by William K. Wimsatt is series of exploration of both the inevitability of death and
essays between (1941 - 1952). uncertainties that surround what happen when people
actually die.
• The Great Tradition (1948) is a book of literary
criticism written by F.R. Leavis. So, the correct pairs • Kath walker's We are Going (1965) examines the
are A, B, and C. consequences of British colonialism in Australia. The
poem also elaborates what has been lost because of
• Hence, the correct answer is option (a).
British conquest and what will be lost in future if
66. Match List I with List II Aboriginal people aren't valued and respected.
List-I List-II Note : Kath Walker is popularly known by her name
A. Rabindranath I. Purdah and other Oodyeroo Noonuccal.
Tagore Poems • Judith Wright's Woman to Man (1949) is her second
B. Muhammad Iqbal II. The Unifinished man collection of poetry and won Grace Leven Prize for
Poetry in 1949.
C. Nissim Ezekiel III. The Child
• Banking Potatoes is authored by American poet and
D. Imtiaz Dharker IV. The Secrets of the
Pulitzer Prize winner (1994) Yusef Komunykaa.
Self
Hence, the correct answer is option (d).
Choose the correct answer from the options
68. Match List I with List II
given below :
List-I List-II
(a) A-III, B-II, C-I, D-IV
(b) A-IV, B-I, C-II, D-III A. Blood I. Phlegmatic
(c) A-III, B-IV, C-II, D-I B. Yellow Bile II. Sanguine
(d) A-III, B-II, C-I, D-IV C. Phlegm III. Melancholy
Ans. (c) : The correct match of List I & list II is (A)-III, D. Black bile IV. Choleric
(B)-IV, (C)-II, (D)-I. Choose the correct answer from the options
List-I/Authors List-II/Works given below :
(a) A-I, B-II, C-III, D-IV
A. Rabindranath Tagore III. The Child (1930)
(b) A-II, B-IV, C-I, D-III
B. Muhammad Igbal IV. The Secrets of the Self (c) A-IV, B-III, C-II, D-I
(1915) (d) A-III, B-I, C-II, D-IV
NTA UGC NET/JRF English Paper II, (Dec 2022) Shift-II 448 YCT
Ans. (b) : The correct match of List-I with List-II is : A- B. Love looks not with the III. A Midsummer
II, B-IV, C-I, D-III. eyes, but with mind, and Night's Dream
List-I List-II therefore is winged cupid
A. Blood II. Sanguine painted blind.
B. Yellow Bile IV. Choleric C. Ill deeds is doubted with II. The Comedy of
an evil word. Errors
C. Phlegm I. Phlegmatic
D. We are such stuff as I. The Tempest.
D. Black Bile III. Melancholy
dreams are made on and
• Comedy of humours is a dramatic genre most closely our little life is rounded
associated with the English playwright Ben Jonson from with sleep.
the late 16th century.
Hence, the correct answer is option (a).
• The term derives from the Latin humor, meaning
70. Match List I with List II
"liquid" and its use in the medieval and Renaissance
medical theory that the human body held a balance of List-I List-II
four liquids, or humours: Blood (Sanguine), phlegm A. Aporia I. Marxism
(phlegmatic), yellow bile (choler), and black bile B. Scapes II. Psychoanalysis
(Melancholy). C. Interpellation III. Deconstruction
• According to Ben Jonson, when these fluids are
D. Mirror Stage IV. Globalisation
Properly balanced, humours are thought to give the
individual a healthy mind in a healthy body. Choose the correct answer from the options
given below :
Hence, the correct answer is option (b).
(a) A-III, B-II, C-I, D-IV
69. Match List I with List II
(b) A-IV, B-I, C-II, D-III
List-I List-II
(c) A-III, B-IV, C-I, D-II
A. Some are born great, I. The Tempest (d) A-II, B-III, C-I, D-IV
others achieve
Ans. (c) : The correct match of List-I with List-II is
greatness.
(A)-III, (B)-IV, (C)-I, (D)-II
B. Love looks not with II. The Comedy of Errors
the eyes, but with the List-I/(Terms) List-II/(Theory)
mind, and therefore is A. Aporia III. Deconstruction
winged Cupid painted B. Scapes IV. Globalization
blind. C. Interpellation I. Marxism
C. Ill deeds is doubled III. A Midsummer Night's D. Mirror Stage II. Psychoanalysis
with an evil word. Dream
• 'Aporia' is a term commonly used in Deconstruction
D. We are such stuff as IV. Twelfth Night theory to describe the difficulty in understanding the
dreams are made on, meaning of the words. The term refers to the
and our little life is unresolvable problems that may arise while reading a
rounded with a sleep. text.
Choose the correct answer from the options • Arjun Appadurai, an anthropologist, defined the five
given below : Scapes to develop the concept of global cultural flow.
(a) A-IV, B-III, C-II, D-I The Scapes can help us understanding the complexity of
(b) A-I, B-II, C-III, D-IV process of globalization. These five Scapes are
(c) A-III, B-IV, C-I, D-II ethnoscapes, technoscapes, ideoscapes, finamescapes,
(d) A-II, B-III, C-I, D-IV and Mediascapes.
Ans. (a) : The correct match of the lines with the play • In Marxist theory, especially that of Louis Althusser,
it has been taken is : interpellation is a culture's or ideologie's creating of
(A)-IV, (B)-III, (C)-II, (D)-I. identity for "individual".
List-I/Line List-II/Play • 'Mirror stage' is a concept from Jaques Lacan's
A. Some are born great, IV. Twelfth Night psychoanalytic theory.
others achieve greatness Hence, the correct answer is option (c).

NTA UGC NET/JRF English Paper II, (Dec 2022) Shift-II 449 YCT
71. Match List I with List II • Nigerian writer Buchi Emecheta's The Bride Price
List-I List-II (1976) focuses on the problems of women in post-
A. The Political Unconscious : I. Joseph Carroll colonial Nigeria. She dedicated this novel to her mother,
Narrative as socially Alice Ogbanje Emecheta.
symbolic act • Half of a Yellow Sun is a novel by Nigerian author
B. The Pleasure of the Text II. Monique Wittig Chimamanda Ngozi Adichie, published in 2006 by 4th
Estate in London, the novel tells the story of the Biafran
C. The Straight Mind and III. Roland Barthes
other Essays war.
D. Literary Darwinism : IV. Fredric Jameson • The Lying Days (1953) is the debut novel of Nobel
Evolution, Human Nature prize winner South African novelist, Nadine Gordimer.
and Literature Hence, the correct answer is option (a).
Choose the correct answer from the options 73. Match List I with List II
given below : List-I List-II
(a) A-II, B-III, C-I, D-IV A. Ratanbai : A Sketch I. Krupabai Satthianadhan
(b) A-II, B-IV, C-I, D-III of Bombay High
(c) A-III, B-IV, C-II, D-I Caste Hindu Young
(d) A-IV, B-III, C-II, D-I Wife
Ans. (d) :The correct match of the given books with B. The Hindoo Wife or II. Shevantibai M.
Their writers is : (A)-IV, (B)-III, (C)-II, (D)-I the Enchanted Fruit Nikambe
• Fredric Jameson's The Political Unconscious : C. Kamala, A Story of III. Toru Dutt
Narrative as a Socially Symbolic Act (1981) suggests Hindu life
reading the text as an allegory, an ideological signifying
D. Bianca or The Young IV. Raj Lakshmi Debi
method which functions in the gap between signifier
Spanish Maiden
and signified.
Choose the correct answer from the options
• The Pleasure of the Text is a 1973 book by the French
essayist, literary theorist, philosopher, critic and given below :
semiotician Roland Barthes. (a) A-I, B-III, C-IV, D-II
• The Straight Mind and other Essays is a 1992 (b) A-III, B-II, C-IV, D-I
collection of essays by Monique Wittig (c) A-II, B-IV, C-I, D-III
• The book Literary Darwinism : Evolution, Human (d) A-IV, B-II, C-I, D-III
Nature and Literature is written by Joseph Carroll Ans. (c) : The correct match of books with their
Hence, the correct answer is option (d). authors is : (A)-II, (B)-IV, (C)-I, (D)-III.
72. Match List I with List II List-I/(Books) List-II/(Authors)
List-I List-II A. Ratanbai : A sketch of II. Shevantibai M.
A. The Famished Road I. Buchi emecheta a Bombay High Caste Nikambe
B. The Bride Price II. Nadine Gordimer Hindu Young Wife
C. Half of a Yellow Sun III. Ben Okri B. The Hindoo Wife or IV. Raj Lakshmi Debi
The Enchanted Fruit
D. The Lying Days IV. Chimamanda Ngozi
Adichie C. Kamala, A story of I. Krupabai
Hindu Life Satthianadhan
Choose the correct answer from the options
given below : D. Bianca or The Young III. Toru Dutt
(a) A-III, B-I, C-IV, D-II Spanish Maiden
(b) A-III, B-II, C-IV, D-I 74. Match List I with List II
(c) A-IV, B-III, C-II, D-I List-I List-II
(d) A-II, B-IV, C-I, D-III A. Richard Wright I. A Mercy
Ans. (a) : The correct match of the novels with their B. Toni Morrison II. Kindred
authors is : (A)-III, (B)-I, (C)-IV, (D)-II.
C. Barbara Chase III. American Hunger
• Winner of the 1991 Booker Prize for fiction, The Riboud
Famished Road by Nigerian author Ben Okri tells the
story of Azaro, a spirit child. D. Octavia Butler IV. Sally Hemings

NTA UGC NET/JRF English Paper II, (Dec 2022) Shift-II 450 YCT
Choose the correct answer from the options • Indian Writing in English: Critical Essays is written
given below : by David McCutchion.
(a) A-III, B-I, C-IV, D-II • The Indian Contribution to English Literature is
(b) A-III, B-II, C-IV, D-I written by A.R. Srinivasa Iyengar.
(c) A-IV, B-III, C-II, D-I
• English in India : Its Present and Future is written by
(d) A-II, B-IV, C-I, D-III
V.K. Gokak.
Ans. (a) : The correct match of the writers with their
Hence, the correct answer is option (c).
books is: (A)-III, (B)-I, (C)-IV, (D)-II.
76. Choose the correct chronological sequence in
List-I/(Writers) List-II/(Books)
which the following texts were published.
A. Richard Wright III. American Hunger
(1977) A. The Tower
B. Toni Morrison I. A Mercy (2008) B. The Hind and the Panther
C. The Wild Swans at Coole
C. Barbara Chase Riboud IV. Sally Hemings (1979)
D. Mac Flecknoe
D. Octavia Butler II. Kindred (1979)
E. The Whitsun Weddings
• Richard Wright's American Hunger (1977) is a
autobiographical work which narrates Wright's Choose the correct answer from the options
experiences after moving to the North. given below :
• Toni Morrison's A Mercy (2008) deals with slavery in (a) A, B, D, E, C (b) B, C, A, E, D
17th century. (c) B, A, C, D, E (d) D, B, C, A, E
• Barbara Chase Riboud's Sally Hemings (1979) is a Ans. (d) : The correct sequence of the publication of the
story based on historical fact recreates the relationship given poems is : D, B, C, A, E.
between Thomas Jefferson and his slave, Shally • John Dryden's Mac Flecknoe (1682) is a mock-heroic
Hemings, who bore him seven children.
satire, a direct attack on anther prominent poet of the
• Octavia E Butler's Kindred (1979) incorporates time
time Thomas Shadwell.
travel and is modeled on slave narratives.
• John Dryden's The Hind and the Panther (1687), a
Hence, the correct answer is option (a).
poem in three parts, is an allegory in heroic couplets.
75. Match List I with List II
• William Butler Yeats' The Wild Swan at Coole (1917)
List-I List-II
is a lyric poem which explores the theme of the fraility
A. M.K. Naik I. The Indian
of human life through his speaker.
Contribution to English
Literature • W.B. Yeats' The Tower (1928) is a powerful poem
B. David McCutchion II. A History of Indian that talks of his deteriorating physical health and his
English Literature growing passion in political and personal matters.
C. A.R. Srinivasa lyengar III. English in India : its • Philip Larkin's The Whitsun Weddings (1964) is a
Present and Future collection of 32 poems which recounts the speaker's
D. V.K. Gokak IV. Indian Writing in train journey from the east of England to London and
English : Critical his observations along the way.
Essays Hence, the correct answer is option (d).
Choose the correct answer from the options 77. Arrange the following poets in accordance with
given below : their years of birth.
(a) A-I, B-II, C-IV, D-III A. George Herbert
(b) A-III, B-II, C-I, D-IV B. Edmunds Spenser
(c) A-II, B-IV, C-I, D-III
C. Philip Sidney
(d) A-IV, B-I, C-III, D-II
D. John Donne
Ans. (c) : The correct match of writers with their books
E. Oliver Goldsmith
is : (A)-II, (B)-IV, (C)-I, (D)-III.
Choose the correct answer from the options
• Madhukar Krishna Naik's (MK Naik) A History of
Indian English Literature traces the course of Indian given below :
English timer, dividing it into convenient periods, in an (a) A, B, D, C, E (b) B, C, D, A, E
analytical, critical and engaging style. (c) E, B, A, D, C (d) A, D, E, B, C

NTA UGC NET/JRF English Paper II, (Dec 2022) Shift-II 451 YCT
Ans. (b) : The correct chronological order of the years 79. Find the chronological order of the writers in
of the birth of given writers is : B, C, D, A, E. terms of their years of birth :
• Edmund Spencer (1552/53-1599) is an English poet A. Jane Austen
whose long allegorical poem The Faerie Queene is one B. Henry Fielding
of the greatest in English Literature. C. James M. Barrie
• Sir Philip Sidney (1554-1586) is an Elizabethan D. Richard Doddridge Blackmore
courtier, statesman, soldier, poet and patron of scholars E. William Makepeace Thackeray
and poets. Choose the correct answer from the options
given below :
• After Shakespeare's sonnets, Sidney's Astrophel and
(a) A, B, C, D, E (b) B, A, E, D, C
Stella is considered the finest Elizabethan sonnet cycle.
(c) C, D, A, B, E (d) D, B, A, E, C
His notable works are Arcadia, Astrophel and Stella,
and The Defence of Poesie. Ans. (b) : Henry Fielding (1707-1754) was an English
novelist, irony writer and dramatist. His comic novel
• John Donne (1572-1631) is a leading English poet of
Tom Jones is still widely appreciated.
the metaphysical poetry, notable for purity and
• Jane Austen (1775-1817) was an English novelist. Her
effectiveness of his choice of words. His notable work
notable works are Emma (1815) Lady Susan, Sense and
is Easter Wings.
Sensibility (1811), Pride and Prejudice (1813), and
• George Herbert (1593-1633) is an English religious Mansfield Park (1814).
poet, a major metaphysical poet, notable for purity and • William Makepeace Thackeray (1811-1863) was a
effectiveness of his choice of words. His notable work British novelist author and illustrator. His notable works
is Easter Wings. are Vanity Fair (1847-48), and The History of Henry
• Olives Goldsmith (1730-1714) is an Anglo Irish Esmond (1852).
essayist, poet, novelist and dramatist. His notable works • Richard Doddridge Blackmore (R.D. Blakemore)
are the series to essays The Citizen of the World, or, (1825-1900) was one of the most famous English
Letters from a Chinese Philosopher (1762), the novel novelists of the second half of the nineteenth century.
The Vicar of Wakefield (1766), and the play She Stoops • James M. Barrie was a Scottish novelist and
to Conquer (1773). playwright born in 1860.
Hence, the correct answer is option (b). Hence, the correct answer is option (b).
78. Find the chronological order of publication of 80. Find the chronological order of the writers in
the given works : terms of the period they belonged to :
A. Darwin's origin of species A. Richard Steele
B. Macaulay's "Essay on Milton" B. Charles Lamb
C. Stevenson's Treasure island C. John Dryden
D. Browning's "Pauline" D. Francis Bacon
E. Matthew Arnold
E. Arnold Bennet's Old Wives' Tale
Choose the correct answer from the options
Choose the correct answer from the options
given below :
given below :
(a) A, B, C, D, E (b) B, D, E, C, A
(a) A, B, C, D, E (b) B, D, A, C, E
(c) C, B, D, A, E (d) D, C, A, B, E
(c) C, D, A, B, E (d) D, E, A, C, B
Ans. (d) : The chronological order of the writers in
Ans. (b) : The chronological order of the publication of terms of the period they belong to is : D, C, A, B, E.
the given works is: B, D, A, C, E. • Francis Bacon is the first great English essayist,
• The chronological order of publication of the books is consider to be the father of English essays belongs to
Thomas Babington Macaulay's In Essay on John Milton Jacobean period (1603-1625).
(1825), Robert Browning's first published poem • John Dryden (1631-1700) belongs to Restoration
Pauline: A Fragment of a Confession (1833), Charles period or the Age of Dryden.
Darwin's On the Origin of Species (1859), Robert Louis • Richard Steele (1672-1729) belongs to the Neo-
Stevenson's adventure novel Treasure Island (The sea Classical Age.
cook : A story of Boys), and Arnold Bennett's (novel) • Charles Lamb (1775-1834), prince of English
The Old Wives' Tale (1908). essayists belongs to Romantic age.
Note : The Old Wives' Tale is a play by George Peele, • Matthew Arnold (1822-1888) an English poet and
published in (1595). cultural critic belongs to English Victorian period.
Hence, the correct answer is option (b). Hence, the correct answer is option (d).

NTA UGC NET/JRF English Paper II, (Dec 2022) Shift-II 452 YCT
81. Find the Chronological order of publication of 83. Find the chronological order of publication of
the given works : the given works :
A. Rajmohan's wife A. Boswell's Life of Johnson
B. A Bend in the Ganges B. Hobbes's Leviathan
C. Kanthapura
C. Pepys's Diary
D. Untouchable
D. Bunyan's Pilgrim's Progress
E. Distant Drum
E. Locke's Human Understanding
Choose the correct answer from the options
given below : Choose the correct answer from the options
(a) A, B, C, D, E (b) A, C, D, E, B given below :
(c) A, D, C, B, E (d) A, E, D, C, B (a) B, C, D, E, A (b) A, C, D, E, B
Ans. (c) : The correct chronological order of the (c) C, D, A, B, E (d) D, E, A, C, B
publication of the given works is : A, D, C, B, E. Ans. (a) : The correct Chronological order to the given
• Bankim Chandra Chatterjee's Rajmohan's Wife (1864) works is : B, C, D, E, A.
is the first published novel in English by an Indian.
• Hobbes' Leviathan (161)
• Mulk Raj Anand's Untouchable (1935) deals with
• Pepys' Diary (1664)
untouchability prevelent in Indian Society at that time.
• Raja Rao's Kanthapura (1938) is one of the finest • Bunyan's Pilgrim's Progress (1678)
novels to come out of mid-twentieth century India • Lock's Human Understanding (1689)
which deals with Gandhi's Struggle for independence • Boswell's Life of Johnson (1791)
from the British Rule.
• Hence, the correct answer is option (a).
• Manohar Malgonkar's novel A Bend in the Ganges
84. Find the chronological order of publication of
(1964) deals with freedom struggle in India and ends
with the partition riots in Punjab. the given works :
• Distant Drum (1974) is a book by Manohar A. Structuralist Poetics
Malgonkar. B. Course in General Linguistics
Hence, the correct answer is option (c). C. The Pursuit of Signs
82. Find the chronological order of publication of D. The Pleasure of the text
Charles Dickens' novels : E. The Implied reader
A. Oliver twist Choose the correct answer from the options
B. Dombey and Sons given below :
C. Pickwick Papers
(a) A, C, D, E, B (b) B, D, E, A, C
D. Bleak House
(c) C, D, A, B, E (d) D, E, A, C, B
E. David Copperfield
Choose the correct answer from the options Ans. (b) : The correct chronological order to
given below publication of the given works is : B, D, E, A, C.
(a) A, D, C, B, E (b) D, E, B, C, A • Ferdinand de Saussure's Course in General
(c) B, D, C, A, E (d) C, A, B, D, E Linguistics was published in 1916 (French).
Ans. (*) : The chronological order of publication of • Roland Barthes' The Pleasure of the Text was
Charles Dickens' novels will be as : published in 1973 (French).
(C) Pickwick Papers – 1836 • Wolfgang Iser's Implied Reader was published in
(A) Oliver Twist – 1838
1974.
(B) Dombey and Sons – 1848
• Jonathan Culler's Structuralist Poetics was published
(E) David Copperfield (1850)
in 1975.
(D) Bleak House (1852)
• So, it should be C, A, B, E, D. • Jonathan Culler's The Pursuit of Signs was published
in (1981).
• None of the given options has the combination.
Hence, NTA has dropped this question. • Hence, the correct answer is option (b).

NTA UGC NET/JRF English Paper II, (Dec 2022) Shift-II 453 YCT
85. Given below are two statements, one is labeled unpredictable change in the play. In light of the
as Assertion (A) and the other is labeled as above statements, choose the correct answer
Reason (R). from the options given below :
Assertion (A) : From a sociolinguistic point of (a) Both Statement I and Statement II are true.
view, mainstream SLA studies remain asocial- (b) Both Statement I and Statement II are false.
the social import of learning to interact
(c) Statement I is true but Statement II is false.
through language remains hidden.
Reason (R) : A sociolinguistic perspective (d) Statement I is false but Statement II is true.
focusses on the linguistic system as well as on a Ans. (b) : In the light of the above statements, Both
concern with specific items of pragmatic and Statement I and Statement II are false.
discourse development and rejects the tendency • "Anagenesis" is not the final part of drama instead it is
of looking at language as a set of norms, at
a mechanism in which one species evolves into another
language diversity and ideologies.
by evolutionary changes within a lineage.
In light of the above statements, choose the
most appropriate answer from the options • "Anagnorisis" is the turning point of the play where
given below : audience observes unpredictable change in the play.
(a) Both (A) and (R) are correct and (R) is the Hence, the correct answer is option (b).
correct explanation of (A). 88. Given below are two statements :
(b) Both (A) and (R) are correct and (R) is the Statement I : It is true that there is an analogy
correct explanation of (A)
between the works of an author and the
(c) (A) is correct but (R) is not correct
experiences of his life.
(d) (A) is not correct but (R) is correct
Statement II : The works may be seen as an
Ans. (c) : In the light of the above statements, the
incomplete translation of the life.
statement given in assertion is absolutely correct while
the reason is not correct. In light of the above statements, choose the
correct answer from the options given below :
86. Given below are two statements, one is labelled
as Assertion (A) and the other is labelled as (a) Both Statement I and Statement II are true.
Reason (R). (b) Both Statement I and Statement II are false.
Assertion (A) : In extensive reading, the (c) Statement I is true but Statement II is false.
teachers play the main role. (d) Statement I is false but Statement II is true.
Reason (R) : The aim of extensive reading is to Ans. (a) : In the light of the above statements both the
enrich learners knowledge.
statements are absolutely correct.
In light of the above statements, choose the
most appropriate answer from the options • It is true that there is an analogy between the work of
given below : an author and the experiences of his life.
(a) Both (A) and (R) are correct and (R) is the • The work may be seen as an incomplete translation of
correct explanation of (A). the life.
(b) Both (A) and (R) are correct and (R) is the Hence, the correct answer is option (a).
correct explanation of (A)
89. Given below are two statements :
(c) (A) is correct but (R) is not correct.
Statement I : Things acquire a rational
(d) (A) is not correct but (R) is correct.
signification, and not only one of simple usage,
Ans. (d) : In the light of the above Statements, assertion
because an other is associated with my
is not correct while the reason is absolutely correct.
relations with them.
• In extensive reading, the teacher does not play the
main role instead the readers/ students have to be very Statement II : In designating a thing, I
active. designate it to the other.
• The aim of extensive reading is to enrich learner's In light of the above statements, choose the
knowledge. correct answer from the options given below :
• Hence, the correct answer is option (d). (a) Both Statement I and Statement II are true
87. Given below are two statements : (b) Both Statement I and Statement II are false.
Statement I : "Anagenesis" is the final part of (c) Statement I is true but Statement II is false.
the drama just after the climax in which there (d) Statement I is false but Statement II is true
is resolution for any conflicts left in the plot. Ans. (a) : In the light of the above statements, both the
Statement II : "Anagenesis" is the turning
statements are absolutely correct.
point of the play where audience observes
NTA UGC NET/JRF English Paper II, (Dec 2022) Shift-II 454 YCT
90. Given below are two statements : (a) the poet
Statement I : The poststructuralists' genre (b) the poet's dog.
critics suggest that the way to 'de-essentialize' (c) a game played by two or more blindfolded
genre is to re-cast it in terms of discourse. persons
Statement II : But while the poststructuralist (d) the poet's mother.
move toward dialectical exchange and ideology Ans. (c) : Moriarty is the name of a game played by two
is both useful and necessary, conceiving of or more blindfolded persons.
genre primarily in terms of discourse reveals • Each player is blindfolded and given a rolled up
certain limitations that are intrinsic to newspaper (anything which is not likely to injure).
poststructuralism's basic approach to discourse
• Here, in the poem the two player in this game are the
and to the relationship envisioned between
poet and his father.
discourse and subjectivity.
Hence, the correct answer is option (c).
In light of the above statements, choose the
correct answer from the options given below : 93. Which one of the following statements is true?
(a) Both Statement I and Statement II are true. (a) The poet imagines that his father tightens his
grip on the rolled-up copy of Times.
(b) Both Statement I and Statement II are false.
(b) The poet's father tightens his grip on the
(c) Statement I is true but Statement II is false
rolled-up copy of times.
(d) Statement I is false but Statement II is true.
(c) The poet is sure that his father tightens his
Ans. (b) : In the light of the above statements regarding grip on the rolled-up copy of Times.
Post Structuralism, both the statements are incorrect. (d) The poet sees his father tightening his grip on
Instruction (91-95): Read the following poem and the rolled-up copy of times.
answer the questions that follows : Ans. (a) : Among the given statements, the first
Are you There? statement mentioned in option (a) is absolutely correct.
My father and I shove back the furniture • The poet imagines that his father frightens his grip on
to the four walls of the sitting room the rolled-up copy of Times (News paper).
then lie on the carpet wearing blindfolds, • The above statement is clearly mentioned in second
his left hand holding my left hand. stanza.
Are you there, Moriarty? he enquires, Hence, the correct answer is option (a).
before tightening (I imagine) the grip 94. The 'Weapon' mentioned in the first line of the
on his rolled-up copy of yesterday's Times. fourth stanza of the poem is
There is only one possible answer to that. (a) a knife
I give it while rolling away to the side (b) a rolled-up newspaper.
but still clasping his hand, still in range, (c) a scissor
and sure enough he manages a direct hit. (d) a stick
Now it is my turn, but the moment I lift Ans. (b) : The 'weapon' mentioned in the first line of
my weapon I realise there is no stillness, the fourth stanza of the poem is "a rolled-up
and the chill and stiffness of his fingers, newspaper".
he has been dead for a good while already. Hence, the correct answer is option (b).
Andrew Motion 95. In the last stanza of the poem,
91. The poet and his father shove back furniture to (a) the poet is sure of his father's death.
(a) sleep well (b) the poet imagines his father to be dead.
(b) to play a game (c) the poet does not know whether his father is
(c) to create space for more furniture alive.
(d) to lie down to contemplate. (d) the poet wildly guesses that his father is dead.
Ans. (b) : The poet and his father shove back the Ans. (a) : In the last stanza of the poem, the poet is sure
furniture to the four walls of the sitting room to play a of his father's death.
game (Are you there, Moriarty). • In the last stanza, the poet realized that there was no
Hence, the correct answer is option (b). reason to continue. He could tell from his stillness and
92. Read the following poem and answer the chill and stillness of his fingers. He was sure that his
questions that follows : father is dead.
Moriarty is the name of Hence, the correct answer is option (a) .

NTA UGC NET/JRF English Paper II, (Dec 2022) Shift-II 455 YCT
Instructions: (96-100): Read the following passage and 97. What is the implication of the phrase, 'there is
answer the questions that follow : In this sense, no essence' of literature whatsoever" in the
one can think of literature less as some inherent passage?
quality or set of qualities displayed by certain (a) There is no sensibility in literary texts
kinds of writing all the way from Beowulf to (b) There is no central meaning in literary texts.
Virginia woolf, than as a number of ways in
(c) There is no aesthetic consideration in
which people relate themselves to writing. It
literature.
would not be easy to isolate, from all that has been
variously called 'literature', some constant set of (d) There is no rational logic in literature.
inherent features. In fact, it would be as Ans. (b) : In the given excerpt of the "Literary Theory"
impossible as trying to identify the single by "Terry Eagleton, The implication of the phrase
distinguishing feature which all games have in "There is no 'essence' of literature whatsoever" is "there
common. There is no 'essence' of literature is no central meaning in literary texts".
whatsoever. Any bit of writing may be read 'non- • Literary texts are interpreted and read in the situation
pragmatically', if that is what reading a text as and it's completely based on the reader.
literature means, just as any writing may be read
Hence, the correct answer is option (b).
'poetically'. If pore over the railway timetable not
to discover a train connection but to stimulate in 98. What is the meaning of the term "non-
myself general reflections on the speed and pragmatic" used in the passage?
complexity of modern existence, then I might be (a) Scientific (b) Rational
said to be reading it as literature. John M. Ellis has (c) Practical (d) Affective
argued that the term 'literature' operates rather like Ans. (d) : The meaning of the term "non pragmatic" in
the word 'weed' : weeds are not particular kinds of the passage means "affective".
plant, but just any kind of plant which for some
Hence, the correct answer is option (d).
reason or another a gardener does not want
around. Perhaps 'literature' means something like 99. What is the significance of the analogy drawn
the opposite : any kind of writing which for some between "weed" and "literature" in the context
reason or another somebody values highly. As the of the passage?
philosophers might say, 'literature' and 'weed' are (a) They do not serve any descriptive function.
functional rather than ontological terms : they tell (b) They serve some prescriptive functions.
us about what we do, not about the fixed being of (c) They are considered expendable by
things. deterministic systems.
96. What is the implication of the statement : In (d) They are both subterranean beings.
this sense, one can think of literature less as
some inherent quality or set of qualities Ans. (c) : Analogy drawn between "weed" and
displayed by certain kinds by certain kinds of "literature" in the context of the passage in " the context
writing all the way from Beowulf to Virginia of the passage is "they are considered expendable by
Woolf, than as a number of ways in which deterministic systems".
people relate themselves to writing"? Hence, the correct answer is option (c).
(a) Literature has values that are constant and 100. What is the meaning of the word
universal for all classes and races. "ontological"?
(b) Literature has a moral dimension, which (a) Dealing with the study of "knowledge
cannot be superseded by any other values. systems".
(c) Literature has values that may be interpreted (b) Dealing with the study of "being".
differently by different subject-positions.
(c) Dealing with the study of "Society".
(d) The inherent quality of literature is its
(d) Dealing with the study of "nature".
literariness.
Ans. (b) : Meaning of "ontological" is "related to the
Ans. (c) : The implication of the given first line of the
passage is literature has values that may be interpreted branch of metaphysics dealing with the nature of
differently be different subject-positions. being".
Hence, the correct answer is option (c). Hence, the correct answer is option (b).

NTA UGC NET/JRF English Paper II, (Dec 2022) Shift-II 456 YCT

You might also like